Sie sind auf Seite 1von 436

1

[G.R. No. 163783.June 22, 2004] After a considered and judicious examination of the arguments raised by petitioner as well as
those presented in the Comments filed by the Solicitor General and respondent Joint
PIMENTEL vs. CONGRESS Committee, this Court finds that the petition has absolutely no basis under the Constitution
and must, therefore, be dismissed.
EN BANC
Petitioner's claim that his arguments are buttressed by "legislative procedure, precedent or
practice [as] borne [out] by the rules of both Houses of Congress" is directly contradicted by
Section 42 of Rule XIV of the Rules adopted by the Senate, of which he is an incumbent
Gentlemen:
member.� This section clearly provides that the Senate shall convene in joint session during
Quoted hereunder, for your information, is a resolution of this Court dated JUN 22 2004. any voluntary or compulsory recess to canvass the votes for President and Vice-President not
later than thirty days after the day of the elections in accordance with Section 4, Article VII of
G. R. No. 163783 (Aquilino Q. Pimentel, Jr. vs. Joint Committee of Congress to Canvass the the Constitution.
Votes Cast for President and Vice-President in the May 10, 2004 Elections.)
Moreover, as pointed out in the Comment filed by the Senate Panel for respondent Joint
RESOLUTION Committee and that of the Office of the Solicitor General, the precedents set by the 1992 and
1998 Presidential Elections do not support the move to stop the ongoing canvassing by the
By the present Petition for Prohibition, petitioner Senator Aquilino Q. Pimentel, Jr. seeks a Joint Committee, they citing the observations of former Senate President Jovito Salonga.
judgment declaring null and void the continued existence of the Joint Committee of Congress
(Joint Committee) to determine the authenticity and due execution of the certificates of Thus, during the 1992 Presidential elections, both Houses of Congress adjourned sine die on
canvass and preliminarily canvass the votes cast for Presidential and Vice-Presidential May 25, 1992. On June 16, 1992, the Joint Committee finished tallying the votes for President
candidates in the May 10, 2004 elections following the adjournment of Congress sine die on and Vice-President.[1]cralaw Thereafter, on June 22, 1992, the Eighth Congress convened in
June 11, 2004. The petition corollarily prays for the issuance of a writ of prohibition directing joint public session as the National Board of Canvassers, and on even date proclaimed Fidel
the Joint Committee to cease and desist from conducting any further proceedings pursuant V. Ramos and Joseph Ejercito Estrada as President and Vice-President, respectively.[2]cralaw
to the Rules of the Joint Public Session of Congress on Canvassing.
Upon the other hand, during the 1998 Presidential elections, both Houses of Congress
Petitioner posits that with "the adjournment sine die on June 11, 2004 by the Twelfth adjourned sine die on May 25, 1998. The Joint Committee completed the counting of the
Congress of its last regular session, [its] term ... terminated and expired on the said day and votes for President and Vice-President on May 27, 1998.[3]cralaw The Tenth Congress then
the said Twelfth Congress serving the term 2001 to 2004 passed out of legal existence." convened in joint public session on May 29, 1998 as the National Board of Canvassers and
Henceforth, petitioner goes on, "all pending matters and proceedings terminate upon the proclaimed Joseph Ejercito Estrada as President and Gloria Macapagal-Arroyo as President
expiration of ... Congress." To advance this view, he relies on "legislative procedure, and Vice-President, respectively.[4]cralaw
precedent or practice [as] borne [out] by the rules of both Houses of Congress."
As for petitioner's argument that "the [e]xistence and [p]roceedings [o]f the Joint Committee
Given the importance of the constitutional issue raised and to put to rest all questions of Congress [a]re [i]nvalid, [i]llegal and [u]nconstitutional [f]ollowing the [a]djournment [s]ine
regarding the regularity, validity or constitutionality of the canvassing of votes fro President [d]ie [o]f [b]oth Houses of Congress [o]f [t]heir [r]egular [s]essions on June 11, 2004," he cites
and Vice-President in the recently concluded national elections, this Court assumes in support thereof Section 15, Article VI of the Constitution which reads:
jurisdiction over the instant petition pursuant to its power and duty "to determine whether
or not there has been a grave abuse of discretion amounting to lack or excess of jurisdiction Sec. 15. The Congress shall convene once every year on the fourth Monday of July for its
on the part of any branch or instrumentality of the Government" under Section 1 of Article regular session, unless a different date is fixed by law, and shall continue to be in session for
VIII of the Constitution and its original jurisdiction over petitions for prohibition under such number of days as it may determine until thirty days before the opening of its next
Section 5 of the same Article. regular session, exclusive of Saturdays, Sundays, and legal holidays. The President may call a
special session at any time.
2

Contrary to petitioner's argument, however, the term of the present Twelfth Congress did approval of the joint public session of both Houses of Congress, which may reconvene
not terminate and expire upon the adjournment sine die of the regular session of both without need of call by the President to a special session.
Houses on June 11, 2004.
WHEREFORE, the instant Petition is hereby DISMISSED.
Section 15, Article VI of the Constitution cited by petitioner does not pertain to the term of
Congress, but to its regular annual legislative sessions and the mandatory 30-day recess Aquilino Q. Pimentel, Jr. vs. Joint Committee of Congress to Canvass the Votes Cast for
before the opening of its next regular session (subject to the power of the President to call a President and Vice-President in the May 10, 2004 Elections.
special session at any time).
G.R. No. 163783 |June 22, 2004 |J. Puno
Section 4 of Article VIII also of the Constitution clearly provides that "[t]he term of office of
The Senate shall convene in joint session during any voluntary or compulsory recess to
the Senators shall be six years and shall commence, unless otherwise provided by law, at
canvass the votes for President and Vice-President not later than thirty days after cvc the
noon on the thirtieth day of June next following their election." Similarly, Section 7 of the
day of the elections in accordance with Section 4, Article VII of the Constitution.
same Article provides that "[t]he Members of the House of Representatives shall be elected
for a term of three years which shall begin, unless otherwise provided by law, at noon on the
Facts:
thirtieth day of June next following their election." Consequently, there being no law to the
contrary, until June 30, 2004, the present Twelfth Congress to which the present legislators Senator Pimentel Jr. seeks to declare null and void the continued existence of the Joint
belong cannot be said to have "passed out of legal existence." Committee and prohibit it with its continuous action. He claims that with the adjournment
on June 11, 2004 by the 12th Congress of its last regular session, its legal existence has
The legislative functions of the Twelfth Congress may have come to a close upon the final
ended thus all pending matters and proceedings end upon the expiration of the Congress.
adjournment of its regular sessions on June 11, 2004, but this does not affect its non-
legislative functions, such as that of being the National Board of Canvassers. In fact, the joint Issue: Whether Senator Pimentel’s action will prosper.
public session of both Houses of Congress convened by express directive of Section 4, Article
VII of the Constitution to canvass the votes for and to proclaim the newly elected President Ruling:
and Vice-President has not, and cannot, adjourn sine die until it has accomplished its
constitutionally mandated tasks. For only when a board of canvassers has completed its NO. Petitioner's claim that his arguments are buttressed by legislative procedure, precedent
functions is it rendered functus officio. Its membership may change, but it retains its or practice as borne out by the rules of both Houses of Congress is directly contradicted by
authority as a board until it has accomplished its purposes. (Pelayo v. Commission on Section 42 of Rule XIV of the Rules adopted by the Senate, of
Elections, 23 SCRA 1374, 1385 [1968], citing Bautista v. Fugoso, 60 Phil. 383, 389 [1934] and which he is an incumbent member. This section clearly provides that the Senate shall
Aquino v. Commission on Elections, L-28392, January 29 1968) convene in joint session during any voluntary or compulsory recess to canvass the votes for
President and Vice-President not later than thirty days after the day of the elections in
Since the Twelfth Congress has not yet completed its non-legislative duty to canvass the accordance with Section 4, Article VII of the Constitution.
votes and proclaim the duly elected President and Vice-President, its existence as the
National Board of Canvassers, as well as that of the Joint Committee to which it referred the Moreover, as pointed out in the Comment filed by the Senate Panel for respondent Joint
preliminary tasks of authenticating and canvassing the certificates of canvass, has not Committee and that of the Office of the Solicitor General, the precedents set by the 1992 and
become functus officio. 1998 Presidential Elections do not support the move to stop the ongoing canvassing by the
Joint Committee, they citing the observations of former Senate President Jovito Salonga.

Thus, during the 1992 Presidential elections, both Houses of Congress adjourned sine die on
In sum, despite the adjournment sine die of Congress, there is no legal impediment to the May 25, 1992. On June 16, 1992, the Joint Committee finished tallying the votes for
Joint Committee completing the tasks assigned to it and transmitting its report for the President and Vice-President.
3

Thereafter, on June 22, 1992, the Eighth Congress convened in jointpublic session as the session (subject to the power of the President to call a special session at any time).Section 4
National Board of Canvassers, and on even date proclaimed Fidel V. Ramos and Joseph of Article VIII provides that "[t]he term of office of the Senators shall be six years and shall
Ejercito Estrada as President and VicePresident, respectively. commence, unlessotherwise provided by law, at noon on the thirtieth day of June next
following their election." Similarly, Section 7 provides that"[t]he Members of the House of
Upon the other hand, during the 1998 Presidential elections, both Houses of Congress Representatives shall be elected for a term of three years. Consequently, there being no law
adjourned sine die on May 25, 1998. The Joint Committee completed the counting of the tothe contrary, until June 30, 2004, the present Twelfth Congress to which the present
votes for President and VicePresident on May 27, 1998. The Tenth Congress then convened legislators belong cannot be said to have"passed out of legal existence.
in joint public session on May 29, 1998 as the National Board of Canvassers and proclaimed
Joseph Ejercito Estrada as President and Gloria MacapagalArroyo as President and Vice- "The legislative functions of the Twelfth Congress may have come to a close upon the final
President, respectively. adjournment of its regular sessions on June 11, 2004, but this does not affect its non-
legislative functions. In fact, the joint public session of both Housesof Congress convened by
PIMENTEL vs CONGRESS express directive of Section 4, Article VII to canvass the votes for and to proclaim the newly
electedPresident and VP has not, and cannot, adjourn sine die until it has accomplished its
Joint Committee of Congress to Canvass the Votes Cast for President and Vice-President in
constitutionally mandated tasks. For onlywhen a board of canvassers has completed its
the May 10, 2004 Elections
functions is it rendered functus officio. Its membership may change, but it retainsits authority
as a board until it has accomplished its purposes.
Facts:
Since the Twelfth Congress has not yet completed its non-legislative duty to canvass the
Petition for Prohibition. Pimentel, Jr. seeks a judgment declaring null and void the continued
votes and proclaim the dulyelected President and VP, its existence as the National Board of
existence of the JointCommittee. The petition corollarily prays for the issuance of a writ of
Canvassers, as well as that of the Joint Committee to which itreferred the preliminary tasks
prohibition directing the Joint Committee to cease anddesist from conducting any further
of authenticating and canvassing the certificates of canvass, has not become functus officio.
proceedings pursuant to the Rules of the Joint Public Session of Congress on Canvassing.
Petitioner posits that with "the adjournment sine die(w/o date fixed) on June 11, 2004 by the
Twelfth Congress of its lastregular session, [its] term ... terminated and expired on the said
day and the said Twelfth Congress serving the term 2001 to 2004 passed out of legal
existence. " Henceforth, petitioner goes on, "all pending matters and proceedings terminate
upon the expiration of Congress.

ISSUE: WON the Joint Committee performing election canvass even after the termination of
congress’ session is constitutional.

RULING:

Sec. 15. Art VI - The Congress shall convene once every year on the fourth Monday of July for
its regular session,unless a different date is fixed by law, and shall continue to be in session
for such number of days as it may determine until thirtydays before the opening of its next
regular session, exclusive of Saturdays, Sundays, and legal holidays. The President may call
aspecial session at any time.Contrary to petitioner's argument, however, the term of the
present Twelfth Congress did not terminate and expire upon theadjournment sine die of the
regular session of both Houses on June 11, 2004.Section 15, Article VI of the Constitution
cited by petitioner does not pertain to the term of Congress, but to its regular annual
legislative sessions and the mandatory 30-day recess before the opening of its next regular
4

EN BANC 2. Petitioner is estopped from assailing the jurisdiction of the PET.

G.R. No. 191618 June 7, 2011 3. The constitution of the PET is "on firm footing on the basis of the grant of
authority to the [Supreme] Court to be the sole judge of all election contests for
ATTY. ROMULO B. MACALINTAL, Petitioner, the President or Vice-President under paragraph 7, Section 4, Article VII of the 1987
vs. Constitution."
PRESIDENTIAL ELECTORAL TRIBUNAL, Respondent.
Except for the invocation of our decision in Louis ‟Barok" C. Biraogo v. The Philippine Truth
RESOLUTION Commission of 2010,3petitioner does not allege new arguments to warrant reconsideration
of our Decision.
NACHURA, J.:
We cannot agree with his insistence that the creation of the PET is unconstitutional. We
reiterate that the abstraction of the Supreme Court acting as a Presidential Electoral Tribunal
Before us is a Motion for Reconsideration filed by petitioner Atty. Romulo B. Macalintal of
from the unequivocal grant of jurisdiction in the last paragraph of Section 4, Article VII of the
our Decision1 in G.R. No. 191618 dated November 23, 2010, dismissing his petition and
Constitution is sound and tenable. The provision reads:
declaring the establishment of respondent Presidential Electoral Tribunal (PET) as
constitutional.
Sec. 4. x x x.
Petitioner reiterates his arguments on the alleged unconstitutional creation of the PET:
The Supreme Court, sitting en banc, shall be the sole judge of all contests relating to the
election, returns, and qualifications of the President or Vice-President, and may promulgate
1. He has standing to file the petition as a taxpayer and a concerned citizen.
its rules for the purpose.

2. He is not estopped from assailing the constitution of the PET simply by virtue of
We mapped out the discussions of the Constitutional Commission on the foregoing provision
his appearance as counsel of former president Gloria Macapagal-Arroyo before
and concluded therefrom that:
respondent tribunal.

The mirabile dictu of the grant of jurisdiction to this Court, albeit found in the Article on the
3. Section 4, Article VII of the Constitution does not provide for the creation of the
executive branch of government, and the constitution of the PET, is evident in the
PET.
discussions of the Constitutional Commission. On the exercise of this Court’s judicial power
as sole judge of presidential and vice-presidential election contests, and to promulgate its
4. The PET violates Section 12, Article VIII of the Constitution. rules for this purpose, we find the proceedings in the Constitutional Commission most
instructive:
To bolster his arguments that the PET is an illegal and unauthorized progeny of Section 4,
Article VII of the Constitution, petitioner invokes our ruling on the constitutionality of the MR. DAVIDE. On line 25, after the words "Vice-President," I propose to add AND MAY
Philippine Truth Commission (PTC).2Petitioner cites the concurring opinion of Justice Teresita PROMULGATE ITS RULES FOR THE PURPOSE. This refers to the Supreme Court sitting en
J. Leonardo-de Castro that the PTC is a public office which cannot be created by the banc. This is also to confer on the Supreme Court exclusive authority to enact the necessary
President, the power to do so being lodged exclusively with Congress. Thus, petitioner rules while acting as sole judge of all contests relating to the election, returns and
submits that if the President, as head of the Executive Department, cannot create the PTC, qualifications of the President or Vice-President.
the Supreme Court, likewise, cannot create the PET in the absence of an act of legislature.
MR. REGALADO. My personal position is that the rule-making power of the Supreme Court
On the other hand, in its Comment to the Motion for Reconsideration, the Office of the with respect to its internal procedure is already implicit under the Article on the Judiciary;
Solicitor General maintains that: considering, however, that according to the Commissioner, the purpose of this is to
indicate the sole power of the Supreme Court without intervention by the legislature in the
1. Petitioner is without standing to file the petition. promulgation of its rules on this particular point, I think I will personally recommend its
acceptance to the Committee.
5

MR. NOLLEDO x x x. The Supreme Court, sitting en banc, shall be the sole judge of all contests relating to the
election, returns and qualifications of the President or Vice-President.
With respect to Sections 10 and 11 on page 8, I understand that the Committee has also
created an Electoral Tribunal in the Senate and a Commission on Appointments which may May I seek clarification as to whether or not the matter of determining the outcome of the
cover membership from both Houses. But my question is: It seems to me that the committee contests relating to the election returns and qualifications of the President or Vice-
report does not indicate which body should promulgate the rules that shall govern the President is purely a political matter and, therefore, should not be left entirely to the
Electoral Tribunal and the Commission on Appointments. Who shall then promulgate the judiciary. Will the above-quoted provision not impinge on the doctrine of separation of
rules of these bodies? powers between the executive and the judicial departments of the government?

MR. DAVIDE. The Electoral Tribunal itself will establish and promulgate its rules because it MR. REGALADO. No, I really do not feel that would be a problem. This is a new provision
is a body distinct and independent already from the House, and so with the Commission on incidentally. It was not in the 1935 Constitution nor in the 1973 Constitution.
Appointments also. It will have the authority to promulgate its own rules.
MR. VILLACORTA. That is right.
On another point of discussion relative to the grant of judicial power, but equally cogent, we
listen to former Chief Justice Roberto Concepcion: MR. REGALADO. We feel that it will not be an intrusion into the separation of powers
guaranteed to the judiciary because this is strictly an adversarial and judicial proceeding.
MR. SUAREZ. Thank you.
MR. VILLACORTA. May I know the rationale of the Committee because this supersedes
Would the Commissioner not consider that violative of the doctrine of separation of powers? Republic Act 7950 which provides for the Presidential Electoral Tribunal?

MR. CONCEPCION. I think Commissioner Bernas explained that this is a contest between FR. BERNAS. Precisely, this is necessary. Election contests are, by their nature, judicial.
two parties. This is a judicial power. Therefore, they are cognizable only by courts. If, for instance, we did not have a
constitutional provision on an electoral tribunal for the Senate or an electoral tribunal for
MR. SUAREZ. We know, but practically the Committee is giving to the judiciary the right to the House, normally, as composed, that cannot be given jurisdiction over contests.
declare who will be the President of our country, which to me is a political action.
So, the background of this is really the case of Roxas v. Lopez. The Gentleman will remember
MR. CONCEPCION. There are legal rights which are enforceable under the law, and these that in that election, Lopez was declared winner. He filed a protest before the Supreme Court
are essentially justiciable questions. because there was a republic act which created the Supreme Court as the Presidential
Electoral Tribunal. The question in this case was whether new powers could be given the
Supreme Court by law. In effect, the conflict was actually whether there was an attempt to
MR. SUAREZ. If the election contest proved to be long, burdensome and tedious, practically
create two Supreme Courts and the answer of the Supreme Court was: "No, this did not
all the time of the Supreme Court sitting en banc would be occupied with it considering
involve the creation of two Supreme Courts, but precisely we are giving new jurisdiction to
that they will be going over millions and millions of ballots or election returns, Madam
the Supreme Court, as it is allowed by the Constitution. Congress may allocate various
President.
jurisdictions."

Echoing the same sentiment and affirming the grant of judicial power to the Supreme Court,
Before the passage of that republic act, in case there was any contest between two
Justice Florenz D. Regalado and Fr. Joaquin Bernas both opined:
presidential candidates or two vice-presidential candidates, no one had jurisdiction over
it. So, it became necessary to create a Presidential Electoral Tribunal. What we have done
MR. VILLACORTA. Thank you very much, Madam President. is to constitutionalize what was statutory but it is not an infringement on the separation of
powers because the power being given to the Supreme Court is a judicial power.
I am not sure whether Commissioner Suarez has expressed his point. On page 2, the fourth
paragraph of Section 4 provides: Unmistakable from the foregoing is that the exercise of our power to judge presidential and
vice-presidential election contests, as well as the rule-making power adjunct thereto, is
plenary; it is not as restrictive as petitioner would interpret it. In fact, former Chief Justice
6

Hilario G. Davide, Jr., who proposed the insertion of the phrase, intended the Supreme Court The Supreme Court, sitting en banc[,] shall be the sole judge of all contests relating to the
to exercise exclusive authority to promulgate its rules of procedure for that purpose. To this, election, returns and qualifications of the President or Vice-President.1avvphi1
Justice Regalado forthwith assented and then emphasized that the sole power ought to be
without intervention by the legislative department. Evidently, even the legislature cannot Are we not giving enormous work to the Supreme Court especially when it is directed to
limit the judicial power to resolve presidential and vice-presidential election contests and our sit en banc as the sole judge of all presidential and vice-presidential election contests?
rule-making power connected thereto.
MR. SUMULONG. That question will be referred to Commissioner Concepcion.
To foreclose all arguments of petitioner, we reiterate that the establishment of the PET
simply constitutionalized what was statutory before the 1987 Constitution. The experiential
MR. CONCEPCION. This function was discharged by the Supreme Court twice and the
context of the PET in our country cannot be denied.4
Supreme Court was able to dispose of each case in a period of one year as provided by law.
Of course, that was probably during the late 1960s and early 1970s. I do not know how the
Stubbornly, despite the explicit reference of the Members of the Constitutional Commission present Supreme Court would react to such circumstances, but there is also the question of
to a Presidential Electoral Tribunal, with Fr. Joaquin Bernas categorically declaring that in who else would hear the election protests.
crafting the last paragraph of Section 4, Article VII of the Constitution, they
"constitutionalize[d] what was statutory," petitioner continues to insist that the last
MR. SUAREZ. We are asking this question because between lines 23 to 25, there are no rules
paragraph of Section 4, Article VII of the Constitution does not provide for the creation of the
provided for the hearings and there is not time limit or duration for the election contest to
PET. Petitioner is adamant that "the fact that [the provision] does not expressly prohibit [the]
be decided by the Supreme Court. Also, we will have to consider the historical background
creation [of the PET] is not an authority for the Supreme Court to create the same."
that when R.A. 1793, which organized the Presidential Electoral Tribunal, was promulgated
on June 21, 1957, at least three famous election contests were presented and two of them
Petitioner is going to town under the misplaced assumption that the text of the provision ended up in withdrawal by the protestants out of sheer frustration because of the delay in
itself was the only basis for this Court to sustain the PET’s constitutionality. the resolution of the cases. I am referring to the electoral protest that was lodged by former
President Carlos P. Garcia against our "kabalen" former President Diosdado Macapagal in
We reiterate that the PET is authorized by the last paragraph of Section 4, Article VII of the 1961 and the vice-presidential election contest filed by the late Senator Gerardo Roxas
Constitution and as supported by the discussions of the Members of the Constitutional against Vice-President Fernando Lopez in 1965.
Commission, which drafted the present Constitution.
MR. CONCEPCION. I cannot answer for what the protestants had in mind. But when that
The explicit reference by the framers of our Constitution to constitutionalizing what was protest of Senator Roxas was withdrawn, the results were already available. Senator Roxas
merely statutory before is not diluted by the absence of a phrase, line or word, mandating did not want to have a decision adverse to him. The votes were being counted already, and
the Supreme Court to create a Presidential Electoral Tribunal. he did not get what he expected so rather than have a decision adverse to his protest, he
withdrew the case.
Suffice it to state that the Constitution, verbose as it already is, cannot contain the specific
wording required by petitioner in order for him to accept the constitutionality of the PET. MR. SUAREZ. I see. So the Commission would not have any objection to vesting in the
Supreme Court this matter of resolving presidential and vice-presidential contests?
In our Decision, we clarified the structure of the PET:
MR. CONCEPCION. Personally, I would not have any objection.
Be that as it may, we hasten to clarify the structure of the PET as a legitimate progeny of
Section 4, Article VII of the Constitution, composed of members of the Supreme Court, MR. SUAREZ. Thank you.
sitting en banc. The following exchange in the 1986 Constitutional Commission should
provide enlightenment: Would the Commissioner not consider that violative of the doctrine of separation of powers?

MR. SUAREZ. Thank you. Let me proceed to line 23, page 2, wherein it is provided, and I MR. CONCEPCION. I think Commissioner Bernas explained that this is a contest between two
quote: parties. This is a judicial power.
7

MR. SUAREZ. We know, but practically the Committee is giving to the judiciary the right to paragraph of Section 4, Article VII of the Constitution to decide presidential and vice-
declare who will be the President of our country, which to me is a political action. presidential elections contests includes the means necessary to carry it into effect. Thus:

MR. CONCEPCION. There are legal rights which are enforceable under the law, and these are Obvious from the foregoing is the intent to bestow independence to the Supreme Court as
essentially justiciable questions. the PET, to undertake the Herculean task of deciding election protests involving presidential
and vice-presidential candidates in accordance with the process outlined by former Chief
MR. SUAREZ. If the election contest proved to be long, burdensome and tedious, practically Justice Roberto Concepcion. It was made in response to the concern aired by delegate Jose E.
all the time of the Supreme Court sitting en banc would be occupied with it considering Suarez that the additional duty may prove too burdensome for the Supreme Court. This
that they will be going over millions and millions of ballots or election returns, Madam explicit grant of independence and of the plenary powers needed to discharge this burden
President. justifies the budget allocation of the PET.

MR. CONCEPCION. The time consumed or to be consumed in this contest for President is The conferment of additional jurisdiction to the Supreme Court, with the duty characterized
dependent upon they key number of teams of revisors. I have no experience insofar as as an "awesome" task, includes the means necessary to carry it into effect under the doctrine
contests in other offices are concerned. of necessary implication. We cannot overemphasize that the abstraction of the PET from the
explicit grant of power to the Supreme Court, given our abundant experience, is not
unwarranted.
MR. SUAREZ. Although there is a requirement here that the Supreme Court is mandated to
sit en banc?
A plain reading of Article VII, Section 4, paragraph 7, readily reveals a grant of authority to
the Supreme Court sitting en banc. In the same vein, although the method by which the
MR. CONCEPCION. Yes.
Supreme Court exercises this authority is not specified in the provision, the grant of power
does not contain any limitation on the Supreme Court's exercise thereof. The Supreme
MR. SUAREZ. I see. Court's method of deciding presidential and vice-presidential election contests, through the
PET, is actually a derivative of the exercise of the prerogative conferred by the aforequoted
MR. CONCEPCION. The steps involved in this contest are: First, the ballot boxes are opened constitutional provision. Thus, the subsequent directive in the provision for the Supreme
before teams of three, generally, a representative each of the court, of the protestant and Court to "promulgate its rules for the purpose."
of the "protestee." It is all a questions of how many teams are organized. Of course, that
can be expensive, but it would be expensive whatever court one would choose. There were The conferment of full authority to the Supreme Court, as a PET, is equivalent to the full
times that the Supreme Court, with sometimes 50 teams at the same time working, would authority conferred upon the electoral tribunals of the Senate and the House of
classify the objections, the kind of problems, and the court would only go over the Representatives, i.e., the Senate Electoral Tribunal (SET) and the House of Representatives
objected votes on which the parties could not agree. So it is not as awesome as it would Electoral Tribunal (HRET), which we have affirmed on numerous occasions.6
appear insofar as the Court is concerned. What is awesome is the cost of the revision of the
ballots because each party would have to appoint one representative for every team, and
Next, petitioner still claims that the PET exercises quasi-judicial power and, thus, its members
that may take quite a big amount.
violate the proscription in Section 12, Article VIII of the Constitution, which reads:

MR. SUAREZ. If we draw from the Commissioner's experience which he is sharing with us,
SEC. 12. The Members of the Supreme Court and of other courts established by law shall not
what would be the reasonable period for the election contest to be decided?
be designated to any agency performing quasi-judicial or administrative functions.

MR. CONCEPCION. Insofar as the Supreme Court is concerned, the Supreme Court always
We dispose of this argument as we have done in our Decision, viz.:
manages to dispose of the case in one year.

The traditional grant of judicial power is found in Section 1, Article VIII of the Constitution
MR. SUAREZ. In one year. Thank you for the clarification.5
which provides that the power "shall be vested in one Supreme Court and in such lower
courts as may be established by law." Consistent with our presidential system of
Judicial power granted to the Supreme Court by the same Constitution is plenary. And under government, the function of "dealing with the settlement of disputes, controversies or
the doctrine of necessary implication, the additional jurisdiction bestowed by the last conflicts involving rights, duties or prerogatives that are legally demandable and
8

enforceable" is apportioned to courts of justice. With the advent of the 1987 Constitution, The Constitution which, in Section 17, Article VI, explicitly provides that three Supreme Court
judicial power was expanded to include "the duty of the courts of justice to settle actual Justices shall sit in the Senate and House Electoral Tribunals, respectively, effectively
controversies involving rights which are legally demandable and enforceable, and to exempts the Justices-Members thereof from the prohibition in Section 12, Article VIII. In the
determine whether or not there has been a grave abuse of discretion amounting to lack or same vein, it is the Constitution itself, in Section 4, Article VII, which exempts the Members
excess of jurisdiction on the part of any branch or instrumentality of the Government." The of the Court, constituting the PET, from the same prohibition.
power was expanded, but it remained absolute.
We have previously declared that the PET is not simply an agency to which Members of the
The set up embodied in the Constitution and statutes characterizes the resolution of Court were designated. Once again, the PET, as intended by the framers of the Constitution,
electoral contests as essentially an exercise of judicial power. is to be an institution independent, but not separate, from the judicial department, i.e., the
Supreme Court. McCulloch v. State of Maryland proclaimed that "[a] power without the
At the barangay and municipal levels, original and exclusive jurisdiction over election means to use it, is a nullity." The vehicle for the exercise of this power, as intended by the
contests is vested in the municipal or metropolitan trial courts and the regional trial courts, Constitution and specifically mentioned by the Constitutional Commissioners during the
respectively. discussions on the grant of power to this Court, is the PET. Thus, a microscopic view, like the
petitioner's, should not constrict an absolute and constitutional grant of judicial power.7
At the higher levels - city, provincial, and regional, as well as congressional and senatorial -
exclusive and original jurisdiction is lodged in the COMELEC and in the House of Finally, petitioner’s application of our decision in Biraogo v. Philippine Truth Commission8 to
Representatives and Senate Electoral Tribunals, which are not, strictly and literally speaking, the present case is an unmitigated quantum leap.
courts of law. Although not courts of law, they are, nonetheless, empowered to resolve
election contests which involve, in essence, an exercise of judicial power, because of the The decision therein held that the PTC "finds justification under Section 17, Article VII of the
explicit constitutional empowerment found in Section 2(2), Article IX-C (for the COMELEC) Constitution." A plain reading of the constitutional provisions, i.e., last paragraph of Section 4
and Section 17, Article VI (for the Senate and House Electoral Tribunals) of the Constitution. and Section 17, both of Article VII on the Executive Branch, reveals that the two are
Besides, when the COMELEC, the HRET, and the SET decide election contests, their decisions differently worded and deal with separate powers of the Executive and the Judicial Branches
are still subject to judicial review - via a petition for certiorari filed by the proper party - if of government. And as previously adverted to, the basis for the constitution of the PET was,
there is a showing that the decision was rendered with grave abuse of discretion tantamount in fact, mentioned in the deliberations of the Members of the Constitutional Commission
to lack or excess of jurisdiction. during the drafting of the present Constitution.

It is also beyond cavil that when the Supreme Court, as PET, resolves a presidential or vice- WHEREFORE, the Motion for Reconsideration is DENIED. Our Decision in G.R. No. 191618
presidential election contest, it performs what is essentially a judicial power. In the landmark STANDS.
case of Angara v. Electoral Commission, Justice Jose P. Laurel enucleated that "it would be
inconceivable if the Constitution had not provided for a mechanism by which to direct the SO ORDERED.
course of government along constitutional channels." In fact, Angara pointed out that "[t]he
Constitution is a definition of the powers of government." And yet, at that time, the 1935
Constitution did not contain the expanded definition of judicial power found in Article VIII,
Section 1, paragraph 2 of the present Constitution.

With the explicit provision, the present Constitution has allocated to the Supreme Court, in
conjunction with latter's exercise of judicial power inherent in all courts, the task of deciding
presidential and vice-presidential election contests, with full authority in the exercise
thereof. The power wielded by PET is a derivative of the plenary judicial power allocated to
courts of law, expressly provided in the Constitution. On the whole, the Constitution draws a
thin, but, nevertheless, distinct line between the PET and the Supreme Court.

If the logic of petitioner is to be followed, all Members of the Court, sitting in the Senate and
House Electoral Tribunals would violate the constitutional proscription found in Section 12,
Article VIII. Surely, the petitioner will be among the first to acknowledge that this is not so.
9

Macalintal vs PET, GR 191618, June 7, 2011 presidential and vice-presidential elections contests includes the means necessary to carry it
into effect.
Posted by Pius Morados on November 13, 2011
No. The traditional grant of judicial power is found in Section 1, Article VIII of the Constitution
(Admin Law, PET, Quasi-judicial power) which provides that the power “shall be vested in one Supreme Court and in such lower
courts as may be established by law.” The set up embodied in the Constitution and statutes
Facts:
characterize the resolution of electoral contests as essentially an exercise of judicial power.
When the Supreme Court, as PET, resolves a presidential or vice-presidential election
Par 7, Sec 4, Art VII of the 1987 Constitution provides: “The Supreme Court, sitting en banc,
contest, it performs what is essentially a judicial power.
shall be the sole judge of all contests relating to the election, returns, and qualifications of
the President or Vice-President, and may promulgate its rules for the purpose.”
The COMELEC, HRET and SET are not, strictly and literally speaking, courts of law. Although
not courts of law, they are, nonetheless, empowered to resolve election contests which
Sec 12, Art. VIII of the Constitution provides: The Members of the Supreme Court and of
involve, in essence, an exercise of judicial power, because of the explicit constitutional
other courts established by law shall not be designated to any agency performing quasi-
empowerment found in Section 2(2), Article IX-C (for the COMELEC) and Section 17, Article VI
judicial or administrative functions.
(for the Senate and House Electoral Tribunals) of the Constitution.
The case at bar is a motion for reconsideration filed by petitioner of the SC’s decision
dismissing the former’s petition and declaring the establishment of the respondent PET as
constitutional.

Petitioner argues that PET is unconstitutional on the ground that Sec 4, Art VII of the
Constitution does not provide for the creation of the PET, and it violates Sec 12, Art VIII of
the Constitution.

The Solicitor General maintains that the constitution of the PET is on firm footing on the basis
of the grant of authority to the Supreme Court to be the sole judge of all election contests for
the President or Vice-President under par 7, Sec 4, Art VII of the Constitution.

Issue:

Whether or not PET is constitutional.

Whether or not PET exercises quasi-judicial power.

Held:

Yes. The explicit reference of the Members of the Constitutional Commission to a


Presidential Electoral Tribunal, with Fr. Joaquin Bernas categorically declaring that in crafting
the last paragraph of Sec. 4, Art VII of the 1987 Constitution, they “constitutionalized what
was statutory.” Judicial power granted to the Supreme Court by the same Constitution is
plenary. And under the doctrine of necessary implication, the additional jurisdiction
bestowed by the last paragraph of Section 4, Article VII of the Constitution to decide
10

OCTOBER TERM, 1996


Syllabus

Syllabus
nity from civil damages litigation arising out of events that occurred before he took office-
cannot be sustained on the basis of precedent. The principal rationale for affording
CLINTON v. JONES Presidents immunity from damages actions based on their official acts-i. e., to enable them
to perform their designated functions effectively without fear that a particular decision may
CERTIORARI TO THE UNITED STATES COURT OF APPEALS FOR THE EIGHTH CIRCUIT give rise to personal liability, see, e. g., Nixon v. Fitzgerald, 457 U. S. 731, 749, 752, and n. 32-
provides no support for an immunity for unofficial conduct. Moreover, immunities for acts
No. 95-1853. Argued January 13, 1997-Decided May 27,1997 clearly within official capacity are grounded in the nature of the function performed, not the
identity of the actor who performed it. Forrester v. White, 484 U. S. 219, 229. The Court is
also unpersuaded by petitioner's historical evidence, which sheds little light on the question
Respondent sued under 42 U. S. C. §§ 1983 and 1985 and Arkansas law to recover damages at issue, and is largely canceled by conflicting evidence that is itself consistent with both the
from petitioner, the current President of the United States, alleging, inter alia, that while he doctrine of Presidential immunity as set forth in Fitzgerald, and rejection of the immunity
was Governor of Arkansas, petitioner made "abhorrent" sexual advances to her, and that her claim in this case. Pp. 692-697.
rejection of those advances led to punishment by her supervisors in the state job she held at
the time. Petitioner promptly advised the Federal District Court that he would file a motion
to dismiss on Presidential immunity grounds, and requested that all other pleadings and (b) The separation-of-powers doctrine does not require federal courts to stay all private
motions be deferred until the immunity issue was resolved. After the court granted that actions against the President until he leaves office. Even accepting the unique importance of
request, petitioner filed a motion to dismiss without prejudice and to toll any applicable the Presidency in the constitutional scheme, it does not follow that that doctrine would be
statutes of limitation during his Presidency. The District Judge denied dismissal on immunity violated by allowing this action to proceed. The doctrine provides a self-executing safeguard
grounds and ruled that discovery could go forward, but ordered any trial stayed until against the encroachment or aggrandizement of one of the three coequal branches of
petitioner's Presidency ended. The Eighth Circuit affirmed the dismissal denial, but reversed Government at the expense of another. Buckley v. Valeo, 424 U. S. 1, 122. But in this case
the trial postponement as the "functional equivalent" of a grant of temporary immunity to there is no suggestion that the Federal Judiciary is being asked to perform any function that
which petitioner was not constitutionally entitled. The court explained that the President, might in some way be described as "executive." Respondent is merely asking the courts to
like other officials, is subject to the same laws that apply to all citizens, that no case had been exercise their core Article III jurisdiction to decide cases and controversies, and, whatever the
found in which an official was granted immunity from suit for his unofficial acts, and that the outcome, there is no possibility that the decision here will curtail the scope of the Executive
rationale for official immunity is inapposite where only personal, private conduct by a Branch's official powers. The Court rejects petitioner's contention that this case-as well as
President is at issue. The court also rejected the argument that, unless immunity is available, the potential additional litigation that an affirmance of the Eighth Circuit's judgment might
the threat of judicial interference with the Executive Branch would violate separation of spawn-may place unacceptable burdens on the President that will hamper the performance
powers. of his official duties. That assertion finds little support either in history, as evidenced by the
paucity of suits against sitting Presidents for their private actions, or in the relatively narrow
compass of the issues raised in this particular case. Of greater significance, it is settled that
Held: the Judiciary may severely burden the Executive Branch by reviewing the legality of the
President's official conduct, see, e. g., Youngstown Sheet & Tube Co. v. Sawyer, 343 U. S. 579,
1. This Court need not address two important constitutional issues not encompassed within and may direct appropriate process to the President himself, see, e. g., United
the questions presented by the certiorari petition: (1) whether a claim comparable to States v. Nixon, 418 U. S. 683. It must follow that the federal courts have power to determine
petitioner's assertion of immunity might succeed in a state tribunal, and (2) whether a court the legality of the President's unofficial conduct. The reasons for rejecting a categorical rule
may compel the President's attendance at any specific time or place. Pp. 689-692. requiring federal courts to stay private actions during the President's term apply as well to a
rule that would, in petitioner's words, require a stay "in all but the most exceptional cases."
2. Deferral of this litigation until petitioner's Presidency ends is not constitutionally required. Pp.697-706.
Pp.692-710.
(c) Contrary to the Eighth Circuit's ruling, the District Court's stay order was not the
(a) Petitioner's principal submission-that in all but the most exceptional cases, the "functional equivalent" of an unconstitutional grant of temporary immunity. Rather, the
Constitution affords the President temporary immu- District Court has broad discretion to stay proceedings as an incident to its power to control
its own docket. See, e. g., Landis v. North American Co., 299 U. S. 248, 254. Moreover, the
potential burdens on the President posed by this litigation are appropriate matters for that
11

court to evaluate in its management of the case, and the high respect owed the Presidency is State of Arkansas. Respondent, Paula Corbin Jones, is a resident of California. In 1991 she
a matter that should inform the conduct of the entire proceeding. Nevertheless, the District lived in Arkansas, and was an employee of the Arkansas Industrial Development Commission.
Court's stay decision was an abuse of discretion because it took no account of the
importance of respondent's interest in bringing the case to trial, and because it was On May 6, 1994, she commenced this action in the United States District Court for the
premature in that there was nothing in the record to enable a judge to assess whether Eastern District of Arkansas by filing a complaint naming petitioner and Danny Ferguson, a
postponement of trial after the completion of discovery would be warranted. Pp.706-708. former Arkansas State Police officer, as defendants. The

(d) The Court is not persuaded of the seriousness of the alleged risks that this decision will * John C. Jeffries, Jr., and Pamela S. Karlan filed a brief for Law Professors as amicus
generate a large volume of politically motivated harassing and frivolous litigation and that curiae urging reversal.
national security concerns might prevent the President from explaining a legitimate need for
a continuance, and has confidence in the ability of federal judges to deal with both concerns.
Christopher A. Hansen and Steven R. Shapiro filed a brief for the American Civil Liberties
If Congress deems it appropriate to afford the President stronger protection, it may respond
Union as amicus curiae urging affirmance.
with legislation. Pp. 708-710.

Briefs of amicus curiae were filed for the Coalition of American Veterans by Laurence A. Elgin;
72 F.3d 1354, affirmed.
and for Law Professors by Ronald D. Rotunda, Albert E. Jenner, Jr., Stephen B. Burbank,
William Cohen, Geoffrey P. Miller, Robert F. Nagel, and Richard Parker complaint alleges two
STEVENS, J., delivered the opinion of the Court, in which REHNQUIST, C. J., and O'CONNOR, federal claims, and two state-law claims over which the federal court has jurisdiction because
SCALIA, KENNEDY, SOUTER, THOMAS, and GINSBURG, JJ., joined. BREYER, J., filed an opinion of the diverse citizenship of the parties.1 As the case comes to us, we are required to assume
concurring in the judgment, post, p. 710. the truth of the detailedbut as yet untested-factual allegations in the complaint.

Robert S. Bennett argued the cause for petitioner. With him on the briefs were Carl S. Rauh, Those allegations principally describe events that are said to have occurred on the afternoon
Alan Kriegel, Amy R. Sabrin, and David A. Strauss. of May 8, 1991, during an official conference held at the Excelsior Hotel in Little Rock,
Arkansas. The Governor delivered a speech at the conference; respondent-working as a state
Acting Solicitor General Dellinger argued the cause for the United States as amicus curiae employee-staffed the registration desk. She alleges that Ferguson persuaded her to leave her
urging reversal. With him on the brief were Assistant Attorney General Hunger, Deputy desk and to visit the Governor in a business suite at the hotel, where he made "abhorrent" 2
Solicitor General Kneedler, Malcolm L. Stewart, and Douglas N. Letter. sexual advances that she vehemently rejected. She further claims that her superiors at work
subsequently dealt with her in a hostile and rude manner, and changed her duties to punish
Gilbert K. Davis argued the cause for respondent. With him on the brief was Joseph her for rejecting those advances. Finally, she alleges that after petitioner was elected
Cammarata. * President, Ferguson defamed her by making a statement to a reporter that implied she had
accepted petitioner's alleged overtures, and that various persons authorized to speak for the
President publicly branded her a liar by denying that the incident had occurred.
JUSTICE STEVENS delivered the opinion of the Court. This case raises a constitutional and a
prudential question concerning the Office of the President of the United States. Respondent,
a private citizen, seeks to recover damages from the current occupant of that office based on Respondent seeks actual damages of $75,000 and punitive damages of $100,000. Her
actions allegedly taken before his term began. The President submits that in all but the most complaint contains four counts. The first charges that petitioner, acting under color of state
exceptional cases the Constitution requires federal courts to defer such litigation until his law, deprived her of rights protected by the Constitution, in violation of Rev. Stat. § 1979, 42
term ends and that, in any event, respect for the office warrants such a stay. Despite the U. S. C. § 1983. The second charges that petitioner and Ferguson engaged in a conspiracy to
force of the arguments supporting the President's submissions, we conclude that they must violate her federal rights, also actionable under federal law. See Rev. Stat. § 1980, 42 U. S. C.
be rejected. § 1985. The third is a state common-law claim for intentional infliction of emotional distress,
grounded primarily on the incident at the hotel. The fourth count, also based on state law, is
for defamation, embracing both the comments allegedly made to the press by Ferguson and
I
the statements of petitioner's agents. Inasmuch as the legal sufficiency of the claims has not
yet been challenged, we assume, without deciding, that each of the four counts states a
Petitioner, William Jefferson Clinton, was elected to the Presidency in 1992, and reelected in cause of action as a matter of law. With the exception of the last charge, which arguably may
1996. His term of office expires on January 20, 2001. In 1991 he was the Governor of the involve conduct within the outer perimeter of the President's official responsibilities, it is
perfectly clear that the alleged misconduct of petitioner was unrelated to any of his official
12

duties as President of the United States and, indeed, occurred before he was elected to that 7 Although, as noted above, the District Court's initial order permitted discovery to go
office.3 forward, the court later stayed discovery pending the outcome of the appeals on the
immunity issue. 879 F. Supp. 86 (ED Ark. 1995).
II
President leaves office as the "functional equivalent" of a grant of temporary immunity, it
In response to the complaint, petitioner promptly advised the District Court that he intended reversed that order. 72 F.3d 1354, 1361, n. 9, 1363 (CA8 1996). Writing for the majority,
to file a motion to dismiss on grounds of Presidential immunity, and requested the court to Judge Bowman explained that "the President, like all other government officials, is subject to
defer all other pleadings and motions until after the immunity issue was resolved.4 Relying the same laws that apply to all other members of our society," id., at 1358, that he could find
on our cases holding that immunity questions should be decided at the earliest possible stage no "case in which any public official ever has been granted any immunity from suit for his
of the litigation, 858 F. Supp. 902, 905 (ED Ark. 1994), our recognition of the "'singular unofficial acts," ibid., and that the rationale for official immunity "is inapposite where only
importance of the President's duties,'" id., at 904 (quoting Nixon v. Fitzgerald, 457 U. S. 731, personal, private conduct by a President is at issue," id., at 1360. The majority specifically
751 (1982)), and the fact that the question did not require any analysis of the allegations of rejected the argument that, unless immunity is available, the threat of judicial interference
the complaint, 858 F. Supp., at 905, the court granted the request. Petitioner thereupon filed with the Executive Branch through scheduling orders, potential contempt citations, and
a motion "to dismiss ... without prejudice and to toll any statutes of limitation [that may be sanctions would violate separation-of-powers principles. Judge Bowman suggested that
applicable] until he is no longer President, at which time the plaintiff may refile the instant "judicial case management sensitive to the burdens of the presidency and the demands of
suit." Record, Doc. No. 17. Extensive submissions were made to the District Court by the the President's schedule" would avoid the perceived danger. Id., at 1361.
parties and the Department of Justice.5
In dissent, Judge Ross submitted that even though the holding in Fitzgerald involved official
The District Judge denied the motion to dismiss on immunity grounds and ruled that acts, the logic of the opinion, which "placed primary reliance on the prospect that the
discovery in the case could go forward, but ordered any trial stayed until the end of President's discharge of his constitutional powers and duties would be impaired if he were
petitioner's Presidency. 869 F. Supp. 690 (ED Ark. 1994). Although she recognized that a "thin subject to suits for damages," applies with equal force to this case. 72 F. 3d, at 1367. In his
majority" in Nixon v. Fitzgerald, 457 U. S. 731 (1982), had held that "the President has view, "unless exigent circumstances can be shown," all private actions for damages against a
absolute immunity from civil damage actions arising out of the execution of official duties of sitting President must be stayed until the completion of his term. Ibid. In this case, Judge
office," she was not convinced that "a President has absolute immunity from civil causes of Ross saw no reason why the stay would prevent respondent from ultimately obtaining an
action arising prior to assuming the office." 6 She was, however, persuaded by some of the adjudication of her claims.
reasoning in our opinion in Fitzgerald that deferring the trial if one were required would be
appropriate.7 869 F. Supp., at 699-700. Relying in part on the fact that respondent had failed In response to the dissent, Judge Beam wrote a separate concurrence. He suggested that a
to bring her complaint until two days before the 3-year period of limitations expired, she prolonged delay may well create a significant risk of irreparable harm to respondent because
concluded that the public interest in avoiding litigation that might hamper the President in of an unforeseeable loss of evidence or the possible death of a party. Id., at 1363-1364.
conducting the duties of his office outweighed any demonstrated need for an immediate Moreover, he argued that in civil rights cases brought under § 1983 there is a "public interest
trial. Id., at 698-699. in an ordinary citizen's timely vindication of ... her most fundamental right against alleged
abuse of power by government officials." Id., at 1365. In his view, the dissent's concern about
Both parties appealed. A divided panel of the Court of Appeals affirmed the denial of the judicial interference with the functioning of the Presidency was "greatly
motion to dismiss, but because it regarded the order postponing the trial until the overstated." Ibid. Neither the involvement of prior Presidents in litigation, either as parties or
as witnesses, nor the character of this "relatively uncomplicated civil litigation," indicated
that the threat was serious. Id., at 1365-1366. Finally, he saw "no basis for staying discovery
5 See App. to Pet. for Cert. 53.
or trial of the claims against Trooper Ferguson." Id., at 1366.8

6869 F. Supp., at 698. She explained: "Nowhere in the Constitution, congressional acts, or the
III
writings of any judge or scholar, may any credible support for such a proposition be found. It
is contrary to our form of government, which asserts as did the English in the Magna Carta
and the Petition of Right, that even the sovereign is subject to God and the law." Ibid. The President, represented by private counsel, filed a petition for certiorari. The Acting
Solicitor General, representing the United States, supported the petition, arguing that the
decision of the Court of Appeals was "fundamentally mistaken" and created "serious risks for
the institution of the Presidency." 9 In her brief in opposition to certiorari, respondent
argued that this "one-of-a-kind case is singularly inappropriate" for the exercise of our
13

certiorari jurisdiction because it did not create any conflict among the Courts of Appeals, it 1442(a); Mesa v. California, 489 U. S. 121, 125-126 (1989). Whether those concerns would
"does not pose any conceivable threat to the functioning of the Executive Branch," and there present a more compelling case for immunity is a question that is not before us.
is no precedent supporting the President's position.10
Second, our decision rejecting the immunity claim and allowing the case to proceed does not
While our decision to grant the petition, 518 U. S. 1016 (1996), expressed no judgment require us to confront the question whether a court may compel the attendance of the
concerning the merits of the case, it does reflect our appraisal of its importance. The President at any specific time or place. We assume that the testimony of the President, both
representations made on behalf of the Executive Branch as to the potential impact of the for discovery and for use at trial, may be taken at the White House at a time that
precedent established by the Court of Appeals merit our respectful and deliberate
consideration. as a proper exercise of judicial discretion, may stay such litigation until the President leaves
office." Our review is confined to these issues. See this Court's Rule 14.1(a).
It is true that we have often stressed the importance of avoiding the premature adjudication
of constitutional questions.ll That doctrine of avoidance, however, is applicable to the entire 13 Because the Supremacy Clause makes federal law "the supreme Law of the Land," Art. VI,
Federal Judiciary, not just to this Court, cf. Arizonansfor Official English v. Arizona, ante, p. cl. 2, any direct control by a state court over the President, who has principal responsibility to
43, and comes into play after the court has acquired jurisdiction of a case. It does not dictate ensure that those laws are "faithfully executed," Art. II, § 3, may implicate concerns that are
a discretionary denial of every certiorari petition raising a novel constitutional question. It quite different from the interbranch separation-of-powers questions addressed here.
does, however, make it appropriate to identify two important constitutional issues not Cf., e. g., Hancock v. Train, 426 U. S. 167, 178-179 (1976); Mayo v. United States, 319 U. S.
encompassed within the questions presented by the petition for certiorari that we need not 441, 445 (1943). See L. Tribe, American Constitutional Law 513 (2d ed. 1988) ("[A]bsent
address today. 12 explicit congressional consent no state may command federal officials ... to take action in
derogation of their ... federal responsibilities") will accommodate his busy schedule, and that,
11 As we have explained: '''If there is one doctrine more deeply rooted than any other in the if a trial is held, there would be no necessity for the President to attend in person, though he
process of constitutional adjudication, it is that we ought not to pass on questions of could elect to do SO.14
constitutionality ... unless such adjudication is unavoidable.' Spector Motor
Service v. McLaughlin, 323 U. S. 101, 105 [(1944)]. It has long been the Court's 'considered IV
practice not to decide abstract, hypothetical or contingent questions ... or to decide any
constitutional question in advance of the necessity for its decision ... or to formulate a rule of
Petitioner's principal submission-that "in all but the most exceptional cases," Brief for
constitutional law broader than is required by the precise facts to which it is to be applied ...
Petitioner i, the Constitution affords the President temporary immunity from civil damages
or to decide any constitutional question except with reference to the particular facts to
litigation arising out of events that occurred before he took office-cannot be sustained on the
which it is to be applied ... .' Alabama State Federation of Labor v. McAdory, 325 U. S. 450,
basis of precedent.
461 [(1945)]. 'It is not the habit of the court to decide questions of a constitutional nature
unless absolutely necessary to a decision of the case.' Burton v. United States, 196 U. S. 283,
295 [(1905)]." Rescue Army v. Municipal Court of Los Angeles, 331 U. S. 549, 570, n. 34 Only three sitting Presidents have been defendants in civil litigation involving their actions
(1947). prior to taking office. Complaints against Theodore Roosevelt and Harry Truman had been
dismissed before they took office; the dismissals were affirmed after their respective
inaugurations.15 Two companion cases arising out of an automobile accident were filed
12The two questions presented in the certiorari petition are: "1. Whether the litigation of a
against John F. Kennedy in 1960 during the Presidential campaign.16 After taking office, he
private civil damages action against an incumbent President must in all but the most
unsuccessfully argued that his status as Commander in Chief gave him a right to a stay under
exceptional cases be deferred until the President leaves office"; and "2. Whether a district
the Soldiers' and Sailors' Civil Relief Act of 1940, 50 U. S. C. App. §§ 501-525. The motion for a
court, First, because the claim of immunity is asserted in a federal court and relies heavily on
stay was denied by the District Court, and the matter was settled out of court.17 Thus, none
the doctrine of separation of powers that restrains each of the three branches of the Federal
of those cases sheds any light on the constitutional issue before us.
Government from encroaching on the domain of the other two, see, e. g.,
Buckley v. Valeo, 424 U. S. 1, 122 (1976) (per curiam), it is not necessary to consider or decide
whether a comparable claim might succeed in a state tribunal. If this case were being heard The principal rationale for affording certain public servants immunity from suits for money
in a state forum, instead of advancing a separation-of-powers argument, petitioner would damages arising out of their official acts is inapplicable to unofficial conduct. In cases
presumably rely on federalism and comity concerns, 13 as well as the interest in protecting involving prosecutors, legislators, and judges we have repeatedly explained that the
federal officials from possible local prejudice that underlies the authority to remove certain immunity serves the public interest in enabling such officials to perform their designated
cases brought against federal officers from a state to a federal court, see 28 U. S. C. §
14

functions effectively without fear that a particular decision may give rise to personal liability. Petitioner draws our attention to dicta in Fitzgerald, which he suggests are helpful to his
18 We explained in Ferri v. Ackerman, 444 U. S. 193 (1979): cause. We noted there that "[b]ecause of the singular importance of the President's duties,
diversion of his energies by concern with private lawsuits would raise unique risks to the
"As public servants, the prosecutor and the judge represent the interest of society as a effective functioning of government," 457 U. S., at 751, and suggested further that
whole. The conduct of their official duties may adversely affect a wide variety of different "[c]ognizance of ... personal vulnerability frequently could distract a President from his public
individuals, each of whom may be a potential source of future controversy. The societal duties," id., at 753. Petitioner argues that in this aspect the Court's concern was parallel to
interest in providing such public officials with the maximum ability to deal fearlessly and the issue he suggests is of great importance in this case, the possibility that a sitting
impartially with the public at large has long been recognized as an acceptable justification for President might be distracted by the need to participate in litigation during the pendency of
official immunity. The point of immunity for such officials is to forestall an atmosphere of his office. In context, however, it is clear that our dominant concern was with the diversion of
intimidation that would conflict with their resolve to perform their designated functions in a the President's attention during the decisionmaking process caused by needless worry as to
principled fashion." Id., at 202-204. the possibility of damages actions stemming from any particular official decision.
Moreover, Fitzgerald did not present the issue raised in this case because that decision
involved claims against a former President.
That rationale provided the principal basis for our holding that a former President of the
United States was "entitled to absolute immunity from damages liability predicated on his
official acts," Fitzgerald, 457 U. S., at 749. See id., at 752 (citing Ferri v. Ackerman). Our Petitioner's effort to construct an immunity from suit for unofficial acts grounded purely in
central concern was to avoid rendering the President "unduly cautious in the discharge of his the identity of his office is unsupported by precedent.
official duties." 457 U. S., at 752, n. 32.19
V
18 Some of these cases defined the immunities of state and local officials in actions filed
under 42 U. S. C. § 1983. See, e. g., Imbler v. Pachtman, 424 U. S. 409,422-423 (1976) We are also unpersuaded by the evidence from the historical record to which petitioner has
(prosecutorial immunity); Tenney v. Brandhove, 341 U. S. 367, 376-377 (1951) (legislative called our attention. He points to a comment by Thomas Jefferson protesting the
immunity); Pierson v. Ray, 386 U. S. 547, 554-555 (1967) (judicial immunity). The rationale subpoena duces tecum Chief Justice Marshall directed to him in the Burr trial,20 a statement
underlying our official immunity jurisprudence in cases alleging constitutional violations in the diaries kept by Senator William Maclay of the first Senate debates, in which thenVice
brought against federal officials is similar. See, e. g., Butz v. Economou, 438 U. S. 478, 500- President John Adams and Senator Oliver Ellsworth are recorded as having said that "the
501 (1978). President personally [is] not ... subject to any process whatever," lest it be "put ... in the
power of a common Justice to exercise any Authority over him and Stop the Whole Machine
This reasoning provides no support for an immunity for unofficial conduct. As we explained of Government," 21 and to a quotation from Justice Story's Commentaries on the
in Fitzgerald, "the sphere of protected action must be related closely to the immunity's Constitution.22 None of these sources sheds much light on the question at hand.23
justifying purposes." Id., at 755. Because of the President's broad responsibilities, we
recognized in that case an immunity from damages claims arising out of official acts 20 In Jefferson's view, the subpoena jeopardized the separation of powers by subjecting the
extending to the "outer perimeter of his authority." Id., at 757. But we have never suggested Executive Branch to judicial command. See 10 Works of Thomas Jefferson 404, n. (P. Ford ed.
that the President, or any other official, has an immunity that extends beyond the scope of 1905); Fitzgerald, 457 U. S., at 751,
any action taken in an official capacity. See id., at 759 (Burger, C. J., concurring) (noting that
"a President, like Members of Congress, judges, prosecutors, or congressional aides-all 219 Documentary History of First Federal Congress of the United States 168 (K. Bowling & H.
having absolute immunity-are not immune for acts outside official duties"); see also id., at Veit eds. 1988) (Diary of William Maclay).
761, n. 4.
22 See 3 J. Story, Commentaries on the Constitution of the United States § 1563, pp. 418-419
(1833).
Moreover, when defining the scope of an immunity for acts clearly taken within an official
capacity, we have applied a functional approach. "Frequently our decisions have held that an
official's absolute immunity should extend only to acts in performance of particular functions 23Jefferson's argument provides little support for petitioner's position.
of his office." Id., at 755. Hence, for example, a judge's absolute immunity does not extend to
actions performed in a purely administrative capacity. See Forrester v. White, 484 U. S. 219, As we explain later, the prerogative Jefferson claimed was denied him by the Chief Justice in
229-230 (1988). As our opinions have made clear, immunities are grounded in "the nature of the very decision Jefferson was protesting, and this Court has subsequently reaffirmed that
the function performed, not the identity of the actor who performed it." Id., at 229. holding. See United States v. Nixon, 418 U. S. 683 (1974). The statements supporting a similar
15

proposition recorded in Senator Maclay's diary are inconclusive of the issue before us here VI
for the same reason. In addition, this material is hardly proof of the unequivocal common
understanding at the time of the founding. Immediately after mentioning the positions of Petitioner's strongest argument supporting his immunity claim is based on the text and
Adams and Ellsworth, structure of the Constitution. He does not contend that the occupant of the Office of the
President is "above the law," in the sense that his conduct is entirely immune from judicial
Respondent, in turn, has called our attention to conflicting historical evidence. Speaking in scrutiny.24 The President argues merely for a postponement of the judicial proceedings that
favor of the Constitution's adoption at the Pennsylvania Convention, James Wilsonwho had will determine whether he violated any law. His argument is grounded in the character of the
participated in the Philadelphia Convention at which the document was drafted-explained office that was created by Article II of the Constitution, and relies on separation-of-powers
that, although the President "is placed [on] high," "not a single privilege is annexed to his principles that have structured our constitutional arrangement since the founding.
character; far from being above the laws, he is amenable to them in his private character as a
citizen, and in his public character by impeachment." 2 J. Elliot, Debates on the Federal As a starting premise, petitioner contends that he occupies a unique office with powers and
Constitution 480 (2d ed. 1863) (emphasis deleted). This description is consistent with both responsibilities so vast and important that the public interest demands that he devote his
the doctrine of Presidential immunity as set forth in Fitzgerald and rejection of the immunity undivided time and attention to his public duties. He submits that-given the nature of the
claim in this case. With respect to acts taken in his "public character"-that is, official actsthe office-the doctrine of separation of powers places limits on the authority of the Federal
President may be disciplined principally by impeachment, not by private lawsuits for Judiciary to interfere with the Executive Branch that would be transgressed by allowing this
damages. But he is otherwise subject to the laws for his purely private acts. action to proceed.

In the end, as applied to the particular question before us, we reach the same conclusion 24 For that reason, the argument does not place any reliance on the English ancestry that
about these historical materials that Justice Jackson described when confronted with an issue informs our common-law jurisprudence; he does not claim the prerogatives of the monarchs
concerning the dimensions of the President's power. who asserted that "[t]he King can do no wrong." See 1 W. Blackstone, Commentaries *246.
Although we have adopted the related doctrine of sovereign immunity, the common-law
Maclay went on to point out in his diary that he virulently disagreed with them, concluding fiction that "[t]he king ... is not only incapable of doing wrong, but even
that his opponents' view "[s]hows clearly how amazingly fond of the old leven many People of thinking wrong," ibid., was rejected at the birth of the Republic. See, e. g.,
are." Diary of Maclay 168. Nevada v. Hall, 440 U. S. 410, 415, and nn. 7-8 (1979); Langford v. United States, 101 U. S.
341, 342-343 (1880).
Finally, Justice Story's comments in his constitutional law treatise provide no substantial
support for petitioner's position. Story wrote that because the President's "incidental We have no dispute with the initial premise of the argument. Former Presidents, from
powers" must include "the power to perform [his duties], without any obstruction," he George Washington to George Bush, have consistently endorsed petitioner's characterization
"cannot, therefore, be liable to arrest, imprisonment, or detention, while he is in the of the office.25 After serving his term, Lyndon Johnson observed: "Of all the 1,886 nights I
discharge of the duties of his office; andfor this purpose his person must be deemed, in civil was President, there were not many when I got to sleep before 1 or 2 a.m., and there were
cases at least, to possess an official inviolability." 3 Story § 1563, at 418-419 (emphasis few mornings when I didn't wake up by 6 or 6:30."26 In 1967, the Twenty-fifth Amendment
added). Story said only that "an official inviolability," ibid. (emphasis added), was necessary to the Constitution was adopted to ensure continuity in the performance of the powers and
to preserve the President's ability to perform the functions of the office; he did not specify duties of the office; 27 one of the sponsors of that Amendment stressed the importance of
the dimensions of the necessary immunity. While we have held that an immunity from suits providing that "at all times" there be a President "who has complete control and will be able
grounded on official acts is necessary to serve this purpose, see Fitzgerald, 457 U. S., at 749, to perform" those duties.28 As Justice Jackson has pointed out, the Presidency concentrates
it does not follow that the broad immunity from all civil damages suits that petitioner seeks is executive authority "in a single head in whose choice the whole Nation has a part, making
also necessary. him the focus of public hopes and expectations. In drama, magnitude and finality his
decisions so far overshadow any others that almost alone he fills the public eye and
"Just what our forefathers did envision, or would have envisioned had they foreseen modern ear." Youngstown Sheet & Tube Co. v. Sawyer, 343 U. S., at 653 (concurring opinion). We
conditions, must be divined from materials almost as enigmatic as the dreams Joseph was have, in short, long recognized the "unique position in the constitutional scheme" that this
called upon to interpret for Pharoah. A century and a half of partisan debate and scholarly office occupies. Fitzgerald,
speculation yields no net result but only supplies more or less apt quotations from respected
sources on each side .... They largely cancel each other." Youngstown Sheet & Tube 25 See, e. g., A. Tourtellot, The Presidents on the Presidency 346-374 (1964) (citing
Co. v. Sawyer, 343 U. S. 579, 634-635 (1952) (concurring opinion). comments of, among others, George Washington, John Quincy Adams, Benjamin Harrison,
Theodore Roosevelt, William Howard Taft, and Woodrow Wilson); H. Finer, The Presidency:
16

Crisis and Regeneration 35-37 (1960) (citing similar remarks by a number of Presidents, of purely advisory opinions to the Executive,33 or permit the federal courts to resolve
including James Monroe, James K. Polk, and Harry Truman). nonjusticiable questions.34

457 U. S., at 749.29 Thus, while we suspect that even in our modern era there remains some 31 See also United States v. Klein, 13 Wall. 128, 147 (1872) (noting that Congress had
truth to Chief Justice Marshall's suggestion that the duties of the Presidency are not entirely "inadvertently passed the limit which separates the legislative from the judicial power").
"unremitting," United States v. Burr, 25 F. Cas. 30, 34 (No. 14,692d) (CC Va. 1807), we accept
the initial premise of the Executive's argument. 32 See also Bowsher v. Synar, 478 U. S. 714, 726 (1986) ("structure of the Constitution does
not permit Congress to execute the laws"). Cf. INS v. Chadha, 462 U. S. 919, 958
It does not follow, however, that separation-of-powers principles would be violated by (1983); Springer v. Philippine Islands, 277 U. S. 189, 202-203 (1928).
allowing this action to proceed. The doctrine of separation of powers is concerned with the
allocation of official power among the three coequal branches of our Government. The 33 See United States v. Ferreira, 13 How. 40 (1852); Hayburn's Case, 2 Dall. 409 (1792). As we
Framers "built into the tripartite Federal Government ... a self-executing safeguard against explained in Chicago & Southern Air Lines, Inc. v. Waterman S. S. Corp., 333 U. S. 103, 113
the encroachment or aggrandizement of one branch at the expense of the (1948): "This Court early and wisely determined that it would not give advisory opinions even
other." Buckley v. Valeo, 424 U. S., at 122.30 Thus, for example, the Congress may not when asked by the Chief Executive." More generally, "we have broadly stated that 'executive
exercise the judicial power to revise final judgments, Plaut v. Spendthrift or administrative duties of a nonjudicial nature may not be imposed on judges holding office
under Art. III of the Constitution.'" Morrison v. Olson, 487 U. S. 654, 677 (1988)
29We noted in Fitzgerald: "Article II, § 1, of the Constitution provides that '[t]he executive (quoting Buckley v. Valeo, 424 U. S. 1, 123 (1976) (per curiam)). These restrictions on judicial
Power shall be vested in a President of the United States ... .' This grant of authority activities "help ensure the independence of the Judicial Branch and to prevent the Judiciary
establishes the President as the chief constitutional officer of the Executive Branch, from encroaching into areas reserved for the other branches." 487 U. S., at 678; see
entrusted with supervisory and policy responsibilities of utmost discretion and sensitivity. also Mistretta v. United States, 488 U. S., at 385.
These include the enforcement of federal law-it is the President who is charged
constitutionally to 'take Care that the Laws be faithfully executed'; the conduct of foreign 34 We have long held that the federal courts may not resolve such matters. See, e. g.,
affairs-a realm in which the Court has recognized that '[i]t would be intolerable that courts, Luther v. Borden, 7 How. 1 (1849). As we explained in Nixon v. United States, 506 U. S.
without the relevant information, should review and perhaps nullify actions of the Executive 224 (1993): "A controversy is nonjusticiable-i. e., involves a political question-where there is a
taken on information properly held secret'; and management of the Executive Brancha task 'textually demonstrable constitutional commitment of the issue to a coordinate political
for which 'imperative reasons requir[e] an unrestricted power [in the President] to remove department; or a lack of judicially discoverable and manageable standards for resolving it ...
the most important of his subordinates in their most important duties.''' 457 U. S., at 749-750 .' Baker v. Carr, 369 U. S. 186,217 (1962). But the courts must, in the first instance, interpret
(footnotes omitted). the text in question and determine whether and to what extent the issue is textually
committed. See ibid.; Powell v. McCormack, 395 U. S. 486, 519 (1969)." Id., at 228.
30 See Loving v. United States, 517 U. S. 748,756-757 (1996); Mistretta v. United States, 488
U. S. 361, 382 (1989) ("[C]oncern of encroachment and aggrandizement ... has animated our
separation-of-powers jurisprudence"); The Federalist No. 51, p. 349 (J. Cooke ed. 1961)
("[T]he great security against a gradual concentration of the several powers in the same
Of course the lines between the powers of the three branches are not always neatly defined.
department, consists in giving to those who administer each department the necessary
See Mistretta v. United States, 488 U. S. 361, 380-381 (1989).35 But in this case there is no
constitutional means, and personal motives, to resist encroachments of the others").
suggestion that the Federal Judiciary is being asked to perform any function that might in
some way be described as "executive." Respondent is merely asking the courts to exercise
Farm, Inc., 514 U. S. 211 (1995),31 or the executive power to manage an airport, their core Article III jurisdiction to decide cases and controversies. Whatever the outcome of
see Metropolitan Washington Airports Authority v. Citizens for Abatement of Aircraft Noise, this case, there is no possibility that the decision will curtail the scope of the official powers
Inc., 501 U. S. 252, 276 (1991) (holding that "[i]f the power is executive, the Constitution does of the Executive Branch. The litigation of questions that relate entirely to the unofficial
not permit an agent of Congress to exercise it").32 See J. W Hampton, Jr., & Co. v. United conduct of the individual who happens to be the President poses no perceptible risk of
States, 276 U. S. 394, 406 (1928) (Congress may not "invest itself or its members with either misallocation of either judicial power or executive power.
executive power or judicial power"). Similarly, the President may not exercise the legislative
power to authorize the seizure of private property for public use. Youngstown, 343 U. S., at
Rather than arguing that the decision of the case will produce either an aggrandizement of
588. And, the judicial power to decide cases and controversies does not include the provision
judicial power or a narrowing of executive power, petitioner contends that-as a byproduct of
17

an otherwise traditional exercise of judicial power-burdens will be placed on the President example of such a case is our holding that President Truman exceeded his constitutional
that will hamper the performance of his official duties. We have recognized that "[e]ven authority when he issued an order directing the Secretary of Commerce to take possession of
when a branch does not arrogate power to itself ... the separation-of-powers doctrine and operate most of the Nation's steel mills in order to avert a national
requires that a branch not impair another in the performance of its constitutional catastrophe. Youngstown Sheet & Tube Co. v. Sawyer, 343 U. S. 579 (1952). Despite the
duties." Loving v. United States, 517 U. S. 748, 757 (1996); see also Nixon v. Administrator of serious impact of that decision on the ability of the Executive Branch to accomplish its
General Services, 433 U. S. 425, 443 (1977). As a factual matter, petitioner contends that this assigned mission, and the substantial time that the President must necessarily have devoted
particular case-as well as the potential additional litigation that an affirmance of the Court of to the matter as a result of judicial involvement, we exercised our Article III jurisdiction to
Appeals judgment might spawn-may impose an unacceptable burden on the President's time decide whether his official conduct conformed to the law. Our holding was an application of
and energy, and thereby impair the effective performance of his office. the principle established in Marbury v. Madison, 1 Cranch 137 (1803), that "[i]t is
emphatically the province and duty of the judicial department to say what the law is." Id., at
Petitioner's predictive judgment finds little support in either history or the relatively narrow 177.
compass of the issues raised in this particular case. As we have already noted, in the more
than 200- year history of the Republic, only three sitting Presidents have been subjected to Second, it is also settled that the President is subject to judicial process in appropriate
suits for their private actions.36 See supra, at 692. If the past is any indicator, it seems circumstances. Although Thomas Jefferson apparently thought otherwise, Chief Justice
unlikely that a deluge of such litigation will ever engulf the Presidency. As for the case at Marshall, when presiding in the treason trial of Aaron Burr, ruled that a subpoena duces
hand, if properly managed by the District Court, it appears to us highly unlikely to occupy any tecum could be directed to the President. United States v. Burr, 25 F. Cas. 30 (No. 14,692d)
substantial amount of petitioner's time. (CC Va. 1807).38 We unequivocally and emphatically endorsed Marshall's position when we
held that President Nixon was obligated to comply with a subpoena commanding him to
Of greater significance, petitioner errs by presuming that interactions between the Judicial produce certain tape recordings of his conversations with his aides. United States v. Nixon,
Branch and the Executive, even quite burdensome interactions, necessarily rise to the level 418 U. S. 683 (1974). As we explained, "neither the doctrine of separation of powers, nor the
of constitutionally forbidden impairment of the Executive's ability to perform its need for confidentiality of high-level communications, without more, can sustain an
constitutionally mandated functions. "[O]ur ... system imposes upon the Branches a degree absolute, unqualified Presidential privilege of immunity from judicial process under all
of overlapping responsibility, a duty of interdependence as well as independence the circumstances." Id., at 706.39
absence of which 'would preclude the establishment of a Nation capable of governing itself
effectively.'" Mistretta, 488 U. S., at 381 (quoting Buckley, 424 U. S., at 121). As Madison Sitting Presidents have responded to court orders to provide testimony and other
explained, separation of powers does not mean that the branches "ought to have no partial information with sufficient frequency that such interactions between the Judicial and
agency in, or no controul over the acts of each other." 37 The fact that a federal court's Executive Branches can scarcely be thought a novelty. President Monroe responded to
exercise of its traditional Article III jurisdiction may significantly burden the time and written interrogatories, see Rotunda, Presidents and Ex-Presidents as Witnesses: A Brief
attention of the Chief Executive is not sufficient to establish a violation of the Constitution. Historical Footnote, 1975 U. Ill. L. Forum 1, 5-6, President Nixonas noted above-produced
Two long-settled propositions, first announced by Chief Justice Marshall, support that tapes in response to a subpoena
conclusion.
38 After the decision was rendered, Jefferson expressed his distress in a letter to a
36 In Fitzgerald, we were able to discount the lack of historical support for the proposition prosecutor at the trial, noting that "[t]he Constitution enjoins [the President's] constant
that official-capacity actions against the President posed a serious threat to the office on the agency in the concerns of 6. millions of people." 10 Works of Thomas Jefferson 404, n. (P.
ground that a right to sue federal officials for damages as a result of constitutional violations Ford ed. 1905). He asked: "Is the law paramount to this, which calls on him on behalf of a
had only recently been recognized. See 457 U. S., at 753, n. 33; Bivens v. Six Unknown Fed. single one?" Ibid.; see also Fitzgerald, 457 U. S., at 751-752, n. 31 (quoting Jefferson's
Narcotics Agents, 403 U. S. 388 (1971). The situation with respect to suits against the comments at length). For Chief Justice Marshall, the answer-quite plainly-was yes.
President for actions taken in his private capacity is quite different because such suits may be
grounded on legal theories that have always been applicable to any potential defendant. 39 Of course, it does not follow that a court may "'proceed against the president as against
Moreover, because the President has contact with far fewer people in his private life than in an ordinary individual,'" United States v. Nixon, 418 U. S., at 715 (quoting United
his official capacity, the class of potential plaintiffs is considerably smaller and the risk of States v. Burr, 25 F. Cas. 187, 192 (No. 14,694) (CC Va. 1807)). Special caution is appropriate if
litigation less intense. the materials or testimony sought by the court relate to a President's official activities, with
respect to which "[t]he interest in preserving confidentiality is weighty indeed and entitled to
First, we have long held that when the President takes official action, the Court has the great respect." 418 U. S., at 712. We have made clear that in a criminal case the powerful
authority to determine whether he has acted within the law. Perhaps the most dramatic interest in the "fair administration of criminal justice" requires that the evidence be given
18

under appropriate circumstances lest the "very integrity of the judicial system" be The Court of Appeals described the District Court's discretionary decision to stay the trial as
eroded. Id., at 709, 711-712. the "functional equivalent" of a grant of temporary immunity. 72 F. 3d, at 1361, n. 9.
Concluding that petitioner was not constitutionally entitled to such an immunity, the court
duces tecum, see United States v. Nixon, President Ford complied with an order to give a held that it was error to grant the stay. Ibid. Although we ultimately conclude that the stay
deposition in a criminal trial, United States v. Fromme, 405 F. Supp. 578 (ED Cal. 1975), and should not have been granted, we think the issue is more difficult than the opinion of the
President Clinton has twice given videotaped testimony in criminal proceedings, see United Court of Appeals suggests.
States v. McDougal, 934 F. Supp. 296 (ED Ark. 1996); United States v. Branscum, No. LRP-CR-
96-49 (ED Ark., June 7, 1996). Moreover, sitting Presidents have also voluntarily complied Strictly speaking the stay was not the functional equivalent of the constitutional immunity
with judicial requests for testimony. President Grant gave a lengthy deposition in a criminal that petitioner claimed, because the District Court ordered discovery to proceed. Moreover,
case under such circumstances, 1 R. Rotunda & J. Nowak, Treatise on Constitutional Law § a stay of either the trial or discovery might be justified by considerations that do not require
7.1 (2d ed. 1992), and President Carter similarly gave videotaped testimony for use at a the recognition of any constitutional immunity. The District Court has broad discretion to
criminal trial, id., § 7.1(b) (Supp. 1997). stay proceedings as an incident to its power to control its own docket.
See, e. g., Landis v. North American Co., 299 U. S. 248, 254 (1936). As we have explained,
In sum, "[i]t is settled law that the separation-of-powers doctrine does not bar every exercise "[e]specially in cases of extraordinary public moment, [a plaintiff] may be required to submit
of jurisdiction over the President of the United States." Fitzgerald, 457 U. S., at 753-754. If to delay not immoderate in extent and not oppressive in its consequences if the public
the Judiciary may severely burden the Executive Branch by reviewing the legality of the welfare or convenience will thereby be promoted." Id., at 256. Although we have rejected
President's official conduct, and if it may direct appropriate process to the President himself, the argument that the potential burdens on the President violate separation-of-powers
it must follow that the federal courts have power to determine the legality of his unofficial principles, those burdens are appropriate matters for the District Court to evaluate in its
conduct. The burden on the President's time and energy that is a mere byproduct of such management of the case. The high respect that is owed to the office of the Chief Executive,
review surely cannot be considered as onerous as the direct burden imposed by judicial though not justifying a rule of categorical immunity, is a matter that should inform the
review and the occasional invalidation of his official actions.40 We therefore hold that the conduct of the entire proceeding, including the timing and scope of discovery.41
doctrine of separation of powers does not require federal courts to stay all private actions
against the President until he leaves office. Nevertheless, we are persuaded that it was an abuse of discretion for the District Court to
defer the trial until after the President leaves office. Such a lengthy and categorical stay takes
40 There is, no doubt, some truth to Learned Hand's comment that a lawsuit should be no account whatever of the respondent's interest in bringing the case to trial. The complaint
"dread[ed] ... beyond almost anything else short of sickness and death." 3 Association of the was filed within the statutory limitations period-albeit near the end of that period-and
Bar of the City of New York, Lectures on Legal Topics 105 (1926). We recognize that a delaying trial would increase the danger of prejudice resulting from the loss of evidence,
President, like any other official or private citizen, may become distracted or preoccupied by including the inability of witnesses to recall specific facts, or the possible death of a party.
pending litigation. Presidents and other officials face a variety of demands on their time,
however, some private, some political, and some as a result of official duty. While such 41 Although these claims are in fact analytically distinct, the District Court does not appear to
distractions may be vexing to those subjected to them, they do not ordinarily implicate have drawn that distinction. Rather than basing its decision on particular factual findings that
constitutional separation-of-powers concerns. might have buttressed an exercise of discretion, the District Court instead suggested that a
discretionary stay was supported by the legal conclusion that such a stay was required
The reasons for rejecting such a categorical rule apply as well to a rule that would require a by Fitzgerald. See 869 F. Supp., at 699. We therefore reject petitioner's argument that we
stay "in all but the most exceptional cases." Brief for Petitioner i. Indeed, if the Framers of lack jurisdiction over respondent's cross-appeal from the District Court's alternative holding
the Constitution had thought it necessary to protect the President from the burdens of that its decision was "also permitted," inter alia, "under the equity powers of the
private litigation, we think it far more likely that they would have adopted a categorical rule Court." Ibid. The Court of Appeals correctly found that pendent appellate jurisdiction over
than a rule that required the President to litigate the question whether a specific case this issue was proper. See 72 F. 3d, at 1357, n. 4. The District Court's legal ruling that the
belonged in the "exceptional case" subcategory. In all events, the question whether a specific President was protected by a temporary immunity from trial-but not discovery-was
case should receive exceptional treatment is more appropriately the subject of the exercise "inextricably intertwined," Swint v. Chambers County Comm'n, 514 U. S. 35, 51 (1995), with
of judicial discretion than an interpretation of the Constitution. Accordingly, we turn to the its suggestion that a discretionary stay having the same effect might be proper; indeed,
question whether the District Court's decision to stay the trial until after petitioner leaves "review of the [latter] decision [is] necessary to ensure meaningful review of the
office was an abuse of discretion. [former]," ibid.

VII
19

The decision to postpone the trial was, furthermore, premature. The proponent of a stay NASCa, Inc., 501 U. S. 32, 50 (1991) (noting that "if in the informed discretion of the court,
bears the burden of establishing its need. Id., at 255. In this case, at the stage at which the neither the statute nor the Rules are up to the task, the court may safely rely on its inherent
District Court made its ruling, there was no way to assess whether a stay of trial after the power" in imposing appropriate sanctions). Those sanctions may be set at a level "sufficient
completion of discovery would be warranted. Other than the fact that a trial may consume to deter repetition of such conduct or comparable conduct by others similarly situated." Fed.
some of the President's time and attention, there is nothing in the record to enable a judge Rule Civ. Proc. 11 (c)(2). As Rule 11 indicates, sanctions may be appropriate where a claim is
to assess the potential harm that may ensue from scheduling the trial promptly after "presented for any improper purpose, such as to harass," including any claim based on
discovery is concluded. We think the District Court may have given undue weight to the "allegations and other factual contentions [lacking] evidentiary support" or unlikely to prove
concern that a trial might generate unrelated civil actions that could conceivably hamper the wellgrounded after reasonable investigation. Rules l1(b)(l), (3).
President in conducting the duties of his office. If and when that should occur, the court's
discretion would permit it to manage those actions in such fashion (including deferral of trial) 43 United States v. Nixon, 418 U. S., at 710-711; see also Fitzgerald, 457 U. S., at 753 ("Courts
that interference with the President's duties would not occur. But no such impingement traditionally have recognized the President's constitutional responsibilities and status as
upon the President's conduct of his office was shown here. factors counseling judicial deference and restraint").

VIII BREYER, J., concurring in judgment

We add a final comment on two matters that are discussed at length in the briefs: the risk the President advocates, Congress, of course, could not repeal it. But our holding today raises
that our decision will generate a large volume of politically motivated harassing and frivolous no barrier to a statutory response to these concerns.
litigation, and the danger that national security concerns might prevent the President from
explaining a legitimate need for a continuance.
The Federal District Court has jurisdiction to decide this case. Like every other citizen who
properly invokes that jurisdiction, respondent has a right to an orderly disposition of her
We are not persuaded that either of these risks is serious. claims. Accordingly, the judgment of the Court of Appeals is affirmed.

Most frivolous and vexatious litigation is terminated at the pleading stage or on summary It is so ordered.
judgment, with little if any personal involvement by the defendant. See Fed. Rules Civ. Proc.
12, 56. Moreover, the availability of sanctions provides a significant deterrent to litigation
CLINTON v. JONES
directed at the President in his unofficial capacity for purposes of political gain or
harassment.42 History indicates that the likelihood that a significant number of such cases
will be filed is remote. Although scheduling problems may arise, there is no reason to assume certiorari to the united states court of appeals for the eighth circuit
that the district courts will be either unable to accommodate the President's needs or
unfaithful to the tradition-especially in matters involving national securityof giving "the No. 95-1853. Argued January 13, 1997 -- Decided May 27, 1997
utmost deference to Presidential responsibilities."43 Several Presidents, including petitioner,
have given testimony without jeopardizing the Nation's security. See supra, at 704-705. In Respondent sued under 42 U.S.C. §§ 1983 and 1985 and Arkansas law to recover damages
short, we have confidence in the ability of our federal judges to deal with both of these from petitioner, the current President of the United States, alleging, inter alia, that while he
concerns. was Governor of Arkansas, petitioner made "abhorrent" sexual advances to her, and that her
rejection of those advances led to punishment by her supervisors in the state job she held at
If Congress deems it appropriate to afford the President stronger protection, it may respond the time. Petitioner promptly advised the Federal District Court that he would file a motion
with appropriate legislation. As petitioner notes in his brief, Congress has enacted more than to dismiss on Presidential immunity grounds, and requested that all other pleadings and
one statute providing for the deferral of civil litigation to accommodate important public motions be deferred until the immunity issue was resolved. After the court granted that
interests. Brief for Petitioner 34-36. See, e. g., 11 U. S. C. § 362 (litigation against debtor
request, petitioner filed a motion to dismiss without prejudice and to toll any applicable
stayed upon filing of bankruptcy petition); Soldiers' and Sailors' Civil Relief Act of 1940, 50 U.
S. C. App. §§ 501-525 (provisions governing, inter alia, tolling or stay of civil claims by or statutes of limitation during his Presidency. The District Judge denied dismissal on immunity
against military personnel during course of active duty). If the Constitution embodied the grounds and ruled that discovery could go forward, but ordered any trial stayed until
rule that petitioner's Presidency ended. The Eighth Circuit affirmed the dismissal denial, but reversed
the trial postponement as the "functional equivalent" of a grant of temporary immunity to
42 See, e. g., Fed. Rule Civ. Proc. 11; 28 U. S. C. § 1927; Chambers v. which petitioner was not constitutionally entitled. The court explained that the President,
20

like other officials, is subject to the same laws that apply to all citizens, that no case had been Branch's official powers. The Court rejects petitioner's contention that this case--as well as
found in which an official was granted immunity from suit for his unofficial acts, and that the the potential additional litigation that an affirmance of the Eighth Circuit's judgment might
rationale for official immunity is inapposite where only personal, private conduct by a spawn--may place unacceptable burdens on the President that will hamper the performance
President is at issue. The court also rejected the argument that, unless immunity is available, of his official duties. That assertion finds little support either in history, as evidenced by the
the threat of judicial interference with the Executive Branch would violate separation of paucity of suits against sitting Presidents for their private actions, or in the relatively narrow
powers. compass of the issues raised in this particular case. Of greater significance, it is settled that
the Judiciary may severely burden the Executive Branch by reviewing the legality of the
Held: President's official conduct, see e.g., Youngstown Sheet & Tube Co. v. Sawyer, 343 U.S. 579,
and may direct appropriate process to the President himself, see e.g., United States v. Nixon,
1. This Court need not address two important constitutional issues not encompassed within
418 U.S. 683. It must follow that the federal courts have power to determine the legality of
the questions presented by the certiorari petition: (1) whether a claim comparable to
the President's unofficial conduct. The reasons for rejecting a categorical rule requiring
petitioner's assertion of immunity might succeed in a state tribunal, and (2) whether a court
federal courts to stay private actions during the President's term apply as well to a rule that
may compel the President's attendance at any specific time or place. Pp. 7-9.
would, in petitioner's words, require a stay "in all but the most exceptional cases." Pp. 15-24.
2. Deferral of this litigation until petitioner's Presidency ends is not constitutionally required.
(c) Contrary to the Eighth Circuit's ruling, the District Court's stay order was not the
Pp. 7-28.
"functional equivalent" of an unconstitutional grant of temporary immunity. Rather, the
District Court has broad discretion to stay proceedings as an incident to its power to control
(a) Petitioner's principal submission--that in all but the most exceptional cases, the
its own docket. See, e.g., Landis v. North American Co., 299 U.S. 248, 254. Moreover, the
Constitution affords the President temporary immunity from civil damages litigation arising
potential burdens on the President posed by this litigation are appropriate matters for that
out of events that occurred before he took office--cannot be sustained on the basis of
court to evaluate in its management of the case, and the high respect owed the Presidency is
precedent. The principal rationale for affording Presidents immunity from damages actions
a matter that should inform the conduct of the entire proceeding. Nevertheless, the District
based on their official acts--i.e., to enable them to perform their designated functions
Court's stay decision was an abuse of discretion because it took no account of the
effectively without fear that a particular decision may give rise to personal liability, see, e.g.,
importance of respondent's interest in bringing the case to trial, and because it was
Nixon v. Fitzgerald, 457 U.S. 731, 749, 752, and n. 32--provides no support for an immunity
premature in that there was nothing in the record to enable a judge to assess whether
for unofficial conduct. Moreover, immunities for acts clearly within official capacity are
postponement of trial after the completion of discovery would be warranted. Pp. 25-27.
grounded in the nature of the function performed, not the identity of the actor who
performed it. Forrester v. White, 484 U.S. 219, 229. The Court is also unpersuaded by
(d) The Court is not persuaded of the seriousness of the alleged risks that this decision will
petitioner's historical evidence, which sheds little light on the question at issue, and is largely
generate a large volume of politically motivated harassing and frivolous litigation and that
canceled by conflicting evidence that is itself consistent with both the doctrine of presidential
national security concerns might prevent the President from explaining a legitimate need for
immunity as set forth in Fitzgerald, and rejection of the immunity claim in this case. Pp. 9-15.
a continuance, and has confidence in the ability of federal judges to deal with both concerns.
If Congress deems it appropriate to afford the President stronger protection, it may respond
(b) The separation of powers doctrine does not require federal courts to stay all private
with legislation. Pp. 27-28.
actions against the President until he leaves office. Even accepting the unique importance of
the Presidency in the constitutional scheme, it does not follow that that doctrine would be
72 F. 3d 1354, affirmed.
violated by allowing this action to proceed. The doctrine provides a self executing safeguard
against the encroachment or aggrandizement of one of the three co equal branches of Stevens, J., delivered the opinion of the Court, in which Rehnquist, C. J., and O'Connor, Scalia,
Government at the expense of another. Buckley v. Valeo, 424 U.S. 1, 122. But in this case Kennedy, Souter, Thomas, and Ginsburg, JJ., joined. Breyer, J., filed an opinion concurring in
there is no suggestion that the Federal Judiciary is being asked to perform any function that the judgment.
might in some way be described as "executive." Respondent is merely asking the courts to
exercise their core Article III jurisdiction to decide cases and controversies, and, whatever the
outcome, there is no possibility that the decision here will curtail the scope of the Executive
21

EN BANC to beaches and asking her questions about Karapatan, an alliance of human rights
organizations. He, however, failed to make an investigation even after Lourdes’
disappearance had been made known to him;
G.R. No. 183871 February 18, 2010

3. A week after Lourdes’ release, another daughter, Jean R. Apruebo (Jean), was
LOURDES D. RUBRICO, JEAN RUBRICO APRUEBO, and MARY JOY RUBRICO
constrained to leave their house because of the presence of men watching them;
CARBONEL, Petitioners,
vs.
GLORIA MACAPAGAL-ARROYO, GEN. HERMOGENES ESPERON, P/DIR. GEN. AVELINO 4. Lourdes has filed with the Office of the Ombudsman a criminal complaint for
RAZON, MAJ. DARWIN SY a.k.a. DARWIN REYES, JIMMY SANTANA, RUBEN ALFARO, CAPT. kidnapping and arbitrary detention and administrative complaint for gross abuse of
ANGELO CUARESMA, a certain JONATHAN, P/SUPT. EDGAR B. ROQUERO, ARSENIO C. authority and grave misconduct against Capt. Angelo Cuaresma (Cuaresma), Ruben
GOMEZ, and OFFICE OF THE OMBUDSMAN, Respondents. Alfaro (Alfaro), Jimmy Santana (Santana) and a certain Jonathan, c/o Headquarters
301st AISS, Fernando Air Base and Maj. Sy/Reyes with address at No. 09
Amsterdam Ext., Merville Subd., Parañaque City, but nothing has happened; and
DECISION
the threats and harassment incidents have been reported to the Dasmariñas
municipal and Cavite provincial police stations, but nothing eventful resulted from
VELASCO, JR., J.: their respective investigations.

In this petition for review under Rule 45 of the Rules of Court in relation to Section 191 of the Two of the four witnesses to Lourdes’ abduction went into hiding after being
Rule on the Writ of Amparo2 (Amparo Rule), Lourdes D. Rubrico, Jean Rubrico Apruebo, and visited by government agents in civilian clothes; and
Mary Joy Rubrico Carbonel assail and seek to set aside the Decision3 of the Court of Appeals
(CA) dated July 31, 2008 in CA-G.R. SP No. 00003, a petition commenced under the Amparo
5. Karapatan conducted an investigation on the incidents. The investigation would
Rule.
indicate that men belonging to the Armed Forces of the Philippines (AFP), namely
Capt. Cuaresma of the Philippine Air Force (PAF), Alfaro, Santana, Jonathan and
The petition for the writ of amparo dated October 25, 2007 was originally filed before this Maj. Darwin Sy/Reyes, led the abduction of Lourdes; that unknown to the
Court. After issuing the desired writ and directing the respondents to file a verified written abductors, Lourdes was able to pilfer a "mission order" which was addressed to CA
return, the Court referred the petition to the CA for summary hearing and appropriate Ruben Alfaro and signed by Capt. Cuaresma of the PAF.
action. The petition and its attachments contained, in substance, the following allegations:
The petition prayed that a writ of amparo issue, ordering the individual respondents to desist
1. On April 3, 2007, armed men belonging to the 301st Air Intelligence and Security from performing any threatening act against the security of the petitioners and for the Office
Squadron (AISS, for short) based in Fernando Air Base in Lipa City abducted Lourdes of the Ombudsman (OMB) to immediately file an information for kidnapping qualified with
D. Rubrico (Lourdes), then attending a Lenten pabasa in Bagong Bayan, the aggravating circumstance of gender of the offended party. It also prayed for damages
Dasmariñas, Cavite, and brought to, and detained at, the air base without charges. and for respondents to produce documents submitted to any of them on the case of Lourdes.
Following a week of relentless interrogation - conducted alternately by hooded
individuals - and what amounts to verbal abuse and mental harassment, Lourdes,
Before the CA, respondents President Gloria Macapagal-Arroyo, Gen. Hermogenes Esperon,
chair of the Ugnayan ng Maralita para sa Gawa Adhikan, was released at
then Armed Forces of the Philippines (AFP) Chief of Staff, Police Director-General (P/Dir.
Dasmariñas, Cavite, her hometown, but only after being made to sign a statement
Gen.) Avelino Razon, then Philippine National Police (PNP) Chief, Police Superintendent
that she would be a military asset.
(P/Supt.) Roquero of the Cavite Police Provincial Office, Police Inspector (P/Insp.) Gomez,
now retired, and the OMB (answering respondents, collectively) filed, through the Office of
After Lourdes’ release, the harassment, coming in the form of being tailed on at the Solicitor General (OSG), a joint return on the writ specifically denying the material
least two occasions at different places, i.e., Dasmariñas, Cavite and Baclaran in inculpatory averments against them. The OSG also denied the allegations against the
Pasay City, by motorcycle-riding men in bonnets, continued; following impleaded persons, namely: Cuaresma, Alfaro, Santana, Jonathan, and Sy/Reyes,
for lack of knowledge or information sufficient to form a belief as to the allegations’ truth.
2. During the time Lourdes was missing, P/Sr. Insp. Arsenio Gomez (P/Insp. Gomez), And by way of general affirmative defenses, answering respondents interposed the following
then sub-station commander of Bagong Bayan, Dasmariñas, Cavite, kept sending defenses: (1) the President may not be sued during her incumbency; and (2) the petition is
text messages to Lourdes’ daughter, Mary Joy R. Carbonel (Mary Joy), bringing her
22

incomplete, as it fails to indicate the matters required by Sec. 5(d) and (e) of the Amparo 3. P/Supt. Roquero – stated conducting, upon receipt of Lourdes’ complaint, an
Rule.4 investigation and submitting the corresponding report to the PNP Calabarzon,
observing that neither Lourdes nor her relatives provided the police with relevant
Attached to the return were the affidavits of the following, among other public officials, information;
containing their respective affirmative defenses and/or statements of what they had
undertaken or committed to undertake regarding the claimed disappearance of Lourdes and 4. P/Insp. Gomez – alleged that Lourdes, her kin and witnesses refused to
the harassments made to bear on her and her daughters: cooperate with the investigating Cavite PNP; and

1. Gen. Esperon – attested that, pursuant to a directive of then Secretary of 5. Overall Deputy Ombudsman Orlando Casimiro - alleged that cases for violation
National Defense (SND) Gilberto C. Teodoro, Jr., he ordered the Commanding of Articles 267 and 124, or kidnapping and arbitrary detention, respectively, have
General of the PAF, with information to all concerned units, to conduct an been filed with, and are under preliminary investigation by the OMB against those
investigation to establish the circumstances behind the disappearance and the believed to be involved in Lourdes’ kidnapping; that upon receipt of the petition for
reappearance of Lourdes insofar as the involvement of alleged personnel/unit is a writ of amparo, proper coordination was made with the Office of the Deputy
concerned. The Provost Marshall General and the Office of the Judge Advocate Ombudsman for the Military and other Law Enforcement Offices (MOLEO) where
General (JAGO), AFP, also undertook a parallel action. the subject criminal and administrative complaints were filed.

Gen. Esperon manifested his resolve to provide the CA with material results of the Commenting on the return, petitioners pointed out that the return was no more than a
investigation; to continue with the probe on the alleged abduction of Lourdes and general denial of averments in the petition. They, thus, pleaded to be allowed to present
to bring those responsible, including military personnel, to the bar of justice when evidence ex parte against the President, Santana, Alfaro, Capt. Cuaresma, Darwin Sy, and
warranted by the findings and the competent evidence that may be gathered in the Jonathan. And with leave of court, they also asked to serve notice of the petition through
investigation process by those mandated to look into the matter;5 publication, owing to their failure to secure the current address of the latter five and thus
submit, as the CA required, proof of service of the petition on them.
2. P/Dir. Gen. Razon - stated that an investigation he immediately ordered upon
receiving a copy of the petition is on-going vis-à-vis Lourdes’ abduction, and that a The hearing started on November 13, 2007.7 In that setting, petitioners’ counsel prayed for
background verification with the PNP Personnel Accounting and Information the issuance of a temporary protection order (TPO) against the answering respondents on
System disclosed that the names Santana, Alfaro, Cuaresma and one Jonathan do the basis of the allegations in the petition. At the hearing of November 20, 2007, the CA
not appear in the police personnel records, although the PNP files carry the name granted petitioners’ motion that the petition and writ be served by the court’s process server
of Darwin Reyes Y. Muga. on Darwin Sy/Reyes, Santana, Alfaro, Capt. Cuaresma, and Jonathan.

Per the initial investigation report of the Dasmariñas municipal police station, The legal skirmishes that followed over the propriety of excluding President Arroyo from the
P/Dir. Gen. Razon disclosed, Lourdes was abducted by six armed men in the petition, petitioners’ motions for service by publication, and the issuance of a TPO are not of
afternoon of April 3, 2007 and dragged aboard a Toyota Revo with plate number decisive pertinence in this recital. The bottom line is that, by separate resolutions, the CA
XRR 428, which plate was issued for a Mitsubishi van to AK Cottage Industry with dropped the President as respondent in the case; denied the motion for a TPO for the court’s
address at 9 Amsterdam St., Merville Subd., Parañaque City. The person residing in want of authority to issue it in the tenor sought by petitioners; and effectively denied the
the apartment on that given address is one Darius/Erwin See @ Darius Reyes motion for notice by publication owing to petitioners’ failure to submit the affidavit required
allegedly working, per the latter’s house helper, in Camp Aguinaldo. under Sec. 17, Rule 14 of the Rules of Court.8

P/Dir. Gen. Razon, however, bemoaned the fact that Mrs. Rubrico never contacted After due proceedings, the CA rendered, on July 31, 2008, its partial judgment, subject of this
nor coordinated with the local police or other investigating units of the PNP after review, disposing of the petition but only insofar as the answering respondents were
her release, although she is in the best position to establish the identity of her concerned. The fallo of the CA decision reads as follows:
abductors and/or provide positive description through composite sketching.
Nonetheless, he manifested that the PNP is ready to assist and protect the WHEREFORE, premises considered, partial judgment is hereby rendered DISMISSING the
petitioners and the key witnesses from threats, harassments and intimidation from instant petition with respect to respondent Gen. Hermogenes Esperon, P/Dir. Gen. Avelino
whatever source and, at the same time, to assist the Court in the implementation
of its orders.61avvphi1
23

Razon, Supt. Edgar B. Roquero, P/Sr. Insp. Arsenio C. Gomez (ret.) and the Office of the This brings us to the correctness of the assailed dismissal of the petition with respect to Gen.
Ombudsman. Esperon, P/Dir. Gen. Razon, P/Supt. Roquero, P/Insp. Gomez, and the OMB.

Nevertheless, in order that petitioners’ complaint will not end up as another unsolved case, None of the four individual respondents immediately referred to above has been implicated
the heads of the Armed Forces of the Philippines and the Philippine National Police are as being connected to, let alone as being behind, the alleged abduction and harassment of
directed to ensure that the investigations already commenced are diligently pursued to bring petitioner Lourdes. Their names were not even mentioned in Lourdes’ Sinumpaang
the perpetrators to justice. The Chief of Staff of the Armed Forces of the Philippines and Salaysay11 of April 2007. The same goes for the respective Sinumpaang
P/Dir. Gen. Avelino Razon are directed to regularly update petitioners and this Court on the Salaysay and/or Karagdagang Sinumpaang Salaysay of Jean12 and Mary Joy.13
status of their investigation.
As explained by the CA, Gen. Esperon and P/Dir. Gen. Razon were included in the case on the
SO ORDERED. theory that they, as commanders, were responsible for the unlawful acts allegedly
committed by their subordinates against petitioners. To the appellate court, "the privilege of
In this recourse, petitioners formulate the issue for resolution in the following wise: the writ of amparo must be denied as against Gen. Esperon and P/Dir. Gen. Razon for the
simple reason that petitioners have not presented evidence showing that those who
allegedly abducted and illegally detained Lourdes and later threatened her and her family
WHETHER OR NOT the [CA] committed reversible error in dismissing [their] Petition and
were, in fact, members of the military or the police force." The two generals, the CA’s holding
dropping President Gloria Macapagal Arroyo as party respondent.
broadly hinted, would have been accountable for the abduction and threats if the actual
malefactors were members of the AFP or PNP.
Petitioners first take issue on the President’s purported lack of immunity from suit during her
term of office. The 1987 Constitution, so they claim, has removed such immunity heretofore
As regards the three other answering respondents, they were impleaded because they
enjoyed by the chief executive under the 1935 and 1973 Constitutions.
allegedly had not exerted the required extraordinary diligence in investigating and
satisfactorily resolving Lourdes’ disappearance or bringing to justice the actual perpetrators
Petitioners are mistaken. The presidential immunity from suit remains preserved under our of what amounted to a criminal act, albeit there were allegations against P/Insp. Gomez of
system of government, albeit not expressly reserved in the present constitution. Addressing acts constituting threats against Mary Joy.
a concern of his co-members in the 1986 Constitutional Commission on the absence of an
express provision on the matter, Fr. Joaquin Bernas, S.J. observed that it was already
While in a qualified sense tenable, the dismissal by the CA of the case as against Gen.
understood in jurisprudence that the President may not be sued during his or her
Esperon and P/Dir. Gen. Razon is incorrect if viewed against the backdrop of the stated
tenure.9 The Court subsequently made it abundantly clear in David v. Macapagal-Arroyo, a
rationale underpinning the assailed decision vis-à-vis the two generals, i.e., command
case likewise resolved under the umbrella of the 1987 Constitution, that indeed the
responsibility. The Court assumes the latter stance owing to the fact that command
President enjoys immunity during her incumbency, and why this must be so:
responsibility, as a concept defined, developed, and applied under international law, has
little, if at all, bearing in amparo proceedings.
Settled is the doctrine that the President, during his tenure of office or actual incumbency,
may not be sued in any civil or criminal case, and there is no need to provide for it in the
The evolution of the command responsibility doctrine finds its context in the development of
Constitution or law. It will degrade the dignity of the high office of the President, the Head of
laws of war and armed combats. According to Fr. Bernas, "command responsibility," in its
State, if he can be dragged into court litigations while serving as such. Furthermore, it is
simplest terms, means the "responsibility of commanders for crimes committed by
important that he be freed from any form of harassment, hindrance or distraction to enable
subordinate members of the armed forces or other persons subject to their control in
him to fully attend to the performance of his official duties and functions. Unlike the
international wars or domestic conflict."14 In this sense, command responsibility is properly a
legislative and judicial branch, only one constitutes the executive branch and anything which
form of criminal complicity. The Hague Conventions of 1907 adopted the doctrine of
impairs his usefulness in the discharge of the many great and important duties imposed upon
command responsibility,15foreshadowing the present-day precept of holding a superior
him by the Constitution necessarily impairs the operation of the Government.10 x x x
accountable for the atrocities committed by his subordinates should he be remiss in his duty
of control over them. As then formulated, command responsibility is "an omission mode of
And lest it be overlooked, the petition is simply bereft of any allegation as to what specific individual criminal liability," whereby the superior is made responsible for crimes
presidential act or omission violated or threatened to violate petitioners’ protected rights. committed by his subordinates for failing to prevent or punish the perpetrators16 (as
opposed to crimes he ordered).
24

The doctrine has recently been codified in the Rome Statute17 of the International Criminal should not be pursued to fix criminal liability on respondents preparatory to criminal
Court (ICC) to which the Philippines is signatory. Sec. 28 of the Statute imposes individual prosecution, or as a prelude to administrative disciplinary proceedings under existing
responsibility on military commanders for crimes committed by forces under their control. administrative issuances, if there be any.
The country is, however, not yet formally bound by the terms and provisions embodied in
this treaty-statute, since the Senate has yet to extend concurrence in its ratification.18 Petitioners, as the CA has declared, have not adduced substantial evidence pointing to
government involvement in the disappearance of Lourdes. To a concrete point, petitioners
While there are several pending bills on command responsibility,19 there is still no Philippine have not shown that the actual perpetrators of the abduction and the harassments that
law that provides for criminal liability under that doctrine.20 followed formally or informally formed part of either the military or the police chain of
command. A preliminary police investigation report, however, would tend to show a link,
It may plausibly be contended that command responsibility, as legal basis to hold however hazy, between the license plate (XRR 428) of the vehicle allegedly used in the
military/police commanders liable for extra-legal killings, enforced disappearances, or abduction of Lourdes and the address of Darwin Reyes/Sy, who was alleged to be working in
threats, may be made applicable to this jurisdiction on the theory that the command Camp Aguinaldo.25 Then, too, there were affidavits and testimonies on events that transpired
responsibility doctrine now constitutes a principle of international law or customary which, if taken together, logically point to military involvement in the alleged disappearance
international law in accordance with the incorporation clause of the Constitution.21 Still, it of Lourdes, such as, but not limited to, her abduction in broad daylight, her being forcibly
would be inappropriate to apply to these proceedings the doctrine of command dragged to a vehicle blindfolded and then being brought to a place where the sounds of
responsibility, as the CA seemed to have done, as a form of criminal complicity through planes taking off and landing could be heard. Mention may also be made of the fact that
omission, for individual respondents’ criminal liability, if there be any, is beyond the reach of Lourdes was asked about her membership in the Communist Party and of being released
amparo. In other words, the Court does not rule in such proceedings on any issue of criminal when she agreed to become an "asset."
culpability, even if incidentally a crime or an infraction of an administrative rule may have
been committed. As the Court stressed in Secretary of National Defense v. Manalo Still and all, the identities and links to the AFP or the PNP of the alleged abductors, namely
(Manalo),22 the writ of amparo was conceived to provide expeditious and effective Cuaresma, Alfaro, Santana, Jonathan, and Sy/Reyes, have yet to be established.
procedural relief against violations or threats of violation of the basic rights to life, liberty,
and security of persons; the corresponding amparo suit, however, "is not an action to Based on the separate sworn statements of Maj. Paul Ciano26 and Technical Sergeant John N.
determine criminal guilt requiring proof beyond reasonable doubt x x x or administrative Romano,27 officer-in-charge and a staff of the 301st AISS, respectively, none of the alleged
liability requiring substantial evidence that will require full and exhaustive proceedings." 23 Of abductors of Lourdes belonged to the 301st AISS based in San Fernando Air Base. Neither
the same tenor, and by way of expounding on the nature and role of amparo, is what the were they members of any unit of the Philippine Air Force, per the certification28 of Col. Raul
Court said in Razon v. Tagitis: Dimatactac, Air Force Adjutant. And as stated in the challenged CA decision, a verification
with the Personnel Accounting and Information System of the PNP yielded the information
It does not determine guilt nor pinpoint criminal culpability for the disappearance [threats that, except for a certain Darwin Reyes y Muga, the other alleged abductors, i.e., Cuaresma,
thereof or extra-judicial killings]; it determines responsibility, or at least accountability, for Alfaro, Santana and Jonathan, were not members of the PNP. Petitioners, when given the
the enforced disappearance [threats thereof or extra-judicial killings] for purposes of opportunity to identify Police Officer 1 Darwin Reyes y Muga, made no effort to confirm if he
imposing the appropriate remedies to address the disappearance [or extra-judicial killings]. was the same Maj. Darwin Reyes a.k.a. Darwin Sy they were implicating in Lourdes’
abduction.
As the law now stands, extra-judicial killings and enforced disappearances in this jurisdiction
are not crimes penalized separately from the component criminal acts undertaken to carry Petitioners, to be sure, have not successfully controverted answering respondents’
out these killings and enforced disappearances and are now penalized under the Revised documentary evidence, adduced to debunk the former’s allegations directly linking Lourdes’
Penal Code and special laws. The simple reason is that the Legislature has not spoken on the abductors and tormentors to the military or the police establishment. We note, in fact, that
matter; the determination of what acts are criminal x x x are matters of substantive law that Lourdes, when queried on cross-examination, expressed the belief that Sy/Reyes was an NBI
only the Legislature has the power to enact.24 x x x agent.29 The Court is, of course, aware of what was referred to in Razon30 as the "evidentiary
difficulties" presented by the nature of, and encountered by petitioners in, enforced
If command responsibility were to be invoked and applied to these proceedings, it should, at disappearance cases. But it is precisely for this reason that the Court should take care too
most, be only to determine the author who, at the first instance, is accountable for, and has that no wrong message is sent, lest one conclude that any kind or degree of evidence, even
the duty to address, the disappearance and harassments complained of, so as to enable the the outlandish, would suffice to secure amparo remedies and protection.
Court to devise remedial measures that may be appropriate under the premises to protect
rights covered by the writ of amparo. As intimated earlier, however, the determination
25

Sec. 17, as complemented by Sec. 18 of the Amparo Rule, expressly prescribes the minimum for his clients and their witnesses’ attitude, "[They] do not trust the government agencies to
evidentiary substantiation requirement and norm to support a cause of action under the protect them."34 The difficulty arising from a situation where the party whose complicity in
Rule, thus: extra-judicial killing or enforced disappearance, as the case may be, is alleged to be the same
party who investigates it is understandable, though.
Sec. 17. Burden of Proof and Standard of Diligence Required.—The parties shall establish
their claims by substantial evidence. The seeming reluctance on the part of the Rubricos or their witnesses to cooperate ought not
to pose a hindrance to the police in pursuing, on its own initiative, the investigation in
Sec. 18. Judgment.—x x x If the allegations in the petition are proven by substantial evidence, question to its natural end. To repeat what the Court said in Manalo, the right to security of
the court shall grant the privilege of the writ and such reliefs as may be proper and persons is a guarantee of the protection of one’s right by the government. And this
appropriate; otherwise, the privilege shall be denied. (Emphasis added.) protection includes conducting effective investigations of extra-legal killings, enforced
disappearances, or threats of the same kind. The nature and importance of an investigation
are captured in the Velasquez Rodriguez case,35 in which the Inter-American Court of Human
Substantial evidence is more than a mere imputation of wrongdoing or violation that would
Rights pronounced:
warrant a finding of liability against the person charged;31 it is more than a scintilla of
evidence. It means such amount of relevant evidence which a reasonable mind might accept
as adequate to support a conclusion, even if other equally reasonable minds might opine [The duty to investigate] must be undertaken in a serious manner and not as a mere
otherwise.32 Per the CA’s evaluation of their evidence, consisting of the testimonies and formality preordained to be ineffective. An investigation must have an objective and be
affidavits of the three Rubrico women and five other individuals, petitioners have not assumed by the State as its own legal duty, not a step taken by private interests that
satisfactorily hurdled the evidentiary bar required of and assigned to them under the depends upon the initiative of the victim or his family or upon offer of proof, without an
Amparo Rule. In a very real sense, the burden of evidence never even shifted to answering effective search for the truth by the government. (Emphasis added.)
respondents. The Court finds no compelling reason to disturb the appellate court’s
determination of the answering respondents’ role in the alleged enforced disappearance of This brings us to Mary Joy’s charge of having been harassed by respondent P/Insp. Gomez.
petitioner Lourdes and the threats to her family’s security. With the view we take of this incident, there is nothing concrete to support the charge, save
for Mary Joy’s bare allegations of harassment. We cite with approval the following self-
Notwithstanding the foregoing findings, the Court notes that both Gen. Esperon and P/Dir. explanatory excerpt from the appealed CA decision:
Gen. Razon, per their separate affidavits, lost no time, upon their receipt of the order to
make a return on the writ, in issuing directives to the concerned units in their respective In fact, during her cross-examination, when asked what specific act or threat P/Sr. Gomez
commands for a thorough probe of the case and in providing the investigators the necessary (ret) committed against her or her mother and sister, Mary Joy replied "None …"36
support. As of this date, however, the investigations have yet to be concluded with some
definite findings and recommendation. Similarly, there appears to be no basis for petitioners’ allegations about the OMB failing to
act on their complaint against those who allegedly abducted and illegally detained Lourdes.
As regards P/Supt. Romero and P/Insp. Gomez, the Court is more than satisfied that they Contrary to petitioners’ contention, the OMB has taken the necessary appropriate action on
have no direct or indirect hand in the alleged enforced disappearance of Lourdes and the said complaint. As culled from the affidavit37 of the Deputy Overall Ombudsman and the joint
threats against her daughters. As police officers, though, theirs was the duty to thoroughly affidavits38 of the designated investigators, all dated November 7, 2007, the OMB had, on the
investigate the abduction of Lourdes, a duty that would include looking into the cause, basis of said complaint, commenced criminal39 and administrative40 proceedings, docketed as
manner, and like details of the disappearance; identifying witnesses and obtaining OMB-P-C-07-0602-E and OMB-P-A 07-567-E, respectively, against Cuaresma, Alfaro, Santana,
statements from them; and following evidentiary leads, such as the Toyota Revo vehicle with Jonathan, and Sy/Reyes. The requisite orders for the submission of counter-affidavits and
plate number XRR 428, and securing and preserving evidence related to the abduction and verified position papers had been sent out.
the threats that may aid in the prosecution of the person/s responsible. As we said
in Manalo,33 the right to security, as a guarantee of protection by the government, is The privilege of the writ of amparo, to reiterate, is a remedy available to victims of extra-
breached by the superficial and one-sided––hence, ineffective––investigation by the military judicial killings and enforced disappearances or threats of similar nature, regardless of
or the police of reported cases under their jurisdiction. As found by the CA, the local police whether the perpetrator of the unlawful act or omission is a public official or employee or a
stations concerned, including P/Supt. Roquero and P/Insp. Gomez, did conduct a preliminary private individual.
fact-finding on petitioners’ complaint. They could not, however, make any headway, owing
to what was perceived to be the refusal of Lourdes, her family, and her witnesses to
cooperate. Petitioners’ counsel, Atty. Rex J.M.A. Fernandez, provided a plausible explanation
26

At this juncture, it bears to state that petitioners have not provided the CA with the correct what to us are amparo reliefs. Consider: the appellate court decreed, and rightly so, that the
addresses of respondents Cuaresma, Alfaro, Santana, Jonathan, and Sy/Reyes. The mailed police and the military take specific measures for the protection of petitioners’ right or
envelopes containing the petition for a writ of amparo individually addressed to each of threatened right to liberty or security. The protection came in the form of directives
them have all been returned unopened. And petitioners’ motion interposed before the specifically to Gen. Esperon and P/Dir. Gen. Razon, requiring each of them (1) to ensure that
appellate court for notice or service via publication has not been accompanied by supporting the investigations already commenced by the AFP and PNP units, respectively, under them
affidavits as required by the Rules of Court. Accordingly, the appealed CA partial judgment–– on the complaints of Lourdes and her daughters are being pursued with urgency to bring to
disposing of the underlying petition for a writ of amparo without (1) pronouncement as to justice the perpetrators of the acts complained of; and (2) to submit to the CA, copy
the accountability, or lack of it, of the four non-answering respondents or (2) outright furnished the petitioners, a regular report on the progress and status of the investigations.
dismissal of the same petition as to them––hews to the prescription of Sec. 20 of the Amparo The directives obviously go to Gen. Esperon in his capacity as head of the AFP and, in a sense,
Rule on archiving and reviving cases.41 Parenthetically, petitioners have also not furnished chief guarantor of order and security in the country. On the other hand, P/Dir. Gen. Razon is
this Court with sufficient data as to where the afore-named respondents may be served a called upon to perform a duty pertaining to the PNP, a crime-preventing, investigatory, and
copy of their petition for review. arresting institution.

Apart from the foregoing considerations, the petition did not allege ultimate facts as would As the CA, however, formulated its directives, no definitive time frame was set in its decision
link the OMB in any manner to the violation or threat of violation of the petitioners’ rights to for the completion of the investigation and the reportorial requirements. It also failed to
life, liberty, or personal security. consider Gen. Esperon and P/Dir. Gen. Razon’s imminent compulsory retirement from the
military and police services, respectively. Accordingly, the CA directives, as hereinafter
The privilege of the writ of amparo is envisioned basically to protect and guarantee the rights redefined and amplified to fully enforce the amparo remedies, are hereby given to, and shall
to life, liberty, and security of persons, free from fears and threats that vitiate the quality of be directly enforceable against, whoever sits as the commanding general of the AFP and the
this life.42 It is an extraordinary writ conceptualized and adopted in light of and in response to PNP.
the prevalence of extra-legal killings and enforced disappearances.43 Accordingly, the remedy
ought to be resorted to and granted judiciously, lest the ideal sought by the Amparo Rule be At this stage, two postulates and their implications need highlighting for a proper disposition
diluted and undermined by the indiscriminate filing of amparo petitions for purposes less of this case.
than the desire to secure amparo reliefs and protection and/or on the basis of
unsubstantiated allegations. First, a criminal complaint for kidnapping and, alternatively, for arbitrary detention rooted in
the same acts and incidents leading to the filing of the subject amparo petition has been
In their petition for a writ of amparo, petitioners asked, as their main prayer, that the Court instituted with the OMB, docketed as OMB-P-C-O7-0602-E. The usual initial steps to
order the impleaded respondents "to immediately desist from doing any acts that would determine the existence of a prima facie case against the five (5) impleaded individuals
threaten or seem to threaten the security of the Petitioners and to desist from approaching suspected to be actually involved in the detention of Lourdes have been set in motion. It
Petitioners, x x x their residences and offices where they are working under pain of contempt must be pointed out, though, that the filing44 of the OMB complaint came before the
of [this] Court." Petitioners, however, failed to adduce the threshold substantive evidence to effectivity of the Amparo Rule on October 24, 2007.
establish the predicate facts to support their cause of action, i.e., the adverted harassments
and threats to their life, liberty, or security, against responding respondents, as responsible Second, Sec. 2245 of the Amparo Rule proscribes the filing of an amparo petition should a
for the disappearance and harassments complained of. This is not to say, however, that criminal action have, in the meanwhile, been commenced. The succeeding Sec. 23,46 on the
petitioners’ allegation on the fact of the abduction incident or harassment is necessarily other hand, provides that when the criminal suit is filed subsequent to a petition for amparo,
contrived. The reality on the ground, however, is that the military or police connection has the petition shall be consolidated with the criminal action where the Amparo Rule shall
not been adequately proved either by identifying the malefactors as components of the AFP nonetheless govern the disposition of the relief under the Rule. Under the terms of said Sec.
or PNP; or in case identification is not possible, by showing that they acted with the direct or 22, the present petition ought to have been dismissed at the outset. But as things stand, the
indirect acquiescence of the government. For this reason, the Court is unable to ascribe the outright dismissal of the petition by force of that section is no longer technically feasible in
authorship of and responsibility for the alleged enforced disappearance of Lourdes and the light of the interplay of the following factual mix: (1) the Court has, pursuant to Sec. 647 of
harassment and threats on her daughters to individual respondents. To this extent, the the Rule, already issued ex parte the writ of amparo; (2) the CA, after a summary hearing, has
dismissal of the case against them is correct and must, accordingly, be sustained. dismissed the petition, but not on the basis of Sec. 22; and (3) the complaint in OMB-P-C-O7-
0602-E named as respondents only those believed to be the actual abductors of Lourdes,
Prescinding from the above considerations, the Court distinctly notes that the appealed while the instant petition impleaded, in addition, those tasked to investigate the kidnapping
decision veritably extended the privilege of the writ of amparo to petitioners when it granted and detention incidents and their superiors at the top. Yet, the acts and/or omissions subject
27

of the criminal complaint and the amparo petition are so linked as to call for the submit certifications of this determination to the OMB with copy
consolidation of both proceedings to obviate the mischief inherent in a multiplicity-of-suits furnished to petitioners, the CA, and this Court;
situation.
(b) Pursue with extraordinary diligence the evidentiary leads relating to
Given the above perspective and to fully apply the beneficial nature of the writ of amparo as Maj. Darwin Sy and the Toyota Revo vehicle with Plate No. XRR 428; and
an inexpensive and effective tool to protect certain rights violated or threatened to be
violated, the Court hereby adjusts to a degree the literal application of Secs. 22 and 23 of the (c) Prepare, with the assistance of petitioners and/or witnesses,
Amparo Rule to fittingly address the situation obtaining under the premises. 48 Towards this cartographic sketches of respondents Maj. Sy/Reyes, Jimmy Santana,
end, two things are at once indicated: (1) the consolidation of the probe and fact-finding Ruben Alfaro, Capt. Angelo Cuaresma, and a certain Jonathan to aid in
aspects of the instant petition with the investigation of the criminal complaint before the positively identifying and locating them.
OMB; and (2) the incorporation in the same criminal complaint of the allegations in this
petition bearing on the threats to the right to security. Withal, the OMB should be furnished
The investigations shall be completed not later than six (6) months from receipt of this
copies of the investigation reports to aid that body in its own investigation and eventual
Decision; and within thirty (30) days after completion of the investigations, the Chief of Staff
resolution of OMB-P-C-O7-0602-E. Then, too, the OMB shall be given easy access to all
of the AFP and the Director-General of the PNP shall submit a full report of the results of the
pertinent documents and evidence, if any, adduced before the CA. Necessarily, Lourdes, as
investigations to the Court, the CA, the OMB, and petitioners.
complainant in OMB-P-C-O7-0602-E, should be allowed, if so minded, to amend her basic
criminal complaint if the consolidation of cases is to be fully effective.
This case is accordingly referred back to the CA for the purpose of monitoring the
investigations and the actions of the AFP and the PNP.
WHEREFORE, the Court PARTIALLY GRANTS this petition for review and makes a decision:

Subject to the foregoing modifications, the Court AFFIRMS the partial judgment dated July
(1) Affirming the dropping of President Gloria Macapagal-Arroyo from the petition
31, 2008 of the CA.
for a writ of amparo;

SO ORDERED.
(2) Affirming the dismissal of the amparo case as against Gen. Hermogenes
Esperon, and P/Dir. Gen. Avelino Razon, insofar as it tended, under the command
responsibility principle, to attach accountability and responsibility to them, as then FACTS:
AFP Chief of Staff and then PNP Chief, for the alleged enforced disappearance of
Lourdes and the ensuing harassments allegedly committed against petitioners. The Rubrico, in her petition, said she was abducted on April 3, 2007 by armed men belonging to
dismissal of the petition with respect to the OMB is also affirmed for failure of the the 301st Air Intelligence and Security Squadron, based at the Philippine Air Force Field
petition to allege ultimate facts as to make out a case against that body for the Station at Fernando Air Base in Lipa City, Batangas. During her detention, the petitioner
enforced disappearance of Lourdes and the threats and harassment that followed; added, her daughters Mary Joy Rubrico Carbonel and Jean Rubrico Apruebo were harassed
and
by Senior Insp. Arsenio Gomez and that there were also armed men following them. The
petitioners prayed that a writ of amparo be issued, ordering the individual respondents to
(3) Directing the incumbent Chief of Staff, AFP, or his successor, and the incumbent
desist from performing any threatening act against the security of the petitioners and for the
Director-General of the PNP, or his successor, to ensure that the investigations
already commenced by their respective units on the alleged abduction of Lourdes Office of the Ombudsman (OMB) to immediately file an information for kidnapping qualified
Rubrico and the alleged harassments and threats she and her daughters were with the aggravating circumstance of gender of the offended party. It also prayed for
made to endure are pursued with extraordinary diligence as required by Sec. damages and for respondents to produce documents submitted to any of them on the case
1749 of the Amparo Rule. They shall order their subordinate officials, in particular, of Lourdes.
to do the following:
The respondents then filed a joint return on the writ specifically denying the material
(a) Determine based on records, past and present, the identities and inculpatory averments against them. Respondents interposed the defense that the President
locations of respondents Maj. Darwin Sy, a.k.a. Darwin Reyes, Jimmy may not be sued during her incumbency. Petitioners pleaded back to be allowed to present
Santana, Ruben Alfaro, Capt. Angelo Cuaresma, and one Jonathan; and
28

evidence ex parte against the President, et al. By a separate resolution, the CA dropped the
President as respondent in the case .

ISSUE: WHETHER OR NOT the [CA] committed reversible error in dismissing [their] Petition
and dropping President Gloria Macapagal Arroyo as party respondent.

HELD:

The presidential immunity from suit remains preserved under our system of government,
albeit not expressly reserved in the present constitution. Addressing a concern of his co-
members in the 1986 Constitutional Commission on the absence of an express provision on
the matter, Fr. Joaquin Bernas, S.J. observed that it was already understood in jurisprudence
that the President may not be sued during his or her tenure.

Settled is the doctrine that the President, during his tenure of office or actual incumbency,
may not be sued in any civil or criminal case, and there is no need to provide for it in the
Constitution or law. It will degrade the dignity of the high office of the President, the Head of
State, if he can be dragged into court litigations while serving as such.

The Court also affirmed the dismissal of the amparo case against other respondents for
failure of the petition to allege ultimate facts as to make out a case against that body for the
enforced disappearance of Lourdes and the threats and harassment that followed.
29

EN BANC The petition is DISMISSED with respect to President Gloria Macapagal-Arroyo on account of
her presidential immunity from suit. Similarly, the petition is DISMISSED with respect to
G.R. No. 191805 November 15, 2011 respondents Calog and George Palacpac or Harry for lack of merit.

IN THE MATTER OF THE PETITION FOR THE WRIT OF AMPARO AND HABEAS DATA IN Petitioner’s prayer for issuance of a temporary protection order and inspection order is
FAVOR OF NORIEL H. RODRIGUEZ, NORIEL H. RODRIGUEZ, Petitioner, DENIED.
vs.
GLORIA MACAPAGAL-ARROYO, GEN. VICTOR S. IBRADO, PDG JESUS AME VERSOZA, LT. Noriel Rodriguez (Rodriguez) is petitioner in G.R. No. 191805 and respondent in G.R. No.
GEN. DELFIN BANGIT, MAJ. GEN. NESTOR Z. OCHOA, P/CSUPT. AMETO G. TOLENTINO, 193160. He is a member of Alyansa Dagiti Mannalon Iti Cagayan (Kagimungan), a peasant
P/SSUPT. JUDE W. SANTOS, COL. REMIGIO M. DE VERA, an officer named MATUTINA, LT. organization affiliated with Kilusang Magbubukid ng Pilipinas (KMP).
COL. MINA, CALOG, GEORGE PALACPAC under the name "HARRY," ANTONIO CRUZ,
ALDWIN "BONG" PASICOLAN and VINCENT CALLAGAN,Respondents. On the other hand, Gloria Macapagal-Arroyo (former President Arroyo), Police Director
General (PDG.) Jesus A. Verzosa, Police Senior Superintendent (P/SSupt.) Jude W. Santos,
DECISION Brigadier General (Brig. Gen.) Remegio M. De Vera, First Lieutenant (1st Lt.) Ryan S.
Matutina, Lieutenant Colonel (Lt. Col.) Laurence E. Mina, Antonio C. Cruz (Cruz), Aldwin C.
SERENO, J.: Pasicolan (Pasicolan) and Vicente A. Callagan (Callagan) are respondents in G.R. No. 191805
and petitioners in G.R. No. 193160. At the time the events relevant to the present Petitions
occurred, former President Arroyo was the President of the Philippines. PDG. Verzosa,
Before this Court are two consolidated cases, namely, (1) Petition for Partial Review on
P/SSupt. Santos, Brig. Gen. De Vera, 1st Lt. Matutina and Lt. Col. Mina were officers of the
Certiorari dated 20 April 2010 (G.R. No. 191805), and (2) Petition for Review on Certiorari
Philippine National Police (PNP). Cruz, Pasicolan and Callagan were Special Investigators of
dated 19 August 2010 (G.R. No. 193160).1 Both Petitions assail the 12 April 2010 Decision of
the Commission on Human Rights (CHR) in Region II.
the Court of Appeals, the dispositive portion of which reads:

Antecedent Facts
WHEREFORE, the petition for writ of amparo and writ of habeas data is GRANTED.

Rodriguez claims that the military tagged KMP as an enemy of the State under the Oplan
Respondents Gen. Victor S. Ibrado, Lt. Gen. Delfin Bangit, Maj. Gen. Nestor Z. Ochoa, PCSupt.
Bantay Laya, making its members targets of extrajudicial killings and enforced
Ameto G. Tolentino, PSSupt. Jude W. Santos, Col. Remigio M. De Vera, Lt. Col. Laurence E.
disappearances.2
Mina and 1Lt. Ryan S. Matutina, or their replacements in their official posts if they have
already vacated the same, are ORDERED to furnish this Court within five (5) days from notice
of this decision, official or unofficial reports pertaining to petitioner – covering but not On 6 September 2009, at 5:00 p.m., Rodriguez had just reached Barangay Tapel, Cagayan
limited to intelligence reports, operation reports and provost marshal reports prior to, during onboard a tricycle driven by Hermie Antonio Carlos (Carlos), when four men forcibly took him
and subsequent to September 6, 2009 – made by the 5th Infantry Division, Philippine Army, and forced him into a car. Inside the vehicle were several men in civilian clothes, one of
its branches and subsidiaries, including the 17th Infantry Battalion, Philippine Army. whom was holding a .45 caliber pistol. Subsequently, three more persons arrived, and one of
them carried a gun at his side. Two men boarded the car, while the others rode on the
tricycle.3
The above-named respondents are also DIRECTED to refrain from using the said reports in
any transaction or operation of the military. Necessarily, the afore-named respondents are
ORDERED to expunge from the records of the military all documents having any reference to The men tied the hands of Rodriguez, ordered him to lie on his stomach, sat on his back and
petitioner. started punching him. The car travelled towards the direction of Sta. Teresita-Mission and
moved around the area until about 2:00 a.m. During the drive, the men forced Rodriguez to
confess to being a member of the New People’s Army (NPA), but he remained silent. The car
Likewise, the afore-named respondents, as well as respondents Police Director General Jesus
then entered a place that appeared to be a military camp. There were soldiers all over the
Ame Versoza, Antonio Cruz, Aldwin Pasicolan and Vicente Callagan are DIRECTED to ensure
area, and there was a banner with the word "Bravo" written on it. Rodriguez later on learned
that no further violation of petitioner’s rights to life, liberty and security is committed against
that the camp belonged to the 17th Infantry Battalion of the Philippine Army.4
the latter or any member of his family.
30

Rodriguez was brought to a canteen, where six men confronted him, ordering him to confess hit him again. After giving him a pen and a piece of paper, they ordered him to write down
to his membership in the NPA. Due to his exhaustion, he unintentionally fell asleep. As a his request for rice from the people. When he refused, the soldiers maltreated him once
result, the men hit him on the head to wake him up. After the interrogation, two of the men more.10
guarded him, but did not allow him to sleep.5
On 13 September 2009, the soldiers forced Rodriguez to sign documents declaring that he
In the morning of 7 September 2009, the men tied the hands of Rodriguez, blindfolded him had surrendered in an encounter in Cumao, and
and made him board a vehicle. While they were in transit, the soldiers repeatedly hit him in
the head and threatened to kill him. When the car stopped after about ten minutes, the that the soldiers did not shoot him because he became a military asset in May. When he
soldiers brought him to a room, removed his blindfold, and forced him to confess to being a refused to sign the document, he received another beating. Thus, he was compelled to sign,
member of the NPA. During the interrogation, the soldiers repeatedly hit him on the head. but did so using a different signature to show that he was merely coerced.11
Thereafter, he was detained inside the room for the entire day. The soldiers tied his stomach
to a papag, and gave him rice and viand. Fearing that the food might be poisoned, he refused
The soldiers showed Rodriguez photographs of different persons and asked him if he knew
to eat anything. He slept on the papag while being tied to it at the waist.6
the men appearing therein. When he told them that he did not recognize the individuals on
the photos, the soldiers instructed him to write down the name of his school and
On 8 September 2009, the men forced Rodriguez into a vehicle, which brought them to organization, but he declined. The soldiers then wrote something on the paper, making it
Bugey and Mission. While passing houses along the way, the men asked him if his contacts appear that he was the one who had written it, and forced him to sign the document. The
lived in those houses. When he failed to answer, a soldier pointed a gun to his head and soldiers took photographs of him while he was signing. Afterwards, the soldiers forced him
threatened to kill him and his family. Because he remained silent, the soldiers beat him and down, held his hands, and sat on his feet. He did not only receive another beating, but was
tied him up. The vehicle returned to the military camp at past 1:00 p.m., where he was again also electrocuted. The torture lasted for about an hour.12
subjected to tactical interrogation about the location of an NPA camp and his alleged NPA
comrades. He suffered incessant mauling every time he failed to answer.7
At 11:00 p.m. on 15 September 2009, the soldiers brought Rodriguez to a military operation
in the mountains, where he saw Matutina again. They all spent the night there.13
At dawn on 9 September 2009, soldiers armed with rifles took Rodriguez and made him their
guide on their way to an NPA camp in Birao. Accompanying them was a man named Harry,
In the morning of 16 September 2009, the soldiers and Rodriguez started their descent.
who, according to the soldiers, was an NPA member who had surrendered to the military.
When they stopped, the soldiers took his photograph and asked him to name the location of
Harry pointed to Rodriguez and called him a member of the NPA. He also heard Harry tell the
the NPA camp. Thereafter, they all returned to the military camp. The soldiers asked him to
soldiers that the latter knew the area well and was acquainted with a man named Elvis. The
take a bath and wear a white polo shirt handed to him. He was then brought to the Enrile
soldiers loaded Rodriguez into a military truck and drove to Tabbak, Bugey. While he was
Medical Center, where Dr. Juliet Ramil (Dr. Ramil) examined him.14 When the doctor asked
walking with the soldiers, he noticed a soldier with the name tag "Matutina," who appeared
him why he had bruises and contusions, he lied and told her that he sustained them when he
to be an official because the other soldiers addressed him as "sir."8
slipped, as he noticed a soldier observing him. Dr. Ramil’s medical certificate indicated that
he suffered from four hematomas in the epigastric area, chest and sternum.15
Upon reaching Birao on foot, the soldiers looked for and was able to locate a certain Elvis and
told him that Rodriguez had identified his whereabouts location. The soldiers forced
Back at the camp, the soldiers let Rodriguez eat with several military officials and took
Rodriguez to convince Elvis to disclose the location of the NPA camp. They brought the two
pictures of him while he was eating with them. They also asked him to point to a map in front
to the mountains, where both were threatened with death. When the soldiers punched Elvis,
of him and again took his photograph. Later, they told him that he would finally see his
Rodriguez told them that he would reveal the location of the NPA camp if they let Elvis go
mother. 16
home. They finally released Elvis around 3:00 p.m. that day. The soldiers and Rodriguez spent
the next three nights in the mountains.9
Rodriguez was brought to another military camp, where he was ordered to sign a piece of
paper stating that he was a surrenderee and was never beaten up. Scared and desperate to
On 12 September 2009, the soldiers again hit Rodriguez and forced him to identify the
end his ordeal, he signed the paper and was warned not to report anything to the media.17
location of the NPA camp. He was blindfolded and warned to get ready because they would
beat him up again in the military camp. Upon arrival therein, they brought him to the same
room where he had first been detained, and two soldiers mauled him again. They repeatedly Around 6:00 a.m. on 17 September 2009, the soldiers instructed petitioner to take a bath.
punched and kicked him. In the afternoon, they let him rest and gave him an Alaxan tablet. They gave him a pair of jeans and perfume. While he was having breakfast, the two soldiers
Thereafter, he fell asleep due to over-fatigue and extreme body pain. The soldiers, however,
31

guarding him repeatedly reminded him not to disclose to the media his experience in the Around 7:00 a.m. on 3 November 2010, Rodriguez and his girlfriend, Aileen Hazel Robles,
camp and to say instead that he had surrendered to the military.18 noticed that several suspicious-looking men followed them at the Metro Rail Transit (MRT),
in the streets and on a jeepney.25
At 9:00 a.m. on the same day, the mother and the brother of Rodriguez arrived surrounded
by several men. His mother, Wilma Rodriguez (Wilma), talked to Lt. Col. Mina. Rodriguez On 7 December 2009, Rodriguez filed before this Court a Petition for the Writ of Amparo and
heard one of the soldiers tell Wilma that he had surrendered to the military and had long Petition for the Writ of Habeas Data with Prayers for Protection Orders, Inspection of Place,
been its asset. His brother, Rodel Rodriguez (Rodel), informed him that the men and Production of Documents and Personal Properties dated 2 December 2009.26 The
accompanying them were from the CHR, namely, Pasicolan, Cruz and Callagan. Upon seeing petition was filed against former President Arroyo, Gen. Ibrado, PDG. Versoza, Lt. Gen.
Rodriguez, Cruz instructed him to lift up his shirt, and one of the CHR employees took Bangit, Major General (Maj. Gen.) Nestor Z. Ochoa, P/CSupt. Tolentino, P/SSupt. Santos, Col.
photographs of his bruises.19 De Vera, 1st Lt. Matutina, Calog, George Palacpac (Palacpac), Cruz, Pasicolan and Callagan.
The petition prayed for the following reliefs:
A soldier tried to convince Wilma to let Rodriguez stay in the camp for another two weeks to
supposedly prevent the NPA from taking revenge on him. Respondent Calog also approached a. The issuance of the writ of amparo ordering respondents to desist from violating
Rodriguez and Rodel and asked them to become military assets. Rodel refused and insisted Rodriguez’s right to life, liberty and security.
that they take Rodriguez home to Manila. Again, the soldiers reminded them to refrain from
facing the media. The soldiers also told them that the latter will be taken to the Tuguegarao b. The issuance of an order to enjoin respondents from doing harm to or
Airport and guarded until they reached home.20 approaching Rodriguez, his family and his witnesses.

Rodriguez and his family missed their flight. Subsequently, the soldiers accompanied them to c. Allowing the inspection of the detention areas of the Headquarters of Bravo Co.,
the CHR office, where Rodriguez was made to sign an affidavit stating that he was neither 5th Infantry Division, Maguing, Gonzaga, Cagayan and another place near where
abducted nor tortured. Afraid and desperate to return home, he was forced to sign the Rodriguez was brought.
document. Cruz advised him not to file a case against his abductors because they had already
freed him. The CHR personnel then led him and his family to the CHR Toyota Tamaraw FX
d. Ordering respondents to produce documents submitted to them regarding any
service vehicle. He noticed that a vehicle with soldiers on board followed them.21
report on Rodriguez, including operation reports and provost marshall reports of
the 5th Infantry Division, the Special Operations Group of the Armed Forces of the
The Tamaraw FX pulled over and respondent 1st Lt. Matutina boarded the vehicle. Upon Philippines (AFP), prior to, on and subsequent to 6 September 2009.1âwphi1
reaching a mall in Isabela, Rodriguez, his family, Callagan, 1st Lt. Matutina and two other
soldiers transferred to an orange Toyota Revo with plate number WTG 579. Upon reaching
e. Ordering records pertinent or in any way connected to Rodriguez, which are in
the boundary of Nueva Ecija and Nueva Viscaya, 1st Lt. Matutina alighted and called
the custody of respondents, to be expunged, disabused, and forever barred from
Rodriguez to a diner. A certain Alan approached Rodriguez and handed him a cellphone with
being used.27
a SIM card. The latter and his family then left and resumed their journey back home.22

On 15 December 2009, we granted the respective writs after finding that the petition
Rodriguez reached his house in Sta. Ana, Manila at 3:00 a.m. on 18 September 2010. Callagan
sufficiently alleged that Rodriguez had been abducted, tortured and later released by
and two soldiers went inside the house, and took photographs and a video footage thereof.
members of the 17th Infantry Battalion of the Philippine Army.28 We likewise ordered
The soldiers explained that the photos and videos would serve as evidence of the fact that
respondents therein to file a verified return on the writs on or before 22 December 2009 and
Rodriguez and his family were able to arrive home safely. Despite Rodriguez’s efforts to
to comment on the petition on or before 4 January 2010.29 Finally, we directed the Court of
confront the soldiers about their acts, they still continued and only left thirty minutes later.23
Appeals to hear the petition on 4 January 2010 and decide on the case within 10 days after
its submission for decision.30
On 19 September 2009, Dr. Reginaldo Pamugas, a physician trained by the International
Committee on Torture and Rehabilitation, examined Rodriguez and issued a Medical
During the initial hearing on 4 January 2010, the Court of Appeals required the parties to
Certificate stating that the latter had been a victim of torture.24
submit affidavits and other pieces of evidence at the next scheduled hearing on 27 January
2010.31
32

On 8 January 2010, respondents therein, through the Office of the Solicitor General (OSG), During the hearing on 27 January 2010, the parties agreed to file additional affidavits and
filed their Return of the Writ, which was likewise considered as their comment on the position papers and to have the case considered submitted for decision after the filing of
petition.32 In their Return, respondents therein alleged that Rodriguez had surrendered to these pleadings.50
the military on 28 May 2009 after he had been put under surveillance and identified as "Ka
Pepito" by former rebels.33 According to his military handlers, Corporal (Cpl.) Rodel On 12 April 2010, the Court of Appeals rendered its assailed Decision.51 Subsequently, on 28
April 2010, respondents therein filed their Motion for Reconsideration.52 Before the Court of
B. Cabaccan and Cpl. Julius P. Navarro, Rodriguez was a former member of the NPA operating Appeals could resolve this Motion for Reconsideration, Rodriguez filed the instant Petition
in Cagayan Valley.34 Wanting to bolt from the NPA, he told Cpl. Cabaccan and Cpl. Navarro for Partial Review on Certiorari (G.R. No. 191805), raising the following assignment of errors:
that he would help the military in exchange for his protection.35
a. The Court of Appeals erred in not granting the Interim Relief for temporary
Upon his voluntary surrender on 28 May 2009, Rodriguez was made to sign an Oath of protection order.
Loyalty and an Agent’s Agreement/Contract, showing his willingness to return to society and
become a military asset.36 Since then, he acted as a double agent, returning to the NPA to b. The Court of Appeals erred in saying: "(H)owever, given the nature of the writ of
gather information.37 However, he feared that his NPA comrades were beginning to suspect amparo, which has the effect of enjoining the commission by respondents of
him of being an infiltrator.38 Thus, with his knowledge and consent, the soldiers planned to violation to petitioner’s right to life, liberty and security, the safety of petitioner is
stage a sham abduction to erase any suspicion about him being a double agent.39 Hence, the ensured with the issuance of the writ, even in the absence of an order preventing
abduction subject of the instant petition was conducted.40 respondent from approaching petitioner."

Meanwhile, Cruz, Pasicolan and Callagan filed a Consolidated Return of the Writ dated 15 c. The Court of Appeals erred in not finding that respondent Gloria Macapagal
January 2010,41 alleging that they had exercised extraordinary diligence in locating Arroyo had command responsibility.53
Rodriguez, facilitating his safe turnover to his family and securing their journey back home to
Manila. More specifically, they alleged that, on 16 September 2009, after Wilma sought their
On the other hand, respondents therein, in their Comment dated 30 July 2010, averred:
assistance in ascertaining the whereabouts of her son, Cruz made phone calls to the military
and law enforcement agencies to determine his location.42 Cruz was able to speak with Lt.
Col. Mina, who confirmed that Rodriguez was in their custody.43 This information was a. The Court of Appeals properly dropped then President Gloria Macapagal Arroyo
transmitted to CHR Regional Director Atty. Jimmy P. Baliga. He, in turn, ordered Cruz, as a party-respondent, as she may not be sued in any case during her tenure of
Pasicolan and Callagan to accompany Wilma to the 17th Infantry Division.44 office or actual incumbency.

When the CHR officers, along with Wilma and Rodel, arrived at the 17th Infantry Battalion at b. Petitioner had not presented any adequate and competent evidence, much less
Masin, Alcala, Cagayan, Brigade Commander Col. de Vera and Battalion Commander Lt. Col. substantial evidence, to establish his claim that public respondents had violated,
Mina alleged that Rodriguez had become one of their assets, as evidenced by the Summary were violating or threatening to violate his rights to life, liberty and security, as well
on the Surrender of Noriel Rodriguez and the latter’s Contract as Agent.45 The CHR officers as his right to privacy. Hence, he was not entitled to the privilege of the writs of
observed his casual and cordial demeanor with the soldiers.46 In any case, Cruz asked him to amparo and habeas data or to the corresponding interim reliefs (i.e. inspection
raise his shirt to see if he had been subjected to any maltreatment. Cruz and Pasicolan did order, production order and temporary protection order) provided under the rule
not see any traces of torture. Thereafter, Rodriguez was released to his family, and they were on the writ of amparo and the rule on the writ of habeas data.54
made to sign a certification to this effect. During the signing of the document, herein CHR
officers did not witness any threat, intimidation or force employed against Rodriguez or his On 19 August 2010, PDG. Verzosa, P/SSupt. Santos, BGen. De Vera, 1st Lt. Matutina, Lt. Col.
family. 47 Mina, Cruz, Pasicolan and Callagan filed a Petition for Review on Certiorari, seeking the
reversal of the 12 April 2010 Decision of the Court of Appeals.55 They alleged that Rodriguez
During their journey back to the home of Rodriguez, the CHR officers observed that he was –
very much at ease with his military escorts, especially with 1st Lt. Matutina.48 Neither was
there any force or intimidation when the soldiers took pictures of his house, as the taking of Has not presented any adequate and competent evidence, must less substantial evidence, to
photographs was performed with Wilma’s consent.49 establish his claim that petitioners have violated, are violating or threatening with violation
his rights to life, liberty and security, as well as his right to privacy; hence, he is not entitled
to the privilege of the writs of amparo and habeas data and their corresponding interim
33

reliefs (i.e., inspection order, production order and temporary protection order) provided information contains erroneous data or information, order its deletion, destruction or
under the Rule on the Writ of Amparo and the Rule on the Writ of Habeas Data.56 rectification.67

In ascertaining whether the Court of Appeals committed reversible error in issuing its First issue: Grant of interim reliefs
assailed Decision and Resolution, the following issues must be resolved:
In the petition in G.R. No. 191805, Rodriguez prays for the issuance of a temporary
I. Whether the interim reliefs prayed for by Rodriguez may be granted after the protection order. It must be underscored that this interim relief is only available before final
writs of amparo and habeas data have already been issued in his favor. judgment. Section 14 of the Rule on the Writ of Amparo clearly provides:

II. Whether former President Arroyo should be dropped as a respondent on the Interim Reliefs. – Upon filing of the petition or at anytime before final judgment, the court,
basis of the presidential immunity from suit. justice or judge may grant any of the following reliefs:

III. Whether the doctrine of command responsibility can be used in amparo and Temporary Protection Order. – The court, justice or judge, upon motion or motu proprio,
habeas data cases. may order that the petitioner or the aggrieved party and any member of the immediate
family be protected in a government agency or by an accredited person or private institution
IV. Whether the rights to life, liberty and property of Rodriguez were violated or capable of keeping and securing their safety. If the petitioner is an organization, association
threatened by respondents in G.R. No. 191805. or institution referred to in Section 3(c) of this Rule, the protection may be extended to the
officers involved.
At the outset, it must be emphasized that the writs of amparo and habeas data were
promulgated to ensure the protection of the people’s rights to life, liberty and security. 57 The The Supreme Court shall accredit the persons and private institutions that shall extend
rules on these writs were issued in light of the alarming prevalence of extrajudicial killings temporary protection to the petitioner or the aggrieved party and any member of the
and enforced disappearances.58 The Rule on the Writ of Amparo took effect on 24 October immediate family, in accordance with guidelines which it shall issue.
2007,59 and the Rule on the Writ of Habeas Data on 2 February 2008.60
The accredited persons and private institutions shall comply with the rules and conditions
The writ of amparo is an extraordinary and independent remedy that provides rapid judicial that may be imposed by the court, justice or judge.
relief, as it partakes of a summary proceeding that requires only substantial evidence to
make the appropriate interim and permanent reliefs available to the petitioner.61 It is not an (a) Inspection Order. – The court, justice or judge, upon verified motion and after due
action to determine criminal guilt requiring proof beyond reasonable doubt, or liability for hearing, may order any person in possession or control of a designated land or other
damages requiring preponderance of evidence, or administrative responsibility requiring property, to permit entry for the purpose of inspecting, measuring, surveying, or
substantial evidence that will require full and exhaustive proceedings.62 Rather, it serves both photographing the property or any relevant object or operation thereon.
preventive and curative roles in addressing the problem of extrajudicial killings and enforced
disappearances.63 It is preventive in that it breaks the expectation of impunity in the The motion shall state in detail the place or places to be inspected. It shall be supported by
commission of these offenses, and it is curative in that it facilitates the subsequent affidavits or testimonies of witnesses having personal knowledge of the enforced
punishment of perpetrators by inevitably leading to subsequent investigation and action.64 disappearance or whereabouts of the aggrieved party.

Meanwhile, the writ of habeas data provides a judicial remedy to protect a person’s right to If the motion is opposed on the ground of national security or of the privileged nature of the
control information regarding oneself, particularly in instances where such information is information, the court, justice or judge may conduct a hearing in chambers to determine the
being collected through unlawful means in order to achieve unlawful ends.65 As an merit of the opposition.
independent and summary remedy to protect the right to privacy – especially the right to
informational privacy66 – the proceedings for the issuance of the writ of habeas data does
The movant must show that the inspection order is necessary to establish the right of the
not entail any finding of criminal, civil or administrative culpability. If the allegations in the
aggrieved party alleged to be threatened or violated.
petition are proven through substantial evidence, then the Court may (a) grant access to the
database or information; (b) enjoin the act complained of; or (c) in case the database or
34

The inspection order shall specify the person or persons authorized to make the inspection It does not determine guilt nor pinpoint criminal culpability for the disappearance; rather, it
and the date, time, place and manner of making the inspection and may prescribe other determines responsibility, or at least accountability, for the enforced disappearance for
conditions to protect the constitutional rights of all parties. The order shall expire five (5) purposes of imposing the appropriate remedies to address the disappearance. Responsibility
days after the date of its issuance, unless extended for justifiable reasons. refers to the extent the actors have been established by substantial evidence to have
participated in whatever way, by action or omission, in an enforced disappearance, as a
(b) Production Order. – The court, justice, or judge, upon verified motion and after due measure of the remedies this Court shall craft, among them, the directive to file the
hearing, may order any person in possession, custody or control of any designated appropriate criminal and civil cases against the responsible parties in the proper courts.
documents, papers, books, accounts, letters, photographs, objects or tangible things, or Accountability, on the other hand, refers to the measure of remedies that should be
objects in digitized or electronic form, which constitute or contain evidence relevant to the addressed to those who exhibited involvement in the enforced disappearance without
petition or the return, to produce and permit their inspection, copying or photographing by bringing the level of their complicity to the level of responsibility defined above; or who are
or on behalf of the movant. imputed with knowledge relating to the enforced disappearance and who carry the burden
of disclosure; or those who carry, but have failed to discharge, the burden of extraordinary
diligence in the investigation of the enforced disappearance. In all these cases, the issuance
The motion may be opposed on the ground of national security or of the privileged nature of
of the Writ of Amparo is justified by our primary goal of addressing the disappearance, so
the information, in which case the court, justice or judge may conduct a hearing in chambers
that the life of the victim is preserved and his liberty and security are restored.70 (Emphasis
to determine the merit of the opposition.
supplied.)

The court, justice or judge shall prescribe other conditions to protect the constitutional rights
Thus, in the case at bar, the Court of Appeals, in its Decision71 found respondents in G.R. No.
of all the parties.
191805 – with the exception of Calog, Palacpac or Harry – to be accountable for the
violations of Rodriguez’s right to life, liberty and security committed by the 17th Infantry
(c) Witness Protection Order. – The court, justice or judge, upon motion or motu proprio, Battalion, 5th Infantry Division of the Philippine Army. 72 The Court of Appeals dismissed the
may refer the witnesses to the Department of Justice for admission to the Witness petition with respect to former President Arroyo on account of her presidential immunity
Protection, Security and Benefit Program, pursuant to Republic Act No. 6981. from suit. Rodriguez contends, though, that she should remain a respondent in this case to
enable the courts to determine whether she is responsible or accountable therefor. In this
The court, justice or judge may also refer the witnesses to other government agencies, or to regard, it must be clarified that the Court of Appeals’ rationale for dropping her from the list
accredited persons or private institutions capable of keeping and securing their safety. of respondents no longer stands since her presidential immunity is limited only to her
(Emphasis supplied) incumbency.

We held in Yano v. Sanchez68 that "[t]hese provisional reliefs are intended to assist the court In Estrada v. Desierto,73 we clarified the doctrine that a non-sitting President does not enjoy
before it arrives at a judicious determination of the amparo petition." Being interim reliefs, immunity from suit, even for acts committed during the latter’s tenure. We emphasize our
they can only be granted before a final adjudication of the case is made. In any case, it must ruling therein that courts should look with disfavor upon the presidential privilege of
be underscored that the privilege of the writ of amparo, once granted, necessarily entails the immunity, especially when it impedes the search for truth or impairs the vindication of a
protection of the aggrieved party. Thus, since we grant petitioner the privilege of the writ of right, to wit:
amparo, there is no need to issue a temporary protection order independently of the former.
The order restricting respondents from going near Rodriguez is subsumed under the privilege We reject [Estrada’s] argument that he cannot be prosecuted for the reason that he must
of the writ. first be convicted in the impeachment proceedings. The impeachment trial of petitioner
Estrada was aborted by the walkout of the prosecutors and by the events that led to his loss
Second issue: Presidential immunity from suit of the presidency. Indeed, on February 7, 2001, the Senate passed Senate Resolution No. 83
"Recognizing that the Impeachment Court is Functus Officio." Since the Impeachment Court
It bears stressing that since there is no determination of administrative, civil or criminal is now functus officio, it is untenable for petitioner to demand that he should first be
liability in amparo and habeas data proceedings, courts can only go as far as ascertaining impeached and then convicted before he can be prosecuted. The plea if granted, would put a
responsibility or accountability for the enforced disappearance or extrajudicial killing. As we perpetual bar against his prosecution. Such a submission has nothing to commend itself for it
held in Razon v. Tagitis:69 will place him in a better situation than a non-sitting President who has not been subjected
to impeachment proceedings and yet can be the object of a criminal prosecution. To be sure,
the debates in the Constitutional Commission make it clear that when impeachment
35

proceedings have become moot due to the resignation of the President, the proper criminal the US President's immunity from suits for money damages arising out of their official acts is
and civil cases may already be filed against him, viz: inapplicable to unofficial conduct.74 (Emphasis supplied)

Mr. Aquino. On another point, if an impeachment proceeding has been filed against the Further, in our Resolution in Estrada v. Desierto,75 we reiterated that the presidential
President, for example, and the President resigns before judgment of conviction has been immunity from suit exists only in concurrence with the president’s incumbency:
rendered by the impeachment court or by the body, how does it affect the impeachment
proceeding? Will it be necessarily dropped? Petitioner stubbornly clings to the contention that he is entitled to absolute immunity from
suit. His arguments are merely recycled and we need not prolong the longevity of the debate
Mr. Romulo. If we decide the purpose of impeachment to remove one from office, then his on the subject. In our Decision, we exhaustively traced the origin of executive immunity in
resignation would render the case moot and academic. However, as the provision says, the our jurisdiction and its bends and turns up to the present time. We held that given the intent
criminal and civil aspects of it may continue in the ordinary courts." of the 1987 Constitution to breathe life to the policy that a public office is a public trust, the
petitioner, as a non-sitting President, cannot claim executive immunity for his alleged
This is in accord with our ruling in In Re: Saturnino Bermudez that "incumbent Presidents are criminal acts committed while a sitting President. Petitioner's rehashed arguments including
immune from suit or from being brought to court during the period of their incumbency and their thinly disguised new spins are based on the rejected contention that he is still
tenure" but not beyond. xxx President, albeit, a President on leave. His stance that his immunity covers his entire term of
office or until June 30, 2004 disregards the reality that he has relinquished the presidency
and there is now a new de jure President.
We now come to the scope of immunity that can be claimed by petitioner as a non-sitting
President. The cases filed against petitioner Estrada are criminal in character. They involve
plunder, bribery and graft and corruption. By no stretch of the imagination can these crimes, Petitioner goes a step further and avers that even a non-sitting President enjoys immunity
especially plunder which carries the death penalty, be covered by the alleged mantle of from suit during his term of office. He buttresses his position with the deliberations of the
immunity of a non-sitting president. Petitioner cannot cite any decision of this Court licensing Constitutional Commission, viz:
the President to commit criminal acts and wrapping him with post-tenure immunity from
liability. It will be anomalous to hold that immunity is an inoculation from liability for "Mr. Suarez. Thank you.
unlawful acts and omissions. The rule is that unlawful acts of public officials are not acts of
the State and the officer who acts illegally is not acting as such but stands in the same footing The last question is with reference to the Committee's omitting in the draft proposal the
as any other trespasser. immunity provision for the President. I agree with Commissioner Nolledo that the Committee
did very well in striking out this second sentence, at the very least, of the original provision
Indeed, a critical reading of current literature on executive immunity will reveal a judicial on immunity from suit under the 1973 Constitution. But would the Committee members not
disinclination to expand the privilege especially when it impedes the search for truth or agree to a restoration of at least the first sentence that the president shall be immune from
impairs the vindication of a right. In the 1974 case of US v. Nixon, US President Richard suit during his tenure, considering that if we do not provide him that kind of an immunity, he
Nixon, a sitting President, was subpoenaed to produce certain recordings and documents might be spending all his time facing litigations, as the President-in-exile in Hawaii is now
relating to his conversations with aids and advisers. Seven advisers of President Nixon's facing litigations almost daily?
associates were facing charges of conspiracy to obstruct justice and other offenses which
were committed in a burglary of the Democratic National Headquarters in Washington's Fr. Bernas: The reason for the omission is that we consider it understood in present
Watergate Hotel during the 1972 presidential campaign. President Nixon himself was named jurisprudence that during his tenure he is immune from suit.
an unindicted co-conspirator. President Nixon moved to quash the subpoena on the ground,
among others, that the President was not subject to judicial process and that he should first
Mr. Suarez: So there is no need to express it here.
be impeached and removed from office before he could be made amenable to judicial
proceedings. The claim was rejected by the US Supreme Court. It concluded that "when the
ground for asserting privilege as to subpoenaed materials sought for use in a criminal trial is Fr. Bernas: There is no need. It was that way before. The only innovation made by the 1973
based only on the generalized interest in confidentiality, it cannot prevail over the Constitution was to make that explicit and to add other things.
fundamental demands of due process of law in the fair administration of criminal justice." In
the 1982 case of Nixon v. Fitzgerald, the US Supreme Court further held that the immunity of Mr. Suarez: On the understanding, I will not press for any more query, madam President.
the President from civil damages covers only "official acts." Recently, the US Supreme Court
had the occasion to reiterate this doctrine in the case of Clinton v. Jones where it held that
36

I thank the Commissioner for the clarification." Precisely in the case at bar, the doctrine of command responsibility may be used to
determine whether respondents are accountable for and have the duty to address the
Petitioner, however, fails to distinguish between term and tenure. The term means the time abduction of Rodriguez in order to enable the courts to devise remedial measures to protect
during which the officer may claim to hold the office as of right, and fixes the interval after his rights. Clearly, nothing precludes this Court from applying the doctrine of command
which the several incumbents shall succeed one another. The tenure represents the term responsibility in amparo proceedings to ascertain responsibility and accountability in
during which the incumbent actually holds office. The tenure may be shorter than the term extrajudicial killings and enforced disappearances. In this regard, the Separate Opinion of
for reasons within or beyond the power of the incumbent. From the deliberations, the intent Justice Conchita Carpio-Morales in Rubrico is worth noting, thus:
of the framers is clear that the immunity of the president from suit is concurrent only with
his tenure and not his term.76(Emphasis supplied) That proceedings under the Rule on the Writ of Amparo do not determine criminal, civil or
administrative liability should not abate the applicability of the doctrine of command
Applying the foregoing rationale to the case at bar, it is clear that former President Arroyo responsibility. Taking Secretary of National Defense v. Manalo and Razon v. Tagitis in proper
cannot use the presidential immunity from suit to shield herself from judicial scrutiny that context, they do not preclude the application of the doctrine of command responsibility to
would assess whether, within the context of amparo proceedings, she was responsible or Amparo cases.
accountable for the abduction of Rodriguez.
Manalo was actually emphatic on the importance of the right to security of person and its
Third issue: Command responsibility in amparo proceedings contemporary signification as a guarantee of protection of one’s rights by the government. It
further stated that protection includes conducting effective investigations, organization of
the government apparatus to extend protection to victims of extralegal killings or enforced
To attribute responsibility or accountability to former President Arroyo, Rodriguez contends
disappearances, or threats thereof, and/or their families, and bringing offenders to the bar of
that the doctrine of command responsibility may be applied. As we explained in Rubrico v.
justice.
Arroyo,77 command responsibility pertains to the "responsibility of commanders for crimes
committed by subordinate members of the armed forces or other persons subject to their
control in international wars or domestic conflict."78 Although originally used for ascertaining Tagitis, on the other hand, cannot be more categorical on the application, at least in
criminal complicity, the command responsibility doctrine has also found application in civil principle, of the doctrine of command responsibility:
cases for human rights abuses.79 In the United States, for example, command responsibility
was used in Ford v. Garcia and Romagoza v. Garcia – civil actions filed under the Alien Tort Given their mandates, the PNP and PNP-CIDG officials and members were the ones who were
Claims Act and the Torture Victim Protection Act.80 This development in the use of command remiss in their duties when the government completely failed to exercise the extraordinary
responsibility in civil proceedings shows that the application of this doctrine has been diligence that the Amparo Rule requires. We hold these organizations accountable through
liberally extended even to cases not criminal in nature. Thus, it is our view that command their incumbent Chiefs who, under this Decision, shall carry the personal responsibility of
responsibility may likewise find application in proceedings seeking the privilege of the writ of seeing to it that extraordinary diligence, in the manner the Amparo Rule requires, is applied
amparo. As we held in Rubrico: in addressing the enforced disappearance of Tagitis.

It may plausibly be contended that command responsibility, as legal basis to hold Neither does Republic Act No. 9851 emasculate the applicability of the command
military/police commanders liable for extra-legal killings, enforced disappearances, or responsibility doctrine to Amparo cases. The short title of the law is the "Philippine Act on
threats, may be made applicable to this jurisdiction on the theory that the command Crimes Against International Humanitarian Law, Genocide, and Other Crimes Against
responsibility doctrine now constitutes a principle of international law or customary Humanity." Obviously, it should, as it did, only treat of superior responsibility as a ground for
international law in accordance with the incorporation clause of the Constitution. criminal responsibility for the crimes
covered.http://www.lawphil.net/judjuris/juri2010/feb2010/gr_183871_2010.html -
If command responsibility were to be invoked and applied to these proceedings, it should, at fnt20cm Such limited treatment, however, is merely in keeping with the statute’s purpose
most, be only to determine the author who, at the first instance, is accountable for, and has and not intended to rule out the application of the doctrine of command responsibility to
the duty to address, the disappearance and harassments complained of, so as to enable the other appropriate cases.
Court to devise remedial measures that may be appropriate under the premises to protect
rights covered by the writ of amparo. As intimated earlier, however, the determination Indeed, one can imagine the innumerable dangers of insulating high-ranking military and
should not be pursued to fix criminal liability on respondents preparatory to criminal police officers from the coverage of reliefs available under the Rule on the Writ of Amparo.
prosecution, or as a prelude to administrative disciplinary proceedings under existing The explicit adoption of the doctrine of command responsibility in the present case will only
administrative issuances, if there be any.81 (Emphasis supplied.) bring Manalo and Tagitis to their logical conclusion.
37

In fine, I submit that the Court should take this opportunity to state what the law ought to be defined above; or (ii) who are imputed with knowledge relating to the enforced
if it truly wants to make the Writ of Amparo an effective remedy for victims of extralegal disappearance and who carry the burden of disclosure; or (iii) those who carry, but have
killings and enforced disappearances or threats thereof. While there is a genuine dearth of failed to discharge, the burden of extraordinary diligence in the investigation of the enforced
evidence to hold respondents Gen. Hermogenes Esperon and P/Dir. Gen. Avelino Razon disappearance. Thus, although there is no determination of criminal, civil or administrative
accountable under the command responsibility doctrine, the ponencia’s hesitant application liabilities, the doctrine of command responsibility may nevertheless be applied to ascertain
of the doctrine itself is replete with implications abhorrent to the rationale behind the Rule responsibility and accountability within these foregoing definitions.
on the Writ of Amparo.82(Emphasis supplied.)
a. Command responsibility of the President
This Separate Opinion was reiterated in the recently decided case of Boac v.
Cadapan,83 likewise penned by Justice Carpio-Morales, wherein this Court ruled: Having established the applicability of the doctrine of command responsibility in amparo
proceedings, it must now be resolved whether the president, as commander-in-chief of the
Rubrico categorically denies the application of command responsibility in amparo cases to military, can be held responsible or accountable for extrajudicial killings and enforced
determine criminal liability. The Court maintains its adherence to this pronouncement as far disappearances. We rule in the affirmative.
as amparo cases are concerned.
To hold someone liable under the doctrine of command responsibility, the following
Rubrico, however, recognizes a preliminary yet limited application of command responsibility elements must obtain:
in amparo cases to instances of determining the responsible or accountable individuals or
entities that are duty-bound to abate any transgression on the life, liberty or security of the a. the existence of a superior-subordinate relationship between the accused as
aggrieved party. superior and the perpetrator of the crime as his subordinate;

If command responsibility were to be invoked and applied to these proceedings, it should, at b. the superior knew or had reason to know that the crime was about to be or had
most, be only to determine the author who, at the first instance, is accountable for, and has been committed; and
the duty to address, the disappearance and harassments complained of, so as to enable the
Court to devise remedial measures that may be appropriate under the premises to protect
c. the superior failed to take the necessary and reasonable measures to prevent the
rights covered by the writ of amparo. As intimated earlier, however, the determination
criminal acts or punish the perpetrators thereof.84
should not be pursued to fix criminal liability on respondents preparatory to criminal
prosecution, or as a prelude to administrative disciplinary proceedings under existing
administrative issuances, if there be any. The president, being the commander-in-chief of all armed forces,85 necessarily possesses
control over the military that qualifies him as a superior within the purview of the command
responsibility doctrine. 86
In other words, command responsibility may be loosely applied in amparo cases in order to
identify those accountable individuals that have the power to effectively implement
whatever processes an amparo court would issue. In such application, the amparo court does On the issue of knowledge, it must be pointed out that although international tribunals apply
not impute criminal responsibility but merely pinpoint the superiors it considers to be in the a strict standard of knowledge, i.e., actual knowledge, such may nonetheless be established
best position to protect the rights of the aggrieved party. through circumstantial evidence.87 In the Philippines, a more liberal view is adopted and
superiors may be charged with constructive knowledge. This view is buttressed by the
enactment of Executive Order No. 226, otherwise known as the Institutionalization of the
Such identification of the responsible and accountable superiors may well be a preliminary
Doctrine of ‘Command Responsibility’ in all Government Offices, particularly at all Levels of
determination of criminal liability which, of course, is still subject to further investigation by
Command in the Philippine National Police and other Law Enforcement Agencies (E.O.
the appropriate government agency. (Emphasis supplied.)
226).88 Under E.O. 226, a government official may be held liable for neglect of duty under the
doctrine of command responsibility if he has knowledge that a crime or offense shall be
As earlier pointed out, amparo proceedings determine (a) responsibility, or the extent the committed, is being committed, or has been committed by his subordinates, or by others
actors have been established by substantial evidence to have participated in whatever way, within his area of responsibility and, despite such knowledge, he did not take preventive or
by action or omission, in an enforced disappearance, and (b) accountability, or the measure corrective action either before, during, or immediately after its commission.89 Knowledge of
of remedies that should be addressed to those (i) who exhibited involvement in the enforced the commission of irregularities, crimes or offenses is presumed when (a) the acts are
disappearance without bringing the level of their complicity to the level of responsibility widespread within the government official’s area of jurisdiction; (b) the acts have been
38

repeatedly or regularly committed within his area of responsibility; or (c) members of his a. The totality of evidence proved by substantial evidence the responsibility or accountability
immediate staff or office personnel are involved.90 of respondents for the violation of or threat to Rodriguez’s right to life, liberty and security.

Meanwhile, as to the issue of failure to prevent or punish, it is important to note that as the After a careful examination of the records of these cases, we are convinced that the Court of
commander-in-chief of the armed forces, the president has the power to effectively Appeals correctly found sufficient evidence proving that the soldiers of the 17th Infantry
command, control and discipline the military.91 Battalion, 5th Infantry Division of the military abducted Rodriguez on 6 September 2009, and
detained and tortured him until 17 September 2009.
b. Responsibility or accountability of former President Arroyo
Rodriguez’s Sinumpaang Salaysay dated 4 December 2009 was a meticulous and
The next question that must be tackled is whether Rodriguez has proven through substantial straightforward account of his horrific ordeal with the military, detailing the manner in which
evidence that former President Arroyo is responsible or accountable for his abduction. We he was captured and maltreated on account of his suspected membership in the NPA.96 His
rule in the negative. narration of his suffering included an exhaustive description of his physical surroundings,
personal circumstances and perceived observations. He likewise positively identified
respondents 1st Lt. Matutina and Lt. Col. Mina to be present during his abduction, detention
Rodriguez anchors his argument on a general allegation that on the basis of the "Melo
and torture,97 and respondents Cruz, Pasicolan and Callagan as the CHR representatives who
Commission" and the "Alston Report," respondents in G.R. No. 191805 already had
appeared during his release.98
knowledge of and information on, and should have known that a climate of enforced
disappearances had been perpetrated on members of the NPA.92 Without even attaching, or
at the very least, quoting these reports, Rodriguez contends that the Melo Report points to More particularly, the fact of Rodriguez’s abduction was corroborated by Carlos in his
rogue military men as the perpetrators. While the Alston Report states that there is a policy Sinumpaang Salaysay dated 16 September 2009,99 wherein he recounted in detail the
allowing enforced disappearances and pins the blame on the President, we do not circumstances surrounding the victim’s capture.
automatically impute responsibility to former President Arroyo for each and every count of
forcible disappearance.93 Aside from Rodriguez’s general averments, there is no piece of As regards the allegation of torture, the respective Certifications of Dr. Ramil and Dr.
evidence that could establish her responsibility or accountability for his abduction. Neither Pamugas validate the physical maltreatment Rodriguez suffered in the hands of the soldiers
was there even a clear attempt to show that she should have known about the violation of of the 17th Infantry Battalion, 5th Infantry Division. According to the Certification dated 12
his right to life, liberty or security, or that she had failed to investigate, punish or prevent it. October 2009 executed by Dr. Ramil,100 she examined Rodriguez in the Alfonso Ponce Enrile
Memorial District Hospital on 16 September 2009 and arrived at the following findings:
Fourth issue: Responsibility or accountability of respondents in G.R. No. 191805
FACE
The doctrine of totality of evidence in amparo cases was first laid down in this Court’s ruling
in Razon,94 to wit: - 10cm healed scar face right side

The fair and proper rule, to our mind, is to consider all the pieces of evidence adduced in - 2cm healed scar right eyebrow (lateral area)
their totality, and to consider any evidence otherwise inadmissible under our usual rules to
be admissible if it is consistent with the admissible evidence adduced. In other words, we - 2cm healed scar right eye brow (median area)
reduce our rules to the most basic test of reason – i.e., to the relevance of the evidence to
the issue at hand and its consistency with all other pieces of adduced evidence. Thus, even
- 4cm x 2cm hematoma anterior chest at the sternal area right side
hearsay evidence can be admitted if it satisfies this basic minimum test.95 (Emphasis
supplied.)
- 3cm x 2cm hematoma sternal area left side
In the case at bar, we find no reason to depart from the factual findings of the Court of
Appeals, the same being supported by substantial evidence. A careful examination of the - 6cm x 1cm hematoma from epigastric area to ant. chest left side
records of this case reveals that the totality of the evidence adduced by Rodriguez
indubitably prove the responsibility and accountability of some respondents in G.R. No. - 6cm x 1cm hematoma from epigastric area to ant. chest right side
191805 for violating his right to life, liberty and security.
39

- Multiple healed rashes (brownish discoloration) both forearm congenial. This contention cannot be sustained, as it is far removed from ordinary human
experience.
- Multiple healed rashes (brownish discoloration)
If it were true that Rodriguez maintained amicable relations with the military, then he should
- both leg arm have unhesitatingly assured his family on 17 September 2009 that he was among friends.
Instead, he vigorously pleaded with them to get him out of the military facility. In fact, in the
Sinumpaang Salaysay dated 4 December 2009104 Wilma executed, she made the following
- hip area/lumbar area101
averments:

Dr. Pamugas performed a separate medical examination of Rodriguez on 19 September 2009,


18. Na nang Makita ko ang aking anak ay nakaramdam ako sa kanya ng awa dahil sa
the results of which confirmed that the injuries suffered by the latter were inflicted through
mukha syang pagod at malaki ang kanyang ipinayat.
torture. Dr. Pamugas thus issued a Medical Report dated 23 September 2009,102 explicitly
stating that Rodriguez had been tortured during his detention by the military, to wit:
19. Na niyakap ko sya at sa aming pagkakayakap ay binulungan nya ako na wag ko
syang iiwan sa lugar na iyon;
X. Interpretation of Findings

23. Na sinabihan ako ng mga sundalo na kung pwede daw ay maiwan muna ng
The above physical and psychological findings sustained by the subject are related to the
dalawang linggo sa kampo ako at si Noriel para daw matrain pa si Noriel sa loob ng
torture and ill-treatment done to him. The multiple circular brown to dark brown spots found
kampo;
on both legs and arms were due to the insect bites that he sustained when he was forced to
join twice in the military operations. The abrasions could also be due to the conditions
related during military operations. The multiple pin-point blood spots found on his left ear is 24. Na hindi ako pumayag na maiwan ang aking anak;
a result of an unknown object placed inside his left ear. The areas of tenderness he felt
during the physical examination were due to the overwhelming punching and kicking on his 33. Na sa kasalukuhan, hanggang ngayon ay nag-aalala pa ako sa paa (sic) sa
body. The occasional difficulty of sleeping is a symptom experience (sic) by the subject as a kaligtasan ng aming buong pamilya, lalo na kay Noriel; xxx105
result of the psychological trauma he encountered during his detention.
Also, Rodel made the following supporting averments in his Sinumpaang Salaysay
XI. Conclusions and Recommendations dated 3 December 2009:106

The physical injuries and psychological trauma suffered by the subject are secondary to the 24. Na nang makita ko si Noriel, hindi sya makalakad ng diretso, hinang-hina sya,
torture and ill-treatment done to him while in detention for about 11 days. The physical malaki ang ipinayat at nanlalalim ang mga mata;
injuries sustained by the subject, of which the age is compatible with the alleged date of
infliction (sic).103 (Emphasis supplied.) 25. Na nang makita ko ang aking kapatid ay nakaramdam ako ng awa dahil nakilala
ko syang masigla at masayahin;
In assessing the weight of the Certifications, the Court of Appeals correctly relied on the
medical finding that the injuries suffered by Rodriguez matched his account of the 26. Na ilang minuto lang ay binulugan nya ako ng "Kuya, ilabas mo ako dito,
maltreatment inflicted on him by the soldiers of the 17th Infantry Battalion, 5th Infantry papatayin nila ako."
Division of the Philippine Army. Further, the kind of injuries he sustained showed that he
could not have sustained them from merely falling, thus making respondents’ claim highly
27. Na sinabihan kami ni Lt. Col. Mina na baka pwedeng maiwan pa ng dalwang
implausible.
linggo ang aking kapatid sa kanila para raw ma-train sya.

Despite these medical findings that overwhelmingly supported and lent credibility to the
28. Na hindi kami pumayag ng aking nanay; xxx107
allegations of Rodriguez in his Sinumpaang Salaysay, respondents in G.R. No. 191805 still
stubbornly clung to their argument that he was neither abducted nor detained. Rather, they
claimed that he was a double agent, whose relationship with the military was at all times Moreover, the Court of Appeals likewise aptly pointed out the illogical, if not outrightly
contradictory, contention of respondents in G.R. No. 191805 that while Rodriguez had
40

complained of his exhaustion from his activities as a member of the CPP-NPA, he insufficient competence in facilitating and ensuring the safe release of Rodriguez after his
nevertheless willingly volunteered to return to his life in the NPA to become a double-agent ordeal.
for the military. The lower court ruled in this manner:
b. The failure to conduct a fair and effect investigation amounted to a violation of or threat
In the Return of the Writ, respondent AFP members alleged that petitioner confided to his to Rodriguez’s rights to life, liberty and security.
military handler, Cpl. Navarro, that petitioner could no longer stand the hardships he
experienced in the wilderness, and that he wanted to become an ordinary citizen again The Rule on the Writ of Amparo explicitly states that the violation of or threat to the right to
because of the empty promises of the CPP-NPA. However, in the same Return, respondents life, liberty and security may be caused by either an act or an omission of a public
state that petitioner agreed to become a double agent for the military and wanted to re- official.111 Moreover, in the context of amparo proceedings, responsibility may refer to the
enter the CPP-NPA, so that he could get information regarding the movement directly from participation of the respondents, by action or omission, in enforced
the source. If petitioner was tired of life in the wilderness and desired to become an ordinary disappearance.112 Accountability, on the other hand, may attach to respondents who are
citizen again, it defies logic that he would agree to become an undercover agent and work imputed with knowledge relating to the enforced disappearance and who carry the burden
alongside soldiers in the mountains – or the wilderness he dreads – to locate the hideout of of disclosure; or those who carry, but have failed to discharge, the burden of extraordinary
his alleged NPA comrades.108 (Emphasis supplied.) diligence in the investigation of the enforced disappearance.113

Furthermore, the appellate court also properly ruled that aside from the abduction, In this regard, we emphasize our ruling in Secretary of National Defense v. Manalo114 that the
detention and torture of Rodriguez, respondents, specifically 1st Lt. Matutina, had violated right to security of a person includes the positive obligation of the government to ensure the
and threatened the former’s right to security when they made a visual recording of his observance of the duty to investigate, viz:
house, as well as the photos of his relatives, to wit:
Third, the right to security of person is a guarantee of protection of one's rights by the
In the videos taken by the soldiers – one of whom was respondent Matutina – in the house of government. In the context of the writ of Amparo, this right is built into the guarantees of
petitioner on September 18, 2009, the soldiers even went as far as taking videos of the the right to life and liberty under Article III, Section 1 of the 1987 Constitution and the right
photos of petitioner’s relatives hung on the wall of the house, as well as videos of the to security of person (as freedom from threat and guarantee of bodily and psychological
innermost part of the house. This Court notes that 1Lt. Matutina, by taking the said videos, integrity) under Article III, Section 2. The right to security of person in this third sense is a
did not merely intend to make proofs of the safe arrival of petitioner and his family in their corollary of the policy that the State "guarantees full respect for human rights" under Article
home. 1Lt. Matutina also desired to instill fear in the minds of petitioner and his family by II, Section 11 of the 1987 Constitution. As the government is the chief guarantor of order and
showing them that the sanctity of their home, from then on, will not be free from the security, the Constitutional guarantee of the rights to life, liberty and security of person is
watchful eyes of the military, permanently captured through the medium of a seemingly rendered ineffective if government does not afford protection to these rights especially
innocuous cellhpone video camera. The Court cannot – and will not – condone such act, as it when they are under threat. Protection includes conducting effective investigations,
intrudes into the very core of petitioner’s right to security guaranteed by the fundamental organization of the government apparatus to extend protection to victims of extralegal
law.109(Emphasis supplied.) killings or enforced disappearances (or threats thereof) and/or their families, and bringing
offenders to the bar of justice. The Inter-American Court of Human Rights stressed the
Taken in their totality, the pieces of evidence adduced by Rodriguez, as well as the importance of investigation in the Velasquez Rodriguez Case, viz:
contradictory defenses presented by respondents in G.R. No. 191805, give credence to his
claim that he had been abducted, detained and tortured by soldiers belonging to the 17th (The duty to investigate) must be undertaken in a serious manner and not as a mere
Infantry Battalion, 5th Infantry Division of the military. formality preordained to be ineffective. An investigation must have an objective and be
assumed by the State as its own legal duty, not as a step taken by private interests that
It must be pointed out, however, that as to respondents Cruz, Pasicolan and Callagan, there depends upon the initiative of the victim or his family or upon their offer of proof, without an
was no substantial evidence to show that they violated, or threatened with violation, effective search for the truth by the government.
Rodriguez’s right to life, liberty and security. Despite the dearth of evidence to show the CHR
officers’ responsibility or accountability, this Court nonetheless emphasizes its criticism as Similarly, the European Court of Human Rights (ECHR) has interpreted the "right to security"
regards their capacity to recognize torture or any similar form of abuse. The CHR, being not only as prohibiting the State from arbitrarily depriving liberty, but imposing a positive
constitutionally mandated to protect human rights and investigate violations duty on the State to afford protection of the right to liberty. The ECHR interpreted the "right
thereof,110 should ensure that its officers are well-equipped to respond effectively to and to security of person" under Article 5(1) of the European Convention of Human Rights in the
address human rights violations. The actuations of respondents unmistakably showed their leading case on disappearance of persons, Kurt v. Turkey. In this case, the claimant's son had
41

been arrested by state authorities and had not been seen since. The family's requests for Clearly, the absence of a fair and effective official investigation into the claims of Rodriguez
information and investigation regarding his whereabouts proved futile. The claimant violated his right to security, for which respondents in G.R. No. 191805 must be held
suggested that this was a violation of her son's right to security of person. The ECHR responsible or accountable.
ruled, viz:
Nevertheless, it must be clarified that Rodriguez was unable to establish any responsibility or
... any deprivation of liberty must not only have been effected in conformity with the accountability on the part of respondents P/CSupt. Tolentino, P/SSupt. Santos, Calog and
substantive and procedural rules of national law but must equally be in keeping with the very Palacpac. Respondent P/CSupt. Tolentino had already retired when the abduction and
purpose of Article 5, namely to protect the individual from arbitrariness... Having assumed torture of Rodriguez was perpetrated, while P/SSupt. Santos had already been reassigned
control over that individual it is incumbent on the authorities to account for his or her and transferred to the National Capital Regional Police Office six months before the subject
whereabouts. For this reason, Article 5 must be seen as requiring the authorities to take incident occurred. Meanwhile, no sufficient allegations were maintained against respondents
effective measures to safeguard against the risk of disappearance and to conduct a prompt Calog and Palacpac.
effective investigation into an arguable claim that a person has been taken into custody
and has not been seen since.115 (Emphasis supplied) From all the foregoing, we rule that Rodriguez was successful in proving through substantial
evidence that respondents Gen. Ibrado, PDG. Verzosa, Lt. Gen. Bangit, Maj. Gen. Ochoa, Brig.
In the instant case, this Court rules that respondents in G.R. No. 191805 are responsible or Gen. De Vera, 1st Lt. Matutina, and Lt. Col. Mina were responsible and accountable for the
accountable for the violation of Rodriguez’s right to life, liberty and security on account of violation of Rodriguez’s rights to life, liberty and security on the basis of (a) his abduction,
their abject failure to conduct a fair and effective official investigation of his ordeal in the detention and torture from 6 September to 17 September 2009, and (b) the lack of any fair
hands of the military. Respondents Gen. Ibrado, PDG. Verzosa, Lt. Gen. Bangit, Maj. Gen. and effective official investigation as to his allegations. Thus, the privilege of the writs of
Ochoa, Col. De Vera and Lt. Col. Mina only conducted a perfunctory investigation, exerting no amparo and habeas data must be granted in his favor. As a result, there is no longer any need
efforts to take Ramirez’s account of the events into consideration. Rather, these respondents to issue a temporary protection order, as the privilege of these writs already has the effect of
solely relied on the reports and narration of the military. The ruling of the appellate court enjoining respondents in G.R. No. 191805 from violating his rights to life, liberty and security.
must be emphasized:
It is also clear from the above discussion that despite (a) maintaining former President
In this case, respondents Ibrado, Verzosa, Bangit, Tolentino, Santos, De Vera, and Mina are Arroyo in the list of respondents in G.R. No. 191805, and (b) allowing the application of the
accountable, for while they were charged with the investigation of the subject incident, the command responsibility doctrine to amparo and habeas data proceedings, Rodriguez failed
investigation they conducted and/or relied on is superficial and one-sided. The records to prove through substantial evidence that former President Arroyo was responsible or
disclose that the military, in investigating the incident complained of, depended on the accountable for the violation of his rights to life, liberty and property. He likewise failed to
Comprehensive Report of Noriel Rodriguez @Pepito prepared by 1Lt. Johnny Calub for the prove through substantial evidence the accountability or responsibility of respondents Maj.
Commanding Officer of the 501st Infantry Brigade, 5th Infantry Division, Philippine Army. Gen. Ochoa, Cruz, Pasicolan and Callagan.
Such report, however, is merely based on the narration of the military. No efforts were
undertaken to solicit petitioner’s version of the subject incident and no witnesses were WHEREFORE, we resolve to GRANT the Petition for Partial Review in G.R. No. 191805 and
questioned regarding the alleged abduction of petitioner. DENY the Petition for Review in G.R. No. 193160. The Decision of the Court of Appeals is
hereby AFFIRMED WITH MODIFICATION.
Respondent PDG Verzosa, as Chief of the PNP, is accountable because Section 24 of Republic
Act No. 6975, otherwise known as the "PNP Law," specifies the PNP as the governmental The case is dismissed with respect to respondents former President Gloria Macapagal-
office with the mandate "to investigate and prevent crimes, effect the arrest of criminal Arroyo, P/CSupt. Ameto G. Tolentino, and P/SSupt. Jude W. Santos, Calog, George Palacpac,
offenders, bring offenders to justice and assist in their prosecution." In this case, PDG Antonio Cruz, Aldwin Pasicolan and Vicent Callagan for lack of merit.
Verzosa failed to order the police to conduct the necessary investigation to unmask the
mystery surrounding petitioner’s abduction and disappearance. Instead, PDG Verzosa
This Court directs the Office of the Ombudsman (Ombudsman) and the Department of
disclaims accountability by merely stating that petitioner has no cause of action against him.
Justice (DOJ) to take the appropriate action with respect to any possible liability or liabilities,
Palpable, however, is the lack of any effort on the part of PDG Verzosa to effectively and
within their respective legal competence, that may have been incurred by respondents Gen.
aggressively investigate the violations of petitioner’s right to life, liberty and security by
Victor Ibrado, PDG. Jesus Verzosa, Lt. Gen. Delfin Bangit, Maj. Gen. Nestor Ochoa, Brig. Gen.
members of the 17th Infantry Battalion, 17th Infantry Division, Philippine Army. 116 (Emphasis
Remegio De Vera, 1st Lt. Ryan Matutina, and Lt. Col. Laurence Mina. The Ombudsman and
supplied.)
the DOJ are ordered to submit to this Court the results of their action within a period of six
months from receipt of this Decision.
42

In the event that herein respondents no longer occupy their respective posts, the directives the search for truth or impairs the vindication of a right. The deliberations of the
mandated in this Decision and in the Court of Appeals are enforceable against the incumbent Constitutional Commission also reveal that the intent of the framers is clear that presidential
officials holding the relevant positions. Failure to comply with the foregoing shall constitute immunity from suit is concurrent only with his tenure and not his term. (The term means the
contempt of court. time during which the officer may claim to hold the office as of right, and fixes the interval
after which the several incumbents shall succeed one another. The tenure represents the
SO ORDERED. term during which the incumbent actually holds office. The tenure may be shorter than the
term for reasons within or beyond the power of the incumbent.) Therefore, former Pres.
GMA cannot use such immunity to shield herself from judicial scrutiny that would assess
#7 IN THE MATTER OF THE PETITION FOR THE WRIT OF AMPARO AND WRIT OF HABEAS
whether, within the context of amparo proceedings, she was responsible or accountable for
DATA IN FAVOR OF NORIEL H. RODRIGUEZ
the abduction of Rodriguez.
GR NO. 191805
NOVEMBER 15, 2011

Facts: 2. Yes. As we explained in Rubrico v. Arroyo, command responsibility pertains to the


"responsibility of commanders for crimes committed by subordinate members of
Petitioner Noriel Rodriguez is a member of AlyansaDagitiMannalonIti Cagayan (Kagimungan), the armed forces or other persons subject to their control in international wars or
a peasant organization affiliated with KilusangMagbubukidngPilipinas (KMP). He claims that domestic conflict." Although originally used for ascertaining criminal complicity, the
the military tagged KMP as an enemy of the State under the OplanBantayLaya, making its command responsibility doctrine has also found application in civil cases for human
members targets of extrajudicial killings and enforced disappearances. rights abuses.This development in the use of command responsibility in civil
proceedings shows that the application of this doctrine has been liberally extended
Rodriguez was abducted by military men and was tortured repeatedly when he refused to even to cases not criminal in nature. Thus, it is our view that command
confess to his membership in the NPA. When released, he filed a Petition for the Writ of responsibility may likewise find application in proceedings seeking the privilege of
Amparo and and Petition for the Writ of Habeas Datawith Prayers for Protection Orders, the writ of amparo.
Inspection of Place, and Production of Documents and Personal Properties. The petition was
filed against former Pres. Arroyo, et al. The writs were granted but the CA dropped Pres Precisely in the case at bar, the doctrine of command responsibility may be used to
Arroyo as party-respondent, as she may not be sued in any case during her tenure of office or determine whether respondents are accountable for and have the duty to address the
actual incumbency. abduction of Rodriguez in order to enable the courts to devise remedial measures to protect
his rights. Clearly, nothing precludes this Court from applying the doctrine of command
Issue: responsibility in amparo proceedings to ascertain responsibility and accountability in
1. Whether former Pres GMA should be dropped as respondent on the basis of extrajudicial killings and enforced disappearances.
presidential immunity from suit
2. Whether the doctrine of command responsibility can be used in amparo and In other words, command responsibility may be loosely applied in amparo cases in order to
habeas data cases. identify those accountable individuals that have the power to effectively implement
3. Whether the president, as commander-in-chief of the military, can be held whatever processes an amparo court would issue. In such application, the amparo court does
responsible or accountable for extrajudicial killings and enforced disappearances. not impute criminal responsibility but merely pinpoint the superiors it considers to be in the
4. Whether Rodriguez has proven through substantial evidence that former President best position to protect the rights of the aggrieved party.Such identification of the
Arroyo is responsible or accountable for his abduction. responsible and accountable superiors may well be a preliminary determination of criminal
Held: liability which, of course, is still subject to further investigation by the appropriate
government agency.
1. No.It bears stressing that since there is no determination of administrative, civil or
criminal liability inamparo and habeas data proceedings, courts can only go as far
Thus, although there is no determination of criminal, civil or administrative liabilities, the
as ascertaining responsibility or accountability for the enforced disappearance or
doctrine of command responsibility may nevertheless be applied to ascertain responsibility
extrajudicial killing.
and accountability within these foregoing definitions.
As was held in the case of Estrada v Desierto, a non-sitting President does not enjoy
immunity from suit, even for acts committed during the latter’s tenure; that courts should 3. Yes.
look with disfavor upon the presidential privilege of immunity, especially when it impedes
43

To hold someone liable under the doctrine of command responsibility, the following
elements must obtain:

a. the existence of a superior-subordinate relationship between the accused as


superior and the perpetrator of the crime as his subordinate;

b. the superior knew or had reason to know that the crime was about to be or had
been committed; and

c. the superior failed to take the necessary and reasonable measures to prevent the
criminal acts or punish the perpetrators thereof.84

The president, being the commander-in-chief of all armed forces,necessarily possesses


control over the military that qualifies him as a superior within the purview of the command
responsibility doctrine.

4. No.Rodriguez anchors his argument on a general allegation that on the basis of the
"Melo Commission" and the "Alston Report," respondents in G.R. No. 191805
already had knowledge of and information on, and should have known that a
climate of enforced disappearances had been perpetrated on members of the
NPA.Without even attaching, or at the very least, quoting these reports, Rodriguez
contends that the Melo Report points to rogue military men as the perpetrators.
While the Alston Report states that there is a policy allowing enforced
disappearances and pins the blame on the President, we do not automatically
impute responsibility to former President Arroyo for each and every count of
forcible disappearance.Aside from Rodriguez’s general averments, there is no piece
of evidence that could establish her responsibility or accountability for his
abduction. Neither was there even a clear attempt to show that she should have
known about the violation of his right to life, liberty or security, or that she had
failed to investigate, punish or prevent it.
44

EN BANC the Acting Solicitor General in place of Solicitor General Devanadera who had been
appointed as the Secretary of Justice;3 that on March 5, 2010, President Arroyo designated
G.R. No. 191644 February 19, 2013 him also as the Acting Secretary of Justice vice Secretary Devanadera who had meanwhile
tendered her resignation in order to run for Congress representing a district in Quezon
Province in the May 2010 elections; that he then relinquished his position as the Government
DENNIS A.B. FUNA, Petitioner,
Corporate Counsel; and that pending the appointment of his successor, Agra continued to
vs.
perform his duties as the Acting Solicitor General.4
CTING SECRETARY OF JUSTICE ALBERTO C. AGRA, IN HIS OFFICIAL CONCURRENT
CAPACITIES AS ACTING SECRETARY OF THE DEPARTMENT OF JUSTICE AND AS ACTING
SOLICITOR GENERAL, EXECUTIVE SECRETARY LEANDRO R. MENDOZA, OFFICE OF THE Notwithstanding the conflict in the versions of the parties, the fact that Agra has admitted to
PRESIDENT, Respondents. holding the two offices concurrently in acting capacities is settled, which is sufficient for
purposes of resolving the constitutional question that petitioner raises herein.
DECISION
The Case
BERSAMIN, J.:
In Funa v. Ermita,5 the Court resolved a petition for certiorari, prohibition
and mandamus brought by herein petitioner assailing the constitutionality of the designation
Section 13, Article VII of the 1987 Constitution expressly prohibits the President, Vice-
of then Undersecretary of the Department of Transportation and Communications (DOTC)
President, the Members of the Cabinet, and their deputies or assistants from holding any
Maria Elena H. Bautista as concurrently the Officer-in-Charge of the Maritime Industry
other office or employment during their tenure unless otherwise provided in the
Authority. The petitioner has adopted here the arguments he advanced in Funa v. Ermita,
Constitution. Complementing the prohibition is Section 7, paragraph (2), Article IX-B of the
and he has rested his grounds of challenge mainly on the pronouncements in Civil Liberties
1987 Constitution, which bans any appointive official from holding any other office or
Union v. Executive Secretary6and Public Interest Center, Inc. v. Elma.7
employment in the Government or any subdivision, agency or instrumentality thereof,
including government-owned or controlled corporations or their subsidiaries, unless
otherwise allowed by law or the primary functions of his position. What may differentiate this challenge from those in the others is that the appointments
being hereby challenged were in acting or temporary capacities. Still, the petitioner submits
that the prohibition under Section 13, Article VII of the 1987 Constitution does not
These prohibitions under the Constitution are at the core of this special civil action
distinguish between an appointment or designation of a Member of the Cabinet in an acting
for certiorari and prohibition commenced on April 7, 2010 to assail the designation of
or temporary capacity, on the one hand, and one in a permanent capacity, on the other
respondent Hon. Alberto C. Agra, then the Acting Secretary of Justice, as concurrently the
hand; and that Acting Secretaries, being nonetheless Members of the Cabinet, are not
Acting Solicitor General.
exempt from the constitutional ban. He emphasizes that the position of the Solicitor General
is not an ex officio position in relation to the position of the Secretary of Justice, considering
Antecedents that the Office of the Solicitor General (OSG) is an independent and autonomous office
attached to the Department of Justice (DOJ).8 He insists that the fact that Agra was extended
The petitioner alleges that on March 1, 2010, President Gloria M. Macapagal-Arroyo an appointment as the Acting Solicitor General shows that he did not occupy that office in
appointed Agra as the Acting Secretary of Justice following the resignation of Secretary Agnes an ex officio capacity because an ex officio position does not require any further warrant or
VST Devanadera in order to vie for a congressional seat in Quezon Province; that on March 5, appointment.
2010, President Arroyo designated Agra as the Acting Solicitor General in a concurrent
capacity;1 that on April 7, 2010, the petitioner, in his capacity as a taxpayer, a concerned Respondents contend, in contrast, that Agra’s concurrent designations as the Acting
citizen and a lawyer, commenced this suit to challenge the constitutionality of Agra’s Secretary of Justice and Acting Solicitor General were only in a temporary capacity, the only
concurrent appointments or designations, claiming it to be prohibited under Section 13, effect of which was to confer additional duties to him. Thus, as the Acting Solicitor General
Article VII of the 1987 Constitution; that during the pendency of the suit, President Benigno and Acting Secretary of Justice, Agra was not "holding" both offices in the strict constitutional
S. Aquino III appointed Atty. Jose Anselmo I. Cadiz as the Solicitor General; and that Cadiz sense.9 They argue that an appointment, to be covered by the constitutional prohibition,
assumed as the Solicitor General and commenced his duties as such on August 5, 2010.2 must be regular and permanent, instead of a mere designation.

Agra renders a different version of the antecedents. He represents that on January 12, 2010, Respondents further contend that, even on the assumption that Agra’s concurrent
he was then the Government Corporate Counsel when President Arroyo designated him as designation constituted "holding of multiple offices," his continued service as the Acting
45

Solicitor General was akin to a hold-over; that upon Agra’s designation as the Acting have a personal and substantial interest in the case such that he has sustained, or will
Secretary of Justice, his term as the Acting Solicitor General expired in view of the sustain, direct injury as a result of its enforcement; (3) the question of constitutionality must
constitutional prohibition against holding of multiple offices by the Members of the Cabinet; be raised at the earliest opportunity; and (4) the issue of constitutionality must be the very lis
that under the principle of hold-over, Agra continued his service as the Acting Solicitor mota of the case.18
General "until his successor is elected and qualified"10 to "prevent a hiatus in the government
pending the time when a successor may be chosen and inducted into office;"11 and that Here, the OSG does not dispute the justiciability and ripeness for consideration and
during his continued service as the Acting Solicitor General, he did not receive any salaries resolution by the Court of the matter raised by the petitioner. Also, the locus standi of the
and emoluments from the OSG after becoming the Acting Secretary of Justice on March 5, petitioner as a taxpayer, a concerned citizen and a lawyer to bring a suit of this nature has
2010.12 already been settled in his favor in rulings by the Court on several other public law litigations
he brought. In Funa v. Villar,19 for one, the Court has held:
Respondents point out that the OSG’s independence and autonomy are defined by the
powers and functions conferred to that office by law, not by the person appointed to head To have legal standing, therefore, a suitor must show that he has sustained or will sustain a
such office;13 and that although the OSG is attached to the DOJ, the DOJ’s authority, control "direct injury" as a result of a government action, or have a "material interest" in the issue
and supervision over the OSG are limited only to budgetary purposes.14 affected by the challenged official act. However, the Court has time and again acted liberally
on the locus standi requirements and has accorded certain individuals, not otherwise
In his reply, petitioner counters that there was no "prevailing special circumstance" that directly injured, or with material interest affected, by a Government act, standing to sue
justified the non-application to Agra of Section 13, Article VII of the 1987 Constitution;15 that provided a constitutional issue of critical significance is at stake. The rule on locus standi is
the temporariness of the appointment or designation is not an excuse to disregard the after all a mere procedural technicality in relation to which the Court, in a catena of cases
constitutional ban against holding of multiple offices by the Members of the Cabinet;16 that involving a subject of transcendental import, has waived, or relaxed, thus allowing non-
Agra’s invocation of the principle of hold-over is misplaced for being predicated upon an traditional plaintiffs, such as concerned citizens, taxpayers, voters or legislators, to sue in
erroneous presentation of a material fact as to the time of his designation as the Acting the public interest, albeit they may not have been personally injured by the operation of a
Solicitor General and Acting Secretary of Justice; that Agra’s concurrent designations further law or any other government act. In David, the Court laid out the bare minimum norm
violated the Administrative Code of 1987 which mandates that the OSG shall be autonomous before the so-called "non-traditional suitors" may be extended standing to sue, thusly:
and independent.17
1.) For taxpayers, there must be a claim of illegal disbursement of public funds or
Issue that the tax measure is unconstitutional;

Did the designation of Agra as the Acting Secretary of Justice, concurrently with his position 2.) For voters, there must be a showing of obvious interest in the validity of the
of Acting Solicitor General, violate the constitutional prohibition against dual or multiple election law in question;
offices for the Members of the Cabinet and their deputies and assistants?
3.) For concerned citizens, there must be a showing that the issues raised are of
Ruling transcendental importance which must be settled early; and

The petition is meritorious. 4.) For legislators, there must be a claim that the official action complained of
infringes their prerogatives as legislators.
The designation of Agra as Acting Secretary of Justice concurrently with his position of Acting
Solicitor General was unconstitutional and void for being in violation of the constitutional This case before Us is of transcendental importance, since it obviously has "far-reaching
prohibition under Section 13, Article VII of the 1987 Constitution. implications," and there is a need to promulgate rules that will guide the bench, bar, and
the public in future analogous cases. We, thus, assume a liberal stance and allow petitioner
1. Requisites of judicial review not in issue to institute the instant petition.20 (Bold emphasis supplied)

The power of judicial review is subject to limitations, to wit: (1) there must be an actual case In Funa v. Ermita,21 the Court recognized the locus standi of the petitioner as a taxpayer, a
or controversy calling for the exercise of judicial power; (2) the person challenging the act concerned citizen and a lawyer because the issue raised therein involved a subject of
must have the standing to assail the validity of the subject act or issuance, that is, he must
46

transcendental importance whose resolution was necessary to promulgate rules to guide the subdivision, agency, or instrumentality thereof, including government-owned or controlled
Bench, Bar, and the public in similar cases. corporations or their subsidiaries. They shall strictly avoid conflict of interest in the conduct
of their office.
But, it is next posed, did not the intervening appointment of and assumption by Cadiz as the
Solicitor General during the pendency of this suit render this suit and the issue tendered A relevant and complementing provision is Section 7, paragraph (2), Article IX-B of the 1987
herein moot and academic? Constitution, to wit:

A moot and academic case is one that ceases to present a justiciable controversy by virtue of Section 7. x x x
supervening events, so that a declaration thereon would be of no practical use or
value.22 Although the controversy could have ceased due to the intervening appointment of Unless otherwise allowed by law or the primary functions of his position, no appointive
and assumption by Cadiz as the Solicitor General during the pendency of this suit, and such official shall hold any other office or employment in the Government or any subdivision,
cessation of the controversy seemingly rendered moot and academic the resolution of the agency or instrumentality thereof, including government-owned or controlled corporations
issue of the constitutionality of the concurrent holding of the two positions by Agra, the or their subsidiaries.
Court should still go forward and resolve the issue and not abstain from exercising its power
of judicial review because this case comes under several of the well-recognized exceptions
The differentiation of the two constitutional provisions was well stated in Funa v. Ermita,25 a
established in jurisprudence. Verily, the Court did not desist from resolving an issue that a
case in which the petitioner herein also assailed the designation of DOTC Undersecretary as
supervening event meanwhile rendered moot and academic if any of the following
concurrent Officer-in-Charge of the Maritime Industry Authority, with the Court reiterating
recognized exceptions obtained, namely: (1) there was a grave violation of the Constitution;
its pronouncement in Civil Liberties Union v. The Executive Secretary26 on the intent of the
(2) the case involved a situation of exceptional character and was of paramount public
Framers behind these provisions of the Constitution, viz:
interest; (3) the constitutional issue raised required the formulation of controlling principles
to guide the Bench, the Bar and the public; and (4) the case was capable of repetition, yet
evading review.23 Thus, while all other appointive officials in the civil service are allowed to hold other office or
employment in the government during their tenure when such is allowed by law or by the
primary functions of their positions, members of the Cabinet, their deputies and assistants
It is the same here. The constitutionality of the concurrent holding by Agra of the two
may do so only when expressly authorized by the Constitution itself. In other words, Section
positions in the Cabinet, albeit in acting capacities, was an issue that comes under all the
7, Article IX-B is meant to lay down the general rule applicable to all elective and
recognized exceptions. The issue involves a probable violation of the Constitution, and
appointive public officials and employees, while Section 13, Article VII is meant to be the
relates to a situation of exceptional character and of paramount public interest by reason of
exception applicable only to the President, the Vice-President, Members of the Cabinet,
its transcendental importance to the people. The resolution of the issue will also be of the
their deputies and assistants.
greatest value to the Bench and the Bar in view of the broad powers wielded through said
positions. The situation further calls for the review because the situation is capable of
repetition, yet evading review.24 In other words, many important and practical benefits are Since the evident purpose of the framers of the 1987 Constitution is to impose a stricter
still to be gained were the Court to proceed to the ultimate resolution of the constitutional prohibition on the President, Vice-President, members of the Cabinet, their deputies and
issue posed. assistants with respect to holding multiple offices or employment in the government during
their tenure, the exception to this prohibition must be read with equal severity. On its face,
the language of Section 13, Article VII is prohibitory so that it must be understood as
2. Unconstitutionality of Agra’s concurrent designation as Acting Secretary of Justice
intended to be a positive and unequivocal negation of the privilege of holding multiple
and Acting Solicitor General
government offices or employment. Verily, wherever the language used in the constitution is
prohibitory, it is to be understood as intended to be a positive and unequivocal negation. The
At the center of the controversy is the correct application of Section 13, Article VII of the phrase "unless otherwise provided in this Constitution" must be given a literal
1987 Constitution, viz: interpretation to refer only to those particular instances cited in the Constitution itself, to
wit: the Vice-President being appointed as a member of the Cabinet under Section 3, par. (2),
Section 13. The President, Vice-President, the Members of the Cabinet, and their deputies or Article VII; or acting as President in those instances provided under Section 7, pars. (2) and
assistants shall not, unless otherwise provided in this Constitution, hold any other office or (3), Article VII; and, the Secretary of Justice being ex-officio member of the Judicial and Bar
employment during their tenure. They shall not, during said tenure, directly or indirectly Council by virtue of Section 8 (1), Article VIII. (Bold emphasis supplied.)
practice any other profession, participate in any business, or be financially interested in any
contract with, or in any franchise, or special privilege granted by the Government or any
47

Being designated as the Acting Secretary of Justice concurrently with his position of Acting Agra cannot liberally apply in his favor the broad exceptions provided in Section 7, paragraph
Solicitor General, therefore, Agra was undoubtedly covered by Section 13, Article VII, supra, 2, Article IX-B of the Constitution ("Unless otherwise allowed by law or the primary functions
whose text and spirit were too clear to be differently read. Hence, Agra could not validly hold of his position") to justify his designation as Acting Secretary of Justice concurrently with his
any other office or employment during his tenure as the Acting Solicitor General, because the designation as Acting Solicitor General, or vice versa. Thus, the Court has said –
Constitution has not otherwise so provided.27
[T]he qualifying phrase "unless otherwise provided in this Constitution" in Section 13, Article
It was of no moment that Agra’s designation was in an acting or temporary capacity. The text VII cannot possibly refer to the broad exceptions provided under Section 7, Article IX-B of the
of Section 13, supra, plainly indicates that the intent of the Framers of the Constitution was 1987 Constitution. To construe said qualifying phrase as respondents would have us do,
to impose a stricter prohibition on the President and the Members of his Cabinet in so far as would render nugatory and meaningless the manifest intent and purpose of the framers of
holding other offices or employments in the Government or in government-owned or the Constitution to impose a stricter prohibition on the President, Vice-President, Members
government controlled-corporations was concerned.28 In this regard, to hold an office means of the Cabinet, their deputies and assistants with respect to holding other offices or
to possess or to occupy the office, or to be in possession and administration of the office, employment in the government during their tenure. Respondents’ interpretation that
which implies nothing less than the actual discharge of the functions and duties of the Section 13 of Article VII admits of the exceptions found in Section 7, par. (2) of Article IX-B
office.29 Indeed, in the language of Section 13 itself, supra, the Constitution makes no would obliterate the distinction so carefully set by the framers of the Constitution as to when
reference to the nature of the appointment or designation. The prohibition against dual or the highranking officials of the Executive Branch from the President to Assistant Secretary, on
multiple offices being held by one official must be construed as to apply to all appointments the one hand, and the generality of civil servants from the rank immediately below Assistant
or designations, whether permanent or temporary, for it is without question that the avowed Secretary downwards, on the other, may hold any other office or position in the government
objective of Section 13, supra, is to prevent the concentration of powers in the Executive during their tenure.35
Department officials, specifically the President, the Vice-President, the Members of the
Cabinet and their deputies and assistants.30 To construe differently is to "open the veritable To underscore the obvious, it is not sufficient for Agra to show that his holding of the other
floodgates of circumvention of an important constitutional disqualification of officials in the office was "allowed by law or the primary functions of his position." To claim the exemption
Executive Department and of limitations on the President’s power of appointment in the of his concurrent designations from the coverage of the stricter prohibition under Section
guise of temporary designations of Cabinet Members, undersecretaries and assistant 13, supra, he needed to establish herein that his concurrent designation was expressly
secretaries as officers-in-charge of government agencies, instrumentalities, or government- allowed by the Constitution. But, alas, he did not do so.
owned or controlled corporations."31
To be sure, Agra’s concurrent designations as Acting Secretary of Justice and Acting Solicitor
According to Public Interest Center, Inc. v. Elma,32 the only two exceptions against the holding General did not come within the definition of an ex officio capacity. Had either of his
of multiple offices are: (1) those provided for under the Constitution, such as Section 3, concurrent designations been in an ex officio capacity in relation to the other, the Court
Article VII, authorizing the Vice President to become a member of the Cabinet; and (2) posts might now be ruling in his favor.
occupied by Executive officials specified in Section 13, Article VII without additional
compensation in ex officio capacities as provided by law and as required by the primary
The import of an ex officio capacity has been fittingly explained in Civil Liberties Union v.
functions of the officials’ offices. In this regard, the decision in Public Interest Center, Inc. v.
Executive Secretary,36 as follows:
Elma adverted to the resolution issued on August 1, 1991 in Civil Liberties Union v. The
Executive Secretary, whereby the Court held that the phrase "the Members of the Cabinet,
and their deputies or assistants" found in Section 13, supra, referred only to the heads of the x x x. The term ex officio means "from office; by virtue of office." It refers to an "authority
various executive departments, their undersecretaries and assistant secretaries, and did not derived from official character merely, not expressly conferred upon the individual character,
extend to other public officials given the rank of Secretary, Undersecretary or Assistant but rather annexed to the official position." Ex officio likewise denotes an "act done in an
Secretary.33 Hence, in Public Interest Center, Inc. v. Elma, the Court opined that the official character, or as a consequence of office, and without any other appointment or
prohibition under Section 13 did not cover Elma, a Presidential Assistant with the rank of authority other than that conferred by the office." An ex officio member of a board is one
Undersecretary.34 who is a member by virtue of his title to a certain office, and without further warrant or
appointment. x x x.
It is equally remarkable, therefore, that Agra’s designation as the Acting Secretary of Justice
was not in an ex officio capacity, by which he would have been validly authorized to The ex officio position being actually and in legal contemplation part of the principal office, it
concurrently hold the two positions due to the holding of one office being the consequence follows that the official concerned has no right to receive additional compensation for his
of holding the other. Being included in the stricter prohibition embodied in Section 13, supra, services in the said position. The reason is that these services are already paid for and
covered by the compensation attached to his principal office. x x x.
48

Under the Administrative Code of 1987, the DOJ is mandated to "provide the government Court, the Court of Appeals, and all other courts or tribunals in all civil actions and
with a principal law agency which shall be both its legal counsel and prosecution arm; special proceedings in which the Government or any officer thereof in his official
administer the criminal justice system in accordance with the accepted processes thereof capacity is a party.
consisting in the investigation of the crimes, prosecution of offenders and administration of
the correctional system; implement the laws on the admission and stay of aliens, citizenship, 2. Investigate, initiate court action, or in any manner proceed against any person,
land titling system, and settlement of land problems involving small landowners and corporation or firm for the enforcement of any contract, bond, guarantee,
members of indigenous cultural minorities; and provide free legal services to indigent mortgage, pledge or other collateral executed in favor of the Government. Where
members of the society."37 The DOJ’s specific powers and functions are as follows: proceedings are to be conducted outside of the Philippines the Solicitor General
may employ counsel to assist in the discharge of the aforementioned
(1) Act as principal law agency of the government and as legal counsel and responsibilities.
representative thereof, whenever so required;
3. Appear in any court in any action involving the validity of any treaty, law,
(2) Investigate the commission of crimes, prosecute offenders and administer the executive order or proclamation, rule or regulation when in his judgment his
probation and correction system; intervention is necessary or when requested by the Court.

(3) Extend free legal assistance/representation to indigents and poor litigants in 4. Appear in all proceedings involving the acquisition or loss of Philippine
criminal cases and non-commercial civil disputes; citizenship.

(4) Preserve the integrity of land titles through proper registration; 5. Represent the Government in all land registration and related proceedings.
Institute actions for the reversion to the Government of lands of the public domain
(5) Investigate and arbitrate untitled land disputes involving small landowners and and improvements thereon as well as lands held in violation of the Constitution.
members of indigenous cultural communities;
6. Prepare, upon request of the President or other proper officer of the National
(6) Provide immigration and naturalization regulatory services and implement the Government, rules and guidelines for government entities governing the
laws governing citizenship and the admission and stay of aliens; preparation of contracts, making investments, undertaking of transactions, and
drafting of forms or other writings needed for official use, with the end in view of
facilitating their enforcement and insuring that they are entered into or prepared
(7) Provide legal services to the national government and its functionaries,
conformably with law and for the best interests of the public.
including government-owned or controlled corporations and their subsidiaries; and

7. Deputize, whenever in the opinion of the Solicitor General the public interest
(8) Perform such other functions as may be provided by law.38
requires, any provincial or city fiscal to assist him in the performance of any
function or discharge of any duty incumbent upon him, within the jurisdiction of
On the other hand, the Administrative Code of 1987 confers upon the Office of the Solicitor the aforesaid provincial or city fiscal. When so deputized, the fiscal shall be under
General the following powers and functions, to wit: the control and supervision of the Solicitor General with regard to the conduct of
the proceedings assigned to the fiscal, and he may be required to render reports or
The Office of the Solicitor General shall represent the Government of the Philippines, its furnish information regarding the assignment.
agencies and instrumentalities and its officials and agents in any litigation, proceeding,
investigation or matter requiring the services of lawyers. When authorized by the President 8. Deputize legal officers of government departments, bureaus, agencies and
or head of the office concerned, it shall also represent government owned or controlled offices to assist the Solicitor General and appear or represent the Government in
corporations. The Office of the Solicitor General shall discharge duties requiring the services cased involving their respective offices, brought before the courts and exercise
of lawyers. It shall have the following specific powers and functions: supervision and control over such legal Officers with respect to such cases.

1. Represent the Government in the Supreme Court and the Court of Appeals in all 9. Call on any department, bureau, office, agency or instrumentality of the
criminal proceedings; represent the Government and its officers in the Supreme Government for such service, assistance and cooperation as may be necessary in
49

fulfilling its functions and responsibilities and for this purpose enlist the services of knowledge and expertise, particularly at this stage of our national and economic
any government official or employee in the pursuit of his tasks. development, far outweigh the benefits, if any, that may be gained from a department head
spreading himself too thin and taking in more than what he can handle.
10. Departments, bureaus, agencies, offices, instrumentalities and corporations to
whom the Office of the Solicitor General renders legal services are authorized to It is not amiss to observe, lastly, that assuming that Agra, as the Acting Solicitor General, was
disburse funds from their sundry operating and other funds for the latter Office. not covered by the stricter prohibition under Section 13, supra, due to such position being
For this purpose, the Solicitor General and his staff are specifically authorized to merely vested with a cabinet rank under Section 3, Republic Act No. 9417, he nonetheless
receive allowances as may be provided by the Government offices, remained covered by the general prohibition under Section 7, supra. Hence, his concurrent
instrumentalities and corporations concerned, in addition to their regular designations were still subject to the conditions under the latter constitutional provision. In
compensation. this regard, the Court aptly pointed out in Public Interest Center, Inc. v. Elma:46

11. Represent, upon the instructions of the President, the Republic of the The general rule contained in Article IX-B of the 1987 Constitution permits an appointive
Philippines in international litigations, negotiations or conferences where the legal official to hold more than one office only if "allowed by law or by the primary functions of his
position of the Republic must be defended or presented. position." In the case of Quimson v. Ozaeta, this Court ruled that, "[t]here is no legal
objection to a government official occupying two government offices and performing the
12. Act and represent the Republic and/or the people before any court, tribunal, functions of both as long as there is no incompatibility." The crucial test in determining
body or commission in any matter, action or proceedings which, in his opinion whether incompatibility exists between two offices was laid out in People v. Green - whether
affects the welfare of the people as the ends of justice may require; and one office is subordinate to the other, in the sense that one office has the right to interfere
with the other.
13. Perform such other functions as may be provided by law.39
[I]ncompatibility between two offices, is an inconsistency in the functions of the two; x x x
Where one office is not subordinate to the other, nor the relations of the one to the other
The foregoing provisions of the applicable laws show that one position was not derived from
such as are inconsistent and repugnant, there is not that incompatibility from which the law
the other. Indeed, the powers and functions of the OSG are neither required by the primary
declares that the acceptance of the one is the vacation of the other. The force of the word, in
functions nor included by the powers of the DOJ, and vice versa. The OSG, while attached to
its application to this matter is, that from the nature and relations to each other, of the two
the DOJ,40 is not a constituent unit of the latter,41 as, in fact, the Administrative Code of
places, they ought not to be held by the same person, from the contrariety and antagonism
1987 decrees that the OSG is independent and autonomous.42 With the enactment of
which would result in the attempt by one person to faithfully and impartially discharge the
Republic Act No. 9417,43 the Solicitor General is now vested with a cabinet rank, and has the
duties of one, toward the incumbent of the other. X x x The offices must subordinate, one
same qualifications for appointment, rank, prerogatives, salaries, allowances, benefits and
[over] the other, and they must, per se, have the right to interfere, one with the other,
privileges as those of the Presiding Justice of the Court of Appeals. 44
before they are incompatible at common law. x x x.

Moreover, the magnitude of the scope of work of the Solicitor General, if added to the
While Section 7, Article IX-B of the 1987 Constitution applies in general to all elective and
equally demanding tasks of the Secretary of Justice, is obviously too much for any one official
appointive officials, Section 13, Article VII, thereof applies in particular to Cabinet secretaries,
to bear. Apart from the sure peril of political pressure, the concurrent holding of the two
undersecretaries and assistant secretaries. In the Resolution in Civil Liberties Union v.
positions, even if they are not entirely incompatible, may affect sound government
Executive Secretary, this Court already clarified the scope of the prohibition provided in
operations and the proper performance of duties. Heed should be paid to what the Court has
Section 13, Article VII of the 1987 Constitution. Citing the case of US v. Mouat, it specifically
pointedly observed in Civil Liberties Union v. Executive Secretary: 45
identified the persons who are affected by this prohibition as secretaries, undersecretaries
and assistant secretaries; and categorically excluded public officers who merely have the
Being head of an executive department is no mean job. It is more than a full-time job, rank of secretary, undersecretary or assistant secretary.
requiring full attention, specialized knowledge, skills and expertise. If maximum benefits are
to be derived from a department head’s ability and expertise, he should be allowed to attend
Another point of clarification raised by the Solicitor General refers to the persons affected by
to his duties and responsibilities without the distraction of other governmental offices or
the constitutional prohibition. The persons cited in the constitutional provision are the
employment. He should be precluded from dissipating his efforts, attention and energy
"Members of the Cabinet, their deputies and assistants." These terms must be given their
among too many positions of responsibility, which may result in haphazardness and
common and general acceptation as referring to the heads of the executive departments,
inefficiency. Surely the advantages to be derived from this concentration of attention,
their undersecretaries and assistant secretaries. Public officials given the rank equivalent to a
50

Secretary, Undersecretary, or Assistant Secretary are not covered by the prohibition, nor is the virtue of actual services rendered in the questioned positions may therefore be retained by
Solicitor General affected thereby. (Italics supplied). them.

It is clear from the foregoing that the strict prohibition under Section 13, Article VII of the A de facto officer is one who derives his appointment from one having colorable authority to
1987 Constitution is not applicable to the PCGG Chairman nor to the CPLC, as neither of them appoint, if the office is an appointive office, and whose appointment is valid on its face.51 He
is a secretary, undersecretary, nor an assistant secretary, even if the former may have the may also be one who is in possession of an office, and is discharging its duties under color of
same rank as the latter positions. authority, by which is meant authority derived from an appointment, however irregular or
informal, so that the incumbent is not a mere volunteer.52 Consequently, the acts of the de
It must be emphasized, however, that despite the non-applicability of Section 13, Article VII facto officer are just as valid for all purposes as those of a de jure officer, in so far as the
of the 1987 Constitution to respondent Elma, he remains covered by the general prohibition public or third persons who are interested therein are concerned. 53
under Section 7, Article IX-B and his appointments must still comply with the standard of
compatibility of officers laid down therein; failing which, his appointments are hereby In order to be clear, therefore, the Court holds that all official actions of Agra as a de
pronounced in violation of the Constitution.47 facto Acting Secretary of Justice, assuming that was his later designation, were presumed
valid, binding and effective as if he was the officer legally appointed and qualified for the
Clearly, the primary functions of the Office of the Solicitor General are not related or office. 54 This clarification is necessary in order to protect the sanctity of the dealings by the
necessary to the primary functions of the Department of Justice. Considering that the nature public with persons whose ostensible authority emanates from the State. 55 Agra's official
and duties of the two offices are such as to render it improper, from considerations of public actions covered by this claritlcation extend to but are not limited to the promulgation of
policy, for one person to retain both,48 an incompatibility between the offices exists, further resolutions on petitions for review filed in the Department of Justice, and the issuance of
warranting the declaration of Agra’s designation as the Acting Secretary of Justice, department orders, memoranda and circulars relative to the prosecution of criminal cases.
concurrently with his designation as the Acting Solicitor General, to be void for being in
violation of the express provisions of the Constitution. WHEREFORE, the Comi GRANTS the petition for certiorari and prohibition; ANNULS AND
VOIDS the designation of Hon. Alberto C. Agra as the Acting Secretary of Justice in a
3. Effect of declaration of unconstitutionality of Agra’s concurrent appointment; concurrent capacity with his position as the Acting Solicitor General for being
the de facto officer doctrine unconstitutional and violative of Section 13, Article VII of the 1987 Constitution;
and DECLARES that l-Ion. Alberto C. Agra was a de facto officer during his tenure as Acting
Secretary of Justice.
In view of the application of the stricter prohibition under Section 13, supra, Agra did not
validly hold the position of Acting Secretary of Justice concurrently with his holding of the
position of Acting Solicitor General. Accordingly, he was not to be considered as a de No pronouncement on costs of suit.
jure officer for the entire period of his tenure as the Acting Secretary of Justice. A de
jure officer is one who is deemed, in all respects, legally appointed and qualified and whose SO ORDERED.
term of office has not expired.49
Funa vs Agra
That notwithstanding, Agra was a de facto officer during his tenure as Acting Secretary of G.R. No. 191644 February 19, 2013
Justice. In Civil Liberties Union v. Executive Secretary,50 the Court said:
Facts:
During their tenure in the questioned positions, respondents may be considered de
facto officers and as such entitled to emoluments for actual services rendered. It has been The petitioner alleges that on March 1, 2010, President Gloria M. Macapagal Arroyo
held that "in cases where there is no de jure, officer, a de facto officer, who, in good faith has appointed Agra as the Acting Secretary of Justice following the resignation of Secretary Agnes
had possession of the office and has discharged the duties pertaining thereto, is legally VST Devanadera in order to vie for a congressional seat in Quezon Province; that on March 5,
entitled to the emoluments of the office, and may in an appropriate action recover the 2010, President Arroyo designated Agra as the Acting Solicitor General in a concurrent
salary, fees and other compensations attached to the office. This doctrine is, undoubtedly, capacity; that on April 7, 2010, the petitioner, in his capacity as a taxpayer, a concerned
supported on equitable grounds since it seems unjust that the public should benefit by the citizen and a lawyer, commenced this suit to challenge the constitutionality of Agra’s
services of an officer de facto and then be freed from all liability to pay any one for such concurrent appointments or designations, claiming it to be prohibited under Section 13,
services. Any per diem, allowances or other emoluments received by the respondents by Article VII of the 1987 Constitution; that during the pendency of the suit, President Benigno
51

S. Aquino III appointed Atty. Jose Anselmo I. Cadiz as the Solicitor General; and that Cadiz It was of no moment that Agra’s designation was in an acting or temporary capacity. The text
assumed as the Solicitor General and commenced his duties as such on August 5, 2010. Agra of Section 13, supra, plainly indicates that the intent of the Framers of the Constitution was
renders a different version of the antecedents. He represents that on January 12, 2010, he to impose a stricter prohibition on the President and the Members of his Cabinet in so far as
was then the Government Corporate Counsel when President Arroyo designated him as the holding other offices or employments in the Government or in government-owned or
Acting Solicitor General in place of Solicitor General Devanadera who had been appointed as government controlled-corporations was concerned. In this regard, to hold an office means
the Secretary of Justice; that on March 5, 2010, President Arroyo designated him also as the to possess or to occupy the office, or to be in possession and administration of the office,
Acting Secretary of Justice vice Secretary Devanadera who had meanwhile tendered her which implies nothing less than the actual discharge of the functions and duties of the office.
resignation in order to run for Congress representing a district in Quezon Province in the May Indeed, in the language of Section 13 itself, supra, the Constitution makes no reference to
2010 elections; that he then relinquished his position as the Government Corporate Counsel; the nature of the appointment or designation. The prohibition against dual or multiple offices
and that pending the appointment of his successor, Agra continued to perform his duties as being held by one official must be construed as to apply to all appointments or designations,
the Acting Solicitor General. Notwithstanding the conflict in the versions of the parties, the whether permanent or temporary, for it is without question that the avowed objective of
fact that Agra has admitted to holding the two offices concurrently in acting capacities is Section 13, supra, is to prevent the concentration of powers in the Executive Department
settled, which is sufficient for purposes of resolving the constitutional question that officials, specifically the President, the Vice-President, the Members of the Cabinet and their
petitioner raises herein. deputies and assistants. To construe differently is to “open the veritable floodgates of
circumvention of an important constitutional disqualification of officials in the Executive
Issue: Whether or not Agra’s holding of concurrent position is unconstitutional. Department and of limitations on the Presidents power of appointment in the guise of
temporary designations of Cabinet Members, undersecretaries and assistant secretaries as
officers-in-charge of government agencies, instrumentalities, or government-owned or
Held:
controlled corporations.

Yes. At the center of the controversy is the correct application of Section 13, Article VII of the
It is not amiss to observe, lastly, that assuming that Agra, as the Acting Solicitor General, was
1987 Constitution, viz:
not covered by the stricter prohibition under Section 13, supra, due to such position being
merely vested with a cabinet rank under Section 3, Republic Act No. 9417, he nonetheless
Section 13. The President, Vice-President, the Members of the Cabinet, and their deputies or remained covered by the general prohibition under Section 7, supra. Hence, his concurrent
assistants shall not, unless otherwise provided in this Constitution, hold any other office or designations were still subject to the conditions under the latter constitutional provision. In
employment during their tenure. They shall not, during said tenure, directly or indirectly this regard, the Court aptly pointed out in Public Interest Center, Inc. v. Elma:
practice any other profession, participate in any business, or be financially interested in any
contract with, or in any franchise, or special privilege granted by the Government or any
The general rule contained in Article IX-B of the 1987 Constitution permits an appointive
subdivision, agency, or instrumentality thereof, including government-owned or controlled
official to hold more than one office only if “allowed by law or by the primary functions of his
corporations or their subsidiaries. They shall strictly avoid conflict of interest in the conduct of
position.” In the case of Quimson v. Ozaeta, this Court ruled that, “[t]here is no legal
their office.
objection to a government official occupying two government offices and performing the
functions of both as long as there is no incompatibility.” The crucial test in determining
A relevant and complementing provision is Section 7, paragraph (2), Article IX-B of the 1987 whether incompatibility exists between two offices was laid out in People v. Green – whether
Constitution, to wit: one office is subordinate to the other, in the sense that one office has the right to interfere
with the other.
Section 7. x x x Unless otherwise allowed by law or the primary functions of his position, no
appointive official shall hold any other office or employment in the Government or any 1.) FUNA vs. AGRA
subdivision, agency or instrumentality thereof, including government-owned or controlled
corporations or their subsidiaries.
FACTS:

Being designated as the Acting Secretary of Justice concurrently with his position of Acting
The petitioner alleges that on March 1, 2010, President Gloria M. Macapagal-Arroyo
Solicitor General, therefore, Agra was undoubtedly covered by Section 13, Article VII, supra,
appointed Agra as the Acting Secretary of Justice following the resignation of Secretary Agnes
whose text and spirit were too clear to be differently read. Hence, Agra could not validly hold
VST Devanadera in order to vie for a congressional seat in Quezon Province; that on March 5,
any other office or employment during his tenure as the Acting Solicitor General, because the
2010, President Arroyo designated Agra as the Acting Solicitor General in a concurrent
Constitution has not otherwise so provided.
capacity; that on April 7, 2010, the petitioner, in his capacity as a taxpayer, a concerned
52

citizen and a lawyer, commenced this suit to challenge the constitutionality of Agra’s It was of no moment that Agra’s designation was in an acting or temporary capacity. The text
concurrent appointments or designations, claiming it to be prohibited under Section 13, of Section 13, supra, plainly indicates that the intent of the Framers of the Constitution was
Article VII of the 1987 Constitution; that during the pendency of the suit, President Benigno to impose a stricter prohibition on the President and the Members of his Cabinet in so far as
S. Aquino III appointed Atty. Jose Anselmo I. Cadiz as the Solicitor General; and that Cadiz
holding other offices or employments in the Government or in government-owned or
assumed as the Solicitor General and commenced his duties as such on August 5, 2010.
government controlled-corporations was concerned. In this regard, to hold an office means
to possess or to occupy the office, or to be in possession and administration of the office,
Agra renders a different version of the antecedents. He represents that on January 12, 2010,
which implies nothing less than the actual discharge of the functions and duties of the office.
he was then the Government Corporate Counsel when President Arroyo designated him as
Indeed, in the language of Section 13 itself, supra, the Constitution makes no reference to
the Acting Solicitor General in place of Solicitor General Devanadera who had been
the nature of the appointment or designation. The prohibition against dual or multiple offices
appointed as the Secretary of Justice; that on March 5, 2010, President Arroyo designated
being held by one official must be construed as to apply to all appointments or designations,
him also as the Acting Secretary of Justice vice Secretary Devanadera who had meanwhile
whether permanent or temporary, for it is without question that the avowed objective of
tendered her resignation in order to run for Congress representing a district in Quezon
Section 13, supra, is to prevent the concentration of powers in the Executive Department
Province in the May 2010 elections; that he then relinquished his position as the Government
officials, specifically the President, the Vice-President, the Members of the Cabinet and their
Corporate Counsel; and that pending the appointment of his successor, Agra continued to
deputies and assistants.
perform his duties as the Acting Solicitor General.
The only two exceptions against the holding of multiple offices are: (1) those provided for
Notwithstanding the conflict in the versions of the parties, the fact that Agra has admitted to
under the Constitution, such as Section 3, Article VII, authorizing the Vice President to
holding the two offices concurrently in acting capacities is settled, which is sufficient for
become a member of the Cabinet; and (2) posts occupied by Executive officials specified in
purposes of resolving the constitutional question that petitioner raises herein.
Section 13, Article VII without additional compensation in ex officio capacities as provided by
ISSUE: WON the designation of Agra as Acting Secretary of Justice and Acting Solicitor law and as required by the primary functions of the officials’ offices. in Civil Liberties Union
General is unconstitutional v. The Executive Secretary, whereby the Court held that the phrase "the Members of the
Cabinet, and their deputies or assistants" found in Section 13, supra, referred only to the
HELD: heads of the various executive departments, their undersecretaries and assistant secretaries,
and did not extend to other public officials given the rank of Secretary, Undersecretary or
At the center of the controversy is the correct application of Section 13, Article VII of the Assistant Secretary.
1987 Constitution, viz:
It is equally remarkable, therefore, that Agra’s designation as the Acting Secretary of Justice
Section 13. The President, Vice-President, the Members of the Cabinet, and their deputies or was not in an ex officio capacity, by which he would have been validly authorized to
assistants shall not, unless otherwise provided in this Constitution, hold any other office or concurrently hold the two positions due to the holding of one office being the consequence
employment during their tenure. They shall not, during said tenure, directly or indirectly of holding the other. Being included in the stricter prohibition embodied in Section 13, supra,
practice any other profession, participate in any business, or be financially interested in any Agra cannot liberally apply in his favor the broad exceptions provided in Section 7, paragraph
contract with, or in any franchise, or special privilege granted by the Government or any 2, Article IX-B of the Constitution ("Unless otherwise allowed by law or the primary functions
subdivision, agency, or instrumentality thereof, including government-owned or controlled of his position") to justify his designation as Acting Secretary of Justice concurrently with his
corporations or their subsidiaries. They shall strictly avoid conflict of interest in the conduct designation as Acting Solicitor General, or vice versa.
of their office.
To underscore the obvious, it is not sufficient for Agra to show that his holding of the other
Being designated as the Acting Secretary of Justice concurrently with his position of Acting office was "allowed by law or the primary functions of his position." To claim the exemption
Solicitor General, therefore, Agra was undoubtedly covered by Section 13, Article VII, supra, of his concurrent designations from the coverage of the stricter prohibition under Section 13,
whose text and spirit were too clear to be differently read. Hence, Agra could not validly hold supra, he needed to establish herein that his concurrent designation was expressly allowed
any other office or employment during his tenure as the Acting Solicitor General, because the by the Constitution. But, alas, he did not do so.
Constitution has not otherwise so provided.
53

To be sure, Agra’s concurrent designations as Acting Secretary of Justice and Acting Solicitor It must be emphasized, however, that despite the non-applicability of Section 13, Article VII
General did not come within the definition of an ex officio capacity. Had either of his of the 1987 Constitution to respondent Elma, he remains covered by the general prohibition
concurrent designations been in an ex officio capacity in relation to the other, the Court under Section 7, Article IX-B and his appointments must still comply with the standard of
might now be ruling in his favor. compatibility of officers laid down therein; failing which, his appointments are hereby
pronounced in violation of the Constitution.
The import of an ex officio capacity has been fittingly explained in Civil Liberties Union v.
Executive Secretary as follows: Clearly, the primary functions of the Office of the Solicitor General are not related or
necessary to the primary functions of the Department of Justice. Considering that the nature
x x x. The term ex officio means "from office; by virtue of office." It refers to an "authority and duties of the two offices are such as to render it improper, from considerations of public
derived from official character merely, not expressly conferred upon the individual character, policy, for one person to retain both an incompatibility between the offices exists, further
but rather annexed to the official position." Ex officio likewise denotes an "act done in an warranting the declaration of Agra’s designation as the Acting Secretary of Justice,
official character, or as a consequence of office, and without any other appointment or concurrently with his designation as the Acting Solicitor General, to be void for being in
authority other than that conferred by the office." An ex officio member of a board is one violation of the express provisions of the Constitution.
who is a member by virtue of his title to a certain office, and without further warrant or
appointment. The ex officio position being actually and in legal contemplation part of the
principal office, it follows that the official concerned has no right to receive additional
compensation for his services in the said position. The reason is that these services are
already paid for and covered by the compensation attached to his principal office.

While Section 7, Article IX-B of the 1987 Constitution applies in general to all elective and
appointive officials, Section 13, Article VII, thereof applies in particular to Cabinet secretaries,
undersecretaries and assistant secretaries. In the Resolution in Civil Liberties Union v.
Executive Secretary, this Court already clarified the scope of the prohibition provided in
Section 13, Article VII of the 1987 Constitution. Citing the case of US v. Mouat, it specifically
identified the persons who are affected by this prohibition as secretaries, undersecretaries
and assistant secretaries; and categorically excluded public officers who merely have the
rank of secretary, undersecretary or assistant secretary.

Another point of clarification raised by the Solicitor General refers to the persons affected by
the constitutional prohibition. The persons cited in the constitutional provision are the
"Members of the Cabinet, their deputies and assistants." These terms must be given their
common and general acceptation as referring to the heads of the executive departments,
their undersecretaries and assistant secretaries. Public officials given the rank equivalent to a
Secretary, Undersecretary, or Assistant Secretary are not covered by the prohibition, nor is
the Solicitor General affected thereby. (Italics supplied).

It is clear from the foregoing that the strict prohibition under Section 13, Article VII of the
1987 Constitution is not applicable to the PCGG Chairman nor to the CPLC, as neither of them
is a secretary, undersecretary, nor an assistant secretary, even if the former may have the
same rank as the latter positions.
54

EN BANC Joseph H. Durano Tourism 23 August 2004


Michael T. Defensor Environment and Natural Resources 23 August 2004
G.R. No. 164978 October 13, 2005
The appointment papers are uniformly worded as follows:
AQUILINO Q. PIMENTEL, JR., EDGARDO J. ANGARA, JUAN PONCE ENRILE, LUISA P.
EJERCITO-ESTRADA, JINGGOY E. ESTRADA, PANFILO M. LACSON, ALFREDO S. LIM, JAMBY
Sir:
A.S. MADRIGAL, and SERGIO R. OSMEÑA III, Petitioners
vs.
EXEC. SECRETARY EDUARDO R. ERMITA, FLORENCIO B. ABAD, AVELINO J. CRUZ, JR., Pursuant to the provisions of existing laws, you are hereby appointed ACTING SECRETARY,
MICHAEL T. DEFENSOR, JOSEPH H. DURANO, RAUL M. GONZALEZ, ALBERTO G. ROMULO, DEPARTMENT OF (appropriate department) vice (name of person replaced).
RENE C. VILLA, and ARTHUR C. YAP, Respondents.
By virtue hereof, you may qualify and enter upon the performance of the duties and
DECISION functions of the office, furnishing this Office and the Civil Service Commission with copies of
your Oath of Office.
CARPIO, J.:
Gloria Arroyo
The Case
Respondents took their oath of office and assumed duties as acting secretaries.
This is a petition for certiorari and prohibition1 with a prayer for the issuance of a writ of
preliminary injunction to declare unconstitutional the appointments issued by President On 8 September 2004, Aquilino Q. Pimentel, Jr. ("Senator Pimentel"), Edgardo J. Angara
Gloria Macapagal-Arroyo ("President Arroyo") through Executive Secretary Eduardo R. Ermita ("Senator Angara"), Juan Ponce Enrile ("Senator Enrile"), Luisa P. Ejercito-Estrada ("Senator
("Secretary Ermita") to Florencio B. Abad, Avelino J. Cruz, Jr., Michael T. Defensor, Joseph H. Ejercito-Estrada"), Jinggoy E. Estrada ("Senator Estrada"), Panfilo M. Lacson ("Senator
Durano, Raul M. Gonzalez, Alberto G. Romulo, Rene C. Villa, and Arthur C. Yap Lacson"), Alfredo S. Lim ("Senator Lim"), Jamby A.S. Madrigal ("Senator Madrigal"), and
("respondents") as acting secretaries of their respective departments. The petition also seeks Sergio R. Osmeña, III ("Senator Osmeña") ("petitioners") filed the present petition as
to prohibit respondents from performing the duties of department secretaries. Senators of the Republic of the Philippines.

Antecedent Facts Congress adjourned on 22 September 2004. On 23 September 2004, President Arroyo
issued ad interimappointments3 to respondents as secretaries of the departments to which
they were previously appointed in an acting capacity. The appointment papers are uniformly
The Senate and the House of Representatives ("Congress") commenced their regular session
worded as follows:
on 26 July 2004. The Commission on Appointments, composed of Senators and
Representatives, was constituted on 25 August 2004.
Sir:
Meanwhile, President Arroyo issued appointments2 to respondents as acting secretaries of
their respective departments. Pursuant to the provisions of existing laws, you are hereby appointed SECRETARY [AD
INTERIM], DEPARTMENT OF (appropriate department).

Appointee Department Date of Appointment


By virtue hereof, you may qualify and enter upon the performance of the duties and
Arthur C. Yap Agriculture 15 August 2004 functions of the office, furnishing this Office and the Civil Service Commission with copies of
Alberto G. Romulo Foreign Affairs 23 August 2004 your oath of office.
Raul M. Gonzalez Justice 23 August 2004
Florencio B. Abad Education 23 August 2004 (signed)
Avelino J. Cruz, Jr. National Defense 23 August 2004
Rene C. Villa Agrarian Reform 23 August 2004 Gloria Arroyo
55

Issue xxx The Commission on Appointments is a creature of the Constitution. Although its
membership is confined to members of Congress, said Commission is independent of
The petition questions the constitutionality of President Arroyo’s appointment of Congress. The powers of the Commission do not come from Congress, but emanate directly
respondents as acting secretaries without the consent of the Commission on Appointments from the Constitution. Hence, it is not an agent of Congress. In fact, the functions of the
while Congress is in session. Commissioner are purely executive in nature. xxx9

The Court’s Ruling On Petitioners’ Standing

The petition has no merit. The Solicitor General states that the present petition is a quo warranto proceeding because,
with the exception of Secretary Ermita, petitioners effectively seek to oust respondents for
unlawfully exercising the powers of department secretaries. The Solicitor General further
Preliminary Matters
states that petitioners may not claim standing as Senators because no power of the
Commission on Appointments has been "infringed upon or violated by the President. xxx If at
On the Mootness of the Petition all, the Commission on Appointments as a body (rather than individual members of the
Congress) may possess standing in this case."10
The Solicitor General argues that the petition is moot because President Arroyo had
extended to respondents ad interim appointments on 23 September 2004 immediately after Petitioners, on the other hand, state that the Court can exercise its certiorari jurisdiction over
the recess of Congress. unconstitutional acts of the President.11 Petitioners further contend that they possess
standing because President Arroyo’s appointment of department secretaries in an acting
As a rule, the writ of prohibition will not lie to enjoin acts already done.4 However, as an capacity while Congress is in session impairs the powers of Congress. Petitioners
exception to the rule on mootness, courts will decide a question otherwise moot if it is cite Sanlakas v. Executive Secretary12 as basis, thus:
capable of repetition yet evading review.5
To the extent that the powers of Congress are impaired, so is the power of each member
In the present case, the mootness of the petition does not bar its resolution. The question of thereof, since his office confers a right to participate in the exercise of the powers of that
the constitutionality of the President’s appointment of department secretaries in an acting institution.
capacity while Congress is in session will arise in every such appointment.
An act of the Executive which injures the institution of Congress causes a derivative but
On the Nature of the Power to Appoint nonetheless substantial injury, which can be questioned by a member of Congress. In such a
case, any member of Congress can have a resort to the courts.
The power to appoint is essentially executive in nature, and the legislature may not interfere
with the exercise of this executive power except in those instances when the Constitution Considering the independence of the Commission on Appointments from Congress, it is error
expressly allows it to interfere.6 Limitations on the executive power to appoint are construed for petitioners to claim standing in the present case as members of Congress. President
strictly against the legislature.7 The scope of the legislature’s interference in the executive’s Arroyo’s issuance of acting appointments while Congress is in session impairs no power of
power to appoint is limited to the power to prescribe the qualifications to an appointive Congress. Among the petitioners, only the following are members of the Commission on
office. Congress cannot appoint a person to an office in the guise of prescribing qualifications Appointments of the 13th Congress: Senator Enrile as Minority Floor Leader, Senator Lacson
to that office. Neither may Congress impose on the President the duty to appoint any as Assistant Minority Floor Leader, and Senator Angara, Senator Ejercito-Estrada, and
particular person to an office.8 Senator Osmeña as members.

However, even if the Commission on Appointments is composed of members of Congress, Thus, on the impairment of the prerogatives of members of the Commission on
the exercise of its powers is executive and not legislative. The Commission on Appointments Appointments, only Senators Enrile, Lacson, Angara, Ejercito-Estrada, and Osmeña have
does not legislate when it exercises its power to give or withhold consent to presidential standing in the present petition. This is in contrast to Senators Pimentel, Estrada, Lim, and
appointments. Thus: Madrigal, who, though vigilant in protecting their perceived prerogatives as members of
Congress, possess no standing in the present petition.
56

The Constitutionality of President Arroyo’s Issuance SEC. 16. Power of Appointment. — The President shall exercise the power to appoint such
officials as provided for in the Constitution and laws.
of Appointments to Respondents as Acting Secretaries
SEC. 17. Power to Issue Temporary Designation. — (1) The President may temporarily
Petitioners contend that President Arroyo should not have appointed respondents as acting designate an officer already in the government service or any other competent person to
secretaries because "in case of a vacancy in the Office of a Secretary, it is only an perform the functions of an office in the executive branch, appointment to which is vested
Undersecretary who can be designated as Acting Secretary."13 Petitioners base their in him by law, when: (a) the officer regularly appointed to the office is unable to perform
argument on Section 10, Chapter 2, Book IV of Executive Order No. 292 ("EO 292"), 14 which his duties by reason of illness, absence or any other cause; or (b) there exists a vacancy[.]
enumerates the powers and duties of the undersecretary. Paragraph 5 of Section 10 reads:
(2) The person designated shall receive the compensation attached to the position, unless he
SEC. 10. Powers and Duties of the Undersecretary. - The Undersecretary shall: is already in the government service in which case he shall receive only such additional
compensation as, with his existing salary, shall not exceed the salary authorized by law for
the position filled. The compensation hereby authorized shall be paid out of the funds
(5) Temporarily discharge the duties of the Secretary in the latter’s absence or inability to
appropriated for the office or agency concerned.
discharge his duties for any cause or in case of vacancy of the said office, unless otherwise
provided by law. Where there are more than one Undersecretary, the Secretary shall allocate
the foregoing powers and duties among them. The President shall likewise make the (3) In no case shall a temporary designation exceed one (1) year. (Emphasis supplied)
temporary designation of Acting Secretary from among them; and
Petitioners and respondents maintain two diametrically opposed lines of thought. Petitioners
Petitioners further assert that "while Congress is in session, there can be no appointments, assert that the President cannot issue appointments in an acting capacity to department
whether regular or acting, to a vacant position of an office needing confirmation by the secretaries while Congress is in session because the law does not give the President such
Commission on Appointments, without first having obtained its consent."15 power. In contrast, respondents insist that the President can issue such appointments
because no law prohibits such appointments.
In sharp contrast, respondents maintain that the President can issue appointments in an
acting capacity to department secretaries without the consent of the Commission on The essence of an appointment in an acting capacity is its temporary nature. It is a stop-gap
Appointments even while Congress is in session. Respondents point to Section 16, Article VII measure intended to fill an office for a limited time until the appointment of a permanent
of the 1987 Constitution. Section 16 reads: occupant to the office.16 In case of vacancy in an office occupied by an alter ego of the
President, such as the office of a department secretary, the President must necessarily
appoint an alter ego of her choice as acting secretary before the permanent appointee of her
SEC. 16. The President shall nominate and, with the consent of the Commission on
choice could assume office.
Appointments, appoint the heads of the executive departments, ambassadors, other public
ministers and consuls, or officers of the armed forces from the rank of colonel or naval
captain, and other officers whose appointments are vested in him in this Constitution. He Congress, through a law, cannot impose on the President the obligation to appoint
shall also appoint all other officers of the Government whose appointments are not automatically the undersecretary as her temporary alter ego. An alter ego, whether
otherwise provided for by law, and those whom he may be authorized by law to appoint. The temporary or permanent, holds a position of great trust and confidence. Congress, in the
Congress may, by law, vest the appointment of other officers lower in rank in the President guise of prescribing qualifications to an office, cannot impose on the President who her alter
alone, in the courts, or in the heads of departments, agencies, commissions, or boards. ego should be.

The President shall have the power to make appointments during the recess of the Congress, The office of a department secretary may become vacant while Congress is in session. Since a
whether voluntary or compulsory, but such appointments shall be effective only until department secretary is the alter ego of the President, the acting appointee to the office
disapproval by the Commission on Appointments or until the next adjournment of the must necessarily have the President’s confidence. Thus, by the very nature of the office of a
Congress. department secretary, the President must appoint in an acting capacity a person of her
choice even while Congress is in session. That person may or may not be the permanent
appointee, but practical reasons may make it expedient that the acting appointee will also be
Respondents also rely on EO 292, which devotes a chapter to the President’s power of
the permanent appointee.
appointment. Sections 16 and 17, Chapter 5, Title I, Book III of EO 292 read:
57

The law expressly allows the President to make such acting appointment. Section 17, Chapter While Congress was in session, due to vacancies in the cabinet, then president Gloria
5, Title I, Book III of EO 292 states that "[t]he President may temporarily designate an officer Macapagal-Arroyo (GMA) appointed Arthur Yap et al as secretaries of their respective
already in the government service or any other competent person to perform the functions departments. They were appointed in an acting capacity only. Senator Aquilino Pimentel
of an office in the executive branch." Thus, the President may even appoint in an acting together with 7 other senators filed a complaint against the appointment of Yap et al.
capacity a person not yet in the government service, as long as the President deems that Pimentel averred that GMA cannot make such appointment without the consent of the
person competent. Commission on Appointment; that, in accordance with Section 10, Chapter 2, Book IV of
Executive Order No. 292, only the undersecretary of the respective departments should
Petitioners assert that Section 17 does not apply to appointments vested in the President by be designated in an acting capacity and not anyone else.
the Constitution, because it only applies to appointments vested in the President by law. On the contrary, then Executive Secretary Eduardo Ermita averred that the president is
Petitioners forget that Congress is not the only source of law. "Law" refers to the empowered by Section 16, Article VII of the 1987 Constitution to issue appointments in an
Constitution, statutes or acts of Congress, municipal ordinances, implementing rules issued acting capacity to department secretaries without the consent of the Commission on
pursuant to law, and judicial decisions.17 Appointments even while Congress is in session. Further, EO 292 itself allows the president
to issue temporary designation to an officer in the civil service provided that the temporary
Finally, petitioners claim that the issuance of appointments in an acting capacity is designation shall not exceed one year.
susceptible to abuse. Petitioners fail to consider that acting appointments cannot exceed one
During the pendency of said case, Congress adjourned and GMA issued ad
year as expressly provided in Section 17(3), Chapter 5, Title I, Book III of EO 292. The law has
interim appointments re-appointing those previously appointed in acting capacity.
incorporated this safeguard to prevent abuses, like the use of acting appointments as a way
to circumvent confirmation by the Commission on Appointments. ISSUE: Whether or not the appointments made by ex PGMA is valid.
HELD:
In distinguishing ad interim appointments from appointments in an acting capacity, a noted
textbook writer on constitutional law has observed: Yes. The argument raised by Ermita is correct. Further, EO 292 itself provided the safeguard
so that such power will not be abused hence the provision that the temporary designation
shall not exceed one year. In this case, in less than a year after the initial appointments made
Ad-interim appointments must be distinguished from appointments in an acting capacity. by GMA, and when the Congress was in recess, GMA issued the ad interim appointments –
Both of them are effective upon acceptance. But ad-interim appointments are extended only this also proves that the president was in good faith.
during a recess of Congress, whereas acting appointments may be extended any time there is
a vacancy. Moreover ad-interim appointments are submitted to the Commission on It must also be noted that cabinet secretaries are the alter egos of the president. The choice
Appointments for confirmation or rejection; acting appointments are not submitted to the is the president’s to make and the president normally appoints those whom he/she can trust.
Commission on Appointments. Acting appointments are a way of temporarily filling She cannot be constrained to choose the undersecretary. She has the option to choose. An
important offices but, if abused, they can also be a way of circumventing the need for alter ego, whether temporary or permanent, holds a position of great trust and confidence.
confirmation by the Commission on Appointments.18 Congress, in the guise of prescribing qualifications to an office, cannot impose on the
President who her alter ego should be.
However, we find no abuse in the present case. The absence of abuse is readily apparent The office of a department secretary may become vacant while Congress is in session. Since a
from President Arroyo’s issuance of ad interim appointments to department secretary is the alter ego of the President, the acting appointee to the office
respondents immediately upon the recess of Congress, way before the lapse of one year. must necessarily have the President’s confidence. That person may or may not be the
permanent appointee, but practical reasons may make it expedient that the acting appointee
WHEREFORE, we DISMISS the present petition for certiorari and prohibition. will also be the permanent appointee.
Anent the issue that GMA appointed “outsiders”, such is allowed. EO 292 also provides that
SO ORDERED. the president “may temporarily designate an officer already in the government service or any
other competent person to perform the functions of an office in the executive branch.” Thus,
472 SCRA 587 – Political Law – Commission on Appointment – Ad Interim Appointments vs the President may even appoint in an acting capacity a person not yet in the government
Appointments in an Acting Capacity service, as long as the President deems that person competent.

Law on Public Officers – Modes and Kinds of Appointment NOTE: Ad Interim Appointments vs Appointments in an Acting Capacity
58

The Solicitor General states that the present petition is a quo warrantoproceeding because,
with the exception of Secretary Ermita, petitioners effectively seek to oust respondents for
Ad Interim Appointments Appointments in an Acting unlawfully exercising the powers of department secretaries. The Solicitor General further
Capacity states that petitioners may not claim standing as Senators because no power of the
Commission on Appointments has been “infringed upon or violated by the President. If at all,
the Commission on Appointments as a body (rather than individual members of the
It is a permanent appointment Acting appointments are a Congress) may possess standing in this case.” Petitioners, on the other hand, state that the
because it takes effect immediately way of temporarily filling Court can exercise its certiorari jurisdiction over unconstitutional acts of the President.
and can no longer be withdrawn by important offices but, if Petitioners further contend that they possess standing because President Arroyo’s
the President once the appointee has abused, they can also be a appointment of department secretaries in an acting capacity while Congress is in session
Description impairs the powers of Congress.
qualified into office. The fact that it is way of circumventing the
subject to confirmation by the COA need for confirmation by the
does not alter its permanent Commission on Thus, on the impairment of the prerogatives of members of the Commission on
character (Matibag vs Benipayo) Appointments. Appointments, only Senators Enrile, Lacson, Angara, Ejercito-Estrada, and Osmeña have
standing in the present petition. This is in contrast to Senators Pimentel, Estrada, Lim, and
Madrigal, who, though vigilant in protecting their perceived prerogatives as members of
When Upon Acceptance by Congress, possess no standing in the present petition.
Upon Acceptance by Appointee
Effective Appointee

Pimentel, Jr. vs Ermita, 472 SCRA 587, Oct. 13, 2005


Any time when there is
When Made When Congress is in recess Facts:
vacancy

President Arroyo issued appointments to respondents as acting secretaries without the


Submitted consent of the Commission on Appointments, while Congress is in their regular session.
Yes No
to the COA Subsequently after the Congress had adjourned, President Arroyo issued ad interim
appointments to respondents as secretaries of the departments to which they were
previously appointed in an acting capacity. Petitioners senators assailing the constitutionality
FACTS: of the appointments, assert that “while Congress is in session, there can be no appointments,
whether regular or acting, to a vacant position of an office needing confirmation by the
Commission on Appointments, without first having obtained its consent. espondent
Petitioners file for certiorari and prohibition with a prayer for the issuance of a writ of
secretaries maintain that the President can issue appointments in an acting capacity to
preliminary injunction to declare unconstitutional the appointments issued by President
department secretaries without the consent of the Commission on Appointments even while
Gloria Macapagal-Arroyo (“President Arroyo”) through Executive Secretary Eduardo R. Ermita
(“Secretary Ermita”) to Florencio B. Abad, Avelino J. Cruz, Jr., Michael T. Defensor, Joseph H. Congress is in session.!" #$#, which devotes a chapter to the President%s power of
Durano, Raul M. Gonzalez, Alberto G. Romulo, Rene C. Villa, and Arthur C. Yap appointment. Sections &' and &(, Chapter ), *itle +, oo- +++ of !" #$#.
(“respondents”) as acting secretaries of their respective departments.The petition also seeks
to prohibit respondents from performing the duties of department secretaries. !ss"e

"/ the President can issue appointments in an acting capacity to department secretaries
ISSUE: Do the petitioners have legal standing?
while Congress is in session.

RULING: #el$
59

0es. *he essence of an appointment in an acting capacity is its temporary nature. +t is a SEC. 16. Power of Appointment. — The President shall exercise the power to appoint such
measure intended to fill an office for a limited time until the appointment of a permanent officials as provided for in the Constitution and laws.
occupant to the office. +n case of vacancy in an office occupied by an alter ego of the
President, such as the office of a department secretary, the President must necessarily SEC. 17. Power to Issue Temporary Designation. — (1) The President may temporarily
appoint an alter ego of her choice as acting secretary before the permanent appointee of her designate an officer already in the government service or any other competent person to
choice could assume office.*he power to appoint is essentially e1ecutive in nature, and the perform the functions of an office in the executive branch, appointment to which is vested in
legislature may not interfere with the e1ercise of this e1ecutive power e1cept in those him by law, when: (a) the officer regularly appointed to the office is unable to perform his
instances when the Constitution e1pressly allows it to interfere. *he power to appoint is duties by reason of illness, absence or any other cause; or (b) there exists a vacancy[.]
essentially e1ecutive in nature, and the legislature may not interfere with the e1ercise of this
Issue: WON the President can issue appointments in an acting capacity to department
e1ecutive power e1cept in those instances when the Constitution e1pressly allows it to
secretaries while Congress is in session.
interfere. 2imitations on the e1ecutive power to appoint are construed strictly against the
legislature. *he scope of the legislature%s interference in the e1ecutive%s power to appoint
Held:
is limited to the power to prescribe the qualifications to an appointive office. Congress
cannot appoint a person to an office in the guise of prescribing qualifications to that office. Yes. The essence of an appointment in an acting capacity is its temporary nature. It is a stop-
/either may Congress impose on the President the duty to appoint any particular person to gap measure intended to fill an office for a limited time until the appointment of a
an office. permanent occupant to the office. In case of vacancy in an office occupied by an alter ego of
the President, such as the office of a department secretary, the President must necessarily
Public Officer, Difference Between Ad-Interim and Acting Appointments)
appoint an alter ego of her choice as acting secretary before the permanent appointee of her
choice could assume office.
Facts:
The office of a department secretary may become vacant while Congress is in session. Since
President Arroyo issued appointments to respondents as acting secretaries of their
a department secretary is the alter ego of the President, the acting appointee to the office
respective departments without the consent of the Commission on Appointments, while
must necessarily have the President’s confidence. Thus, by the very nature of the office of a
Congress is in their regular session.
department secretary, the President must appoint in an acting capacity a person of her
Subsequently after the Congress had adjourned, President Arroyo issued ad interim choice even while Congress is in session.
appointments to respondents as secretaries of the departments to which they were
Ad interim appointments and acting appointments are both effective upon acceptance. But
previously appointed in an acting capacity.
ad-interim appointments are extended only during a recess of Congress, whereas acting
Petitioners senators assailing the constitutionality of the appointments, assert that “while appointments may be extended any time there is a vacancy. Moreover ad-interim
Congress is in session, there can be no appointments, whether regular or acting, to a vacant appointments are submitted to the Commission on Appointments for confirmation or
position of an office needing confirmation by the Commission on Appointments, without first rejection; acting appointments are not submitted to the Commission on Appointments.
having obtained its consent. Acting appointments are a way of temporarily filling important offices but, if abused, they
can also be a way of circumventing the need for confirmation by the Commission on
Respondent secretaries maintain that the President can issue appointments in an acting Appointments.
capacity to department secretaries without the consent of the Commission on Appointments
even while Congress is in session. The absence of abuse is readily apparent from President Arroyo’s issuance of ad interim
appointments to respondents immediately upon the recess of Congress, way before the
EO 292, which devotes a chapter to the President’s power of appointment. Sections 16 and lapse of one year.
17, Chapter 5, Title I, Book III of EO 292 read:
60

Note: Can Congress impose the automatic appointment of the undersecretary?

Congress, through a law, cannot impose on the President the obligation to appoint
automatically the undersecretary as her temporary alter ego.

The power to appoint is essentially executive in nature, and the legislature may not interfere
with the exercise of this executive power except in those instances when the Constitution
expressly allows it to interfere. Limitations on the executive power to appoint are construed
strictly against the legislature. The scope of the legislature’s interference in the executive’s
power to appoint is limited to the power to prescribe the qualifications to an appointive
office. Congress cannot appoint a person to an office in the guise of prescribing
qualifications to that office. Neither may Congress impose on the President the duty to
appoint any particular person to an office.
61

EN BANC WHEREFORE, judgment is hereby rendered

G.R. No. 139554 July 21, 2006 1) Declaring petitioners [the Endriga group] to have a clear right to their respective
offices to which they were elected by the CCP Board up to the expiration of their 4-
ARMITA B. RUFINO, ZENAIDA R. TANTOCO, LORENZO CALMA, RAFAEL SIMPAO, JR., and year term,
FREDDIE GARCIA, petitioners,
vs. 2) Ousting respondents [the Rufino group], except respondent Zenaida R. Tantoco,
BALTAZAR N. ENDRIGA, MA. PAZ D. LAGDAMEO, PATRICIA C. SISON, IRMA PONCE-ENRILE from their respective offices and excluding them therefrom, and
POTENCIANO, and DOREEN FERNANDEZ, respondents.
3) Dismissing the case against respondent Zenaida R. Tantoco.
DECISION
SO ORDERED.8
CARPIO, J.:
In G.R. No. 139565, petitioners Baltazar N. Endriga ("Endriga"), Ma. Paz D. Lagdameo
Presidential Decree No. 15 (PD 15) created the Cultural Center of the Philippines (CCP) for ("Lagdameo"), Patricia C. Sison ("Sison"), Irma Ponce-Enrile Potenciano ("Potenciano"), and
the primary purpose of propagating arts and culture in the Philippines.1 The CCP is to awaken Doreen Fernandez ("Fernandez"), collectively referred to as the Endriga group, assail the
the consciousness of the Filipino people to their artistic and cultural heritage and encourage Resolution dated 3 August 1999 issued by the Court of Appeals in the same case insofar as it
them to preserve, promote, enhance, and develop such heritage. 2 denied their Motion for Immediate Execution of the Decision dated 14 May 1999.

PD 15 created a Board of Trustees ("Board") to govern the CCP. PD 15 mandates the Board to The Antecedents
draw up programs and projects that (1) cultivate and enhance public interest in, and
appreciation of, Philippine art; (2) discover and develop talents connected with Philippine On 25 June 1966, then President Ferdinand E. Marcos issued Executive Order No. 30 (EO 30)
cultural pursuits; (3) create opportunities for individual and national self-expression in creating the Cultural Center of the Philippines as a trust governed by a Board of Trustees of
cultural affairs; and (4) encourage the organization of cultural groups and the staging of seven members to preserve and promote Philippine culture. The original founding trustees,
cultural exhibitions.3 The Board administers and holds in trust real and personal properties of who were all appointed by President Marcos, were Imelda Romualdez-Marcos, Juan Ponce-
the CCP for the benefit of the Filipino people.4 The Board invests income derived from its Enrile, Andres Soriano, Jr., Antonio Madrigal, Father Horacio Dela Costa, S.J., I.P. Soliongco,
projects and operations in a Cultural Development Fund set up to attain the CCP's and Ernesto Rufino.
objectives.5
On 5 October 1972, or soon after the declaration of Martial Law, President Marcos issued PD
The consolidated petitions in the case at bar stem from a quo warranto proceeding involving 15,9 the CCP's charter, which converted the CCP under EO 30 into a non-municipal public
two sets of CCP Boards. The controversy revolves on who between the contending groups, corporation free from the "pressure or influence of politics."10 PD 15 increased the members
both claiming as the rightful trustees of the CCP Board, has the legal right to hold office. The of CCP's Board from seven to nine trustees. Later, Executive Order No. 1058, issued on 10
resolution of the issue boils down to the constitutionality of the provision of PD 15 on the October 1985, increased further the trustees to 11.
manner of filling vacancies in the Board.
After the People Power Revolution in 1986, then President Corazon C. Aquino asked for the
The Case courtesy resignations of the then incumbent CCP trustees and appointed new trustees to the
Board. Eventually, during the term of President Fidel V. Ramos, the CCP Board included
Before us are two consolidated Petitions for Review on Certiorari under Rule 45 of the 1997 Endriga, Lagdameo, Sison, Potenciano, Fernandez, Lenora A. Cabili ("Cabili"), and Manuel T.
Rules of Civil Procedure. In G.R. No. 139554, petitioners Armita B. Rufino ("Rufino"), Zenaida Mañosa ("Mañosa").
R. Tantoco ("Tantoco"),6 Lorenzo Calma ("Calma"), Rafael Simpao, Jr. ("Simpao"), and Freddie
Garcia ("Garcia"), represented by the Solicitor General and collectively referred to as the On 22 December 1998, then President Joseph E. Estrada appointed seven new trustees to the
Rufino group, seek to set aside the Decision7 dated 14 May 1999 of the Court of Appeals in CCP Board for a term of four years to replace the Endriga group as well as two other
CA-G.R. SP No. 50272 as well as the Resolution dated 3 August 1999 denying the motion for incumbent trustees. The seven new trustees were:
reconsideration. The dispositive portion of the appellate court's decision reads:
62

1. Armita B. Rufino - President, vice Baltazar N. Endriga The Court referred the Endriga group's petition to the Court of Appeals "for appropriate
action" in observance of the hierarchy of courts.
2. Zenaida R. Tantoco - Member, vice Doreen Fernandez
On 14 May 1999, the Court of Appeals rendered the Decision under review granting the quo
3. Federico Pascual - Member, vice Lenora A. Cabili warranto petition. The Court of Appeals declared the Endriga group lawfully entitled to hold
office as CCP trustees. On the other hand, the appellate court's Decision ousted the Rufino
group from the CCP Board.
4. Rafael Buenaventura - Member, vice Manuel T. Mañosa

In their motion for reconsideration, the Rufino group asserted that the law could only
5. Lorenzo Calma - Member, vice Ma. Paz D. Lagdameo
delegate to the CCP Board the power to appoint officers lower in rank than the trustees of
the Board. The law may not validly confer on the CCP trustees the authority to appoint or
6. Rafael Simpao, Jr. - Member, vice Patricia C. Sison elect their fellow trustees, for the latter would be officers of equal rank and not of lower
rank. Section 6(b) of PD 15 authorizing the CCP trustees to elect their fellow trustees should
7. Freddie Garcia - Member, vice Irma Ponce-Enrile Potenciano be declared unconstitutional being repugnant to Section 16, Article VII of the 1987
Constitution allowing the appointment only of "officers lower in rank" than the appointing
Except for Tantoco, the Rufino group took their respective oaths of office and assumed the power.
performance of their duties in early January 1999.
On 3 August 1999, the Court of Appeals denied the Rufino group's motion for
On 6 January 1999, the Endriga group filed a petition for quo warranto before this Court reconsideration. The Court of Appeals also denied the Endriga group's motion for immediate
questioning President Estrada's appointment of seven new members to the CCP Board. The execution of the 14 May 1999 Decision.
Endriga group alleged that under Section 6(b) of PD 15, vacancies in the CCP Board "shall be
filled by election by a vote of a majority of the trustees held at the next regular meeting x x Hence, the instant consolidated petitions.
x." In case "only one trustee survive[s], the vacancies shall be filled by the surviving trustee
acting in consultation with the ranking officers of the [CCP]." The Endriga group claimed that Meanwhile, Angara filed a Petition-in-Intervention before this Court alleging that although
it is only when the CCP Board is entirely vacant may the President of the Philippines fill such she was not named as a respondent in the quo warranto petition, she has an interest in the
vacancies, acting in consultation with the ranking officers of the CCP. case as the then incumbent CCP Board Chairperson. Angara adopted the same position and
offered the same arguments as the Rufino group.
The Endriga group asserted that when former President Estrada appointed the Rufino group,
only one seat was vacant due to the expiration of Mañosa's term. The CCP Board then had 10 The Ruling of the Court of Appeals
incumbent trustees, namely, Endriga, Lagdameo, Sison, Potenciano, Fernandez, together
with Cabili, Father Bernardo P. Perez ("Fr. Perez"), Eduardo De los Angeles ("De los Angeles"),
The Court of Appeals held that Section 6(b) of PD 15 providing for the manner of filling
Ma. Cecilia Lazaro ("Lazaro"), and Gloria M. Angara ("Angara"). President Estrada retained Fr.
vacancies in the CCP Board is clear, plain, and free from ambiguity. Section 6(b) of PD 15
Perez, De los Angeles, Lazaro, and Angara as trustees.
mandates the remaining trustees to fill by election vacancies in the CCP Board. Only when
the Board is entirely vacant, which is not the situation in the present case, may the President
Endriga's term was to expire on 26 July 1999, while the terms of Lagdameo, Sison, exercise his power to appoint.
Potenciano, and Fernandez were to expire on 6 February 1999. The Endriga group
maintained that under the CCP Charter, the trustees' fixed four-year term could only be
The Court of Appeals stated that the legislative history of PD 15 shows a clear intent "to
terminated "by reason of resignation, incapacity, death, or other cause." Presidential action
insulate the position of trustee from the pressure or influence of politics by abandoning
was neither necessary nor justified since the CCP Board then still had 10 incumbent trustees
appointment by the President of the Philippines as the mode of filling"11 vacancies in the CCP
who had the statutory power to fill by election any vacancy in the Board.
Board. The Court of Appeals held that until Section 6(b) of PD 15 is declared unconstitutional
in a proper case, it remains the law. The Court of Appeals also clarified that PD 15 vests on
The Endriga group refused to accept that the CCP was under the supervision and control of the CCP Chairperson the power to appoint all officers, staff, and personnel of the CCP,
the President. The Endriga group cited Section 3 of PD 15, which states that the CCP "shall subject to confirmation by the Board.
enjoy autonomy of policy and operation x x x."
63

The Court of Appeals denied the Rufino group's motion for reconsideration for failure to raise A. Endriga et al. are estopped from instituting the quo warranto action since they
new issues except the argument that Section 6(b) of PD 15 is unconstitutional. The Court of recognized and benefited from the administrative construction regarding the filling
Appeals declined to rule on the constitutionality of Section 6(b) of PD 15 since the Rufino of vacancies in the CCP Board of Trustees x x x;
group raised this issue for the first time in the motion for reconsideration. The Court of
Appeals also held, "Nor may the President's constitutional and/or statutory power of B. x x x [Endriga et al.'s] terms did not legally commence as [they] were not validly
supervision and control over government corporations restrict or modify the application of elected under PD 15;
the CCP Charter."12
C. assuming that [Endriga et al.] were validly elected, they lost their right to retain
The Court of Appeals, moreover, denied the Endriga group's motion for immediate execution their offices because their terms as trustees expired on 31 December 1998;
of judgment on the ground that the reasons submitted to justify execution pending appeal
were not persuasive.
D. [Endriga et al.] assumed positions in conflict x x x with their offices in the CCP
and were thus not entitled to retain the same;
The Issues
V
In G.R. No. 139554, the Rufino group, through the Solicitor General, contends that the Court
of Appeals committed reversible error:
x x x in not dismissing the quo warranto petition for being moot x x x;

I
VI

x x x in holding that it was "not actuated" to pass upon the constitutionality of


x x x in holding that [Rufino et al.'s] prayer [that the] disputed offices [be declared]
Section 6(b) of PD 15 inasmuch as the issue was raised for the first time in [Rufino
entirely as vacant is bereft of basis and amounts to "an admission of their lack of
et al.'s] motion for reconsideration;
right to the office they claim."13

II
In G.R. No. 139565, the Endriga group raises the following issue:

x x x in not holding that Section 6(b) of PD 15 is unconstitutional considering that:


whether a writ of quo warranto involving a public office should be declared a self-
executing judgment and deemed immediately executory under Rule 39, Section 4
A. x x x [it] is an invalid delegation of the President's appointing power under the of the Rules of Court.14
Constitution;
The Court's Ruling
B. x x x [it] effectively deprives the President of his constitutional power of control
and supervision over the CCP;
The petition in G.R. No. 139554 has merit.

III
The battle for CCP's leadership between the Rufino and Endriga groups dealt a blow to the
country's artistic and cultural activities. The highly publicized leadership row over the CCP
x x x in declaring the provisions of PD 15 as clear and complete and in failing to created discord among management, artists, scholars, employees, and even the public
apply the executive/administrative construction x x x which has been consistently because of the public interest at stake.
recognized and accepted since 1972;
Subsequently, the assumption to office of a new President in 2001 seemingly restored
IV normalcy to the CCP leadership. After then Vice-President Gloria Macapagal-Arroyo assumed
the Presidency on 20 January 2001, the Rufino group tendered their respective resignations
x x x in finding that [Endriga et al.] have a clear legal right to be the incumbent on 24-29 January 2001 as trustees of the CCP Board. On 12 July 2001, President Macapagal-
trustees and officers of the CCP considering that:
64

Arroyo appointed 11 trustees to the CCP Board with the corresponding positions set opposite susceptible of judicial resolution.16 The case must not be moot or based on extra-legal or
their names: other similar considerations not cognizable by courts of justice.17 A case becomes moot when
its purpose has become stale.18
1. Baltazar N. Endriga - Chairman
The purpose of the quo warranto petition was to oust the Rufino group from the CCP Board
2. Nestor O. Jardin - President and to declare the Endriga group as the rightful trustees of the CCP Board. It may appear that
supervening events have rendered this case moot with the resignation of the Rufino group as
well as the expiration of the terms of the Endriga group based on their appointments by then
3. Ma. Paz D. Lagdameo - Member
President Ramos. A "new" set of CCP trustees had been appointed by President Macapagal-
Arroyo and subsequently elected by the CCP Board.
4. Teresita O. Luz - Member
However, there are times when the controversy is of such character that to prevent its
5. Irma P.E. Potenciano - Member recurrence, and to assure respect for constitutional limitations, this Court must pass on the
merits of a case. This is one such case.
6. Eduardo D. De los Angeles - Member
The issues raised here are no longer just determinative of the respective rights of the
7. Patricia C. Sison - Member contending parties. The issues pertaining to circumstances personal to the Endriga group
may have become stale. These issues are (1) whether the Endriga group is estopped from
8. Benjamin H. Cervantes - Member bringing the quo warranto for they themselves were appointed by the incumbent President;
(2) whether they were validly elected by the remaining CCP trustees; (3) whether their terms
expired on 31 December 1998 as specified in their appointment papers; and (4) whether they
9. Sonia M. Roco - Member are entitled to immediate execution of judgment.

10. Ruperto S. Nicdao, Jr. - Member However, the constitutional question that gave rise to these issues will continue to spawn
the same controversy in the future, unless the threshold constitutional question is resolved
11. Lina F. Litton - Member — the validity of Section 6(b) and (c) of PD 15 on the manner of filling vacancies in the CCP
Board. While the issues may be set aside in the meantime, they are certain to recur every
In its special meeting on 13 July 2001, the CCP Board elected these 11 newly-appointed four years, especially when a new President assumes office, generating the same controversy
trustees to the same positions and as trustees of the CCP Board. In the same meeting, the all over again. Thus, the issues raised here are capable of repetition, yet evading review if
Board also elected the Chairman and President. compromises are resorted every time the same controversy erupts and the constitutionality
of Section 6(b) and (c) of PD 15 is not resolved.
On 21 December 2001, the Solicitor General submitted to this Court a manifestation stating
that the "election of the trustees was made without prejudice to the resolution of the The Court cannot refrain from passing upon the constitutionality of Section 6(b) and (c) of PD
constitutional issues before this Honorable Court in G.R. Nos. 139554 and 139565, x x x."15 15 if only to prevent a repeat of this regrettable controversy and to protect the CCP from
being periodically wracked by internecine politics. Every President who assumes office
naturally wants to appoint his or her own trustees to the CCP Board. A frontal clash will thus
The Issue of Mootness
periodically arise between the President's constitutional power to appoint under Section 16,
Article VII of the 1987 Constitution and the CCP trustees' power to elect their fellow trustees
We first consider the Rufino group's contention that the Endriga group's quo warranto suit under Section 6(b) and (c) of PD 15.
should have been dismissed for being moot. The Rufino group argued that when the Endriga
group's terms subsequently expired, there was no more actual controversy for the Court to
This Court may, in the exercise of its sound discretion, brush aside procedural barriers19 and
decide.
take cognizance of constitutional issues due to their paramount importance. It is the Court's
duty to apply the 1987 Constitution in accordance with what it says and not in accordance
For the Court to exercise its power of adjudication, there must be an actual case or with how the Legislature or the Executive would want it interpreted.20 This Court has the
controversy — one that involves a conflict of legal rights, an assertion of opposite legal claims
65

final word on what the law means.21 The Court must assure respect for the constitutional The source of the President's power to appoint, as well as the Legislature's authority to
limitations embodied in the 1987 Constitution. delegate the power to appoint, is found in Section 16, Article VII of the 1987 Constitution
which provides:
Interpreting Section 6(b) and (c) of PD 15
The President shall nominate and, with the consent of the Commission on
At the heart of the controversy is Section 6(b) of PD 15, as amended, which reads: Appointments, appoint the heads of the executive departments, ambassadors,
other public ministers and consuls, or officers of the armed forces from the rank of
colonel or naval captain, and other officers whose appointments are vested in him
Board of Trustees. — The governing powers and authority of the corporation shall
in this Constitution. He shall also appoint all other officers of the Government
be vested in, and exercised by, a Board of eleven (11) Trustees who shall serve
whose appointments are not otherwise provided for by law, and those whom he
without compensation.
may be authorized by law to appoint. The Congress may, by law, vest the
appointment of other officers lower in rank in the President alone, in the courts,
(b) Vacancies in the Board of Trustees due to termination of term, resignation, or in the heads of departments, agencies, commissions, or boards.
incapacity, death or other cause as may be provided in the By-laws, shall be filled
by election by a vote of a majority of the trustees held at the next regular
The President shall have the power to make appointments during the recess of the
meeting following occurrence of such vacancy. The elected trustee shall then hold
Congress, whether voluntary or compulsory, but such appointments shall be
office for a complete term of four years unless sooner terminated by reason of
effective only until disapproval by the Commission on Appointments or until the
resignation, incapacity, death or other cause. Should only one trustee survive, the
next adjournment of the Congress. (Emphasis supplied)
vacancies shall be filled by the surviving trustee acting in consultation with the
ranking officers of the Center. Such officers shall be designated in the Center's
Code of By-Laws. Should for any reason the Board be left entirely vacant, the same The power to appoint is the prerogative of the President, except in those instances when the
shall be filled by the President of the Philippines acting in consultation with the Constitution provides otherwise. Usurpation of this fundamentally Executive power by the
aforementioned ranking officers of the Center. (Emphasis supplied) Legislative and Judicial branches violates the system of separation of powers that inheres in
our democratic republican government.22
Inextricably related to Section 6(b) is Section 6(c) which limits the terms of the trustees, as
follows: Under Section 16, Article VII of the 1987 Constitution, the President appoints three groups of
officers. The first group refers to the heads of the Executive departments, ambassadors,
other public ministers and consuls, officers of the armed forces from the rank of colonel or
(c) No person may serve as trustee who is not a resident of the Philippines, of good
naval captain, and other officers whose appointments are vested in the President by the
moral standing in the community and at least 25 years of age: Provided, That there
Constitution. The second group refers to those whom the President may be authorized by
shall always be a majority of the trustees who are citizens of the Philippines.
law to appoint. The third group refers to all other officers of the Government whose
Trustees may not be reelected for more than two (2) consecutive terms. (Emphasis
appointments are not otherwise provided by law.
supplied)

Under the same Section 16, there is a fourth group of lower-ranked officers whose
The clear and categorical language of Section 6(b) of PD 15 states that vacancies in the CCP
appointments Congress may by law vest in the heads of departments, agencies,
Board shall be filled by a majority vote of the remaining trustees. Should only one trustee
commissions, or boards. The present case involves the interpretation of Section 16, Article VII
survive, the vacancies shall be filled by the surviving trustee acting in consultation with the
of the 1987 Constitution with respect to the appointment of this fourth group of officers.23
ranking officers of the CCP. Should the Board become entirely vacant, the vacancies shall
be filled by the President of the Philippines acting in consultation with the same ranking
officers of the CCP. Thus, the remaining trustees, whether one or more, elect their fellow The President appoints the first group of officers with the consent of the Commission on
trustees for a fixed four-year term. On the other hand, Section 6(c) of PD 15 does not allow Appointments. The President appoints the second and third groups of officers without the
trustees to reelect fellow trustees for more than two consecutive terms. consent of the Commission on Appointments. The President appoints the third group of
officers if the law is silent on who is the appointing power, or if the law authorizing the head
of a department, agency, commission, or board to appoint is declared unconstitutional. Thus,
The Power of Appointment
if Section 6(b) and (c) of PD 15 is found unconstitutional, the President shall appoint the
trustees of the CCP Board because the trustees fall under the third group of officers.
66

The Scope of the Appointment Power of the Heads of The framers of the 1987 Constitution clearly intended that Congress could by law vest the
Departments, Agencies, Commissions, or Boards appointment of lower-ranked officers in the heads of departments, agencies, commissions,
or boards. The deliberations26 of the 1986 Constitutional Commission explain this intent
The original text of Section 16, Article VII of the 1987 Constitution, as written in Resolution beyond any doubt.27
No. 51724 of the Constitutional Commission, is almost a verbatim copy of the one found in
the 1935 Constitution. Constitutional Commissioner Father Joaquin Bernas, S.J., explains the The framers of the 1987 Constitution changed the qualifying word "inferior" to the less
evolution of this provision and its import, thus: disparaging phrase "lower in rank" purely for style. However, the clear intent remained
that these inferior or lower in rank officers are the subordinates of the heads of
The last sentence of the first paragraph of Section 16 x x x is a relic from the 1935 departments, agencies, commissions, or boards who are vested by law with the power to
and 1973 Constitutions, x x x. appoint. The express language of the Constitution and the clear intent of its framers point to
only one conclusion — the officers whom the heads of departments, agencies, commissions,
or boards may appoint must be of lower rank than those vested by law with the power to
Under the 1935 Constitution, the provision was: "but the Congress may by law vest
appoint.
the appointment of inferior officers in the President alone, in the courts, or in the
heads of departments." As already seen, it meant that, while the general rule was
that all presidential appointments needed confirmation by the Commission on Congress May Vest the Authority to Appoint Only in the Heads of the Named Offices
Appointments, Congress could relax this rule by vesting the power to appoint
"inferior officers" in "the President alone, in the courts, or in the heads of Further, Section 16, Article VII of the 1987 Constitution authorizes Congress to vest "in the
departments." It also meant that while, generally, appointing authority belongs to heads of departments, agencies, commissions, or boards" the power to appoint lower-
the President, Congress could let others share in such authority. And the word ranked officers. Section 16 provides:
"inferior" was understood to mean not petty or unimportant but lower in rank
than those to whom appointing authority could be given. The Congress may, by law, vest the appointment of other officers lower in rank in
the President alone, in the courts, or in the heads of departments, agencies,
Under the 1973 Constitution, according to which the power of the President to commissions, or boards. (Emphasis supplied)
appoint was not limited by any other body, the provision read: "However, the
Batasang Pambansa may by law vest in members of the Cabinet, courts, heads of In a department in the Executive branch, the head is the Secretary. The law may not
agencies, commissions, and boards the power to appoint inferior officers in their authorize the Undersecretary, acting as such Undersecretary, to appoint lower-ranked
respective offices." No mention was made of the President. The premise was that officers in the Executive department. In an agency, the power is vested in the head of the
the power to appoint belonged to the President; but the Batasan could diffuse agency for it would be preposterous to vest it in the agency itself. In a commission, the head
this authority by allowing it to be shared by officers other than the President. is the chairperson of the commission. In a board, the head is also the chairperson of the
board. In the last three situations, the law may not also authorize officers other than the
The 1987 provision also has the evident intent of allowing Congress to give to heads of the agency, commission, or board to appoint lower-ranked officers.
officers other than the President the authority to appoint. To that extent therefore
reference to the President is pointless. And by using the word "alone," copying the The grant of the power to appoint to the heads of agencies, commissions, or boards is a
tenor of the 1935 provision, it implies, it is submitted, that the general rule in the matter of legislative grace. Congress has the discretion to grant to, or withhold from, the
1935 Constitution of requiring confirmation by the Commission on Appointments heads of agencies, commissions, or boards the power to appoint lower-ranked officers. If it
had not been changed. Thereby the picture has been blurred. This confused text, so grants, Congress may impose certain conditions for the exercise of such legislative
however, should be attributed to oversight. Reference to the President must be delegation, like requiring the recommendation of subordinate officers or the concurrence of
ignored and the whole sentence must be read merely as authority for Congress to the other members of the commission or board.
vest appointing power in courts, in heads of departments, agencies, commissions,
or boards after the manner of the 1973 text.
This is in contrast to the President's power to appoint which is a self-executing power vested
by the Constitution itself and thus not subject to legislative limitations or conditions.28 The
Incidentally, the 1987 text, in order to eschew any pejorative connotation, avoids power to appoint conferred directly by the Constitution on the Supreme Court en banc29 and
the phrase "inferior officers" and translates it instead into "officers lower in rank," on the Constitutional Commissions30 is also self-executing and not subject to legislative
that is, lower in rank than the courts or the heads of departments, agencies, limitations or conditions.
commissions, or boards.25 (Emphasis supplied)
67

The Constitution authorizes Congress to vest the power to appoint lower-ranked officers 1987 Constitution speaks of the power to appoint lower-ranked officers in the Executive
specifically in the "heads" of the specified offices, and in no other person.31 The word branch, it vests the power "in the heads of departments, agencies, commissions, or boards."
"heads" refers to the chairpersons of the commissions or boards and not to their members,
for several reasons. In addition, the 1987 Constitution expressly provides that in the case of the constitutional
commissions, the power to appoint lower-ranked officers is vested in the commission as a
First, a plain reading of the last sentence of the first paragraph of Section 16, Article VII of the body. Thus, Section 4, Article IX-A of the 1987 Constitution provides, "The Constitutional
1987 Constitution shows that the word "heads" refers to all the offices succeeding that term, Commissions shall appoint their officials and employees in accordance with law."
namely, the departments, agencies, commissions, or boards. This plain reading is consistent
with other related provisions of the Constitution. Sixth, the last clause of the pertinent sentence in Section 16, Article VII of the 1987
Constitution is an enumerationof offices whose heads may be vested by law with the power
Second, agencies, like departments, have no collegial governing bodies but have only chief to appoint lower-ranked officers. This is clear from the framers' deliberations of the 1987
executives or heads of agencies. Thus, the word "heads" applies to agencies. Any other Constitution, thus:
interpretation is untenable.
THE PRESIDENT: Commissioner Davide is recognized.
Third, all commissions or boards have chief executives who are their heads. Since the
Constitution speaks of "heads" of offices, and all commissions or boards have chief MR. DAVIDE: On page 8, line 3, change the period (.) after "departments" to a
executives or heads, the word "heads" could only refer to the chief executives or heads of comma (,) and add AGENCIES, COMMISSIONS, OR BOARDS. This is just to complete
the commissions or boards. the enumeration in the 1935 Constitution from which this additional clause was
taken.
Fourth, the counterpart provisions of Section 16, Article VII of the 1987 Constitution in the
1935 and 1973 Constitutions uniformly refer to "heads" of offices. The 1935 Constitution THE PRESIDENT: Does the Committee accept?
limited the grant of the appointment power only to "heads of departments." 32 The 1973
Constitution expanded such grant to other officers, namely, "members of the Cabinet, x x x,
MR. SUMULONG: We accept the amendment.
courts, heads of agencies, commissions, and boards x x x."33

MR. ROMULO: The Committee has accepted the amendment, Madam President.
If the 1973 Constitution intended to extend the grant to members of commissions or boards,
it could have followed the same language used for "members of the Cabinet" so as to state
"members of commissions or boards." Alternatively, the 1973 Constitution could have placed THE PRESIDENT: Is there any objection to the addition of the words "AGENCIES,
the words commissions and boards after the word "courts" so as to state "members of the COMMISSIONS, OR BOARDS" on line 3, page 8? (Silence) The Chair hears none; the
Cabinet, x x x, courts, commissions and boards." Instead, the 1973 Constitution used "heads amendment is approved.34 (Italicization in the original; boldfacing supplied)
of agencies, commissions, and boards."
As an enumeration of offices, what applies to the first office in the enumeration also applies
Fifth, the 1935, 1973, and 1987 Constitutions make a clear distinction whenever granting the to the succeeding offices mentioned in the enumeration. Since the words "in the heads of"
power to appoint lower-ranked officers to members of a collegial body or to the head of that refer to "departments," the same words "in the heads of" also refer to the other offices
collegial body. Thus, the 1935 Constitution speaks of vesting the power to appoint "in the listed in the enumeration, namely, "agencies, commissions, or boards."
courts, or in the heads of departments." Similarly, the 1973 Constitution speaks of "members
of the Cabinet, courts, heads of agencies, commissions, and boards." The Chairperson of the CCP Board is the Head of CCP

Also, the 1987 Constitution speaks of vesting the power to appoint "in the courts, or in the The head of the CCP is the Chairperson of its Board. PD 15 and its various amendments
heads of departments, agencies, commissions, or boards." This is consistent with Section constitute the Chairperson of the Board as the head of CCP. Thus, Section 8 of PD 15
5(6), Article VIII of the 1987 Constitution which states that the "Supreme Court shall x x x provides:
[a]ppoint all officials and employees of the Judiciary in accordance with the Civil Service
Law," making the Supreme Court en banc the appointing power. In sharp contrast, when the Appointment of Personnel. — The Chairman, with the confirmation of the Board,
shall have the power to appoint all officers, staff and personnel of the Center with
68

such compensation as may be fixed by the Board, who shall be residents of the trustees of the CCP Board to elect their co-trustees, Section 6(b) and (c) of PD 15 is
Philippines. The Center may elect membership in the Government Service unconstitutional because it violates Section 16, Article VII of the 1987 Constitution.
Insurance System and if it so elects, its officers and employees who qualify shall
have the same rights and privileges as well as obligations as those enjoyed or borne It does not matter that Section 6(b) of PD 15 empowers the remaining trustees to "elect" and
by persons in the government service. Officials and employees of the Center shall not "appoint" their fellow trustees for the effect is the same, which is to fill vacancies in the
be exempt from the coverage of the Civil Service Law and Rules. CCP Board. A statute cannot circumvent the constitutional limitations on the power to
appoint by filling vacancies in a public office through election by the co-workers in that office.
Section 3 of the Revised Rules and Regulations of the CCP recognizes that the head of the Such manner of filling vacancies in a public office has no constitutional basis.
CCP is the Chairman of its Board when it provides:
Further, Section 6(b) and (c) of PD 15 makes the CCP trustees the independent appointing
CHAIRMAN OF THE BOARD. — The Board of Trustees shall elect a Chairman who power of their fellow trustees. The creation of an independent appointing power inherently
must be one of its members, and who shall be the presiding officer of the Board of conflicts with the President's power to appoint. This inherent conflict has spawned recurring
Trustees, with power among others, to appoint, within the compensation fixed by controversies in the appointment of CCP trustees every time a new President assumes office.
the Board, and subject to confirmation of the Board, remove, discipline all officers
and personnel of the Center, and to do such other acts and exercise such other In the present case, the incumbent President appointed the Endriga group as trustees, while
powers as may be determined by the Board of Trustees. The Chairman shall the remaining CCP trustees elected the same Endriga group to the same positions. This has
perform his duties and exercise his powers as such until such time as the Board of been the modus vivendi in filling vacancies in the CCP Board, allowing the President
Trustees, by a majority vote, shall elect another Chairman. The Chairman shall be to appoint and the CCP Board to elect the trustees. In effect, there are two appointing
concurrently President, unless the Board otherwise elects another President. powers over the same set of officers in the Executive branch. Each appointing power insists
on exercising its own power, even if the two powers are irreconcilable. The Court must put
Thus, the Chairman of the CCP Board is the "head" of the CCP who may be vested by law, an end to this recurring anomaly.
under Section 16, Article VII of the 1987 Constitution, with the power to appoint lower-
ranked officers of the CCP. The President's Power of Control

Under PD 15, the CCP is a public corporation governed by a Board of Trustees. Section 6 of There is another constitutional impediment to the implementation of Section 6(b) and (c) of
PD 15, as amended, states: PD 15. Under our system of government, all Executive departments, bureaus, and offices are
under the control of the President of the Philippines. Section 17, Article VII of the 1987
Board of Trustees. — The governing powers and authority of the corporation shall Constitution provides:
be vested in, and exercised by, a Board of eleven (11) Trustees who shall serve
without compensation. The President shall have control of all the executive departments, bureaus, and
offices. He shall ensure that the laws be faithfully executed. (Emphasis supplied)
The CCP, being governed by a board, is not an agency but a board for purposes of Section 16,
Article VII of the 1987 Constitution. The presidential power of control over the Executive branch of government extends to all
executive employees from the Department Secretary to the lowliest clerk. 35 This
Section 6(b) and (c) of PD 15 Repugnant to Section 16, Article VII of the 1987 Constitution constitutional power of the President is self-executing and does not require any
implementing law. Congress cannot limit or curtail the President's power of control over the
Section 6(b) and (c) of PD 15 is thus irreconcilably inconsistent with Section 16, Article VII of Executive branch.36
the 1987 Constitution. Section 6(b) and (c) of PD 15 empowers the remaining trustees of the
CCP Board to fill vacancies in the CCP Board, allowing them to elect their fellow trustees. On The 1987 Constitution has established three branches of government — the Executive,
the other hand, Section 16, Article VII of the 1987 Constitution allows heads of departments, Legislative and Judicial. In addition, there are the independent constitutional bodies — like
agencies, commissions, or boards to appoint only "officers lower in rank" than such "heads the Commission on Elections, Commission on Audit, Civil Service Commission, and the
of departments, agencies, commissions, or boards." This excludes a situation where the Ombudsman. Then there are the hybrid or quasi-judicial agencies,37 exercising jurisdiction in
appointing officer appoints an officer equal in rank as him. Thus, insofar as it authorizes the specialized areas, that are under the Executive branch for administrative supervision
purposes, but whose decisions are reviewable by the courts. Lastly, there are the local
69

government units, which under the Constitution enjoy local autonomy38 subject only to power of the Supreme Court. This will destroy the system of checks and balances finely
limitations Congress may impose by law.39 Local government units are subject to general structured in the 1987 Constitution among the Executive, Legislative, and Judicial branches.
supervision by the President.40
Of course, the President's power of control does not extend to quasi-judicial bodies whose
Every government office, entity, or agency must fall under the Executive, Legislative, or proceedings and decisions are judicial in nature and subject to judicial review, even as such
Judicial branches, or must belong to one of the independent constitutional bodies, or must quasi-judicial bodies may be under the administrative supervision of the President. It also
be a quasi-judicial body or local government unit. Otherwise, such government office, entity, does not extend to local government units, which are merely under the general supervision
or agency has no legal and constitutional basis for its existence. of the President.

The CCP does not fall under the Legislative or Judicial branches of government. The CCP is Section 6(b) and (c) of PD 15, which authorizes the trustees of the CCP Board to fill vacancies
also not one of the independent constitutional bodies. Neither is the CCP a quasi-judicial in the Board, runs afoul with the President's power of control under Section 17, Article VII of
body nor a local government unit. Thus, the CCP must fall under the Executive branch. Under the 1987 Constitution. The intent of Section 6(b) and (c) of PD 15 is to insulate the CCP from
the Revised Administrative Code of 1987, any agency "not placed by law or order creating political influence and pressure, specifically from the President.44 Section 6(b) and (c) of PD
them under any specific department" falls "under the Office of the President."41 15 makes the CCP a self-perpetuating entity, virtually outside the control of the President.
Such a public office or board cannot legally exist under the 1987 Constitution.
Since the President exercises control over "all the executive departments, bureaus, and
offices," the President necessarily exercises control over the CCP which is an office in the Section 3 of PD 15, as amended, states that the CCP "shall enjoy autonomy of policy and
Executive branch. In mandating that the President "shall have control of all executive x x x operation x x x."45 This provision does not free the CCP from the President's control, for if it
offices," Section 17, Article VII of the 1987 Constitution does not exempt any executive office does, then it would be unconstitutional. This provision may give the CCP Board a free hand in
— one performing executive functions outside of the independent constitutional bodies — initiating and formulating policies and undertaking activities, but ultimately these policies
from the President's power of control. There is no dispute that the CCP performs executive, and activities are all subject to the President's power of control.
and not legislative, judicial, or quasi-judicial functions.
The CCP is part of the Executive branch. No law can cut off the President's control over the
The President's power of control applies to the acts or decisions of all officers in the CCP in the guise of insulating the CCP from the President's influence. By stating that the
Executive branch. This is true whether such officers are appointed by the President or by "President shall have control of all the executive x x x offices," the 1987 Constitution
heads of departments, agencies, commissions, or boards. The power of control means the empowers the President not only to influence but even to control all offices in the Executive
power to revise or reverse the acts or decisions of a subordinate officer involving the exercise branch, including the CCP. Control is far greater than, and subsumes, influence.
of discretion.42
WHEREFORE, we GRANT the petition in G.R. No. 139554. We
In short, the President sits at the apex of the Executive branch, and exercises "control of all declare UNCONSTITUTIONAL Section 6(b) and (c) of Presidential Decree No. 15, as amended,
the executive departments, bureaus, and offices." There can be no instance under the insofar as it authorizes the remaining trustees to fill by election vacancies in the Board of
Constitution where an officer of the Executive branch is outside the control of the President. Trustees of the Cultural Center of the Philippines. In view of this ruling in G.R. No. 139554, we
The Executive branch is unitary since there is only one President vested with executive power find it unnecessary to rule on G.R. No. 139565.
exercising control over the entire Executive branch.43 Any office in the Executive branch that
is not under the control of the President is a lost command whose existence is without any SO ORDERED.
legal or constitutional basis.
FACTS:
The Legislature cannot validly enact a law that puts a government office in the Executive
branch outside the control of the President in the guise of insulating that office from politics On 25 June 1966, then President Ferdinand E. Marcos issued Executive Order No. 30 (EO 30)
or making it independent. If the office is part of the Executive branch, it must remain subject
creating the Cultural Center of the Philippines as a trust governed by a Board of Trustees of
to the control of the President. Otherwise, the Legislature can deprive the President of his
constitutional power of control over "all the executive x x x offices." If the Legislature can do seven members to preserve and promote Philippine culture.
this with the Executive branch, then the Legislature can also deal a similar blow to the
Judicial branch by enacting a law putting decisions of certain lower courts beyond the review On 5 October 1972, or soon after the declaration of Martial Law, President Marcos issued
PD 15, the CCP’s charter, which converted the CCP under EO 30 into a non-municipal public
70

corporation free from the “pressure or influence of politics.” PD 15 increased the members The Endriga group asserted that when former President Estrada appointed the Rufino group,
of CCP’s Board from seven to nine trustees. Later, Executive Order No. 1058, issued on 10 only one seat was vacant due to the expiration of Mañosa’s term. The CCP Board then had
October 1985, increased further the trustees to 11. 10 incumbent trustees.

After the People Power Revolution in 1986, then President Corazon C. Aquino asked for the The Endriga group refused to accept that the CCP was under the supervision and control of
courtesy resignations of the then incumbent CCP trustees and appointed new trustees to the the President. The Endriga group cited Section 3 of PD 15, which states that the CCP “shall
Board. Eventually, during the term of President Fidel V. Ramos, the CCP Board included enjoy autonomy of policy and operation x x x.”
Endriga, Lagdameo, Sison, Potenciano, Fernandez, Lenora A. Cabili (“Cabili”), and Manuel
T. Mañosa (“Mañosa”). On 14 May 1999, the Court of Appeals granted the quo warranto petition. The Court of
Appeals declared the Endriga group lawfully entitled to hold office as CCP trustees. On the
On 22 December 1998, then President Joseph E. Estrada appointed seven new trustees to the other hand, the appellate court’s Decision ousted the Rufino group from the CCP Board.
CCP Board for a term of four years to replace the Endriga group as well as two other
incumbent trustees. The seven new trustees were: In their motion for reconsideration, the Rufino group asserted that the law could only
delegate to the CCP Board the power to appoint officers lower in rank than the trustees of
1. Armita B. Rufino - President, vice Baltazar the Board. The law may not validly confer on the CCP trustees the authority to appoint or
elect their fellow trustees, for the latter would be officers of equal rank and not of lower
N. Endriga rank. Section 6(b) of PD 15 authorizing the CCP trustees to elect their fellow trustees should
be declared unconstitutional being repugnant to Section 16, Article VII of the 1987
2. Zenaida R. Tantoco - Member, vice Doreen Fernandez
Constitution allowing the appointment only of “officers lower in rank” than the appointing
power.
3. Federico Pascual - Member, vice Lenora A. Cabili
On 3 August 1999, the Court of Appeals denied the Rufino group’s motion for
4. Rafael Buenaventura - Member, vice Manuel T. Mañosa
reconsideration. The Court of Appeals also denied the Endriga group’s motion for immediate
5. Lorenzo Calma - Member, vice Ma. Paz D. Lagdameo execution of the 14 May 1999 Decision.

6. Rafael Simpao, Jr. - Member, vice Patricia C. Sison Hence, the instant consolidated petitions.

7. Freddie Garcia - Member, vice Irma Ponce-Enrile ISSUE: Whether or not Sec. 6 (b) of PD 15 is constitutional and CCP trustees have the
authority to appoint and elect their fellow trustees when there is vacancy.
Potenciano
RULING:
Except for Tantoco, the Rufino group took their respective oaths of office and assumed the
performance of their duties in early January 1999. NO. The SC ruled that Sec. 6 (b) and (c) of PD 15 as amended which authorizes the remaining
trustees to fill by election vacancies in the Board of Trustees of CCP is unconstitutional.
On 6 January 1999, the Endriga group filed a petition for quo warranto before this Court
questioning President Estrada’s appointment of seven new members to the CCP Board. The Section 6(b) and (c) of PD 15, which authorizes the trustees of the CCP Board to fill vacancies
Endriga group alleged that under Section 6(b) of PD 15, vacancies in the CCP Board “shall be in the Board, runs afoul with the President’s power of control under Section 17, Article VII of
filled by election by a vote of a majority of the trustees held at the next regular meeting x x the 1987 Constitution. The intent of Section 6(b) and (c) of PD 15 is to insulate the CCP from
x.” In case “only one trustee survive[s], the vacancies shall be filled by the surviving trustee political influence and pressure, specifically from the President. Section 6(b) and (c) of PD 15
acting in consultation with the ranking officers of the [CCP].” The Endriga group claimed that makes the CCP a self-perpetuating entity, virtually outside the control of the President. Such
it is only when the CCP Board is entirely vacant may the President of the Philippines fill such a public office or board cannot legally exist under the 1987 Constitution.
vacancies, acting in consultation with the ranking officers of the CCP.
71

Section 3 of PD 15, as amended, states that the CCP “shall enjoy autonomy of policy and Article 7, Section 16 (1) since both positions are equal in nature. CCP cannot invoke
operation x x x.” This provision does not free the CCP from the President’s control, for if it autonomy prescribed in its charter as an exemption from the limitation of delegative
does, then it would be unconstitutional. This provision may give the CCP Board a free hand appointing power because such invocation puts CCP outside the control of the President.
in initiating and formulating policies and undertaking activities, but ultimately these policies
and activities are all subject to the President’s power of control.

The CCP is part of the Executive branch. No law can cut off the President’s control over the
CCP in the guise of insulating the CCP from the President’s influence. By stating that the
“President shall have control of all the executive x x x offices,” the 1987 Constitution
empowers the President not only to influence but even to control all offices in the Executive
branch, including the CCP. Control is far greater than, and subsumes, influence.

CASE MAIN POINT:

Appointing authority may be given to other officials than the President provided the
appointment is in a rank lower than the appointing official. (ARTICLE 7, SECTION 16:
APPOINTING POWER CAN BE VESTED IN OTHER OFFICIALS)

FACTS:

Two groups of appointed members of the Board of Trustees of CCP are contesting each
other’s appointment. The Endriga group, sitting as current members, was appointed by then-
President Ramos and is assailing the appointment of the Rufino group, replacing all 7
members of the Endriga group, by then-President Estrada. Endriga group avers that the
appointment into the Board of the Rufino group transgressed PD 15 – creation of Board of
Trustees of CCP. As stated in PD 15, specifically Section 6, appointment into the Board shall
only be made by a majority vote of the trustees; presidential appointments can only be made
when the Board is entirely vacant to uphold the CCP’s charter of independence from
pressure or politics. Meanwhile, Rufino group stands by their appointment since the
provision on appointments stated in Section 6, PD 15 is violative of Section 16, Article 7 of
the Constitution. The Board cannot invoke the charter of autonomy to extend to
appointment of its members.

ISSUE: Whether or not PD 15, Section 6 allowing appointments made by trustees of their
fellow members is constitutional?

HELD:

No, PD 15, Section 6 allowing appointments of members by the trustees themselves is


UNCONSTITUTIONAL. While it is stated that appointing powers may be delegated by the
President, such power is limited in scope to include only ranks lower than the appointing
authority. In the case, an appointment of a member made by a fellow member transgresses
72

EN BANC Motions for Reconsideration

G.R. No. 191002 April 20, 2010 Petitioners Jaime N. Soriano (G.R. No. 191032), Amador Z. Tolentino and Roland B. Inting
(G.R. No. 191342), and Philippine Bar Association (G.R. No. 191420), as well as intervenors
ARTURO M. DE CASTRO, Petitioner, Integrated Bar of the Philippines-Davao del Sur (IBP-Davao del Sur, et al.); Christian Robert S.
vs. Lim; Peter Irving Corvera; Bagong Alyansang Bayan and others (BAYAN, et al.); Alfonso V. Tan,
JUDICIAL AND BAR COUNCIL (JBC) and PRESIDENT GLORIA MACAPAGAL - Jr.; the Women Trial Lawyers Organization of the Philippines (WTLOP); Marlou B. Ubano;
ARROYO, Respondents. Mitchell John L. Boiser; and Walden F. Bello and Loretta Ann P. Rosales (Bello, et al.), filed
their respective motions for reconsideration. Also filing a motion for reconsideration was
Senator Aquilino Q. Pimentel, Jr., whose belated intervention was allowed.
RESOLUTION

We summarize the arguments and submissions of the various motions for reconsideration, in
BERSAMIN, J.:
the aforegiven order:

On March 17, 2010, the Court promulgated its decision, holding:


Soriano

WHEREFORE, the Court:


1. The Court has not squarely ruled upon or addressed the issue of whether or not
the power to designate the Chief Justice belonged to the Supreme Court en banc.
1. Dismisses the petitions for certiorari and mandamus in G.R. No. 191002 and G.R.
No. 191149, and the petition for mandamus in G.R. No. 191057 for being
2. The Mendoza petition should have been dismissed, because it sought a mere
premature;
declaratory judgment and did not involve a justiciable controversy.

2. Dismisses the petitions for prohibition in G.R. No. 191032 and G.R. No. 191342
3. All Justices of the Court should participate in the next deliberations. The mere
for lack of merit; and
fact that the Chief Justice sits as ex officio head of the JBC should not prevail over
the more compelling state interest for him to participate as a Member of the Court.
3. Grants the petition in A.M. No. 10-2-5-SC and, accordingly, directs the Judicial
and Bar Council:
Tolentino and Inting

(a) To resume its proceedings for the nomination of candidates to fill the
1. A plain reading of Section 15, Article VII does not lead to an interpretation that
vacancy to be created by the compulsory retirement of Chief Justice
exempts judicial appointments from the express ban on midnight appointments.
Reynato S. Puno by May 17, 2010;

2. In excluding the Judiciary from the ban, the Court has made distinctions and has
(b) To prepare the short list of nominees for the position of Chief Justice;
created exemptions when none exists.

(c) To submit to the incumbent President the short list of nominees for
3. The ban on midnight appointments is placed in Article VII, not in Article VIII,
the position of Chief Justice on or before May 17, 2010; and
because it limits an executive, not a judicial, power.

(d) To continue its proceedings for the nomination of candidates to fill


4. Resort to the deliberations of the Constitutional Commission is superfluous, and
other vacancies in the Judiciary and submit to the President the short list
is powerless to vary the terms of the clear prohibition.
of nominees corresponding thereto in accordance with this decision.

5. The Court has given too much credit to the position taken by Justice Regalado.
SO ORDERED.
Thereby, the Court has raised the Constitution to the level of a venerated text
73

whose intent can only be divined by its framers as to be outside the realm of cannot tell the JBC what to do, how to do it, or when to do it, especially in the
understanding by the sovereign people that ratified it. absence of a real and justiciable case assailing any specific action or inaction of the
JBC.
6. Valenzuela should not be reversed.
9. The Court has engaged in rendering an advisory opinion and has indulged in
7. The petitioners, as taxpayers and lawyers, have the clear legal standing to speculations.
question the illegal composition of the JBC.
10. The constitutional ban on appointments being already in effect, the Court’s
Philippine Bar Association directing the JBC to comply with the decision constitutes a culpable violation of the
Constitution and the commission of an election offense.
1. The Court’s strained interpretation of the Constitution violates the basic
principle that the Court should not formulate a rule of constitutional law broader 11. The Court cannot reverse on the basis of a secondary authority a doctrine
than what is required by the precise facts of the case. unanimously formulated by the Court en banc.

2. Considering that Section 15, Article VII is clear and straightforward, the only duty 12. The practice has been for the most senior Justice to act as Chief Justice
of the Court is to apply it. The provision expressly and clearly provides a general whenever the incumbent is indisposed. Thus, the appointment of the successor
limitation on the appointing power of the President in prohibiting the appointment Chief Justice is not urgently necessary.
of any person to any position in the Government without any qualification and
distinction. 13. The principal purpose for the ban on midnight appointments is to arrest any
attempt to prolong the outgoing President’s powers by means of proxies. The
3. The Court gravely erred in unilaterally ignoring the constitutional safeguard attempt of the incumbent President to appoint the next Chief Justice is undeniably
against midnight appointments. intended to perpetuate her power beyond her term of office.

4. The Constitution has installed two constitutional safeguards:- the prohibition IBP-Davao del Sur, et al.
against midnight appointments, and the creation of the JBC. It is not within the
authority of the Court to prefer one over the other, for the Court’s duty is to apply 1. Its language being unambiguous, Section 15, Article VII of the Constitution
the safeguards as they are, not as the Court likes them to be. applies to appointments to the Judiciary. Hence, no cogent reason exists to warrant
the reversal of the Valenzuela pronouncement.
5. The Court has erred in failing to apply the basic principles of statutory
construction in interpreting the Constitution. 2. Section 16, Article VII of the Constitution provides for presidential appointments
to the Constitutional Commissions and the JBC with the consent of the Commission
6. The Court has erred in relying heavily on the title, chapter or section headings, on Appointments. Its phrase "other officers whose appointments are vested in him
despite precedents on statutory construction holding that such headings carried in this Constitution" is enough proof that the limitation on the appointing power of
very little weight. the President extends to appointments to the Judiciary. Thus, Section 14, Section
15, and Section 16 of Article VII apply to all presidential appointments in the
Executive and Judicial Branches of the Government.
7. The Constitution has provided a general rule on midnight appointments, and the
only exception is that on temporary appointments to executive positions.
3. There is no evidence that the framers of the Constitution abhorred the idea of an
Acting Chief Justice in all cases.
8. The Court has erred in directing the JBC to resume the proceedings for the
nomination of the candidates to fill the vacancy to be created by the compulsory
retirement of Chief Justice Puno with a view to submitting the list of nominees for Lim
Chief Justice to President Arroyo on or before May 17, 2010. The Constitution
grants the Court only the power of supervision over the JBC; hence, the Court
74

1. There is no justiciable controversy that warrants the Court’s exercise of judicial 3. In ruling that Section 15, Article VII is in conflict with Section 4(1), Article VIII, the
review. Court has violated the principle of ut magis valeat quam pereat (which mandates
that the Constitution should be interpreted as a whole, such that any conflicting
2. The election ban under Section 15, Article VII applies to appointments to fill a provisions are to be harmonized as to fully give effect to all). There is no conflict
vacancy in the Court and to other appointments to the Judiciary. between the provisions; they complement each other.

3. The creation of the JBC does not justify the removal of the safeguard under 4. The form and structure of the Constitution’s titles, chapters, sections, and
Section 15 of Article VII against midnight appointments in the Judiciary. draftsmanship carry little weight in statutory construction. The clear and plain
language of Section 15, Article VII precludes interpretation.
Corvera
Tan, Jr.
1. The Court’s exclusion of appointments to the Judiciary from the Constitutional
ban on midnight appointments is based on an interpretation beyond the plain and 1. The factual antecedents do not present an actual case or controversy. The clash
unequivocal language of the Constitution. of legal rights and interests in the present case are merely anticipated. Even if it is
anticipated with certainty, no actual vacancy in the position of the Chief Justice has
yet occurred.
2. The intent of the ban on midnight appointments is to cover appointments in
both the Executive and Judicial Departments. The application of the principle of
verba legis (ordinary meaning) would have obviated dwelling on the organization 2. The ruling that Section 15, Article VII does not apply to a vacancy in the Court
and arrangement of the provisions of the Constitution. If there is any ambiguity in and the Judiciary runs in conflict with long standing principles and doctrines of
Section 15, Article VII, the intent behind the provision, which is to prevent political statutory construction. The provision admits only one exception, temporary
partisanship in all branches of the Government, should have controlled. appointments in the Executive Department. Thus, the Court should not distinguish,
because the law itself makes no distinction.
3. A plain reading is preferred to a contorted and strained interpretation based on
compartmentalization and physical arrangement, especially considering that the 3. Valenzuela was erroneously reversed. The framers of the Constitution clearly
Constitution must be interpreted as a whole. intended the ban on midnight appointments to cover the members of the Judiciary.
Hence, giving more weight to the opinion of Justice Regalado to reverse the en
banc decision in Valenzuela was unwarranted.
4. Resort to the deliberations or to the personal interpretation of the framers of
the Constitution should yield to the plain and unequivocal language of the
Constitution. 4. Section 15, Article VII is not incompatible with Section 4(1), Article VIII. The 90-
day mandate to fill any vacancy lasts until August 15, 2010, or a month and a half
after the end of the ban. The next President has roughly the same time of 45 days
5. There is no sufficient reason for reversing Valenzuela, a ruling that is reasonable
as the incumbent President (i.e., 44 days) within which to scrutinize and study the
and in accord with the Constitution.
qualifications of the next Chief Justice. Thus, the JBC has more than enough
opportunity to examine the nominees without haste and political
BAYAN, et al. uncertainty.1avvphi1

1. The Court erred in granting the petition in A.M. No. 10-2-5-SC, because the 5. When the constitutional ban is in place, the 90-day period under Section 4(1),
petition did not present a justiciable controversy. The issues it raised were not yet Article VIII is suspended.
ripe for adjudication, considering that the office of the Chief Justice was not yet
vacant and that the JBC itself has yet to decide whether or not to submit a list of
6. There is no basis to direct the JBC to submit the list of nominees on or before
nominees to the President.
May 17, 2010. The directive to the JBC sanctions a culpable violation of the
Constitution and constitutes an election offense.
2. The collective wisdom of Valenzuela Court is more important and compelling
than the opinion of Justice Regalado.
75

7. There is no pressing necessity for the appointment of a Chief Justice, because the 1. Under Section 15, Article VII, the only exemption from the ban on midnight
Court sits en banc, even when it acts as the sole judge of all contests relative to the appointments is the temporary appointment to an executive position. The
election, returns and qualifications of the President and Vice-President. Fourteen limitation is in keeping with the clear intent of the framers of the Constitution to
other Members of the Court can validly comprise the Presidential Electoral place a restriction on the power of the outgoing Chief Executive to make
Tribunal. appointments.

WTLOP 2. To exempt the appointment of the next Chief Justice from the ban on midnight
appointments makes the appointee beholden to the outgoing Chief Executive, and
1. The Court exceeded its jurisdiction in ordering the JBC to submit the list of compromises the independence of the Chief Justice by having the outgoing
nominees for Chief Justice to the President on or before May 17, 2010, and to President be continually influential.
continue its proceedings for the nomination of the candidates, because it granted a
relief not prayed for; imposed on the JBC a deadline not provided by law or the 3. The Court’s reversal of Valenzuela without stating the sufficient reason violates
Constitution; exercised control instead of mere supervision over the JBC; and the principle of stare decisis.
lacked sufficient votes to reverse Valenzuela.
Bello, et al.
2. In interpreting Section 15, Article VII, the Court has ignored the basic principle of
statutory construction to the effect that the literal meaning of the law must be 1. Section 15, Article VII does not distinguish as to the type of appointments an
applied when it is clear and unambiguous; and that we should not distinguish outgoing President is prohibited from making within the prescribed period. Plain
where the law does not distinguish. textual reading and the records of the Constitutional Commission support the view
that the ban on midnight appointments extends to judicial appointments.
3. There is no urgency to appoint the next Chief Justice, considering that the
Judiciary Act of 1948 already provides that the power and duties of the office 2. Supervision of the JBC by the Court involves oversight. The subordinate subject
devolve on the most senior Associate Justice in case of a vacancy in the office of to oversight must first act not in accord with prescribed rules before the act can be
the Chief Justice. redone to conform to the prescribed rules.

Ubano 3. The Court erred in granting the petition in A.M. No. 10-2-5-SC, because the
petition did not present a justiciable controversy.
1. The language of Section 15, Article VII, being clear and unequivocal, needs no
interpretation Pimentel

2. The Constitution must be construed in its entirety, not by resort to the 1. Any constitutional interpretative changes must be reasonable, rational, and
organization and arrangement of its provisions. conformable to the general intent of the Constitution as a limitation to the powers
of Government and as a bastion for the protection of the rights of the people. Thus,
3. The opinion of Justice Regalado is irrelevant, because Section 15, Article VII and in harmonizing seemingly conflicting provisions of the Constitution, the
the pertinent records of the Constitutional Commission are clear and unambiguous. interpretation should always be one that protects the citizenry from an ever
expanding grant of authority to its representatives.
4. The Court has erred in ordering the JBC to submit the list of nominees to the
President by May 17, 2010 at the latest, because no specific law requires the JBC to 2. The decision expands the constitutional powers of the President in a manner
submit the list of nominees even before the vacancy has occurred. totally repugnant to republican constitutional democracy, and is tantamount to a
judicial amendment of the Constitution without proper authority.
Boiser
Comments
76

The Office of the Solicitor General (OSG) and the JBC separately represent in their respective 2. The administrative matter he brought invoked the Court’s power of supervision
comments, thus: over the JBC as provided by Section 8(1), Article VIII of the Constitution, as
distinguished from the Court’s adjudicatory power under Section 1, Article VIII. In
OSG the former, the requisites for judicial review are not required, which was
why Valenzuela was docketed as an administrative matter. Considering that the JBC
itself has yet to take a position on when to submit the short list to the proper
1. The JBC may be compelled to submit to the President a short list of its nominees
appointing authority, it has effectively solicited the exercise by the Court of its
for the position of Chief Justice.
power of supervision over the JBC.

2. The incumbent President has the power to appoint the next Chief Justice.
3. To apply Section 15, Article VII to Section 4(1) and Section 9, Article VIII is to
amend the Constitution.
3. Section 15, Article VII does not apply to the Judiciary.
4. The portions of the deliberations of the Constitutional Commission quoted in the
4. The principles of constitutional construction favor the exemption of the Judiciary dissent of Justice Carpio Morales, as well as in some of the motions for
from the ban on midnight appointments.1awph!1 reconsideration do not refer to either Section 15, Article VII or Section 4(1), Article
VIII, but to Section 13, Article VII (on nepotism).
5. The Court has the duty to consider and resolve all issues raised by the parties as
well as other related matters. Ruling

JBC We deny the motions for reconsideration for lack of merit, for all the matters being thereby
raised and argued, not being new, have all been resolved by the decision of March 17, 2010.
1. The consolidated petitions should have been dismissed for prematurity, because
the JBC has not yet decided at the time the petitions were filed whether the Nonetheless, the Court opts to dwell on some matters only for the purpose of clarification
incumbent President has the power to appoint the new Chief Justice, and because and emphasis.
the JBC, having yet to interview the candidates, has not submitted a short list to
the President.
First: Most of the movants contend that the principle of stare decisis is controlling, and
accordingly insist that the Court has erred in disobeying or abandoning Valenzuela.1
2. The statement in the decision that there is a doubt on whether a JBC short list is
necessary for the President to appoint a Chief Justice should be struck down as
The contention has no basis.
bereft of constitutional and legal basis. The statement undermines the
independence of the JBC.
Stare decisis derives its name from the Latin maxim stare decisis et non quieta movere, i.e.,
to adhere to precedent and not to unsettle things that are settled. It simply means that a
3. The JBC will abide by the final decision of the Court, but in accord with its
principle underlying the decision in one case is deemed of imperative authority, controlling
constitutional mandate and its implementing rules and regulations.
the decisions of like cases in the same court and in lower courts within the same jurisdiction,
unless and until the decision in question is reversed or overruled by a court of competent
For his part, petitioner Estelito P. Mendoza (A.M. No. 10-2-5-SC) submits his comment even if authority. The decisions relied upon as precedents are commonly those of appellate courts,
the OSG and the JBC were the only ones the Court has required to do so. He states that the because the decisions of the trial courts may be appealed to higher courts and for that
motions for reconsideration were directed at the administrative matter he initiated and reason are probably not the best evidence of the rules of law laid down. 2
which the Court resolved. His comment asserts:
Judicial decisions assume the same authority as a statute itself and, until authoritatively
1. The grounds of the motions for reconsideration were already resolved by the abandoned, necessarily become, to the extent that they are applicable, the criteria that must
decision and the separate opinion. control the actuations, not only of those called upon to abide by them, but also of those
duty-bound to enforce obedience to them.3 In a hierarchical judicial system like ours, the
decisions of the higher courts bind the lower courts, but the courts of co-ordinate authority
77

do not bind each other. The one highest court does not bind itself, being invested with the Last: The movants take the majority to task for holding that Section 15, Article VII does not
innate authority to rule according to its best lights.4 apply to appointments in the Judiciary. They aver that the Court either ignored or refused to
apply many principles of statutory construction.
The Court, as the highest court of the land, may be guided but is not controlled by precedent.
Thus, the Court, especially with a new membership, is not obliged to follow blindly a The movants gravely err in their posture, and are themselves apparently contravening their
particular decision that it determines, after re-examination, to call for a rectification.5 The avowed reliance on the principles of statutory construction.
adherence to precedents is strict and rigid in a common-law setting like the United Kingdom,
where judges make law as binding as an Act of Parliament. 6 But ours is not a common-law For one, the movants, disregarding the absence from Section 15, Article VII of the express
system; hence, judicial precedents are not always strictly and rigidly followed. A judicial extension of the ban on appointments to the Judiciary, insist that the ban applied to the
pronouncement in an earlier decision may be followed as a precedent in a subsequent case Judiciary under the principle of verba legis. That is self-contradiction at its worst.
only when its reasoning and justification are relevant, and the court in the latter case accepts
such reasoning and justification to be applicable to the case. The application of the
Another instance is the movants’ unhesitating willingness to read into Section 4(1) and
precedent is for the sake of convenience and stability.
Section 9, both of Article VIII, the express applicability of the ban under Section 15, Article VII
during the period provided therein, despite the silence of said provisions thereon. Yet,
For the intervenors to insist that Valenzuela ought not to be disobeyed, or abandoned, or construction cannot supply the omission, for doing so would generally constitute an
reversed, and that its wisdom should guide, if not control, the Court in this case is, therefore, encroachment upon the field of the Constitutional Commission. Rather, Section 4(1) and
devoid of rationality and foundation. They seem to conveniently forget that the Constitution Section 9 should be left as they are, given that their meaning is clear and explicit, and no
itself recognizes the innate authority of the Court en banc to modify or reverse a doctrine or words can be interpolated in them.9Interpolation of words is unnecessary, because the law is
principle of law laid down in any decision rendered en banc or in division.7 more than likely to fail to express the legislative intent with the interpolation. In other words,
the addition of new words may alter the thought intended to be conveyed. And, even where
Second: Some intervenors are grossly misleading the public by their insistence that the the meaning of the law is clear and sensible, either with or without the omitted word or
Constitutional Commission extended to the Judiciary the ban on presidential appointments words, interpolation is improper, because the primary source of the legislative intent is in the
during the period stated in Section 15, Article VII. language of the law itself.10

The deliberations that the dissent of Justice Carpio Morales quoted from the records of the Thus, the decision of March 17, 2010 has fittingly observed:
Constitutional Commission did not concern either Section 15, Article VII or Section 4(1),
Article VIII, but only Section 13, Article VII, a provision on nepotism. The records of the Had the framers intended to extend the prohibition contained in Section 15, Article VII to the
Constitutional Commission show that Commissioner Hilario G. Davide, Jr. had proposed to appointment of Members of the Supreme Court, they could have explicitly done so. They
include judges and justices related to the President within the fourth civil degree of could not have ignored the meticulous ordering of the provisions. They would have easily
consanguinity or affinity among the persons whom the President might not appoint during and surely written the prohibition made explicit in Section 15, Article VII as being equally
his or her tenure. In the end, however, Commissioner Davide, Jr. withdrew the proposal to applicable to the appointment of Members of the Supreme Court in Article VIII itself, most
include the Judiciary in Section 13, Article VII "(t)o avoid any further complication,"8 such that likely in Section 4 (1), Article VIII. That such specification was not done only reveals that the
the final version of the second paragraph of Section 13, Article VII even completely omits any prohibition against the President or Acting President making appointments within two
reference to the Judiciary, to wit: months before the next presidential elections and up to the end of the President’s or Acting
President’s term does not refer to the Members of the Supreme Court.
Section 13. xxx
We cannot permit the meaning of the Constitution to be stretched to any unintended point
The spouse and relatives by consanguinity or affinity within the fourth civil degree of the in order to suit the purposes of any quarter.
President shall not during his tenure be appointed as Members of the Constitutional
Commissions, or the Office of the Ombudsman, or as Secretaries, Undersecretaries, chairmen Final Word
or heads of bureaus or offices, including government-owned or controlled corporations and
their subsidiaries.
It has been insinuated as part of the polemics attendant to the controversy we are resolving
that because all the Members of the present Court were appointed by the incumbent
78

President, a majority of them are now granting to her the authority to appoint the successor Carpio Morales; Associate Justice Presbitero J. Velasco, Jr.; and Associate Justice Antonio
of the retiring Chief Justice. Eduardo B. Nachura. However, the last two declined their nomination through letters dated
January 18, 2010 and January 25, 2010, respectively.
The insinuation is misguided and utterly unfair.
The OSG contends that the incumbent President may appoint the next Chief Justice, because
the prohibition under Section 15, Article VII of the Constitution does not apply to
The Members of the Court vote on the sole basis of their conscience and the merits of the
appointments in the Supreme Court. It argues that any vacancy in the Supreme Court must
issues. Any claim to the contrary proceeds from malice and condescension. Neither the
be filled within 90 days from its occurrence, pursuant to Section 4(1), Article VIII of the
outgoing President nor the present Members of the Court had arranged the current situation
Constitution; that had the framers intended the prohibition to apply to Supreme Court
to happen and to evolve as it has. None of the Members of the Court could have prevented
appointments, they could have easily expressly stated so in the Constitution, which explains
the Members composing the Court when she assumed the Presidency about a decade ago
why the prohibition found in Article VII (Executive Department) was not written in Article VIII
from retiring during her prolonged term and tenure, for their retirements were mandatory.
(Judicial Department); and that the framers also incorporated in Article VIII ample restrictions
Yet, she is now left with an imperative duty under the Constitution to fill up the vacancies
or limitations on the President’s power to appoint members of the Supreme Court to ensure
created by such inexorable retirements within 90 days from their occurrence. Her official
its independence from “political vicissitudes” and its “insulation from political pressures,”
duty she must comply with. So must we ours who are tasked by the Constitution to settle the
such as stringent qualifications for the positions, the establishment of the JBC, the specified
controversy.
period within which the President shall appoint a Supreme Court Justice.

ACCORDINGLY, the motions for reconsideration are denied with finality. A part of the question to be reviewed by the Court is whether the JBC properly initiated the
process, there being an insistence from some of the oppositors-intervenors that the JBC
SO ORDERED. could only do so once the vacancy has occurred (that is, after May 17, 2010). Another part is,
of course, whether the JBC may resume its process until the short list is prepared, in view of
1.) ARTURO M. DE CASTRO vs. JUDICIAL AND BAR COUNCIL (JBC) and PRESIDENT the provision of Section 4(1), Article VIII, which unqualifiedly requires the President to
GLORIA MACAPAGAL – ARROYO appoint one from the short list to fill the vacancy in the Supreme Court (be it the Chief Justice
or an Associate Justice) within 90 days from the occurrence of the vacancy.
G.R. No. 191002
ISSUE: Whether the incumbent President can appoint the successor of Chief Justice Puno
upon his retirement.
FACTS: The compulsory retirement of Chief Justice Reynato S. Puno by May 17, 2010 occurs
just days after the coming presidential elections on May 10, 2010.
HELD:
These cases trace their genesis to the controversy that has arisen from the forthcoming
Prohibition under Section 15, Article VII does not apply to appointments to fill a vacancy in
compulsory retirement of Chief Justice Puno on May 17, 2010, or seven days after the
the Supreme Court or to other appointments to the Judiciary.
presidential election. Under Section 4(1), in relation to Section 9, Article VIII, that “vacancy
shall be filled within ninety days from the occurrence thereof” from a “list of at least three
Two constitutional provisions are seemingly in conflict.
nominees prepared by the Judicial and Bar Council for every vacancy.” Also considering that
Section 15, Article VII (Executive Department) of the Constitution prohibits the President or
The first, Section 15, Article VII (Executive Department), provides: Section 15. Two months
Acting President from making appointments within two months immediately before the next
immediately before the next presidential elections and up to the end of his term, a President
presidential elections and up to the end of his term, except temporary appointments to
or Acting President shall not make appointments, except temporary appointments to
executive positions when continued vacancies therein will prejudice public service or
executive positions when continued vacancies therein will prejudice public service or
endanger public safety.
endanger public safety.
The JBC, in its en banc meeting of January 18, 2010, unanimously agreed to start the process
The other, Section 4 (1), Article VIII (Judicial Department), states: Section 4. (1). The Supreme
of filling up the position of Chief Justice.
Court shall be composed of a Chief Justice and fourteen Associate Justices. It may sit en banc
or in its discretion, in division of three, five, or seven Members. Any vacancy shall be filled
Conformably with its existing practice, the JBC “automatically considered” for the position of
within ninety days from the occurrence thereof.
Chief Justice the five most senior of the Associate Justices of the Court, namely: Associate
Justice Antonio T. Carpio; Associate Justice Renato C. Corona; Associate Justice Conchita
79

Had the framers intended to extend the prohibition contained in Section 15, Article VII to the I diverged fully from the Decision on the question of whether we should maintain or reverse
appointment of Members of the Supreme Court, they could have explicitly done so. They our ruling in Valenzuela. I maintained that it is still good law; no reason exists to touch the
could not have ignored the meticulous ordering of the provisions. They would have easily ruling as its main focus – the application of the election ban on the appointment of lower
and surely written the prohibition made explicit in Section 15, Article VII as being equally court judges under Article VIII, Section 9 of the Constitution – is not even an issue in the
applicable to the appointment of Members of the Supreme Court in Article VIII itself, most present case and was discussed only because the petitions incorrectly cited the ruling as
likely in Section 4 (1), Article VIII. That such specification was not done only reveals that the authority on the issue of the Chief Justice’s appointment. The Decision proposed to reverse
prohibition against the President or Acting President making appointments within two Valenzuela but only secured the support of five (5) votes, while my Separate Opinion in
months before the next presidential elections and up to the end of the President’s or Acting support of Valenzuela had four (4) votes. Thus, on the whole, the Decision did not prevail in
President’s term does not refer to the Members of the Supreme Court. reversing Valenzuela, as it only had five (5) votes in a field of 12 participating Members of the
Court. Valenzuela should therefore remain, as of the filing of this Opinion, as a valid
Section 14, Section 15, and Section 16 are obviously of the same character, in that they affect precedent.
the power of the President to appoint. The fact that Section 14 and Section 16 refer only to
appointments within the Executive Department renders conclusive that Section 15 also Acting on the present motions for reconsideration, I join the majority in denying the motions
applies only to the Executive Department. This conclusion is consistent with the rule that with respect to the Chief Justice issue, although we differ in some respects on the reasons
every part of the statute must be interpreted with reference to the context, i.e. that every supporting the denial. I dissent from the conclusion that the Valenzuela ruling should be
part must be considered together with the other parts, and kept subservient to the general reversed. My divergence from the majority’s reasons and conclusions compels me to write
intent of the whole enactment. It is absurd to assume that the framers deliberately situated this Concurring and Dissenting Opinion.
Section 15 between Section 14 and Section 16, if they intended Section 15 to cover all kinds
of presidential appointments. If that was their intention in respect of appointments to the
The Basic Requisites / Justiciability
Judiciary, the framers, if only to be clear, would have easily and surely inserted a similar
prohibition in Article VIII, most likely within Section 4 (1) thereof.
One marked difference between the Decision and my Separate Opinion is our approach on
the basic requisites/justiciability issues. The Decision apparently glossed over this aspect of
CONCURRING AND DISSENTING OPINION
the case, while I fully explained why the De Castro4 and Peralta5 petitions should be
dismissed outright. In my view, these petitions violated the most basic requirements of their
BRION, J.: chosen medium for review – a petition for certiorari and mandamus under Rule 65 of the
Rules of Court.
The Motions for Reconsideration
The petitions commonly failed to allege that the Judicial and Bar Council (JBC) performs
After sifting through the motions for reconsideration, I found that the arguments are largely judicial or quasi-judicial functions, an allegation that the petitions could not really make,
the same arguments that we have passed upon, in one form or another, in the various since the JBC does not really undertake these functions and, for this reason, cannot be the
petitions. Essentially, the issues boil down to justiciability; the conflict of constitutional subject of a petition for certiorari; hence, the petitions should be dismissed outright. They
provisions; the merits of the cited constitutional deliberations; and the status and effect of likewise failed to facially show any failure or refusal by the JBC to undertake a constitutional
the Valenzuela1 ruling. Even the motion for reconsideration of the Philippine Bar Association duty to justify the issuance of a writ of mandamus; they invoked judicial notice that we could
(G.R. No. 191420), whose petition I did not expressly touch upon in my Separate Opinion, not give because there was, and is, no JBC refusal to act.6 Thus, the mandamus aspects of
basically dwells on these issues. these petitions should have also been dismissed outright. The ponencia, unfortunately, failed
to fully discuss these legal infirmities.
I have addressed most, if not all, of these issues and I submit my Separate Opinion2 as my
basic response to the motions for reconsideration, supplemented by the discussions below. The motions for reconsideration lay major emphasis on the alleged lack of an actual case or
controversy that made the Chief Justice’s appointment a justiciable issue. They claim that the
As I reflected in my Separate Opinion (which three other Justices joined),3 the election Court cannot exercise the power of judicial review where there is no clash of legal rights and
appointment ban under Article VII, Section 15 of the Constitution should not apply to the interests or where this clash is merely anticipated, although the anticipated event shall come
appointment of Members of the Supreme Court whose period for appointment is separately with certainty.7
provided for under Article VIII, Section 4(1). I shared this conclusion with the Court’s Decision
although our reasons differed on some points. What the movants apparently forgot, focused as they were on their respective petitions, is
that the present case is not a single-petition case that rises or falls on the strength of that
80

single petition. The present case involves various petitions and interventions,8 not necessarily confronts.14 To me, this was "a point no less critical, from the point of view of supervision,
pulling towards the same direction, although each one is focused on the issue of whether the than the appointment of the two judges during the election ban period in Valenzuela."15
election appointment ban under Article VII, Section 15 of the Constitution should apply to
the appointment of the next Chief Justice of the Supreme Court. In making this conclusion, I pointed out in my Separate Opinion the unavoidable surrounding
realities evident from the confluence of events, namely: (1) an election to be held on May 10,
Among the petitions filed were those of Tolentino (G.R. No. 191342), Soriano (G.R. No. 2010; (2) the retirement of the Chief Justice on May 17, 2010; (3) the lapse of the terms of
191032) and Mendoza (A.M. No. 10-2-5-SC). The first two are petitions for prohibition under the elective officials from the President to the congressmen on June 30, 2010; (4) the delay
Section 2 of Rule 65 of the Rules of Court.9 While they commonly share this medium of before the Congress can organize and send its JBC representatives; and (5) the expiration of
review, they differ in their supporting reasons. The Mendoza petition, on the other hand, is the term of a non-elective JBC member in July 2010.16 All these – juxtaposed with the Court’s
totally different – it is a petition presented as an administrative matter (A.M.) in the manner supervision over the JBC, the latter’s need for guidance, and the existence of an actual
that the Valenzuela case was an A.M. case. As I pointed out in the Separate Opinion, the controversy on the same issues bedeviling the JBC – in my view, were sufficient to save the
Court uses the A.M. docket designation on matters relating to its exercise of supervision over Mendoza petition from being a mere request for opinion or a petition for declaratory relief
all courts and their personnel.10 I failed to note then, but I make of record now, that court that falls under the jurisdiction of the lower court. This recognition is beyond the level of
rules and regulations – the outputs in the Court’s rulemaking function – are also docketed as what this Court can do in handling a moot and academic case – usually, one that no longer
A.M. cases. presents a judiciable controversy but one that can still be ruled upon at the discretion of the
court when the constitutional issue is of paramount public interest and controlling principles
That an actual case or controversy involving a clash of rights and interests exists is are needed to guide the bench, the bar and the public.17
immediately and patently obvious in the Tolentino and Soriano petitions. At the time the
petitions were filed, the JBC had started its six-phase nomination process that would To be sure, this approach in recognizing when a petition is actionable is novel. An overriding
culminate in the submission of a list of nominees to the President of the Philippines for reason for this approach can be traced to the nature of the petition, as it rests on the Court’s
appointive action. Tolentino and Soriano – lawyers and citizens with interest in the strict supervisory authority and relates to the exercise of the Court’s administrative rather than its
observance of the election ban – sought to prohibit the JBC from continuing with this judicial functions (other than these two functions, the Court also has its rulemaking function
process. The JBC had started to act, without any prodding from the Court, because of its duty under Article VIII, Section 5(5) of the Constitution). Strictly speaking, the Mendoza petition
to start the nomination process but was hampered by the petitions filed and the legal calls for directions from the Court in the exercise of its power of supervision over the
questions raised that only the Supreme Court can settle with finality.11 Thus, a clash of JBC,18 not on the basis of the power of judicial review.19 In this sense, it does not need the
interests based on law existed between the petitioners and the JBC. To state the obvious, a actual clash of interests of the type that a judicial adjudication requires. All that must be
decision in favor of Tolentino or Soriano would result in a writ of prohibition that would shown is the active need for supervision to justify the Court’s intervention as supervising
direct the JBC not to proceed with the nomination process. authority.

The Mendoza petition cited the effect of a complete election ban on judicial appointments Under these circumstances, the Court’s recognition of the Mendoza petition was not an
(in view of the already high level of vacancies and the backlog of cases) as basis, and undue stretch of its constitutional powers. If the recognition is unusual at all, it is so only
submitted the question as an administrative matter that the Court, in the exercise of its because of its novelty; to my knowledge, this is the first time ever in Philippine jurisprudence
supervisory authority over the Judiciary and the JBC itself, should act upon. At the same time, that the supervisory authority of the Court over an attached agency has been highlighted in
it cited the "public discourse and controversy" now taking place because of the application of this manner. Novelty, per se, however, is not a ground for objection nor a mark of infirmity
the election ban on the appointment of the Chief Justice, pointing in this regard to the very for as long as the novel move is founded in law. In this case, as in the case of the writ of
same reasons mentioned in Valenzuela about the need to resolve the issue and avoid the amparo and habeas data that were then novel and avowedly activist in character, sufficient
recurrence of conflict between the Executive and the Judiciary, and the need to "avoid legal basis exists to actively invoke the Court’s supervisory authority – granted under the
polemics concerning the matter."12 Constitution, no less – as basis for action.

I recognized in the Separate Opinion that, unlike in Valenzuela where an outright defiance of To partly quote the wording of the Constitution, Article VIII, Section 8(1) and (5) provide that
the election ban took place, no such obvious triggering event transpired in the Mendoza "A Judicial and Bar Council is hereby created under the supervision of the Supreme Court… It
petition.13 Rather, the Mendoza petition looked to the supervisory power of the Court over may exercise such other functions and duties as the Supreme Court may assign to it."
judicial personnel and over the JBC as basis to secure a resolution of the election ban issue. Supervision, as a legal concept, more often than not, is defined in relation with the concept
The JBC, at that time, had indicated its intent to look up to the Court’s supervisory power and of control.20 In Social Justice Society v. Atienza,21 we defined "supervision" as follows:
role as the final interpreter of the Constitution to guide it in responding to the challenges it
81

[Supervision] means overseeing or the power or authority of an officer to see that I hasten to add that the JBC’s constitutional task is not as simple as some people think it to
subordinate officers perform their duties. If the latter fail or neglect to fulfill them, the be. The process of preparing and submitting a list of nominees is an arduous and time-
former may take such action or step as prescribed by law to make them perform their duties. consuming task that cannot be done overnight. It is a six-step process lined with standards
Control, on the other hand, means the power of an officer to alter or modify or nullify or set requiring the JBC to attract the best available candidates, to examine and investigate them,
aside what a subordinate officer ha[s] done in the performance of his duties and to to exhibit transparency in all its actions while ensuring that these actions conform to
substitute the judgment of the former for that of the latter. constitutional and statutory standards (such as the election ban on appointments), to submit
the required list of nominees on time, and to ensure as well that all these acts are politically
Under this definition, the Court cannot dictate on the JBC the results of its assigned task, i.e., neutral. On the time element, the JBC list for the Supreme Court has to be submitted on or
who to recommend or what standards to use to determine who to recommend. It cannot before the vacancy occurs given the 90-day deadline that the appointing President is given in
even direct the JBC on how and when to do its duty, but it can, under its power of making the appointment. The list will be submitted, not to the President as an outgoing
supervision, direct the JBC to "take such action or step as prescribed by law to make them President, nor to the election winner as an incoming President, but to the President of the
perform their duties," if the duties are not being performed because of JBC’s fault or Philippines whoever he or she may be. If the incumbent President does not act on the JBC list
inaction, or because of extraneous factors affecting performance. Note in this regard that, within the time left in her term, the same list shall be available to the new President for him
constitutionally, the Court can also assign the JBC other functions and duties – a power that to act upon. In all these, the Supreme Court bears the burden of overseeing that the JBC’s
suggests authority beyond what is purely supervisory. duty is done, unerringly and with utmost dispatch; the Court cannot undertake this
supervision in a manner consistent with the Constitution’s expectation from the JBC unless it
adopts a pro-active stance within the limits of its supervisory authority.
Where the JBC itself is at a loss on how to proceed in light of disputed constitutional
provisions that require interpretation,22 the Court is not legally out of line – as the final
authority on the interpretation of the Constitution and as the entity constitutionally-tasked The Disputed Provisions
to supervise the JBC – in exercising its oversight function by clarifying the interpretation of
the disputed constitutional provision to guide the JBC. In doing this, the Court is not simply The movants present their arguments on the main issue at several levels. Some argue that
rendering a general legal advisory; it is providing concrete and specific legal guidance to the the disputed constitutional provisions – Article VII, Section 15 and Article VIII, Section 4(1) –
JBC in the exercise of its supervisory authority, after the latter has asked for assistance in this are clear and speak for themselves on what the Constitution covers in banning appointments
regard. That the Court does this while concretely resolving actual controversies (the during the election period.23 One even posits that there is no conflict because both provisions
Tolentino and Soriano petitions) on the same issue immeasurably strengthens the intrinsic can be given effect without one detracting against the full effectiveness of the
correctness of the Court’s action. other,24 although the effect is to deny the sitting President the option to appoint in favor of a
deferment for the incoming President’s action. Still others, repeating their original
It may be asked: why does the Court have to recognize the Mendoza petition when it can arguments, appeal to the principles of interpretation and latin maxims to prove their point.25
resolve the conflict between Article VII, Section 15 and Article VIII, Section 4(1) through the
Tolentino and Soriano petitions? In my discussions in the Separate Opinion, I stated upfront my views on how the disputed
provisions interact with each other. Read singly and in isolation, they appear clear (this
The answer is fairly simple and can be read between the lines of the above explanation on reading applies the "plain meaning rule" that Tolentino advocates in his motion for
the relationship between the Court and the JBC. First, administrative is different from judicial reconsideration, as explained below). Arrayed side by side with each other and considered in
function and providing guidance to the JBC can only be appropriate in the discharge of the relation with the other provisions of the Constitution, particularly its structure and
Court’s administrative function. Second, the resolution of the Tolentino and Soriano petitions underlying intents, the conflict however becomes obvious and unavoidable.
will lead to rulings directly related to the underlying facts of these petitions, without clear
guidelines to the JBC on the proper parameters to observe vis-à-vis the constitutional dispute Section 15 on its face disallows any appointment in clear negative terms ("shall not make")
along the lines the JBC needs. In fact, concrete guidelines addressed to the JBC in the without specifying the appointments covered by the prohibition.26 From this literal and
resolution of the Tolentino/Soriano petitions may even lead to accusations that the Court’s isolated reading springs the argument that no exception is provided (except that found in
resolution is broader than is required by the facts of the petitions. The Mendoza petition, Section 15 itself) so that even the Judiciary is covered by the ban on appointments.
because it pertains directly to the performance of the JBC’s duty and the Court’s supervisory
authority, allows the issuance of precise guidelines that will enable the JBC to fully and On the other hand, Section 4(1) is likewise very clear and categorical in its terms: any vacancy
seasonably comply with its constitutional mandate. in the Court shall be filled within 90 days from its occurrence.27 In the way of Section 15,
Section 4(1) is also clear and categorical and provides no exception; the appointment refers
82

solely to the Members of the Supreme Court and does not mention any period that would A holistic reading of the Constitution – a must in constitutional interpretation – dictates as a
interrupt, hold or postpone the 90-day requirement. general rule that the tasks assigned to each department and their limitations should be given
full effect to fulfill the constitutional purposes under the check and balance principle, unless
From this perspective, the view that no conflict exists cannot be seriously made, unless with the Constitution itself expressly indicates its preference for one task, concern or standard
the mindset that one provision controls and the other should yield. Many of the petitions in over the others,32 or unless this Court, in its role as interpreter of the Constitution, has
fact advocate this kind of reading, some of them openly stating that the power of spoken on the appropriate interpretation that should be made.33
appointment should be reserved for the incoming President.28 The question, however, is
whether – from the viewpoint of strict law and devoid of the emotionalism and political In considering the interests of the Executive and the Judiciary, a holistic approach starts from
partisanship that permeate the present Philippine political environment – this kind of the premise that the constitutional scheme is to grant the President the power of
mindset can really be adopted in reading and applying the Constitution. appointment, subject to the limitation provided under Article VII, Section 15. At the same
time, the Judiciary is assured, without qualifications under Article VIII, Section 4(1), of the
In my view, this kind of mindset and the conclusion it inevitably leads to cannot be adopted; immediate appointment of Members of the Supreme Court, i.e., within 90 days from the
the provisions of the Constitution cannot be read in isolation from what the whole contains. occurrence of the vacancy. If both provisions would be allowed to take effect, as I believe
To be exact, the Constitution must be read and understood as a whole, reconciling and they should, the limitation on the appointment power of the President under Article VII,
harmonizing apparently conflicting provisions so that all of them can be given full force and Section 15 should itself be limited by the appointment of Members of the Court pursuant to
effect,29 unless the Constitution itself expressly states otherwise. 30 Article VIII, Section 4(1), so that the provision applicable to the Judiciary can be given full
effect without detriment to the President’s appointing authority. This harmonization will
result in restoring to the President the full authority to appoint Members of the Supreme
Not to be forgotten in reading and understanding the Constitution are the many established
Court pursuant to the combined operation of Article VII, Section 15 and Article VIII, Section
underlying constitutional principles that we have to observe and respect if we are to be true
4(1).
to the Constitution. These principles – among them the principles of checks and balances and
separation of powers – are not always expressly stated in the Constitution, but no one who
believes in and who has studied the Constitution can deny that they are there and deserve Viewed in this light, there is essentially no conflict, in terms of the authority to appoint,
utmost attention, respect, and even priority consideration. between the Executive and Judiciary; the President would effectively be allowed to exercise
the Executive’s traditional presidential power of appointment while respecting the Judiciary’s
own prerogative. In other words, the President retains full powers to appoint Members of
In establishing the structures of government, the ideal that the Constitution seeks to achieve
the Court during the election period, and the Judiciary is assured of a full membership within
is one of balance among the three great departments of government – the Executive, the
the time frame given.
Legislative and the Judiciary, with each department undertaking its constitutionally-assigned
task as a check against the exercise of power by the others, while all three departments
move forward in working for the progress of the nation. Thus, the Legislature makes the laws Interestingly, the objection to the full application of Article VIII, Section 4(1) comes, not from
and is supreme in this regard, in the way that the Executive is supreme in enforcing and the current President, but mainly from petitioners echoing the present presidential
administering the law, while the Judiciary interprets both the Constitution and the law. Any candidates, one of whom shall soon be the incoming President. They do not, of course, cite
provision in each of the Articles on these three departments31 that intrudes into the other reasons of power and the loss of the opportunity to appoint the Chief Justice; many of the
must be closely examined if the provision affects and upsets the desired balance. petitioners/intervenors oppose the full application of Article VIII, Section 4(1) based on the
need to maintain the integrity of the elections through the avoidance of a "midnight
appointment."
Under the division of powers, the President as Chief Executive is given the prerogative of
making appointments, subject only to the legal qualification standards, to the checks
provided by the Legislature’s Commission on Appointments (when applicable) and by the JBC This "integrity" reason is a given in a democracy and can hardly be opposed on the
for appointments in the Judiciary, and to the Constitution’s own limitations. Conflict comes in theoretical plane, as the integrity of the elections must indeed prevail in a true democracy.
when the Constitution laid down Article VII, Section 15 limiting the President’s appointing The statement, however, begs a lot of questions, among them the question of whether the
power during the election period. This limitation of power would have been all- appointment of a full Court under the terms of Article VIII, Section 4(1) will adversely affect
encompassing and would, thus, have extended to all government positions the President can or enhance the integrity of the elections.
fill, had the Constitution not inserted a provision, also on appointments, in the Article on the
Judiciary with respect to appointments to the Supreme Court. This conflict gives rise to the In my Separate Opinion, I concluded that the appointment of a Member of the Court even
questions: which provision should prevail, or should both be given effect? Or should both during the election period per se implies no adverse effect on the integrity of the election; a
provisions yield to a higher concern – the need to maintain the integrity of our elections?
83

full Court is ideal during this period in light of the Court’s unique role during elections. I Be it remembered that if any EDSA-type situation arises in the coming elections, it will be
maintain this view and fully concur in this regard with the majority. compounded by the lack of leaders because of the lapse of the President’s term by June 30,
2010; by a possible failure of succession if for some reason the election of the new leadership
During the election period, the court is not only the interpreter of the Constitution and the becomes problematic; and by the similar absence of congressional leadership because
election laws; other than the Commission on Elections and the lower courts to a limited Congress has not yet convened to organize itself.34 In this scenario, only the Judiciary of the
extent, the Court is likewise the highest impartial recourse available to decisively address any three great departments of government stands unaffected by the election and should at
problem or dispute arising from the election. It is the leader and the highest court in the least therefore be complete to enable it to discharge its constitutional role to its fullest
Judiciary, the only one of the three departments of government directly unaffected by the potential and capacity. To state the obvious, leaving the Judiciary without any permanent
election. The Court is likewise the entity entrusted by the Constitution, no less, with the leader in this scenario may immeasurably complicate the problem, as all three departments
gravest election-related responsibilities. In particular, it is the sole judge of all contests in the of government will then be leaderless.
election of the President and the Vice-President, with leadership and participation as well in
the election tribunals that directly address Senate and House of Representatives electoral To stress what I mentioned on this point in my Separate Opinion, the absence of a Chief
disputes. With this grant of responsibilities, the Constitution itself has spoken on the trust it Justice will make a lot of difference in the effectiveness of the Court as he or she heads the
reposes on the Court on election matters. This reposed trust, to my mind, renders academic Judiciary, sits as Chair of the JBC and of the Presidential Electoral Tribunal, presides over
any question of whether an appointment during the election period will adversely affect the impeachment proceedings, and provides the moral suasion and leadership that only the
integrity of the elections – it will not, as the maintenance of a full Court in fact contributes to permanent mantle of the Chief Justice can bestow. EDSA II is just one of the many lessons
the enforcement of the constitutional scheme to foster a free and orderly election. from the past when the weightiest of issues were tackled and promptly resolved by the
Court. Unseen by the general public in all these was the leadership that was there to ensure
In reading the motions for reconsideration against the backdrop of the partisan political that the Court would act as one, in the spirit of harmony and stability although divergent in
noise of the coming elections, one cannot avoid hearing echoes from some of the arguments their individual views, as the Justices individually make their contributions to the collegial
that the objection is related, more than anything else, to their lack of trust in an appointment result. To some, this leadership may only be symbolic, as the Court has fully functioned in the
to be made by the incumbent President who will soon be bowing out of office. They label the past even with an incomplete membership or under an Acting Chief Justice. But as I said
incumbent President’s act as a "midnight appointment" – a term that has acquired a before, an incomplete Court "is not a whole Supreme Court; it will only be a Court with 14
pejorative meaning in contemporary society. members who would act and vote on all matters before it." To fully recall what I have said on
this matter:
As I intimated in my Separate Opinion, the imputation of distrust can be made against any
appointing authority, whether outgoing or incoming. The incoming President himself will be The importance of the presence of one Member of the Court can and should never be
before this Court if an election contest arises; any President, past or future, would also underestimated, particularly on issues that may gravely affect the nation. Many a case has
naturally wish favorable outcomes in legal problems that the Court would resolve. These been won or lost on the basis of one vote. On an issue of the constitutionality of a law, treaty
possibilities and the potential for continuing influence in the Court, however, cannot be or statute, a tie vote – which is possible in a 14 member court – means that the
active considerations in resolving the election ban issue as they are, in their present form and constitutionality is upheld. This was our lesson in Isagani Cruz v. DENR Secretary.
presentation, all speculative. If past record is to be the measure, the record of past Chief
Justices and of this Court speaks for itself with respect to the Justices’ relationship with, and More than the vote, Court deliberation is the core of the decision-making process and one
deferral to, the appointing authority in their decisions. voice is less is not only a vote less but a contributed opinion, an observation, or a cautionary
word less for the Court. One voice can be a big difference if the missing voice is that of the
What should not be forgotten in examining the records of the Court, from the prism of Chief Justice.
problems an electoral exercise may bring, is the Court’s unique and proven capacity to
intervene and diffuse situations that are potentially explosive for the nation. EDSA II Without meaning to demean the capability of an Acting Chief Justice, the ascendancy in the
particularly comes to mind in this regard (although it was an event that was not rooted in Court of a permanent sitting Chief Justice cannot be equaled. He is the first among equals – a
election problems) as it is a perfect example of the potential for damage to the nation that primus inter pares – who sets the tone for the Court and the Judiciary, and who is looked up
the Court can address and has addressed. When acting in this role, a vacancy in the Court is to on all matters, whether administrative or judicial. To the world outside the Judiciary, he is
not only a vote less, but a significant contribution less in the Court’s deliberations and the personification of the Court and the whole Judiciary. And this is not surprising since, as
capacity for action, especially if the missing voice is the voice of the Chief Justice. Chief Justice, he not only chairs the Court en banc, but chairs as well the Presidential
Electoral Tribunal that sits in judgment over election disputes affecting the President and the
Vice-President. Outside of his immediate Court duties, he sits as Chair of the Judicial and Bar
84

Council, the Philippine Judicial Academy and, by constitutional command, presides over the If Valenzuela did prominently figure at all in the present case, the prominence can be
impeachment of the President. To be sure, the Acting Chief Justice may be the ablest, but he attributed to the petitioners’ mistaken reading that this case is primary authority for the
is not the Chief Justice without the mantle and permanent title of the Office, and even his dictum that Article VII, Section 15 completely bans all appointments to the Judiciary,
presence as Acting Chief Justice leaves the Court with one member less. Sadly, this member including appointments to the Supreme Court, during the election period up to the end of
is the Chief Justice; even with an Acting Chief Justice, the Judiciary and the Court remains the incumbent President’s term.
headless. 35
In reality, this mistaken reading is an obiter dictum in Valenzuela, and hence, cannot be cited
Given these views, I see no point in re-discussing the finer points of technical interpretation for its primary precedential value. This legal situation still holds true as Valenzuela was not
and their supporting latin maxims that I have addressed in my Separate Opinion and now feel doctrinally reversed as its proposed reversal was supported only by five (5) out of the 12
need no further elaboration; maxims can be found to serve a pleader’s every need and in any participating Members of the Court. In other words, this ruling on how Article VII, Section 15
case are the last interpretative tools in constitutional interpretation. Nor do I see any point in is to be interpreted in relation with Article VIII, Section 9, should continue to stand unless
discussing arguments based on the intent of the framers of the Constitution now cited by the otherwise expressly reversed by this Court.
parties in the contexts that would serve their own ends. As may be evident in these
discussions, other than the texts of the disputed provisions, I prefer to examine their But separately from the mistaken use of an obiter ruling as primary authority, I believe that I
purposes and the consequences of their application, understood within the context of should sound the alarm bell about the Valenzuela ruling in light of a recent vacancy in the
democratic values. Past precedents are equally invaluable for the lead, order, and stability position of Presiding Justice of the Sandiganbayan resulting from Presiding Justice Norberto
they contribute, but only if they are in point, certain, and still alive to current realities, while Geraldez’s death soon after we issued the decision in the present case. Reversing the
the history of provisions, including the intents behind them, are primarily important to Valenzuela ruling now, in the absence of a properly filed case addressing an appointment at
ascertain the purposes the provisions serve. this time to the Sandiganbayan or to any other vacancy in the lower courts, will be an
irregular ruling of the first magnitude by this Court, as it will effectively be a shortcut that lifts
From these perspectives and without denigrating the framers’ historical contributions, I say the election ban on appointments to the lower courts without the benefit of a case whose
that it is the Constitution that now primarily speaks to us in this case and what we hear are facts and arguments would directly confront the continued validity of the Valenzuela ruling.
its direct words, not merely the recorded isolated debates reflecting the personal intents of This is especially so after we have placed the Court on notice that a reversal of Valenzuela is
the constitutional commissioners as cited by the parties to fit their respective theories. The uncalled for because its ruling is not the litigated issue in this case.
voice speaking the words of the Constitution is our best guide, as these words will
unalterably be there for us to read in the context of their purposes and the nation’s needs In any case, let me repeat what I stressed in my Separate Opinion about Valenzuela which
and circumstances. This Concurring and Dissenting Opinion hears and listens to that voice. rests on the reasoning that the evils Section 15 seeks to remedy – vote buying, midnight
appointments and partisan reasons to influence the elections – exist, thus justifying an
The Valenzuela Decision election appointment ban. In particular, the "midnight appointment" justification, while fully
applicable to the more numerous vacancies at the lower echelons of the Judiciary (with an
The ponencia’s ruling reversing Valenzuela, in my view, is out of place in the present case, alleged current lower court vacancy level of 537 or a 24.5% vacancy rate), should not apply to
since at issue here is the appointment of the Chief Justice during the period of the election the Supreme Court which has only a total of 15 positions that are not even vacated at the
ban, not the appointment of lower court judges that Valenzuela resolved. To be perfectly same time. The most number of vacancies for any one year occurred only last year (2009)
clear, the conflict in the constitutional provisions is not confined to Article VII, Section 15 and when seven (7) positions were vacated by retirement, but this vacancy rate is not expected
Article VIII, Section 4(1) with respect to the appointment of Members of the Supreme Court; to be replicated at any time within the next decade. Thus "midnight appointments" to the
even before the Valenzuela ruling, the conflict already existed between Article VII, Section 15 extent that they were understood in Aytona36 will not occur in the vacancies of this Court as
and Article VIII, Section 9 – the provision on the appointment of the justices and judges of nominations to its vacancies are all processed through the JBC under the public’s close
courts lower than the Supreme Court. After this Court’s ruling in Valenzuela, no amount of scrutiny. As already discussed above, the institutional integrity of the Court is hardly an issue.
hairsplitting can result in the conclusion that Article VII, Section 15 applied the election ban If at all, only objections personal to the individual Members of the Court or against the
over the whole Judiciary, including the Supreme Court, as the facts and the fallo of individual applicants can be made, but these are matters addressed in the first place by the
Valenzuela plainly spoke of the objectionable appointment of two Regional Trial Court JBC before nominees are submitted. There, too, are specific reasons, likewise discussed
judges. To reiterate, Valenzuela only resolved the conflict between Article VII, Section 15 and above, explaining why the election ban should not apply to the Supreme Court. These
appointments to the Judiciary under Article VIII, Section 9. exempting reasons, of course, have yet to be shown to apply to the lower courts. Thus, on
the whole, the reasons justifying the election ban in Valenzuela still obtain in so far as the
lower courts are concerned, and have yet to be proven otherwise in a properly filed case.
85

Until then, Valenzuela, except to the extent that it mentioned Section 4(1), should remain an concern either Sec. 15, Art. VII or Sec. 4(1), Art. VIII, but only Sec. 13, Art. VII, a provision on
authoritative ruling of this Court. nepotism.

CONCLUSION Election ban on appointments does not extend to the Supreme Court. The Court upheld its
March 17, 2010 decision ruling that the prohibition under Art. VII, Sec. 15 of the Constitution
In light of these considerations, a writ of prohibition cannot issue to prevent the JBC from against presidential appointments immediately before the next presidential elections and up
performing its principal function, under the Constitution, of recommending nominees for the to the end of the term of the outgoing president does not apply to vacancies in the Supreme
position of Chief Justice. Thus, I vote to deny with finality the Tolentino and Soriano motions Court.
for reconsideration.
Facts:
The other motions for reconsideration in so far as they challenge the conclusion that the
President can appoint the Chief Justice even during the election period are likewise denied The compulsory retirement of Chief Justice Reynato S. Puno by May 17, 2010 occurs just days
with finality for lack of merit, but are granted in so far as they support the continued validity after thecoming presidential elections on May 10, 2010. Even before the event actually
of the ruling of this Court in In Re: Valenzuela and Vallarta, A.M. No. 98-5-01-SC, November
happens, it is giving rise tomany legal dilemmas.This dilemma is rooted in consideration of
9, 1998.
Section 15, Art VII of the Constitution prohibiting the President or Acting President from
making appointments within two months immediately before the next presidentialelection
My opinion on the Mendoza petition stands.
and up to the end of his term, except when temporary appointments to executive positions
whencontinued vacancies will prejudice public service or endanger public safety.However,
ARTURO D. BRION
Associate Justice Section 4 (1), Art VIII of the Constitution also provides that any vacancy in the Supreme Court
shallbe filled within 90 days from occurrence. The question leads to who should appoint the
Facts: next Chief Justice and may the JBC resume the process of screening candidates should the
incumbent president not prohibited to do so. May a mandamus lie to compel the submission
This is a Motion for Reconsideration on the March 17, 2010 decision of the Court. The said of JBC¶s nominees to the president?
decision directs the Judicial and Bar Council to resume its proceedings for the nomination of
candidates to fill the vacancy created by the compulsory retirement of Chief Justice Reynato This issue at hand truly is impressed with transcendental importance to the Nation. A lot of
S. Puno by May 17, 2010, and to prepare the short list of nominees and submit it to the petitions were received by the court from a mandamus to prohibitions. We limit our
incumbent President. Movants argue that the disputed constitutional provision, Art. VII, Sec. discussion with GR191002 for brevity.
15 and Art. VIII, Sec. 4(1), clearly intended the ban on midnight appointments to cover the
Issues: Whether or not the case at bar is an actual controversy.Whether or not the
members of the Judiciary, and they contended that the principle of stare decisis is
petitioners have legal standing to file said petition.
controlling, and insisted that the Court erred in disobeying or abandoning the Valenzuela
ruling. Ratio Decidendi:

ISSUE (Section 4): Did the Constitutional Commission extend to the Judiciary the ban on The court held the case being premature because the Judicial and Bar Council has until May
presidential appointments during the period stated in Sec. 15, Article VII? 17, 2010 at theleast within which to submit the list of nominees to the President to fill the
vacancy created by the compulsoryretirement of Chief Justice Puno.The petitioner here
RULING:
asserts his right as citizen filing the petition on behalf of the public who are directly
The Constitutional Commission did not extend to the Judiciary the ban on presidential affectedby the issue of the appointment. The question raised before the court is in fact of
appointments during the period stated in Sec. 15, Art. VII. The deliberations that the dissent transcendental importance.The court dispels all doubt to remove any obstacle or obstruction
of Justice Carpio Morales quoted from the records of the Constitutional Commission did not to the resolution of the essential issuesquarely presented. Standing is a peculiar concept to
constitutional law because in some cases, suits are notbrought by parties who have been
personally injured by the operation of law or any other government act butby concerned
86

citizens, taxpayers or voters who actually sue in the public interest.The court dismissed the appointments, they could have easily expressly stated so in the Constitution, which explains
petitions for certiorari and mandamus in GR 191002 and GR 191149 and the petitionfor why the prohibition found in Article VII (Executive Department) was not written in Article VIII
mandamus in GR no. 191057 for being premature; dismissal of the petitions for prohibition in (Judicial Department); and that the framers also incorporated in Article VIII ample restrictions
GR 191032and GR 191342 for lack of merit; and grants the in AM No. 10-2-5-SC and or limitations on the President’s power to appoint members of the Supreme Court to ensure
accordingly directs the JBC to:resume proceedings for the nomination of candidates, prepare its independence from “political vicissitudes” and its “insulation from political pressures,”
short list of nominees for the said position,submit to the incumbent President the short list such as stringent qualifications for the positions, the establishment of the JBC, the specified
of nominees, and to continue proceedings for the nomination of candidates to fill other period within which the President shall appoint a Supreme Court Justice.
vacancies in the Judiciary and submit to the President the short list of
nomineescorresponding thereto in accordance with this decision. A part of the question to be reviewed by the Court is whether the JBC properly initiated the
process, there being an insistence from some of the oppositors-intervenors that the JBC
FACTS: could only do so once the vacancy has occurred (that is, after May 17, 2010). Another part is,
of course, whether the JBC may resume its process until the short list is prepared, in view of
The compulsory retirement of Chief Justice Reynato S. Puno by May 17, 2010 occurs just days the provision of Section 4(1), Article VIII, which unqualifiedly requires the President to
after the coming presidential elections on May 10, 2010. appoint one from the short list to fill the vacancy in the Supreme Court (be it the Chief Justice
or an Associate Justice) within 90 days from the occurrence of the vacancy.
These cases trace their genesis to the controversy that has arisen from the forthcoming
compulsory retirement of Chief Justice Puno on May 17, 2010, or seven days after the ISSUE: Whether the incumbent President can appoint the successor of Chief Justice Puno
presidential election. Under Section 4(1), in relation to Section 9, Article VIII, that “vacancy upon his retirement.
shall be filled within ninety days from the occurrence thereof” from a “list of at least three
nominees prepared by the Judicial and Bar Council for every vacancy.” Also considering that HELD:
Section 15, Article VII (Executive Department) of the Constitution prohibits the President or
Acting President from making appointments within two months immediately before the next Prohibition under Section 15, Article VII does not apply to appointments to fill a vacancy in
presidential elections and up to the end of his term, except temporary appointments to the Supreme Court or to other appointments to the Judiciary.
executive positions when continued vacancies therein will prejudice public service or
Two constitutional provisions are seemingly in conflict.
endanger public safety.
The first, Section 15, Article VII (Executive Department), provides: Section 15. Two months
The JBC, in its en banc meeting of January 18, 2010, unanimously agreed to start the process
immediately before the next presidential elections and up to the end of his term, a President
of filling up the position of Chief Justice.
or Acting President shall not make appointments, except temporary appointments to
Conformably with its existing practice, the JBC “automatically considered” for the position of executive positions when continued vacancies therein will prejudice public service or
Chief Justice the five most senior of the Associate Justices of the Court, namely: Associate endanger public safety.
Justice Antonio T. Carpio; Associate Justice Renato C. Corona; Associate Justice Conchita
The other, Section 4 (1), Article VIII (Judicial Department), states: Section 4. (1). The Supreme
Carpio Morales; Associate Justice Presbitero J. Velasco, Jr.; and Associate Justice Antonio
Court shall be composed of a Chief Justice and fourteen Associate Justices. It may sit en banc
Eduardo B. Nachura. However, the last two declined their nomination through letters dated
or in its discretion, in division of three, five, or seven Members. Any vacancy shall be filled
January 18, 2010 and January 25, 2010, respectively.
within ninety days from the occurrence thereof.
The OSG contends that the incumbent President may appoint the next Chief Justice, because
Had the framers intended to extend the prohibition contained in Section 15, Article VII to the
the prohibition under Section 15, Article VII of the Constitution does not apply to
appointment of Members of the Supreme Court, they could have explicitly done so. They
appointments in the Supreme Court. It argues that any vacancy in the Supreme Court must
could not have ignored the meticulous ordering of the provisions. They would have easily
be filled within 90 days from its occurrence, pursuant to Section 4(1), Article VIII of the
and surely written the prohibition made explicit in Section 15, Article VII as being equally
Constitution; that had the framers intended the prohibition to apply to Supreme Court
applicable to the appointment of Members of the Supreme Court in Article VIII itself, most
87

likely in Section 4 (1), Article VIII. That such specification was not done only reveals that the APPOINTMENTS, EXCEPT TEMPORARY APPOINTMENTS TO EXECUTIVE POSITIONS WHEN
prohibition against the President or Acting President making appointments within two CONTINUED VACANCIES THEREIN WILL PREJUDICE PUBLIC SERVICE OR ENDANGER PUBLIC
months before the next presidential elections and up to the end of the President’s or Acting SAFETY.
President’s term does not refer to the Members of the Supreme Court.
IN RELATION TO ART. VIII, UNDER JUDICIAL DEPARTMENT,
Had the framers intended to extend the prohibition contained in Section 15, Article VII to the
appointment of Members of the Supreme Court, they could have explicitly done so. They ART. VIII, SEC. 9. THE MEMBERS OF THE SUPREME COURT AND JUDGES OF THE LOWER
could not have ignored the meticulous ordering of the provisions. They would have easily COURTS SHALL BE APPOINTED BY THE PRESIDENT FROM A LIST OF AT LEAST THREE
and surely written the prohibition made explicit in Section 15, Article VII as being equally NOMINEES PREPARED BY THE JUDICIAL AND BAR COUNCIL FOR EVERY VACANCY. SUCH
applicable to the appointment of Members of the Supreme Court in Article VIII itself, most APPOINTMENTS NEED NO CONFIRMATION.
likely in Section 4 (1), Article VIII. That such specification was not done only reveals that the
FOR THE LOWER COURTS, THE PRESIDENT SHALL ISSUE THE APPOINTMENTS WITHIN NINETY
prohibition against the President or Acting President making appointments within two
DAYS FROM THE SUBMISSION OF THE LIST.
months before the next presidential elections and up to the end of the President’s or Acting
President’s term does not refer to the Members of the Supreme Court.
ISSUE: Whether the prohibition against presidential appointments under Art. VII, Sec. 15
(Midnight Appointment Ban), does not extend to appointments in the Judiciary.
Section 14, Section 15, and Section 16 are obviously of the same character, in that they affect
the power of the President to appoint. The fact that Section 14 and Section 16 refer only to
HELD:
appointments within the Executive Department renders conclusive that Section 15 also
applies only to the Executive Department. This conclusion is consistent with the rule that Prohibition under Art. VII, Sec.15 does not apply to appointments to fill a vacancy in the
every part of the statute must be interpreted with reference to the context, i.e. that every Supreme Court or to other appointments to the Judiciary.
part must be considered together with the other parts, and kept subservient to the general
intent of the whole enactment. It is absurd to assume that the framers deliberately situated As can be seen, Article VII is devoted to the Executive Department, and, among others, it lists
Section 15 between Section 14 and Section 16, if they intended Section 15 to cover all kinds the powers vested by the Constitution in the President. The presidential power of
of presidential appointments. If that was their intention in respect of appointments to the appointment is dealt with in Sections 14, 15 and 16 of the Article. Article VIII is dedicated to
Judiciary, the framers, if only to be clear, would have easily and surely inserted a similar the Judicial Department and defines the duties and qualifications of Members of the
prohibition in Article VIII, most likely within Section 4 (1) thereof. Supreme Court, among others. Sec. 4(1) and Sec. 9 of this Article are the provisions
specifically providing for the appointment of Supreme Court Justices. In particular, Sec. 9
FACTS: states that the appointment of Supreme Court Justices can only be made by the President
upon the submission of a list of at least three nominees by the JBC; Sec. 4(1) of the Article
This is a consolidated case regarding the appointment of President Gloria Macapagal-Arroyo
mandates the President to fill the vacancy within 90 days from the occurrence of the
to Associate Justice Renato Corona as Chief Justice of the Supreme Court.
vacancy.
All the petitions to the Court pose as the principal legal question whether the incumbent
Had the framers intended to extend the prohibition contained in Art. VII, Sec. 15 to the
President can appoint the successor of Chief Justice Puno upon his retirement. The question
appointment of Members of the Supreme Court, they could have explicitly done so. They
is undoubtedly impressed with transcendental importance to the nation because the
could not have ignored the meticulous ordering of the provisions. That such specification was
appointment of the Chief Justice is any President’s most important appointment. The
not done only reveals that the prohibition against the President or Acting President making
conflicting provisions are Art. VII, Sec. 15 and Art. VIII, Sec. 9.
appointments within two months before the next presidential elections and up to the end of
the President’s or Acting President’s term does not refer to the Members of the Supreme
CONSIDERING, ART. VII, UNDER EXECUTIVE DEPARTMENT,
Court.
ART. VII, SEC. 15. TWO MONTHS IMMEDIATELY BEFORE THE NEXT PRESIDENTIAL ELECTIONS
AND UP TO THE END OF HIS TERM, A PRESIDENT OR ACTING PRESIDENT SHALL NOT MAKE
88

Taken into consideration also that the appointment of the next Chief Justice by the
incumbent President is preferable to having the Associate Justice who is first in precedence
take over. Under the Constitution, the heads of the Legislative and Executive Departments
are popularly elected, and whoever are elected and proclaimed at once become the leaders
of their respective Departments. However, the lack of any appointed occupant of the office
of Chief Justice harms the independence of the Judiciary, because the Chief Justice is the
head of the entire Judiciary. The Chief Justice performs functions absolutely significant to the
life of the nation. With the entire Supreme Court being the Presidential Electoral Tribunal,
the Chief Justice is the Chairman of the Tribunal. There being no obstacle to the appointment
of the next Chief Justice, aside from its being mandatory for the incumbent President to
make within the 90-day period from May 17, 2010, there is no justification to insist that the
successor of Chief Justice Puno be appointed by the next President.
89

EN BANC composition of the Sandiganbayan from nine to fifteen Justices who would sit in five divisions
of three members each. Republic Act No. 10660,7 recently enacted on April 16, 2015, created
G.R. No. 224302, November 29, 2016 two more divisions of the Sandiganbayan with three Justices each, thereby resulting in six
vacant positions.
HON. PHILIP A. AGUINALDO, HON. REYNALDO A. ALHAMBRA, HON. DANILO S. CRUZ, HON.
On July 20, 2015, the Judicial and Bar Council (JBC) published in the Philippine Star and
BENJAMIN T. POZON, HON. SALVADOR V. TIMBANG, JR., AND THE INTEGRATED BAR OF
Philippine Daily Inquirer and posted on the JBC website an announcement calling for
THE PHILIPPINES (IBP), Petitioners, v. HIS EXCELLENCY PRESIDENT BENIGNO SIMEON C.
applications or recommendations for the six newly created positions of Associate Justice of
AQUINO III, HON. EXECUTIVE SECRETARY PAQUITO N. OCHOA, HON. MICHAEL FREDERICK
the Sandiganbayan.8 After screening and selection of applicants, the JBC submitted to
L. MUSNGI, HON. MA. GERALDINE FAITH A. ECONG, HON. DANILO S. SANDOVAL, HON.
President Aquino six shortlists contained in six separate letters, all dated October 26, 2015,
WILHELMINA B. JORGE-WAGAN, HON. ROSANA FE ROMERO-MAGLAYA, HON. MERIANTHE
which read:
PACITA M. ZURAEK, HON. ELMO M. ALAMEDA, AND HON. VICTORIA C. FERNANDEZ-
BERNARDO, Respondent.
1) For the 16th Sandiganbayan Associate Justice:

DECISION Your Excellency:

LEONARDO-DE CASTRO, J.: chanRoblesvirtualLawlibraryPursuant to Article VIII, Section 9 of the Constitution, the Judicial
and Bar Council (JBC) has the honor to submit the following nominations for the vacancy for
Before this Court is a Petition for Quo Warranto under Rule 66 and Certiorari and Prohibition the SIXTEENTH ASSOCIATE JUSTICE of the SANDIGANBAYAN, with their respective votes:
under Rule 65 with Application for Issuance of Injunctive Writs1 filed by petitioners Judge
Philip A. Aguinaldo (Aguinaldo) of the Regional Trial Court (RTC), Muntinlupa City, Branch 1. AGUINALDO, PHILIP A. - 5 votes �
207; Judge Reynaldo A. Alhambra (Alhambra) of RTC, Manila, Branch 53; Judge Danilo S. Cruz
(D. Cruz) of RTC, Pasig City, Branch 152; Judge Benjamin T. Pozon (Pozon) of RTC, Makati City, 2. ALHAMBRA, REYNALDO A. - 5 votes �
Branch 139; Judge Salvador V. Timbang, Jr. (Timbang) of RTC, Las Pi�as City, Branch 253; and
the Integrated Bar of the Philippines (IBP), against respondents former President Benigno 3. CRUZ, DANILO S. - 5 votes �
Simeon C. Aquino III (Aquino), Executive Secretary Paquito N. Ochoa (Ochoa), Sandiganbayan
Associate Justice Michael Frederick L. Musngi (Musngi), Sandiganbayan Associate Justice Ma. 4. POZON, BENJAMIN T. - 5 votes �
Geraldine Faith A. Econg (Econg), Atty. Danilo S. Sandoval (Sandoval), Atty. Wilhelmina B.
Jorge-Wagan (Jorge-Wagan), Atty. Rosana Fe Romero-Maglaya (Romero� Maglaya), Atty. 5. SANDOVAL, DANILO S. - 5 votes �
Merianthe Pacita M. Zuraek (Zuraek), Atty. Elmo M. Alameda (Alameda), and Atty. Victoria C.
Fernandez-Bernardo (Fernandez�-Bernardo). The Petition assails President Aquino's 6. TIMBANG, SALVADOR JR. - 5 votes9 �
appointment of respondents Musngi and Econg as Associate Justices of the Sandiganbayan.

I 2) For the 17th Sandiganbayan Associate Justice:


FACTUAL ANTECEDENTS
Your Excellency:
On June 11, 1978, then President Ferdinand E. Marcos (Marcos) issued Presidential Decree
No. 1486, creating a special court called the Sandiganbayan, composed of a Presiding Judge to Article VIII, Section 9 of the Constitution, the Judicial and Bar Council (JBC) has the honor
and eight Associate Judges to be appointed by the President, which shall have jurisdiction to submit the following nominations for the vacancy for the SEVENTEENTH ASSOCIATE
over criminal and civil cases involving graft and corrupt practices and such other offenses JUSTICE of the SANDIGANBAYAN, with their respective votes:
committed by public officers and employees, including those in government� owned or
controlled corporations.3 A few months later, on December 10, 1978, President Marcos also
issued Presidential Decree No. 1606,4 which elevated the rank of the members of the
1. CORPUS-MA�ALAC, MARYANN E. - 6 votes �
Sandiganbayan from Judges to Justices, co-equal in rank with the Justices of the Court of
Appeals; and provided that the Sandiganbayan shall sit in three divisions of three Justices
each.5Republic Act No. 79756 was approved into law on March 30, 1995 and it increased the
2. MENDOZA-ARCEGA, MARIA THERESA V. - 6 votes �
90

3. QUIMBO, RODOLFO NOEL S. - 6 votes � 5. RAMOS, RENAN E. - 5 votes

4. DIZON, MA. ANTONIA EDITA CLARIDADES - 5 votes � 6. ROXAS, RUBEN REYNALDO G. - 5 votes12

5. SORIANO, ANDRES BARTOLOME - 5 votes10 �


5) For the 20th Sandiganbayan Associate Justice:

3) For the 18th Sandiganbayan Associate Justice: Your Excellency:

Your Excellency: Pursuant to Article VIII, Section 9 of the Constitution, the Judicial and Bar Council (JBC) has
the honor to submit the following nominations for the vacancy for the TWENTIETH
Pursuant to Article VIII, Section 9 of the Constitution, the Judicial and Bar Council (JBC) has ASSOCIATE JUSTICE of the SANDIGANBAYAN, with their respective votes.
the honor to submit the following nominations for the vacancy for the EIGHTEENTH
ASSOCIATE JUSTICE of the SANDIGANBAYAN, with their respective votes:
1. MIRANDA, KARL B. - 6 votes
chanRoblesvirtualLawlibrary
2. ATAL-PA�O, PERPETUA - 5 votes
1. BAGUIO, CELSO O. - 5 votes �
3. BUNYI-MEDINA, THELMA - 5 votes
2. DE GUZMAN-ALVAREZ, MA. TERESA E. - 5 votes �
4. CORTEZ, LUISITO G. - 5 votes
3. FERNANDEZ, BERNELITO R. - 5 votes �
5. FIEL-MACARAIG, GERALDINE C. - 5 votes
4. PANGANIBAN, ELVIRA DE CASTRO - 5 votes �
6. QUIMPO-SALE, ANGELENE MARY W. - 5 votes
5. SAGUN, FERNANDO JR. T. - 5 votes �
7. JACINTO, BAYANI H. - 4 votes13
6. TRESPESES, ZALDY V. -5 votes11 �

6) For the 21st Sandiganbayan Associate Justice:


4) For the 19th Sandiganbayan Associate Justice:
Your Excellency:
Your Excellency:
Pursuant to Article VIII, Section 9 of the Constitution, the Judicial and Bar Council (JBC) has
Pursuant to Article VIII, Section 9 of the Constitution, the Judicial and Bar Council (JBC) has the honor to submit the following nominations for the vacancy for the TWENTY-FIRST
the honor to submit the following nominations for the vacancy for the NINETEENTH ASSOCIATE JUSTICE of the SANDIGANBAYAN, with their respective votes:
ASSOCIATE JUSTICE of the SANDIGANBAYAN, with their respective votes:

1. JORGE-WAGAN, WILHELMINA B. - 6 votes


1. GUANZON, FRANCES V. - 6 votes
2. ECONG, GERALDINE FAITH A. - 5 votes
2. MACARAIG-GUILLEN, MARISSA - 6 votes
3. ROMERO-MAGLAYA, ROSANNA FE - 5 votes
3. CRUZ, REYNALDO P. - 5 votes
4. ZURAEK, MERIANTHE PACITA M. - 5 votes
4. PAUIG, VILMA T. - 5 votes
91

5. ALAMEDA, ELMO M. - 4 votes


(C) THE APPOINTMENTS MADE WERE NOT IN ACCORDANCE WITH THE SHORTLISTS
SUBMITTED BY THE JUDICIAL AND BAR COUNCIL FOR EACH VACANCY, THUS AFFECTING THE
6. FERNANDEZ-BERNARDO, VICTORIA C. - 4 votes
ORDER OF SENIORITY OF THE ASSOCIATE JUSTICES.16chanroblesvirtuallawlibrary
According to petitioners, the JBC was created under the 1987 Constitution to reduce the
7. MUSNGI, MICHAEL FREDERICK L. - 4 votes14
politicization of the appointments to the Judiciary, i.e., "to rid the process of appointments to
President Aquino issued on January 20, 2015 the appointment papers for the six new the Judiciary from the political pressure and partisan activities."
Sandiganbayan Associate Justices, namely: (1) respondent Musngi; (2) Justice Reynaldo P.
Cruz (R. Cruz); (3) respondent Econg; (4) Justice Maria Theresa V. Mendoza-Arcega Article VIII, Section 9 of the 1987 Constitution contains the mandate of the JBC, as well as the
(Mendoza-Arcega); (5) Justice Karl B. Miranda (Miranda); and (6) Justice Zaldy V. Trespeses limitation on the President's appointing power to the Judiciary, thus:
(Trespeses). The appointment papers were transmitted on January 25, 2016 to the six new
Sandiganbayan Associate Justices, who took their oaths of office on the same day all at the Sec. 9. The Members of the Supreme Court and judges of lower courts shall be appointed by
Supreme Court Dignitaries Lounge. Respondent Econg, with Justices Mendoza-Arcega and the President from a list of at least three nominees prepared by the Judicial and Bar Council
Trespeses, took their oaths of office before Supreme Court Chief Justice Maria Lourdes P. A. for every vacancy. Such appointments need no confirmation.
Sereno (Sereno); while respondent Musngi, with Justices R. Cruz and Miranda, took their
oaths of office before Supreme Court Associate Justice Francis H. Jardeleza (Jardeleza). For the lower courts, the President shall issue the appointments within ninety days from the
submission of the list.
Arguments of the Petitioners
It is the function of the JBC to search, screen, and select nominees recommended for
Petitioners Aguinaldo, Alhambra, D. Cruz, Pozon, and Timbang (Aguinaldo, et al.), were all appointment to the Judiciary. It shall prepare a list with at least three qualified nominees for
nominees in the shortlist for the 16th Sandiganbayan Associate Justice. They assert that they a particular vacancy in the Judiciary to be submitted to the President, who, in turn, shall
possess the legal standing or locus standi to file the instant Petition since they suffered a appoint from the shortlist for said specific vacancy. Petitioners emphasize that Article VIII,
direct injury from President Aquino's failure to appoint any of them as the Section 9 of the 1987 Constitution is clear and unambiguous as to the mandate of the JBC to
16th Sandiganbayan Associate Justice. submit a shortlist of nominees to the President for "every vacancy" to the Judiciary, as well
as the limitation on the President's authority to appoint members of the Judiciary from
Petitioner IBP avers that it comes before this Court through a taxpayer's suit, by which among the nominees named in the shortlist submitted by the JBC.
taxpayers may assail an alleged illegal official action where there is a claim that public funds
are illegally disbursed, deflected to an improper use, or wasted through the enforcement of In this case, the JBC submitted six separate lists, with five to seven nominees each, for the six
an invalid or unconstitutional law. Petitioner IBP also maintains that it has locus vacancies in the Sandiganbayan, particularly, for the 16th, 17th, 18th, 19th, 20th and
standi considering that the present Petition involves an issue of transcendental importance 21st Associate Justices. Petitioners contend that only nominees for the position of the
to the people as a whole, an assertion of a public right, and a subject matter of public 16th Sandiganbayan Associate Justice may be appointed as the 16th Sandiganbayan Associate
interest. Lastly, petitioner IBP contends that as the association of all lawyers in the country, Justice, and the same goes for the nominees for each of the vacancies for the 17th, 18th, 19th,
with the fundamental purpose of safeguarding the administration of justice, it has a direct 20th, and 21st Sandiganbayan Associate Justices. However, on January 20, 2016, President
interest in the validity of the appointments of the members of the Judiciary. Aquino issued the appointment papers for the six new Sandiganbayan Associate Justices, to
wit:
Petitioners base their instant Petition on the following arguments:
VACANCY IN THE BAR CODE SHORTLISTED
PERSON APPOINTED
SANDIGANBAYAN NO. FOR
PRESIDENT AQUINO VIOLATED SECTION 9, ARTICLE VIII OF THE 1987 CONSTITUTION IN THAT:
Michael Frederick L. 21st Associate
chanRoblesvirtualLawlibrary(A) HE DID NOT APPOINT ANYONE FROM THE SHORTLIST 16th Associate Justice PNOY019445
Musngi Justice
SUBMITTED BY THE JBC FOR THE VACANCY FOR POSITION OF THE 16 TH ASSOCIATE JUSTICE
OF THE SANDIGANBAYAN; AND
19th Associate
17th Associate Justice Reynaldo P. Cruz PNOY019446
Justice
(B) HE APPOINTED UNDERSECRETARY MUSNGI AND JUDGE ECONG AS ASSOCIATE JUSTICES
OF THE SANDIGANBAYAN TO THE VACANCY FOR THE POSITION OF 21STASSOCIATE JUSTICE
18th Associate Justice Geraldine Faith A. PNOY019447 21st Associate
OF THE SANDIGANBAYAN.
92

On matters of procedure, the OSG argues, as follows:


Econg Justice
chanRoblesvirtualLawlibraryFirst, President Aquino should be dropped as a respondent m the
Maria Theresa V. 17th Associate
19th Associate Justice PNOY019448 instant case on the ground of his immunity from suit.
Mendoza-Arcega Justice
Second, petitioners Aguinaldo, et al. cannot institute an action for quo warranto because
20th Associate
20th Associate Justice Karl B. Miranda PNOY019449 usurpation of public office, position, or franchise is a public wrong, and not a private injury.
Justice
Hence, only the State can file such an action through the Solicitor General or public
prosecutor, under Sections 2 and 3, Rule 6620of the Rules of Court. As an exception, an
18th Associate
21st Associate Justice Zaldy V. Trespeses PNOY019450 individual may commence an action for quo warranto in accordance with Section 5, Rule
Justice
6621 of the Rules of Court if he/she claims entitlement to a public office or position. However,
Petitioners observe the following infirmities in President Aquino's for said individual's action for quo warranto to prosper, he/she must prove that he/she
appointments:ChanRoblesVirtualawlibrary suffered a direct injury as a result of the usurpation of public office or position; and that
he/she has a clear right, and not merely a preferential right, to the contested office or
position. Herein petitioners Aguinaldo, et al. have failed to show that they are entitled to the
a. Michael Frederick L. Musngi, nominated for the vacancy of the 21st Associate
positions now being held by respondents Musngi and Econg, as the inclusion of petitioners
Justice, was appointed as the 16th Associate Justice;
Aguinaldo, et al. in the shortlist for the 16th Sandiganbayan Associate Justice had only given
them the possibility, not the certainty, of appointment to the Sandiganbayan. Petitioners
b. Reynaldo P. Cruz, nominated for the vacancy of the 19th Associate Justice, was Aguinaldo, et al., as nominees, only had an expectant right because their appointment to the
appointed as the 17th Associate Justice; Sandiganbayan would still be dependent upon the President's discretionary appointing
power.
c. Geraldine Faith A. Econg, also nominated for the vacancy of the 21st Associate
Justice, but was appointed as the 18th Associate Justice; Third, petitioner IBP can only institute the certiorari and prohibition case, but not the action
for quo warranto against respondents Musngi and Econg because it cannot comply with the
d. Maria Theresa V. Mendoza[-Arcega], nominated for the vacancy of the direct injury requirement for the latter. Petitioner IBP justifies its locus standi to file the
17thAssociate Justice, but was appointed as the 19th Associate Justice; petition for certiorari and prohibition by invoking the exercise by this Court of its expanded
power of judicial review and seeking to oust respondents Musngi and Econg as
e. Zaldy V. Trespeses, nominated for the vacancy of the 18th Associate Justice, but was Sandiganbayan Associate Justices based on the alleged unconstitutionality of their
appointed as the 21st Associate Justice. appointments, and not on a claim of usurpation of a public office. Yet, based on Topacio v.
Ong,22 a petition for certiorari or prohibition is a collateral attack on a public officer's title,
60. Only the appointment of Karl B. Miranda as the 20th Associate Justice is in which cannot be permitted. Title to a public office can only be contested directly in a quo
accordance with his nomination.18 warranto proceeding.

Moreover, it is the JBC, not petitioner IBP, which has legal standing to file the present suit, as
Petitioners insist that President Aquino could only choose one nominee from each of the six
the dispute here is between the JBC and the OP. The fundamental question in this case is
separate shortlists submitted by the JBC for each specific vacancy, and no other; and any
"whether the JBC can corral the discretion of the President to appoint, a core constitutional
appointment made in deviation of this procedure is a violation of the Constitution. Hence,
prerogative, by designating qualified nominees within specific, artificial numerical categories
petitioners pray, among other reliefs, that the appointments of respondents Musngi and
and forcing the President to appoint in accordance with those artificial numerical categories."
Econg, who belonged to the same shortlist for the position of 21st Associate Justice, be
The Court, though, is barred from deciding said question because the JBC is not a party
declared null and void for these were made in violation of Article VIII, Section 9 of the 1987
herein.
Constitution.

Arguments of the Respondents Fourth, petitioners have erroneously included Jorge-Wagan, Romero� Maglaya, Zuraek,
Alameda, and Fernandez-Bernardo (Jorge-Wagan, et al.) as unwilling co-petitioners in the
The Office of the Solicitor General (OSG), on behalf of the Office of the President (OP), filed a Petition at bar. Apart from the fact that Jorge-Wagan, et al. do not claim entitlement to the
Comment,19 seeking the dismissal of the Petition on procedural and substantive grounds. positions occupied by respondents Musngi and Econg, non-appointed nominees for the
positions of 16th and 21st Associate Justices of the Sandiganbayan cannot simultaneously
93

claim right to assume two vacancies in said special court. unreasonably restricts the President's choices to only a few nominees even when the JBC
recognized 37 nominees qualified for the position of Sandiganbayan Associate Justice. This
And fifth, petitioners disregarded the hierarchy of courts by directly filing the instant Petition gives the JBC, apart from its power to recommend qualified nominees, the power to dictate
for Quo warranto and Certiorari and Prohibition before this Court. Even in cases where the upon the President which among the qualified nominees should be contending for a
Court is vested with original concurrent jurisdiction, it remains a court of last resort, not a particular vacancy. By dividing nominees into groups and artificially designating each group a
court of first instance. numerical value, the JBC creates a substantive qualification to various judicial posts, which
potentially impairs the President's prerogatives in appointing members of the Judiciary.
The OSG next addresses the substantive issues.
The OSG additionally points out that the JBC made a categorical finding that respondents
The OSG submits that the core argument of petitioners stems from their erroneous premise Musngi and Econg were "suitably best" for appointment as Sandiganbayan Associate Justice.
that there are existing numerical positions in the Sandiganbayan: the 1st being the Presiding The functions of the 16th Sandiganbayan Associate Justice are no different from those of the
Justice, and the succeeding 2nd to the 21st being the Associate Justices. It is the assertion of 17th, 18th, 19th, 20th, or 21stSandiganbayan Associate Justice. Since respondents Musngi and
the OSG that the Sandiganbayan is composed of a Presiding Justice and 20 Associate Justices, Econg were indubitably qualified and obtained sufficient votes, it was the ministerial duty of
without any numerical designations. Presidential Decree No. 1606 and its amendments do the JBC to include them as nominees for any of the six vacancies in the Sandiganbayan
not mention vacancies for the positions of "2nd Associate Justice," "3rd Associate Justice," etc. presented for the President's final consideration.
There are no such items in the Judiciary because such numerical designations are only used
to refer to the seniority or order of precedence of Associate Justices in collegiate courts such Furthermore, the OSG alleges that it is highly unjust to remove respondents Musngi and
as the Supreme Court, Court of Appeals, Court of Tax Appeals, and Sandiganbayan. Econg from their current positions on the sole ground that the nominees were divided into
six groups. The JBC announced "the opening/reopening, for application or recommendation"
The OSG further contends that the power to determine the order of precedence of the of "[s]ix (6) newly-created positions of Associate Justice of the Sandiganbayan." Respondents
Associate Justices of the Sandiganbayan is reposed in the President, as part of his Musngi and Econg applied for the vacancy of "Associate Justice of the Sandiganbayan." In its
constitutional power to appoint. Citing Section 1, third paragraph of Presidential Decree No. announcements for interview, the JBC stated that it would be interviewing applicants for "six
160623 and Rule II, Section 1 of the Revised Internal Rules of the Sandiganbayan,24 the OSG (6) newly created positions of Associate Justice of the Sandiganbayan." It was only on
explains that the order of precedence of the Associate Justices of the Sandiganbayan shall be October 26, 2015, the date of submission of the shortlists, when the nominees had been
according to the order of their appointments, that is, according to the dates of their clustered into six groups. The OSG notes that there are no JBC rules on the division of
respective commissions, or, when two or more commissions bear the same date, according nominees in cases where there are several vacancies in a collegiate court. In this case, the
to the order in which their commissions had been issued by the President. It is the averment OSG observes that there were no measurable standards or parameters for dividing the 37
of the OSG that the constitutional power of the JBC to recommend nominees for nominees into the six groups. The clustering of nominees was not based on the number of
appointment to the Judiciary does not include the power to determine their seniority. votes the nominees had garnered. The nominees were not evenly distributed among the six
President Aquino correctly disregarded the order of precedence in the shortlists submitted groups, i.e., there were five nominees for 17th Sandiganbayan Associate Justice; six nominees
by the JBC and exercised his statutory power to determine the seniority of the appointed for 16th, 18th, and 19th Sandiganbayan Associate Justices; and seven nominees for the
Sandiganbayan Associate Justices. 20th and 21st Sandiganbayan Associate Justices.

The OSG interprets Article VIII, Section 9 of the 1987 Constitution differently from The OSG then refers to several examples demonstrating that the previous practice of the JBC
petitioners. According to the OSG, said provision neither requires nor allows the JBC to was to submit only one shortlist for several vacancies in a collegiate court.
cluster nominees for every vacancy in the Judiciary; it only mandates that for every vacancy,
the JBC shall present at least three nominees, among whom the President shall appoint a The other respondents had likewise filed their respective Comments or Manifestations:
member of the Judiciary. As a result, if there are six vacancies for Sandiganbayan Associate
Justice, the JBC shall present, for the President's consideration, at least 18 nominees for said chanRoblesvirtualLawlibrary1) In respondent Fernandez-Bernardo's
vacancies. In the case at bar, the JBC submitted 37 nominees for the six vacancies in the Comment,25cralawred she recognizes the legal, substantial, and paramount significance of
Sandiganbayan; and from said pool of 37 nominees, the President appointed the six the ruling of the Court on the interpretation and application of Article VIII, Section 9 of the
Sandiganbayan Associate Justices, in faithful compliance with the Constitution. 1987 Constitution, which will serve as a judicial precedent for the guidance of the Executive
and Legislative Departments, the JBC, the Bench, and the Bar.
It is also the position of the OSG that the President has the absolute discretion to determine
who is best suited for appointment among all the qualified nominees. The very narrow 2) Respondent Musngi states in his Manifestation26 that he will no longer file a separate
reading of Article VIII, Section 9 of the 1987 Constitution proposed by petitioners Comment and that he adopts all the averments, issues, arguments, discussions, and reliefs in
94

the Comment of the OSG. announcement of the opening for application/recommendation was for the six newly-
created positions of Sandiganbayan Associate Justice; (b) respondent Econg's application was
3) In her Comment,27 respondent Jorge-Wagan maintains that she is not the proper party to for the six newly-created positions of Sandiganbayan Associate Justice; and (c) the
assail the validity of the appointment of the 16th Sandiganbayan Associate Justice as she was announcement of the public interview of candidates was for the six newly-created positions
nominated for the 21st Sandiganbayan Associate Justice; and that she is also not the proper of Sandiganbayan Associate Justice.
party to seek the nullification of the appointments of respondents Musngi and Econg as
Sandiganbayan Associate Justices. Not being a proper party-in-interest, respondent Jorge- Thus, respondent Econg prays for, among other reliefs, the dismissal of the instant Petition
Wagan argues that she cannot be considered an "unwilling co-plaintiff." for Quo Warranto and Certiorari and Prohibition for lack of merit, and the declaration that
the appointments of respondents Musngi and Econg as Sandiganbayan Associate Justices are
4) Respondent Romero-Maglaya makes the following averments in her valid.
Manifestation/Comment28: that she should not have been impleaded as a respondent or an
unwilling co-plaintiff in the instant Petition because her rights as a nominee for judicial 6) In respondent Sandoval's Comment,30 he avows that he opts not to join the petitioners as
appointment were not violated; that she had no claim of entitlement to the position of he subscribes to the principle that the heart and core of the President's power to appoint is
Sandiganbayan Associate Justice; and that she had no participation in the alleged violation of the freedom to choose. The power to appoint rests on the President and the President alone.
the Constitution or exercise of grave abuse of discretion amounting to lack or excess of Respondent Sandoval has already accepted the fact that he was not appointed despite being
jurisdiction. nominated by the JBC for the position of Sandiganbayan Associate Justice and he is looking
forward to another opportunity to apply for a higher position in the Judiciary.
5) Respondent Econg manifests in her Comment29 that while she is adopting in toto the
arguments in the Comment of the OSG, she is also making certain factual clarifications and Respondents Zuraek and Almeda have not filed their comments despite notice and are
additional procedural and substantive averments. deemed to have waived their right to do so.

Respondent Econg clarifies that her real name is Geraldine Faith A. Econg, and not Ma. On November 26, 2016, the JBC belatedly filed a Motion for Intervention in the Petition at
Geraldine Faith A. Econg. bar, or more than six months from the filing of the herein Petition on May 17, 2016 and after
Chief Justice Sereno, the Chairperson of the JBC herself, administered the oath of office of
Respondent Econg believes that the present Petition is really for quo warranto because it respondent Econg, whose appointment is now being questioned for having been done in
seeks to declare null and void the respective appointments of respondents Musngi and disregard of the clustering of nominees by the JBC.
Econg. Respondent Econg, however, asseverates that petitioners Aguinaldo, et al. have no
clear, unquestionable franchise to the Office of Associate Justice of the Sandiganbayan II
simply because they had been included in the shortlist submitted for the President's The Ruling of the Court
consideration. Nomination is not equivalent to appointment and the removal of respondents
Musngi and Econg will not automatically grant petitioners Aguinaldo, et al. the right to the The Court takes cognizance of the present Petition despite several procedural infirmities
Office of Associate Justice of the Sandiganbayan. Petitioners Aguinaldo, et al., except for given the transcendental importance of the constitutional issue raised herein.
petitioner Alhambra, are even uncertain about their right to the position/s of 16th and/or
21st Sandiganbayan Associate Justice/s as they have also applied for the position of The Petition at bar is for (a) Quo Warranto under Rule 66 of the Revised Rules of Court; and
Sandiganbayan Associate Justice in lieu of Sandiganbayan Associate Justice Teresita V. Diaz- (b) Certiorari and Prohibition under Rule 65 of the same Rules.
Baldos, who eventually retired on July 22, 2016. Even assuming for the sake of argument that
petitioners' alternative remedy of certiorari is proper, respondent Econg contends that Rule 66 of the Revised Rules of Court particularly identifies who can file a special civil action
petitioners only had 60 days to file such a petition from January 20, 2016, the date she and of Quo Warranto, to wit:
respondent Musngi were appointed. Petitioners belatedly filed their Petition before the
Court on May 17, 2016. RULE 66
Quo Warranto
Respondent Econg also raises the concern that if the Court affirms the petitioners' position
that there are no valid appointments for the 16th and 21st Sandiganbayan Associate Justices, Sec. 1. Action by Government against individuals. - An action for the usurpation of a public
the seniority or order of precedence among the Sandiganbayan Associate Justices will be office, position or franchise may be commenced by a verified petition brought in the name of
adversely affected. Respondent Econg avers that there was only one list of nominees for the the Republic of the Philippines against:
six vacant positions of Sandiganbayan Associate Justice, considering that: (a) the
95

chanRoblesvirtualLawlibrary(a) A person who usurps, intrudes into, or unlawfully holds or Neither can the IBP initiate a quo warranto proceeding to oust respondents Musngi and
exercises a public office, position or franchise; Econg from their currents posts as Sandiganbayan Associate Justices for the IBP does not
qualify under Rule 66, Section 5 of the Revised Rules of Court as an individual claiming to be
(b) A public officer who does or suffers an act which, by the provision of law, constitutes a entitled to the positions in question.
ground for the forfeiture of his office; or
Nevertheless, the Court takes in consideration the fact that the present Petition is also
(c) An association which acts as a corporation within the Philippines without being legally for Certiorari and Prohibition under Rule 65 of the Revised Rules of Court, which alleges that
incorporated or without lawful authority so to act President Aquino violated Article VIII, Section 9 of the 1987 Constitution and committed
grave abuse of discretion amounting to lack or excess of jurisdiction in his appointment of
Sec. 2. When Solicitor General or public prosecutor must commence action. - The Solicitor respondents Musngi and Econg as Sandiganbayan Associate Justices.
General or a public prosecutor, when directed by the President of the Philippines, or when
upon complaint or otherwise he has good reason to believe that any case specified in the Article VIII, Section 1 of the 1987 Constitution vests upon the Court the expanded power of
preceding section can be established by proof, must commence such action. judicial review, thus:
Article VIII
Sec. 3. When Solicitor General or public prosecutor may commence action with permission of
court. - The Solicitor General or a public prosecutor may, with the permission of the court in Sec. 1. The judicial power shall be vested in one Supreme Court and in such lower courts as
which the action is to be commenced, bring such an action at the request and upon the may be established by law.
relation of another person; but in such case the officer bringing it may first require an
indemnity for the expenses and costs of the action in an amount approved by and to be Judicial power includes the duty of the courts of justice to settle actual controversies
deposited in the court by the person at whose request and upon whose relation the same is involving rights which are legally demandable and enforceable, and to determine whether or
brought. not there has been a grave abuse of discretion amounting to lack or excess of jurisdiction on
the part of any branch or instrumentality of the Government.
Sec. 5. When an individual may commence such an action. - A person claiming to be entitled The Court recognized in Jardeleza v. Sereno (Jardeleza Decision)32 that a "petition
to a public office or position usurped or unlawfully held or exercised by another may bring an for certiorari is a proper remedy to question the act of any branch or instrumentality of the
action therefor in his own name. government on the ground of grave abuse of discretion amounting to lack or excess of
In Topacio v. Ong,31 the Court pronounced that:ChanRoblesVirtualawlibrary jurisdiction by any branch or instrumentality of the government, even if the latter does not
A quo warranto proceeding is the proper legal remedy to determine the right or title to the exercise judicial, quasi-judicial or ministerial functions."
contested public office and to oust the holder from its enjoyment. It is brought against the
person who is alleged to have usurped, intruded into, or unlawfully held or exercised the In opposing the instant Petition for Certiorari and Prohibition, the OSG cites Topacio in which
public office, and may be commenced by the Solicitor General or a public prosecutor, as the the Court declares that title to a public office may not be contested except directly, by quo
case may be, or by any person claiming to be entitled to the public office or position usurped warranto proceedings; and it cannot be assailed collaterally, such as by certiorari and
or unlawfully held or exercised by another. prohibition.

Nothing is more settled than the principle, which goes back to the 1905 case of Acosta v. However, Topacio is not on all fours with the instant case. In Topacio, the writs
Flor, reiterated in the recent 2008 case of Feliciano v. Villasin, that for a quo of certiorari and prohibition were sought against Sandiganbayan Associate Justice Gregory S.
warrantopetition to be successful, the private person suing must show a clear right to the Ong on the ground that he lacked the qualification of Filipino citizenship for said position. In
contested office. In fact, not even a mere preferential right to be appointed thereto can contrast, the present Petition for Certiorari and Prohibition puts under scrutiny, not any
lend a modicum of legal ground to proceed with the action. disqualification on the part of respondents Musngi and Econg, but the act of President
Aquino in appointing respondents Musngi and Econg as Sandiganbayan Associate Justices
Petitioners Aguinaldo, et al., as nominees for the 16th Saridiganbayan Associate Justice, did without regard for the clustering of nominees into six separate shortlists by the JBC, which
not have a clear right to said position, and therefore not proper parties to a quo allegedly violated the Constitution and constituted grave abuse of discretion amounting to
warranto proceeding. Being included in the list of nominees had given them only the lack or excess of jurisdiction. This would not be the first time that the Court, in the exercise of
possibility, but not the certainty, of being appointed to the position, given the discretionary its expanded power of judicial review, takes cognizance of a petition for certiorari that
power of the President in making judicial appointments. It is for this same reason that challenges a presidential appointment for being unconstitutional or for having been done in
respondents Jorge-Wagan, et al., nominees for the 21st Sandiganbayan Associate Justice, may grave abuse of discretion. As the Court held in Funa v. Villar34:
not be impleaded as respondents or unwilling plaintiffs in a quo warranto proceeding.
96

Anent the aforestated posture of the OSG, there is no serious disagreement as to the He could be suing as a "stranger," or in the category of a "citizen," or "taxpayer." In either
propriety of the availment of certiorari as a medium to inquire on whether the assailed case, he has to adequately show that he is entitled to seek judicial protection. In other
appointment of respondent Villar as COA Chairman infringed the constitution or was infected words, he has to make out a sufficient interest in the vindication of the public order and the
with grave abuse of discretion. For under the expanded concept of judicial review under the securing of relief as a "citizen" or "taxpayer."
1987 Constitution, the corrective hand of certiorari may be invoked not only "to settle actual
controversies involving rights which are legally demandable and enforceable," but also "to Case law in most jurisdictions now allows both "citizen" and "taxpayer" standing in public
determine whether or not there has been a grave abuse of discretion amounting to lack or actions. The distinction was first laid down in Beauchamp v. Silk, where it was held that the
excess of jurisdiction on the part of any branch or instrumentality of the government." plaintiff in a taxpayer's suit is in a different category from the plaintiff in a citizen's suit. In the
"Grave abuse of discretion" denotes: such capricious and whimsical exercise of judgment as former, the plaintiff is affected by the expenditure of public funds, while in the latter, he is
is equivalent to lack of jurisdiction, or, in other words, where the power is exercised in an but the mere instrument of the public concern. As held by the New York Supreme Court
arbitrary or despotic manner by reason of passion or personal hostility, and it must be so in People ex rel Case v. Collins: "In matter of mere public right, however ... the people are the
patent and gross as to amount to an evasion of positive duty or to a virtual refusal to perform real parties... It is at least the right, if not the duty, of every citizen to interfere and see that a
the duty enjoined or to act in contemplation of law. public offence be properly pursued and punished, and that a public grievance be remedied."
With respect to taxpayer's suits, Terr v. Jordan held that "the right of a citizen and a taxpayer
We find the remedy of certiorari applicable to the instant case in view of the allegation that to maintain an action in courts to restrain the unlawful use of public funds to his injury
then President Macapagal-Arroyo exercised her appointing power in a manner constituting cannot be denied."
grave abuse of discretion. (Citations omitted.)
However, being a mere procedural technicality, the requirement of locus standi may be
Even so, the Court finds it proper to drop President Aquino as respondent taking into account waived by the Court in the exercise of its discretion. This was done in the 1949 Emergency
that when this Petition was filed on May 17, 2016, he was still then the incumbent President Powers Cases, Araneta v. Dinglasan, where the "transcendental importance" of the cases
who enjoyed immunity from suit. The presidential immunity from suit remains preserved in prompted the Court to act liberally. Such liberality was neither a rarity nor accidental.
the system of government of this country, even though not expressly reserved in the 1987 In Aquino v. Comelec, this Court resolved to pass upon the issues raised due to the "far-
Constitution.35 The President is granted the privilege of immunity from suit "to assure the reaching implications" of the petition notwithstanding its categorical statement that
exercise of Presidential duties and functions free from any hindrance or distraction, petitioner therein had no personality to file the suit. Indeed, there is a chain of cases where
considering that being the Chief Executive of the Government is a job that, aside from this liberal policy has been observed, allowing ordinary citizens, members of Congress, and
requiring all of the office-holder's time, also demands undivided attention."36 It is sufficient civic organizations to prosecute actions involving the constitutionality or validity of laws,
that former Executive Secretary Ochoa is named as respondent herein as he was then the regulations and rulings.
head of the OP and was in-charge of releasing presidential appointments, including those to
the Judiciary. Thus, the Court has adopted a rule that even where the petitioners have failed to show direct
injury, they have been allowed to sue under the principle of "transcendental importance."
Since the Petition at bar involves a question of constitutionality, the Court must determine Pertinent are the following cases:
the locus standi or legal standing of petitioners to file the same. The Court will exercise its
power of judicial review only if the case is brought before it by a party who has the legal (1) Chavez v. Public Estates Authority, where the Court ruled that the enforcement of the
standing to raise the constitutional or legal question. "Legal standing" means a personal and constitutional right to information and the equitable diffusion of natural resources are
substantial interest in the case such that the party has sustained or will sustain direct injury matters of transcendental importance which clothe the petitioner with locus standi;
as a result of the governmental act that is being challenged; while "interest" refers to
material interest, an interest in issue and to be affected by the decree or act assailed, as (2) Bagong Alyansang Makabayan v. Zamora, wherein the Court held that "given the
distinguished from mere interest in the question involved, or a mere incidental interest. The transcendental importance of the issues involved, the Court may relax the standing
interest of the plaintiff must be personal and not one based on a desire to vindicate the requirements and allow the suit to prosper despite the lack of direct injury to the parties
constitutional right of some third and unrelated party. seeking judicial review" of the Visiting Forces Agreement;

In David v. Macapagal-Arroyo,39 the Court acknowledged exceptional circumstances which (3) Lim v. Executive Secretary, while the Court noted that the petitioners may not file suit in
justified liberality and relaxation of the rules on legal standing:ChanRoblesVirtualawlibrary their capacity as taxpayers absent a showing that "Balikatan 02-01" involves the exercise of
The difficulty of determining locus standi arises in public suits. Here, the plaintiff who asserts Congress' taxing or spending powers, it reiterated its ruling in Bagong Alyansang Makabayan
a "public right" in assailing an allegedly illegal official action, does so as a representative of v. Zamora, that in cases of transcendental importance, the cases must be settled promptly
the general public. He may be a person who is affected no differently from any other person. and definitely and standing requirements may be relaxed.
97

By way of summary, the following rules may be culled from the cases decided by this Court. of public interest, and for the above-mentioned reasons, the Court shall accord petitioners
Taxpayers, voters, concerned citizens, and legislators may be accorded standing to sue, the legal standing to sue.
provided that the following requirements are met:
The instant Petition fundamentally challenges President Aquino's appointment of
(1) the cases involve constitutional issues; respondents Musngi and Econg as the 16th and 18th Sandiganbayan Associate Justices.
Petitioners contend that only one of them should have been appointed as both of them were
included in one cluster of nominees for the 21stSandiganbayan Associate Justice. The Petition
(2) for taxpayers, there must be a claim of illegal disbursement of public funds or that the presents for resolution of the Court the issue of whether President Aquino violated Article
tax measure is unconstitutional; VIII, Section 9 of the 1987 Constitution and gravely abused his discretionary power to appoint
members of the Judiciary when he disregarded the clustering by the JBC of the nominees for
each specific vacant position of Sandiganbayan Associate Justice. The issue is of paramount
(3) for voters, there must be a showing of obvious interest in the validity of the election importance for it affects the validity of appointments to collegiate courts and, ultimately, the
law in question; administration of justice, for if there are questions as to the right of the appointee to his
position as judge/justice, then doubts shall likewise shadow all his acts as such. This will
indubitably undermine the faith of the public in the judicial system. Since at hand is a
(4) for concerned citizens, there must be a showing that the issues raised are of constitutional issue of first impression, which will likely arise again when there are
transcendental importance which must be settled early; and simultaneous vacancies in collegiate courts, it is imperative for the Court to already resolve
the same for the guidance of the Bench and Bar, and the general public as well.

(5) for legislators, there must be a claim that the official action complained of infringes The OSG also prays for the dismissal of this Petition on the additional ground that petitioners,
upon their prerogatives as legislators. by coming directly before this Court, violated the hierarchy of courts. Relevant to this matter
are the following pronouncements of the Court in Querubin v. Commission on Elections42:
While neither petitioners Aguinaldo, et al. nor petitioner IBP have legal standing to file a
petition for quo warranto, they have legal standing to institute a petition for certiorari.
Notwithstanding the non-exclusivity of the original jurisdiction over applications for the
issuance of writs of certiorari, however, the doctrine of hierarchy of courts dictates that
The clustering of nominees by the JBC, which the President, for justifiable reasons, did not
recourse must first be made to the lower-ranked court exercising concurrent jurisdiction with
follow, could have caused all nominees direct injury, thus, vesting them with personal and
a higher court. The rationale behind the principle is explained in Ba�ez, Jr. v. Concepcion in
substantial interest, as the clustering limited their opportunity to be considered for
the following wise:
appointment to only one of the six vacant positions for Sandiganbayan Associate Justice
instead of all the six vacant positions to which the JBC found them as qualified for
The Court must enjoin the observance of the policy on the hierarchy of courts, and now
appointment. This is the far-reaching adverse consequence to petitioners Aguinaldo, et
affirms that the policy is not to be ignored without serious consequences. The strictness of
al. that they have missed. More importantly, for a complete resolution of this Petition, the
the policy is designed to shield the Court from having to deal with causes that are also well
Court must inevitably address the issue of the validity of the clustering of nominees by the
within the competence of the lower courts, and thus leave time to the Court to deal with the
JBC for simultaneous vacancies in collegiate courts, insofar as it seriously impacts on the
more fundamental and more essential tasks that the Constitution has assigned to it. The
constitutional power of the President to appoint members of the Judiciary, which will be
Court may act on petitions for the extraordinary writs of certiorari, prohibition
explained below.
and mandamus only when absolutely necessary or when serious and important reasons exist
to justify an exception to the policy.
One of the fundamental purposes of the IBP is to improve the administration of justice.40 As
the association of all lawyers in the country, petitioner IBP has an interest in ensuring the
Petitioners do not have the absolute and unrestrained freedom of choice of the court to
validity of the appointments to the Judiciary. It is recognized that the administration of
which an application for certiorari will be directed. Indeed, referral to the Supreme Court as
justice is primarily a joint responsibility of the judge and the lawyer.41 Definitely, lawyers
the court of last resort will simply be empty rhetoric if party-litigants are able to flout judicial
cannot effectively discharge their duties if they entertain doubts, or worse, had lost their
hierarchy at will. The Court reserves the direct invocation of its jurisdiction only when there
faith in judges and/or justices. It is clearly imperative for the IBP to prevent that situation
are special and important reasons clearly and especially set out in the petition that would
from happening by exercising vigilance and ensurmg that the judicial appointment process
justify the same.
remains transparent and credible.

Given that the constitutional issue in the Petition at bar is of transcendental importance and
98

In the leading case of The Diocese of Bacolod v. Comelec, the Court enumerated the specific obtained copies of the shortlists for the vacancies for the 16th to the 21st Sandiganbayan
instances when direct resort to this Court is allowed, to wit: Associate Justices on March 22, 2016. Counting the 60-day period from March 22, 2016,
petitioners allege that they had until May 21, 2016 to file their Petition.
(a) When there are genuine issues of constitutionality that must be addressed at the most
immediate time; Rule 65, Section 4 of the Revised Rules of Court explicitly states that certiorari should be
instituted within a period of 60 days from notice of the judgment, order, or resolution sought
to be assailed. The 60-day period is inextendible to avoid any unreasonable delay that would
(b) When the issues involved are of transcendental importance; violate the constitutional rights of parties to a speedy disposition of their case. The question
though is when said 60-day period began to run in this case. The Court refers to its ruling
in Velicaria-Garafil v. Office of the President.43 In said case, the Court declared that
(c) Cases of first impression; appointment is a process. For an appointment to be valid, complete, and effective, four
elements must always concur, to wit: "(1) authority to appoint and evidence of the exercise
of authority, (2) transmittal of the appointment paper and evidence of the transmittal, (3) a
(d) When the constitutional issues raised are best decided by this Court; vacant position at the time of appointment, and (4) receipt of the appointment paper and
acceptance of the appointment by the appointee who possesses all the qualifications and
none of the disqualifications." The Court expounded on the importance of the last element
(e) When the time element presented in this case cannot be ignored; as follows:
Acceptance is indispensable to complete an appointment. Assuming office and taking the
oath amount to acceptance of the appointment. An oath of office is a qualifying requirement
(f) When the petition reviews the act of a constitutional organ; for a public office, a prerequisite to the full investiture of the office.

Javier v. Reyes is instructive in showing how acceptance is indispensable to complete an


(g) When there is no other plain, speedy, and adequate remedy in the ordinary course of appointment. On 7 November 1967, petitioner Isidro M. Javier (Javier) was appointed by
law; then Mayor Victorino B. Aldaba as the Chief of Police of Malolos, Bulacan. The Municipal
Council confirmed and approved Javier's appointment on the same date. Javier took his oath
of office on 8 November 1967, and subsequently discharged the rights, prerogatives, and
(h) When public welfare and the advancement of public policy so dictates, or when duties of the office. On 3 January 1968, while the approval of Javier's appointment was
demanded by the broader interest of justice; pending with the CSC, respondent Purificacion C. Reyes (Reyes), as the new mayor of
Malolos, sent to the CSC a letter to recall Javier's appointment. Reyes also designated Police
Lt. Romualdo F. Clemente as Officer-in-Charge of the police department. The CSC approved
(i) When the orders complained of are patent nullities; and Javier's appointment as permanent on 2 May 1968, and even directed Reyes to reinstate
Javier. Reyes, on the other hand, pointed to the appointment of Bayani Bernardo as Chief of
Police of Malolos, Bulacan on 4 September 1967. This Court ruled that Javier's appointment
(j) When appeal is considered as clearly an inappropriate remedy. (Citations omitted.) prevailed over that of Bernardo. It cannot be said that Bernardo accepted his appointment
because he never assumed office or took his oath.
Inasmuch as the Petition at bar involves a constitutional question of transcendental
importance and of first impression and demanded by the broader interest of justice, the Excluding the act of acceptance from the appointment process leads us to the very evil which
Court, in the exercise of its discretion, resolves to exercise primary jurisdiction over the we seek to avoid (i.e., antedating of appointments). Excluding the act of acceptance will only
same. provide more occasions to honor the Constitutional provision in the breach. The inclusion of
acceptance by the appointee as an integral part of the entire appointment process prevents
Lastly, respondent Econg opposes the Petition at bar for being filed out of time. According to the abuse of the Presidential power to appoint. It is relatively easy to antedate appointment
respondent Econg, the 60-day period for petitioners to file this Petition commenced on papers and make it appear that they were issued prior to the appointment ban, but it is more
January 20, 2016, the date she and her co-respondent Musngi were appointed by President difficult to simulate the entire appointment process up until acceptance by the
Aquino. Based on respondent Econg's argument, the 60-day period ended on March 20, appointee.44 (Citations omitted.)
2016, Sunday, so petitioners only had until March 21, 2016, Monday, to timely file the
Petition. For their part, petitioners aver that after learning of the appointments of
respondents Musngi and Econg as Sandiganbayan Associate Justices from the media, they
99

The records show that on January 25, 2016, the appointment papers were transmitted to and disregarding the clustering of nominees into six separate shortlists for the six vacancies for
received by the six newly-appointed Sandiganbayan Associate Justices, including respondents Sandiganbayan Associate Justice.
Musngi and Econg, who, on the same day, already took their oaths of office. Therefore,
pursuant to Velicaria-Garafil, the appointment process became complete and effective on Article VIII, Section 9 of the 1987 Constitution provides that "[t]he Members of the Supreme
January 25, 2016. If the Court is to count the 60-day reglementary period for filing a petition Court and judges of lower courts shall be appointed by the President from a list of at least
for certiorari from January 25, 2016, it expired on March 25, 2016. The present Petition three nominees prepared by the Judicial and Bar Council for every vacancy."
for Certiorari and Prohibition was filed on May 17, 2016.
The appointment process for the Judiciary seems simple enough if there is only one vacancy
Just like any rule, however, there are recognized exceptions to the strict observance of the to consider at a time. The power of the President to appoint members of the Judiciary is
60-day period for filing a petition for certiorari, viz.: (1) most persuasive and weighty reasons; beyond question, subject to the limitation that the President can only appoint from a list of
(2) to relieve a litigant from an injustice not commensurate with his failure to comply with at least three nominees submitted by the JBC for every vacancy. However, the controversy in
the prescribed procedure; (3) good faith of the defaulting party by immediately paying within this case arose because by virtue of Republic Act No. 10660, creating two new divisions of
a reasonable time from the time of the default; (4) the existence of special or compelling the Sandiganbayan with three members each, there were six simultaneous vacancies for
circumstances; (5) the merits of the case; (6) a cause not entirely attributable to the fault or Associate Justice of said collegiate court; and that the JBC submitted six separate shortlists
negligence of the party favored by the suspension of the rules; (7) a lack of any showing that for the vacancies for the 16th to the 21st Sandiganbayan Associate Justices.
the review sought is merely frivolous and dilatory; (8) the other party will not be unjustly
prejudiced thereby; (9) fraud, accident, mistake, or excusable negligence without appellant's On one hand, petitioners assert that President Aquino's power to appoint is limited to each
fault; (10) peculiar legal and equitable circumstances attendant to each case; (11) in the shortlist submitted by the JBC. President Aquino should have appointed the
name of substantial justice and fair play; (12) importance of the issues involved; and (13) 16th Sandiganbayan Associate Justice from the nominees in the shortlist for the
exercise of sound discretion by the judge guided by all the attendant circumstances. There 16th Sandiganbayan Associate Justice, the 17th Sandiganbayan Associate Justice from the
should be an effort, though, on the part of the party invoking liberality to advance a nominees in the shortlist for the 17th Sandiganbayan Associate Justice, and so on and so
reasonable or meritorious explanation for his/her failure to comply with the rules. forth. By totally overlooking the nominees for the 16th Sandiganbayan Associate Justice and
appointing respondents Musngi and Econg, who were both nominees for the
The peculiar circumstances of this case, plus the importance of the issues involved herein, 21st Sandiganbayan Associate Justice, as the 16th and 18th Sandiganbayan Associate Justices,
justify the relaxation of the 60-day period for the filing of this Petition for Certiorari and respectively, President Aquino violated the 1987 Constitution and committed grave abuse of
Prohibition. Indeed, the official act assailed by petitioners is the appointment by President discretion amounting to lack or excess of jurisdiction.
Aquino of respondents Musngi and Econg as Sandiganbayan Associate Justices, which was
completed on January 25, 2016 when said respondents took their oaths of office. Yet, Respondents, on the other hand, maintain that President Aquino acted in accordance with
petitioners could not have sought remedy from the Court at that point. As basis for the 1987 Constitution and well-within his discretionary power to appoint members of the
petitioners' opposition to the said appointments, they needed to see and secure copies of Judiciary when he disregarded the clustering of nominees by the JBC into six separate
the shortlists for the 16th to the 21st Sandiganbayan Associate Justices. It was only after shortlists and collectively considered all 37 nominees named in said shortlists for the six
petitioners obtained copies of all six shortlists on March 22, 2016 that petitioners would have vacancies for Sandiganbayan Associate Justice.
been able to confirm that no one from the shortlist for the 16th Sandiganbayan Associate
Justice was appointed to any of the six vacancies for Sandiganbayan Associate Justice; and The primordial question then for resolution of the Court is whether President Aquino, under
that respondents Musngi and Econg, both in the shortlist for the 21st Sandiganbayan the circumstances, was limited to appoint only from the nominees in the shortlist submitted
Associate Justice, were appointed as the 16th and 18thSandiganbayan Associate Justices, by the JBC for each specific vacancy.
respectively. In addition, respondent Econg is not unjustly prejudiced by the delay, but will
even benefit from the Court resolving once and for all the questions on her right to the The Court answers in the negative.
position of Sandiganbayan Associate Justice.
The JBC was created under the 1987 Constitution with the principal function of
The Court reiterates that there can be no valid objection to its discretion to waive one or recommending appointees to the Judiciary.47 It is a body, representative of all the
some procedural requirements if only to remove any impediment to address and resolve the stakeholders in the judicial appointment process, intended to rid the process of
constitutional question of transcendental importance raised in this Petition, the same having appointments to the Judiciary of the evils of political pressure and partisan activities.48 The
far�reaching implications insofar as the administration of justice is concemed. extent of the role of the JBC in recommending appointees vis-a-vis the power of the
President to appoint members of the Judiciary was discussed during the deliberations of the
President Aquino did not violate the Constitution or commit grave abuse of discretion in Constitutional Commission (CONCOM) on July 10, 1986, thus:ChanRoblesVirtualawlibrary
100

MR. RODRIGO: Let me go to another point then. This is clear under Section 1, paragraph 3 of Presidential Decree No. 1606, which
reads:ChanRoblesVirtualawlibrary
On page 2, Section 5, there is a novel provision about appointments of members of the Sec. 1. Sandiganbayan; composition; qualifications; tenure; removal and compensation
Supreme Court and of judges of lower courts. At present it is the President who appoints
them. If there is a Commission on Appointments, then it is the President with the The Presiding Justice shall be so designated in his commission and the other Justices shall
confirmation of the Commission on Appointments. In this proposal, we would like to have precedence according to the dates of their respective commissions, or, when the
establish a new office, a sort of a board composed of seven members, called the Judicial and commissions of two or more of them shall bear the same date, according to the order in
Bar Council. And while the President will still appoint the members of the judiciary, he will be which their commissions have been issued by the President.
limited to the recommendees of this Council. Consistent with the foregoing, Rule II, Section 1(b) of the Revised Internal Rules of the
Sandiganbayan similarly provides:
MR. CONCEPCION: That is correct.
Sec. 1. Composition of the Court and Rule on Precedence.-
MR. RODRIGO: And the Council will, whenever there is a vacancy, recommend three.
(b) Rule on Precedence - The Presiding Justice shall enjoy precedence over the other
MR. CONCEPCION: At least three for every vacancy. members of the Sandiganbayan in all official functions. The Associate Justices shall have
precedence according to the order of their appointments.
MR. RODRIGO: And the President cannot appoint anybody outside of the three Apropos herein is the following ruling of the Court in Re: Seniority Among the Four (4) Most
recommendees. Recent Appointments to the Position of Associate Justices of the Court of Appeals,50 which
involved the Court of Appeals, another collegiate court:ChanRoblesVirtualawlibrary
MR. CONCEPCION: Nomination by the Council would be one of the qualifications for For purposes of appointments to the judiciary, therefore, the date the commission has been
appointment.49chanroblesvirtuallawlibrary signed by the President (which is the date appearing on the face of such document) is the
It is apparent from the aforequoted CONCOM deliberations that nomination by the JBC shall date of the appointment. Such date will determine the seniority of the members of the Court
be a qualification for appointment to the Judiciary, but this only means that the President of Appeals in connection with Section 3, Chapter I of BP 129, as amended by RA 8246. In
cannot appoint an individual who is not nominated by the JBC. It cannot be disputed herein other words, the earlier the date of the commission of an appointee, the more senior
that respondents Musngi and Econg were indeed nominated by the JBC and, hence, qualified he/she is over the other subsequent appointees. It is only when the appointments of two
to be appointed as Sandiganbayan Associate Justices. or more appointees bear the same date that the order of issuance of the appointments by
the President becomes material. This provision of statutory law (Section 3, Chapter I of BP
It should be stressed that the power to recommend of the JBC cannot be used to restrict or 129, as amended by RA 8246) controls over the provisions of the 2009 IRCA which gives
limit the President's power to appoint as the latter's prerogative to choose someone whom premium to the order of appointments as transmitted to this Court. Rules implementing a
he/she considers worth appointing to the vacancy in the Judiciary is still paramount. As long particular law cannot override but must give way to the law they seek to implement.
as in the end, the President appoints someone nominated by the JBC, the appointment is
valid. On this score, the Court finds herein that President Aquino was not obliged to appoint Evidently, based on law, rules, and jurisprudence, the numerical order of the Sandiganbayan
one new Sandiganbayan Associate Justice from each of the six shortlists submitted by the Associate Justices cannot be determined until their actual appointment by the President.
JBC, especially when the clustering of nominees into the six shortlists encroached on
President Aquino's power to appoint members of the Judiciary from all those whom the JBC It bears to point out that part of the President's power to appoint members of a collegiate
had considered to be qualified for the same positions of Sandiganbayan Associate Justice. court, such as the Sandiganbayan, is the power to determine the seniority or order of
preference of such newly appointed members by controlling the date and order of issuance
Moreover, in the case at bar, there were six simultaneous vacancies for the position of of said members' appointment or commission papers. By already designating the numerical
Sandiganbayan Associate Justice, and the JBC cannot, by clustering of the nominees, order of the vacancies, the JBC would be establishing the seniority or order of preference of
designate a numerical order of seniority of the prospective appointees. The Sandiganbayan, a the new Sandiganbayan Associate Justices even before their appointment by the President
collegiate court, is composed of a Presiding Justice and 20 Associate Justices divided into and, thus, unduly arrogating unto itself a vital part of the President's power of appointment.
seven divisions, with three members each. The numerical order of the seniority or order of
preference of the 20 Associate Justices is determined pursuant to law by the date and order There is also a legal ground why the simultaneous vacant positions of Sandiganbayan
of their commission or appointment by the President. Associate Justice should not each be assigned a specific number by the JBC. The
Sandiganbayan Associate Justice positions were created without any distinction as to rank in
seniority or order of preference in the collegiate court. The President appoints his choice
101

nominee to the post of Sandiganbayan Associate Justice, but not to a Sandiganbayan 79. In June 2012, there were 3 vacancies for Associate Justice of the CA. Out of 53
Associate Justice position with an identified rank, which is automatically determined by the candidates, the JBC submitted to the President only 1 short list of 14 nominees who obtained
order of issuance of appointment by the President. The appointment does not specifically the required number of votes. Based on this short list, President Aquino appointed Associate
pertain to the 16th, 17th, 18th, 19th, 20th, or 21st Sandiganbayan Associate Justice, because the Justices Henri Jean Paul B. Inting, Oscar V. Badelies, and Marie Christine Azcarraga Jacob.
Sandiganbayan Associate Justice's ranking is temporary and changes every time a vacancy
occurs in said collegiate court. In fact, by the end of 2016, there will be two more vacancies Additionally, in 1995, when Republic Act No. 7975 increased the divisions in the
for Sandiganbayan Associate Justice.51 These vacancies will surely cause movement in the Sandiganbayan from three to five, which similarly created six simultaneous vacant positions
ranking within the Sandiganbayan. At the time of his/her appointment, a Sandiganbayan of Sandiganbayan Associate Justice, the JBC, with then Supreme Court Chief Justice Andres R.
Associate Justice might be ranked 16th, but because of the two vacancies occurring in the Narvasa as Chairman, submitted a single list of nominees from which former President Fidel
court, the same Sandiganbayan Associate Justice may eventually be higher ranked. V. Ramos subsequently chose his six appointees. Reproduced in full below was the
nomination submitted by the JBC on said occasion:
Furthermore, the JBC, in sorting the qualified nominees into six clusters, one for every July 17, 1997
vacancy, could influence the appointment process beyond its constitutional mandate of
recommending qualified nominees to the President. Clustering impinges upon the HIS EXCELLENCY
President's power of appointment, as well as restricts the chances for appointment of the PRESIDENT FIDEL V. RAMOS
qualified nominees, because (1) the President's option for every vacancy is limited to the five Malaca�an, Manila
to seven nominees in the cluster; and (2) once the President has appointed from one cluster,
then he is proscribed from considering the other nominees in the same cluster for the other Dear Mr. President:
vacancies. The said limitations are utterly without legal basis and in contravention of the
President's appointing power. chanRoblesvirtualLawlibraryPursuant to the provisions of Article VIII, Section 9 of the
Constitution, the Judicial and Bar Council has the honor to submit the nominations (in
To recall, the JBC invited applications and recommendations and conducted interviews for alphabetical order) for six (6) positions of Associate Justice of the Sandiganbayan, per the JBC
the "six newly created positions of Associate Justice of the Sandiganbayan." Applicants, Minutes of July 9 and 16, 1997:
including respondents Musngi and Econg, applied for the vacancy for "Associate Justice of 1. Asuncion, Elvi John S.
the Sandiganbayan." Throughout the application process before the JBC, the six newly- 2. Badoy Jr., Anacleto D.
created positions of Sandiganbayan Associate Justice were not specifically identified and 3. Casta�eda Jr., Catalina D.
differentiated from one another for the simple reason that there was really no legal 4. De Castro, Teresita Leonardo
justification to do so. The requirements and qualifications, as well as the power, duties, and 5. Fineza, Antonio J.
responsibilities are the same for all the Sandiganbayan Associate Justices. If an individual is 6. Flores, Alfredo C.
found to be qualified for one vacancy, then he/she is also qualified for all the other 7. Gustilo, Alfredo J.
vacancies. It was only at the end of the process that the JBC precipitously clustered the 37 8. Hernandez, Jose R.
qualified nominees into six separate shortlists for each of the six vacant positions. 9. Ilarde, Ricardo M.
10. Laggui, Pedro N.
The Court notes that the clustering of nominees is a totally new practice of the JBC. 11. Lee Jr., German G.
Previously, the JBC submitted only one shortlist for two or more vacancies in a collegiate 12. Legaspi, Godofredo L.
court. Worth reproducing below are the examples cited by the OSG: 13. Makasiar, Ramon P.
14. Mallillin, Hesiquio R.
For instance, in June 2011, there were 2 vacancies for Associate Justice of the Supreme 15. Martinez, Wilfredo C.
Court. Out of 30 candidates, the JBC submitted to the President only 1 short list of 6 16. Mirasol, Teodulo E.
nominees. Based on this short list, President Aquino appointed Associate Justices Bienvenido 17. Nario, Narciso S.
L. Reyes, and Estela Perlas-Bernabe. 18. Navarro, Flordelis Ozaeta
19. Ortile, Senecio D.
In January 2012, there were 3 vacancies for Associate Justice of the CA. Out of sixty-three 20. Pineda, Ernesto L.
(63) candidates, the JBC prepared only 1 short list of 13 nominees for these 3 vacancies. 21. Ponferrada, Bernardo T.
Based on this short list, President Aquino appointed Associate Justices Ma. Luisa C. 22. Quimsing, Godofredo P.
Quijano�Padilla, Renate C. Francisco, and Jhosep Y. Lopez. 23. Rivera, Candido V.
102

24. Rosario Jr., Eriberto U. Manila


25. Salonga, Josefina Guevara
26. Sultan, Justo M. Your Excellency:
27. Umali, Mariano M.
Their respective curriculum vitae are hereto attached. chanRoblesvirtualLawlibraryPursuant to Article VIII, Section 9 of the Constitution, the Judicial
Once more, on November 23, 2009, the JBC, then headed by Supreme Court Chief Justice and Bar Council has the honor to submit nominations for the two (2) positions of ASSOCIATE
Reynato S. Puno (Puno), submitted to former President Gloria Macapagal-Arroyo JUSTICE of the SUPREME COURT, per the JBC Minutes of even date, as follows:
(Macapagal-Arroyo) a single list of nominees for two vacant positions of Supreme Court
Associate Justice, from which President Macapagal-Arroyo ultimately appointed Associate 1. Reyes, Jose, Jr. C. - 7 votes �
Justices Jose P. Perez and Jose C. Mendoza. The letter of nomination of the JBC reads:
November 23, 2009 2. Robles, Rodolfo D. - 7 votes �

Her Excellency 3. De Leon, Magdangal M. - 6 votes �


President Gloria Macapagal Arroyo
Malaca�ang Palace 4. Reyes, Bienvenido L. - 6 votes �
Manila
5. Bernabe, Estela Perlas - 5 votes �
Your Excellency:
6. Dimaampao, Japar B. - 5 votes �
chanRoblesvirtualLawlibraryPursuant to Section 9, Article VIII of the Constitution, the Judicial
Their respective curriculum vitae are hereto attached.
and Bar Council has the honor to submit nominations for two (2) positions of Associate
There is no explanation for the shift in practice by the JBC, which impaired the power of the
Justice of the Supreme Court (vice Hon. Leonardo A. Quisumbing and Hon. Minita V. Chico-
President to appoint under the 1987 Constitution and his statutory authority to determine
Nazario), per the JBC Minutes of even date, to wit: seniority in a collegiate court. The clustering by the JBC of the qualified nominees for the six
1. Abdulwahid, Hakim S. - 6 votes � vacancies for Sandiganbayan Associate Justice appears to have been done arbitrarily, there
being no clear basis, standards, or guidelines for the same. The number of nominees was not
2. Mendoza, Jose C. - 6 votes � even equally distributed among the clusters.

3. Perez, Jose P. - 5 votes � In view of the foregoing, President Aquino validly exercised his discretionary power to
appoint members of the Judiciary when he disregarded the clustering of nominees into six
4. Villaruz, Francisco, Jr. H. - 5 votes � separate shortlists for the vacancies for the 16th, 17th, 18th, 19th, 20th and 21st Sandiganbayan
Associate Justices. President Aquino merely maintained the well-established practice,
5. De Leon, Magdangal M. - 4 votes � consistent with the paramount Presidential constitutional prerogative, to appoint the six new
Sandiganbayan Associate Justices from the 37 qualified nominees, as if embodied in one JBC
6. Tijam, Noel G. - 4 votes � list. This does not violate Article VIII, Section 9 of the 1987 Constitution which requires the
President to appoint from a list of at least three nominees submitted by the JBC for every
Their respective curriculum vitae are hereto attached. vacancy. To meet the minimum requirement under said constitutional provision of three
And, as mentioned by the OSG, the JBC, during the Chairmanship of Supreme Court Chief nominees per vacancy, there should at least be 18 nominees from the JBC for the six
Justice Renato C. Corona, submitted to President Aquino on June 21, 2011 just one list of vacancies for Sandiganbayan Associate Justice; but the minimum requirement was even
nominees for two vacant positions of Supreme Court Associate Justice, from which President exceeded herein because the JBC submitted for the President's consideration a total of 37
Aquino eventually appointed Associate Justices Bienvenido L. Reyes and Estela M. Perlas- qualified nominees. All the six newly appointed Sandiganbayan Associate Justices met the
Bernabe. Such list is fully quoted hereunder:��� requirement of nomination by the JBC under Article VIII, Section 9 of the 1987 Constitution.
June 21, 2011 Hence, the appointments of respondents Musngi and Econg, as well as the other four new
Sandiganbayan Associate Justices, are valid and do not suffer from any constitutional
His Excellency infirmity.
President Benigno Simeon C. Aquino III
Malaca�ang Palace The ruling of the Court in this case shall similarly apply to the situation wherein there are
103

closely successive vacancies in a collegiate court, to which the President shall make
appointments on the same occasion, regardless of whether the JBC carried out combined or The Court is unconvinced.
separate application process/es for the vacancies. The President is not bound by the
clustering of nominees by the JBC and may consider as one the separate shortlists of The instant Petition was filed before this Court on May 17, 2016, yet, the JBC filed its Motion
nominees concurrently submitted by the JBC. As the Court already ratiocinated herein, the for Intervention only on November 26, 2016, more than six months later, and even praying
requirements and qualifications, as well as the power, duties, and responsibilities are the for an additional 30-day period from notice to submit its complaint-in-intervention.
same for all the vacant posts in a collegiate court; and if an individual is found to be qualified Therefore, allowing the intervention will undoubtedly delay the resolution of the case; and
for one vacancy, then he/she is also qualified for all the other vacancies. It is worthy of note further delay in the resolution of this case will only perpetuate the doubts on the legitimacy
that the JBC, in previous instances of closely successive vacancies in collegiate courts, such as of the appointments of respondents Musngi and Econg as Sandiganbayan Associate Justices,
the Court of Appeals and the Supreme Court, faithfully observed the practice of submitting to the detriment of said court, in particular, and the entire justice system, in general. What is
only a single list of nominees for all the available vacancies, with at least three nominees for more, unless promptly resolved by the Court, the instant case is capable of repetition given
every vacancy, from which the President made his appointments on the same occasion. This the forthcoming vacancies in collegiate courts, particularly, the Supreme Court.
is in keeping with the constitutional provisions on the President's exclusive power to appoint
members of the Judiciary and the mandate of the JBC to recommend qualified nominees for Even if the intervention of the JBC will evidently cause delay in the resolution of this case and
appointment to the Judiciary. prejudice to the original parties herein, are there compelling substantive grounds to still
allow the intervention of the JBC? The JBC, through its own fault, did not provide the Court
The Court denies the Motion for Intervention of the JBC in this Petition. with a way to make such a determination. The Revised Rules of Court explicitly requires that
the pleading-in-intervention already be attached to the motion for intervention. 55 The JBC
In its Motion for Intervention, the JBC echoes the arguments of the OSG in the latter's could have already argued the merits of its case in its complaint-in-intervention. However,
Comment that the dispute is between the JBC and the OP and it cannot be decided by the the JBC not only failed to attach its complaint-in-intervention to its Motion for Intervention,
Court since the JBC is not a party, much less, a complaining party in this case. The JBC asserts but it also did not provide any explanation for such failure.
that it has legal interest in the matter of litigation because it will be adversely affected by the
judgment or decision in the present case, having submitted the controverted shortlists of The Court can reasonably assume, as well, that the JBC is well-aware of President Aquino's
nominees to the OP. The JBC likewise claims that its intervention will not unduly delay or appointment of the six Sandiganbayan Associate Justices, including respondents Musngi and
prejudice the adjudication of the rights of the original parties in the case. The JBC, thus, prays Econg, on January 20, 2015. The six newly-appointed Sandiganbayan Associate Justices all
that it be allowed to intervene in the instant case and to submit its complaint-in-intervention took their oaths of office on January 25, 2016 at the Supreme Court Dignitaries Lounge.
within 30 days from receipt of notice allowing its intervention. Respondent Econg, with Justices Mendoza-Arcega and Trespeses, took their oaths of office
before Chief Justice Sereno, who is also the Chairperson of the JBC; while respondent
Intervening in a case is not a matter of right but of sound discretion of the Court.53 The Musngi, with Justices R. Cruz and Miranda, took their oaths of office before Supreme Court
allowance or disallowance of a motion for intervention rests on the sound discretion of the Associate Justice Jardeleza on the same occasion and at the same venue. Despite its
court after consideration of the appropriate circumstances. It is not an absolute right. The knowledge of the appointment and assumption of office of respondents Musngi and Econg in
statutory rules or conditions for the right of intervention must be shown. The procedure to January 2016, the JBC did not take any action to challenge the same on the ground that
secure the right to intervene is to a great extent fixed by the statute or rule, and intervention President Aquino appointed respondents Musngi and Econg in disregard of the clustering of
can, as a rule, be secured only in accordance with the terms of the applicable provision. nominees by the JBC through the separate shortlists for the six vacancies for Sandiganbayan
Associate Justice. The silence of the JBC all this while, for a period of eleven (11) months, can
It bears to point out that petitioners did not name the JBC as a respondent in this case already be deemed as acquiescence to President Aquino's appointment of respondents
because petitioners precisely wanted the shortlists submitted by the JBC upheld; they were Musngi and Econg.
on the same side. Petitioners already presented the arguments for the constitutionality of
and strict adherence by the President to the separate shortlists submitted by the JBC for the For the foregoing reasons, the Court denies the Motion for Intervention of the JBC.
six simultaneous vacancies for Sandiganbayan Associate Justice. Significantly, not one of the
parties moved, and not even the Court motu proprio ordered, to implead the JBC as an There are several other new rules and practices adopted by the JBC which the Court takes
indispensable party herein. cognizance of as a separate administrative matter.

The JBC avers in its Motion for Intervention that it has a legal interest in the Petition at bar The Court takes cognizance of several other matters covered by the new rules and practices
and its intervention will not unduly delay or prejudice the adjudication of the rights of the adopted by the JBC.
original parties in the case.
104

Item No. 1: The Court takes judicial notice of the fact that the JBC promulgated on institutionalizes Chief Justice Sereno's unilateral decision to abandon a well-established rule,
September 20, 2016 JBC No. 2016-1, "The Revised Rules of the Judicial and Bar Council" procedure, and practice observed by the Court, and completely precludes the incumbent
(Revised JBC Rules), to take effect on October 24, 2016. Notably, the Revised JBC Rules Supreme Court Justices from expressing their views on the qualifications of the applicants to
explicitly states among its Whereas clauses: the vacancies in the Supreme Court.

WHEREAS, the President of the Philippines may appoint only one from the list of at least The Court calls attention to the fact that the JBC, in JBC-009 and the Revised JBC Rules,
three nominees for every vacancy officially transmitted by the Council to the Office of the invites the public to give any comment or opposition against the applicants to the Judiciary.
President[.]
According to Rule 1, Section 9 of JBC-009:
This is an obvious attempt by the JBC to institutionalize through the Revised JBC Rules its
newly-introduced practice of clustering nominees for simultaneous vacancies in collegiate Sec. 9. Publication of list of applicants. The list of applicants or recommendees which the
courts. The timing likewise is disturbing as the instant case is pending resolution by this Court Council shall consider in a given time shall be published once in a newspaper of general
and with existing and upcoming vacancies in several collegiate courts, i.e., the circulation in the Philippines and once in a newspaper of local circulation in the province or
Sandiganbayan, the Court of Appeals, and even this Court. As the Court has categorically city where the vacancy is located. The publication shall invite the public to inform the
declared herein, the clustering by the JBC of nominees for simultaneous vacancies in Council within the period fixed therein of any complaint or derogatory information against
collegiate courts constitute undue limitation on and impairment of the power of the the applicant.
President to appoint members of the Judiciary under the 1987 Constitution. It also deprives
qualified nominees equal opportunity to be considered for all vacancies, not just a specific A similar provision can be found in the Revised JBC Rules as Rule 1, Section 8:
one. Incorporating such Whereas clause into the Revised JBC Rules will not serve to legitimize
an unconstitutional and unfair practice. Accordingly, such Whereas clause shall not bind the Sec. 8. Publication of List of Applicants. The list of applicants who meet the minimum
President pursuant to the pronouncements of the Court in the present Petition. qualifications and the Council's evaluative criteria prescribed in Sections 2 and 3 of Rule 3 of
these Rules, which the Council shall consider in a given time, shall be published once in two
Item No. 2: The same Revised JBC Rules deleted a significant part of JBC-009, the former JBC newspapers of general circulation in the Philippines.
Rules, specifically, Rule 8, Section 1, which provided:
The publication shall inform the public that any complaint or opposition against applicants
Sec. 1. Due weight and regard to the recommendees of the Supreme Court. - In every case may be filed with the secretariat of the Council. A copy of the list shall likewise be posted in
involving an appointment to a seat in the Supreme Court, the Council shall give due weight the JBC website. (Emphasis supplied.)
and regard to the recommendees of the Supreme Court. For this purpose, the Council shall Yet, Chief Justice Sereno, without consulting the Court en banc, has done away with the
submit to the Court a list of candidates for any vacancy in the Court with an executive settled practice of seeking the views of the incumbent Justices on the applicants to the
summary of its evaluation and assessment of each of them, together with all relevant records vacant positions in the Supreme Court.
concerning the candidates from whom the Court may base the selection of its
recommendees. To recall, Chief Justice Sereno had previously disregarded Rule 8, Section 1 of JBC-009, during
the nomination process for the vacancy of Supreme Court Associate Justice following the
The deletion of this provision will likewise institutionalize the elimination by Chief Justice retirement of Associate Justice Roberto A. Abad on May 22, 2014. As Associate Justice Arturo
Sereno of the voting by the Supreme Court Justices on who among the applicants to the D. Brion narrated in his Separate Concurring Opinion in the Jardeleza Decision56:
Supreme Court they believe are most deserving.
[Of particular note in this regard is this Court's own experience when it failed to vote for its
Through Rule 8, Section 1 of JBC-009, the JBC had accorded through the years due weight recornmendees for the position vacated by retired Associate Justice Roberto A. Abad,
and regard to the recommendees of the Supreme Court for the vacancies in said Court. The because of a letter dated May 29, 2014 from the Chief Justice representing to the Court that
JBC had consistently complied with said rule and furnished the Court in prior years with the "several Justices" requested that the Court do away with the voting for Court
list of candidates for vacancies in the Court, together with an executive summary of the recornmendees, as provided in Section 1, Rule 8 of JBC-009. When subsequently confronted
evaluation and assessment of each candidate by the JBC and all relevant documents on who these Justices were, the Chief Justice failed to name anyone. As a result, applicants
concerning the candidates, for the incumbent Justices' consideration, but stopped doing so who could have been recommended by the Court (Jardeleza, among them), missed their
ever since Chief Justice Sereno became the Chairperson of the JBC. Although the JBC was not chance to be nominees.]
bound by the list of recommendees of the Court, the JBC at least took the list under
advisement. The deletion of the foregoing provision from the Revised JBC Rules formally
105

The Supreme Court Justices were also not given the opportunity to know the applicants to Jr. and Teresita J. Leonardo� De Castro as JBC consultants, and in their stead, the Chief
the succeeding vacant position in the Court (to which Associate Justice Alfredo Benjamin S. Justice appointed retired Chief Justices Hilario G. Davide, Jr., Artemio V. Panganiban, and
Caguioa was eventually appointed) as Rule 8, Section 1 of JBC-009 was again not followed. Reynato S. Puno as JBC consultants. The experience and wisdom of the three retired Chief
Justices are undisputed. However, practicality and prudence also dictate that incumbent
Item No. 3: The JBC currently has no incumbent Supreme Court Associate Justice as Associate Justices of the Court should be retained as JBC consultants since their interest in
consultant. By practice, since the creation of the JBC, the two (2) most senior Supreme Court the Judiciary is real, actual, and direct. Incumbent Associate Justices of the Court are aware
Associate Justices had acted as consultants of the JBC. From 1987 until 2016, the following of the present state, needs, and concerns of the Judiciary, and consultants from the Court,
Associate Justices of this Court, during their incumbency, served as JBC consultants: even if they have no right to vote, have served, from the organization of the JBC, as the only
link to the supervisory authority of the Court over the JBC under the 1987 Constitution.
Supreme Court Associate Justices as JBC Moreover, Hon. Angelina Sandoval-Gutierrez already sits as a regular member of the JBC
Period
Consultants representing the Retired Supreme Court Justices, pursuant to Article VIII, Section 8(1) of the
1987 Constitution, which expressly describes the composition of the JBC, as follows:
Pedro L. Yap+ December 10, 1987 to April 13, 1988
Sec. 8. (1) A Judicial and Bar Council is hereby created under the supervision of the Supreme
Marcelo B. Fernan+ January 5, 1988 to June 29, 1988 Court composed of the Chief Justice as Chairman, the Secretary of Justice, and a
representative of the Congress as Members, a representative of the Integrated Bar, a
Andres R. Narvasa May 6, 1988 to December 5, 1991 professor of law, a retired Member of the Supreme Court, and a representative of the
private sector. (Emphasis supplied.)
Leo M. Medialdea+ July 21, 1988 to November 4, 1992
These changes in settled rules and practices recently adopted by the JBC under Chief Justice
Ameurfina M. Herrera January 16, 1992 to March 30, 1992 Sereno are disconcerting. There appears to be a systematic move by the JBC, under Chief
Justice Sereno to arrogate to itself more power and influence than it is actually granted by
December 21, 1993 to November 13, the Constitution and this Court, and at the same time, to ease out the Court from any
Josue N. Bellosillo
2003 legitimate participation in the nomination process for vacancies in the Judiciary, specifically,
in the Supreme Court. This behooves the Court, through the exercise of its power of
Jose C. Vitug November 20, 2003 to July 14, 2004 supervision over the JBC, to take a closer look into the new rules and practices of the JBC and
ensure that these are in accord with the 1987 Constitution, the pertinent laws, and the
Artemio V. Panganiban July 21, 2004 to December 19, 2005 governmental policies of transparency and accountability in the nomination process for
vacancies in the Judiciary.
Leonardo A. Quisumbing January 1, 2006 to November 5, 2009
Article VIII, Section 8 of the 1987 Constitution gives the JBC the principal function of
Consuelo Y. Santiago December 11, 2006 to October 4, 2009 "recommending appointees to the Judiciary," but it also explicitly states that the JBC shall be
"under the supervision of the Court" and that "[i]t may exercise such other functions and
Renato C. Corona November 6, 2009 to May 16, 2010 duties as the Supreme Court may assign to it."

Antonio T. Carpio October 5, 2009 to May 16, 2010 Book IV, Chapter 7, Section 38(2) of Executive Order No. 292, otherwise known as The
September 10, 2012 to January 28, Administrative Code of the Philippines, defines supervision as follows:
2014
Sec. 38. Definition of Administrative Relationship. - Unless otherwise expressly stated in the
Presbiterio J. Velasco, Jr. June 4, 2012 to August 23, 2012 Code or in other laws defining the special relationships of particular agencies, administrative
September 10, 2012 to [August 2016] relationships shall be categorized and defmed as follows:

Teresita J. Leonardo-De Castro June 4, 2012 to August 23, 2012 (2) Administrative Supervision. - (a) Administrative supervlSlon which shall govern the
[February 1, 2014] to [August 2016]58 administrative relationship between a department or its equivalent and regulatory agencies
or other agencies as may be provided by law, shall be limited to the authority of the
Without notice, warning, or explanation to the Supreme Court En Banc, Chief Justice Sereno department or its equivalent to generally oversee the operations of such agencies and to
recently unceremoniously relieved Supreme Court Associate Justices Presbiterio J. Velasco,
106

insure that they are managed effectively, efficiently and economically but without conditions in order to render the power real and effective."
interference with day-to-day activities; or require the submission of reports and cause the
conduct of management audit, performance evaluation and inspection to determine In the exercise of its power of supervision over the JBC, the Court shall take up the
compliance with policies, standards and guidelines of the department; to take such action as aforementioned Item Nos. 2 and 3 as a separate administrative matter and direct the JBC to
may be necessary for the proper performance of official functions, including rectification of file its comment on the same.
violations, abuses and other forms of maladministration; and to review and pass upon
budget proposals of such agencies but may not increase or add to them; WHEREFORE, premises considered, the Court DISMISSES the instant Petition for Quo
Warranto and Certiorari and Prohibition for lack of merit. The Court DECLARES the clustering
(b) Such authority shall not, however, extend to: (1) appointments and other personnel of nominees by the Judicial and Bar Council UNCONSTITUTIONAL, and the appointments of
actions in accordance with the decentralization of personnel functions under the Code, respondents Associate Justices Michael Frederick L. Musngi and Geraldine Faith A. Econg,
except when appeal is made from an action of the appointing authority, in which case the together with the four other newly-appointed Associate Justices of the Sandiganbayan,
appeal shall be initially sent to the department or its equivalent, subject to appeal in as VALID. The Court further DENIES the Motion for Intervention of the Judicial and Bar
accordance with law; (2) contracts entered into by the agency in the pursuit of its objectives, Council in the present Petition, but ORDERS the Clerk of Court En Banc to docket as a
the review of which and other procedures related thereto shall be governed by appropriate separate administrative matter the new rules and practices of the Judicial and Bar Council
laws, rules and regulations; and (3) the power to review, reverse, revise, or modify the which the Court took cognizance of in the preceding discussion as Item No.2: the deletion or
decisions of regulatory agencies in the exercise of their regulatory or quasi-judicial functions; non-inclusion in JBC No. 2016-1, or the Revised Rules of the Judicial and Bar Council, of Rule
and cralawlawlibrary 8, Section 1 of JBC-009; and Item No. 3: the removal of incumbent Senior Associate Justices
of the Supreme Court as consultants of the Judicial and Bar Council, referred to in pages 35
(c) Unless a different meaning is explicitly provided in the specific law governing the to 40 of this Decision. The Court finally DIRECTS the Judicial and Bar Council to file its
relationship of particular agencies, the word "supervision" shall encompass administrative comment on said Item Nos. 2 and 3 within thirty (30) days from notice.
supervision as defined in this paragraph.
The Court also provided the following definition of supervision in the Jardeleza Decision59: SO ORDERED.

As a meaningful guidepost, jurisprudence provides the definition and scope of supervision. It


is the power of oversight, or the authority to see that subordinate officers perform their
duties. It ensures that the laws and the rules governing the conduct of a government entity
are observed and complied with. Supervising officials see to it that rules are followed, but
they themselves do not lay down such rules, nor do they have the discretion to modify or
replace them. If the rules are not observed, they may order the work done or redone, but
only to conform to such rules. They may not prescribe their own manner of execution of the
act. They have no discretion on this matter except to see to it that the rules are followed.

"Supervision" is differentiated from "control," thus:

Supervisory power, when contrasted with control, is the power of mere oversight over an
inferior body; it does not include any restraining authority over such body. Officers in control
lay down the rules in the doing of an act. If they are not followed, it is discretionary on his
part to order the act undone or re-done by his subordinate or he may even decide to do it
himself. Supervision does not cover such authority. Supervising officers merely sees to it that
the rules are followed, but he himself does not lay down such rules, nor does he have the
discretion to modify or replace them. If the rules are not observed, he may order the work
done or re-done to conform to the prescribed rules. He cannot prescribe his own manner for
the doing of the act.60 (Citations omitted.)

The Court had recognized that "[s]upervision is not a meaningless thing. It is an active power.
It is certainly not without limitation, but it at least implies authority to inquire into facts and
107

EN BANC At the outset, the Court notes the revelation of the JBC in its Motion for Reconsideration-in-
Intervention that it is not taking any position in this particular case on President Aquino's
February 21, 2017 appointments to the six newly-created positions of Sandiganbayan Associate Justice. The
Court quotes the relevant portions from the Motion, as follows:
G.R. No. 224302
The immediate concern of the JBC is this Court's pronouncement that the former's act of
submitting six lists for six vacancies was unconstitutional. Whether the President can cross-
HON. PHILIP A. AGUINALDO, HON. REYNALDO A. ALHAMBRA, HON. DANILO S. CRUZ, HON.
reach into the lists is not the primary concern of the JBC in this particular case. At another
BENJAMIN T. POZON, HON. SALVADOR V. TIMBANG, JR., and the INTEGRATED BAR OF THE
time, perhaps, it may take a position. But not in this particular situation involving the
PHILIPPINES (IBP),Petitioners
newly created positions in the Sandiganbayan in view of the lack of agreement by the JBC
vs.
Members on that issue.
HIS EXCELLENCY PRESIDENT BENIGNO SIMEON C. AQUINO III, HON. EXECUTIVE SECRETARY
PAQUITO N. OCHOA, HON. MICHAEL FREDERICK L. MUSNGI, HON. MA. GERALDINE FAITH
A. ECONG, HON. DANILO S. SANDOVAL, HON. WILHELMINA B. JORGE-WAGAN, HON. What the President did with the lists, for the purpose of this particular dispute alone as far
ROSANA FE ROMERO-MAGLAYA, HON. MERIANTHE PACITA M. ZURAEK, HON. ELMO M. as the JBC is concerned, was the President's exclusive domain. 2
ALAMEDA, and HON. VICTORIA C. FERNANDEZ-BERNARDO, Respondents
Nonetheless, the JBC did not categorically withdraw the arguments raised in its previous
RESOLUTION Motions, and even reiterated and further discussed said arguments, and raised additional
points in its Motion for Reconsideration-in-Intervention. Hence, the Court is still constrained
to address said arguments in this Resolution.
LEONARDO-DE CASTRO, J.:

In its Motion for Reconsideration (with Motion for Inhibition of the Ponente) the JBC argues
In its Decision dated November 29, 2016, the Court En Banc held:
as follows: (a) Its Motion for Intervention was timely filed on November 26, 2016, three days
before the promulgation of the Decision in the instant case; (b) The JBC has a legal interest in
WHEREFORE, premises considered, the Court DISMISSES the instant Petition for Quo this case, and its intervention would not have unduly delayed or prejudiced the adjudication
Warranto and Certiorari and Prohibition for lack of merit. The Court DECLARES the clustering of the rights of the original parties; (c) Even assuming that the Motion for Intervention
of nominees by the Judicial and Bar Council UNCONSTITUTIONAL, and the appointments of suffers procedural infirmities, said Motion should have been granted for a complete
respondents Associate Justices Michael Frederick L. Musngi and Geraldine Faith A. Econg, resolution of the case and to afford the JBC due process; and (d) Unless its Motion for
together with the four other newly-appointed Associate Justices of the Sandiganbayan, Intervention is granted by the Court, the JBC is not bound by the questioned Decision
as VALID. The Court further DENIES the Motion for Intervention of the Judicial and Bar because the JBC was neither a party litigant nor impleaded as a party in the case, the JBC was
Council in the present Petition, but ORDERS the Clerk of Court En Banc to docket as a deprived of due process, the assailed Decision is a judgment in personam and not a
separate administrative matter the new rules and practices of the Judicial and Bar Council judgment in rem, and a decision rendered in violation of a party's right to due process is void
which the Court took cognizance of in the preceding discussion as Item No. 2: the deletion or for lack of jurisdiction.
non-inclusion in JBC No. 2016-1, or the Revised Rules of the Judicial and Bar Council, of Rule
8, Section 1 of JBC-009; and Item No. 3: the removal of incumbent Senior Associate Justices
On the merits of the case, the JBC asserts that in submitting six short lists for six vacancies, it
of the Supreme Court as consultants of the Judicial and Bar Council, referred to in pages 3 5
was only acting in accordance with the clear and unambiguous mandate of Article VIII,
to 40 of this Decision. The Court finally DIRECTS the Judicial and Bar Council to file its
Section 93 of the 1987 Constitution for the JBC to submit a list for every vacancy. Considering
comment on said Item Nos. 2 and 3 within thirty (30) days from notice.1
its independence as a constitutional body, the JBC has the discretion and wisdom to perform
its mandate in any manner as long as it is consistent with the Constitution. According to the
I JBC, its new practice of "clustering," in fact, is more in accord with the purpose of the JBC to
THE JBC MOTIONS rid the appointment process to the Judiciary from political pressure as the President has to
choose only from the nominees for one particular vacancy. Otherwise, the President can
The Judicial and Bar Council (JBC) successively filed a Motion for Reconsideration (with choose whom he pleases, and thereby completely disregard the purpose for the creation of
Motion for the Inhibition of the Ponente) on December 27, 2016 and a Motion for the JBC. The JBC clarifies that it numbered the vacancies, not to influence the order of
Reconsideration-in-Intervention (Of the Decision dated 29 November 2016) on February 6, precedence, but for practical reasons, i.e., to distinguish one list from the others and to avoid
2017. confusion. The JBC also points out that the acts invoked against the JBC are based on practice
108

or custom, but "practice, no matter how long continued, cannot give rise to any vested would stay neutral and not take any legal position on the constitutionality of said
right." The JBC, as a constitutional body, enjoys independence, and as such, it may change its appointments since it "did not have any legal interest in the offices of Associate Justices of
practice from time to time in accordance with its wisdom. the Sandiganbayan"; (b) None of the parties prayed that the act of clustering by the JBC be
declared unconstitutional; and (c) The JBC believed that the Court would apply the doctrine
Lastly, the JBC moves for the inhibition of the ponente of the assailed Decision based on of presumption of regularity in the discharge by the JBC of its official functions and if the
Canon 3, Section 5 of the New Code of Judicial Conduct for Philippine Judiciary.4 The JBC Court would have been inclined to delve into the validity of the act of clustering by the JBC, it
alleges that the ponente, as consultant of the JBC from 2014 to 2016, had personal would order the JBC to comment on the matter.
knowledge of the voting procedures and format of the short lists, which are the subject
matters of this case. The ponente was even present as consultant during the meeting on The JBC impugns the significance accorded by the ponente to the fact that Chief Justice Maria
October 26, 2015 when the JBC voted upon the candidates for the six new positions of Lourdes P. A. Sereno (Sereno), Chairperson of the JBC, administered the oath of office of
Associate Justice of the Sandiganbayan created under Republic Act No. 10660. The JBC then respondent Econg as Sandiganbayan Associate Justice on January 25, 2016. Chief Justice
expresses its puzzlement over the ponente 's participation in the present proceedings, Serena's act should not be taken against the JBC because, the JBC reasons, Chief Justice
espousing a position contrary to that of the JBC. The JBC questions why it was only in her Sereno only chairs the JBC, but she is not the JBC, and the administration of the oath of office
Decision in the instant case did the ponente raise her disagreement with the JBC as to the was a purely ministerial act.
clustering of nominees for each of the six simultaneous vacancies for Sandiganbayan
Associate Justice. The JBC further quoted portions of the assailed Decision that it claims The JBC likewise disputes the ponente 's observation that clustering is a totally new practice
bespoke of the ponente 's "already-arrived-at" conclusion as to the alleged ill acts and of the JBC. The JBC avers that even before Chief Justice Sereno's Chairmanship, the JBC has
intentions of the JBC. Hence, the JBC submits that such formed inference will not lend to an generally followed the rule of one short list for every vacancy in all first and second level trial
even-handed consideration by the ponente should she continue to participate in the case. courts. The JBC has followed the "one list for every vacancy" rule even for appellate courts
since 2013. The JBC even recalls that it submitted on August 17, 2015 to then President
Ultimately, the JBC prays: Benigno Simeon C. Aquino III (Aquino) four separate short lists for four vacancies in the Court
of Appeals; and present during the JBC deliberations were the ponente and Supreme Court
IN VIEW OF THE FOREGOING, it is respectfully prayed that the DECISION dated 29 November Associate Justice Presbitero J. Velasco, Jr. (Velasco) as consultants, who neither made any
2016 be reconsidered and set aside and a new one be issued granting the Motion for comment on the preparation of the short lists.
Intervention of the JBC.
On the merits of the Petition, the JBC maintains that it did not exceed its authority and, in
It is likewise prayed that the ponente inhibit herself from further participating in this case fact, it only faithfully complied with the literal language of Article VIII, Section 9 of the 1987
and that the JBC be granted such other reliefs as are just and equitable under the premises.5 Constitution, when it prepared six short lists for the six vacancies in the Sandiganbayan. It
cites the cases of Atong Paglaum, Inc. v. Commission on Elections6and Ocampo v.
Enriquez,7 wherein the Court allegedly adopted the textualist approach of constitutional
The JBC subsequently filed a Motion for Reconsideration-in-Intervention (Of the Decision
interpretation.
dated 29 November 2016), praying at the very beginning that it be deemed as sufficient
remedy for the technical deficiency of its Motion for Intervention (i.e., failure to attach the
pleading-in-intervention) and as Supplemental Motion for Reconsideration of the denial of its The JBC renounces any duty to increase the chances of appointment of every candidate it
Motion for Intervention. adjudged to have met the minimum qualifications. It asserts that while there might have
been favorable experiences with the past practice of submitting long consolidated short lists,
past practices cannot be used as a source of rights and obligations to override the duty of the
The JBC, in its latest Motion, insists on its legal interest, injury, and standing to intervene in
JBC to observe a straightforward application of the Constitution.
the present case, as well as on the timeliness of its Motion for Intervention.

The JBC posits that clustering is a matter of legal and operational necessity for the JBC and
The JBC proffers several reasons for not immediately seeking to intervene in the instant case
the only safe standard operating procedure for making short lists. It presents different
despite admitting that it received copies of the appointments of the six Sandiganbayan
scenarios which demonstrate the need for clustering, viz., (a) There are two different sets of
Associate Justices from the Office of the President (OP) on January 25, 2016, to wit: (a) Even
applicants for the vacancies; (b) There is a change in the JBC composition during the interval
as its individual Members harbored doubts as to the validity of the appointments of
in the deliberations on the vacancies as the House of Representatives and the Senate
respondents Michael Frederick L. Musngi (Musngi) and Geraldine Faith A. Econg (Econg) as
alternately occupy the ex officio seat for the Legislature; (c) The applicant informs the JBC of
Sandiganbayan Associate Justices, the JBC agreed as a body in an executive session that it
his/her preference for assignment in the Cebu Station or Cagayan de Oro Station of the Court
109

of Appeals because of the location or the desire to avoid mingling with certain personalities; 1. the Court's pronouncements in the Decision dated 29 November 2016 with respect to the
(d) The multiple vacancies in newly-opened first and second level trial courts; and (e) The JBC's submission of six shortlists of nominees to the Sandiganbayan be modified to reflect
dockets to be inherited in the appellate court are overwhelming so the JBC chooses that the JBC is deemed to have followed Section 9, Article VIII of the Constitution in its
nominees for those particular posts with more years of service as against those near practice of submitting one shortlist of nominees for every vacancy, including in submitting on
retirement. 28 October 2015 six lists to former President Benigno Simeon C. Aquino III for the six
vacancies of the Sandiganbayan, or for the Court to be completely silent on the matter; and
To the JBC, it seems that the Court was in a hurry to promulgate its Decision on November
29, 2016, which struck down the practice of clustering by the JBC. The JBC supposes that it 2. the Court delete the treatment as a separate administrative matter of the alleged new
was in anticipation of the vacancies in the Court as a result of the retirements of Supreme rules and practices of the JBC, particularly the following: (1) the deletion or non-inclusion of
Court Associate Justices Jose P. Perez (Perez) and Arturo D. Brion (Brion) on December 14, Rule 8, Section 1 of JBC-009 in JBC No. 2016-1, or the Revised Rules of the Judicial and Bar
2016 and December 29, 2016, respectively. The JBC then claims that it had no choice but to Council; and (2) the removal of incumbent Senior Associate Justices of the Supreme Court as
submit two separate short lists for said vacancies in the Court because there were two sets of consultants of the JBC, referred to in pages 35 to 40 of the Decision. And as a consequence,
applicants for the same, i.e., there were 14 applicants for the seat vacated by Justice Perez the Court excuse the JBC from filing the required comment on the said matters.9
and 17 applicants for the seat vacated by Justice Brion.
II
The JBC further contends that since each vacancy creates discrete and possibly unique THE RULING OF THE COURT
situations, there can be no general rule against clustering. Submitting separate, independent
short lists for each vacancy is the only way for the JBC to observe the constitutional There is no legal or factual basis for the ponente to inhibit herself from the instant case.
standards of (a) one list for every vacancy, and (b) choosing candidates of competence,
independence, probity, and integrity for every such vacancy.
The Motion for Inhibition of the Ponente filed by the JBC is denied.

It is also the asseveration of the JBC that it did not encroach on the President's power to
The present Motion for Inhibition has failed to comply with Rule 8, Section 2 of the Internal
appoint members of the Judiciary. The JBC alleges that its individual Members gave several
Rules of the Supreme Court,10 which requires that "[a] motion for inhibition must be in
reasons why there was an apparent indication of seniority assignments in the six short lists
writing and under oath and shall state the grounds therefor." Yet, even if technical rules are
for the six vacancies for Sandiganbayan Associate Justice, particularly: (a) The JBC can best
relaxed herein, there is still no valid ground for the inhibition of the ponente.
perform its job by indicating who are stronger candidates by giving higher priority to those in
the lower-numbered list; (b) The indication could head off the confusion encountered in Re:
Seniority Among the Four Most Recent Appointments to the Position of Associate Justices of There is no ground11 for the mandatory inhibition of the ponente from the case at bar.
the Court of Appeals;8 and (c) The numbering of the lists from 16th to 21st had nothing to do
with seniority in the Sandiganbayan, but was only an ordinal designation of the cluster to The ponente has absolutely no personal interest in this case. The ponente is not a counsel,
which the candidates were included. partner, or member of a law firm that is or was the counsel in the case; the ponente or her
spouse, parent, or child has no pecuniary interest in the case; and the ponente is not related
The JBC ends with a reiteration of the need for the ponente to inhibit herself from the instant to any of the parties in the case within the sixth degree of consanguinity or affinity, or to an
case as she appears to harbor hostility possibly arising from the termination of her JBC attorney or any member of a law firm who is counsel of record in the case within the fourth
consultancy. degree of consanguinity or affinity.

The prayer of the JBC in its Motion for Reconsideration-in-Intervention reads: The ponente is also not privy to any proceeding in which the JBC discussed and decided to
adopt the unprecedented method of clustering the nominees for the six simultaneous
vacancies for Sandiganbayan Associate Justice into six separate short lists, one for every
IN VIEW OF THE FOREGOING, it is respectfully prayed that JBC's Motion for Reconsideration-
vacancy. The ponente does not know when, how, and why the JBC adopted the clustering
in-Intervention, Motion for Intervention and Motion for Reconsideration with Motion for
method of nomination for appellate courts and even the Supreme Court.
Inhibition of Justice Teresita J. Leonardo-De Castro of the JBC be granted and/or given due
course and that:
With due respect to Chief Justice Sereno, it appears that when the JBC would deliberate on
highly contentious, sensitive, and important issues, it was her policy as Chairperson of the
JBC to hold executive sessions, which excluded the Supreme Court consultants. At the JBC
110

meeting held on October 26, 2015, Chief Justice Sereno immediately mentioned at the Sandiganbayan Associate Justice during one of its executive sessions prior to October 26,
beginning of the deliberations "that, as the Council had always done in the past when there 2015.
are multiple vacancies, the voting would be on a per vacancy basis."12 Chief Justice Sereno
went on to state that the manner of voting had already been explained to the two ex Hence, even though the ponente and the other JBC consultants were admittedly present
officio members of the JBC who were not present during the meeting, namely, Senator during the meeting on October 26, 2015, the clustering of the nominees· for the six
Aquilino L. Pimentel III (Pimentel) and then Department of Justice (DOJ) Secretary ALFREDO simultaneous vacancies for Sandiganbayan Associate Justice was already fait
BENJAMIN S. Caguioa (Caguioa).13 Then the JBC immediately proceeded with the voting of accompli. Questions as to why and how the JBC came to agree on the clustering of nominees
nominees. This ponente was not consulted before the JBC decision to cluster nominees was were no longer on the table for discussion during the said meeting. As the minutes of the
arrived at and, therefore, she did not have the opportunity to study and submit her meeting on October 26, 2015 bear out, the JBC proceedings focused on the voting of
recommendation to the JBC on the clustering of nominees. nominees. It is stressed that the crucial issue in the present case pertains to the clustering of
nominees and not the nomination and qualifications of any of the nominees.
It is evident that prior to the meeting on October 26, 2015, the JBC had already reached an This ponente only had the opportunity to express her opinion on the issue of the clustering of
agreement on the procedure it would follow in voting for nominees, i.e., the clustering of the nominees for simultaneous and closely successive vacancies in collegiate courts in
nominees into six separate short lists, with one short list for each of the six newly-created her ponencia in the instant case. As a Member of the Supreme Court, the ponente is duty-
positions of Sandiganbayan Associate Justice. That Senator Pimentel and DOJ Secretary bound to render an opinion on a matter that has grave constitutional implications.
Caguioa, who were not present at the meeting on October 26, 2015, were informed
beforehand of the clustering of nominees only proves that the JBC had already agreed upon Neither is there any basis for the ponente 's voluntary inhibition from the case at bar. Other
the clustering of nominees prior to the said meeting. than the bare allegations of the JBC, there is no clear and convincing evidence of the ponente
's purported bias and prejudice, sufficient to overcome the presumption that she had
Notably, Chief Justice Sereno inaccurately claimed at the very start of the deliberations that rendered her assailed ponencia in the regular performance of her official and sacred duty of
the JBC had been voting on a per vacancy basis "as the Council had always done," giving the dispensing justice according to law and evidence and without fear or favor. Significant herein
impression that the JBC was merely following established procedure, when in truth, the is the following disquisition of the Court on voluntary inhibition of judges in Gochan v.
clustering of nominees for simultaneous or closely successive vacancies in a collegiate court Gochan,15 which is just as applicable to Supreme Court Justices:
was a new practice only adopted by the JBC under her Chairmanship. In the Decision dated
November 29, 2016, examples were already cited how, in previous years, the JBC submitted In a string of cases, the Supreme Court has said that bias and prejudice, to be considered
just one short list for simultaneous or closely successive vacancies in collegiate courts, valid reasons for the voluntary inhibition of judges, must be proved with clear and
including the Supreme Court, which will again be presented hereunder. convincing evidence. Bare allegations of their partiality will not suffice. It cannot be
presumed, especially if weighed against the sacred oaths of office of magistrates, requiring
As previously mentioned, it is the practice of the JBC to hold executive sessions when taking them to administer justice fairly and equitably - both to the poor and the rich, the weak and
up sensitive matters. The ponente and Associate Justice Velasco, incumbent Justices of the the strong, the lonely and the well-connected. (Emphasis supplied.)
Supreme Court and then JBC consultants, as well as other JBC consultants, were excluded
from such executive sessions. Consequently, the ponente and Associate Justice Velasco were Furthermore, it appears from the admitted lack of consensus on the part of the JBC Members
unable to participate in and were kept in the dark on JBC proceedings/decisions, particularly, as to the validity of the clustering shows that the conclusion reached by the ponente did not
on matters involving the nomination of candidates for vacancies in the appellate courts and arise from personal hostility but from her objective evaluation of the adverse constitutional
the Supreme Court. The matter of the nomination to the Supreme Court of now Supreme implications of the clustering of the nominees for the vacant posts of Sandiganbayan
Court Associate Justice FRANCIS H. Jardeleza (Jardeleza), which became the subject matter Associate Justice. It is unfortunate that the JBC stooped so low in casting aspersion on the
of Jardeleza v. Sereno,14 was taken up by the JBC in such an executive session. person of this ponente instead of focusing on sound legal arguments to support its position.
This ponente also does not know when and why the JBC deleted from JBC No. 2016-1, "The There is absolutely no factual basis for the uncalled for and unfair imputation of the JBC that
Revised Rules of the Judicial and Bar Council," what was Rule 8, Section 1 of JBC-009, the the ponente harbors personal hostility against the JBC presumably due to her removal as
former JBC Rules, which gave due weight and regard to the recommendees of the Supreme consultant. The ponente 's removal as consultant was the decision of Chief Justice Sereno,
Court for vacancies in the Court. The amendment of the JBC Rules could have been decided not the JBC. The ponente does not bear any personal grudge or resentment against the JBC
upon by the JBC when the ponente and Associate Justice Velasco were already relieved by for her removal as consultant. The ponente does not view Chief Justice Sereno's move as
Chief Justice Sereno of their duties as consultants of the JBC. The JBC could have similarly particularly directed against her as Associate Justice Velasco had been similarly removed as
taken up and decided upon the clustering of nominees for the six vacant posts of JBC consultant. The ponente has never been influenced by personal motive in deciding cases.
The ponente, instead, perceives the removal of incumbent Supreme Court Justices as
111

consultants of the JBC as an affront against the Supreme Court itself as an institution, since whether before the OP or the courts, the Court will nevertheless now allow the JBC
the evident intention of such move was to keep the Supreme Court in the dark on the intervention by considering the issues raised and arguments adduced in the Motion for
changes in rules and practices subsequently adopted by the JBC, which, to the mind of Reconsideration and Motion for Reconsideration-in-Intervention of the JBC in the interest of
this ponente, may adversely affect the exercise of the supervisory authority over the JBC substantial justice.
vested upon the Supreme Court by the Constitution.
Incidentally, it should be mentioned that the JBC reproaches the Court for supposedly
All the basic issues raised in the Petition had been thoroughly passed upon by the Court in hurrying the promulgation of its Decision on November 29, 2016 in anticipation of the
its Decision dated November 29, 2016 and the JBC already expressed its disinterest to impending vacancies in the Supreme Court due to the retirements of Associate Justices Perez
question President Aquino's "cross-reaching" in his appointment of the six new and Brion in December 2016. On the contrary, it appears that it was the JBC which hurriedly
Sandiganbayan Associate Justices. proceeded with the two separate publications on August 4, 2016 and August 18, 2016 of the
opening of the application for the aforesaid vacancies, respectively, which was contrary to
Even if the Motion for Reconsideration and Motion for Reconsideration-in-Intervention of previous practice, even while the issue of clustering was set to be decided by the Court.
the JBC, praying for the grant of its Motion for Intervention and the reversal of the Decision Moreover, a scrutiny of the process the Petition went through before its promulgation
dated November 29, 2016, are admitted into the records of this case and the issues raised negates any haste on the part of the Court. Bear in mind that the Petition at bar was filed on
and arguments adduced in the said two Motions are considered, there is no cogent reason to May 1 7, 2016 and petitioners' Reply, the last pleading allowed by the Court in this case, was
reverse the Decision dated November 29, 2016, particularly, in view of the admission of the filed on August 3, 2016. The draftponencia was calendared in the agenda of the Supreme
JBC of the lack of unanimity among the JBC members on the issue involving the clustering of Court en bane, called again, and deliberated upon several times before it was actually voted
nominees for the six simultaneous vacancies for Sandiganbayan Associate Justice and their upon on November 29, 2016. Indeed, it appears that it was the JBC which rushed to release
disinterest to question the "cross-reaching" or non-observance by President Aquino of such the separate short lists of nominees for the said Supreme Court vacancies despite knowing
clustering. the pendency of the instant Petition and its own filing of a Motion for Intervention herein on
November 28, 2016. The JBC went ahead with the release of separate short lists of nominees
for the posts of Supreme Court Associate Justice vice retired Associate Justices Perez and
Hence, the Court will no longer belabor the issue that only three JBC Members signed the
Brion on December 2, 2016 and December 9, 2016, respectively.
Motion for Intervention and Motion for Reconsideration and only four JBC Members signed
the Motion for Reconsideration-in-Intervention, as well as the fact that Chief Justice Sereno,
as Chairperson of the JBC, did not sign the three Motions. Even if the Court allows the intervention of the JBC, as it will now do in the case at bar, the
arguments of the JBC on the merits of the case fail to persuade the Court to reconsider its
Decision dated November 29, 2016.
To determine the legal personality of the signatories to file the JBC Motions, the Court has
accorded particular significance to who among the JBC Members signed the Motions and to
Chief Justice Sereno's act of administering the oath of office to three of the newly-appointed a. The clustering of nominees for the six vacancies in the Sandiganbayan by the JBC
Sandiganbayan Associate Justices, including respondent Econg, in resolving the pending impaired the President's power to appoint members of the Judiciary and to determine the
Motions of the JBC. However, in its Motion for Reconsideration-in-Intervention, the JBC now seniority of the newly-appointed Sandiganbayan Associate Justices.
reveals that not all of its Members agree on the official position to take in the case of
President Aquino's appointment of the six new Sandiganbayan Associate Justices. Thus, the Noteworthy is the fact that the Court unanimously voted that in this case of six simultaneous
position of the JBC on the clustering of the nominees for the six simultaneous vacancies for vacancies for Sandiganbayan Associate Justice, the JBC acted beyond its constitutional
Sandiganbayan Associate Justice rests on shaky legal ground. mandate in clustering the nominees into six separate short lists and President Aquino did not
commit grave abuse of discretion in disregarding the said clustering.
The JBC takes exception as to why the Court allowed the Petition at bar even when it did not
strictly comply with the rules, as it was filed beyond the 60-day period for filing a petition The JBC invokes its independence, discretion, and wisdom, and maintains that it deemed it
for certiorari. The Court, in its Decision dated November 29, 2016, gave consideration to wiser and more in accord with Article VIII, Section 9 of the 1987 Constitution to cluster the
petitioners' assertion that they had to secure first official copies of the six short lists before nominees for the six simultaneous vacancies for Sandiganbayan Associate Justice into six
they were able to confirm that President Aquino, in appointing the six new Sandiganbayan separate short lists. The independence and discretion of the JBC, however, is not without
Associate Justices, actually disregarded the clustering of nominees into six separate short limits. It cannot impair the President's power to appoint members of the Judiciary and his
lists. While the Court is hard-pressed to extend the same consideration to the JBC which statutory power to determine the seniority of the newly-appointed Sandiganbayan Associate
made no immediate effort to explain its failure to timely question or challenge the Justices. The Court cannot sustain the strained interpretation of Article VIII, Section 9 of the
appointments of respondents Econg and Musngi as Sandiganbayan Associate Justices
112

1987 Constitution espoused by the JBC, which ultimately curtailed the President's appointing 16th Associate Michael 21st Undersecretary for Special Concerns/
power. Justice Frederick L. Associate Chief of Staff of the Executive
Musngi Justice Secretary, OP, for 5 years
In its Decision dated November 29, 2016, the Court ruled that the clustering impinged upon
the President's appointing power in the following ways: The President's option for every 17th Associate Reynaldo P. 19th Undersecretary, Office of the
vacancy was limited to the five to seven nominees in each cluster. Once the President had Justice Cruz Associate Executive Secretary, OP, for 4-1/2
appointed a nominee from one cluster, then he was proscribed from considering the other Justice years
nominees in the same cluster for the other vacancies. All the nominees applied for and were
18th Associate Geraldine Faith 21st Former Judge, Regional Trial Court
found to be qualified for appointment to any of the vacant Associate Justice positions in the
Justice A. Econg Associate (RTC), Cebu, for 6 years Chief of
Sandiganbayan, but the JBC failed to explain why one nominee should be considered for
Justice Office, Philippine Mediation Center
appointment to the position assigned to one specific cluster only. Correspondingly, the
(PMC) Philippine Judicial Academy
nominees' chance for appointment was restricted to the consideration of the one cluster in
(PHILJA)
which they were included, even though they applied and were found to be qualified for all
the vacancies. Moreover, by designating the numerical order of the vacancies, the JBC 19th Associate Maria Theresa 17th Judge, RTC, Malolos Bulacan, for 10
established the seniority or order of preference of the new Sandiganbayan Associate Justices, Justice V. Mendoza- Associate years
a power which the law (Section 1, paragraph 3 of Presidential Decree No. 160616), rules (Rule Arcega Justice
II, Section 1 (b) of the Revised Internal Rules of the Sandiganbayan17), and jurisprudence (Re:
Seniority Among the Four Most Recent Appointments to the Position of Associate Justices of 20th Associate Karl B. Miranda 20th Assistant Solicitor General, Office of
the Court of Appeals18), vest exclusively upon the President. Justice Associate the Solicitor General (OSG), for 15
Justice years
b. Clustering can be used as a device to favor or prejudice a qualified nominee. 21st Associate Zaldy V. 18th Judicial Staff Head, Office of the Chief
Justice Trespeses Associate Justice (OCJ), Supreme Court, for 2
The JBC avers that it has no duty to increase the chances of appointment of every candidate Justice years
it has adjudged to have met the minimum qualifications for a judicial post. The Court does
not impose upon the JBC such duty, it only requires that the JBC gives all qualified
It would be safe to say that all the aforementioned six nominees were strong contenders. If
nominees fair and equal opportunity to be appointed. The clustering by the JBC of nominees
all six nominees were placed in the same cluster, then only one of them would have been
for simultaneous or closely successive vacancies in collegiate courts can actually be a device
actually appointed as Sandiganbayan Associate Justice and the other five could no longer be
to favor or prejudice a particular nominee. A favored nominee can be included in a cluster
considered for the still unfilled vacancies. If then Atty. Zaldy V. Trespeses (Trespeses), Judicial
with no other strong contender to ensure his/her appointment; or conversely, a nominee can
Staff Head, OCJ, was included in the cluster with respondent Econg, PHILJA Chief of Office for
be placed in a cluster with many strong contenders to minimize his/her chances of
PMC, and respondent Musngi, Undersecretary for Special Concerns and Chief of Staff of the
appointment.
Executive Secretary, OP, then he would have lesser chance of being appointed as he would
have to vie for a single vacancy with two other strong contenders; and only one of the three
Without casting aspersion or insinuating ulterior motive on the part of the JBC - which would would have been appointed. Evidently, the appointments to the six simultaneous vacancies
only be highly speculative on the part of the Court - hereunder are different scenarios, using for Sandiganbayan Associate Justice would have been different by simply jumbling the
the very same circumstances and nominees in this case, to illustrate how clustering could be clusters of nominees. Even if we go back in history, had the JBC clustered the nominees for
used to favor or prejudice a particular nominee and subtly influence President Aquino's the posts vacated by Supreme Court Associate Justices Leonardo A. Quisumbing
appointing power, had President Aquino faithfully observed the clustering. (Quisumbing) and Minita V. Chico-Nazario (Chico-Nazario), and if Associate Justices Perez and
Jose Catral Mendoza (Mendoza) were together in the same cluster, then only one of them
The six nominees actually appointed by President Aquino as Sandiganbayan Associate would have been appointed. Also, had the JBC clustered the nominees for the vacancies
Justices were the following: resulting from the retirements of Supreme Court Associate Justices Antonio Eduardo B.
Nachura (Nachura) and Conchita Carpio Morales (Carpio Morales), and if Associate Justices
Bienvenido L. Reyes (Reyes) and ESTELA M. Perlas-Bernabe (Perlas-Bernabe) were together in
VACANCY IN THE PERSON SHORT FORMER POSITION HELD the same cluster, then the appointment of one of them would have already excluded the
SANDIGANBAYAN APPOINTED LISTED FOR other.
113

c. There are no objective criteria, standards, or guidelines for the clustering of nominees by Motion for Reconsideration-in-Intervention for assigning a nominee to a particular
the JBC. cluster/vacancy. The Court highlights that without objective criteria, standards, or guidelines
in determining which nominees are to be included in which cluster, the clustering of
The problem is that the JBC has so far failed to present a legal, objective, and rational basis nominees for specific vacant posts seems to be at the very least, totally arbitrary. The lack of
for determining which nominee shall be included in a cluster. Simply saying that it is the such criteria, standards, or guidelines may open the clustering to manipulation to favor or
result of the deliberation and voting by the JBC for every vacancy is unsatisfactory. A review prejudice a qualified nominee.
of the voting patterns by the JBC Members for the six simultaneous vacancies for
Sandiganbayan Associate Justice only raises more questions and doubts than answers. It d. There is technically no clustering of nominees for first and second level trial courts.
would seem, to the casual observer, that the Chief Justice and the four regular JBC Members
exercised block voting most of the time. Out of the 89 candidates for the six vacancies, there The Court further points out that its Decision dated November 29, 2016 only discussed
were a total of 3 7 qualified nominees spread across six separate short lists. Out of the 37 vacancies in collegiate courts. The constant referral by the JBC to separate short lists of
qualified nominees, the Chief Justice and the four regular JBC Members coincidentally voted nominees for vacant judgeship posts in first and second level trial courts as proof of previous
for the same 28 nominees in precisely the same clusters, only varying by just one vote for clustering is inapt. The separate short lists in such situations are technically not clustering as
the other nine nominees. the vacancies happened and were announced at different times and candidates applied for
specific vacancies, based on the inherent differences in the location and jurisdiction of the
It is also interesting to note that all the nominees were listed only once in just one cluster, trial courts, as well as the qualifications of nominees to the same, hence, justifying a separate
and all the nominees subsequently appointed as Sandiganbayan Associate Justice were short list for each vacant post.
distributed among the different clusters, except only for respondents Econg and Musngi. Was
this by chance or was there already an agreement among the Chief Justice and the regular e. While clustering of nominees was observed in the nominations for vacancies in the Court
JBC Members to limit the nomination of a candidate to a specific cluster for one specific of Appeals in 2015, it escaped scrutiny as the appointments to said vacancies were not
vacancy, thus, excluding the same candidate from again being nominated in a different challenged before the Court.
cluster for another vacancy? It is understandable that the Chief Justice and the four regular
JBC Members would agree on whom to nominate because their nominations were based on
As an example of previous clustering in a collegiate court, the JBC attached to its Motion for
the qualifications of the candidates. What is difficult to comprehend is how they determined
Reconsideration-in-Intervention a transmittal letter dated August 1 7, 2015 of the JBC
the distribution of the nominees to the different clusters in the absence of any criteria or
addressed to President Aquino, which divided the nominees into four clusters for the four
standard to be observed in the clustering of nominees. This was never explained by the JBC
vacancies for Court of Appeals Associate Justice. The JBC contends that during the
in any of its Motions even when the issue of clustering is vital to this case. Resultantly, the
deliberations on said nominations, the ponente and Supreme Court Associate Justice Velasco
Court also asks why were respondents Econg and Musngi nominated in a single cluster? And
were both present as JBC consultants but did not raise any objection.
why was then Atty. Trespeses not included in the same cluster as respondents Econg and
Musngi, or the clusters of then Undersecretary Reynaldo P. Cruz, RTC Judge Maria Theresa V.
Mendoza-Arcega, or Assistant Solicitor General Karl B. Miranda? Furthermore, what criteria While it may be true that the JBC already observed clustering in 2015, it is still considered a
was used when Chief Justice Sereno and the other four regular JBC Members voted for then relatively new practice, adopted only under Chief Justice Sereno's Chairmanship of the JBC.
Atty. Trespeses for only one particular cluster, i.e., for the 18th Sandiganbayan Associate The clustering then escaped scrutiny as no party questioned the appointments to the said
Justice, and nowhere else? Atty. Trespeses did not receive any vote in the other clusters vacancies. The view of the consultants was also not solicited or requested by the JBC. The
except for the lone vote for him of an ex officio JBC Member for the vacancy for the Court now observes that the vacancies for Court of Appeals Associate Justice in 2015 were
21st Sandiganbayan Associate Justice. not all simultaneous or closely successive, most of which occurring months apart, specifically,
vice the late Associate Justice Michael P. Elbinias who passed away on November 20, 2014;
vice retired Associate Justice Rebecca De Guia-Salvador, who opted for early retirement
The Court emphasizes that the requirements and qualifications, as well as the powers, duties,
effective on January 31, 2015; vice Associate Justice Hakim S. Abdulwahid, who compulsorily
and responsibilities are the same for all vacant posts in a collegiate court, such as the
retired on June 12, 2015; and vice Associate Justice Isaias P. Dicdican who compulsorily
Sandiganbayan; and if an individual is found to be qualified for one vacancy, then he/she is
retired on July 4, 2015. Even so, the JBC published a single announcement for all four
found to be qualified for all the other vacancies - there are no distinctions among the vacant
vacancies on March 15, 2015, with the same deadlines for submission of applications and
posts. It is improbable that the nominees expressed their desire to be appointed to only a
supporting documents. This is in stark contrast to the two-week interval between the
specific vacant position and not the other vacant positions in the same collegiate court, when
compulsory retirements of Supreme Court Associate Justices Perez and Brion on December
neither the Constitution nor the law provides a specific designation or distinctive description
14, 2016 and December 29, 2016, respectively, for which the JBC still made separate
for each vacant position in the collegiate court. The JBC did not cite any cogent reason in its
publications, required submission of separate applications, separately processed the
114

applications, and submitted separate short lists. Additionally, it is noteworthy that the There had been no similar problems in the past because the JBC jointly announced
nominations for the four vacant posts of Court of Appeals Associate Justice were contained in simultaneous or closely successive vacancies in the Supreme Court in a single publication,
a single letter dated August 1 7, 2015, addressed to President Aquino, through then invited the filing by a candidate of a single application for all the vacancies on the same
Executive Secretary Paquito N. Ochoa, Jr., whereas in the case of the Sandiganbayan, the JBC deadline, jointly processed all applications, and submitted a single list of qualified nominees
submitted six separate letters, all dated October 26, 2015, transmitting one short list for each to the President, thus, resulting in a simple, inexpensive, and efficient process of nomination.
of the six vacancies. The separate letters of transmittal further reinforce the intention of the Such was the case when the JBC announced the two vacancies for Supreme Court Associate
JBC to prevent the President from "cross-reaching" or disregarding the clustering of Justice following the retirements of Associate Justices Quisumbing and Chico-Nazario in
nominees for the six vacancies for Sandiganbayan Associate Justice and, thus, unduly limit 2009. Pertinent portions of the JBC publication are reproduced below:
the President's exercise of his power to appoint members of the Judiciary.
The Judicial and Bar Council (JBC) announces the opening, for application or
f. The separate short lists for the current vacancies in the Supreme Court are not in issue in recommendation, of the: two (2) forthcoming vacant positions of ASSOCIATE JUSTICE OF THE
this case, but has been brought up by the JBC in its Motion for Reconsideration-in- SUPREME COURT vice Hon. Leonardo A. Quisumbing and Hon. Minita V. Chico-Nazario,
Intervention. who will compulsorily retire on 6 November and 5 December 2009, respectively, x x x

The Court takes the occasion herein to clarify that the application of its ruling in the Decision Applications or recommendation for the two (2) positions in the Supreme Court must be
dated November 29, 2017 to the situation involving closely successive vacancies in a submitted not later than 28 September 2009 (Monday) x x x to the JBC Secretariat, 2nd Flr.
collegiate court may be properly addressed in an actual case which squarely raises the issue. Centennial Bldg., Supreme Court, Padre Faura St., Manila (Tel. No. 552-9512; Fax No. 552-
It also bears to stress that the current vacancies in the Supreme Court as a result of the 9607; email address jbc _supreme court@yahoo.com.ph or jbc@sc.judiciary.gov.ph).
compulsory retirements of Associate Justices Perez and Brion are not in issue in this case, Applicants or recommendees must submit six (6) copies of the following:
but has been brought to the fore by the JBC itself in its Motion for Reconsideration-in-
Intervention. Therefore, the Court will refrain from making any pronouncements on the The JBC, then headed by Supreme Court Chief Justice Reynato S. Puno, submitted to
separate short lists of nominees submitted by the JBC to President Rodrigo Roa Duterte President Gloria Macapagal-Arroyo (Macapagal-Arroyo) a single short list dated November
(Duterte) on December 2, 2016 and December 9, 2016 so as not to preempt the President's 29, 2009 with a total of six nominees for the two vacancies for Supreme Court Associate
decision on how to treat the separate short lists of nominees for the two current vacancies in Justice, from which, President Macapagal-Arroyo appointed Associate Justices Perez and
the Supreme Court. The Court will only address the statements made by the JBC in relation to Mendoza.
said short lists by reciting some relevant historical facts relating to the filling-up of previous
vacancies in the Supreme Court.
The JBC again announced the two vacancies for Supreme Court Associate Justice due to the
retirements of Associate Justices Nachura and Carpio Morales, thus:
The JBC avers that it had no choice but to submit separate short lists of nominees to
President Duterte for the vacancies for Supreme Court Associate Justice vice Associate
The Judicial and Bar Council (JBC) announces the opening, for application or
Justices Perez and Brion, who retired on December 14, 2016 and December 29, 2016,
recommendation, of the following positions:
respectively, because there were different sets of applicants for each, with 14 applicants for
the seat vacated by Associate Justice Perez and 17 applicants for the seat vacated by
Associate Justice Brion. The situation is the own doing of the JBC, as the JBC announced the 1. ASSOCIATE JUSTICE OF THE SUPREME COURT (vice Hon. Antonio Eduardo B. Nachura and
expected vacancies left by the compulsory retirements of Associate Justices Perez and Brion, Hon. Conchita Carpio Morales, who will compulsorily retire on 13 and 19 June 2011,
which were merely two weeks apart, through two separately paid publications on August 4, respectively);
2016 and August 18, 2016, respectively, in newspapers of general circulation; invited the
filing of separate applications for the vacancies with different deadlines; and separately Applications or recommendations for vacancies in nos. 1-3 must be filed on or before 28
processed the applications of candidates to the said vacancies. The JBC would inevitably end March 2011 (Monday) x x x to the JBC Secretariat, 2nd Flr. Centennial Bldg., Supreme Court,
up with two different sets of nominees, one set for the position vacated by Justice Perez and Padre Faura St., Manila (Tel. No. 552-9512; Fax No. 552-9598; email
another set for that vacated by Justice Brion, notwithstanding that the JBC undeniably found address jbcsupremecourt@yahoo.com.ph. Those who applied before these vacancies were
all nominees in both sets to be qualified to be appointed as Associate Justice of the Supreme declared open must manifest in writing their interest on or before the said deadline. In case
Court, as they all garnered at least four votes. of recommendations, the recommendees must signify their acceptance either in the
recommendation letter itself or in a separate document.
115

New applicants or recommendees for positions in the appellate courts must submit the submitted by the JBC to President Aquino explicitly stated that the nominees were for the
following on or before 4 April 2011 (Monday) x x x: Sixteenth (16th), Seventeenth (17th), Eighteenth (18th), Nineteenth (19th), Twentieth (20th),
and Twenty-First (2 Pt) Sandiganbayan Associate Justice, respectively; and on the faces of
The single short list dated June 21, 2011, submitted by the JBC, under the Chairmanship of said short lists, it could only mean that President Aquino was to make the appointments in
Supreme Court Chief Justice Renato C. Corona, presented, for President Aquino's the order of seniority pre-determined by the JBC, and that nominees who applied for any of
consideration, six nominees for the two vacant posts of Supreme Court Associate Justice, the vacant positions, requiring the same qualifications, were deemed to be qualified to be
with President Aquino subsequently appointing Associate Justices Reyes and Perlas-Bernabe. considered for appointment only to the one vacant position to which his/her cluster was
specifically assigned. Whatever the intentions of the individual JBC Members were, they
cannot go against what has been clearly established by law,19 rules,20 and jurisprudence.21 In
How the new procedure adopted by the JBC of submitting two separate lists of nominees will
its Decision dated November 29, 2016, the Court already adjudged that:
also affect the seniority of the two Supreme Court Associate Justices to be appointed to the
current vacancies is another issue that may arise because of the new JBC procedure. Unlike
the present two separate lists of nominees specifying the vacant post to which they are Evidently, based on law, rules, and jurisprudence, the numerical order of the Sandiganbayan
short-listed for appointment, the short list of nominees submitted by the JBC before did not Associate Justices cannot be determined until their actual appointment by the President.
identify to which of the vacant positions, when there are more than one existing vacancies, a
qualified candidate is nominated to as there was only one list of nominees for all vacancies It also bears to point out that part of the President's power to appoint members of a
submitted to the President. Correspondingly, the appointment papers issued by the collegiate court, such as the Sandiganbayan, is the power to determine the seniority or order
President, as in the cases of Supreme Court Associate Justices Perez, Mendoza, Reyes, and of preference of such newly appointed members by controlling the date and order of
Perlas-Bernabe, did not specify the particular vacant post to which each of them was issuance of said members' appointment or commission papers. By already designating the
appointed. The appointment papers of the afore-named Supreme Court Associate Justices numerical order of the vacancies, the JBC would be establishing the seniority or order of
were all similarly worded as follows: preference of the new Sandiganbayan Associate Justices even before their appointment by
the President and, thus, unduly arrogating unto itself a vital part of the President's power of
Pursuant to the provisions of existing laws, you are hereby appointed ASSOCIATE JUSTICE OF appointment.22
THE SUPREME COURT.
It is also not clear to the Court how, as the JBC avowed in its Motion for Reconsideration, the
By virtue hereof, you may qualify and enter upon the performance of the duties and clustering of nominees for simultaneous vacancies in collegiate courts into separate short
functions of the office, furnishing this Office and the Civil Service Commission with copies of lists can rid the appointment process to the Judiciary of political pressure; or conversely, how
your Oath of Office. the previous practice of submitting a single list of nominees to the President for
simultaneous vacancies in collegiate courts, requiring the same qualifications, made the
appointment process more susceptible to political pressure. The 1987 Constitution itself, by
As earlier stated, the Court makes no ruling on the above-mentioned divergence between
creating the JBC and requiring that the President can only appoint judges and Justices from
the procedures in the nomination for existing vacancies in the Supreme Court followed by
the nominees submitted by the JBC, already sets in place the mechanism to protect the
the JBC before and by the present JBC as it may be premature to do so and may prejudge
appointment process from political pressure. By arbitrarily clustering the nominees for
whatever action President Duterte may take on the two separate short lists of nominees
appointment to the six simultaneous vacancies for Sandiganbayan Associate Justice into
for the current Supreme Court vacancies which were submitted by the JBC.
separate short lists, the JBC influenced the appointment process and encroached on the
President's power to appoint members of the Judiciary and determine seniority in the said
g. The designation by the JBC of numbers to the vacant Sandiganbayan Associate Justice court, beyond its mandate under the 1987 Constitution. As the Court pronounced in its
posts encroached on the President's power to determine the seniority of the justices Decision dated November 29, 2016, the power to recommend of the JBC cannot be used to
appointed to the said court. restrict or limit the President's power to appoint as the latter's prerogative to choose
someone whom he/she considers worth appointing to the vacancy in the Judiciary is still
The JBC contends in its Motion for Reconsideration-in-Intervention that its individual paramount. As long as in the end, the President appoints someone nominated by the JBC,
members have different reasons for designating numbers to the vacant Sandiganbayan the appointment is valid, and he, not the JBC, determines the seniority of appointees to a
Associate Justice posts. The varying reason/s of each individual JBC Members raises the collegiate court.
concern whether they each fully appreciated the constitutional and legal consequences of
their act, i.e., that it encroached on the power, solely vested in the President, to determine Finally, the JBC maintains that it is not bound by the Decision dated November 29, 2016 of
the seniority of the justices appointed to a collegiate court. Each of the six short lists the Court in this case on the ground that it is not a party herein. The JBC prays in its Motion
116

for Reconsideration and Motion for Reconsideration-in-Intervention, among other reliefs and being given the opportunity to submit its comment and be heard on the same. The
remedies, for the Court to reverse its ruling in the Decision dated November 29, 2016 administrative matter was already raffled to another ponente, thus, any incident concerning
denying the Motion for Intervention of the JBC in the present case. However, the Court has the same should be consolidated in the said administrative matter.
now practically allowed the intervention of the JBC in this case, by taking into consideration
the issues raised and arguments adduced in its Motion for Reconsideration and Motion for Regarding the Separate Opinion of Associate Justice Caguioa, it must be pointed out that he
Reconsideration-in-Intervention, but which the Court found to be unmeritorious. has conceded that the President did not commit an unconstitutional act in "disregarding the
clustering done by the JBC" when he chose Associate Justices of the Sandiganbayan "outside"
To recapitulate, the Petition at bar challenged President Aquino's appointment of of the "clustered" lists provided by the JBC.
respondents Econg and Musngi as Sandiganbayan Associate Justices, which disregarded the
clustering by the JBC of the nominees for the six simultaneous vacancies in said collegiate WHEREFORE, premises considered, except for its motion/prayer for intervention, which the
court into six separate short lists. The Court ultimately decreed in its Decision dated Court has now granted, the Motion for Reconsideration (with Motion for the Inhibition of
November 29, 2016 that: the Ponente) and the Motion for Reconsideration-in-Intervention (Of the Decision dated 29
November 2016) of the Judicial and Bar Council are DENIED for lack of merit.
President Aquino validly exercised his discretionary power to appoint members of the
Judiciary when he disregarded the clustering of nominees into six separate shortlists for the Nota bene: The Court has agreed not to issue a ruling herein on the separate short lists of
vacancies for the 16th, 17th, 18th, 19th, 20th, and 21st Sandiganbayan Associate Justices. nominees submitted by the Judicial and Bar Council to President Rodrigo Roa Duterte for the
President Aquino merely maintained the well-established practice, consistent with the present vacancies in the Supreme Court resulting from the compulsory retirements of
paramount Presidential constitutional prerogative, to appoint the six new Sandiganbayan Associate Justices Jose P. Perez and Arturo D. Brion because these were not in issue nor
Associate Justices from the 37 qualified nominees, as if embodied in one JBC list. This does deliberated upon in this case, and in order not to preempt the decision the President may
not violate Article VIII, Section 9 of the 1987 Constitution which requires the President to take on the said separate short lists in the exercise of his power to appoint members of the
appoint from a list of at least three nominees submitted by the JBC for every vacancy. To Judiciary under the Constitution.
meet the minimum requirement under said constitutional provision of three nominees per
vacancy, there should at least be 18 nominees from the JBC for the six vacancies for
SO ORDERED.
Sandiganbayan Associate Justice; but the minimum requirement was even exceeded herein
because the JBC submitted for the President's consideration a total of 37 qualified nominees.
All the six newly appointed Sandiganbayan Associate Justices met the requirement of G.R. No. 224302 November 29, 2016
nomination by the JBC under Article VIII, Section 9 of the 1987 Constitution. Hence, the
appointments of respondents Musngi and Econg, as well as the other four new Hon. Philip A. Aguinaldo v. His Excellency President Benigno Simeon C. Aquino III
Sandiganbayan Associate Justices, are valid and do not suffer from any constitutional
infirmity.23 FACTS:

The declaration of the Court that the clustering of nominees by the JBC for the simultaneous The Judicial & Bar Council submitted several lists of nominees for possible appointees to the
vacancies that occurred by the creation of six new positions of Associate Justice of the vacancies in the Sandiganbayan. The nominees were clustered into several lists like for
Sandiganbayan is unconstitutional was only incidental to its ruling that President Aquino is example 17th, 18th, 19th, 20th, 21st with five (5) nominees for each cluster. President
not bound by such clustering in making his appointments to the vacant Sandiganbayan Aquino appointed Justice’s to the vacant positions, but did not pick the appointees from the
Associate Justice posts. Other than said declaration, the Court did not require the JBC to do clusters concerned but appointed justices from one cluster to another position. Petitioners
or to refrain from doing something insofar as the issue of clustering of the nominees to the
who were listed in the cluster for the 17th Justice questioned the appointments.
then six vacant posts of Sandiganbayan Associate Justice was concerned.
They contended that the President could only choose one nominee from each of the six
As for the other new rules and practices adopted by the JBC which the Court has taken
separate shortlists for each specific vacancy and no other and the appointment made in
cognizance of and docketed as a separate administrative matter (viz., Item No. 2: the
deviation of this procedure is a violation of the Constitution.
deletion or non-inclusion in JBC No. 2016-1, or the Revised Rules of the Judicial and Bar
Council, of Rule 8, Section 1 of JBC-009; and Item No. 3: the removal of incumbent Senior
Associate Justices of the Supreme Court as consultants of the Judicial and Bar Council, ISSUES: Whether or not the clustering done by the JBC is constitutional.
referred to in pages 45 to 51 of the Decision dated November 29, 2016), the JBC is actually
117

HELD: Section 1 of JBC-009; and Item No. 3: the removal of incumbent Senior Associate Justices of
the Supreme Court as consultants of the Judicial and Bar Council, referred to in pages 35 to
The Supreme Court (SC) has affirmed its decision striking down as unconstitutional the 40 of this Decision. The Court finally DIRECTS the Judicial and Bar Council to file its comment
clustering of shortlisted nominees made by the Judicial and Bar Council (JBC) last year in on said Item Nos. 2 and 3 within thirty (30) days from notice.
connection with Sandiganbayan vacancies. The power to recommend of the JBC cannot be
used to restrict or limit the President's power to appoint as the latter's prerogative to choose SO ORDERED.”
someone whom he/she considers worth appointing to the vacancy in the Judiciary is still
paramount. As long as in the end, the President appoints someone nominated by the JBC, SUBJECTS/DOCTRINES/DIGEST:
the appointment is valid. President Aquino was not obliged to appoint one new
“Without notice, warning, or explanation to the Supreme Court En Banc, Chief Justice Sereno
Sandiganbayan Associate Justice from each of the six shortlists submitted by the JBC,
recently unceremoniously relieved Supreme Court Associate Justices Presbiterio J. Velasco,
especially when the clustering of nominees into the six shortlists encroached on President
Jr. and Teresita J. LeonardoDe Castro as JBC consultants, and in their stead, the Chief Justice
Aquino's power to appoint members of the Judiciary from all those whom the JBC had
appointed retired Chief Justices Hilario G. Davide, Jr., Artemio V. Panganiban, and Reynato S.
considered to be qualified for the same positions of Sandiganbayan Associate Justice.
Puno as JBC consultants. The experience and wisdom of the three retired Chief Justices are
The JBC, in sorting the qualified nominees into six clusters, one for every vacancy, could undisputed. However, practicality and prudence also dictate that incumbent Associate
influence the appointment process beyond its constitutional mandate of recommending Justices of the Court should be retained as JBC consultants since their interest in the Judiciary
qualified nominees to the President. Clustering impinges upon the President's power of is real, actual, and direct. Incumbent Associate Justices of the Court are aware of the present
appointment, as well as restricts the chances for appointment of the qualified nominees, state, needs, and concerns of the Judiciary, and consultants from the Court, even if they have
because (1) the President's option for every vacancy is limited to the five to seven nominees no right to vote, have served, from the organization of the JBC, as the only link to the
in the cluster; and (2) once the President has appointed from one cluster, then he is supervisory authority of the Court over the JBC under the 1987 Constitution. . . . . .”
proscribed from considering the other nominees in the same cluster for the other vacancies.
“These changes in settled rules and practices recently adopted by the JBC under Chief Justice
The said limitations are utterly without legal basis and in contravention of the President's Sereno are disconcerting. There appears to be a systematic move by the JBC, under Chief
appointing power. In view of the foregoing, President Aquino validly exercised his Justice Sereno to arrogate to itself more power and influence than it is actually granted by
discretionary power to appoint members of the Judiciary when he disregarded the clustering the Constitution and this Court, and at the same time, to ease out the Court from any
of nominees into six separate shortlists for the different vacancies. This did not violate Article legitimate participation in the nomination process for vacancies in the Judiciary, specifically,
VIII, Section 9 of the 1987 Constitution which requires the President to appoint from a list of in the Supreme Court. This behooves the Court, through the exercise of its power of
at least three nominees submitted by the JBC for every vacancy. supervision over the JBC, to take a closer look into the new rules and practices of the JBC and
ensure that these are in accord with the 1987 Constitution, the pertinent laws, and the
DISPOSITIVE: governmental policies of transparency and accountability in the nomination process for
vacancies in the Judiciary.”
“WHEREFORE, premises considered, the Court DISMISSES the instant Petition for Quo
Warranto and Certiorari and Prohibition for lack of merit. The Court DECLARES the clustering “The Court had recognized that “[ s ]upervision is not a meaningless thing. It is an active
of nominees by the Judicial and Bar Council UNCONSTITUTIONAL, and the appointments of power. It is certainly not without limitation, but it at least implies authority to inquire into
respondents Associate Justices Michael Frederick L. Musngi and Geraldine Faith A. Econg, facts and conditions in order to render the power real and effective.”
together with the four other newly-appointed Associate Justices of the Sandiganbayan, as
VALID. The Court further DENIES the Motion for Intervention of the Judicial and Bar Council “In the exercise of its power of supervision over the JBC, the Court shall take up the
in the present Petition, but ORDERS the Clerk of Court En Banc to docket as a separate aforementioned Item Nos. 2 and 3 as a separate administrative matter and direct the JBC to
administrative matter the new rules and practices of the Judicial and Bar Council which the file its comment on the same.”
Court took cognizance of in the preceding discussion as Item No. 2: the deletion or non-
inclusion in JBC No. 2016-1, or the Revised Rules of the Judicial and Bar Council, of Rule 8,
118

G.R. No 224302 O The JBC also points out that the acts invoked against the JBC are based on practice or
custom, but "practice, no matter how long continued, cannot give rise to any vested right."
TOPIC: Power of Appointment
O The JBC, as a constitutional body, enjoys independence, and as such, it may change its
CASE LAW/ DOCTRINE: practice from time to time in accordance with its wisdom.

The power to recommend of the JBC cannot be used to restrict or limit the President's power • The JBC maintains that it did not exceed its authority and, in fact, it only faithfully complied
to appoint as the latter's prerogative to choose someone whom he/she considers worth with the literal language of Article VIII, Section 9 of the 1987 Constitution, when it prepared
appointing to the vacancy in the Judiciary is still paramount. As long as in the end, the six short lists for the six vacancies in the Sandiganbayan.
President appoints someone nominated by the JBC, the appointment is valid, and he, not the
JBC, determines the seniority of appointees to a collegiate court. • The JBC renounces any duty to increase the chances of appointment of every candidate it
adjudged to have met the minimum qualifications. It asserts that while there might have
Emergency Recit: been favorable experiences with the past practice of submitting long consolidated short lists,
past practices cannot be used as a source of rights and obligations to override the duty of the
The JBC filed a Motion for Reconsideration-in-Intervention (Of the Decision dated 29
JBC to observe a straightforward application of the Constitution.
November 2016) on February 6, 2017. It alleged that the clustering of nominees was in
accordance with the Constitution. The Supreme Court reiterated its 2016 decision that as • The JBC further contends that since each vacancy creates discrete and possibly unique
long as in the end, the President appoints someone nominated by the JBC, the appointment situations, there can be no general rule against clustering. Submitting separate, independent
is valid, and he, not the JBC, determines the seniority of appointees to a collegiate court. short lists for each vacancy is the only way for the JBC to observe the constitutional
standards of (a) one list for every vacancy, and (b) choosing candidates of competence,
FACTS:
independence, probity, and integrity for every such vacancy.
• The JBC successively filed a Motion for Reconsideration (with Motion for the Inhibition of
• It is also the asseveration of the JBC that it did not encroach on the President's power to
the Ponente) on December 27, 2016 and a Motion for Reconsideration-in-Intervention (Of
appoint members of the Judiciary.
the Decision dated 29 November 2016) on February 6, 2017.
• The numbering of the lists from 16th to 21st had nothing to do with seniority in the
O The immediate concern of the JBC is this Court's pronouncement that the former's act of
Sandiganbayan, but was only an ordinal designation of the cluster to which the candidates
submitting six lists for six vacancies was unconstitutional.
were included.
• The JBC asserts that in submitting six short lists for six vacancies, it was only acting in
ISSUE(S): Whether or not the Decision dated 29 November 2016, which ruled the clustering
accordance with the clear and unambiguous mandate of Article VIII, Section 9[3] of the 1987
of nominees by the Judicial and Bar Council as unconstitional, must be reversed.
Constitution for the JBC to submit a list for every vacancy.
RATIO:
• According to the JBC, its new practice of "clustering," in fact, is more in accord with the
purpose of the JBC to rid the appointment process to the Judiciary from political pressure as No. The Supreme Court sustained the Nov. 2016 Decision.
the President has to choose only from the nominees for one particular vacancy.
• The clustering of nominees for the six vacancies in the Sandiganbayan by the JBC impaired
O Otherwise, the President can choose whom he pleases, and thereby completely disregard the President's power to appoint members of the Judiciary and to determine the seniority of
the purpose for the creation of the JBC. the newly-appointed Sandiganbayan Associate Justices.

O The JBC clarifies that it numbered the vacancies, not to influence the order of precedence, O The Court ruled that the clustering impinged upon the President's appointing power in the
but for practical reasons, i.e., to distinguish one list from the others and to avoid confusion. following ways:
119

O The President's option for every vacancy was limited to the five to seven nominees in each appointed members by controlling the date and order of issuance of said members'
cluster. Once the President had appointed a nominee from one cluster, then he was appointment or commission papers.
proscribed from considering the other nominees in the same cluster for the other vacancies.
• By already designating the numerical order of the vacancies, the JBC would be establishing
O All the nominees applied for and were found to be qualified for appointment to any of the the seniority or order of preference of the new Sandiganbayan Associate Justices even before
vacant Associate Justice positions in the Sandiganbayan, but the JBC failed to explain why their appointment by the President and, thus, unduly arrogating unto itself a vital part of the
one nominee should be considered for appointment to the position assigned to one specific President's power of appointment.
cluster only.

O Correspondingly, the nominees' chance for appointment was restricted to the


consideration of the one cluster in which they were included, even though they applied and
were found to be qualified for all the vacancies.

O Moreover, by designating the numerical order of the vacancies, the JBC established the
seniority or order of preference of the new Sandiganbayan Associate Justices, a power which
the law (Section 1, paragraph 3 of Presidential Decree No. 1606), rules (Rule II, Section 1(b) of
the Revised Internal Rules of the Sandiganbayan), and jurisprudence (Re: Seniority Among
the Four Most Recent Appointments to the Position of Associate Justices of the Court of
Appeals), vest exclusively upon the President.

• Clustering can be used as a device to favor or prejudice a qualified nominee.

O The clustering by the JBC of nominees for simultaneous or closely successive vacancies in
collegiate courts can actually be a device to favor or prejudice a particular nominee. A
favored nominee can be included in a cluster with no other strong contender to ensure
his/her appointment; or conversely, a nominee can be placed in a cluster with many strong
contenders to minimize his/her chances of appointment.

• There are no objective criteria, standards, or guidelines for the clustering of nominees by
the JBC.

O The JBC has so far failed to present a legal, objective, and rational basis for determining
which nominee shall be included in a cluster. Simply saying that it is the result of the
deliberation and voting by the JBC for every vacancy is unsatisfactory.

• The designation by the JBC of numbers to the vacant Sandiganbayan Associate Justice posts
encroached on the President's power to determine the seniority of the justices appointed to
the said court.

• Part of the President's power to appoint members of a collegiate court, such as the
Sandiganbayan, is the power to determine the seniority or order of preference of such newly
120

EN BANC The ban on midnight appointments in Section 15, Article VII of the 1987 Constitution reads:

G.R. No. 203372 June 16, 2015 Two months immediately before the next presidential elections and up to the end of his
term, a President or Acting President shall not make appointments, except temporary
ATTY. CHELOY E. VELICARIA-GARAFIL, Petitioner, appointments to executive positions when continued vacancies therein will prejudice public
vs. service or endanger public safety.
OFFICE OF THE PRESIDENT and HON. SOLICITOR GENERAL JOSE ANSELMO I.
CADIZ, Respondents. Thus, for purposes of the 2010 elections, 10 March 2010 was the cutoff date for valid
appointments and the next day, 11 March 2010, was the start of the ban on midnight
DECISION appointments. Section 15, Article VII of the 1987 Constitution recognizes as an exception to
the ban on midnight appointments only "temporary appointments to executive positions
when continued vacancies therein will prejudice public service or endanger public safety."
CARPIO, J.:
None of the petitioners claim that their appointments fall under this exception.

The present consolidated cases involve four petitions: G.R. No. 203372 with Atty. Cheloy E.
Appointments
Velicaria-Garafil (Atty. Velicaria-Garafil), who was appointed State Solicitor II at the Office of
the Solicitor General (OSG), as petitioner; G.R. No. 206290 with Atty. Dindo G. Venturanza
(Atty. Venturanza), who was appointed Prosecutor IV (City Prosecutor) of Quezon City, as G.R. No. 203372
petitioner; G.R. No. 209138 with Irma A. Villanueva (Villanueva), who was appointed
Administrator for Visayas of the Board of Administrators of the Cooperative Development The paper evidencing Atty. Velicaria-Garafil's appointment as State Solicitor II at the OSG was
Authority (CDA), and Francisca B. Rosquita (Rosquita), who was appointed Commissioner of dated 5 March 2010.13 There was a transmittal letter dated 8 March 2010 of the
the National Commission of Indigenous Peoples (NCIP), as petitioners; and G.R. No. 212030 appointment paper from the Office of the President (OP), but this transmittal letter was
with Atty. Eddie U. Tamondong (Atty. Tamondong), who was appointed member of the received by the Malacañang Records Office (MRO) only on 13 May 2010. There was no
Board of Directors of the Subic Bay Metropolitan Authority (SBMA), as petitioner. All indication as to the OSG's date of receipt of the appointment paper. On 19 March 2010, the
petitions question the constitutionality of Executive Order No. 2 (EO 2) for being inconsistent OSG's Human Resources Department called up Atty. Velicaria-Garafil to schedule her oath-
with Section 15, Article VII of the 1987 Constitution. taking. Atty. Velicaria-Garafil took her oath of office as State Solicitor II on 22 March 2010
and assumed her position on 6 April 2010.
Petitioners seek the :reversal of the separate Decisions of the Court of Appeals (CA) that
dismissed their petitions and upheld the constitutionality of EO 2. G.R. No. 203372 filed by G.R. No. 206290
Atty. Velicaria-Garafil is a Petition for Review on Certiorari,1 assailing the Decision2 dated 31
August 2012 of the CA in CA-G.R. SP No. 123662. G.R. No. 206290 filed by Atty. Venturanza is The paper evidencing Atty. Venturanza's appointment as Prosecutor IV (City Prosecutor) of
a Petition for Review on Certiorari,3 assailing the Decision4 dated 31 August 2012 and Quezon City was dated 23 February 2010.14 It is apparent, however, that it was only on 12
Resolution5 dated 12 March 2013 of the CA in CA-G.R. SP No. 123659. G.R. No. 209138 filed March 2010 that the OP, in a letter dated 9 March 2010, transmitted Atty. Venturanza's
by Villanueva and Rosquita is a Petition for Certiorari,6 seeking to nullify the Decision7 dated appointment paper to then Department of Justice (DOJ) Secretary Alberto C. Agra.15 During
28 August 2013 of the CA in CA-G.R. SP Nos. 123662, 123663, and 123664.8 Villanueva and the period between 23 February and 12 March 2010, Atty. Venturanza, upon verbal advice
Rosquita filed a Petition-in-Intervention in the consolidated cases before the CA. G.R. No. from Malacañang of his promotion but without an official copy of his appointment paper,
212030 is a Petition for Review on Certiorari,9 assailing the Decision10 dated 31 August 2012 secured clearances from the Civil Service Commission (CSC),16 Sandiganbayan,17 and the
of the CA in CAG.R. SP No. 123664 and Resolution11 dated 7 April 2014 of the CA in CAG.R. SP DOJ.18 Atty. Venturanza took his oath of office on 15 March 2010, and assumed office on the
Nos. 123662, 123663, and 123664.12 same day.

Facts of the Cases G.R. No. 209138

Prior to the conduct of the May 2010 elections, then President Gloria Macapagal-Arroyo The paper evidencing Villanueva's appointment as Administrator for Visayas of the Board of
(President Macapagal-Arroyo) issued more than 800 appointments to various positions in Administrators of the CDA was dated 3 March 2010.19 There was no transmittal letter of the
several government offices. appointment paper from the OP. Villanueva took her oath of office on 13 April 2010.
121

The paper evidencing Rosquita's appointment as Commissioner, representing Region I and The entirety of EO 2 reads:
the Cordilleras, of the NCIP was dated 5 March 2010.20 Like Villanueva, there was no
transmittal letter of the appointment paper from the OP. Rosquita took her oath of office on EXECUTIVE ORDER NO. 2
18 March 2010. G.R. No. 212030
RECALLING, WITHDRAWING, AND REVOKING APPOINTMENTS ISSUED BY THE PREVIOUS
The paper evidencing Atty. Tamondong's appointment as member, representing the private ADMINISTRATION IN VIOLATION OF THE CONSTITUTIONAL BAN ON MIDNIGHT
sector, of the SBMA Board of Directors was dated 1 March 2010.21 Atty. Tamondong APPOINTMENTS, AND FOR OTHER PURPOSES.
admitted that the appointment paper was received by the Office of the SBMA Chair on 25
March 201022 and that he took his oath of office on the same day.23 He took another oath of
WHEREAS, Sec. 15, Article VII of the 1987 Constitution provides that "Two months
office on 6 July 2010 as "an act of extra caution because of the rising crescendo of noise from
immediately before the next presidential elections and up to the end of his term, a President
the new political mandarins against the so-called 'midnight appointments."'24
or Acting President shall not make appointments, except temporary appointments to
executive positions when continued vacancies therein will prejudice public service or
To summarize, the pertinent dates for each petitioner are as follows: endanger public safety."; WHEREAS, in the case of "In re: Appointments dated March 30,
1998 of Hon. Mateo Valenzuela and Hon. Vallarta as Judges of the Regional Trial Court of
G.R. No. Date of Date of Date of Date of Assumption Branch 62 of Bago City and Branch 24 of Cabanatuan City, respectively" (A.M. No. 98-5-01-SC
Appointment Transmittal Receipt by Oath of of Office Nov. 9, 1998), the Supreme Court interpreted this provision to mean that the President is
Letter Letter MRO Office neither required to make appointments nor allowed to do so during the two months
immediately before the next presidential elections and up to the end of her term. The only
203372 known exceptions to this prohibition are (1) temporary appointments in the executive
(Atty. 13 May 22 March positions when continued vacancies will prejudice public service or endanger public safety
5 March 2010 8 March 2010 6 April 2010
Velicaria- 2010 2010 and in the light of the recent Supreme Court decision in the case of De Castro, et al. vs. JBC
Garafil) and PGMA, G.R. No. 191002, 17 March 2010, (2) appointments to the Judiciary;

206290
23 February 12 March 15 March 15 March WHEREAS, Section 261 of the Omnibus Election Code provides that:
(Atty. 9 March 2010
2010 2010 2010 2010
Venturanza)
"Section 261. Prohibited Acts.-The following shall be guilty of an election offense:
209138 4 May 13 April
3 March 2010
(Villanueva) 2010 2010
(g) Appointments of new employees, creation of new position,
209138 13 May 18 March promotion, or giving salary increases. - During the period of forty-five
5 March 2010 days before a regular election and thirty days before a special election.
(Rosquita) 2010 2010

25 March (1) Any head, official or appointing officer of a government office, agency
212030
2010 and or instrumentality, whether national or local, including government-
(Atty. 1 March 2010
6 July owned or controlled corporations, who appoints or hires any new
Tamondong)
2010 employee, whether provisional, temporary or casual, or creates and fills
any new position, except upon prior authority to the Commission. The
Issuance of EO 2 Commission shall not grant the authority sought unless it is satisfied that
the position to be filled is essential to the proper functioning of the office
or agency concerned, and that the position shall not be filled in a manner
On 30 June 2010, President Benigno S. Aquino III (President Aquino) took his oath of office as
that may influence the election.
President of the Republic of the Philippines. On 30 July 2010, President Aquino issued EO 2
recalling, withdrawing, and revoking appointments issued by President Macapagal-Arroyo
which violated the constitutional ban on midnight appointments. As an exception to the foregoing provisions, a new employee may be
appointed in the case of urgent need:
122

Provided, however, that notice of the appointment shall be given to the SECTION 2. Recall, Withdraw, and Revocation of Midnight Appointments. Midnight
Commission within three days from the date of the appointment. Any appointments, as defined under Section 1, are hereby recalled, withdrawn, and revoked. The
appointment or hiring in violation of this provision shall be null and void. positions covered or otherwise affected are hereby declared vacant.

(2) Any government official who promotes or gives any increase of salary SECTION 3. Temporary designations. - When necessary to maintain efficiency in public service
or remuneration or privilege to any government official or employee, and ensure the continuity of government operations, the Executive Secretary may designate
including those in government-owned or controlled corporations."; an officer-in-charge (OIC) to perform the duties and discharge the responsibilities of any of
those whose appointment has been recalled, until the replacement of the OIC has been
WHEREAS, it appears on record that a number of appointments were made on or about 10 appointed and qualified.
March 2010 in complete disregard of the intent and spirit of the constitutional ban on
midnight appointment and which deprives the new administration of the power to make its SECTION 4. Repealing Clause. - All executive issuances, orders, rules and regulations or part
own appointment; thereof inconsistent with the provisions of this Executive Order are hereby repealed or
modified accordingly.
WHEREAS, based on established jurisprudence, an appointment is deemed complete only
upon acceptance of the appointee; SECTION 5. Separability Clause. - If any section or provision of this executive order shall be
declared unconstitutional or invalid, the other sections or provision not affected thereby
WHEREAS, in order to strengthen the civil service system, it is necessary to uphold the shall remain in full force and effect.
principle that appointments to the civil service must be made on the basis of merit and
fitness, it is imperative to recall, withdraw, and revoke all appointments made in violation of SECTION 6. Effectivity. - This Executive order shall take effect immediately.
the letter and spirit of the law;
DONE in the City of Manila, this 30th day of July, in the year Two Thousand and Ten.
NOW, THEREFORE, I, BENIGNO S. AQUINO III, by virtue of the powers vested in me by the
Constitution as President of the Philippines, do hereby order and direct that: By the President:

SECTION 1. Midnight Appointments Defined. - The following appointments made by the (Sgd.) PAQUITO N. OCHOA, JR.
former President and other appointing authorities in departments, agencies, offices, and Executive Secretary25
instrumentalities, including government-owned or controlled corporations, shall be
considered as midnight appointments:
(Sgd.) BENIGNO S. AQUINO III

(a) Those made on or after March 11, 2010, including all appointments bearing
Effect of the Issuance of EO 2
dates prior to March 11, 2010 where the appointee has accepted, or taken his
oath, or assumed public office on or after March 11, 2010, except temporary
appointments in the executive positions when continued vacancies will prejudice G.R. No. 203372
public service or endanger public safety as may be determined by the appointing
authority. On 5 August 2010, Jose Anselmo Cadiz assumed office as Solicitor General (Sol. Gen. Cadiz).
On 6 August 2010, Sol. Gen. Cadiz instructed a Senior Assistant Solicitor General to inform
(b) Those made prior to March 11, 2010, but to take effect after said date or the officers and employees affected by EO 2 that they were terminated from service effective
appointments to office that would be vacant only after March 11, 2010. the next day.

(c) Appointments and promotions made during the period of 45 days prior to the Atty. Velicaria-Garafil reported for work on 9 August 2010 without any knowledge of her
May 10, 2010 elections in violation of Section 261 of the Omnibus Election Code. termination. She was made to return the office-issued laptop and cellphone, and was told
that her salary ceased as of 7 August 2010. On 12 August 2010, Atty. Velicaria-Garafil was
informed that her former secretary at the OSG received a copy of a memorandum on her
behalf. The memorandum, dated 9 August 2010, bore the subject "Implementation of
123

Executive Order No. 2 dated 30 July 2010" and was addressed to the OSG's Director of Referral to CA
Finance and Management Service.
There were several petitions31 and motions for intervention32 that challenged the
Atty. Velicaria-Garafil filed a petition for certiorari (G.R. No. 193327) before this Court on 1 constitutionality of EO 2.
September 2010. The petition prayed for the nullification of EO 2, and for her reinstatement
as State Solicitor II without loss of seniority, rights and privileges, and with full backwages On 31 January 2012, this Court issued a Resolution referring the petitions, motions for
from the time that her salary was withheld.26 intervention, as well as various letters, to the CA for further proceedings, including the
reception and assessment of the evidence from all parties. We defined the issues as follows:
G.R. No. 206290
1. Whether the appointments of the petitioners and intervenors were midnight
On 1 September 2010, Atty. Venturanza received via facsimile transmission an undated copy appointments within the coverage of EO 2;
of DOJ Order No. 556. DOJ Order No. 556, issued by DOJ Secretary Leila M. De Lima (Sec. De
Lima), designated Senior Deputy State Prosecutor Richard Anthony D. Fadullon (Pros. 2. Whether all midnight appointments, including those of petitioners and
Fadullon) as Officer-in-Charge of the Office of the City Prosecutor in Quezon City. In a letter intervenors, were invalid;
to Sec. De Lima dated 15 September 2010, Atty. Venturanza asked for clarification of his
status, duties, and functions since DOJ Order No. 556 did not address the same. Atty.
3 . Whether the appointments of the petitioners and intervenors were made with
Venturanza also asked for a status quo ante order to prevent Pros. Fadullon ·from usurping
undue haste, hurried maneuvers, for partisan reasons, and not in accordance with
the position and functions of the City Prosecutor of Quezon City. Atty. Venturanza also wrote
good faith; and
a letter to President Aquino on the same day, and sought reaffirmation of his promotion as
City Prosecutor of Quezon City.
4. Whether EO 2 violated the Civil Service Rules on Appointment.33
On 6 October 2010, Atty. Venturanza received a letter dated 25 August 2010 from Sec. De
Lima which directed him to relinquish the office to which he was appointed, and to cease This Court gave the CA the authority to resolve all pending matters and applications, and to
from performing its functions. decide the issues as if these cases were originally filed with the CA.

Atty. Venturanza filed a Petition for Certiorari, Prohibition, Mandamus with Urgent Prayer for Rulings of the CA
Status Quo Ante Order, Temporary Restraining Order and/or Preliminary Mandatory
Injunction (G.R. No. 193 867) before this Court on 14 October 2010.27 Even though the same issues were raised in the different petitions, the CA promulgated
separate Decisions for the petitions. The CA consistently ruled that EO 2 is constitutional. The
G.R. No. 209138 CA, however, issued different rulings as to the evaluation of the circumstances of petitioners'
appointments. In the cases of Attys. Velicaria-Garafil and Venturanza, the CA stated that the
OP should consider the circumstances of their appointments. In the cases of Villanueva,
The OP withheld the salaries of Villanueva and Rosquita on the basis of EO 2. On 3 August
Rosquita, and Atty. Tamondong, the CA explicitly stated that · the revocation of their
2010, Villanueva and Rosquita sought to intervene in G.R. No. 192991.28 On 1 October 2010,
appointments was proper because they were midnight appointees.
Executive Secretary Paquito N. Ochoa, Jr. revoked Rosquita's appointment as NCIP
Commissioner.29 On 13 October 2010, Villanueva and Rosquita notified this Court that they
wanted to intervene in Atty. Tamondong's petition (G.R. No. 192987) instead. G.R. No. 203372 (CA-G.R. SP No. 123662)

G.R. No. 212030 The CA promulgated its Decision in CA-G.R. SP No. 123662 on 31 August 2012. The CA ruled
that EO 2 is not unconstitutional. However, the CA relied on Sales v. Carreon34 in ruling that
the OP should evaluate whether Atty. Velicaria-Garafil's appointment had extenuating
Atty. Tamondong was removed from the SBMA Board of Directors on 30 July 2010. He filed a
circumstances that might make it fall outside the ambit of EO 2.
petition for prohibition, declaratory relief and preliminary injunction with prayer for
temporary restraining order (G.R. No. 192987) before this Court on 9 August 2010. The
petition prayed for the prohibition of the implementation of EO 2, the declaration of his The dispositive portion of the CA's Decision reads:
appointment as legal, and the declaration of EO 2 as unconstitutional.30
124

WHEREFORE, the petition for certiorari and mandamus [is] DENIED. ponencia and the dissent both agree that all the appointments in these cases are midnight
appointments in violation of Section 15, Article VII of the 1987 Constitution.
Executive Order No. 2, dated July 30, 2010, is NOT unconstitutional.
Constitutionality of EO 2
The issue on whether or not to uphold petitioner's appointment as State Solicitor II at the
OSG is hereby referred to the Office of the President which has the sole authority and Based on prevailing jurisprudence, appointment to a government post is a process that takes
discretion to pass upon the same. several steps to complete. Any valid appointment, including one made under the exception
provided in Section 15, Article VII of the 1987 Constitution, must consist of the President
SO ORDERED.35 signing an appointee's appointment paper to a vacant office, the official transmittal of the
appointment paper (preferably through the MRO), receipt of the appointment paper by the
appointee, and acceptance of the appointment by the appointee evidenced by his or her
G.R. No. 212030 (CA-G.R. SP No. 123664)
oath of office or his or her assumption to office.

On 31 August 2012, the CA promulgated its Decision in CA-G.R. SP No. 123664. The
Aytona v. Castillo (Aytona)42 is the basis for Section 15, Article VII of the 1987 Constitution.
dispositive portion reads as follows:
Aytona defined "midnight or last minute" appointments for Philippine
jurisprudence.1âwphi1 President Carlos P. Garcia submitted on 29 December 1961, his last
WHEREFORE, premises considered, the instant Petition is hereby DISMISSED. Executive Order day in office, 350 appointments, including that of Dominador R. Aytona for Central Bank
No. 2 is hereby declared NOT UNCONSTITUTIONAL. Accordingly, the revocation of Atty. Eddie Governor. President Diosdado P. Macapagal assumed office on 30 December 1961, and
Tamondong's appointment as Director of Subic Bay Metropolitan Authority is VALID for being issued on 31 December 1961 Administrative Order No. 2 recalling, withdrawing, and
a midnight appointment. cancelling all appointments made by President Garcia after 13 December 1961 (President
Macapagal's proclamation date). President Macapagal appointed Andres V. Castillo as
SO ORDERED.39 Central Bank Governor on 1 January 1962. This Court dismissed Aytona's quo warranto
proceeding against Castillo, and upheld Administrative Order No. 2's cancellation of the
The Issues for Resolution "midnight or last minute" appointments. We wrote:

We resolve the following issues in these petitions: (1) whether petitioners' appointments x x x But the issuance of 350 appointments in one night and the planned induction of almost
violate Section 15, Article VII of the 1987 Constitution, and (2) whether EO 2 is constitutional. all of them a few hours before the inauguration of the new President may, with some reason,
Ruling of the Court be regarded by the latter as an abuse of Presidential prerogatives, the steps taken being
apparently a mere partisan effort to fill all vacant positions irrespective of fitness and other
conditions, and thereby to deprive the new administration of an opportunity to make the
The petitions have no merit. All of petitioners' appointments are midnight appointments and corresponding appointments.
are void for violation of Section 15, Article VII of the 1987 Constitution. EO 2 is constitutional.
Villanueva and Rosquita, petitioners in G.R. No. 209138, did not appeal the CA's ruling under
Rule 45, but instead filed a petition for certiorari under Rule 65. This procedural error alone x x x Now it is hard to believe that in signing 350 appointments in one night, President Garcia
warrants an outright dismissal of G.R. No. 209138. Even if it were correctly filed under Rule exercised such "double care" which was required and expected of him; and therefore, there
45, the petition should still be dismissed for being filed out of time.40 There was also no seems to be force to the contention that these appointments fall beyond the intent and spirit
explanation as to why they did not file a motion for reconsideration of the CA's Decision. of the constitutional provision granting to the Executive authority to issue ad interim
Midnight Appointments appointments.

This ponencia and the dissent both agree that the facts in all these cases show that "none of Under the circumstances above described, what with the separation of powers, this Court
the petitioners have shown that their appointment papers (and transmittal letters) have resolves that it must decline to disregard the Presidential .Administrative Order No. 2,
been issued (and released) before the ban."41The dates of receipt by the MRO, which in cancelling such "midnight" or "last-minute" appointments.
these cases are the only reliable evidence of actual transmittal of the appointment papers by
President Macapagal-Arroyo, are dates clearly falling during the appointment ban. Thus, this Of course the Court is . aware of many precedents to the effect that once an appointment
has been issued, it cannot be reconsidered, specially where the appointee has qualified. But
125

none of them refer to mass ad interim appointments (three hundred and fifty), issued in the acceptance are made after the ban. However, the dissent holds that an appointment is valid,
last hours of an outgoing Chief Executive, in a setting similar to that outlined herein. On the or "efficacious," if the appointment and transmittal are made before the ban even if the
other hand, the authorities admit of exceptional circumstances justifying revocation and if acceptance is made after the ban. In short, the dissent allows an appointment to take effect
any circumstances justify revocation, those described herein should fit the exception. during the ban, as long as the President signed and transmitted the appointment before the
ban, even if the appointee never received the appointment paper before the ban and
Incidentally, it should be stated that the underlying reason for denying the power to revoke accepted the appointment only during the ban.
after the appointee has qualified is the latter's equitable rights. Yet it is doubtful if such
equity might be successfully set up in the present situation, considering the rush conditional The dissent's view will lead to glaring absurdities. Allowing the dissent's proposal that an
appointments, hurried maneuvers and other happenings detracting from that degree of good appointment is complete merely upon the signing of an appointment paper and its
faith, morality and propriety which form the basic foundation of claims to equitable relief. transmittal, excluding the appointee's acceptance from the appointment process, will lead to
The appointees, it might be argued, wittingly or unwittingly cooperated with the stratagem the absurdity that, in case of non-acceptance, the position is considered occupied and
to beat the deadline, whatever the resultant consequences to the dignity and efficiency of nobody else may be appointed to it. Moreover, an incumbent public official, appointed to
the public service. Needless to say, there are instances wherein not only strict legality, but another public office by the President, will automatically be deemed to occupy the new
also fairness, justice and righteousness should be taken into account.43 public office and to have automatically resigned from his first office upon transmittal of his
appointment paper, even if he refuses to accept the new appointment. This will result in
During the deliberations for the 1987 Constitution, then Constitutional Commissioner (now chaos in public service.
retired Supreme Court Chief Justice) Hilario G. Davide, Jr. referred to this Court's ruling in
Aytona and stated that his proposal seeks to prevent a President, whose term is about to Even worse, a President who is unhappy with an incumbent public official can simply appoint
end, from preempting his successor by appointing his own people to sensitive positions. him to another public office, effectively removing him from his first office without due
process. The mere transmittal of his appointment paper will remove the public official from
MR. DAVIDE: The idea of the proposal is that about the end of the term of the President, he office without due process and even without cause, in violation of the Constitution.
may prolong his rule indirectly by appointing people to these sensitive positions, like the
commissions, the Ombudsman, the judiciary, so he could perpetuate himself in power even The dissent's proferred excuse (that the appointee is not alluded to in Section 15, Article VII)
beyond his term of office; therefore foreclosing the right of his successor to make for its rejection of "acceptance by the appointee" as an integral part of the appointment
appointments to these positions. We should realize that the term of the President is six years process ignores the reason for the limitation of the President's power to appoint, which is .to
and under what we had voted on, there is no reelection for him. Yet he can continue to rule prevent the outgoing President from continuing to rule the country indirectly after the end of
the country through appointments made about the end of his term to these sensitive his term. The 1986 Constitutional Commission installed a definite cut-off date as an objective
positions.44 and unbiased marker against which this once-in-every-six-years prohibition should be
measured.
The 1986 Constitutional Commission put a definite period, or an empirical value, on Aytona's
intangible "stratagem to beat the deadline," and also on the act of "preempting the The dissent's assertion that appointment should be viewed in its narrow sense (and is not a
President's successor," which shows a lack of "good faith, morality and propriety." Subject to process) only during the prohibited period is selective and time-based, and ignores well-
only one exception, appointments made during this period are thus automatically prohibited settled jurisprudence. For purposes of complying with the time limit imposed by the
under the Constitution, regardless of the appointee's qualifications or even of the President's appointment ban, the dissent' s position cuts short the appointment process to the signing of
motives. The period for prohibited appointments covers two months before the elections the appointment paper and its transmittal, excluding the receipt of the appointment paper
until the end of the President's term. The Constitution, with a specific exception, ended the and acceptance of the appointment by the appointee.
President's power to appoint "two months immediately before the next presidential
elections." For an appointment to be valid, it must be made outside of the prohibited period The President exercises only one kind of appointing power. There is no need to differentiate
or, failing that, fall under the specified exception. the exercise of the President's appointing power outside, just before, or during the
appointment ban. The Constitution allows the President to exercise the power of
The dissent insists that, during the prohibited period, an appointment should be viewed in its appointment during the period not covered by the appointment ban, and disallows (subject
"narrow sense." In its narrow sense, an appointment is not a process, but is only an to an exception) the President from exercising the power of appointment during the period
"executive act that the President unequivocally exercises pursuant to his discretion."45 The covered by the appointment ban. The concurrence of all steps in the appointment process is
dissent makes acceptance of the appointment inconsequential. The dissent holds that an admittedly required for appointments outside the appointment ban. There is no justification
appointment is void if the appointment is made before the ban but the transmittal and whatsoever to remove acceptance as a requirement in the appointment process for
126

appointments just before the start of the appointment ban, or during the appointment ban because their acceptance is not required during the ban. Adoption by this Court of the
in appointments falling within the exception. The existence of the appointment ban makes dissent's singular exception will certainly wreak havoc on the civil service.
no difference in the power of the President to appoint; it is still the same power to appoint.
In fact, considering the purpose of the appointment ban, the concurrence of all steps in the The following elements should always concur in the making of a valid (which should be
appointment process must be strictly applied on appointments made just before or during understood as both complete and effective) appointment: (1) authority to appoint and
the appointment ban. evidence of the exercise of the authority; (2) transmittal of the appointment paper and
evidence of the transmittal; (3) a vacant position at the time of appointment; and (4) receipt
In attempting to extricate itself from the obvious consequences of its selective application, of the appointment paper and acceptance of the appointment by the appointee who
the dissent glaringly contradicts itself: possesses all the qualifications and none of the disqualifications. The concurrence of all these
elements should always apply, regardless of when the appointment is made, whether
Thus, an acceptance is still necessary in order for the appointee to validly assume his post outside, just before, or during the appointment ban. These steps in the appointment process
and discharge the functions of his new office, and thus make the appointment effective. should always concur and operate as a single process. There is no valid appointment if the
There can never be an instance where the appointment of an incumbent will automatically process lacks even one step. And, unlike the dissent's proposal, there is no need to further
result in his resignation from his present post and his subsequent assumption of his new distinguish between an effective and an ineffective appointment when an appointment is
position; or where the President can simply remove an incumbent from his current office by valid.
appointing him to another one. I stress that acceptance through oath or any positive act is
still indispensable before any assumption of office may occur.46 (Emphasis added) Appointing Authority

The dissent proposes that this Court ignore well-settled jurisprudence during the The President's exercise of his power to appoint officials is provided for in the Constitution
appointment ban, but apply the same jurisprudence outside of the appointment ban. and laws.48 Discretion is an integral part in the exercise of the power of
appointment.49 Considering that appointment calls for a selection, the appointing power
[T]he well-settled rule in our jurisprudence, that an appointment is a process that begins with necessarily exercises a discretion. According to Woodbury, J., "the choice of a person to fill
the selection by the appointing power and ends with acceptance of the appointment by the an office constitutes the essence of his appointment," and Mr. Justice Malcolm adds that an
appointee, stands. As early as the 1949 case of Lacson v. Romero, this Court laid down the "[a]ppointment to office is intrinsically an executive act involving the exercise of discretion."
rule that acceptance by the appointee is the last act needed to make an appointment In Pamantasan ng Lungsod ng Maynila v. Intermediate Appellate Court we held:
complete. The Court reiterated this rule in the 1989 case of Javier v. Reyes. In the 1996 case
of Garces v. Court of Appeals, this Court emphasized that acceptance by the appointee is The power to appoint is, in essence, discretionary. The appointing power has the right of
indispensable to complete an appointment. The 1999 case of Bermudez v. Executive choice which he may exercise freely according to his judgment, deciding for himself who is
Secretary, cited in the ponencia, affirms this standing rule in our jurisdiction, to wit: best qualified among those who have the necessary qualifications and eligibilities. It is a
prerogative of the appointing power x x x x
"The appointment is deemed complete once the last act required of the appointing authority
has been complied with and its acceptance thereafter by the appointee in order to render it Indeed, the power of choice is the heart of the power to appoint. Appointment involves an
effective."47 exercise of discretion of whom to appoint; it is not a ministerial act of issuing appointment
papers to the appointee. In other words, the choice of the appointee is a fundamental
The dissent's assertion creates a singular exception to the well-settled doctrine that component of the appointing power.
appointment is a process that begins with the signing of the appointment paper, followed by
the transmittal and receipt of the appointment paper, and becomes complete with the Hence, when Congress clothes the President with the power to appoint an officer, it
acceptance of the appointment. The dissent makes the singular exception that during the (Congress) cannot at the same time limit the choice of the President to only one candidate.
constitutionally mandated ban on appointments, acceptance is not necessary to complete Once the power of appointment is conferred on the President, such conferment necessarily
the appointment. The dissent gives no reason why this Court should make such singular carries the discretion of whom to appoint. Even on the pretext of prescribing the
exception, which is contrary to the express provision of the Constitution prohibiting the qualifications of the officer, Congress may not abuse such power as to divest the appointing
President from making appointments during the ban. The dissent's singular exception will authority, directly or indirectly, of his discretion to pick his own choice. Consequently, when
allow the President, during the ban on appointments, to remove from office incumbents the qualifications prescribed by Congress can only be met by one individual, such enactment
without cause by simply appointing them to another office and transmitting the appointment effectively eliminates the discretion of the appointing power to choose and constitutes an
papers the day before the ban begins, appointments that the incumbents cannot refuse irregular restriction on the power of appointment.50
127

Transmittal i. Prepare and submit to the approving authority, periodic disposition schedules of
non-current records which have no historical, legal and/or claim value.
It is not enough that the President signs the appointment paper. There should be evidence
that the President intended the appointment paper to be issued. It could happen that an j. With the approval of the Executive Secretary, assist other offices in the
appointment paper may be dated and signed by the President months before the installation or improvement of their records management system; and
appointment ban, but never left his locked drawer for the entirety of his term. Release of the
appointment paper through the MRO is an unequivocal act that signifies the President's k. Give instructions or deliver lectures and conduct practical training to in-service
intent of its issuance. trainees from other offices and to students from educational institutions on
records management.51
The MRO was created by Memorandum Order No. 1, Series of 1958, Governing the
Organization and Functions of the Executive Office and General Matters of Procedure The Records Division was elevated to an Office in 1975, with the addition of the
Therein. Initially called the Records Division, the MRO functioned as an administrative unit of following functions:
the Executive Office. Memorandum Order No. 1 assigned the following functions:
1. Maintain and control vital documents and essential records to support
a. Receive, record and screen all incoming correspondence, telegrams, documents the functions of the OP in its day to day activities;
and papers, and
2. Monitor the flow of communications' from their time of receipt up to
(1) Forward those of a personal and unofficial nature to the President's their dispatch;
Private Office; and
3. Service the documentary, information and reference requirements of
(2) Distribute those requiring action within the Office or requiring staff top management and action officers of the OP, and the reference and
work prior to presentation to the President to the appropriate units research needs of other government agencies and the general public;
within the Office.
4. Ensure the proper storage, maintenance, protection and preservation
b. Follow up on correspondence forwarded to entities outside the Office to assure of vital and presidential documents, and the prompt disposal of obsolete
that prompt replies are made and copies thereof furnished the Office. and valueless records;

c. Dispatch outgoing correspondence and telegrams. 5. Effect the prompt publication/dissemination of laws, presidential
issuances and classified documents;
d. Have custody of records of the Office, except personal papers of the President,
and keep them in such condition as to meet the documentary and reference 6. Provide computerized integrated records management support
requirements of the Office. services for easy reference and retrieval of data and information; and

e. Keep and maintain a filing and records system for acts, memoranda, orders, 7. To be able to represent the OP and OP officials in response to
circulars, correspondence and other documents affecting the Office for ready Subpoena Duces Tecum and Testificandum served by courts and other
reference and use. investigating bodies.52

f. Issue certified true copies of documents on file in the Division m accordance with For purposes of verification of the appointment paper's existence and authenticity, the
prevailing standard operating procedure. appointment paper must bear the security marks (i.e., handwritten signature of the
President, bar code, etc.) and must be accompanied by a transmittal letter from the MRO.
g. Keep a separate record of communications or documents of confidential nature.
The testimony of Mr. Mariani to Dimaandal, Director IV of the MRO, underscores the
h. Have custody of the Great Seal of the Republic of the Philippines. purpose of the release of papers through his office.
128

Q: What are the functions of the MRO? A: The MRO Service Guide was issued pursuant to Memorandum Circular No. 35, Series of
2003 and Memorandum Circular No. 133, Series of 2007.
A: The MRO is mandated under Memorandum Order No. 1, series of 1958 to (1) receive,
record, and screen all incoming correspondence, telegrams, documents, and papers; (2) Q: Do you exercise any discretion in the release of documents forwarded to the MRO for
follow up on correspondence forwarded to entities outside the Office of the President ("OP") transmittal to various offices?
to assure that prompt replies are made and copies thereof furnished the OP; (3) timely
dispatch all outgoing documents and correspondence; (4) have custody of records of the OP, A: No. We are mandated to immediately release all documents and correspondence
except personal papers of the President, and keep them in such condition as to meet the forwarded to us for transmittal.
documentary and reference requirements of the Office; (5) keep and maintain a filing and
records system for Acts, Memoranda, Orders, Circulars, correspondence, and other pertinent
Q: If a document is forwarded by the OES to the MRO today, when is it officially released by
documents for ready reference and use; ( 6) issue certified copies of documents on file as
the MRO to the department or agency concerned?
requested and in accordance with prevailing standard operating procedures; (7) maintain
and control vital documents and essential records to support the OP in its day-to-day
activities; (8) monitor the flow of communications from the time of receipt up to their A: The document is released within the day by the MRO if the addressee is within Metro
dispatch; and (9) other related functions. Manila. For example, in the case of the appointment paper of Dindo Venturanza, the OES
forwarded to the MRO on March 12, 2010 his original appointment paper dated February 23,
2010 and the transmittal letter dated March 9, 2010 prepared by the OES. The MRO released
Q: As you previously mentioned, the MRO is the custodian of all documents emanating from
his appointment paper on the same day or on March 12, 2010, and was also received by the
Malacañang pursuant to its mandate under Memorandum Order No. 1, Series of 1958. Is the
DOJ on March 12, 2010 as shown by the delivery receipt.
MRO required to follow a specific procedure in dispatching outgoing documents?

Q: What is the effect if a document is released by an office or department within Malacañan


A: Yes.
without going through the MRO?

Q: Is this procedure observed for the release of an appointment paper signed by the
A: If a document does not pass through the MRO contrary to established procedure, the
President? A: Yes. It is observed for the release of the original copy of the appointment paper
MRO cannot issue a certified true copy of the same because as far as the MRO is concerned,
signed by the President.
it does not exist in our official records, hence, not an official document from the Malacañang.
There is no way of verifying the document's existence and authenticity unless the document
Q: Can you briefly illustrate the procedure for the release of the original copy of the is on file with the MRO even if the person who claims to have in his possession a genuine
appointment paper signed by the President? document furnished to him personally by the President. As a matter of fact, it is only the
MRO which is authorized to issue certified true copies of documents emanating from
A: After an appointment paper is signed by the President, the Office of the Executive Malacañan being the official custodian and central repository of said documents. Not even
Secretary (OES) forwards the appointment paper bearing the stamp mark, barcode, and the OES can issue a certified true copy of documents prepared by them.
hologram of the Office of the President, together with a transmittal letter, to the MRO for
official release. Within the same day, the MRO sends the original copy of the appointment Q: Why do you say that, Mr. Witness?
paper together with the transmittal letter and a delivery receipt which contains appropriate
spaces for the name of the addressee, the date released, and the date received by the
A: Because the MRO is the so-called "gatekeeper" of the Malacañang Palace. All incoming
addressee. Only a photocopy of the appointment is retained for the MRO's official file.
and outgoing documents and correspondence must pass through the MRO. As the official
custodian, the MRO is in charge of the official release of documents.
Q: What is the basis for the process you just discussed?
Q: What if an appointment paper was faxed by the Office of the Executive Secretary to the
A: The Service Guide of the MRO. appointee, is that considered an official release by the MRO?

Q: What is the legal basis for the issuance of the MRO Service Guide, if any? A: No. It is still the MRO which will furnish the original copy of the appointment paper to the
appointee. That appointment paper is, at best, only an "advanced copy."
129

Q: Assuming the MRO has already received the original appointment paper signed by the A: Yes.
President together with the transmittal letter prepared by the OES, you said that the MRO is
bound to transmit these documents immediately, that is, on the same day? Q: I am showing you a Certification containing the number of presidential appointees per
month since January 2009 until June 2010, and a graphical representation thereof. Can you
A: Yes. go over these documents and tell us the relation of these documents to the ones you
previously mentioned?
Q: Were there instances when the President, after the original appointment paper has
already been forwarded to the MRO, recalls the appointment and directs the MRO not to A: These are [sic] the Certification with the table of statistics I prepared after we counted the
transmit the documents? appointments, as well as the graph thereof.

A: Yes, there were such instances. Q: Out of the more than 800 appointees made in March 2010, how many appointment
papers and transmittal letters were released through the MRO?
Q: How about if the document was already transmitted by the MRO, was there any instance
when it was directed to recall the appointment and retrieve the documents already A: Only 133 appointment papers were released through the MRO.
transmitted? A: Yes, but only in a few instances. Sometimes, when the MRO messenger is
already in transit or while he is already in the agency or office concerned, we get a call to Q: In some of these transmittal letters and appointment papers which were not released
hold the delivery. Q: You previously outlined the procedure governing the transmittal of through the MRO but apparently through the OES, there were portions on the stamp of the
original copies of appointment papers to the agency or office concerned. Would you know if OES which supposedly indicated the date and time it was actually received by the agency or
this procedure was followed by previous administrations? office concerned but were curiously left blank, is this regular or irregular?

A: Yes. Since I started working in the MRO in 1976, the procedure has been followed. A: It is highly irregular.
However, it was unusually disregarded when the appointments numbering more than 800
were made by then President Arroyo in March 2010. The MRO did not even know about
Q: Why do you say so?
some of these appointments and we were surprised when we learned about them in the
newspapers.
A: Usually, if the document released by the MRO, the delivery receipt attached to the
transmittal letter is filled out completely because the dates when the original appointment
Q: You mentioned that then President Arroyo appointed more than 800 persons in the
papers were actually received are very material. It is a standard operating procedure for the
month of March alone. How were you able to determine this number?
MRO personnel to ask the person receiving the documents to write his/her name, his
signature, and the date and time when he/she received it.
A: My staff counted all the appointments made by then President An-oyo within the period
starting January 2009 until June 2010.
Q: So, insofar as these transmittal letters and appointment papers apparently released by the
OES are concerned, what is the actual date when the agency or the appointee concerned
Q: What did you notice, if any, about these appointments? received it?

A: There was a steep rise in the number of appointments made by then President Arroyo in A: I cannot answer. There is no way of knowing when they were actually received because
the month of March 2010 compared to the other months. the date and time were deliberately or inadvertently left blank.

Q: Do you have any evidence to show this steep rise? Q: Can we say that the date appearing on the face of the transmittal letters or the
appointment papers is the actual date when it was released by the OES?
A: Yes. I prepared a Certification showing these statistics and the graphical representation
thereof. A: We cannot say that for sure. That is why it is very unusual that the person who received
these documents did not indicate the date and time when it was received because these
Q: If those documents will be shown to you, will you be able to recognize them? details are very important.53
130

The MRO's exercise of its mandate does not prohibit the President or the Executive Secretary (a) The portion with the name "EDDIE U. TAMONDONG" as "Member,
from giving the appointment paper directly to the appointee. However, a problem may arise representing the Private Sector, Board of Directors" as Exhibit "2-L-2";
if an appointment paper is not coursed through the MRO and the appointment paper is lost
or the appointment is questioned. The appointee would then have to prove that the (b) The portion rubber stamped by the Office of the Executive Secretary
appointment paper was directly given to him. located at the back of the last page of the letter showing receipt by Ma.
Carissa O. Coscuella with blank spaces for the date and time when it was
Dimaandal's counsel made this manifestation about petitioners' appointment papers and actually received as Exhibit "2-L-3";
their transmittal:
8. A) The Transmittal Letter pertinent to the appointments of x x x
Your Honors, we respectfully request for the following markings to be made: FRANCISCA BESTOYONG-ROSQUITA dated March 8, 2010 but turned over
to the MRO on May 13, 2010 as Exhibit "2-T" for the respondents;
1. A) The Transmittal Letter pertinent to the appointment of petitioner DINDO
VENTURANZA dated March 9, 2010 as Exhibit "2-F" for the respondents; (c) The portion with the name "FRANCISCA BESTOYONGROSQUIT A" as
"Commissioner, Representing Region I and the Cordilleras" as Exhibit "2-
B) The delivery receipt attached in front of the letter bearing the date T-3·"
March 12, 2010 as Exhibit "2-F-l";
(d) The portion rubber stamped by the Office of the Executive Secretary
C) The Appointment Paper of DINDO VENTURANZA dated February 23, at the back thereof showing receipt by Masli A. Quilaman of NCIP-QC on
2010 as Exhibit "2-G" for the respondents; March 15, 2010 as Exhibit "2-T-4;"

2. A) The Transmittal Letter pertinent to the appointment of CHELOY E. VELICARIA- D) The Appointment Paper of FRANCISCA BESTOYONGROSQUIT A dated March 5,
GARAFIL turned over to the MRO on May 13, 2010 consisting of seven (7) pages as 2010 as Exhibit "2-W" for the respondents;
Exhibits "2-H," "2-H-l," "2-H-2," "2-H-3," "2-H-4," "2-H-5," and "2-H-6" respectively
for the respondents; 9. A) The Transmittal Letter pertinent to the appointment of IRMA A. VILLANUEVA
as Administrator for Visayas, Board of Administrators, Cooperative Development
i. The portion with the name "CHELOY E. VELICARIAGARAFIL" as Authority, Department of Finance dated March 8, 2010 as Exhibit "2-X" for the
"State Solicitor II, Office of the Solicitor General" located on the respondents;
first page of the letter as Exhibit "2-H-7;"
(a) The portion rubber stamped by the Office of the Executive Secretary
ii. The portion rubber stamped by the Office of the Executive at the back thereof showing receipt by DOF with blank spaces for the
Secretary located at the back of the last page of the -letter date and time when it was actually received as Exhibit "2-X-1 ;"
showing receipt by the DOJ with blank spaces for the date and
time when it was actually received as Exhibit "2-H-8;" B) The Appointment Paper of IRMA A. VILLANUEVA dated March 3, 2010 as Exhibit
"2-Y" for the respondents.54
B) The Appointment Paper of CHELOY E. VELICARIA-GARAFIL dated March
5, 2010 as Exhibit "2-I" for the respondents; The testimony of Ellenita G. Gatbunton, Division Chief of File Maintenance and Retrieval
Division of the MRO, supports Dimaandal's counsel's manifestation that the transmittal of
4. A) The Transmittal Letter pertinent to the appointment of EDDIE U. petitioners' appointment papers is questionable.
TAMONDONG dated 8 March 2010 but turned over to the MRO only on May 6,
2010 consisting of two (2) pages as Exhibits "2-L" and "2-L-l" respectively for the Q: In the case of Cheloy E. Velicaria-Garafil, who was appointed as State Solicitor II of the
respondents; Office of the Solicitor General, was her appointment paper released through the MRO?
131

A: No. Her appointment paper dated March 5, 2010, with its corresponding transmittal letter, Q: What is your basis?
was merely turned over to the MRO on May 13, 2010. The transmittal letter that was turned
over to the MRO was already stamped "released" by the Office of the Executive Secretary, A: The transmittal letter and appointment paper turned over to the MR0.55
but the date and time as to when it was actually received were unusually left blank.
The possession of the original appointment paper is not indispensable to authorize an
Q: What is your basis? appointee to assume office. If it were indispensable, then a loss of the original appointment
paper, which could be brought about by negligence, accident, fraud, fire or theft,
A: The transmittal letter and appointment paper turned over to the MRO. corresponds to a loss of the office.56 However, in case of loss of the original appointment
paper, the appointment must be evidenced by a certified true copy issued by the proper
Q: In the case of Eddie U. Tamondong, who was appointed as member of the Board of office, in this case the MRO. Vacant Position
Directors of Subic Bay Metropolitan Authority, was her [sic] appointment paper released
through the MRO? An appointment can be made only to a vacant office. An appointment cannot be made to an
occupied office. The incumbent must first be legally removed, or his appointment validly
A: No. His appointment paper dated March 1, 2010, with its corresponding transmittal letter, terminated, before one could be validly installed to succeed him. 57
was merely turned over to the MRO on May 6, 2010. The transmittal letter that was turned
over to the MRO was already stamped "released" by the Office of the Executive Secretary, To illustrate: in Lacson v. Romero,58 Antonio Lacson (Lacson) occupied the post of provincial
but the date and time as to when it was actually received were unusually left blank. fiscal of Negros Oriental. He was later nominated and confirmed as provincial fiscal of Tarlac.
The President nominated and the Commission on Appointments confirmed Honorio Romero
Q: What is your basis? (Romero) as provincial fiscal of Negros Oriental as Lacson's replacement. Romero took his
oath of office, but Lacson neither accepted the appointment nor assumed office as provincial
fiscal of Tarlac. This Court ruled that Lacson remained as provincial fiscal of Negros Oriental,
A: The transmittal letter and appointment paper turned over to the MRO.
having declined the appointment as provincial fiscal of Tarlac. There was no vacancy to which
Romero could be legally appointed; hence, Romero's appointment as provincial fiscal
Q: In the case of Francisca Bestoyong-Resquita who was appointed as Commissioner of the ofNegros Oriental vice Lacson was invalid.
National Commission on Indigenous Peoples, representing Region 1 and the Cordilleras, was
her appointment paper released thru the MRO?
The appointment to a government post like that of provincial fiscal to be complete involves
several steps. First, comes the nomination by the President. Then to make that nomination
A: No. Her appointment paper dated March 5, 2010, with its corresponding transmittal letter, valid and permanent, the Commission on Appointments of the Legislature has to confirm said
was merely turned over to the MRO on May 13, 2010. The transmittal letter that was turned nomination. The last step is the acceptance thereof by the appointee by his assumption of
over to the MRO was already stamped "released" by the Office of the Executive Secretary office. The first two steps, nomination and confirmation, constitute a mere offer of a post.
and received on March 15, 2010. They are acts of the Executive and Legislative departments of the Government. But the last
necessary step to make the appointment complete and effective rests solely with the
Q: What is your basis? appointee himself. He may or he may not accept the appointment or nomination. As held in
the case of Borromeo vs. Mariano, 41 Phil. 327, "there is no power in this country which can
A: The transmittal letter and appointment paper turned over to the MRO. compel a man to accept an office." Consequently, since Lacson has declined to accept his
appointment as provincial fiscal of Tarlac and no one can compel him to do so, then he
continues as provincial fiscal of Negros Oriental and no vacancy in said office was created,
Q: In the case of Irma A. Villanueva who was appointed as Administrator for Visayas of the unless Lacson had been lawfully removed as such fiscal of Negros Oriental.59
Cooperative Development Authority, was her appointment paper released thru the MRO?
Paragraph (b ), Section 1 of EO 2 considered as midnight appointments those appointments
A: No. Her appointment paper dated March 3, 2010, with its corresponding transmittal letter, to offices that will only be vacant on or after 11 March 2010 even though the appointments
was merely turned over to the MRO on May 4, 2010. The transmittal letter that was turned are made prior to 11 March 2010. EO 2 remained faithful to the intent of Section 15, Article
over to the MRO was already stamped "released" by the Office of the Executive Secretary, VII of the 1987 Constitution: the outgoing President is prevented from continuing to rule the
but the date and time as to when it was actually received were unusually left blank. country indirectly after the end of his term.
132

Acceptance by the Qualified Appointee SO ORDERED.

Acceptance is indispensable to complete an appointment. Assuming office and taking the The present consolidated cases involve four petitions: G.R. No. 203372 with Atty. Cheloy E.
oath amount to acceptance of the appointment.60 An oath of office is a qualifying Velicaria-Garafil (Atty. Velicaria-Garafil), who was appointed State Solicitor II at the Office of
requirement for a public office, a prerequisite to the full investiture of the office.61 the Solicitor General (OSG), as petitioner; G.R. No. 206290 with Atty. Dindo G. Venturanza
(Atty. Venturanza), who was appointed Prosecutor IV (City Prosecutor) of Quezon City, as
Javier v. Reyes62 is instructive in showing how acceptance is indispensable to complete an petitioner; G.R. No. 209138 with Irma A. Villanueva (Villanueva), who was appointed
appointment. On 7 November 1967, petitioner Isidro M. Javier (Javier) was appointed by
Administrator for Visayas of the Board of Administrators of the Cooperative Development
then Mayor Victorino B. Aldaba as the Chief of Police of Malolos, Bulacan. The Municipal
Council confirmed and approved Javier's appointment on the same date. Javier took his oath Authority (CDA), and Francisca B. Rosquita (Rosquita), who was appointed Commissioner of
of office on 8 November 1967, and subsequently discharged the rights, prerogatives, and the National Commission of Indigenous Peoples (NCIP), as petitioners; and G.R. No. 212030
duties of the office. On 3 January 1968, while the approval of Javier's appointment was with Atty. Eddie U. Tamondong (Atty. Tamondong), who was appointed member of the
pending with the CSC, respondent Purificacion C. Reyes (Reyes), as the new mayor of Board of Directors of the Subic Bay Metropolitan Authority (SBMA), as petitioner. All
Malolos, sent to the . CSC a letter to recall Javier's appointment. Reyes also designated Police petitions question the constitutionality of Executive Order No. 2 (EO 2) for being inconsistent
Lt. Romualdo F. Clemente as Officer-in-Charge of the police department. The CSC approved with Section 15, Article VII of the 1987 Constitution.
Javier's appointment as permanent on 2 May 1968, and even directed Reyes to reinstate
Javier. Reyes, on the other hand, pointed to the appointment of Bayani Bernardo as Chief of
Prior to the conduct of the May 2010 elections, then President Gloria Macapagal-Arroyo
Police of Malolos, Bulacan on 4 September 1967. This Court ruled that Javier's appointment
(President Macapagal-Arroyo) issued more than 800 appointments to various positions in
prevailed over that of Bernardo. It cannot be said that Bernardo accepted his appointment
because he never assumed office or took his oath. several government offices.

The ban on midnight appointments in Section 15, Article VII of the 1987 Constitution reads:
Excluding the act of acceptance from the appointment process leads us to the very evil which
we seek to avoid (i.e., antedating of appointments). Excluding the act of acceptance will only
Two months immediately before the next presidential elections and up to the end of his
provide more occasions to honor the Constitutional provision in the breach. The inclusion of
acceptance by the appointee as an integral part of the entire appointment process prevents term, a President or Acting President shall not make appointments, except temporary
the abuse of the Presidential power to appoint. It is relatively easy to antedate appointment appointments to executive positions when continued vacancies therein will prejudice public
papers and make it appear that they were issued prior to the appointment ban, but it is more service or endanger public safety.
difficult to simulate the entire appointment process up until acceptance by the appointee.
Thus, for purposes of the 2010 elections, 10 March 2010 was the cutoff date for valid
Petitioners have failed to show compliance with all four elements of a valid appointment. appointments and the next day, 11 March 2010, was the start of the ban on midnight
They cannot prove with certainty that their appointment papers were transmitted before the appointments. Section 15, Article VII of the 1987 Constitution recognizes as an exception to
appointment ban took effect. On the other hand, petitioners admit that they took their oaths the ban on midnight appointments only "temporary appointments to executive positions
of office during the appointment ban.
when continued vacancies therein will prejudice public service or endanger public safety."

Petitioners have failed to raise any valid ground for the Court to declare EO 2, or any part of None of the petitioners claim that their appointments fall under this exception.
it, unconstitutional. Consequently, EO 2 remains valid and constitutional.
On 30 June 2010, President Benigno S. Aquino III (President Aquino) took his oath of office as
WHEREFORE, the petitions in G.R. Nos. 203372, 206290, and 212030 are DENIED, and the President of the Republic of the Philippines. On 30 July 2010, President Aquino issued EO 2
petition in G.R. No. 209138 is DISMISSED. The appointments of petitioners Atty. Cheloy E. recalling, withdrawing, and revoking appointments issued by President Macapagal-Arroyo
Velicaria-Garafil (G.R. No. 203372), Atty. Dindo G. Venturanza (G.R. No. 206290), Irma A.
which violated the constitutional ban on midnight appointments.
Villanueva, and Francisca B. Rosquita (G.R. No. 209138), and Atty. Eddie U. Tamondong (G.R.
No. 212030) are declared VOID. We DECLARE that Executive Order No. 2 dated 30 July 2010
is VALID and CONSTITUTIONAL.
133

Issue: (4) receipt of the appointment paper and acceptance of the appointment by the appointee
who possesses all the qualifications and none of the disqualifications.
(1) whether petitioners' appointments violate Section 15, Article VII of the 1987 Constitution,
and Acceptance is indispensable to complete an appointment. Assuming office and taking the
oath amount to acceptance of the appointment. An oath of office is a qualifying requirement
(2) whether EO 2 is constitutional. for a public office, a prerequisite to the full investiture of the office.

Held: Petitioners have failed to show compliance with all four elements of a valid appointment.
They cannot prove with certainty that their appointment papers were transmitted before the
The following elements should always concur in the making of a valid (which should be
appointment ban took effect. On the other hand, petitioners admit that they took their oaths
understood as both complete and effective) appointment:
of office during the appointment ban.
(1) authority to appoint and evidence of the exercise of the authority;
Petitioners have failed to raise any valid ground for the Court to declare EO 2, or any part of
it, unconstitutional. Consequently, EO 2 remains valid and constitutional.
The President's exercise of his power to appoint officials is provided for in the Constitution
and laws. Discretion is an integral part in the exercise of the power of appointment.
PRESIDENTATTY. CHELOY E. VELICARIA-GARAFIL vs. OFFICE OF THE PRESIDENT
Considering that appointment calls for a selection, the appointing power necessarily
exercises a discretion.
FACTS:
The power to appoint is, in essence, discretionary. The appointing power has the right of
choice which he may exercise freely according to his judgment, deciding for himself who is Prior to the conduct of the May 2010 elections, then President Gloria Macapagal-Arroyo
(President Macapagal-Arroyo) issued more than 800 appointments to various positions in
best qualified among those who have the necessary qualifications and eligibilities.
several government offices.
(2) transmittal of the appointment paper and evidence of the transmittal;
For purposes of the 2010 elections, 10 March 2010 was the cutoff date for valid
It is not enough that the President signs the appointment paper. There should be evidence appointments and the next day, 11 March 2010, was the start of the ban on midnight
appointments. Section 15, Article VII of the 1987 Constitution recognizes as an exception to
that the President intended the appointment paper to be issued. It could happen that an
the ban on midnight appointments only "temporary appointments to executive positions
appointment paper may be dated and signed by the President months before the when continued vacancies therein will prejudice public service or endanger public safety."
appointment ban, but never left his locked drawer for the entirety of his term. Release of the None of the petitioners claim that their appointments fall under this exception.
appointment paper through the MRO is an unequivocal act that signifies the President's
intent of its issuance. The present consolidated cases involve four petitions: G.R. No. 203372 with Atty. Cheloy E.
Velicaria-Garafil (Atty. Velicaria-Garafil), who was appointed State Solicitor II at the Office of
For purposes of verification of the appointment paper's existence and authenticity, the the Solicitor General (OSG), as petitioner; G.R. No. 206290 with Atty. Dindo G. Venturanza
appointment paper must bear the security marks (i.e., handwritten signature of the (Atty. Venturanza), who was appointed Prosecutor IV (City Prosecutor) of Quezon City, as
President, bar code, etc.) and must be accompanied by a transmittal letter from the MRO. petitioner; G.R. No. 209138 with Irma A. Villanueva (Villanueva), who was appointed
Administrator for Visayas of the Board of Administrators of the Cooperative Development
(3) a vacant position at the time of appointment; and Authority (CDA), and Francisca B. Rosquita (Rosquita), who was appointed Commissioner of
the National Commission of Indigenous Peoples (NCIP), as petitioners; and G.R. No. 212030
Petitioners have failed to raise any valid ground for the Court to declare EO 2, or any part of with Atty. Eddie U. Tamondong (Atty. Tamondong), who was appointed member of the
Board of Directors of the Subic Bay Metropolitan Authority (SBMA), as petitioner. All
it, unconstitutional. Consequently, EO 2 remains valid and constitutional.
petitions question the constitutionality of Executive Order No. 2 (EO 2) for being inconsistent
with Section 15, Article VII of the 1987 Constitution.
134

To summarize, the pertinent dates for each petitioner are as follows: RULING:

G.R. No. Date of Date of Date of Date of Assumption All of petitioners' appointments are midnight appointments and are void for violation of
Appointment Transmittal Receipt by Oath of of Office Section 15, Article VII of the 1987 Constitution. EO 2 is constitutional.
Letter Letter MRO Office
This ponencia and the dissent both agree that the facts in all these cases show that "none of
203372 the petitioners have shown that their appointment papers (and transmittal letters) have
(Atty. 13 May 22 March been issued (and released) before the ban." The dates of receipt by the MRO, which in these
5 March 2010 8 March 2010 6 April 2010
Velicaria- 2010 2010 cases are the only reliable evidence of actual transmittal of the appointment papers by
Garafil) President Macapagal-Arroyo, are dates clearly falling during the appointment ban. Thus, this
ponencia and the dissent both agree that all the appointments in these cases are midnight
206290
23 February 12 March 15 March 15 March appointments in violation of Section 15, Article VII of the 1987 Constitution.
(Atty. 9 March 2010
2010 2010 2010 2010
Venturanza)
CONSTITUTIONALITY OF EO 2
209138 4 May 13 April
3 March 2010
(Villanueva) 2010 2010
Any valid appointment, including one made under the exception provided in Section 15,
209138 13 May 18 March Article VII of the 1987 Constitution, must consist of the President signing an appointee's
5 March 2010 appointment paper to a vacant office, the official transmittal of the appointment paper
(Rosquita) 2010 2010
(preferably through the MRO), receipt of the appointment paper by the appointee, and
25 March acceptance of the appointment by the appointee evidenced by his or her oath of office or his
212030
2010 and or her assumption to office.
(Atty. 1 March 2010
6 July
Tamondong)
2010 During the deliberations for the 1987 Constitution, then Constitutional Commissioner (now
retired Supreme Court Chief Justice) Hilario G. Davide, Jr. referred to this Court's ruling in
On 30 June 2010, President Benigno S. Aquino III (President Aquino) took his oath of office as Aytona and stated that his proposal seeks to prevent a President, whose term is about to
President of the Republic of the Philippines. On 30 July 2010, President Aquino issued EO 2 end, from preempting his successor by appointing his own people to sensitive positions.
recalling, withdrawing, and revoking appointments issued by President Macapagal-Arroyo
which violated the constitutional ban on midnight appointments. [T]he well-settled rule in our jurisprudence, that an appointment is a process that begins
with the selection by the appointing power and ends with acceptance of the appointment
The CA consistently ruled that EO 2 is constitutional. The CA, however, issued different by the appointee, stands.
rulings as to the evaluation of the circumstances of petitioners' appointments. In the cases of
Attys. Velicaria-Garafil and Venturanza, the CA stated that the OP should consider the The dissent's assertion creates a singular exception to the well-settled doctrine that
circumstances of their appointments. In the cases of Villanueva, Rosquita, and Atty. appointment is a process that begins with the signing of the appointment paper, followed by
Tamondong, the CA explicitly stated that · the revocation of their appointments was proper the transmittal and receipt of the appointment paper, and becomes complete with the
because they were midnight appointees. acceptance of the appointment. The dissent makes the singular exception that during the
constitutionally mandated ban on appointments, acceptance is not necessary to complete
ISSUES: the appointment. The dissent gives no reason why this Court should make such singular
exception, which is contrary to the express provision of the Constitution prohibiting the
President from making appointments during the ban. The dissent's singular exception will
(1) Whether petitioners' appointments violate Section 15, Article VII of the 1987
allow the President, during the ban on appointments, to remove from office incumbents
Constitution;
without cause by simply appointing them to another office and transmitting the appointment
papers the day before the ban begins, appointments that the incumbents cannot refuse
(2) Whether EO 2 is constitutional. Ruling of the Court because their acceptance is not required during the ban. Adoption by this Court of the
dissent's singular exception will certainly wreak havoc on the civil service.
135

The following elements should always concur in the making of a valid (which should be
understood as both complete and effective) appointment: (1) authority to appoint and
evidence of the exercise of the authority; (2) transmittal of the appointment paper and
evidence of the transmittal; (3) a vacant position at the time of appointment; and (4) receipt
of the appointment paper and acceptance of the appointment by the appointee who
possesses all the qualifications and none of the disqualifications. The concurrence of all these
elements should always apply, regardless of when the appointment is made, whether
outside, just before, or during the appointment ban. These steps in the appointment process
should always concur and operate as a single process. There is no valid appointment if the
process lacks even one step. And, unlike the dissent's proposal, there is no need to further
distinguish between an effective and an ineffective appointment when an appointment is
valid.
136

EN BANC this Act. The Board of Directors is authorized to provide for separation benefits for those who
cannot be accommodated in the new structure. All those who shall retire or are separated
G.R. No. 168613 March 5, 2013 from the service on account of the reorganization under the preceding Section shall be
entitled to such incentives, as are authorized by the Corporation, which shall be in addition
to all gratuities and benefits to which they may be entitled under existing laws.
ATTY. MA. ROSARIO MANALANG-DEMIGILLO, Petitioner,
vs.
TRADE AND INVESTMENT DEVELOPMENT CORPORATION OF THE PHILIPPINES (TIDCORP), In Opinion No. 221 dated September 13, 2002,1 then Government Corporate Counsel Amado
and its BOARD OF DIRECTORS, Respondents. D. Valdez opined as follows:

DECISION There is no question on the power of the PhilEXIM (also known as TIDCORP) Board of
Directors to undertake a reorganization of the corporation’s present organizational set-up. In
fact, the authority to provide for the corporation’s organizational structure is among the
BERSAMIN, J.:
express powers granted to PhilEXIM through its Board.

A reorganization undertaken pursuant to a specific statutory authority by the Board of


As to the one-year period to implement a reorganization mentioned in Section 8 of RA 8494,
Directors of a government-owned and government-controlled corporation is valid.
it is our considered opinion that the same provision refers to the initial reorganization to
effect transition from the Philippine Export and Foreign Loan Guarantee Corporation
Antecedents (Philguarantee) to what is now known as the Trade and Investment Corporation of the
Philippines (TIDCORP). The one-year period does not, however, operate as a limitation that
On February 12, 1998, the Philippine Export and Foreign Loan Guarantee was renamed Trade any subsequent changes in the organizational set-up pursuant to the authority of the Board
and Investment Development Corporation of the Philippines (TIDCORP) pursuant to Republic to determine the corporation’s organizational structure under Section 7 of RA 8494, which is
Act No. 8494 entitled An Act Further Amending Presidential Decree No. 1080, As Amended, designed to make the corporation more attuned to the needs of the people or, in this case,
by Reorganizing And Renaming the Philippine Export and Foreign Loan Guarantee the sector of the Philippine economy that it serves, can only be made during the same one-
Corporation, Expanding Its Primary Purpose, and for Other Purposes. year period.

Republic Act No. 8494 reorganized the structure of TIDCORP. The issuance of appointments On the basis of OGCC Opinion No. 221, the Board of Directors passed Resolution No. 1365,
in accordance with the reorganization ensued. Petitioner Rosario Manalang-Demigillo Series of 2002, on October 22, 2002 to approve a so-called Organizational
(Demigillo) was appointed as Senior Vice President (PG 15) with permanent status, and was Refinement/Restructuring Plan to implement a new organizational structure and staffing
assigned to the Legal and Corporate Services Department (LCSD) of TIDCORP. pattern, a position classification system, and a new set of qualification standards.

In 2002, TIDCORP President Joel C. Valdes sought an opinion from the Office of the During the implementation of the Organizational Refinement/Restructuring Plan, the LCSD
Government Corporate Counsel (OGCC) relative to TIDCORP’s authority to undertake a was abolished. According to the List of Appointed Employees under the New Organizational
reorganization under the law, whose Section 7 and Section 8 provide as follows: Structure of TIDCORP as of November 1, 2002, Demigillo, albeit retaining her position as a
Senior Vice President, was assigned to head the Remedial and Credit Management Support
Section 7. The Board of Directors shall provide for an organizational structure and staffing Sector (RCMSS). On the same date, President Valdes issued her appointment as head of
pattern for officers and employees of the Trade and Investment Development Corporation of RCMSS, such appointment being in nature a reappointment under the reorganization plan.
the Philippines (TIDCORP) and upon recommendation of its President, appoint and fix their
remuneration, emoluments and fringe benefits: Provided, That the Board shall have exclusive On December 13, 2002, President Valdes issued a memorandum informing all officers and
and final authority to appoint, promote, transfer, assign and re-assign personnel of the employees of TIDCORP that the Board of Directors had approved on December 11, 2002 the
TIDCORP, any provision of existing law to the contrary notwithstanding. x x x appointments issued pursuant to the newly approved positions under the Organizational
Refinement/Restructuring Plan.
Section 8. All incumbent personnel of the Philippine Export and Foreign Loan Guarantee
Corporation shall continue to exercise their duties and functions as personnel of the TIDCORP In her letter dated December 23, 2002 that she sent to TIDCORP Chairman Jose Isidro
until reorganization is fully implemented but not to exceed one (1) year from the approval of Camacho, however, Demigillo challenged before the Board of Directors the validity of
137

Resolution No. 1365 and of her assignment to the RCMSS. She averred that she had been In view of the foregoing and your failure to prove that you have effectively and efficiently
thereby illegally removed from her position of Senior Vice President in the LCSD to which she performed the duties, functions and responsibility (sic) of your position, I am constrained to
had been previously assigned during the reorganization of July 1998. She insisted that give you a rating of "Poor" for your 2002 performance.3
contrary to OGCC Opinion No. 221 dated September 13, 2002 the Board of Directors had not
been authorized to undertake the reorganization and corporate restructuring. On April 28, 2003, Demigillo formally communicated to Atty. Florencio P. Gabriel Jr.,
Executive Vice President of the Operations Group, appealing the "poor rating" given her by
On January 31, 2003, pending determination of her challenge by the Board of Directors, President Valdes.
Demigillo appealed to the Civil Service Commission (CSC), raising the same issues.
In a memorandum dated May 6, 2003, Atty. Gabriel informed Demigillo that he could not act
TIDCORP assailed the propriety of Demigillo’s appeal to the CSC, alleging that her elevation of on her appeal because of her "failure to state facts and arguments constituting the grounds
the case to the CSC without the Board of Directors having yet decided her challenge had for the appeal and submit any evidence to support the same."4
been improper and a clear case of forum-shopping.
On May 6, 2003, President Valdes issued a memorandum to Demigillo stating that he found
Later on, however, TIDCORP furnished to the CSC a copy of Board Decision No. 03-002 no justification to change the poor rating given to her for the year 2002.
dismissing Demigillo’s appeal for its lack of merit, thereby rendering the question about the
propriety of Demigillo’s appeal moot and academic. Board Decision No. 03-002 pertinently On August 12, 2003, Demigillo received a memorandum from President Valdes stating that
reads as follows: her performance rating for the period from January 1, 2003 to June 2003 "needs
improvement," attaching the pertinent Performance Evaluation Report Form that she was
Atty. Demigillo failed to show to the Board that she was prejudiced in the implementation of instructed to return "within 24 hours from receipt."5
the TIDCORP organizational refinements/restructuring. She was reappointed to the same
position she was holding before the reorganization. She was not demoted in terms of salary, Not in conformity with the performance rating, Demigillo scribbled on the right corner of the
rank and status. There was a (sic) substantial compliance with the requirements of RA 6656, memorandum the following comments: "I do not agree and accept. I am questioning the
particularly on transparency. More importantly, the said organizational refinements done same. This is pure harassment."
and adoption of a new compensation structure were made in accordance with what is
mandated under the Charter of the Corporation.
She then appealed the poor performance rating on August 14, 2003, calling the rating a part
of Valdes’ "unremitting harassment and oppression on her."6
WHEREFORE, foregoing premises considered, the Board decided as it hereby decides to
DISMISS the appeal of Atty. Ma Rosario Demigillo for lack of merit.2
On August 19, 2003, Demigillo reported for work upon the expiration of the 90-day
preventive suspension imposed by the Board of Directors in a separate administrative case
In the meanwhile, by letter dated April 14, 2003, President Valdes informed Demigillo of her for grave misconduct, conduct prejudicial to the best interest of the service, insubordination
poor performance rating for the period from January 1, 2002 to December 31, 2002, to wit: and gross discourtesy. In her memorandum of that date, she informed Atty. Gabriel Jr. of her
readiness to resume her duties and responsibilities, but requested to be allowed to
After a thorough evaluation/assessment of your job performance for the rating period reproduce documents in connection with the appeal of her performance rating. She further
January 1 to December 21, 2002, it appears that your over-all performance is ‘Poor’. requested that the relevant grievance process should commence.

Records show that you consistently behaved as an obstructionist in the implementation of It appears that the Board of Directors rendered Decision No. 03-003 dated August 15, 2003
the Corporate Business Plan. You failed to demonstrate cooperation, respect and concern unanimously dropping Demigillo from the rolls.7 Demigillo received the copy of Decision No.
towards authority and other members of the company. You also failed to abide by Civil 03-003 on August 25, 2003.
Service and company policies, rules and regulation. You miserably failed to adapt and
respond to changes. You were very resentful to new approaches as shown by your vehement Decision of the CSC
objection to new improved policies and programs. Instead of helping raise the morale of
subordinate at high levels (sic) and promote career and professional growth of subordinates,
On October 14, 2004, the CSC ruled through Resolution No. 0410928 that the 2002
you tried to block such efforts towards this end.
Organizational Refinements or Restructuring Plan of TIDCORP had been valid for being
138

authorized by Republic Act. No. 6656; that Section 7 of Republic Act No. 8498 granted a On June 27, 2005, the CA’s Fourth Division promulgated its decision in CA–G.R. SP No.
continuing power to TIDCORP’s Board of Directors to prescribe the agency’s organizational 87285,12 which, albeit affirming the ruling of the CSC, rendered a legal basis different from
structure, staffing pattern and compensation packages; and that such grant continued until that given by the CSC, to wit:
declared invalid by a court of competent jurisdiction or revoked by Congress.
In numerous cases citing Section 20 and Section 31, Book III of Executive Order No. 292,
The CSC held, however, that TIDCORP’s implementation of its reorganization did not comply otherwise known as the Administrative Code of 1987, the Supreme Court ruled in the
with Section 6 of Republic Act No. 6656;9 that although there was no diminution in affirmative that the President of the Philippines has the continuing authority to reorganize
Demigillo’s rank, salary and status, there was nonetheless a demotion in her functions and the administrative structure of the Office of the President.
authority, considering that the 2002 reorganization reduced her authority and functions from
being the highest ranking legal officer in charge of all the legal and corporate affairs of Hence, being the alter ego of the President of the Philippines, the Board of Directors of the
TIDCORP to being the head of the RCMSS reporting to the Executive Vice President and private respondent-appellee is authorized by law to have a continuous power to reorganize
having only two departments under her supervision; and that the functions of Demigillo’s its agency.13
office were in fact transferred to the Operations Group.
Anent Demigillo’s contention that the 2002 reorganization effected was invalid, the CA ruled:
The CSC further held that the dropping from the rolls of Demigillo did not comply with the
mandatory requirement under Section 2, particularly 2.2 Rule XII of the Revised Omnibus
x x x. In this jurisdiction, reorganizations have been regarded as valid provided they are
Rules on Appointments and Other Personnel Actions Memorandum Circular No. 40, Series of
pursued in good faith. Reorganization is carried out in good faith if it is for the purpose of
1998.
economy or to make bureaucracy more efficient.

Subsequently, TIDCORP reinstated Demigillo to the position of Senior Vice President in


In the case at bench, it is our considered opinion that except for her allegations, the
RCMSS, a position she accepted without prejudice to her right to appeal the decision of the
petitioner-appellant (Demigillo) failed to present sufficient evidence that the reorganization
CSC.
effected in 2002 did not bear the earmarks of economy and efficiency. Good faith is always
presumed.14
Ruling of the CA
The CA held that Demigillo could not be reinstated to her previous position of Senior Vice
Both Demigillo and TIDCORP appealed the decision of the CSC to the Court of Appeals (CA). President of the LCSD in view of the legality of the 2002 reorganization being upheld.15
Demigillo’s appeal was docketed as CA-G.R. SP No. 87285. On the other hand, TIDCORP’s
appeal was docketed as CA-G.R. SP No. 87295.
With respect to CA-G.R. SP No. 87295, the CA’s Special Former Thirteenth Division
promulgated a decision on November 28, 2008,16 denying TIDCORP’s appeal, and holding
In CA-G.R. SP No. 87285, Demigillo partially assailed the CSC’s decision, claiming that the CSC that Demigillo had been demoted and invalidly dropped from the rolls by TIDCORP,
erred: (1) in holding that Section 7 of Republic Act No. 8494 granted the Board of Directors of explaining:
TIDCORP a continuing power to reorganize; (2) in holding that the 2002 TIDCORP
reorganization had been authorized by law; and (3) in not holding that the 2002 TIDCORP
We do not need to stretch Our imagination that respondent Demigillo, one of the highest
reorganization was void ab initio because it was not authorized by law and because the
ranking officers of the corporation, was indeed demoted when she was designated to be the
reorganization did not comply with Republic Act No. 6656.10
head of merely one sector. She may have retained her title as SVP, but she was deprived of
the authority she previously enjoyed and stripped of the duties and responsibilities assigned
In CA-G.R. SP No. 87295, TIDCORP contended that the CSC erred: (1) in ruling that Demigillo to her under the Legal and Corporate Services. In utter disregard of respondent Demigillo’s
had been demoted as a result of the 2002 TIDCORP reorganization; and (2) in ruling that right to security of tenure, petitioner TIDCORP demoted her in the guise of "reorganization."
TIDCORP had failed to observe the provisions of Section 2, particularly 2.2 Rule XII of the
Revised Omnibus Rules on Appointments and Other Personnel Actions (Memorandum
Next, petitioner TIDCORP asserts that respondent Demigillo was legally dropped from the
Circular No. 40, Series of 1998) on dropping from the rolls, to the prejudice of Demigillo’s
rolls. This is a delirious supposition which does not deserve merit at all.
right to due process.11
139

Petitioner TIDCORP did not bother to adduce proof that it complied with the rudiments of Mayor Jose D. Villena and found him guilty of bribery, extortion, and abuse of authority. The
due process before dropping Demigillo from the rolls. She was not given the chance to Secretary of Interior then recommended to the President the suspension from office of
present evidence refuting the contentious ratings as her employer refused to discuss how it Mayor Villena. Upon approval by the President of the recommendation, the Secretary of
arrived at such assessment. Her unceremonious dismissal was made even more apparent as Interior suspended Mayor Villena. Unyielding, Mayor Villena challenged his suspension,
she was never advised of the possibility that she may be separated from service if her rating asserting that the Secretary of Interior had no authority to suspend him from office because
would not improve for the next evaluation period.17 there was no specific law granting such power to the Secretary of Interior; and that it was the
President alone who was empowered to suspend local government officials. The Court
Issues disagreed with Mayor Villena and upheld his suspension, holding that the doctrine of
qualified political agency warranted the suspension by the Secretary of Interior. Justice
Laurel, writing for the Court, opined:
Demigillo filed before this Court a petition for review on certiorari assailing the CA decision in
CA-G.R. SP No. 87285 (G.R. No. 168613), asserting that the CA gravely erred: (1) in holding
that the Board of Directors of TIDCORP was an alter ego of the President who had the After serious reflection, we have decided to sustain the contention of the government in this
continuing authority to reorganize TIDCORP; and (2) in holding that the reorganization of case on the broad proposition, albeit not suggested, that under the presidential type of
TIDCORP effected in 2002 was valid considering her alleged failure to present evidence government which we have adopted and considering the departmental organization
sufficiently showing that the reorganization did not bear the earmarks of economy and established and continued in force by paragraph 1, section 12, Article VII, of our Constitution,
efficiency.18 Corollarily, she sought her reinstatement to a position comparable to her former all executive and administrative organizations are adjuncts of the Executive Department, the
position as Senior Vice President in the LCSD.19 heads of the various executive departments are assistants and agents of the Chief Executive,
and, except in cases where the Chief Executive is required by the Constitution or the law to
act in person or the exigencies of the situation demand that he act personally, the
Likewise, TIDCORP appealed through a petition for review on certiorari, praying for the
multifarious executive and administrative functions of the Chief Executive are performed by
reversal of the decision promulgated in CA-G.R. SP No. 87295 (G.R. No. 185571), contending
and through the executive departments, and the acts of the secretaries of such departments,
that the CA erred: (1) in ruling that Demigillo had been demoted as a result of the TIDCORP
performed and promulgated in the regular course of business, are, unless disapproved or
2002 reorganization; and (2) in ruling that Demigillo had not been legally dropped from the
reprobated by the Chief Executive, presumptively the acts of the Chief Executive. (Runkle vs.
rolls.20
United States [1887], 122 U. S., 543; 30 Law. ed., 1167; 7 Sup. Ct. Rep., 1141; see also U. S. vs.
Eliason [1839], 16 Pet., 291; 10 Law. ed., 968; Jones vs. U. S. [1890], 137 U. S., 202; 34 Law.
On March 8, 2011, the Court En Banc consolidated G.R. No. 168613 and G.R. No. 185571.21 ed., 691; 11 Sup. Ct., Rep., 80; Wolsey vs. Chapman [1880], 101 U. S., 755; 25 Law. ed., 915;
Wilcox vs. Jackson [1836], 13 Pet., 498; 10 Law. ed., 264.)
Ruling of the Court
Fear is expressed by more than one member of this court that the acceptance of the principle
We deny the petition for review of Demigillo (G.R. No. 168613) for its lack of merit, but grant of qualified political agency in this and similar cases would result in the assumption of
the petition for review of TIDCORP (G.R. No. 185571). responsibility by the President of the Philippines for acts of any member of his cabinet,
however illegal, irregular or improper may be these acts. The implications, it is said, are
G.R. No. 168613 serious. Fear, however, is no valid argument against the system once adopted, established
and operated. Familiarity with the essential background of the type of Government
established under our Constitution, in the light of certain well-known principles and practices
In its comment in G.R. No. 168613,22 TIDCORP argues for the application of the doctrine of that go with the system, should offer the necessary explanation. With reference to the
qualified political agency, contending that the acts of the Board of Directors of TIDCORP, an Executive Department of the government, there is one purpose which is crystal-clear and is
attached agency of the Department of Finance whose head, the Secretary of Finance, was an readily visible without the projection of judicial searchlight, and that is the establishment of a
alter ego of the President, were also the acts of the President. single, not plural, Executive. The first section of Article VII of the Constitution, dealing with
the Executive Department, begins with the enunciation of the principle that "The executive
TIDCORP’s argument is unfounded. power shall be vested in a President of the Philippines." This means that the President of the
Philippines is the Executive of the Government of the Philippines, and no other. The heads of
The doctrine of qualified political agency, also known as the alter ego doctrine, was the executive departments occupy political positions and hold office in an advisory capacity,
introduced in the landmark case of Villena v. The Secretary of Interior.23 In said case, the and, in the language of Thomas Jefferson, "should be of the President’s bosom confidence"
Department of Justice, upon the request of the Secretary of Interior, investigated Makati (7 Writings, Ford ed., 498), and in the language of Attorney-General Cushing (7 Op., Attorney-
General, 453), "are subject to the direction of the President." Without minimizing the
140

importance of the heads of the various departments, their personality is in reality but the Section 7. The Board of Directors shall provide for an organizational structure and staffing
projection of that of the President. Stated otherwise, and as forcibly characterized by Chief pattern for officers and employees of the Trade and Investment Development Corporation of
Justice Taft of the Supreme Court of the United States, "each head of a department is, and the Philippines (TIDCORP) and upon recommendation of its President, appoint and fix their
must be, the President's alter ego in the matters of that department where the President is remuneration, emoluments and fringe benefits: Provided, That the Board shall have exclusive
required by law to exercise authority." (Myers vs. United States, 47 Sup. Ct. Rep., 21 at 30; and final authority to appoint, promote, transfer, assign and re-assign personnel of the
272 U.S. 52 at 133; 71 Law. Ed., 160). x x x. TIDCORP, any provision of existing law to the contrary notwithstanding.

The doctrine of qualified political agency essentially postulates that the heads of the various In this connection, too, we reiterate that we cannot disturb but must respect the ruling of the
executive departments are the alter egos of the President, and, thus, the actions taken by CSC that deals with specific cases coming within its area of technical knowledge and
such heads in the performance of their official duties are deemed the acts of the President expertise,26 absent a clear showing of grave abuse of discretion on its part. That clear
unless the President himself should disapprove such acts. This doctrine is in recognition of showing was not made herein. Such deference proceeds from our recognition of the
the fact that in our presidential form of government, all executive organizations are adjuncts important role of the CSC as the central personnel agency of the Government having the
of a single Chief Executive; that the heads of the Executive Departments are assistants and familiarity with and expertise on the matters relating to the career service.
agents of the Chief Executive; and that the multiple executive functions of the President as
the Chief Executive are performed through the Executive Departments. The doctrine has Worthy to stress, lastly, is that the reorganization was not arbitrary and whimsical. It had
been adopted here out of practical necessity, considering that the President cannot be been formulated following lengthy consultations and close coordination with the affected
expected to personally perform the multifarious functions of the executive office. offices within TIDCORP in order for them to come up with various functional statements
relating to the new organizational setup. In fact, the Board of Directors decided on the need
But the doctrine of qualified political agency could not be extended to the acts of the Board to reorganize in 2002 to achieve several worthy objectives, as follows:
of Directors of TIDCORP despite some of its members being themselves the appointees of the
President to the Cabinet. Under Section 10 of Presidential Decree No. 1080, as further (1) To make the organization more viable in terms of economy, efficiency,
amended by Section 6 of Republic Act No. 8494,24 the five ex officio members were the effectiveness and make it more responsive to the needs of its clientèles by
Secretary of Finance, the Secretary of Trade and Industry, the Governor of the Bangko eliminating or minimizing any overlaps and duplication of powers and functions;
Sentral ng Pilipinas, the Director-General of the National Economic and Development
Authority, and the Chairman of the Philippine Overseas Construction Board, while the four
(2) To come up with an organizational structure which is geared towards the
other members of the Board were the three from the private sector (at least one of whom
strengthening of the Corporation's overall financial and business operations
should come from the export community), who were elected by the ex officio members of
through resource allocation shift; and
the Board for a term of not more than two consecutive years, and the President of TIDCORP
who was concurrently the Vice-Chairman of the Board. Such Cabinet members sat on the
Board of Directors of TIDCORP ex officio, or by reason of their office or function, not because (3) To rationalize corporate operations to maximize resources and achieve
of their direct appointment to the Board by the President. Evidently, it was the law, not the optimum sustainable corporate performance vis-a-vis revised corporate policies,
President, that sat them in the Board. objectives and directions by focusing the Corporation's efforts and resources to its
vital and core functions.27
Under the circumstances, when the members of the Board of Directors effected the assailed
2002 reorganization, they were acting as the responsible members of the Board of Directors The result of the lengthy consultations and close coordination was the comprehensive
of TIDCORP constituted pursuant to Presidential Decree No. 1080, as amended by Republic reorganization plan that included a new organizational structure, position classification and
Act No. 8494, not as the alter egos of the President. We cannot stretch the application of a staffing pattern, qualification standards, rules and regulations to implement the
doctrine that already delegates an enormous amount of power. Also, it is settled that the reorganization, separation incentive packages and timetable of implementation.
delegation of power is not to be lightly inferred.25 Undoubtedly, TIDCORP effected the reorganization within legal bounds and in response to
the perceived need to make the agency more attuned to the changing times.
Nonetheless, we uphold the 2002 reorganization and declare it valid for being done in
accordance with the exclusive and final authority expressly granted under Republic Act No. Having found the 2002 reorganization to be valid and made pursuant to Republic Act No.
8494, further amending Presidential Decree No. 1080, the law creating TIDCORP itself, to wit: 8494, we declare that there are no legal and practical bases for reinstating Demigillo to her
former position as Senior Vice President in the LCSD. To be sure, the reorganization plan
abolished the LCSD, and put in place a set-up completely different from the previous one,
141

including a new staffing pattern in which Demigillo would be heading the RCMSS, still as a 2.2 Unsatisfactory or Poor Performance
Senior Vice President of TIDCORP. With that abolition, reinstating her as Senior Vice
President in the LCSD became legally and physically impossible. a. An official or employee who is given two (2) consecutive unsatisfactory ratings
may be dropped from the rolls after due notice. Notice shall mean that the officer
Demigillo’s contention that she was specifically appointed to the position of Senior Vice or employee concerned is informed in writing of his unsatisfactory performance for
President in the LCSD was bereft of factual basis. The records indicate that her permanent a semester and is sufficiently warned that a succeeding unsatisfactory performance
appointment pertained only to the position of Senior Vice President.28 Her appointment did shall warrant his separation from the service. Such notice shall be given not later
not indicate at all that she was to hold that specific post in the LCSD. Hence, her re- than 30 days from the end of the semester and shall contain sufficient information
assignment to the RCMSS was by no means a diminution in rank and status considering that which shall enable the employee to prepare an explanation.
she maintained the same rank of Senior Vice President with an accompanying increase in pay
grade. b. An official or employee, who for one evaluation period is rated poor in
performance, may be dropped from the rolls after due notice. Notice shall mean
The assignment to the RCMSS did not also violate Demigillo’s security of tenure as protected that the officer or employee is informed in writing of the status of his performance
by Republic Act No. 6656. We have already upheld reassignments In the Civil Service not later than the 4th month of that rating period with sufficient warning that
resulting from valid reorganizations.29 Nor could she claim that her reassignment was invalid failure to improve his performance within the remaining period of the semester
because it caused the reduction in her rank, status or salary. On the contrary, she was shall warrant his separation from the service. Such notice shall also contain
reappointed as Senior Vice President, a position that was even upgraded like all the other sufficient information which shall enable the employee to prepare an explanation.
similar positions to Pay Grade 16, Step 4, Level II.30 In every sense, the position to which she
was reappointed under the 2002 reorganization was comparable with, if not similar to her Under Section (b), supra, an official or employee may be dropped from the rolls provided the
previous position. following requisites are present, namely: (1) the official or employee was rated poor in
performance for one evaluation period; (2) the official or employee was notified in writing of
That the RCMSS was a unit smaller than the LCSD did not necessarily result in or cause a the status of her performance not later than the 4th month of the rating period with
demotion for Demigillo. Her new position was but the consequence of the valid sufficient warning that failure to improve her performance within the remaining period of
reorganization, the authority to implement which was vested in the Board of Directors by the semester shall warrant her separation from the service; and (3) such notice contained
Republic Act No. 8494. Indeed, we do not consider to be a violation of the civil servant’s right adequate information that would enable her to prepare an explanation.
to security of tenure the exercise by the agency where she works of the essential prerogative
to change the work assignment or to transfer the civil servant to an assignment where she All of the requisites were duly established herein.
would be most useful and effective. More succinctly put, that prerogative inheres with the
employer,31 whether public or private.
As to the first requisite, there is no dispute that President Valdes gave Demigillo a poor
performance rating for the annual rating period from January 1, 2002 to December 31, 2002.
G.R. No. 185571
The second requisite speaks of a sixth-month or per semester rating period. Although
As earlier stated, TIDCORP’s petition for review in G.R. No. 185571 is meritorious. Demigillo’s poor rating was made on an annual basis, that was allowed by the implementing
rules of Executive Order No. 292.32 Regarding the need to give her the written notice of her
Anent the first issue in G.R. No. 185571, we have already explained that Demigillo was not performance status not later than the 4th month of the rating period, or at the half of the
demoted because she did not suffer any diminution in her rank, status and salary under the semester, the requirement did not apply here because her rating was made on an annual
reorganization. Her reassignment to the RCMSS, a smaller unit compared to the LCSD, basis. By analogy, however, the written notice for an annual rating period could be sent on
maintained for her the same rank of Senior Vice-President with a corresponding increase in the 6th month or in the middle of the year. Nevertheless, this was not expressly provided for
pay grade. The reassignment resulted from the valid reorganization. in the Civil Service rules.1âwphi1 In any case, it is emphasized that the purpose of the written
notice being sent to the affected officer or employee not later than the 4th month of the
With respect to the second issue, Demigillo was validly dropped from the rolls by TIDCORP as rating period has been to give her the sufficient time to improve her performance and
the consequence of the application of the rules governing her employment. Section 2 (2.2), thereby avert her separation from the service. That purpose is the very essence of due
Rule XII of the Revised Omnibus Rules on Appointments and Other Personnel Actions process.
(Memorandum Circular No. 40, Series of 1998) provides:
142

In Demigillo’s case, therefore, what was crucial was whether she had been allowed to SO ORDERED.
enhance her performance within a sufficient time from her receipt of the written notice of
the poor performance rating up to her receipt of the written notice of her dropping from the
rolls. The records show that she was, indeed, given enough time for her to show
improvement. She received on April 21, 2003 a letter from President Valdes that indicated
her poor performance rating for the period of January 1, 2002 to December 31, 2002.33 The
Board of Directors issued on August 15, 2003 the decision dropping her from rolls.34 She
received a copy of the decision on August 25, 2003.35Thereby, she was given almost four
months to improve her performance before she was finally dropped from the rolls.

The second requisite further mentions that the written notice must contain sufficient
warning that failure to improve her performance within the remaining period of the
semester shall warrant separation from the service. Although the letter informing Demigillo
of her poor performance rating did not expressly state such a warning to her, it stated her
gross failures in the performance of her duties.36 The Performance Evaluation Report Form
corresponding to her, which was attached to the memorandum given to her, reflected her
poor performance.36 She was notified in writing of the denial of her appeal of the poor
rating.37 It cannot be denied that the letter of poor rating, the Performance Evaluation Repmi
Form, and the denial of her appeal all signified to her that she could be removed from the
service unless she would improve her performance. Thereby, she was given ample warning to
improve, or else be separated from the service. In that regard, she was certainly not a witless
person who could have missed the significance of such events. She was not only a
lawyer. 38 She was also a mid-level ranking government official who had been in the
government corporate sector for almost 20 years.39 Her familiarity with the dire
consequences of a failure to improve a poor rating under Civil Service rules was justifiably
assumed.

Anent the third requisite, the letter of President Valdes plainly stated the reasons for her
poor rating. Her Performance Evaluation Repmi Form, which was attached to the letter,
enumerated several criteria used in measuring her management skills and the corresponding
rating per criterion. The letter even suggested that in order for her to enhance her
performance she should undergo extensive training on business management, a
comprehensive lecture program on Civil Service rules and regulations, and a training on
effective public relations. The letter indicated that the contents of the Performance
Evaluation Report had been discussed with her. Moreover, Demigillo formally appealed the
poor performance rating, except that TIDCORP denied her appeal. 40All these circumstances
show that she was given more than enough information about the bases for her poor
performance rating, enabling her to appeal properly.

WHEREFORE, we DENY the petition for review on certiorari in G.R. No. 168613; AFFIRM the
decision promulgated on June 27, 2005 by the Court of Appeals in its CA-G.R. No. 87285;
GRANT the petition for review on certiorari in G.R. No. 185571; SET ASIDE the decision
promulgated on November 28, 2008 by the Court of Appeals in its CA-G.R. No. 87295; and
ORDER Atty. MA. ROSARIO MANALANG-DEMIGILLO to pay the costs of suit.
143

Manalang-Demigillo vs Trade and Investment Development Corporation of the Philippines Demigillo’s contention that she was specifically appointed to the position of Senior Vice
President in the LCSD was bereft of factual basis. The records indicate that her permanent
G.R. No. 168613 March 5, 2013 appointment pertained only to the position of Senior Vice President. Her appointment did
not indicate at all that she was to hold that specific post in the LCSD. Hence, her re-
Facts: On February 12, 1998, the Philippine Export and Foreign Loan Guarantee was renamed
assignment to the RCMSS was by no means a diminution in rank and status considering that
Trade and Investment Development Corporation of the Philippines (TIDCORP) pursuant to
she maintained the same rank of Senior Vice President with an accompanying increase in pay
Republic Act No. 8494 entitled An Act Further Amending Presidential Decree No. 1080, As
grade.
Amended, by Reorganizing And Renaming the Philippine Export and Foreign Loan Guarantee
Corporation, Expanding Its Primary Purpose, and for Other Purposes. Republic Act No. 8494 The assignment to the RCMSS did not also violate Demigillo’s security of tenure as protected
reorganized the structure of TIDCORP. The issuance of appointments in accordance with the by Republic Act No. 6656. We have already upheld reassignments In the Civil Service
reorganization ensued. Petitioner Rosario Manalang-Demigillo (Demigillo) was appointed as resulting from valid reorganizations. Nor could she claim that her reassignment was invalid
Senior Vice President (PG 15) with permanent status, and was assigned to the Legal and because it caused the reduction in her rank, status or salary. On the contrary, she was
Corporate Services Department (LCSD) of TIDCORP. Petitioner was evaluated and given a reappointed as Senior Vice President, a position that was even upgraded like all the other
‘poor’ rating for two consecutive evaluations due to her unimproved performance resulting similar positions to Pay Grade 16, Step 4, Level II. In every sense, the position to which she
to her name being dropped from the rolls of TIDCORP. was reappointed under the 2002 reorganization was comparable with, if not similar to her
previous position.
Issue: Whether or not the reorganization is valid resulting to Demigillo’s reassignment valid.

Held:

Yes. Under the circumstances, when the members of the Board of Directors effected the
assailed 2002 reorganization, they were acting as the responsible members of the Board of
Directors of TIDCORP constituted pursuant to Presidential Decree No. 1080, as amended by
Republic Act No. 8494, not as the alter egos of the President. We cannot stretch the
application of a doctrine that already delegates an enormous amount of power. Also, it is
settled that the delegation of power is not to be lightly inferred.

The result of the lengthy consultations and close coordination was the comprehensive
reorganization plan that included a new organizational structure, position classification and
staffing pattern, qualification standards, rules and regulations to implement the
reorganization, separation incentive packages and timetable of implementation.
Undoubtedly, TIDCORP effected the reorganization within legal bounds and in response to
the perceived need to make the agency more attuned to the changing times.

Having found the 2002 reorganization to be valid and made pursuant to Republic Act No.
8494, we declare that there are no legal and practical bases for reinstating Demigillo to her
former position as Senior Vice President in the LCSD. To be sure, the reorganization plan
abolished the LCSD, and put in place a setup completely different from the previous one,
including a new staffing pattern in which Demigillo would be heading the RCMSS, still as a
Senior Vice President of TIDCORP. With that abolition, reinstating her as Senior Vice
President in the LCSD became legally and physically impossible.
144

EN BANC certified the bill on January 7, 2005 for immediate enactment. On January 27, 2005, the
House of Representatives approved the bill on second and third reading.
G.R. No. 168056 September 1, 2005
House Bill No. 37053 on the other hand, substituted House Bill No. 3105 introduced by Rep.
ABAKADA GURO PARTY LIST (Formerly AASJAS) OFFICERS SAMSON S. ALCANTARA and ED Salacnib F. Baterina, and House Bill No. 3381 introduced by Rep. Jacinto V. Paras. Its "mother
VINCENT S. ALBANO, Petitioners, bill" is House Bill No. 3555. The House Committee on Ways and Means approved the bill on
vs. February 2, 2005. The President also certified it as urgent on February 8, 2005. The House of
THE HONORABLE EXECUTIVE SECRETARY EDUARDO ERMITA; HONORABLE SECRETARY OF Representatives approved the bill on second and third reading on February 28, 2005.
THE DEPARTMENT OF FINANCE CESAR PURISIMA; and HONORABLE COMMISSIONER OF
INTERNAL REVENUE GUILLERMO PARAYNO, JR., Respondent. Meanwhile, the Senate Committee on Ways and Means approved Senate Bill No. 19504 on
March 7, 2005, "in substitution of Senate Bill Nos. 1337, 1838 and 1873, taking into
DECISION consideration House Bill Nos. 3555 and 3705." Senator Ralph G. Recto sponsored Senate Bill
No. 1337, while Senate Bill Nos. 1838 and 1873 were both sponsored by Sens. Franklin M.
Drilon, Juan M. Flavier and Francis N. Pangilinan. The President certified the bill on March 11,
AUSTRIA-MARTINEZ, J.:
2005, and was approved by the Senate on second and third reading on April 13, 2005.

The expenses of government, having for their object the interest of all, should be borne by
On the same date, April 13, 2005, the Senate agreed to the request of the House of
everyone, and the more man enjoys the advantages of society, the more he ought to hold
Representatives for a committee conference on the disagreeing provisions of the proposed
himself honored in contributing to those expenses.
bills.

-Anne Robert Jacques Turgot (1727-1781)


Before long, the Conference Committee on the Disagreeing Provisions of House Bill No. 3555,
House Bill No. 3705, and Senate Bill No. 1950, "after having met and discussed in full free and
French statesman and economist conference," recommended the approval of its report, which the Senate did on May 10,
2005, and with the House of Representatives agreeing thereto the next day, May 11, 2005.
Mounting budget deficit, revenue generation, inadequate fiscal allocation for education,
increased emoluments for health workers, and wider coverage for full value-added tax On May 23, 2005, the enrolled copy of the consolidated House and Senate version was
benefits … these are the reasons why Republic Act No. 9337 (R.A. No. 9337)1 was enacted. transmitted to the President, who signed the same into law on May 24, 2005. Thus, came
Reasons, the wisdom of which, the Court even with its extensive constitutional power of R.A. No. 9337.
review, cannot probe. The petitioners in these cases, however, question not only the wisdom
of the law, but also perceived constitutional infirmities in its passage.
July 1, 2005 is the effectivity date of R.A. No. 9337.5 When said date came, the Court issued a
temporary restraining order, effective immediately and continuing until further orders,
Every law enjoys in its favor the presumption of constitutionality. Their arguments enjoining respondents from enforcing and implementing the law.
notwithstanding, petitioners failed to justify their call for the invalidity of the law. Hence, R.A.
No. 9337 is not unconstitutional.
Oral arguments were held on July 14, 2005. Significantly, during the hearing, the Court
speaking through Mr. Justice Artemio V. Panganiban, voiced the rationale for its issuance of
LEGISLATIVE HISTORY the temporary restraining order on July 1, 2005, to wit:

R.A. No. 9337 is a consolidation of three legislative bills namely, House Bill Nos. 3555 and J. PANGANIBAN : . . . But before I go into the details of your presentation, let me just tell you
3705, and Senate Bill No. 1950. a little background. You know when the law took effect on July 1, 2005, the Court issued a
TRO at about 5 o’clock in the afternoon. But before that, there was a lot of complaints aired
House Bill No. 35552 was introduced on first reading on January 7, 2005. The House on television and on radio. Some people in a gas station were complaining that the gas prices
Committee on Ways and Means approved the bill, in substitution of House Bill No. 1468, went up by 10%. Some people were complaining that their electric bill will go up by 10%.
which Representative (Rep.) Eric D. Singson introduced on August 8, 2004. The President Other times people riding in domestic air carrier were complaining that the prices that they’ll
have to pay would have to go up by 10%. While all that was being aired, per your
145

presentation and per our own understanding of the law, that’s not true. It’s not true that the added as an issue in this case, even if it’s tangentially taken up by the pleadings of the
e-vat law necessarily increased prices by 10% uniformly isn’t it? parties, the confusion in the implementation of the E-vat. Our people were subjected to the
mercy of that confusion of an across the board increase of 10%, which you yourself now
ATTY. BANIQUED : No, Your Honor. admit and I think even the Government will admit is incorrect. In some cases, it should be 3%
only, in some cases it should be 6% depending on these mitigating measures and the location
and situation of each product, of each service, of each company, isn’t it?
J. PANGANIBAN : It is not?

ATTY. BANIQUED : Yes, Your Honor.


ATTY. BANIQUED : It’s not, because, Your Honor, there is an Executive Order that granted the
Petroleum companies some subsidy . . . interrupted
J. PANGANIBAN : Alright. So that’s one reason why we had to issue a TRO pending the
clarification of all these and we wish the government will take time to clarify all these by
J. PANGANIBAN : That’s correct . . .
means of a more detailed implementing rules, in case the law is upheld by this Court. . . .6

ATTY. BANIQUED : . . . and therefore that was meant to temper the impact . . . interrupted
The Court also directed the parties to file their respective Memoranda.

J. PANGANIBAN : . . . mitigating measures . . .


G.R. No. 168056

ATTY. BANIQUED : Yes, Your Honor.


Before R.A. No. 9337 took effect, petitioners ABAKADA GURO Party List, et al., filed a petition
for prohibition on May 27, 2005. They question the constitutionality of Sections 4, 5 and 6 of
J. PANGANIBAN : As a matter of fact a part of the mitigating measures would be the R.A. No. 9337, amending Sections 106, 107 and 108, respectively, of the National Internal
elimination of the Excise Tax and the import duties. That is why, it is not correct to say that Revenue Code (NIRC). Section 4 imposes a 10% VAT on sale of goods and properties, Section
the VAT as to petroleum dealers increased prices by 10%. 5 imposes a 10% VAT on importation of goods, and Section 6 imposes a 10% VAT on sale of
services and use or lease of properties. These questioned provisions contain a
ATTY. BANIQUED : Yes, Your Honor. uniform proviso authorizing the President, upon recommendation of the Secretary of
Finance, to raise the VAT rate to 12%, effective January 1, 2006, after any of the following
J. PANGANIBAN : And therefore, there is no justification for increasing the retail price by 10% conditions have been satisfied, to wit:
to cover the E-Vat tax. If you consider the excise tax and the import duties, the Net Tax would
probably be in the neighborhood of 7%? We are not going into exact figures I am just trying . . . That the President, upon the recommendation of the Secretary of Finance, shall, effective
to deliver a point that different industries, different products, different services are hit January 1, 2006, raise the rate of value-added tax to twelve percent (12%), after any of the
differently. So it’s not correct to say that all prices must go up by 10%. following conditions has been satisfied:

ATTY. BANIQUED : You’re right, Your Honor. (i) Value-added tax collection as a percentage of Gross Domestic Product (GDP) of the
previous year exceeds two and four-fifth percent (2 4/5%); or
J. PANGANIBAN : Now. For instance, Domestic Airline companies, Mr. Counsel, are at present
imposed a Sales Tax of 3%. When this E-Vat law took effect the Sales Tax was also removed (ii) National government deficit as a percentage of GDP of the previous year exceeds one and
as a mitigating measure. So, therefore, there is no justification to increase the fares by 10% one-half percent (1 ½%).
at best 7%, correct?
Petitioners argue that the law is unconstitutional, as it constitutes abandonment by Congress
ATTY. BANIQUED : I guess so, Your Honor, yes. of its exclusive authority to fix the rate of taxes under Article VI, Section 28(2) of the 1987
Philippine Constitution.
J. PANGANIBAN : There are other products that the people were complaining on that first
day, were being increased arbitrarily by 10%. And that’s one reason among many others this
Court had to issue TRO because of the confusion in the implementation. That’s why we
146

G.R. No. 168207 Petitioners’ argument is premised on the constitutional right of non-deprivation of life,
liberty or property without due process of law under Article III, Section 1 of the Constitution.
On June 9, 2005, Sen. Aquilino Q. Pimentel, Jr., et al., filed a petition for certiorari likewise According to petitioners, the contested sections impose limitations on the amount of input
assailing the constitutionality of Sections 4, 5 and 6 of R.A. No. 9337. tax that may be claimed. Petitioners also argue that the input tax partakes the nature of a
property that may not be confiscated, appropriated, or limited without due process of law.
Petitioners further contend that like any other property or property right, the input tax credit
Aside from questioning the so-called stand-by authority of the President to increase the VAT
may be transferred or disposed of, and that by limiting the same, the government gets to tax
rate to 12%, on the ground that it amounts to an undue delegation of legislative power,
a profit or value-added even if there is no profit or value-added.
petitioners also contend that the increase in the VAT rate to 12% contingent on any of the
two conditions being satisfied violates the due process clause embodied in Article III, Section
1 of the Constitution, as it imposes an unfair and additional tax burden on the people, in that: Petitioners also believe that these provisions violate the constitutional guarantee of equal
(1) the 12% increase is ambiguous because it does not state if the rate would be returned to protection of the law under Article III, Section 1 of the Constitution, as the limitation on the
the original 10% if the conditions are no longer satisfied; (2) the rate is unfair and creditable input tax if: (1) the entity has a high ratio of input tax; or (2) invests in capital
unreasonable, as the people are unsure of the applicable VAT rate from year to year; and (3) equipment; or (3) has several transactions with the government, is not based on real and
the increase in the VAT rate, which is supposed to be an incentive to the President to raise substantial differences to meet a valid classification.
the VAT collection to at least 2 4/5 of the GDP of the previous year, should only be based on
fiscal adequacy. Lastly, petitioners contend that the 70% limit is anything but progressive, violative of Article
VI, Section 28(1) of the Constitution, and that it is the smaller businesses with higher input
Petitioners further claim that the inclusion of a stand-by authority granted to the President tax to output tax ratio that will suffer the consequences thereof for it wipes out whatever
by the Bicameral Conference Committee is a violation of the "no-amendment rule" upon last meager margins the petitioners make.
reading of a bill laid down in Article VI, Section 26(2) of the Constitution.
G.R. No. 168463
G.R. No. 168461
Several members of the House of Representatives led by Rep. Francis Joseph G. Escudero
Thereafter, a petition for prohibition was filed on June 29, 2005, by the Association filed this petition for certiorari on June 30, 2005. They question the constitutionality of R.A.
of Pilipinas Shell Dealers, Inc., et al., assailing the following provisions of R.A. No. 9337: No. 9337 on the following grounds:

1) Section 8, amending Section 110 (A)(2) of the NIRC, requiring that the input tax on 1) Sections 4, 5, and 6 of R.A. No. 9337 constitute an undue delegation of legislative power,
depreciable goods shall be amortized over a 60-month period, if the acquisition, excluding in violation of Article VI, Section 28(2) of the Constitution;
the VAT components, exceeds One Million Pesos (₱1, 000,000.00);
2) The Bicameral Conference Committee acted without jurisdiction in deleting the no pass
2) Section 8, amending Section 110 (B) of the NIRC, imposing a 70% limit on the amount of on provisions present in Senate Bill No. 1950 and House Bill No. 3705; and
input tax to be credited against the output tax; and
3) Insertion by the Bicameral Conference Committee of Sections 27, 28, 34, 116, 117, 119,
3) Section 12, amending Section 114 (c) of the NIRC, authorizing the Government or any of its 121, 125,7 148, 151, 236, 237 and 288, which were present in Senate Bill No. 1950, violates
political subdivisions, instrumentalities or agencies, including GOCCs, to deduct a 5% final Article VI, Section 24(1) of the Constitution, which provides that all appropriation, revenue or
withholding tax on gross payments of goods and services, which are subject to 10% VAT tariff bills shall originate exclusively in the House of Representatives
under Sections 106 (sale of goods and properties) and 108 (sale of services and use or lease
of properties) of the NIRC. G.R. No. 168730

Petitioners contend that these provisions are unconstitutional for being arbitrary, oppressive, On the eleventh hour, Governor Enrique T. Garcia filed a petition for certiorari and
excessive, and confiscatory. prohibition on July 20, 2005, alleging unconstitutionality of the law on the ground that the
limitation on the creditable input tax in effect allows VAT-registered establishments to retain
a portion of the taxes they collect, thus violating the principle that tax collection and revenue
should be solely allocated for public purposes and expenditures. Petitioner Garcia further
147

claims that allowing these establishments to pass on the tax to the consumers is inequitable, 1. Whether Sections 4, 5 and 6 of R.A. No. 9337, amending Sections 106, 107 and 108 of the
in violation of Article VI, Section 28(1) of the Constitution. NIRC, violate the following provisions of the Constitution:

RESPONDENTS’ COMMENT a. Article VI, Section 28(1), and

The Office of the Solicitor General (OSG) filed a Comment in behalf of respondents. b. Article VI, Section 28(2)
Preliminarily, respondents contend that R.A. No. 9337 enjoys the presumption of
constitutionality and petitioners failed to cast doubt on its validity. 2. Whether Section 8 of R.A. No. 9337, amending Sections 110(A)(2) and 110(B) of the NIRC;
and Section 12 of R.A. No. 9337, amending Section 114(C) of the NIRC, violate the following
Relying on the case of Tolentino vs. Secretary of Finance, 235 SCRA provisions of the Constitution:

630 (1994), respondents argue that the procedural issues raised by petitioners, i.e., legality of a. Article VI, Section 28(1), and
the bicameral proceedings, exclusive origination of revenue measures and the power of the
Senate concomitant thereto, have already been settled. With regard to the issue of undue b. Article III, Section 1
delegation of legislative power to the President, respondents contend that the law is
complete and leaves no discretion to the President but to increase the rate to 12% once any
RULING OF THE COURT
of the two conditions provided therein arise.

As a prelude, the Court deems it apt to restate the general principles and concepts of value-
Respondents also refute petitioners’ argument that the increase to 12%, as well as the 70%
added tax (VAT), as the confusion and inevitably, litigation, breeds from a fallacious notion of
limitation on the creditable input tax, the 60-month amortization on the purchase or
its nature.
importation of capital goods exceeding ₱1,000,000.00, and the 5% final withholding tax by
government agencies, is arbitrary, oppressive, and confiscatory, and that it violates the
constitutional principle on progressive taxation, among others. The VAT is a tax on spending or consumption. It is levied on the sale, barter, exchange or
lease of goods or properties and services.8 Being an indirect tax on expenditure, the seller of
goods or services may pass on the amount of tax paid to the buyer,9 with the seller acting
Finally, respondents manifest that R.A. No. 9337 is the anchor of the government’s fiscal
merely as a tax collector.10 The burden of VAT is intended to fall on the immediate buyers
reform agenda. A reform in the value-added system of taxation is the core revenue measure
and ultimately, the end-consumers.
that will tilt the balance towards a sustainable macroeconomic environment necessary for
economic growth.
In contrast, a direct tax is a tax for which a taxpayer is directly liable on the transaction or
business it engages in, without transferring the burden to someone else.11 Examples are
ISSUES
individual and corporate income taxes, transfer taxes, and residence taxes.12

The Court defined the issues, as follows:


In the Philippines, the value-added system of sales taxation has long been in existence, albeit
in a different mode. Prior to 1978, the system was a single-stage tax computed under the
PROCEDURAL ISSUE "cost deduction method" and was payable only by the original sellers. The single-stage
system was subsequently modified, and a mixture of the "cost deduction method" and "tax
Whether R.A. No. 9337 violates the following provisions of the Constitution: credit method" was used to determine the value-added tax payable.13 Under the "tax credit
method," an entity can credit against or subtract from the VAT charged on its sales or
a. Article VI, Section 24, and outputs the VAT paid on its purchases, inputs and imports.14

b. Article VI, Section 26(2) It was only in 1987, when President Corazon C. Aquino issued Executive Order No. 273, that
the VAT system was rationalized by imposing a multi-stage tax rate of 0% or 10% on all sales
using the "tax credit method."15
SUBSTANTIVE ISSUES
148

E.O. No. 273 was followed by R.A. No. 7716 or the Expanded VAT Law,16 R.A. No. 8241 or the procedure, the respective rules of each house of Congress provided for the creation of a
Improved VAT Law,17 R.A. No. 8424 or the Tax Reform Act of 1997,18 and finally, the presently Bicameral Conference Committee.
beleaguered R.A. No. 9337, also referred to by respondents as the VAT Reform Act.
Thus, Rule XIV, Sections 88 and 89 of the Rules of House of Representatives provides as
The Court will now discuss the issues in logical sequence. follows:

PROCEDURAL ISSUE Sec. 88. Conference Committee. – In the event that the House does not agree with the Senate
on the amendment to any bill or joint resolution, the differences may be settled by the
I. conference committees of both chambers.

Whether R.A. No. 9337 violates the following provisions of the Constitution: In resolving the differences with the Senate, the House panel shall, as much as possible,
adhere to and support the House Bill. If the differences with the Senate are so substantial
that they materially impair the House Bill, the panel shall report such fact to the House for
a. Article VI, Section 24, and
the latter’s appropriate action.

b. Article VI, Section 26(2)


Sec. 89. Conference Committee Reports. – . . . Each report shall contain a detailed, sufficiently
explicit statement of the changes in or amendments to the subject measure.
A. The Bicameral Conference Committee
The Chairman of the House panel may be interpellated on the Conference Committee Report
Petitioners Escudero, et al., and Pimentel, et al., allege that the Bicameral Conference prior to the voting thereon. The House shall vote on the Conference Committee Report in the
Committee exceeded its authority by: same manner and procedure as it votes on a bill on third and final reading.

1) Inserting the stand-by authority in favor of the President in Sections 4, 5, and 6 of R.A. No. Rule XII, Section 35 of the Rules of the Senate states:
9337;
Sec. 35. In the event that the Senate does not agree with the House of Representatives on
2) Deleting entirely the no pass-on provisions found in both the House and Senate bills; the provision of any bill or joint resolution, the differences shall be settled by a conference
committee of both Houses which shall meet within ten (10) days after their composition. The
3) Inserting the provision imposing a 70% limit on the amount of input tax to be credited President shall designate the members of the Senate Panel in the conference committee with
against the output tax; and the approval of the Senate.

4) Including the amendments introduced only by Senate Bill No. 1950 regarding other kinds Each Conference Committee Report shall contain a detailed and sufficiently explicit
of taxes in addition to the value-added tax. statement of the changes in, or amendments to the subject measure, and shall be signed by
a majority of the members of each House panel, voting separately.
Petitioners now beseech the Court to define the powers of the Bicameral Conference
Committee. A comparative presentation of the conflicting House and Senate provisions and a reconciled
version thereof with the explanatory statement of the conference committee shall be
It should be borne in mind that the power of internal regulation and discipline are intrinsic in attached to the report.
any legislative body for, as unerringly elucidated by Justice Story, "[i]f the power did not
exist, it would be utterly impracticable to transact the business of the nation, either at all, The creation of such conference committee was apparently in response to a problem, not
or at least with decency, deliberation, and order."19 Thus, Article VI, Section 16 (3) of the addressed by any constitutional provision, where the two houses of Congress find
Constitution provides that "each House may determine the rules of its proceedings." themselves in disagreement over changes or amendments introduced by the other house in
Pursuant to this inherent constitutional power to promulgate and implement its own rules of a legislative bill. Given that one of the most basic powers of the legislative branch is to
formulate and implement its own rules of proceedings and to discipline its members, may
149

the Court then delve into the details of how Congress complies with its internal rules or how provisions in the House and Senate bills. Akin to the Fariñas case,22 the present petitions also
it conducts its business of passing legislation? Note that in the present petitions, the issue is raise an issue regarding the actions taken by the conference committee on matters regarding
not whether provisions of the rules of both houses creating the bicameral conference Congress’ compliance with its own internal rules. As stated earlier, one of the most basic and
committee are unconstitutional, but whether the bicameral conference committee has inherent power of the legislature is the power to formulate rules for its proceedings and the
strictly complied with the rules of both houses, thereby remaining within the jurisdiction discipline of its members. Congress is the best judge of how it should conduct its own
conferred upon it by Congress. business expeditiously and in the most orderly manner. It is also the sole

In the recent case of Fariñas vs. The Executive Secretary,20 the Court En concern of Congress to instill discipline among the members of its conference committee if it
Banc, unanimously reiterated and emphasized its adherence to the "enrolled bill doctrine," believes that said members violated any of its rules of proceedings. Even the expanded
thus, declining therein petitioners’ plea for the Court to go behind the enrolled copy of the jurisdiction of this Court cannot apply to questions regarding only the internal operation of
bill. Assailed in said case was Congress’s creation of two sets of bicameral conference Congress, thus, the Court is wont to deny a review of the internal proceedings of a co-equal
committees, the lack of records of said committees’ proceedings, the alleged violation of said branch of government.
committees of the rules of both houses, and the disappearance or deletion of one of the
provisions in the compromise bill submitted by the bicameral conference committee. It was Moreover, as far back as 1994 or more than ten years ago, in the case of Tolentino vs.
argued that such irregularities in the passage of the law nullified R.A. No. 9006, or the Fair Secretary of Finance,23 the Court already made the pronouncement that "[i]f a change is
Election Act. desired in the practice [of the Bicameral Conference Committee] it must be sought in
Congress since this question is not covered by any constitutional provision but is only an
Striking down such argument, the Court held thus: internal rule of each house." 24 To date, Congress has not seen it fit to make such changes
adverted to by the Court. It seems, therefore, that Congress finds the practices of the
Under the "enrolled bill doctrine," the signing of a bill by the Speaker of the House and the bicameral conference committee to be very useful for purposes of prompt and efficient
Senate President and the certification of the Secretaries of both Houses of Congress that it legislative action.
was passed are conclusive of its due enactment. A review of cases reveals the Court’s
consistent adherence to the rule. The Court finds no reason to deviate from the salutary Nevertheless, just to put minds at ease that no blatant irregularities tainted the proceedings
rule in this case where the irregularities alleged by the petitioners mostly involved the of the bicameral conference committees, the Court deems it necessary to dwell on the issue.
internal rules of Congress, e.g., creation of the 2nd or 3rd Bicameral Conference Committee The Court observes that there was a necessity for a conference committee because a
by the House. This Court is not the proper forum for the enforcement of these internal comparison of the provisions of House Bill Nos. 3555 and 3705 on one hand, and Senate Bill
rules of Congress, whether House or Senate. Parliamentary rules are merely procedural No. 1950 on the other, reveals that there were indeed disagreements. As pointed out in the
and with their observance the courts have no concern. Whatever doubts there may be as petitions, said disagreements were as follows:
to the formal validity of Rep. Act No. 9006 must be resolved in its favor.The Court reiterates
its ruling in Arroyo vs. De Venecia, viz.: House Bill No. 3555 House Bill No.3705 Senate Bill No. 1950
With regard to "Stand-By Authority" in favor of President
But the cases, both here and abroad, in varying forms of expression, all deny to the courts Provides for 12% VAT on every Provides for 12% VAT in general Provides for a single rate
the power to inquire into allegations that, in enacting a law, a House of Congress failed to sale of goods or properties on sales of goods or properties of 10% VAT on sale of
comply with its own rules, in the absence of showing that there was a violation of a (amending Sec. 106 of NIRC); and reduced rates for sale of goods or properties
constitutional provision or the rights of private individuals. In Osmeña v. Pendatun, it was 12% VAT on importation of certain locally manufactured (amending Sec. 106 of
held: "At any rate, courts have declared that ‘the rules adopted by deliberative bodies are goods (amending Sec. 107 of goods and petroleum products NIRC), 10% VAT on sale
subject to revocation, modification or waiver at the pleasure of the body adopting NIRC); and 12% VAT on sale of and raw materials to be used in of services including sale
them.’ And it has been said that "Parliamentary rules are merely procedural, and with their services and use or lease of the manufacture thereof of electricity by
observance, the courts have no concern. They may be waived or disregarded by the properties (amending Sec. 108 (amending Sec. 106 of NIRC); generation companies,
legislative body." Consequently, "mere failure to conform to parliamentary usage will not of NIRC) 12% VAT on importation of transmission and
invalidate the action (taken by a deliberative body) when the requisite number of goods and reduced rates for distribution companies,
members have agreed to a particular measure."21 (Emphasis supplied) certain imported products and use or lease of
including petroleum products properties (amending
The foregoing declaration is exactly in point with the present cases, where petitioners allege (amending Sec. 107 of NIRC); and Sec. 108 of NIRC)
irregularities committed by the conference committee in introducing changes or deleting 12% VAT on sale of services and
150

use or lease of properties and a The disagreements between the provisions in the House bills and the Senate bill were with
reduced rate for certain services regard to (1) what rate of VAT is to be imposed; (2) whether only the VAT imposed on
including power generation electricity generation, transmission and distribution companies should not be passed on to
(amending Sec. 108 of NIRC) consumers, as proposed in the Senate bill, or both the VAT imposed on electricity generation,
With regard to the "no pass-on" provision transmission and distribution companies and the VAT imposed on sale of petroleum products
No similar provision Provides that the VAT imposed Provides that the VAT should not be passed on to consumers, as proposed in the House bill; (3) in what manner
on power generation and on the imposed on sales of input tax credits should be limited; (4) and whether the NIRC provisions on corporate income
sale of petroleum products shall electricity by generation taxes, percentage, franchise and excise taxes should be amended.
be absorbed by generation companies and services
companies or sellers, of transmission There being differences and/or disagreements on the foregoing provisions of the House and
respectively, and shall not be companies and Senate bills, the Bicameral Conference Committee was mandated by the rules of both houses
passed on to consumers distribution companies, of Congress to act on the same by settling said differences and/or disagreements. The
as well as those of Bicameral Conference Committee acted on the disagreeing provisions by making the
franchise grantees of following changes:
electric utilities shall not
apply to residential 1. With regard to the disagreement on the rate of VAT to be imposed, it would appear from
the Conference Committee Report that the Bicameral Conference Committee tried to bridge
end-users. VAT shall be the gap in the difference between the 10% VAT rate proposed by the Senate, and the various
absorbed by generation, rates with 12% as the highest VAT rate proposed by the House, by striking a compromise
transmission, and whereby the present 10% VAT rate would be retained until certain conditions arise, i.e., the
distribution companies. value-added tax collection as a percentage of gross domestic product (GDP) of the previous
With regard to 70% limit on input tax credit year exceeds 2 4/5%, or National Government deficit as a percentage of GDP of the previous
Provides that the input tax No similar provision Provides that the input year exceeds 1½%, when the President, upon recommendation of the Secretary of Finance
credit for capital goods on tax credit for capital shall raise the rate of VAT to 12% effective January 1, 2006.
which a VAT has been paid goods on which a VAT
shall be equally distributed has been paid shall be 2. With regard to the disagreement on whether only the VAT imposed on electricity
over 5 years or the depreciable equally distributed over 5 generation, transmission and distribution companies should not be passed on to consumers
life of such capital goods; the years or the depreciable or whether both the VAT imposed on electricity generation, transmission and distribution
input tax credit for goods and life of such capital goods; companies and the VAT imposed on sale of petroleum products may be passed on to
services other than capital the input tax credit for consumers, the Bicameral Conference Committee chose to settle such disagreement by
goods shall not exceed 5% of goods and services other altogether deleting from its Report any no pass-on provision.
the total amount of such goods than capital goods shall
and services; and for persons not exceed 90% of the 3. With regard to the disagreement on whether input tax credits should be limited or not, the
engaged in retail trading of output VAT. Bicameral Conference Committee decided to adopt the position of the House by putting a
goods, the allowable input tax limitation on the amount of input tax that may be credited against the output tax, although it
credit shall not exceed 11% of crafted its own language as to the amount of the limitation on input tax credits and the
the total amount of goods manner of computing the same by providing thus:
purchased.
With regard to amendments to be made to NIRC provisions regarding income and excise (A) Creditable Input Tax. – . . .
taxes
No similar No similar Provided for amendments to several NIRC provisions
Provided, The input tax on goods purchased or imported in a calendar month for use in trade
provision provision regarding corporate income, percentage, franchise and excise
or business for which deduction for depreciation is allowed under this Code, shall be spread
taxes
evenly over the month of acquisition and the fifty-nine (59) succeeding months if the
aggregate acquisition cost for such goods, excluding the VAT component thereof, exceeds
one million Pesos (₱1,000,000.00): PROVIDED, however, that if the estimated useful life of
151

the capital good is less than five (5) years, as used for depreciation purposes, then the input . . . the thinking was just to keep the VAT law or the VAT bill simple. And we were thinking
VAT shall be spread over such shorter period: . . . that no sector should be a beneficiary of legislative grace, neither should any sector be
discriminated on. The VAT is an indirect tax. It is a pass on-tax. And let’s keep it plain and
(B) Excess Output or Input Tax. – If at the end of any taxable quarter the output tax exceeds simple. Let’s not confuse the bill and put a no pass-on provision. Two-thirds of the world
the input tax, the excess shall be paid by the VAT-registered person. If the input tax exceeds have a VAT system and in this two-thirds of the globe, I have yet to see a VAT with a no pass-
the output tax, the excess shall be carried over to the succeeding quarter or quarters: though provision. So, the thinking of the Senate is basically simple, let’s keep the VAT
PROVIDED that the input tax inclusive of input VAT carried over from the previous quarter simple.26 (Emphasis supplied)
that may be credited in every quarter shall not exceed seventy percent (70%) of the output
VAT: PROVIDED, HOWEVER, THAT any input tax attributable to zero-rated sales by a VAT- Rep. Teodoro Locsin further made the manifestation that the no pass-on provision "never
registered person may at his option be refunded or credited against other internal revenue really enjoyed the support of either House."27
taxes, . . .
With regard to the amount of input tax to be credited against output tax, the Bicameral
4. With regard to the amendments to other provisions of the NIRC on corporate income tax, Conference Committee came to a compromise on the percentage rate of the limitation or
franchise, percentage and excise taxes, the conference committee decided to include such cap on such input tax credit, but again, the change introduced by the Bicameral Conference
amendments and basically adopted the provisions found in Senate Bill No. 1950, with some Committee was totally within the intent of both houses to put a cap on input tax that may be
changes as to the rate of the tax to be imposed.
credited against the output tax. From the inception of the subject revenue bill in the House
Under the provisions of both the Rules of the House of Representatives and Senate Rules, the of Representatives, one of the major objectives was to "plug a glaring loophole in the tax
Bicameral Conference Committee is mandated to settle the differences between the policy and administration by creating vital restrictions on the claiming of input VAT tax
disagreeing provisions in the House bill and the Senate bill. The term "settle" is synonymous credits . . ." and "[b]y introducing limitations on the claiming of tax credit, we are capping a
to "reconcile" and "harmonize."25 To reconcile or harmonize disagreeing provisions, the major leakage that has placed our collection efforts at an apparent disadvantage." 28
Bicameral Conference Committee may then (a) adopt the specific provisions of either the
House bill or Senate bill, (b) decide that neither provisions in the House bill or the provisions As to the amendments to NIRC provisions on taxes other than the value-added tax proposed
in the Senate bill would in Senate Bill No. 1950, since said provisions were among those referred to it, the conference
committee had to act on the same and it basically adopted the version of the Senate.
be carried into the final form of the bill, and/or (c) try to arrive at a compromise between the
disagreeing provisions. Thus, all the changes or modifications made by the Bicameral Conference Committee were
germane to subjects of the provisions referred
In the present case, the changes introduced by the Bicameral Conference Committee on
disagreeing provisions were meant only to reconcile and harmonize the disagreeing to it for reconciliation. Such being the case, the Court does not see any grave abuse of
provisions for it did not inject any idea or intent that is wholly foreign to the subject discretion amounting to lack or excess of jurisdiction committed by the Bicameral Conference
embraced by the original provisions. Committee. In the earlier cases of Philippine Judges Association vs. Prado29 and Tolentino vs.
Secretary of Finance,30 the Court recognized the long-standing legislative practice of giving
The so-called stand-by authority in favor of the President, whereby the rate of 10% VAT said conference committee ample latitude for compromising differences between the Senate
wanted by the Senate is retained until such time that certain conditions arise when the 12% and the House. Thus, in the Tolentino case, it was held that:
VAT wanted by the House shall be imposed, appears to be a compromise to try to bridge the
difference in the rate of VAT proposed by the two houses of Congress. Nevertheless, such . . . it is within the power of a conference committee to include in its report an entirely new
compromise is still totally within the subject of what rate of VAT should be imposed on provision that is not found either in the House bill or in the Senate bill. If the committee can
taxpayers. propose an amendment consisting of one or two provisions, there is no reason why it cannot
propose several provisions, collectively considered as an "amendment in the nature of a
The no pass-on provision was deleted altogether. In the transcripts of the proceedings of the substitute," so long as such amendment is germane to the subject of the bills before the
Bicameral Conference Committee held on May 10, 2005, Sen. Ralph Recto, Chairman of the committee. After all, its report was not final but needed the approval of both houses of
Senate Panel, explained the reason for deleting the no pass-on provision in this wise: Congress to become valid as an act of the legislative department. The charge that in this case
152

the Conference Committee acted as a third legislative chamber is thus without any Coming to the issue of the validity of the amendments made regarding the NIRC provisions
basis.31 (Emphasis supplied) on corporate income taxes and percentage, excise taxes. Petitioners refer to the following
provisions, to wit:
B. R.A. No. 9337 Does Not Violate Article VI, Section 26(2) of the Constitution on the "No-
Amendment Rule" Section 27 Rates of Income Tax on Domestic Corporation
28(A)(1) Tax on Resident Foreign Corporation
Article VI, Sec. 26 (2) of the Constitution, states: 28(B)(1) Inter-corporate Dividends
34(B)(1) Inter-corporate Dividends
No bill passed by either House shall become a law unless it has passed three readings on 116 Tax on Persons Exempt from VAT
separate days, and printed copies thereof in its final form have been distributed to its 117 Percentage Tax on domestic carriers and keepers of Garage
Members three days before its passage, except when the President certifies to the necessity 119 Tax on franchises
of its immediate enactment to meet a public calamity or emergency. Upon the last reading of
121 Tax on banks and Non-Bank Financial Intermediaries
a bill, no amendment thereto shall be allowed, and the vote thereon shall be taken
immediately thereafter, and the yeas and nays entered in the Journal. 148 Excise Tax on manufactured oils and other fuels
151 Excise Tax on mineral products
Petitioners’ argument that the practice where a bicameral conference committee is allowed 236 Registration requirements
to add or delete provisions in the House bill and the Senate bill after these had passed three 237 Issuance of receipts or sales or commercial invoices
readings is in effect a circumvention of the "no amendment rule" (Sec. 26 (2), Art. VI of the 288 Disposition of Incremental Revenue
1987 Constitution), fails to convince the Court to deviate from its ruling in the Tolentino case
that: Petitioners claim that the amendments to these provisions of the NIRC did not at all originate
from the House. They aver that House Bill No. 3555 proposed amendments only regarding
Nor is there any reason for requiring that the Committee’s Report in these cases must have Sections 106, 107, 108, 110 and 114 of the NIRC, while House Bill No. 3705 proposed
undergone three readings in each of the two houses. If that be the case, there would be no amendments only to Sections 106, 107,108, 109, 110 and 111 of the NIRC; thus, the other
end to negotiation since each house may seek modification of the compromise bill. . . . sections of the NIRC which the Senate amended but which amendments were not found in
the House bills are not intended to be amended by the House of Representatives. Hence,
Art. VI. § 26 (2) must, therefore, be construed as referring only to bills introduced for the they argue that since the proposed amendments did not originate from the House, such
first time in either house of Congress, not to the conference committee report. 32 (Emphasis amendments are a violation of Article VI, Section 24 of the Constitution.
supplied)
The argument does not hold water.
The Court reiterates here that the "no-amendment rule" refers only to the procedure to be
followed by each house of Congress with regard to bills initiated in each of said respective Article VI, Section 24 of the Constitution reads:
houses, before said bill is transmitted to the other house for its concurrence or
amendment. Verily, to construe said provision in a way as to proscribe any further changes Sec. 24. All appropriation, revenue or tariff bills, bills authorizing increase of the public debt,
to a bill after one house has voted on it would lead to absurdity as this would mean that the bills of local application, and private bills shall originate exclusively in the House of
other house of Congress would be deprived of its constitutional power to amend or Representatives but the Senate may propose or concur with amendments.
introduce changes to said bill. Thus, Art. VI, Sec. 26 (2) of the Constitution cannot be taken to
mean that the introduction by the Bicameral Conference Committee of amendments and
In the present cases, petitioners admit that it was indeed House Bill Nos. 3555 and 3705 that
modifications to disagreeing provisions in bills that have been acted upon by both houses of
initiated the move for amending provisions of the NIRC dealing mainly with the value-added
Congress is prohibited.
tax. Upon transmittal of said House bills to the Senate, the Senate came out with Senate Bill
No. 1950 proposing amendments not only to NIRC provisions on the value-added tax but also
C. R.A. No. 9337 Does Not Violate Article VI, Section 24 of the Constitution on Exclusive amendments to NIRC provisions on other kinds of taxes. Is the introduction by the Senate of
Origination of Revenue Bills provisions not dealing directly with the value- added tax, which is the only kind of tax being
amended in the House bills, still within the purview of the constitutional provision
153

authorizing the Senate to propose or concur with amendments to a revenue bill that One of the challenges faced by the present administration is the urgent and daunting task of
originated from the House? solving the country’s serious financial problems. To do this, government expenditures must
be strictly monitored and controlled and revenues must be significantly increased. This may
The foregoing question had been squarely answered in the Tolentino case, wherein the Court be easier said than done, but our fiscal authorities are still optimistic the government will be
held, thus: operating on a balanced budget by the year 2009. In fact, several measures that will result to
significant expenditure savings have been identified by the administration. It is supported
with a credible package of revenue measures that include measures to improve tax
. . . To begin with, it is not the law – but the revenue bill – which is required by the
administration and control the leakages in revenues from income taxes and the value-
Constitution to "originate exclusively" in the House of Representatives. It is important to
added tax (VAT). (Emphasis supplied)
emphasize this, because a bill originating in the House may undergo such extensive changes
in the Senate that the result may be a rewriting of the whole. . . . At this point, what is
important to note is that, as a result of the Senate action, a distinct bill may be produced. To Rep. Eric D. Singson, in his sponsorship speech for House Bill No. 3555, declared that:
insist that a revenue statute – and not only the bill which initiated the legislative process
culminating in the enactment of the law – must substantially be the same as the House bill In the budget message of our President in the year 2005, she reiterated that we all
would be to deny the Senate’s power not only to "concur with amendments" but also to acknowledged that on top of our agenda must be the restoration of the health of our fiscal
"propose amendments." It would be to violate the coequality of legislative power of the two system.
houses of Congress and in fact make the House superior to the Senate.
In order to considerably lower the consolidated public sector deficit and eventually achieve a
…Given, then, the power of the Senate to propose amendments, the Senate can propose its balanced budget by the year 2009, we need to seize windows of opportunities which might
own version even with respect to bills which are required by the Constitution to originate seem poignant in the beginning, but in the long run prove effective and beneficial to the
in the House. overall status of our economy. One such opportunity is a review of existing tax rates,
evaluating the relevance given our present conditions.34 (Emphasis supplied)
Indeed, what the Constitution simply means is that the initiative for filing revenue, tariff or
tax bills, bills authorizing an increase of the public debt, private bills and bills of local Notably therefore, the main purpose of the bills emanating from the House of
application must come from the House of Representatives on the theory that, elected as they Representatives is to bring in sizeable revenues for the government
are from the districts, the members of the House can be expected to be more sensitive to
the local needs and problems. On the other hand, the senators, who are elected at large, to supplement our country’s serious financial problems, and improve tax administration and
are expected to approach the same problems from the national perspective. Both views control of the leakages in revenues from income taxes and value-added taxes. As these
are thereby made to bear on the enactment of such laws.33 (Emphasis supplied) house bills were transmitted to the Senate, the latter, approaching the measures from the
point of national perspective, can introduce amendments within the purposes of those bills.
Since there is no question that the revenue bill exclusively originated in the House of It can provide for ways that would soften the impact of the VAT measure on the
Representatives, the Senate was acting within its consumer, i.e., by distributing the burden across all sectors instead of putting it entirely on
the shoulders of the consumers. The sponsorship speech of Sen. Ralph Recto on why the
constitutional power to introduce amendments to the House bill when it included provisions provisions on income tax on corporation were included is worth quoting:
in Senate Bill No. 1950 amending corporate income taxes, percentage, excise and franchise
taxes. Verily, Article VI, Section 24 of the Constitution does not contain any prohibition or All in all, the proposal of the Senate Committee on Ways and Means will raise ₱64.3 billion in
limitation on the extent of the amendments that may be introduced by the Senate to the additional revenues annually even while by mitigating prices of power, services and
House revenue bill. petroleum products.

Furthermore, the amendments introduced by the Senate to the NIRC provisions that had not However, not all of this will be wrung out of VAT. In fact, only ₱48.7 billion amount is from
been touched in the House bills are still in furtherance of the intent of the House in initiating the VAT on twelve goods and services. The rest of the tab – ₱10.5 billion- will be picked by
the subject revenue bills. The Explanatory Note of House Bill No. 1468, the very first House corporations.
bill introduced on the floor, which was later substituted by House Bill No. 3555, stated:
154

What we therefore prescribe is a burden sharing between corporate Philippines and the The other sections amended by the Senate pertained to matters of tax administration which
consumer. Why should the latter bear all the pain? Why should the fiscal salvation be only on are necessary for the implementation of the changes in the VAT system.
the burden of the consumer?
To reiterate, the sections introduced by the Senate are germane to the subject matter and
The corporate world’s equity is in form of the increase in the corporate income tax from 32 purposes of the house bills, which is to supplement our country’s fiscal deficit, among others.
to 35 percent, but up to 2008 only. This will raise ₱10.5 billion a year. After that, the rate will Thus, the Senate acted within its power to propose those amendments.
slide back, not to its old rate of 32 percent, but two notches lower, to 30 percent.
SUBSTANTIVE ISSUES
Clearly, we are telling those with the capacity to pay, corporations, to bear with this
emergency provision that will be in effect for 1,200 days, while we put our fiscal house in I.
order. This fiscal medicine will have an expiry date.
Whether Sections 4, 5 and 6 of R.A. No. 9337, amending Sections 106, 107 and 108 of the
For their assistance, a reward of tax reduction awaits them. We intend to keep the length of NIRC, violate the following provisions of the Constitution:
their sacrifice brief. We would like to assure them that not because there is a light at the end
of the tunnel, this government will keep on making the tunnel long.
a. Article VI, Section 28(1), and

The responsibility will not rest solely on the weary shoulders of the small man. Big business
b. Article VI, Section 28(2)
will be there to share the burden.35

A. No Undue Delegation of Legislative Power


As the Court has said, the Senate can propose amendments and in fact, the amendments
made on provisions in the tax on income of corporations are germane to the purpose of the
house bills which is to raise revenues for the government. Petitioners ABAKADA GURO Party List, et al., Pimentel, Jr., et al., and Escudero, et al. contend
in common that Sections 4, 5 and 6 of R.A. No. 9337, amending Sections 106, 107 and 108,
respectively, of the NIRC giving the President the stand-by authority to raise the VAT rate
Likewise, the Court finds the sections referring to other percentage and excise taxes germane
from 10% to 12% when a certain condition is met, constitutes undue delegation of the
to the reforms to the VAT system, as these sections would cushion the effects of VAT on
legislative power to tax.
consumers. Considering that certain goods and services which were subject to percentage
tax and excise tax would no longer be VAT-exempt, the consumer would be burdened more
as they would be paying the VAT in addition to these taxes. Thus, there is a need to amend The assailed provisions read as follows:
these sections to soften the impact of VAT. Again, in his sponsorship speech, Sen. Recto said:
SEC. 4. Sec. 106 of the same Code, as amended, is hereby further amended to read as
However, for power plants that run on oil, we will reduce to zero the present excise tax on follows:
bunker fuel, to lessen the effect of a VAT on this product.
SEC. 106. Value-Added Tax on Sale of Goods or Properties. –
For electric utilities like Meralco, we will wipe out the franchise tax in exchange for a VAT.
(A) Rate and Base of Tax. – There shall be levied, assessed and collected on every sale, barter
And in the case of petroleum, while we will levy the VAT on oil products, so as not to destroy or exchange of goods or properties, a value-added tax equivalent to ten percent (10%) of the
the VAT chain, we will however bring down the excise tax on socially sensitive products such gross selling price or gross value in money of the goods or properties sold, bartered or
as diesel, bunker, fuel and kerosene. exchanged, such tax to be paid by the seller or transferor: provided, that the President,
upon the recommendation of the Secretary of Finance, shall, effective January 1, 2006,
raise the rate of value-added tax to twelve percent (12%), after any of the following
What do all these exercises point to? These are not contortions of giving to the left hand
conditions has been satisfied.
what was taken from the right. Rather, these sprang from our concern of softening the
impact of VAT, so that the people can cushion the blow of higher prices they will have to pay
as a result of VAT.36 (i) value-added tax collection as a percentage of Gross Domestic Product (GDP) of the
previous year exceeds two and four-fifth percent (2 4/5%) or
155

(ii) national government deficit as a percentage of GDP of the previous year exceeds one Petitioners allege that the grant of the stand-by authority to the President to increase the
and one-half percent (1 ½%). VAT rate is a virtual abdication by Congress of its exclusive power to tax because such
delegation is not within the purview of Section 28 (2), Article VI of the Constitution, which
SEC. 5. Section 107 of the same Code, as amended, is hereby further amended to read as provides:
follows:
The Congress may, by law, authorize the President to fix within specified limits, and may
SEC. 107. Value-Added Tax on Importation of Goods. – impose, tariff rates, import and export quotas, tonnage and wharfage dues, and other duties
or imposts within the framework of the national development program of the government.
(A) In General. – There shall be levied, assessed and collected on every importation of goods
a value-added tax equivalent to ten percent (10%) based on the total value used by the They argue that the VAT is a tax levied on the sale, barter or exchange of goods and
Bureau of Customs in determining tariff and customs duties, plus customs duties, excise properties as well as on the sale or exchange of services, which cannot be included within the
taxes, if any, and other charges, such tax to be paid by the importer prior to the release of purview of tariffs under the exempted delegation as the latter refers to customs duties, tolls
such goods from customs custody: Provided, That where the customs duties are determined or tribute payable upon merchandise to the government and usually imposed on goods or
on the basis of the quantity or volume of the goods, the value-added tax shall be based on merchandise imported or exported.
the landed cost plus excise taxes, if any: provided, further, that the President, upon the
recommendation of the Secretary of Finance, shall, effective January 1, 2006, raise the rate Petitioners ABAKADA GURO Party List, et al., further contend that delegating to the President
of value-added tax to twelve percent (12%) after any of the following conditions has been the legislative power to tax is contrary to republicanism. They insist that accountability,
satisfied. responsibility and transparency should dictate the actions of Congress and they should not
pass to the President the decision to impose taxes. They also argue that the law also
(i) value-added tax collection as a percentage of Gross Domestic Product (GDP) of the effectively nullified the President’s power of control, which includes the authority to set
previous year exceeds two and four-fifth percent (2 4/5%) or aside and nullify the acts of her subordinates like the Secretary of Finance, by mandating the
fixing of the tax rate by the President upon the recommendation of the Secretary of Finance.
(ii) national government deficit as a percentage of GDP of the previous year exceeds one
and one-half percent (1 ½%). Petitioners Pimentel, et al. aver that the President has ample powers to cause, influence or
create the conditions provided by the law to bring about either or both the conditions
precedent.
SEC. 6. Section 108 of the same Code, as amended, is hereby further amended to read as
follows:
On the other hand, petitioners Escudero, et al. find bizarre and revolting the situation that
the imposition of the 12% rate would be subject to the whim of the Secretary of Finance, an
SEC. 108. Value-added Tax on Sale of Services and Use or Lease of Properties –
unelected bureaucrat, contrary to the principle of no taxation without representation. They
submit that the Secretary of Finance is not mandated to give a favorable recommendation
(A) Rate and Base of Tax. – There shall be levied, assessed and collected, a value-added tax and he may not even give his recommendation. Moreover, they allege that no guiding
equivalent to ten percent (10%) of gross receipts derived from the sale or exchange of standards are provided in the law on what basis and as to how he will make his
services: provided, that the President, upon the recommendation of the Secretary of recommendation. They claim, nonetheless, that any recommendation of the Secretary of
Finance, shall, effective January 1, 2006, raise the rate of value-added tax to twelve percent Finance can easily be brushed aside by the President since the former is a mere alter ego of
(12%), after any of the following conditions has been satisfied. the latter, such that, ultimately, it is the President who decides whether to impose the
increased tax rate or not.
(i) value-added tax collection as a percentage of Gross Domestic Product (GDP) of the
previous year exceeds two and four-fifth percent (2 4/5%) or A brief discourse on the principle of non-delegation of powers is instructive.

(ii) national government deficit as a percentage of GDP of the previous year exceeds one The principle of separation of powers ordains that each of the three great branches of
and one-half percent (1 ½%). (Emphasis supplied) government has exclusive cognizance of and is supreme in matters falling within its own
constitutionally allocated sphere.37 A logical
156

corollary to the doctrine of separation of powers is the principle of non-delegation of the delegate, who is not allowed to step into the shoes of the legislature and exercise a
powers, as expressed in the Latin maxim: potestas delegata non delegari potest which means power essentially legislative.44
"what has been delegated, cannot be delegated."38 This doctrine is based on the ethical
principle that such as delegated power constitutes not only a right but a duty to be In People vs. Vera,45 the Court, through eminent Justice Jose P. Laurel, expounded on the
performed by the delegate through the instrumentality of his own judgment and not through concept and extent of delegation of power in this wise:
the intervening mind of another.39
In testing whether a statute constitutes an undue delegation of legislative power or not, it is
With respect to the Legislature, Section 1 of Article VI of the Constitution provides that "the usual to inquire whether the statute was complete in all its terms and provisions when it left
Legislative power shall be vested in the Congress of the Philippines which shall consist of a the hands of the legislature so that nothing was left to the judgment of any other appointee
Senate and a House of Representatives." The powers which Congress is prohibited from or delegate of the legislature.
delegating are those which are strictly, or inherently and exclusively, legislative. Purely
legislative power, which can never be delegated, has been described as the authority to
‘The true distinction’, says Judge Ranney, ‘is between the delegation of power to make the
make a complete law – complete as to the time when it shall take effect and as to whom it
law, which necessarily involves a discretion as to what it shall be, and conferring an
shall be applicable – and to determine the expediency of its enactment.40 Thus, the rule is
authority or discretion as to its execution, to be exercised under and in pursuance of the
that in order that a court may be justified in holding a statute unconstitutional as a
law. The first cannot be done; to the latter no valid objection can be made.’
delegation of legislative power, it must appear that the power involved is purely legislative in
nature – that is, one appertaining exclusively to the legislative department. It is the nature of
the power, and not the liability of its use or the manner of its exercise, which determines the It is contended, however, that a legislative act may be made to the effect as law after it
validity of its delegation. leaves the hands of the legislature. It is true that laws may be made effective on certain
contingencies, as by proclamation of the executive or the adoption by the people of a
particular community. In Wayman vs. Southard, the Supreme Court of the United States
Nonetheless, the general rule barring delegation of legislative powers is subject to the
ruled that the legislature may delegate a power not legislative which it may itself rightfully
following recognized limitations or exceptions:
exercise. The power to ascertain facts is such a power which may be delegated. There is
nothing essentially legislative in ascertaining the existence of facts or conditions as the
(1) Delegation of tariff powers to the President under Section 28 (2) of Article VI of the basis of the taking into effect of a law. That is a mental process common to all branches of
Constitution; the government. Notwithstanding the apparent tendency, however, to relax the rule
prohibiting delegation of legislative authority on account of the complexity arising from social
(2) Delegation of emergency powers to the President under Section 23 (2) of Article VI of the and economic forces at work in this modern industrial age, the orthodox pronouncement of
Constitution; Judge Cooley in his work on Constitutional Limitations finds restatement in Prof. Willoughby's
treatise on the Constitution of the United States in the following language — speaking of
(3) Delegation to the people at large; declaration of legislative power to administrative agencies: The principle which permits the
legislature to provide that the administrative agent may determine when the
circumstances are such as require the application of a law is defended upon the ground
(4) Delegation to local governments; and
that at the time this authority is granted, the rule of public policy, which is the essence of
the legislative act, is determined by the legislature. In other words, the legislature, as it is
(5) Delegation to administrative bodies. its duty to do, determines that, under given circumstances, certain executive or
administrative action is to be taken, and that, under other circumstances, different or no
In every case of permissible delegation, there must be a showing that the delegation itself is action at all is to be taken. What is thus left to the administrative official is not the
valid. It is valid only if the law (a) is complete in itself, setting forth therein the policy to be legislative determination of what public policy demands, but simply the ascertainment of
executed, carried out, or implemented by the delegate;41 and (b) fixes a standard — the what the facts of the case require to be done according to the terms of the law by which he
limits of which are sufficiently determinate and determinable — to which the delegate must is governed. The efficiency of an Act as a declaration of legislative will must, of course,
conform in the performance of his functions.42 A sufficient standard is one which defines come from Congress, but the ascertainment of the contingency upon which the Act shall
legislative policy, marks its limits, maps out its boundaries and specifies the public agency to take effect may be left to such agencies as it may designate. The legislature, then, may
apply it. It indicates the circumstances under which the legislative command is to be provide that a law shall take effect upon the happening of future specified contingencies
effected.43 Both tests are intended to prevent a total transference of legislative authority to leaving to some other person or body the power to determine when the specified
contingency has arisen. (Emphasis supplied).46
157

In Edu vs. Ericta,47 the Court reiterated: (i) Value-added tax collection as a percentage of Gross Domestic Product (GDP) of the
previous year exceeds two and four-fifth percent (2 4/5%); or
What cannot be delegated is the authority under the Constitution to make laws and to alter
and repeal them; the test is the completeness of the statute in all its terms and provisions (ii) National government deficit as a percentage of GDP of the previous year exceeds one and
when it leaves the hands of the legislature. To determine whether or not there is an undue one-half percent (1 ½%).
delegation of legislative power, the inquiry must be directed to the scope and definiteness of
the measure enacted. The legislative does not abdicate its functions when it describes what The case before the Court is not a delegation of legislative power. It is simply a delegation of
job must be done, who is to do it, and what is the scope of his authority. For a complex ascertainment of facts upon which enforcement and administration of the increase rate
economy, that may be the only way in which the legislative process can go forward. A under the law is contingent. The legislature has made the operation of the 12% rate effective
distinction has rightfully been made between delegation of power to make the laws which January 1, 2006, contingent upon a specified fact or condition. It leaves the entire operation
necessarily involves a discretion as to what it shall be, which constitutionally may not be or non-operation of the 12% rate upon factual matters outside of the control of the
done, and delegation of authority or discretion as to its execution to be exercised under executive.
and in pursuance of the law, to which no valid objection can be made. The Constitution is
thus not to be regarded as denying the legislature the necessary resources of flexibility and
No discretion would be exercised by the President. Highlighting the absence of discretion is
practicability. (Emphasis supplied).48
the fact that the word shall is used in the common proviso. The use of the
word shall connotes a mandatory order. Its use in a statute denotes an imperative obligation
Clearly, the legislature may delegate to executive officers or bodies the power to determine and is inconsistent with the idea of discretion.53 Where the law is clear and unambiguous, it
certain facts or conditions, or the happening of contingencies, on which the operation of a must be taken to mean exactly what it says, and courts have no choice but to see to it that
statute is, by its terms, made to depend, but the legislature must prescribe sufficient the mandate is obeyed.54
standards, policies or limitations on their authority.49 While the power to tax cannot be
delegated to executive agencies, details as to the enforcement and administration of an
Thus, it is the ministerial duty of the President to immediately impose the 12% rate upon the
exercise of such power may be left to them, including the power to determine the existence
existence of any of the conditions specified by Congress. This is a duty which cannot be
of facts on which its operation depends.50
evaded by the President. Inasmuch as the law specifically uses the word shall, the exercise of
discretion by the President does not come into play. It is a clear directive to impose the 12%
The rationale for this is that the preliminary ascertainment of facts as basis for the VAT rate when the specified conditions are present. The time of taking into effect of the 12%
enactment of legislation is not of itself a legislative function, but is simply ancillary to VAT rate is based on the happening of a certain specified contingency, or upon the
legislation. Thus, the duty of correlating information and making recommendations is the ascertainment of certain facts or conditions by a person or body other than the legislature
kind of subsidiary activity which the legislature may perform through its members, or which itself.
it may delegate to others to perform. Intelligent legislation on the complicated problems of
modern society is impossible in the absence of accurate information on the part of the
The Court finds no merit to the contention of petitioners ABAKADA GURO Party List, et al.
legislators, and any reasonable method of securing such information is proper. 51 The
that the law effectively nullified the President’s power of control over the Secretary of
Constitution as a continuously operative charter of government does not require that
Finance by mandating the fixing of the tax rate by the President upon the recommendation
Congress find for itself
of the Secretary of Finance. The Court cannot also subscribe to the position of petitioners

every fact upon which it desires to base legislative action or that it make for itself detailed
Pimentel, et al. that the word shall should be interpreted to mean may in view of the phrase
determinations which it has declared to be prerequisite to application of legislative policy to
"upon the recommendation of the Secretary of Finance." Neither does the Court find
particular facts and circumstances impossible for Congress itself properly to investigate. 52
persuasive the submission of petitioners Escudero, et al. that any recommendation by the
Secretary of Finance can easily be brushed aside by the President since the former is a mere
In the present case, the challenged section of R.A. No. 9337 is the common proviso in alter ego of the latter.
Sections 4, 5 and 6 which reads as follows:
When one speaks of the Secretary of Finance as the alter ego of the President, it simply
That the President, upon the recommendation of the Secretary of Finance, shall, effective means that as head of the Department of Finance he is the assistant and agent of the Chief
January 1, 2006, raise the rate of value-added tax to twelve percent (12%), after any of the Executive. The multifarious executive and administrative functions of the Chief Executive are
following conditions has been satisfied: performed by and through the executive departments, and the acts of the secretaries of such
158

departments, such as the Department of Finance, performed and promulgated in the regular allegations which are manifestly conjectural, as these may not exist at all. The Court deals
course of business, are, unless disapproved or reprobated by the Chief Executive, with facts, not fancies; on realities, not appearances. When the Court acts on appearances
presumptively the acts of the Chief Executive. The Secretary of Finance, as such, occupies a instead of realities, justice and law will be short-lived.
political position and holds office in an advisory capacity, and, in the language of Thomas
Jefferson, "should be of the President's bosom confidence" and, in the language of Attorney- B. The 12% Increase VAT Rate Does Not Impose an Unfair and Unnecessary Additional Tax
General Cushing, is "subject to the direction of the President."55 Burden

In the present case, in making his recommendation to the President on the existence of Petitioners Pimentel, et al. argue that the 12% increase in the VAT rate imposes an unfair and
either of the two conditions, the Secretary of Finance is not acting as the alter ego of the additional tax burden on the people. Petitioners also argue that the 12% increase, dependent
President or even her subordinate. In such instance, he is not subject to the power of control on any of the 2 conditions set forth in the contested provisions, is ambiguous because it does
and direction of the President. He is acting as the agent of the legislative department, to not state if the VAT rate would be returned to the original 10% if the rates are no longer
determine and declare the event upon which its expressed will is to take effect.56 The satisfied. Petitioners also argue that such rate is unfair and unreasonable, as the people are
Secretary of Finance becomes the means or tool by which legislative policy is determined and unsure of the applicable VAT rate from year to year.
implemented, considering that he possesses all the facilities to gather data and information
and has a much broader perspective to properly evaluate them. His function is to gather and
Under the common provisos of Sections 4, 5 and 6 of R.A. No. 9337, if any of the two
collate statistical data and other pertinent information and verify if any of the two conditions
conditions set forth therein are satisfied, the President shall increase the VAT rate to 12%.
laid out by Congress is present. His personality in such instance is in reality but a projection of
The provisions of the law are clear. It does not provide for a return to the 10% rate nor does
that of Congress. Thus, being the agent of Congress and not of the President, the President
it empower the President to so revert if, after the rate is increased to 12%, the VAT collection
cannot alter or modify or nullify, or set aside the findings of the Secretary of Finance and to
goes below the 24/5 of the GDP of the previous year or that the national government deficit
substitute the judgment of the former for that of the latter.
as a percentage of GDP of the previous year does not exceed 1½%.

Congress simply granted the Secretary of Finance the authority to ascertain the existence of
Therefore, no statutory construction or interpretation is needed. Neither can conditions or
a fact, namely, whether by December 31, 2005, the value-added tax collection as a
limitations be introduced where none is provided for. Rewriting the law is a forbidden
percentage of Gross Domestic Product (GDP) of the previous year exceeds two and four-fifth
ground that only Congress may tread upon.60
percent (24/5%) or the national government deficit as a percentage of GDP of the previous
year exceeds one and one-half percent (1½%). If either of these two instances has occurred,
the Secretary of Finance, by legislative mandate, must submit such information to the Thus, in the absence of any provision providing for a return to the 10% rate, which in this
President. Then the 12% VAT rate must be imposed by the President effective January 1, case the Court finds none, petitioners’ argument is, at best, purely speculative. There is no
2006. There is no undue delegation of legislative power but only of the discretion as to the basis for petitioners’ fear of a fluctuating VAT rate because the law itself does not provide
execution of a law. This is constitutionally permissible.57 Congress does not abdicate its that the rate should go back to 10% if the conditions provided in Sections 4, 5 and 6 are no
functions or unduly delegate power when it describes what job must be done, who must do longer present. The rule is that where the provision of the law is clear and unambiguous, so
it, and what is the scope of his authority; in our complex economy that is frequently the only that there is no occasion for the court's seeking the legislative intent, the law must be taken
way in which the legislative process can go forward.58 as it is, devoid of judicial addition or subtraction.61

As to the argument of petitioners ABAKADA GURO Party List, et al. that delegating to the Petitioners also contend that the increase in the VAT rate, which was allegedly an incentive
President the legislative power to tax is contrary to the principle of republicanism, the same to the President to raise the VAT collection to at least 2 4/5 of the GDP of the previous year,
deserves scant consideration. Congress did not delegate the power to tax but the mere should be based on fiscal adequacy.
implementation of the law. The intent and will to increase the VAT rate to 12% came from
Congress and the task of the President is to simply execute the legislative policy. That Petitioners obviously overlooked that increase in VAT collection is not the only condition.
Congress chose to do so in such a manner is not within the province of the Court to inquire There is another condition, i.e., the national government deficit as a percentage of GDP of
into, its task being to interpret the law.59 the previous year exceeds one and one-half percent (1 ½%).

The insinuation by petitioners Pimentel, et al. that the President has ample powers to cause, Respondents explained the philosophy behind these alternative conditions:
influence or create the conditions to bring about either or both the conditions precedent
does not deserve any merit as this argument is highly speculative. The Court does not rule on 1. VAT/GDP Ratio > 2.8%
159

The condition set for increasing VAT rate to 12% have economic or fiscal meaning. If The third thing that I’d like to point out is the environment that we are presently operating in
VAT/GDP is less than 2.8%, it means that government has weak or no capability of is not as benign as what it used to be the past five years.
implementing the VAT or that VAT is not effective in the function of the tax collection.
Therefore, there is no value to increase it to 12% because such action will also be ineffectual. What do I mean by that?

2. Nat’l Gov’t Deficit/GDP >1.5% In the past five years, we’ve been lucky because we were operating in a period of basically
global growth and low interest rates. The past few months, we have seen an inching up, in
The condition set for increasing VAT when deficit/GDP is 1.5% or less means the fiscal fact, a rapid increase in the interest rates in the leading economies of the world. And,
condition of government has reached a relatively sound position or is towards the direction therefore, our ability to borrow at reasonable prices is going to be challenged. In fact,
of a balanced budget position. Therefore, there is no need to increase the VAT rate since the ultimately, the question is our ability to access the financial markets.
fiscal house is in a relatively healthy position. Otherwise stated, if the ratio is more than
1.5%, there is indeed a need to increase the VAT rate.62 When the President made her speech in July last year, the environment was not as bad as it
is now, at least based on the forecast of most financial institutions. So, we were assuming
That the first condition amounts to an incentive to the President to increase the VAT that raising 80 billion would put us in a position where we can then convince them to
collection does not render it unconstitutional so long as there is a public purpose for which improve our ability to borrow at lower rates. But conditions have changed on us because the
the law was passed, which in this case, is mainly to raise revenue. In fact, fiscal interest rates have gone up. In fact, just within this room, we tried to access the market for a
adequacy dictated the need for a raise in revenue. billion dollars because for this year alone, the Philippines will have to borrow 4 billion dollars.
Of that amount, we have borrowed 1.5 billion. We issued last January a 25-year bond at 9.7
The principle of fiscal adequacy as a characteristic of a sound tax system was originally stated percent cost. We were trying to access last week and the market was not as favorable and up
by Adam Smith in his Canons of Taxation (1776), as: to now we have not accessed and we might pull back because the conditions are not very
good.
IV. Every tax ought to be so contrived as both to take out and to keep out of the pockets of
the people as little as possible over and above what it brings into the public treasury of the So given this situation, we at the Department of Finance believe that we really need to front-
state.63 end our deficit reduction. Because it is deficit that is causing the increase of the debt and we
are in what we call a debt spiral. The more debt you have, the more deficit you have because
interest and debt service eats and eats more of your revenue. We need to get out of this
It simply means that sources of revenues must be adequate to meet government
debt spiral. And the only way, I think, we can get out of this debt spiral is really have a front-
expenditures and their variations.64
end adjustment in our revenue base.65

The dire need for revenue cannot be ignored. Our country is in a quagmire of financial woe.
The image portrayed is chilling. Congress passed the law hoping for rescue from an inevitable
During the Bicameral Conference Committee hearing, then Finance Secretary Purisima
catastrophe. Whether the law is indeed sufficient to answer the state’s economic dilemma is
bluntly depicted the country’s gloomy state of economic affairs, thus:
not for the Court to judge. In the Fariñas case, the Court refused to consider the various
arguments raised therein that dwelt on the wisdom of Section 14 of R.A. No. 9006 (The Fair
First, let me explain the position that the Philippines finds itself in right now. We are in a Election Act), pronouncing that:
position where 90 percent of our revenue is used for debt service. So, for every peso of
revenue that we currently raise, 90 goes to debt service. That’s interest plus amortization of
. . . policy matters are not the concern of the Court. Government policy is within the exclusive
our debt. So clearly, this is not a sustainable situation. That’s the first fact.
dominion of the political branches of the government. It is not for this Court to look into the
wisdom or propriety of legislative determination. Indeed, whether an enactment is wise or
The second fact is that our debt to GDP level is way out of line compared to other peer unwise, whether it is based on sound economic theory, whether it is the best means to
countries that borrow money from that international financial markets. Our debt to GDP is achieve the desired results, whether, in short, the legislative discretion within its prescribed
approximately equal to our GDP. Again, that shows you that this is not a sustainable limits should be exercised in a particular manner are matters for the judgment of the
situation. legislature, and the serious conflict of opinions does not suffice to bring them within the
range of judicial cognizance.66
160

In the same vein, the Court in this case will not dawdle on the purpose of Congress or the Petitioners claim that the contested sections impose limitations on the amount of input tax
executive policy, given that it is not for the judiciary to "pass upon questions of wisdom, that may be claimed. In effect, a portion of the input tax that has already been paid cannot
justice or expediency of legislation."67 now be credited against the output tax.

II. Petitioners’ argument is not absolute. It assumes that the input tax exceeds 70% of the
output tax, and therefore, the input tax in excess of 70% remains uncredited. However, to
Whether Section 8 of R.A. No. 9337, amending Sections 110(A)(2) and 110(B) of the NIRC; the extent that the input tax is less than 70% of the output tax, then 100% of such input tax is
and Section 12 of R.A. No. 9337, amending Section 114(C) of the NIRC, violate the following still creditable.
provisions of the Constitution:
More importantly, the excess input tax, if any, is retained in a business’s books of accounts
a. Article VI, Section 28(1), and and remains creditable in the succeeding quarter/s. This is explicitly allowed by Section
110(B), which provides that "if the input tax exceeds the output tax, the excess shall be
carried over to the succeeding quarter or quarters." In addition, Section 112(B) allows a VAT-
b. Article III, Section 1
registered person to apply for the issuance of a tax credit certificate or refund for any unused
input taxes, to the extent that such input taxes have not been applied against the output
A. Due Process and Equal Protection Clauses taxes. Such unused input tax may be used in payment of his other internal revenue taxes.

Petitioners Association of Pilipinas Shell Dealers, Inc., et al. argue that Section 8 of R.A. No. The non-application of the unutilized input tax in a given quarter is not ad infinitum, as
9337, amending Sections 110 (A)(2), 110 (B), and Section 12 of R.A. No. 9337, amending petitioners exaggeratedly contend. Their analysis of the effect of the 70% limitation is
Section 114 (C) of the NIRC are arbitrary, oppressive, excessive and confiscatory. Their incomplete and one-sided. It ends at the net effect that there will be unapplied/unutilized
argument is premised on the constitutional right against deprivation of life, liberty of inputs VAT for a given quarter. It does not proceed further to the fact that such
property without due process of law, as embodied in Article III, Section 1 of the Constitution. unapplied/unutilized input tax may be credited in the subsequent periods as allowed by the
carry-over provision of Section 110(B) or that it may later on be refunded through a tax credit
Petitioners also contend that these provisions violate the constitutional guarantee of equal certificate under Section 112(B).
protection of the law.
Therefore, petitioners’ argument must be rejected.
The doctrine is that where the due process and equal protection clauses are invoked,
considering that they are not fixed rules but rather broad standards, there is a need for proof On the other hand, it appears that petitioner Garcia failed to comprehend the operation of
of such persuasive character as would lead to such a conclusion. Absent such a showing, the the 70% limitation on the input tax. According to petitioner, the limitation on the creditable
presumption of validity must prevail.68 input tax in effect allows VAT-registered establishments to retain a portion of the taxes they
collect, which violates the principle that tax collection and revenue should be for public
Section 8 of R.A. No. 9337, amending Section 110(B) of the NIRC imposes a limitation on the purposes and expenditures
amount of input tax that may be credited against the output tax. It states, in part:
"[P]rovided, that the input tax inclusive of the input VAT carried over from the previous As earlier stated, the input tax is the tax paid by a person, passed on to him by the seller,
quarter that may be credited in every quarter shall not exceed seventy percent (70%) of the when he buys goods. Output tax meanwhile is the tax due to the person when he sells goods.
output VAT: …" In computing the VAT payable, three possible scenarios may arise:

Input Tax is defined under Section 110(A) of the NIRC, as amended, as the value-added tax First, if at the end of a taxable quarter the output taxes charged by the seller are equal to the
due from or paid by a VAT-registered person on the importation of goods or local purchase of input taxes that he paid and passed on by the suppliers, then no payment is required;
good and services, including lease or use of property, in the course of trade or business, from
a VAT-registered person, and Output Tax is the value-added tax due on the sale or lease of
Second, when the output taxes exceed the input taxes, the person shall be liable for the
taxable goods or properties or services by any person registered or required to register under
excess, which has to be paid to the Bureau of Internal Revenue (BIR);69 and
the law.
161

Third, if the input taxes exceed the output taxes, the excess shall be carried over to the Provided, That the input tax on goods purchased or imported in a calendar month for use in
succeeding quarter or quarters. Should the input taxes result from zero-rated or effectively trade or business for which deduction for depreciation is allowed under this Code, shall be
zero-rated transactions, any excess over the output taxes shall instead be refunded to the spread evenly over the month of acquisition and the fifty-nine (59) succeeding months if the
taxpayer or credited against other internal revenue taxes, at the taxpayer’s option.70 aggregate acquisition cost for such goods, excluding the VAT component thereof, exceeds
One million pesos (₱1,000,000.00): Provided, however, That if the estimated useful life of the
Section 8 of R.A. No. 9337 however, imposed a 70% limitation on the input tax. Thus, a capital goods is less than five (5) years, as used for depreciation purposes, then the input VAT
person can credit his input tax only up to the extent of 70% of the output tax. In layman’s shall be spread over such a shorter period: Provided, finally, That in the case of purchase of
term, the value-added taxes that a person/taxpayer paid and passed on to him by a seller can services, lease or use of properties, the input tax shall be creditable to the purchaser, lessee
only be credited up to 70% of the value-added taxes that is due to him on a taxable or license upon payment of the compensation, rental, royalty or fee.
transaction. There is no retention of any tax collection because the person/taxpayer has
already previously paid the input tax to a seller, and the seller will subsequently remit such The foregoing section imposes a 60-month period within which to amortize the creditable
input tax to the BIR. The party directly liable for the payment of the tax is the seller.71 What input tax on purchase or importation of capital goods with acquisition cost of ₱1 Million
only needs to be done is for the person/taxpayer to apply or credit these input taxes, as pesos, exclusive of the VAT component. Such spread out only poses a delay in the crediting of
evidenced by receipts, against his output taxes. the input tax. Petitioners’ argument is without basis because the taxpayer is not permanently
deprived of his privilege to credit the input tax.
Petitioners Association of Pilipinas Shell Dealers, Inc., et al. also argue that the input tax
partakes the nature of a property that may not be confiscated, appropriated, or limited It is worth mentioning that Congress admitted that the spread-out of the creditable input tax
without due process of law. in this case amounts to a 4-year interest-free loan to the government.76 In the same breath,
Congress also justified its move by saying that the provision was designed to raise an annual
The input tax is not a property or a property right within the constitutional purview of the revenue of 22.6 billion.77 The legislature also dispelled the fear that the provision will fend off
due process clause. A VAT-registered person’s entitlement to the creditable input tax is a foreign investments, saying that foreign investors have other tax incentives provided by law,
mere statutory privilege. and citing the case of China, where despite a 17.5% non-creditable VAT, foreign investments
were not deterred.78 Again, for whatever is the purpose of the 60-month amortization, this
involves executive economic policy and legislative wisdom in which the Court cannot
The distinction between statutory privileges and vested rights must be borne in mind for
intervene.
persons have no vested rights in statutory privileges. The state may change or take away
rights, which were created by the law of the state, although it may not take away property,
which was vested by virtue of such rights.72 With regard to the 5% creditable withholding tax imposed on payments made by the
government for taxable transactions, Section 12 of R.A. No. 9337, which amended Section
114 of the NIRC, reads:
Under the previous system of single-stage taxation, taxes paid at every level of distribution
are not recoverable from the taxes payable, although it becomes part of the cost, which is
deductible from the gross revenue. When Pres. Aquino issued E.O. No. 273 imposing a 10% SEC. 114. Return and Payment of Value-added Tax. –
multi-stage tax on all sales, it was then that the crediting of the input tax paid on purchase or
importation of goods and services by VAT-registered persons against the output tax was (C) Withholding of Value-added Tax. – The Government or any of its political subdivisions,
introduced.73 This was adopted by the Expanded VAT Law (R.A. No. 7716),74 and The Tax instrumentalities or agencies, including government-owned or controlled corporations
Reform Act of 1997 (R.A. No. 8424).75 The right to credit input tax as against the output tax is (GOCCs) shall, before making payment on account of each purchase of goods and services
clearly a privilege created by law, a privilege that also the law can remove, or in this case, which are subject to the value-added tax imposed in Sections 106 and 108 of this Code,
limit. deduct and withhold a final value-added tax at the rate of five percent (5%) of the gross
payment thereof: Provided, That the payment for lease or use of properties or property
Petitioners also contest as arbitrary, oppressive, excessive and confiscatory, Section 8 of R.A. rights to nonresident owners shall be subject to ten percent (10%) withholding tax at the
No. 9337, amending Section 110(A) of the NIRC, which provides: time of payment. For purposes of this Section, the payor or person in control of the payment
shall be considered as the withholding agent.
SEC. 110. Tax Credits. –
The value-added tax withheld under this Section shall be remitted within ten (10) days
following the end of the month the withholding was made.
(A) Creditable Input Tax. – …
162

Section 114(C) merely provides a method of collection, or as stated by respondents, a more SEC. 114. Return and Payment of Value-added Tax. –
simplified VAT withholding system. The government in this case is constituted as a
withholding agent with respect to their payments for goods and services. (C) Withholding of Creditable Value-added Tax. – The Government or any of its political
subdivisions, instrumentalities or agencies, including government-owned or controlled
Prior to its amendment, Section 114(C) provided for different rates of value-added taxes to corporations (GOCCs) shall, before making payment on account of each purchase of goods
be withheld -- 3% on gross payments for purchases of goods; 6% on gross payments for from sellers and services rendered by contractors which are subject to the value-added tax
services supplied by contractors other than by public works contractors; 8.5% on gross imposed in Sections 106 and 108 of this Code, deduct and withhold the value-added tax due
payments for services supplied by public work contractors; or 10% on payment for the lease at the rate of three percent (3%) of the gross payment for the purchase of goods and six
or use of properties or property rights to nonresident owners. Under the present Section percent (6%) on gross receipts for services rendered by contractors on every sale or
114(C), these different rates, except for the 10% on lease or property rights payment to installment payment which shall be creditable against the value-added tax liability of the
nonresidents, were deleted, and a uniform rate of 5% is applied. seller or contractor: Provided, however, That in the case of government public works
contractors, the withholding rate shall be eight and one-half percent (8.5%): Provided,
The Court observes, however, that the law the used the word final. In tax usage, final, as further, That the payment for lease or use of properties or property rights to nonresident
opposed to creditable, means full. Thus, it is provided in Section 114(C): "final value-added owners shall be subject to ten percent (10%) withholding tax at the time of payment. For this
tax at the rate of five percent (5%)." purpose, the payor or person in control of the payment shall be considered as the
withholding agent.
In Revenue Regulations No. 02-98, implementing R.A. No. 8424 (The Tax Reform Act of 1997),
the concept of final withholding tax on income was explained, to wit: The valued-added tax withheld under this Section shall be remitted within ten (10) days
following the end of the month the withholding was made. (Emphasis supplied)
SECTION 2.57. Withholding of Tax at Source
As amended, the use of the word final and the deletion of the word creditable exhibits
Congress’s intention to treat transactions with the government differently. Since it has not
(A) Final Withholding Tax. – Under the final withholding tax system the amount of income tax
been shown that the class subject to the 5% final withholding tax has been unreasonably
withheld by the withholding agent is constituted as full and final payment of the income tax
narrowed, there is no reason to invalidate the provision. Petitioners, as petroleum dealers,
due from the payee on the said income. The liability for payment of the tax rests primarily on
are not the only ones subjected to the 5% final withholding tax. It applies to all those who
the payor as a withholding agent. Thus, in case of his failure to withhold the tax or in case of
deal with the government.
underwithholding, the deficiency tax shall be collected from the payor/withholding agent. …

Moreover, the actual input tax is not totally lost or uncreditable, as petitioners believe.
(B) Creditable Withholding Tax. – Under the creditable withholding tax system, taxes
Revenue Regulations No. 14-2005 or the Consolidated Value-Added Tax Regulations 2005
withheld on certain income payments are intended to equal or at least approximate the tax
issued by the BIR, provides that should the actual input tax exceed 5% of gross payments, the
due of the payee on said income. … Taxes withheld on income payments covered by the
excess may form part of the cost. Equally, should the actual input tax be less than 5%, the
expanded withholding tax (referred to in Sec. 2.57.2 of these regulations) and compensation
difference is treated as income.81
income (referred to in Sec. 2.78 also of these regulations) are creditable in nature.

Petitioners also argue that by imposing a limitation on the creditable input tax, the
As applied to value-added tax, this means that taxable transactions with the government are
government gets to tax a profit or value-added even if there is no profit or value-added.
subject to a 5% rate, which constitutes as full payment of the tax payable on the transaction.
This represents the net VAT payable of the seller. The other 5% effectively accounts for the
standard input VAT (deemed input VAT), in lieu of the actual input VAT directly or Petitioners’ stance is purely hypothetical, argumentative, and again, one-sided. The Court
attributable to the taxable transaction.79 will not engage in a legal joust where premises are what ifs, arguments, theoretical and facts,
uncertain. Any disquisition by the Court on this point will only be, as Shakespeare describes
life in Macbeth,82 "full of sound and fury, signifying nothing."
The Court need not explore the rationale behind the provision. It is clear that Congress
intended to treat differently taxable transactions with the government.80 This is supported by
the fact that under the old provision, the 5% tax withheld by the government remains What’s more, petitioners’ contention assumes the proposition that there is no profit or
creditable against the tax liability of the seller or contractor, to wit: value-added. It need not take an astute businessman to know that it is a matter of exception
163

that a business will sell goods or services without profit or value-added. It cannot be The rule of taxation shall be uniform and equitable. The Congress shall evolve a progressive
overstressed that a business is created precisely for profit. system of taxation.

The equal protection clause under the Constitution means that "no person or class of Uniformity in taxation means that all taxable articles or kinds of property of the same class
persons shall be deprived of the same protection of laws which is enjoyed by other persons shall be taxed at the same rate. Different articles may be taxed at different amounts
or other classes in the same place and in like circumstances."83 provided that the rate is uniform on the same class everywhere with all people at all times.86

The power of the State to make reasonable and natural classifications for the purposes of In this case, the tax law is uniform as it provides a standard rate of 0% or 10% (or 12%) on all
taxation has long been established. Whether it relates to the subject of taxation, the kind of goods and services. Sections 4, 5 and 6 of R.A. No. 9337, amending Sections 106, 107 and
property, the rates to be levied, or the amounts to be raised, the methods of assessment, 108, respectively, of the NIRC, provide for a rate of 10% (or 12%) on sale of goods and
valuation and collection, the State’s power is entitled to presumption of validity. As a rule, properties, importation of goods, and sale of services and use or lease of properties. These
the judiciary will not interfere with such power absent a clear showing of unreasonableness, same sections also provide for a 0% rate on certain sales and transaction.
discrimination, or arbitrariness.84
Neither does the law make any distinction as to the type of industry or trade that will bear
Petitioners point out that the limitation on the creditable input tax if the entity has a high the 70% limitation on the creditable input tax, 5-year amortization of input tax paid on
ratio of input tax, or invests in capital equipment, or has several transactions with the purchase of capital goods or the 5% final withholding tax by the government. It must be
government, is not based on real and substantial differences to meet a valid classification. stressed that the rule of uniform taxation does not deprive Congress of the power to classify
subjects of taxation, and only demands uniformity within the particular class.87
The argument is pedantic, if not outright baseless. The law does not make any classification
in the subject of taxation, the kind of property, the rates to be levied or the amounts to be R.A. No. 9337 is also equitable. The law is equipped with a threshold margin. The VAT rate of
raised, the methods of assessment, valuation and collection. Petitioners’ alleged distinctions 0% or 10% (or 12%) does not apply to sales of goods or services with gross annual sales or
are based on variables that bear different consequences. While the implementation of the receipts not exceeding ₱1,500,000.00.88Also, basic marine and agricultural food products in
law may yield varying end results depending on one’s profit margin and value-added, the their original state are still not subject to the tax,89 thus ensuring that prices at the grassroots
Court cannot go beyond what the legislature has laid down and interfere with the affairs of level will remain accessible. As was stated in Kapatiran ng mga Naglilingkod sa Pamahalaan
business. ng Pilipinas, Inc. vs. Tan:90

The equal protection clause does not require the universal application of the laws on all The disputed sales tax is also equitable. It is imposed only on sales of goods or services by
persons or things without distinction. This might in fact sometimes result in unequal persons engaged in business with an aggregate gross annual sales exceeding ₱200,000.00.
protection. What the clause requires is equality among equals as determined according to a Small corner sari-sari stores are consequently exempt from its application. Likewise exempt
valid classification. By classification is meant the grouping of persons or things similar to each from the tax are sales of farm and marine products, so that the costs of basic food and other
other in certain particulars and different from all others in these same particulars.85 necessities, spared as they are from the incidence of the VAT, are expected to be relatively
lower and within the reach of the general public.
Petitioners brought to the Court’s attention the introduction of Senate Bill No. 2038 by Sens.
S.R. Osmeña III and Ma. Ana Consuelo A.S. – Madrigal on June 6, 2005, and House Bill No. It is admitted that R.A. No. 9337 puts a premium on businesses with low profit margins, and
4493 by Rep. Eric D. Singson. The proposed legislation seeks to amend the 70% limitation by unduly favors those with high profit margins. Congress was not oblivious to this. Thus, to
increasing the same to 90%. This, according to petitioners, supports their stance that the 70% equalize the weighty burden the law entails, the law, under Section 116, imposed a 3%
limitation is arbitrary and confiscatory. On this score, suffice it to say that these are still percentage tax on VAT-exempt persons under Section 109(v), i.e., transactions with gross
proposed legislations. Until Congress amends the law, and absent any unequivocal basis for annual sales and/or receipts not exceeding ₱1.5 Million. This acts as a equalizer because in
its unconstitutionality, the 70% limitation stays. effect, bigger businesses that qualify for VAT coverage and VAT-exempt taxpayers stand on
equal-footing.
B. Uniformity and Equitability of Taxation
Moreover, Congress provided mitigating measures to cushion the impact of the imposition of
Article VI, Section 28(1) of the Constitution reads: the tax on those previously exempt. Excise taxes on petroleum products91 and natural
164

gas92 were reduced. Percentage tax on domestic carriers was removed.93 Power producers Nevertheless, the Constitution does not really prohibit the imposition of indirect taxes, like
are now exempt from paying franchise tax.94 the VAT. What it simply provides is that Congress shall "evolve a progressive system of
taxation." The Court stated in the Tolentino case, thus:
Aside from these, Congress also increased the income tax rates of corporations, in order to
distribute the burden of taxation. Domestic, foreign, and non-resident corporations are now The Constitution does not really prohibit the imposition of indirect taxes which, like the VAT,
subject to a 35% income tax rate, from a previous 32%.95 Intercorporate dividends of non- are regressive. What it simply provides is that Congress shall ‘evolve a progressive system of
resident foreign corporations are still subject to 15% final withholding tax but the tax credit taxation.’ The constitutional provision has been interpreted to mean simply that ‘direct taxes
allowed on the corporation’s domicile was increased to 20%.96 The Philippine Amusement are . . . to be preferred [and] as much as possible, indirect taxes should be minimized.’ (E.
and Gaming Corporation (PAGCOR) is not exempt from income taxes anymore.97 Even the FERNANDO, THE CONSTITUTION OF THE PHILIPPINES 221 (Second ed. 1977)) Indeed, the
sale by an artist of his works or services performed for the production of such works was not mandate to Congress is not to prescribe, but to evolve, a progressive tax system. Otherwise,
spared. sales taxes, which perhaps are the oldest form of indirect taxes, would have been prohibited
with the proclamation of Art. VIII, §17 (1) of the 1973 Constitution from which the present
All these were designed to ease, as well as spread out, the burden of taxation, which would Art. VI, §28 (1) was taken. Sales taxes are also regressive.
otherwise rest largely on the consumers. It cannot therefore be gainsaid that R.A. No. 9337 is
equitable. Resort to indirect taxes should be minimized but not avoided entirely because it is difficult, if
not impossible, to avoid them by imposing such taxes according to the taxpayers' ability to
C. Progressivity of Taxation pay. In the case of the VAT, the law minimizes the regressive effects of this imposition by
providing for zero rating of certain transactions (R.A. No. 7716, §3, amending §102 (b) of the
NIRC), while granting exemptions to other transactions. (R.A. No. 7716, §4 amending §103 of
Lastly, petitioners contend that the limitation on the creditable input tax is anything but
the NIRC)99
regressive. It is the smaller business with higher input tax-output tax ratio that will suffer the
consequences.
CONCLUSION
Progressive taxation is built on the principle of the taxpayer’s ability to pay. This principle was
also lifted from Adam Smith’s Canons of Taxation, and it states: It has been said that taxes are the lifeblood of the government. In this case, it is just an
enema, a first-aid measure to resuscitate an economy in distress. The Court is neither blind
nor is it turning a deaf ear on the plight of the masses. But it does not have the panacea for
I. The subjects of every state ought to contribute towards the support of the government, as
the malady that the law seeks to remedy. As in other cases, the Court cannot strike down a
nearly as possible, in proportion to their respective abilities; that is, in proportion to the
law as unconstitutional simply because of its yokes.
revenue which they respectively enjoy under the protection of the state.

Let us not be overly influenced by the plea that for every wrong there is a remedy, and that
Taxation is progressive when its rate goes up depending on the resources of the person
the judiciary should stand ready to afford relief. There are undoubtedly many wrongs the
affected.98
judicature may not correct, for instance, those involving political questions. . . .

The VAT is an antithesis of progressive taxation. By its very nature, it is regressive. The
Let us likewise disabuse our minds from the notion that the judiciary is the repository of
principle of progressive taxation has no relation with the VAT system inasmuch as the VAT
remedies for all political or social ills; We should not forget that the Constitution has
paid by the consumer or business for every goods bought or services enjoyed is the same
judiciously allocated the powers of government to three distinct and separate
regardless of income. In
compartments; and that judicial interpretation has tended to the preservation of the
independence of the three, and a zealous regard of the prerogatives of each, knowing full
other words, the VAT paid eats the same portion of an income, whether big or small. The well that one is not the guardian of the others and that, for official wrong-doing, each may be
disparity lies in the income earned by a person or profit margin marked by a business, such brought to account, either by impeachment, trial or by the ballot box.100
that the higher the income or profit margin, the smaller the portion of the income or profit
that is eaten by VAT. A converso, the lower the income or profit margin, the bigger the part
The words of the Court in Vera vs. Avelino101 holds true then, as it still holds true now. All
that the VAT eats away. At the end of the day, it is really the lower income group or
things considered, there is no raison d'être for the unconstitutionality of R.A. No. 9337.
businesses with low-profit margins that is always hardest hit.
165

WHEREFORE, Republic Act No. 9337 not being unconstitutional, the petitions in G.R. Nos. Issues:
168056, 168207, 168461, 168463, and 168730, are hereby DISMISSED.
Whether or not R.A. No. 9337 has violated the provisions in Article VI, Section 24, and Article
There being no constitutional impediment to the full enforcement and implementation of VI, Section 26 (2) of the Constitution.
R.A. No. 9337, the temporary restraining order issued by the Court on July 1, 2005
is LIFTED upon finality of herein decision. Whether or not there was an undue delegation of legislative power in violation of Article VI
Sec 28 Par 1 and 2 of the Constitution.
SO ORDERED.
Whether or not there was a violation of the due process and equal protection under Article
Facts: III Sec. 1 of the Constitution.

Petitioners ABAKADA GURO Party List challenged the constitutionality of R.A. No. 9337 Discussions:
particularly Sections 4, 5 and 6, amending Sections 106, 107 and 108, respectively, of the
Basing from the ruling of Tolentino case, it is not the law, but the revenue bill which is
National Internal Revenue Code (NIRC). These questioned provisions contain a uniform
required by the Constitution to “originate exclusively” in the House of Representatives, but
proviso authorizing the President, upon recommendation of the Secretary of Finance, to raise
Senate has the power not only to propose amendments, but also to propose its own version
the VAT rate to 12%, effective January 1, 2006, after any of the following conditions have
even with respect to bills which are required by the Constitution to originate in the House.
been satisfied, to wit:
the Constitution simply means is that the initiative for filing revenue, tariff or tax bills, bills
. . . That the President, upon the recommendation of the Secretary of Finance, shall, effective authorizing an increase of the public debt, private bills and bills of local application must
January 1, 2006, raise the rate of value-added tax to twelve percent (12%), after any of the come from the House of Representatives on the theory that, elected as they are from the
following conditions has been satisfied: districts, the members of the House can be expected to be more sensitive to the local needs
and problems. On the other hand, the senators, who are elected at large, are expected to
(i) Value-added tax collection as a percentage of Gross Domestic Product (GDP) of the approach the same problems from the national perspective. Both views are thereby made to
previous year exceeds two and four-fifth percent (2 4/5%); or bear on the enactment of such laws.

(ii) National government deficit as a percentage of GDP of the previous year exceeds one and In testing whether a statute constitutes an undue delegation of legislative power or not, it is
one-half percent (1 ½%). usual to inquire whether the statute was complete in all its terms and provisions when it left
the hands of the legislature so that nothing was left to the judgment of any other appointee
Petitioners argue that the law is unconstitutional, as it constitutes abandonment by Congress or delegate of the legislature.
of its exclusive authority to fix the rate of taxes under Article VI, Section 28(2) of the 1987
Philippine Constitution. They further argue that VAT is a tax levied on the sale or exchange of The equal protection clause under the Constitution means that “no person or class of
goods and services and cannot be included within the purview of tariffs under the exemption persons shall be deprived of the same protection of laws which is enjoyed by other persons
delegation since this refers to customs duties, tolls or tribute payable upon merchandise to or other classes in the same place and in like circumstances.”
the government and usually imposed on imported/exported goods. They also said that the
President has powers to cause, influence or create the conditions provided by law to bring Rulings:
about the conditions precedent. Moreover, they allege that no guiding standards are made
R.A. No. 9337 has not violated the provisions. The revenue bill exclusively originated in the
by law as to how the Secretary of Finance will make the recommendation. They claim,
House of Representatives, the Senate was acting within its constitutional power to introduce
nonetheless, that any recommendation of the Secretary of Finance can easily be brushed
amendments to the House bill when it included provisions in Senate Bill No. 1950 amending
aside by the President since the former is a mere alter ego of the latter, such that, ultimately,
corporate income taxes, percentage, excise and franchise taxes. Verily, Article VI, Section 24
it is the President who decides whether to impose the increased tax rate or not.
of the Constitution does not contain any prohibition or limitation on the extent of the
amendments that may be introduced by the Senate to the House revenue bill.
166

There is no undue delegation of legislative power but only of the discretion as to the Issue: Whether or not the R.A. No. 9337 or the Vat Reform Act is constitutional?
execution of a law. This is constitutionally permissible. Congress does not abdicate its
functions or unduly delegate power when it describes what job must be done, who must do Held:
it, and what is the scope of his authority; in our complex economy that is frequently the only
The Court is not persuaded. Article VI, Section 24 of the Constitution provides that All
way in which the legislative process can go forward.
appropriation, revenue or tariff bills, bills authorizing increase of the public debt, bills of local
Supreme Court held no decision on this matter. The power of the State to make reasonable application, and private bills shall originate exclusively in the House of Representatives, but
and natural classifications for the purposes of taxation has long been established. Whether it the Senate may propose or concur with amendments.
relates to the subject of taxation, the kind of property, the rates to be levied, or the amounts
The Court reiterates that in making his recommendation to the President on the existence of
to be raised, the methods of assessment, valuation and collection, the State’s power is
either of the two conditions, the Secretary of Finance is not acting as the alter ego of the
entitled to presumption of validity. As a rule, the judiciary will not interfere with such power
President or even her subordinate. He is acting as the agent of the legislative department, to
absent a clear showing of unreasonableness, discrimination, or arbitrariness.
determine and declare the event upon which its expressed will is to take effect. The
Facts: Secretary of Finance becomes the means or tool by which legislative policy is determined and
implemented, considering that he possesses all the facilities to gather data and information
Motions for Reconsideration filed by petitioners, ABAKADA Guro party List Officer and et al., and has a much broader perspective to properly evaluate them. His function is to gather and
insist that the bicameral conference committee should not even have acted on the no pass- collate statistical data and other pertinent information and verify if any of the two conditions
on provisions since there is no disagreement between House Bill Nos. 3705 and 3555 on the laid out by Congress is present.
one hand, and Senate Bill No. 1950 on the other, with regard to the no pass-on provision for
the sale of service for power generation because both the Senate and the House were in In the same breath, the Court reiterates its finding that it is not a property or a property
agreement that the VAT burden for the sale of such service shall not be passed on to the right, and a VAT-registered person’s entitlement to the creditable input tax is a mere
end-consumer. As to the no pass-on provision for sale of petroleum products, petitioners statutory privilege. As the Court stated in its Decision, the right to credit the input tax is a
argue that the fact that the presence of such a no pass-on provision in the House version and mere creation of law. More importantly, the assailed provisions of R.A. No. 9337 already
the absence thereof in the Senate Bill means there is no conflict because “a House provision involve legislative policy and wisdom. So long as there is a public end for which R.A. No. 9337
cannot be in conflict with something that does not exist.” was passed, the means through which such end shall be accomplished is for the legislature to
choose so long as it is within constitutional bounds.
Escudero, et. al., also contend that Republic Act No. 9337 grossly violates the constitutional
imperative on exclusive origination of revenue bills under Section 24 of Article VI of the The Motions for Reconsideration are hereby DENIED WITH FINALITY. The temporary
Constitution when the Senate introduced amendments not connected with VAT. restraining order issued by the Court is LIFTED.

Petitioners Escudero, et al., also reiterate that R.A. No. 9337’s stand- by authority to the FACTS:
Executive to increase the VAT rate, especially on account of the recommendatory power
RA 9337, an act amending certain sections of the National Internal Revenue Code of 1997, is
granted to the Secretary of Finance, constitutes undue delegation of legislative power. They
questioned by petitioners for being unconstitutional. Procedural issues raised by petitioners
submit that the recommendatory power given to the Secretary of Finance in regard to the
are the legality of the bicameral proceedings, exclusive origination of revenue measures and
occurrence of either of two events using the Gross Domestic Product (GDP) as a benchmark
the power of the Senate concomitant thereto. Also, an issue was raised with regard to the
necessarily and inherently required extended analysis and evaluation, as well as policy
undue delegation of legislative power to the President to increase the rate of value-added
making.
tax to 12%.
Petitioners also reiterate their argument that the input tax is a property or a property right.
Petitioners also argue that the increase to 12%, as well as the 70% limitation on the
Petitioners also contend that even if the right to credit the input VAT is merely a statutory
creditable input tax, the 60- month amortization on the purchase or importation of capital
privilege, it has already evolved into a vested right that the State cannot remove.
goods exceeding P1,000,000.00, and the 5% final withholding tax by government agencies, is
167

arbitrary, oppressive, and confiscatory, and that it violates the constitutional principle on
progressive taxation, among others.

ISSUE: Whether RA 9337 is constitutional

RULING:

Yes. Mounting budget deficit, revenue generation, inadequate fiscal allocation for education,
increased emoluments for health workers, and wider coverage for full value-added tax
benefits ... these are the reasons why Republic Act No. 9337 (R.A. No. 9337) was enacted.
Reasons, the wisdom of which, the Court even with its extensive constitutional power of
review, cannot probe.

It has been said that taxes are the lifeblood of the government. In this case, it is just an
enema, a first-aid measure to resuscitate an economy in distress. The Court is neither blind
nor is it turning a deaf ear on the plight of the masses. But it does not have the panacea for
the malady that the law seeks to remedy. As in other cases, the Court cannot strike down a
law as unconstitutional simply because of its yokes.
168

EN BANC The second case, G.R. No. 193036, is a special civil action for certiorari and prohibition filed
by petitioners Edcel C. Lagman, Rodolfo B. Albano Jr., Simeon A. Datumanong, and Orlando B.
LOUIS BAROK C. BIRAOGO, Fua, Sr. (petitioners-legislators) as incumbent members of the House of Representatives.

Petitioner, The genesis of the foregoing cases can be traced to the events prior to the historic May 2010
elections, when then Senator Benigno Simeon Aquino III declared his staunch condemnation
- versus -
of graft and corruption with his slogan, Kung walang corrupt, walang mahirap. The Filipino
people, convinced of his sincerity and of his ability to carry out this noble objective,
THE PHILIPPINE TRUTH COMMISSION OF 2010,
catapulted the good senator to the presidency.
Respondent.
To transform his campaign slogan into reality, President Aquino found a need for a special
DECISION body to investigate reported cases of graft and corruption allegedly committed during the
previous administration.
MENDOZA, J.:
Thus, at the dawn of his administration, the President on July 30, 2010, signed Executive
When the judiciary mediates to allocate constitutional boundaries, it does not assert any Order No. 1 establishing the Philippine Truth Commission of 2010 (Truth Commission).
superiority over the other departments; it does not in reality nullify or invalidate an act of the Pertinent provisions of said executive order read:
legislature, but only asserts the solemn and sacred obligation assigned to it by the
Constitution to determine conflicting claims of authority under the Constitution and to EXECUTIVE ORDER NO. 1
establish for the parties in an actual controversy the rights which that instrument secures
CREATING THE PHILIPPINE TRUTH COMMISSION OF 2010
and guarantees to them.
WHEREAS, Article XI, Section 1 of the 1987 Constitution of the Philippines solemnly enshrines
--- Justice Jose P. Laurel[1]
the principle that a public office is a public trust and mandates that public officers and
The role of the Constitution cannot be overlooked. It is through the Constitution that the employees, who are servants of the people, must at all times be accountable to the latter,
fundamental powers of government are established, limited and defined, and by which these serve them with utmost responsibility, integrity, loyalty and efficiency, act with patriotism
powers are distributed among the several departments.[2] The Constitution is the basic and and justice, and lead modest lives;
paramount law to which all other laws must conform and to which all persons, including the
WHEREAS, corruption is among the most despicable acts of defiance of this principle and
highest officials of the land, must defer.[3] Constitutional doctrines must remain steadfast no
notorious violation of this mandate;
matter what may be the tides of time. It cannot be simply made to sway and accommodate
the call of situations and much more tailor itself to the whims and caprices of government
WHEREAS, corruption is an evil and scourge which seriously affects the political, economic,
and the people who run it.[4]
and social life of a nation; in a very special way it inflicts untold misfortune and misery on the
poor, the marginalized and underprivileged sector of society;
For consideration before the Court are two consolidated cases[5] both of which essentially
assail the validity and constitutionality of Executive Order No. 1, dated July 30, 2010, entitled
WHEREAS, corruption in the Philippines has reached very alarming levels, and undermined
Creating the Philippine Truth Commission of 2010.
the peoples trust and confidence in the Government and its institutions;
The first case is G.R. No. 192935, a special civil action for prohibition instituted by petitioner
WHEREAS, there is an urgent call for the determination of the truth regarding certain reports
Louis Biraogo (Biraogo) in his capacity as a citizen and taxpayer. Biraogo assails Executive
of large scale graft and corruption in the government and to put a closure to them by the
Order No. 1 for being violative of the legislative power of Congress under Section 1, Article VI
filing of the appropriate cases against those involved, if warranted, and to deter others from
of the Constitution[6] as it usurps the constitutional authority of the legislature to create a
public office and to appropriate funds therefor.[7]
169

committing the evil, restore the peoples faith and confidence in the Government and in their a) Identify and determine the reported cases of such graft and corruption which it will
public servants; investigate;

WHEREAS, the Presidents battlecry during his campaign for the Presidency in the last b) Collect, receive, review and evaluate evidence related to or regarding the cases of large
elections kung walang corrupt, walang mahirap expresses a solemn pledge that if elected, he scale corruption which it has chosen to investigate, and to this end require any agency,
would end corruption and the evil it breeds; official or employee of the Executive Branch, including government-owned or controlled
corporations, to produce documents, books, records and other papers;
WHEREAS, there is a need for a separate body dedicated solely to investigating and finding
out the truth concerning the reported cases of graft and corruption during the previous c) Upon proper request or representation, obtain information and documents from the
administration, and which will recommend the prosecution of the offenders and secure Senate and the House of Representatives records of investigations conducted by committees
justice for all; thereof relating to matters or subjects being investigated by the Commission;

WHEREAS, Book III, Chapter 10, Section 31 of Executive Order No. 292, otherwise known as d) Upon proper request and representation, obtain information from the courts, including
the Revised Administrative Code of the Philippines, gives the President the continuing the Sandiganbayan and the Office of the Court Administrator, information or documents in
authority to reorganize the Office of the President. respect to corruption cases filed with the Sandiganbayan or the regular courts, as the case
may be;
NOW, THEREFORE, I, BENIGNO SIMEON AQUINO III, President of the Republic of the
Philippines, by virtue of the powers vested in me by law, do hereby order: e) Invite or subpoena witnesses and take their testimonies and for that purpose,
administer oaths or affirmations as the case may be;
SECTION 1. Creation of a Commission. There is hereby created the PHILIPPINE TRUTH
COMMISSION, hereinafter referred to as the COMMISSION, which shall primarily seek and f) Recommend, in cases where there is a need to utilize any person as a state witness to
find the truth on, and toward this end, investigate reports of graft and corruption of such ensure that the ends of justice be fully served, that such person who qualifies as a state
scale and magnitude that shock and offend the moral and ethical sensibilities of the people, witness under the Revised Rules of Court of the Philippines be admitted for that purpose;
committed by public officers and employees, their co-principals, accomplices and accessories
from the private sector, if any, during the previous administration; and thereafter g) Turn over from time to time, for expeditious prosecution, to the appropriate
recommend the appropriate action or measure to be taken thereon to ensure that the full prosecutorial authorities, by means of a special or interim report and recommendation, all
measure of justice shall be served without fear or favor. evidence on corruption of public officers and employees and their private sector co-
principals, accomplices or accessories, if any, when in the course of its investigation the
The Commission shall be composed of a Chairman and four (4) members who will act as an Commission finds that there is reasonable ground to believe that they are liable for graft and
independent collegial body. corruption under pertinent applicable laws;

SECTION 2. Powers and Functions. The Commission, which shall have all the powers of an h) Call upon any government investigative or prosecutorial agency such as the Department
investigative body under Section 37, Chapter 9, Book I of the Administrative Code of 1987, is of Justice or any of the agencies under it, and the Presidential Anti-Graft Commission, for
primarily tasked to conduct a thorough fact-finding investigation of reported cases of graft such assistance and cooperation as it may require in the discharge of its functions and duties;
and corruption referred to in Section 1, involving third level public officers and higher, their
co-principals, accomplices and accessories from the private sector, if any, during the previous i) Engage or contract the services of resource persons, professionals and other personnel
administration and thereafter submit its finding and recommendations to the President, determined by it as necessary to carry out its mandate;
Congress and the Ombudsman.
j) Promulgate its rules and regulations or rules of procedure it deems necessary to
In particular, it shall: effectively and efficiently carry out the objectives of this Executive Order and to ensure the
orderly conduct of its investigations, proceedings and hearings, including the presentation of
evidence;
170

k) Exercise such other acts incident to or are appropriate and necessary in connection with prior administrations, such mandate may be so extended accordingly by way of a
the objectives and purposes of this Order. supplemental Executive Order.

SECTION 3. Staffing Requirements. x x x. SECTION 18. Separability Clause. If any provision of this Order is declared unconstitutional,
the same shall not affect the validity and effectivity of the other provisions hereof.
SECTION 4. Detail of Employees. x x x.
SECTION 19. Effectivity. This Executive Order shall take effect immediately.
SECTION 5. Engagement of Experts. x x x
DONE in the City of Manila, Philippines, this 30th day of July 2010.
SECTION 6. Conduct of Proceedings. x x x.
(SGD.) BENIGNO S. AQUINO III
SECTION 7. Right to Counsel of Witnesses/Resource Persons. x x x.
By the President:
SECTION 8. Protection of Witnesses/Resource Persons. x x x.
(SGD.) PAQUITO N. OCHOA, JR.
SECTION 9. Refusal to Obey Subpoena, Take Oath or Give Testimony. Any government official
or personnel who, without lawful excuse, fails to appear upon subpoena issued by the Executive Secretary
Commission or who, appearing before the Commission refuses to take oath or affirmation,
give testimony or produce documents for inspection, when required, shall be subject to Nature of the Truth Commission
administrative disciplinary action. Any private person who does the same may be dealt with
As can be gleaned from the above-quoted provisions, the Philippine Truth Commission (PTC)
in accordance with law.
is a mere ad hoc body formed under the Office of the President with the primary task to
SECTION 10. Duty to Extend Assistance to the Commission. x x x. investigate reports of graft and corruption committed by third-level public officers and
employees, their co-principals, accomplices and accessories during the previous
SECTION 11. Budget for the Commission. The Office of the President shall provide the administration, and thereafter to submit its finding and recommendations to the President,
necessary funds for the Commission to ensure that it can exercise its powers, execute its Congress and the Ombudsman. Though it has been described as an independent collegial
functions, and perform its duties and responsibilities as effectively, efficiently, and body, it is essentially an entity within the Office of the President Proper and subject to his
expeditiously as possible. control. Doubtless, it constitutes a public office, as an ad hoc body is one.[8]

SECTION 12. Office. x x x. To accomplish its task, the PTC shall have all the powers of an investigative body under
Section 37, Chapter 9, Book I of the Administrative Code of 1987. It is not, however, a quasi-
SECTION 13. Furniture/Equipment. x x x. judicial body as it cannot adjudicate, arbitrate, resolve, settle, or render awards in disputes
between contending parties. All it can do is gather, collect and assess evidence of graft and
SECTION 14. Term of the Commission. The Commission shall accomplish its mission on or
corruption and make recommendations. It may have subpoena powers but it has no power
before December 31, 2012.
to cite people in contempt, much less order their arrest. Although it is a fact-finding body, it
cannot determine from such facts if probable cause exists as to warrant the filing of an
SECTION 15. Publication of Final Report. x x x.
information in our courts of law. Needless to state, it cannot impose criminal, civil or
SECTION 16. Transfer of Records and Facilities of the Commission. x x x. administrative penalties or sanctions.

SECTION 17. Special Provision Concerning Mandate. If and when in the judgment of the The PTC is different from the truth commissions in other countries which have been created
President there is a need to expand the mandate of the Commission as defined in Section 1 as official, transitory and non-judicial fact-finding bodies to establish the facts and context of
hereof to include the investigation of cases and instances of graft and corruption during the serious violations of human rights or of international humanitarian law in a countrys past.[9]
171

They are usually established by states emerging from periods of internal unrest, civil strife or (b) The provision of Book III, Chapter 10, Section 31 of the Administrative Code of 1987
authoritarianism to serve as mechanisms for transitional justice. cannot legitimize E.O. No. 1 because the delegated authority of the President to structurally
reorganize the Office of the President to achieve economy, simplicity and efficiency does not
Truth commissions have been described as bodies that share the following characteristics: (1) include the power to create an entirely new public office which was hitherto inexistent like
they examine only past events; (2) they investigate patterns of abuse committed over a the Truth Commission.
period of time, as opposed to a particular event; (3) they are temporary bodies that finish
their work with the submission of a report containing conclusions and recommendations; (c) E.O. No. 1 illegally amended the Constitution and pertinent statutes when it vested the
and (4) they are officially sanctioned, authorized or empowered by the State.[10] Truth Commission with quasi-judicial powers duplicating, if not superseding, those of the
Commissions members are usually empowered to conduct research, support victims, and Office of the Ombudsman created under the 1987 Constitution and the Department of
propose policy recommendations to prevent recurrence of crimes. Through their Justice created under the Administrative Code of 1987.
investigations, the commissions may aim to discover and learn more about past abuses, or
formally acknowledge them. They may aim to prepare the way for prosecutions and (d) E.O. No. 1 violates the equal protection clause as it selectively targets for investigation
recommend institutional reforms.[11] and prosecution officials and personnel of the previous administration as if corruption is their
peculiar species even as it excludes those of the other administrations, past and present,
Thus, their main goals range from retribution to reconciliation. The Nuremburg and Tokyo who may be indictable.
war crime tribunals are examples of a retributory or vindicatory body set up to try and punish
those responsible for crimes against humanity. A form of a reconciliatory tribunal is the Truth (e) The creation of the Philippine Truth Commission of 2010 violates the consistent and
and Reconciliation Commission of South Africa, the principal function of which was to heal general international practice of four decades wherein States constitute truth commissions
the wounds of past violence and to prevent future conflict by providing a cathartic to exclusively investigate human rights violations, which customary practice forms part of the
experience for victims. generally accepted principles of international law which the Philippines is mandated to
adhere to pursuant to the Declaration of Principles enshrined in the Constitution.
The PTC is a far cry from South Africas model. The latter placed more emphasis on
reconciliation than on judicial retribution, while the marching order of the PTC is the (f) The creation of the Truth Commission is an exercise in futility, an adventure in partisan
identification and punishment of perpetrators. As one writer[12] puts it: hostility, a launching pad for trial/conviction by publicity and a mere populist propaganda to
mistakenly impress the people that widespread poverty will altogether vanish if corruption is
The order ruled out reconciliation. It translated the Draconian code spelled out by Aquino in eliminated without even addressing the other major causes of poverty.
his inaugural speech: To those who talk about reconciliation, if they mean that they would
like us to simply forget about the wrongs that they have committed in the past, we have this (g) The mere fact that previous commissions were not constitutionally challenged is of no
to say: There can be no reconciliation without justice. When we allow crimes to go moment because neither laches nor estoppel can bar an eventual question on the
unpunished, we give consent to their occurring over and over again. constitutionality and validity of an executive issuance or even a statute.[13]

The Thrusts of the Petitions In their Consolidated Comment,[14] the respondents, through the Office of the Solicitor
General (OSG), essentially questioned the legal standing of petitioners and defended the
Barely a month after the issuance of Executive Order No. 1, the petitioners asked the Court assailed executive order with the following arguments:
to declare it unconstitutional and to enjoin the PTC from performing its functions. A perusal
of the arguments of the petitioners in both cases shows that they are essentially the same. 1] E.O. No. 1 does not arrogate the powers of Congress to create a public office because the
The petitioners-legislators summarized them in the following manner: Presidents executive power and power of control necessarily include the inherent power to
conduct investigations to ensure that laws are faithfully executed and that, in any event, the
(a) E.O. No. 1 violates the separation of powers as it arrogates the power of the Congress to Constitution, Revised Administrative Code of 1987 (E.O. No. 292), [15] Presidential Decree
create a public office and appropriate funds for its operation. (P.D.) No. 1416[16] (as amended by P.D. No. 1772), R.A. No. 9970,[17] and settled
jurisprudence that authorize the President to create or form such bodies.
172

2] E.O. No. 1 does not usurp the power of Congress to appropriate funds because there is no Like almost all powers conferred by the Constitution, the power of judicial review is subject
appropriation but a mere allocation of funds already appropriated by Congress. to limitations, to wit: (1) there must be an actual case or controversy calling for the exercise
of judicial power; (2) the person challenging the act must have the standing to question the
3] The Truth Commission does not duplicate or supersede the functions of the Office of the validity of the subject act or issuance; otherwise stated, he must have a personal and
Ombudsman (Ombudsman) and the Department of Justice (DOJ), because it is a fact-finding substantial interest in the case such that he has sustained, or will sustain, direct injury as a
body and not a quasi-judicial body and its functions do not duplicate, supplant or erode the result of its enforcement; (3) the question of constitutionality must be raised at the earliest
latters jurisdiction. opportunity; and (4) the issue of constitutionality must be the very lis mota of the case.[19]

4] The Truth Commission does not violate the equal protection clause because it was validly Among all these limitations, only the legal standing of the petitioners has been put at issue.
created for laudable purposes.
Legal Standing of the Petitioners
The OSG then points to the continued existence and validity of other executive orders and
presidential issuances creating similar bodies to justify the creation of the PTC such as The OSG attacks the legal personality of the petitioners-legislators to file their petition for
Presidential Complaint and Action Commission (PCAC) by President Ramon B. Magsaysay, failure to demonstrate their personal stake in the outcome of the case. It argues that the
Presidential Committee on Administrative Performance Efficiency (PCAPE) by President petitioners have not shown that they have sustained or are in danger of sustaining any
Carlos P. Garcia and Presidential Agency on Reform and Government Operations (PARGO) by personal injury attributable to the creation of the PTC. Not claiming to be the subject of the
President Ferdinand E. Marcos.[18] commissions investigations, petitioners will not sustain injury in its creation or as a result of
its proceedings.[20]
From the petitions, pleadings, transcripts, and memoranda, the following are the principal
issues to be resolved: The Court disagrees with the OSG in questioning the legal standing of the petitioners-
legislators to assail Executive Order No. 1. Evidently, their petition primarily invokes
1. Whether or not the petitioners have the legal standing to file their respective petitions and usurpation of the power of the Congress as a body to which they belong as members. This
question Executive Order No. 1; certainly justifies their resolve to take the cudgels for Congress as an institution and present
the complaints on the usurpation of their power and rights as members of the legislature
2. Whether or not Executive Order No. 1 violates the principle of separation of powers by
before the Court. As held in Philippine Constitution Association v. Enriquez,[21]
usurping the powers of Congress to create and to appropriate funds for public offices,
agencies and commissions; To the extent the powers of Congress are impaired, so is the power of each member thereof,
since his office confers a right to participate in the exercise of the powers of that institution.
3. Whether or not Executive Order No. 1 supplants the powers of the Ombudsman and the
DOJ; An act of the Executive which injures the institution of Congress causes a derivative but
nonetheless substantial injury, which can be questioned by a member of Congress. In such a
4. Whether or not Executive Order No. 1 violates the equal protection clause; and
case, any member of Congress can have a resort to the courts.
5. Whether or not petitioners are entitled to injunctive relief.
Indeed, legislators have a legal standing to see to it that the prerogative, powers and
privileges vested by the Constitution in their office remain inviolate. Thus, they are allowed
Essential requisites for judicial review
to question the validity of any official action which, to their mind, infringes on their
Before proceeding to resolve the issue of the constitutionality of Executive Order No. 1, the prerogatives as legislators.[22]
Court needs to ascertain whether the requisites for a valid exercise of its power of judicial
With regard to Biraogo, the OSG argues that, as a taxpayer, he has no standing to question
review are present.
the creation of the PTC and the budget for its operations.[23] It emphasizes that the funds to
be used for the creation and operation of the commission are to be taken from those funds
already appropriated by Congress. Thus, the allocation and disbursement of funds for the
173

commission will not entail congressional action but will simply be an exercise of the stringent direct injury test in Ex Parte Levitt, later reaffirmed in Tileston v. Ullman. The same
Presidents power over contingent funds. Court ruled that for a private individual to invoke the judicial power to determine the validity
of an executive or legislative action, he must show that he has sustained a direct injury as a
As correctly pointed out by the OSG, Biraogo has not shown that he sustained, or is in danger result of that action, and it is not sufficient that he has a general interest common to all
of sustaining, any personal and direct injury attributable to the implementation of Executive members of the public.
Order No. 1. Nowhere in his petition is an assertion of a clear right that may justify his clamor
for the Court to exercise judicial power and to wield the axe over presidential issuances in This Court adopted the direct injury test in our jurisdiction. In People v. Vera, it held that the
defense of the Constitution. The case of David v. Arroyo[24] explained the deep-seated rules person who impugns the validity of a statute must have a personal and substantial interest in
on locus standi. Thus: the case such that he has sustained, or will sustain direct injury as a result. The Vera doctrine
was upheld in a litany of cases, such as, Custodio v. President of the Senate, Manila Race
Locus standi is defined as a right of appearance in a court of justice on a given question. In Horse Trainers Association v. De la Fuente, Pascual v. Secretary of Public Works and Anti-
private suits, standing is governed by the real-parties-in interest rule as contained in Section Chinese League of the Philippines v. Felix. [Emphases included. Citations omitted]
2, Rule 3 of the 1997 Rules of Civil Procedure, as amended. It provides that every action must
be prosecuted or defended in the name of the real party in interest. Accordingly, the real- Notwithstanding, the Court leans on the doctrine that the rule on standing is a matter of
party-in interest is the party who stands to be benefited or injured by the judgment in the procedure, hence, can be relaxed for nontraditional plaintiffs like ordinary citizens,
suit or the party entitled to the avails of the suit. Succinctly put, the plaintiffs standing is taxpayers, and legislators when the public interest so requires, such as when the matter is of
based on his own right to the relief sought. transcendental importance, of overreaching significance to society, or of paramount public
interest.[25]
The difficulty of determining locus standi arises in public suits. Here, the plaintiff who asserts
a public right in assailing an allegedly illegal official action, does so as a representative of the Thus, in Coconut Oil Refiners Association, Inc. v. Torres,[26] the Court held that in cases of
general public. He may be a person who is affected no differently from any other person. He paramount importance where serious constitutional questions are involved, the standing
could be suing as a stranger, or in the category of a citizen, or taxpayer. In either case, he has requirements may be relaxed and a suit may be allowed to prosper even where there is no
to adequately show that he is entitled to seek judicial protection. In other words, he has to direct injury to the party claiming the right of judicial review. In the first Emergency Powers
make out a sufficient interest in the vindication of the public order and the securing of relief Cases,[27] ordinary citizens and taxpayers were allowed to question the constitutionality of
as a citizen or taxpayer. several executive orders although they had only an indirect and general interest shared in
common with the public.
Case law in most jurisdictions now allows both citizen and taxpayer standing in public
actions. The distinction was first laid down in Beauchamp v. Silk, where it was held that the The OSG claims that the determinants of transcendental importance[28] laid down in CREBA
plaintiff in a taxpayers suit is in a different category from the plaintiff in a citizens suit. In the v. ERC and Meralco[29] are non-existent in this case. The Court, however, finds reason in
former, the plaintiff is affected by the expenditure of public funds, while in the latter, he is Biraogos assertion that the petition covers matters of transcendental importance to justify
but the mere instrument of the public concern. As held by the New York Supreme Court in the exercise of jurisdiction by the Court. There are constitutional issues in the petition which
People ex rel Case v. Collins: In matter of mere public right, howeverthe people are the real deserve the attention of this Court in view of their seriousness, novelty and weight as
partiesIt is at least the right, if not the duty, of every citizen to interfere and see that a public precedents. Where the issues are of transcendental and paramount importance not only to
offence be properly pursued and punished, and that a public grievance be remedied. With the public but also to the Bench and the Bar, they should be resolved for the guidance of
respect to taxpayers suits, Terr v. Jordan held that the right of a citizen and a taxpayer to all.[30] Undoubtedly, the Filipino people are more than interested to know the status of the
maintain an action in courts to restrain the unlawful use of public funds to his injury cannot Presidents first effort to bring about a promised change to the country. The Court takes
be denied. cognizance of the petition not due to overwhelming political undertones that clothe the issue
in the eyes of the public, but because the Court stands firm in its oath to perform its
However, to prevent just about any person from seeking judicial interference in any official constitutional duty to settle legal controversies with overreaching significance to society.
policy or act with which he disagreed with, and thus hinders the activities of governmental
agencies engaged in public service, the United State Supreme Court laid down the more Power of the President to Create the Truth Commission
174

In his memorandum in G.R. No. 192935, Biraogo asserts that the Truth Commission is a power of control over his subordinates in the executive branch, but extends further in the
public office and not merely an adjunct body of the Office of the President.[31] Thus, in order exercise of his other powers, such as his power to discipline subordinates,[41] his power for
that the President may create a public office he must be empowered by the Constitution, a rule making, adjudication and licensing purposes[42] and in order to be informed on matters
statute or an authorization vested in him by law. According to petitioner, such power cannot which he is entitled to know.[43]
be presumed[32] since there is no provision in the Constitution or any specific law that
authorizes the President to create a truth commission.[33] He adds that Section 31 of the The OSG also cites the recent case of Banda v. Ermita,[44] where it was held that the
Administrative Code of 1987, granting the President the continuing authority to reorganize President has the power to reorganize the offices and agencies in the executive department
his office, cannot serve as basis for the creation of a truth commission considering the in line with his constitutionally granted power of control and by virtue of a valid delegation of
aforesaid provision merely uses verbs such as reorganize, transfer, consolidate, merge, and the legislative power to reorganize executive offices under existing statutes.
abolish.[34] Insofar as it vests in the President the plenary power to reorganize the Office of
Thus, the OSG concludes that the power of control necessarily includes the power to create
the President to the extent of creating a public office, Section 31 is inconsistent with the
offices. For the OSG, the President may create the PTC in order to, among others, put a
principle of separation of powers enshrined in the Constitution and must be deemed
closure to the reported large scale graft and corruption in the government.[45]
repealed upon the effectivity thereof.[35]
The question, therefore, before the Court is this: Does the creation of the PTC fall within the
Similarly, in G.R. No. 193036, petitioners-legislators argue that the creation of a public office
ambit of the power to reorganize as expressed in Section 31 of the Revised Administrative
lies within the province of Congress and not with the executive branch of government. They
Code? Section 31 contemplates reorganization as limited by the following functional and
maintain that the delegated authority of the President to reorganize under Section 31 of the
structural lines: (1) restructuring the internal organization of the Office of the President
Revised Administrative Code: 1) does not permit the President to create a public office, much
Proper by abolishing, consolidating or merging units thereof or transferring functions from
less a truth commission; 2) is limited to the reorganization of the administrative structure of
one unit to another; (2) transferring any function under the Office of the President to any
the Office of the President; 3) is limited to the restructuring of the internal organs of the
other Department/Agency or vice versa; or (3) transferring any agency under the Office of
Office of the President Proper, transfer of functions and transfer of agencies; and 4) only to
the President to any other Department/Agency or vice versa. Clearly, the provision refers to
achieve simplicity, economy and efficiency.[36] Such continuing authority of the President to
reduction of personnel, consolidation of offices, or abolition thereof by reason of economy or
reorganize his office is limited, and by issuing Executive Order No. 1, the President
redundancy of functions. These point to situations where a body or an office is already
overstepped the limits of this delegated authority.
existent but a modification or alteration thereof has to be effected. The creation of an office
The OSG counters that there is nothing exclusively legislative about the creation by the is nowhere mentioned, much less envisioned in said provision. Accordingly, the answer to the
President of a fact-finding body such as a truth commission. Pointing to numerous offices question is in the negative.
created by past presidents, it argues that the authority of the President to create public
To say that the PTC is borne out of a restructuring of the Office of the President under
offices within the Office of the President Proper has long been recognized.[37] According to
Section 31 is a misplaced supposition, even in the plainest meaning attributable to the term
the OSG, the Executive, just like the other two branches of government, possesses the
restructure an alteration of an existing structure. Evidently, the PTC was not part of the
inherent authority to create fact-finding committees to assist it in the performance of its
structure of the Office of the President prior to the enactment of Executive Order No. 1. As
constitutionally mandated functions and in the exercise of its administrative functions.[38]
held in Buklod ng Kawaning EIIB v. Hon. Executive Secretary,[46]
This power, as the OSG explains it, is but an adjunct of the plenary powers wielded by the
President under Section 1 and his power of control under Section 17, both of Article VII of
But of course, the list of legal basis authorizing the President to reorganize any department
the Constitution.[39]
or agency in the executive branch does not have to end here. We must not lose sight of the
very source of the power that which constitutes an express grant of power. Under Section
It contends that the President is necessarily vested with the power to conduct fact-finding
31, Book III of Executive Order No. 292 (otherwise known as the Administrative Code of
investigations, pursuant to his duty to ensure that all laws are enforced by public officials and
1987), "the President, subject to the policy in the Executive Office and in order to achieve
employees of his department and in the exercise of his authority to assume directly the
simplicity, economy and efficiency, shall have the continuing authority to reorganize the
functions of the executive department, bureau and office, or interfere with the discretion of
administrative structure of the Office of the President." For this purpose, he may transfer the
his officials.[40] The power of the President to investigate is not limited to the exercise of his
175

functions of other Departments or Agencies to the Office of the President. In Canonizado v. convening of the First Congress, as expressly provided in Section 6, Article XVIII of the 1987
Aguirre [323 SCRA 312 (2000)], we ruled that reorganization "involves the reduction of Constitution. In fact, even the Solicitor General agrees with this view. Thus:
personnel, consolidation of offices, or abolition thereof by reason of economy or redundancy
of functions." It takes place when there is an alteration of the existing structure of ASSOCIATE JUSTICE CARPIO: Because P.D. 1416 was enacted was the last whereas clause of
government offices or units therein, including the lines of control, authority and P.D. 1416 says it was enacted to prepare the transition from presidential to parliamentary.
responsibility between them. The EIIB is a bureau attached to the Department of Finance. It Now, in a parliamentary form of government, the legislative and executive powers are fused,
falls under the Office of the President. Hence, it is subject to the Presidents continuing correct?
authority to reorganize. [Emphasis Supplied]
SOLICITOR GENERAL CADIZ: Yes, Your Honor.
In the same vein, the creation of the PTC is not justified by the Presidents power of control.
ASSOCIATE JUSTICE CARPIO: That is why, that P.D. 1416 was issued. Now would you agree
Control is essentially the power to alter or modify or nullify or set aside what a subordinate
with me that P.D. 1416 should not be considered effective anymore upon the promulgation,
officer had done in the performance of his duties and to substitute the judgment of the
adoption, ratification of the 1987 Constitution.
former with that of the latter.[47] Clearly, the power of control is entirely different from the
power to create public offices. The former is inherent in the Executive, while the latter finds
SOLICITOR GENERAL CADIZ: Not the whole of P.D. [No.] 1416, Your Honor.
basis from either a valid delegation from Congress, or his inherent duty to faithfully execute
the laws. ASSOCIATE JUSTICE CARPIO: The power of the President to reorganize the entire National
Government is deemed repealed, at least, upon the adoption of the 1987 Constitution,
The question is this, is there a valid delegation of power from Congress, empowering the
correct.
President to create a public office?
SOLICITOR GENERAL CADIZ: Yes, Your Honor.[50]
According to the OSG, the power to create a truth commission pursuant to the above
provision finds statutory basis under P.D. 1416, as amended by P.D. No. 1772.[48] The said While the power to create a truth commission cannot pass muster on the basis of P.D. No.
law granted the President the continuing authority to reorganize the national government, 1416 as amended by P.D. No. 1772, the creation of the PTC finds justification under Section
including the power to group, consolidate bureaus and agencies, to abolish offices, to 17, Article VII of the Constitution, imposing upon the President the duty to ensure that the
transfer functions, to create and classify functions, services and activities, transfer laws are faithfully executed. Section 17 reads:
appropriations, and to standardize salaries and materials. This decree, in relation to Section
20, Title I, Book III of E.O. 292 has been invoked in several cases such as Larin v. Executive Section 17. The President shall have control of all the executive departments, bureaus, and
Secretary.[49] offices. He shall ensure that the laws be faithfully executed. (Emphasis supplied).

The Court, however, declines to recognize P.D. No. 1416 as a justification for the President to As correctly pointed out by the respondents, the allocation of power in the three principal
create a public office. Said decree is already stale, anachronistic and inoperable. P.D. No. branches of government is a grant of all powers inherent in them. The Presidents power to
1416 was a delegation to then President Marcos of the authority to reorganize the conduct investigations to aid him in ensuring the faithful execution of laws in this case,
administrative structure of the national government including the power to create offices fundamental laws on public accountability and transparency is inherent in the Presidents
and transfer appropriations pursuant to one of the purposes of the decree, embodied in its powers as the Chief Executive. That the authority of the President to conduct investigations
last Whereas clause: and to create bodies to execute this power is not explicitly mentioned in the Constitution or
in statutes does not mean that he is bereft of such authority.[51] As explained in the
WHEREAS, the transition towards the parliamentary form of government will necessitate landmark case of Marcos v. Manglapus:[52]
flexibility in the organization of the national government.
x x x. The 1987 Constitution, however, brought back the presidential system of government
Clearly, as it was only for the purpose of providing manageability and resiliency during the and restored the separation of legislative, executive and judicial powers by their actual
interim, P.D. No. 1416, as amended by P.D. No. 1772, became functus oficio upon the
176

distribution among three distinct branches of government with provision for checks and objective of the investigative bodies created in the past like the PCAC, PCAPE, PARGO, the
balances. Feliciano Commission, the Melo Commission and the Zenarosa Commission. There being no
changes in the government structure, the Court is not inclined to declare such executive
It would not be accurate, however, to state that "executive power" is the power to enforce power as non-existent just because the direction of the political winds have changed.
the laws, for the President is head of state as well as head of government and whatever
powers inhere in such positions pertain to the office unless the Constitution itself withholds On the charge that Executive Order No. 1 transgresses the power of Congress to appropriate
it. Furthermore, the Constitution itself provides that the execution of the laws is only one of funds for the operation of a public office, suffice it to say that there will be no appropriation
the powers of the President. It also grants the President other powers that do not involve the but only an allotment or allocations of existing funds already appropriated. Accordingly,
execution of any provision of law, e.g., his power over the country's foreign relations. there is no usurpation on the part of the Executive of the power of Congress to appropriate
funds. Further, there is no need to specify the amount to be earmarked for the operation of
On these premises, we hold the view that although the 1987 Constitution imposes limitations the commission because, in the words of the Solicitor General, whatever funds the Congress
on the exercise of specific powers of the President, it maintains intact what is traditionally has provided for the Office of the President will be the very source of the funds for the
considered as within the scope of "executive power." Corollarily, the powers of the President commission.[55] Moreover, since the amount that would be allocated to the PTC shall be
cannot be said to be limited only to the specific powers enumerated in the Constitution. In subject to existing auditing rules and regulations, there is no impropriety in the funding.
other words, executive power is more than the sum of specific powers so enumerated.
Power of the Truth Commission to Investigate
It has been advanced that whatever power inherent in the government that is neither
legislative nor judicial has to be executive. x x x. The Presidents power to conduct investigations to ensure that laws are faithfully executed is
well recognized. It flows from the faithful-execution clause of the Constitution under Article
Indeed, the Executive is given much leeway in ensuring that our laws are faithfully executed. VII, Section 17 thereof.[56] As the Chief Executive, the president represents the government
As stated above, the powers of the President are not limited to those specific powers under as a whole and sees to it that all laws are enforced by the officials and employees of his
the Constitution.[53] One of the recognized powers of the President granted pursuant to this department. He has the authority to directly assume the functions of the executive
constitutionally-mandated duty is the power to create ad hoc committees. This flows from department.[57]
the obvious need to ascertain facts and determine if laws have been faithfully executed.
Thus, in Department of Health v. Camposano,[54] the authority of the President to issue Invoking this authority, the President constituted the PTC to primarily investigate reports of
Administrative Order No. 298, creating an investigative committee to look into the graft and corruption and to recommend the appropriate action. As previously stated, no
administrative charges filed against the employees of the Department of Health for the quasi-judicial powers have been vested in the said body as it cannot adjudicate rights of
anomalous purchase of medicines was upheld. In said case, it was ruled: persons who come before it. It has been said that Quasi-judicial powers involve the power to
hear and determine questions of fact to which the legislative policy is to apply and to decide
The Chief Executives power to create the Ad hoc Investigating Committee cannot be in accordance with the standards laid down by law itself in enforcing and administering the
doubted. Having been constitutionally granted full control of the Executive Department, to same law.[58] In simpler terms, judicial discretion is involved in the exercise of these quasi-
which respondents belong, the President has the obligation to ensure that all executive judicial power, such that it is exclusively vested in the judiciary and must be clearly
officials and employees faithfully comply with the law. With AO 298 as mandate, the legality authorized by the legislature in the case of administrative agencies.
of the investigation is sustained. Such validity is not affected by the fact that the investigating
team and the PCAGC had the same composition, or that the former used the offices and The distinction between the power to investigate and the power to adjudicate was
facilities of the latter in conducting the inquiry. [Emphasis supplied] delineated by the Court in Cario v. Commission on Human Rights.[59] Thus:

It should be stressed that the purpose of allowing ad hoc investigating bodies to exist is to "Investigate," commonly understood, means to examine, explore, inquire or delve or probe
allow an inquiry into matters which the President is entitled to know so that he can be into, research on, study. The dictionary definition of "investigate" is "to observe or study
properly advised and guided in the performance of his duties relative to the execution and closely: inquire into systematically: "to search or inquire into: x x to subject to an official
enforcement of the laws of the land. And if history is to be revisited, this was also the probe x x: to conduct an official inquiry." The purpose of investigation, of course, is to
177

discover, to find out, to learn, obtain information. Nowhere included or intimated is the Section 2(g), highlights this fact and gives credence to a contrary interpretation from that of
notion of settling, deciding or resolving a controversy involved in the facts inquired into by the petitioners. The function of determining probable cause for the filing of the appropriate
application of the law to the facts established by the inquiry. complaints before the courts remains to be with the DOJ and the Ombudsman.[64]

The legal meaning of "investigate" is essentially the same: "(t)o follow up step by step by At any rate, the Ombudsmans power to investigate under R.A. No. 6770 is not exclusive but is
patient inquiry or observation. To trace or track; to search into; to examine and inquire into shared with other similarly authorized government agencies. Thus, in the case of
with care and accuracy; to find out by careful inquisition; examination; the taking of Ombudsman v. Galicia,[65] it was written:
evidence; a legal inquiry;" "to inquire; to make an investigation," "investigation" being in turn
described as "(a)n administrative function, the exercise of which ordinarily does not require a This power of investigation granted to the Ombudsman by the 1987 Constitution and The
hearing. 2 Am J2d Adm L Sec. 257; x x an inquiry, judicial or otherwise, for the discovery and Ombudsman Act is not exclusive but is shared with other similarly authorized government
collection of facts concerning a certain matter or matters." agencies such as the PCGG and judges of municipal trial courts and municipal circuit trial
courts. The power to conduct preliminary investigation on charges against public employees
"Adjudicate," commonly or popularly understood, means to adjudge, arbitrate, judge, decide, and officials is likewise concurrently shared with the Department of Justice. Despite the
determine, resolve, rule on, settle. The dictionary defines the term as "to settle finally (the passage of the Local Government Code in 1991, the Ombudsman retains concurrent
rights and duties of the parties to a court case) on the merits of issues raised: x x to pass jurisdiction with the Office of the President and the local Sanggunians to investigate
judgment on: settle judicially: x x act as judge." And "adjudge" means "to decide or rule upon complaints against local elective officials. [Emphasis supplied].
as a judge or with judicial or quasi-judicial powers: x x to award or grant judicially in a case of
controversy x x." Also, Executive Order No. 1 cannot contravene the power of the Ombudsman to investigate
criminal cases under Section 15 (1) of R.A. No. 6770, which states:
In the legal sense, "adjudicate" means: "To settle in the exercise of judicial authority. To
determine finally. Synonymous with adjudge in its strictest sense;" and "adjudge" means: "To (1) Investigate and prosecute on its own or on complaint by any person, any act or omission
pass on judicially, to decide, settle or decree, or to sentence or condemn. x x. Implies a of any public officer or employee, office or agency, when such act or omission appears to be
judicial determination of a fact, and the entry of a judgment." [Italics included. Citations illegal, unjust, improper or inefficient. It has primary jurisdiction over cases cognizable by the
Omitted] Sandiganbayan and, in the exercise of its primary jurisdiction, it may take over, at any stage,
from any investigatory agency of government, the investigation of such cases.
Fact-finding is not adjudication and it cannot be likened to the judicial function of a court of
justice, or even a quasi-judicial agency or office. The function of receiving evidence and The act of investigation by the Ombudsman as enunciated above contemplates the conduct
ascertaining therefrom the facts of a controversy is not a judicial function. To be considered of a preliminary investigation or the determination of the existence of probable cause. This is
as such, the act of receiving evidence and arriving at factual conclusions in a controversy categorically out of the PTCs sphere of functions. Its power to investigate is limited to
must be accompanied by the authority of applying the law to the factual conclusions to the obtaining facts so that it can advise and guide the President in the performance of his duties
end that the controversy may be decided or resolved authoritatively, finally and definitively, relative to the execution and enforcement of the laws of the land. In this regard, the PTC
subject to appeals or modes of review as may be provided by law.[60] Even respondents commits no act of usurpation of the Ombudsmans primordial duties.
themselves admit that the commission is bereft of any quasi-judicial power.[61]
The same holds true with respect to the DOJ. Its authority under Section 3 (2), Chapter 1,
Contrary to petitioners apprehension, the PTC will not supplant the Ombudsman or the DOJ Title III, Book IV in the Revised Administrative Code is by no means exclusive and, thus, can
or erode their respective powers. If at all, the investigative function of the commission will be shared with a body likewise tasked to investigate the commission of crimes.
complement those of the two offices. As pointed out by the Solicitor General, the
Finally, nowhere in Executive Order No. 1 can it be inferred that the findings of the PTC are to
recommendation to prosecute is but a consequence of the overall task of the commission to
be accorded conclusiveness. Much like its predecessors, the Davide Commission, the
conduct a fact-finding investigation.[62] The actual prosecution of suspected offenders,
Feliciano Commission and the Zenarosa Commission, its findings would, at best, be
much less adjudication on the merits of the charges against them,[63] is certainly not a
recommendatory in nature. And being so, the Ombudsman and the DOJ have a wider degree
function given to the commission. The phrase, when in the course of its investigation, under
of latitude to decide whether or not to reject the recommendation. These offices, therefore,
178

are not deprived of their mandated duties but will instead be aided by the reports of the PTC based on substantial distinctions and is germane to the evils which the Executive Order seeks
for possible indictments for violations of graft laws. to correct.[72] To distinguish the Arroyo administration from past administrations, it recited
the following:
Violation of the Equal Protection Clause
First. E.O. No. 1 was issued in view of widespread reports of large scale graft and corruption
Although the purpose of the Truth Commission falls within the investigative power of the in the previous administration which have eroded public confidence in public institutions.
President, the Court finds difficulty in upholding the constitutionality of Executive Order No. There is, therefore, an urgent call for the determination of the truth regarding certain reports
1 in view of its apparent transgression of the equal protection clause enshrined in Section 1, of large scale graft and corruption in the government and to put a closure to them by the
Article III (Bill of Rights) of the 1987 Constitution. Section 1 reads: filing of the appropriate cases against those involved, if warranted, and to deter others from
committing the evil, restore the peoples faith and confidence in the Government and in their
Section 1. No person shall be deprived of life, liberty, or property without due process of law,
public servants.
nor shall any person be denied the equal protection of the laws.
Second. The segregation of the preceding administration as the object of fact-finding is
The petitioners assail Executive Order No. 1 because it is violative of this constitutional
warranted by the reality that unlike with administrations long gone, the current
safeguard. They contend that it does not apply equally to all members of the same class such
administration will most likely bear the immediate consequence of the policies of the
that the intent of singling out the previous administration as its sole object makes the PTC an
previous administration.
adventure in partisan hostility.[66] Thus, in order to be accorded with validity, the
commission must also cover reports of graft and corruption in virtually all administrations Third. The classification of the previous administration as a separate class for investigation
previous to that of former President Arroyo.[67] lies in the reality that the evidence of possible criminal activity, the evidence that could lead
to recovery of public monies illegally dissipated, the policy lessons to be learned to ensure
The petitioners argue that the search for truth behind the reported cases of graft and
that anti-corruption laws are faithfully executed, are more easily established in the regime
corruption must encompass acts committed not only during the administration of former
that immediately precede the current administration.
President Arroyo but also during prior administrations where the same magnitude of
controversies and anomalies[68] were reported to have been committed against the Filipino Fourth. Many administrations subject the transactions of their predecessors to investigations
people. They assail the classification formulated by the respondents as it does not fall under to provide closure to issues that are pivotal to national life or even as a routine measure of
the recognized exceptions because first, there is no substantial distinction between the due diligence and good housekeeping by a nascent administration like the Presidential
group of officials targeted for investigation by Executive Order No. 1 and other groups or Commission on Good Government (PCGG), created by the late President Corazon C. Aquino
persons who abused their public office for personal gain; and second, the selective under Executive Order No. 1 to pursue the recovery of ill-gotten wealth of her predecessor
classification is not germane to the purpose of Executive Order No. 1 to end corruption.[69] former President Ferdinand Marcos and his cronies, and the Saguisag Commission created by
In order to attain constitutional permission, the petitioners advocate that the commission former President Joseph Estrada under Administrative Order No, 53, to form an ad-hoc and
should deal with graft and grafters prior and subsequent to the Arroyo administration with independent citizens committee to investigate all the facts and circumstances surrounding
the strong arm of the law with equal force.[70] Philippine Centennial projects of his predecessor, former President Fidel V. Ramos.[73]

Position of respondents Concept of the Equal Protection Clause

According to respondents, while Executive Order No. 1 identifies the previous administration One of the basic principles on which this government was founded is that of the equality of
as the initial subject of the investigation, following Section 17 thereof, the PTC will not right which is embodied in Section 1, Article III of the 1987 Constitution. The equal protection
confine itself to cases of large scale graft and corruption solely during the said of the laws is embraced in the concept of due process, as every unfair discrimination offends
administration.[71] Assuming arguendo that the commission would confine its proceedings the requirements of justice and fair play. It has been embodied in a separate clause,
to officials of the previous administration, the petitioners argue that no offense is committed however, to provide for a more specific guaranty against any form of undue favoritism or
against the equal protection clause for the segregation of the transactions of public officers hostility from the government. Arbitrariness in general may be challenged on the basis of the
during the previous administration as possible subjects of investigation is a valid classification
179

due process clause. But if the particular act assailed partakes of an unwarranted partiality or The classification must not be based on existing circumstances only, or so constituted as to
prejudice, the sharper weapon to cut it down is the equal protection clause.[74] preclude addition to the number included in the class. It must be of such a nature as to
embrace all those who may thereafter be in similar circumstances and conditions. It must not
According to a long line of decisions, equal protection simply requires that all persons or leave out or underinclude those that should otherwise fall into a certain classification. As
things similarly situated should be treated alike, both as to rights conferred and elucidated in Victoriano v. Elizalde Rope Workers' Union[85] and reiterated in a long line of
responsibilities imposed.[75] It requires public bodies and institutions to treat similarly cases,[86]
situated individuals in a similar manner.[76] The purpose of the equal protection clause is to
secure every person within a states jurisdiction against intentional and arbitrary The guaranty of equal protection of the laws is not a guaranty of equality in the application of
discrimination, whether occasioned by the express terms of a statue or by its improper the laws upon all citizens of the state. It is not, therefore, a requirement, in order to avoid
execution through the states duly constituted authorities.[77] In other words, the concept of the constitutional prohibition against inequality, that every man, woman and child should be
equal justice under the law requires the state to govern impartially, and it may not draw affected alike by a statute. Equality of operation of statutes does not mean indiscriminate
distinctions between individuals solely on differences that are irrelevant to a legitimate operation on persons merely as such, but on persons according to the circumstances
governmental objective.[78] surrounding them. It guarantees equality, not identity of rights. The Constitution does not
require that things which are different in fact be treated in law as though they were the
The equal protection clause is aimed at all official state actions, not just those of the same. The equal protection clause does not forbid discrimination as to things that are
legislature.[79] Its inhibitions cover all the departments of the government including the different. It does not prohibit legislation which is limited either in the object to which it is
political and executive departments, and extend to all actions of a state denying equal directed or by the territory within which it is to operate.
protection of the laws, through whatever agency or whatever guise is taken. [80]
The equal protection of the laws clause of the Constitution allows classification. Classification
It, however, does not require the universal application of the laws to all persons or things in law, as in the other departments of knowledge or practice, is the grouping of things in
without distinction. What it simply requires is equality among equals as determined speculation or practice because they agree with one another in certain particulars. A law is
according to a valid classification. Indeed, the equal protection clause permits classification. not invalid because of simple inequality. The very idea of classification is that of inequality, so
Such classification, however, to be valid must pass the test of reasonableness. The test has that it goes without saying that the mere fact of inequality in no manner determines the
four requisites: (1) The classification rests on substantial distinctions; (2) It is germane to the matter of constitutionality. All that is required of a valid classification is that it be reasonable,
purpose of the law; (3) It is not limited to existing conditions only; and which means that the classification should be based on substantial distinctions which make
for real differences, that it must be germane to the purpose of the law; that it must not be
(4) It applies equally to all members of the same class.[81] Superficial differences do not
limited to existing conditions only; and that it must apply equally to each member of the
make for a valid classification.[82]
class. This Court has held that the standard is satisfied if the classification or distinction is
based on a reasonable foundation or rational basis and is not palpably arbitrary.
For a classification to meet the requirements of constitutionality, it must include or embrace
all persons who naturally belong to the class.[83] The classification will be regarded as invalid
Applying these precepts to this case, Executive Order No. 1 should be struck down as
if all the members of the class are not similarly treated, both as to rights conferred and
violative of the equal protection clause. The clear mandate of the envisioned truth
obligations imposed. It is not necessary that the classification be made with absolute
commission is to investigate and find out the truth concerning the reported cases of graft
symmetry, in the sense that the members of the class should possess the same
and corruption during the previous administration[87] only. The intent to single out the
characteristics in equal degree. Substantial similarity will suffice; and as long as this is
previous administration is plain, patent and manifest. Mention of it has been made in at least
achieved, all those covered by the classification are to be treated equally. The mere fact that
three portions of the questioned executive order. Specifically, these are:
an individual belonging to a class differs from the other members, as long as that class is
substantially distinguishable from all others, does not justify the non-application of the law WHEREAS, there is a need for a separate body dedicated solely to investigating and finding
to him.[84] out the truth concerning the reported cases of graft and corruption during the previous
administration, and which will recommend the prosecution of the offenders and secure
justice for all;
180

SECTION 1. Creation of a Commission. There is hereby created the PHILIPPINE TRUTH The probability that there would be difficulty in unearthing evidence or that the earlier
COMMISSION, hereinafter referred to as the COMMISSION, which shall primarily seek and reports involving the earlier administrations were already inquired into is beside the point.
find the truth on, and toward this end, investigate reports of graft and corruption of such Obviously, deceased presidents and cases which have already prescribed can no longer be
scale and magnitude that shock and offend the moral and ethical sensibilities of the people, the subjects of inquiry by the PTC. Neither is the PTC expected to conduct simultaneous
committed by public officers and employees, their co-principals, accomplices and accessories investigations of previous administrations, given the bodys limited time and resources. The
from the private sector, if any, during the previous administration; and thereafter law does not require the impossible (Lex non cogit ad impossibilia).[91]
recommend the appropriate action or measure to be taken thereon to ensure that the full
measure of justice shall be served without fear or favor. Given the foregoing physical and legal impossibility, the Court logically recognizes the
unfeasibility of investigating almost a centurys worth of graft cases. However, the fact
SECTION 2. Powers and Functions. The Commission, which shall have all the powers of an remains that Executive Order No. 1 suffers from arbitrary classification. The PTC, to be true
investigative body under Section 37, Chapter 9, Book I of the Administrative Code of 1987, is to its mandate of searching for the truth, must not exclude the other past administrations.
primarily tasked to conduct a thorough fact-finding investigation of reported cases of graft The PTC must, at least, have the authority to investigate all past administrations. While
and corruption referred to in Section 1, involving third level public officers and higher, their reasonable prioritization is permitted, it should not be arbitrary lest it be struck down for
co-principals, accomplices and accessories from the private sector, if any, during the previous being unconstitutional. In the often quoted language of Yick Wo v. Hopkins,[92]
administration and thereafter submit its finding and recommendations to the President,
Congress and the Ombudsman. [Emphases supplied] Though the law itself be fair on its face and impartial in appearance, yet, if applied and
administered by public authority with an evil eye and an unequal hand, so as practically to
In this regard, it must be borne in mind that the Arroyo administration is but just a member make unjust and illegal discriminations between persons in similar circumstances, material to
of a class, that is, a class of past administrations. It is not a class of its own. Not to include their rights, the denial of equal justice is still within the prohibition of the constitution.
past administrations similarly situated constitutes arbitrariness which the equal protection
clause cannot sanction. Such discriminating differentiation clearly reverberates to label the It could be argued that considering that the PTC is an ad hoc body, its scope is limited. The
commission as a vehicle for vindictiveness and selective retribution. Court, however, is of the considered view that although its focus is restricted, the
constitutional guarantee of equal protection under the laws should not in any way be
Though the OSG enumerates several differences between the Arroyo administration and circumvented. The Constitution is the fundamental and paramount law of the nation to
other past administrations, these distinctions are not substantial enough to merit the which all other laws must conform and in accordance with which all private rights
restriction of the investigation to the previous administration only. The reports of determined and all public authority administered.[93] Laws that do not conform to the
widespread corruption in the Arroyo administration cannot be taken as basis for Constitution should be stricken down for being unconstitutional.[94] While the thrust of the
distinguishing said administration from earlier administrations which were also blemished by PTC is specific, that is, for investigation of acts of graft and corruption, Executive Order No. 1,
similar widespread reports of impropriety. They are not inherent in, and do not inure solely to survive, must be read together with the provisions of the Constitution. To exclude the
to, the Arroyo administration. As Justice Isagani Cruz put it, Superficial differences do not earlier administrations in the guise of substantial distinctions would only confirm the
make for a valid classification.[88] petitioners lament that the subject executive order is only an adventure in partisan hostility.
In the case of US v. Cyprian,[95] it was written: A rather limited number of such
The public needs to be enlightened why Executive Order No. 1 chooses to limit the scope of classifications have routinely been held or assumed to be arbitrary; those include: race,
the intended investigation to the previous administration only. The OSG ventures to opine national origin, gender, political activity or membership in a political party, union activity or
that to include other past administrations, at this point, may unnecessarily overburden the membership in a labor union, or more generally the exercise of first amendment rights.
commission and lead it to lose its effectiveness.[89] The reason given is specious. It is without
doubt irrelevant to the legitimate and noble objective of the PTC to stamp out or end To reiterate, in order for a classification to meet the requirements of constitutionality, it
corruption and the evil it breeds.[90] must include or embrace all persons who naturally belong to the class.[96] Such a
classification must not be based on existing circumstances only, or so constituted as to
preclude additions to the number included within a class, but must be of such a nature as to
embrace all those who may thereafter be in similar circumstances and conditions.
181

Furthermore, all who are in situations and circumstances which are relative to the was crafted to tailor-fit the prosecution of officials and personalities of the Arroyo
discriminatory legislation and which are indistinguishable from those of the members of the administration.[105]
class must be brought under the influence of the law and treated by it in the same way as are
the members of the class.[97] The Court tried to seek guidance from the pronouncement in the case of Virata v.
Sandiganbayan,[106] that the PCGG Charter (composed of Executive Orders Nos. 1, 2 and 14)
The Court is not unaware that mere underinclusiveness is not fatal to the validity of a law does not violate the equal protection clause. The decision, however, was devoid of any
under the equal protection clause.[98] Legislation is not unconstitutional merely because it is discussion on how such conclusory statement was arrived at, the principal issue in said case
not all-embracing and does not include all the evils within its reach.[99] It has been written being only the sufficiency of a cause of action.
that a regulation challenged under the equal protection clause is not devoid of a rational
predicate simply because it happens to be incomplete.[100] In several instances, the A final word
underinclusiveness was not considered a valid reason to strike down a law or regulation
The issue that seems to take center stage at present is - whether or not the Supreme Court,
where the purpose can be attained in future legislations or regulations. These cases refer to
in the exercise of its constitutionally mandated power of Judicial Review with respect to
the step by step process.[101] With regard to equal protection claims, a legislature does not
recent initiatives of the legislature and the executive department, is exercising undue
run the risk of losing the entire remedial scheme simply because it fails, through
interference. Is the Highest Tribunal, which is expected to be the protector of the
inadvertence or otherwise, to cover every evil that might conceivably have been
Constitution, itself guilty of violating fundamental tenets like the doctrine of separation of
attacked.[102]
powers? Time and again, this issue has been addressed by the Court, but it seems that the
In Executive Order No. 1, however, there is no inadvertence. That the previous present political situation calls for it to once again explain the legal basis of its action lest it
administration was picked out was deliberate and intentional as can be gleaned from the fact continually be accused of being a hindrance to the nations thrust to progress.
that it was underscored at least three times in the assailed executive order. It must be noted
The Philippine Supreme Court, according to Article VIII, Section 1 of the 1987 Constitution, is
that Executive Order No. 1 does not even mention any particular act, event or report to be
vested with Judicial Power that includes the duty of the courts of justice to settle actual
focused on unlike the investigative commissions created in the past. The equal protection
controversies involving rights which are legally demandable and enforceable, and to
clause is violated by purposeful and intentional discrimination.[103]
determine whether or not there has been a grave of abuse of discretion amounting to lack or
To disprove petitioners contention that there is deliberate discrimination, the OSG clarifies excess of jurisdiction on the part of any branch or instrumentality of the government.
that the commission does not only confine itself to cases of large scale graft and corruption
Furthermore, in Section 4(2) thereof, it is vested with the power of judicial review which is
committed during the previous administration.[104] The OSG points to Section 17 of
the power to declare a treaty, international or executive agreement, law, presidential decree,
Executive Order No. 1, which provides:
proclamation, order, instruction, ordinance, or regulation unconstitutional. This power also
SECTION 17. Special Provision Concerning Mandate. If and when in the judgment of the includes the duty to rule on the constitutionality of the application, or operation of
President there is a need to expand the mandate of the Commission as defined in Section 1 presidential decrees, proclamations, orders, instructions, ordinances, and other regulations.
hereof to include the investigation of cases and instances of graft and corruption during the These provisions, however, have been fertile grounds of conflict between the Supreme
prior administrations, such mandate may be so extended accordingly by way of a Court, on one hand, and the two co-equal bodies of government, on the other. Many times
supplemental Executive Order. the Court has been accused of asserting superiority over the other departments.

The Court is not convinced. Although Section 17 allows the President the discretion to To answer this accusation, the words of Justice Laurel would be a good source of
expand the scope of investigations of the PTC so as to include the acts of graft and corruption enlightenment, to wit: And when the judiciary mediates to allocate constitutional
committed in other past administrations, it does not guarantee that they would be covered boundaries, it does not assert any superiority over the other departments; it does not in
in the future. Such expanded mandate of the commission will still depend on the whim and reality nullify or invalidate an act of the legislature, but only asserts the solemn and sacred
caprice of the President. If he would decide not to include them, the section would then be obligation assigned to it by the Constitution to determine conflicting claims of authority
meaningless. This will only fortify the fears of the petitioners that the Executive Order No. 1
182

under the Constitution and to establish for the parties in an actual controversy the rights SO ORDERED.
which that instrument secures and guarantees to them.[107]
When the judiciary mediates to allocate constitutional boundaries, it does not assert any
Thus, the Court, in exercising its power of judicial review, is not imposing its own will upon a superiority over the other departments; it does not in reality nullify or invalidate an act of the
co-equal body but rather simply making sure that any act of government is done in legislature, but only asserts the solemn and sacred obligation assigned to it by the
consonance with the authorities and rights allocated to it by the Constitution. And, if after Constitution to determine conflicting claims of authority under the Constitution and to
said review, the Court finds no constitutional violations of any sort, then, it has no more establish for the parties in an actual controversy the rights which that instrument secures
authority of proscribing the actions under review. Otherwise, the Court will not be deterred and guarantees to them. --- Justice Jose P. Laurel
to pronounce said act as void and unconstitutional.
Facts:
It cannot be denied that most government actions are inspired with noble intentions, all
geared towards the betterment of the nation and its people. But then again, it is important The genesis of the foregoing cases can be traced to the events prior to the historic May 2010
to remember this ethical principle: The end does not justify the means. No matter how noble elections, when then Senator Benigno Simeon Aquino III declared his staunch condemnation
and worthy of admiration the purpose of an act, but if the means to be employed in of graft and corruption with his slogan, "Kung walang corrupt, walang mahirap." The Filipino
accomplishing it is simply irreconcilable with constitutional parameters, then it cannot still be people, convinced of his sincerity and of his ability to carry out this noble objective,
allowed.[108] The Court cannot just turn a blind eye and simply let it pass. It will continue to catapulted the good senator to the presidency.
uphold the Constitution and its enshrined principles.
The first case is G.R. No. 192935, a special civil action for prohibition instituted by petitioner
The Constitution must ever remain supreme. All must bow to the mandate of this law. Louis Biraogo (Biraogo) in his capacity as a citizen and taxpayer. Biraogo assails Executive
Expediency must not be allowed to sap its strength nor greed for power debase its Order No. 1 for being violative of the legislative power of Congress under Section 1, Article VI
rectitude.[109] of the Constitution as it usurps the constitutional authority of the legislature to create a
public office and to appropriate funds therefor.
Lest it be misunderstood, this is not the death knell for a truth commission as nobly
envisioned by the present administration. Perhaps a revision of the executive issuance so as The second case, G.R. No. 193036, is a special civil action for certiorari and prohibition filed
to include the earlier past administrations would allow it to pass the test of reasonableness by petitioners Edcel C. Lagman, Rodolfo B. Albano Jr., Simeon A. Datumanong, and Orlando B.
and not be an affront to the Constitution. Of all the branches of the government, it is the Fua, Sr. (petitioners-legislators) as incumbent members of the House of Representatives.
judiciary which is the most interested in knowing the truth and so it will not allow itself to be
Thus, at the dawn of his administration, the President on July 30, 2010, signed Executive
a hindrance or obstacle to its attainment. It must, however, be emphasized that the search
Order No. 1 establishing the Philippine Truth Commission of 2010 (Truth Commission).
for the truth must be within constitutional bounds for ours is still a government of laws and
not of men.[110]
Issues:
WHEREFORE, the petitions are GRANTED. Executive Order No. 1 is hereby declared
1. Whether or not the petitioners have the legal standing to file their respective petitions and
UNCONSTITUTIONAL insofar as it is violative of the equal protection clause of the
question Executive Order No. 1;
Constitution.
2. Whether or not Executive Order No. 1 violates the principle of separation of powers by
As also prayed for, the respondents are hereby ordered to cease and desist from carrying out
usurping the powers of Congress to create and to appropriate funds for public offices,
the provisions of Executive Order No. 1.
agencies and commissions;

3. Whether or not Executive Order No. 1 supplants the powers of the Ombudsman and the
DOJ;
183

4. Whether or not Executive Order No. 1 violates the equal protection clause; and In the legal sense, "adjudicate" means: "To settle in the exercise of judicial authority. To
determine finally. Synonymous with adjudge in its strictest sense;" and "adjudge" means: "To
5. Whether or not petitioners are entitled to injunctive relief. pass on judicially, to decide, settle or decree, or to sentence or condemn. x x. Implies a
judicial determination of a fact, and the entry of a judgment."
Held:
Finally, nowhere in Executive Order No. 1 can it be inferred that the findings of the PTC are to
Legal Standing of the Petitioners
be accorded conclusiveness. Much like its predecessors, the Davide Commission, the
Feliciano Commission and the Zenarosa Commission, its findings would, at best, be
The Court, however, finds reason in Biraogo’s assertion that the petition covers matters of
recommendatory in nature. And being so, the Ombudsman and the DOJ have a wider degree
transcendental importance to justify the exercise of jurisdiction by the Court. There are
of latitude to decide whether or not to reject the recommendation. These offices, therefore,
constitutional issues in the petition which deserve the attention of this Court in view of their
are not deprived of their mandated duties but will instead be aided by the reports of the PTC
seriousness, novelty and weight as precedents. Where the issues are of transcendental and
for possible indictments for violations of graft laws.
paramount importance not only to the public but also to the Bench and the Bar, they should
be resolved for the guidance of all.Undoubtedly, the Filipino people are more than interested
Violation of the Equal Protection Clause
to know the status of the President’s first effort to bring about a promised change to the
country. The Court takes cognizance of the petition not due to overwhelming political The petitioners assail Executive Order No. 1 because it is violative of this constitutional
undertones that clothe the issue in the eyes of the public, but because the Court stands firm safeguard. They contend that it does not apply equally to all members of the same class such
in its oath to perform its constitutional duty to settle legal controversies with overreaching that the intent of singling out the "previous administration" as its sole object makes the PTC
significance to society. an "adventure in partisan hostility." Thus, in order to be accorded with validity, the
commission must also cover reports of graft and corruption in virtually all administrations
Power of the President to Create the Truth Commission
previous to that of former President Arroyo.
The Chief Executive’s power to create the Ad hoc Investigating Committee cannot be
The equal protection clause is aimed at all official state actions, not just those of the
doubted. Having been constitutionally granted full control of the Executive Department, to
legislature. Its inhibitions cover all the departments of the government including the political
which respondents belong, the President has the obligation to ensure that all executive
and executive departments, and extend to all actions of a state denying equal protection of
officials and employees faithfully comply with the law. With AO 298 as mandate, the legality
the laws, through whatever agency or whatever guise is taken.
of the investigation is sustained. Such validity is not affected by the fact that the investigating
team and the PCAGC had the same composition, or that the former used the offices and Applying these precepts to this case, Executive Order No. 1 should be struck down as
facilities of the latter in conducting the inquiry. violative of the equal protection clause. The clear mandate of the envisioned truth
commission is to investigate and find out the truth "concerning the reported cases of graft
Power of the Truth Commission to Investigate
and corruption during the previous administration"only. The intent to single out the previous
administration is plain, patent and manifest. Mention of it has been made in at least three
The distinction between the power to investigate and the power to adjudicate was
portions of the questioned executive order.
delineated by the Court in Cariño v. Commission on Human Rights.59 Thus:
Decision
The legal meaning of "investigate" is essentially the same: "(t)o follow up step by step by
patient inquiry or observation. To trace or track; to search into; to examine and inquire into
The issue that seems to take center stage at present is - whether or not the Supreme Court,
with care and accuracy; to find out by careful inquisition; examination; the taking of
in the exercise of its constitutionally mandated power of Judicial Review with respect to
evidence; a legal inquiry;" "to inquire; to make an investigation," "investigation" being in turn
recent initiatives of the legislature and the executive department, is exercising undue
described as "(a)n administrative function, the exercise of which ordinarily does not require a
interference. Is the Highest Tribunal, which is expected to be the protector of the
hearing. 2 Am J2d Adm L Sec. 257; x x an inquiry, judicial or otherwise, for the discovery and
Constitution, itself guilty of violating fundamental tenets like the doctrine of separation of
collection of facts concerning a certain matter or matters."
powers? Time and again, this issue has been addressed by the Court, but it seems that the
184

present political situation calls for it to once again explain the legal basis of its action lest it (c) E.O. No. 1 illegally amended the Constitution and statutes when it vested the “Truth
continually be accused of being a hindrance to the nation’s thrust to progress. Commission” with quasi-judicial powers duplicating, if not superseding, those of the Office of
the Ombudsman created under the 1987 Constitution and the DOJ created under the
WHEREFORE, the petitions are GRANTED. Executive Order No. 1 is hereby declared Administrative Code of 1987.
UNCONSTITUTIONAL insofar as it is violative of the equal protection clause of the
Constitution. (d) E.O. No. 1 violates the equal protection clause as it selectively targets for investigation
and prosecution officials and personnel of the previous administration as if corruption is their
As also prayed for, the respondents are hereby ordered to cease and desist from carrying out peculiar species even as it excludes those of the other administrations, past and present,
the provisions of Executive Order No. 1. who may be indictable.

SO ORDERED. Respondents, through OSG, questioned the legal standing of petitioners and argued that:

FACTS: 1] E.O. No. 1 does not arrogate the powers of Congress because the President’s executive
power and power of control necessarily include the inherent power to conduct investigations
Pres. Aquino signed E. O. No. 1 establishing Philippine Truth Commission of 2010 (PTC) dated
to ensure that laws are faithfully executed and that, in any event, the Constitution, Revised
July 30, 2010.
Administrative Code of 1987, PD No. 141616 (as amended), R.A. No. 9970 and settled
jurisprudence, authorize the President to create or form such bodies.
PTC is a mere ad hoc body formed under the Office of the President with the primary task to
investigate reports of graft and corruption committed by third-level public officers and
2] E.O. No. 1 does not usurp the power of Congress to appropriate funds because there is no
employees, their co-principals, accomplices and accessories during the previous
appropriation but a mere allocation of funds already appropriated by Congress.
administration, and to submit its finding and recommendations to the President, Congress
and the Ombudsman. PTC has all the powers of an investigative body. But it is not a quasi- 3] The Truth Commission does not duplicate or supersede the functions of the Ombudsman
judicial body as it cannot adjudicate, arbitrate, resolve, settle, or render awards in disputes and the DOJ, because it is a fact-finding body and not a quasi-judicial body and its functions
between contending parties. All it can do is gather, collect and assess evidence of graft and do not duplicate, supplant or erode the latter’s jurisdiction.
corruption and make recommendations. It may have subpoena powers but it has no power
to cite people in contempt, much less order their arrest. Although it is a fact-finding body, it 4] The Truth Commission does not violate the equal protection clause because it was validly
cannot determine from such facts if probable cause exists as to warrant the filing of an created for laudable purposes.
information in our courts of law.
ISSUES:
Petitioners asked the Court to declare it unconstitutional and to enjoin the PTC from
performing its functions. They argued that: 1. WON the petitioners have legal standing to file the petitions and question E. O. No. 1;

(a) E.O. No. 1 violates separation of powers as it arrogates the power of the Congress to 2. WON E. O. No. 1 violates the principle of separation of powers by usurping the powers of
create a public office and appropriate funds for its operation. Congress to create and to appropriate funds for public offices, agencies and commissions;

3. WON E. O. No. 1 supplants the powers of the Ombudsman and the DOJ;

(b) The provision of Book III, Chapter 10, Section 31 of the Administrative Code of 1987 4. WON E. O. No. 1 violates the equal protection clause.
cannot legitimize E.O. No. 1 because the delegated authority of the President to structurally
RULING:
reorganize the Office of the President to achieve economy, simplicity and efficiency does not
include the power to create an entirely new public office which was hitherto inexistent like
The power of judicial review is subject to limitations, to wit: (1) there must be an actual case
the “Truth Commission.”
or controversy calling for the exercise of judicial power; (2) the person challenging the act
185

must have the standing to question the validity of the subject act or issuance; otherwise of the recognized powers of the President granted pursuant to this constitutionally-
stated, he must have a personal and substantial interest in the case such that he has mandated duty is the power to create ad hoc committees. This flows from the obvious need
sustained, or will sustain, direct injury as a result of its enforcement; (3) the question of to ascertain facts and determine if laws have been faithfully executed. The purpose of
constitutionality must be raised at the earliest opportunity; and (4) the issue of allowing ad hoc investigating bodies to exist is to allow an inquiry into matters which the
constitutionality must be the very lis mota of the case. President is entitled to know so that he can be properly advised and guided in the
performance of his duties relative to the execution and enforcement of the laws of the land.

2. There will be no appropriation but only an allotment or allocations of existing funds


1. The petition primarily invokes usurpation of the power of the Congress as a body to which already appropriated. There is no usurpation on the part of the Executive of the power of
they belong as members. To the extent the powers of Congress are impaired, so is the power Congress to appropriate funds. There is no need to specify the amount to be earmarked for
of each member thereof, since his office confers a right to participate in the exercise of the the operation of the commission because, whatever funds the Congress has provided for the
powers of that institution. Office of the President will be the very source of the funds for the commission. The amount
that would be allocated to the PTC shall be subject to existing auditing rules and regulations
Legislators have a legal standing to see to it that the prerogative, powers and privileges
so there is no impropriety in the funding.
vested by the Constitution in their office remain inviolate. Thus, they are allowed to question
the validity of any official action which, to their mind, infringes on their prerogatives as 3. PTC will not supplant the Ombudsman or the DOJ or erode their respective powers. If at
legislators. all, the investigative function of the commission will complement those of the two offices.
The function of determining probable cause for the filing of the appropriate complaints
With regard to Biraogo, he has not shown that he sustained, or is in danger of sustaining, any
before the courts remains to be with the DOJ and the Ombudsman. PTC’s power to
personal and direct injury attributable to the implementation of E. O. No. 1.
investigate is limited to obtaining facts so that it can advise and guide the President in the
performance of his duties relative to the execution and enforcement of the laws of the land.
Locus standi is “a right of appearance in a court of justice on a given question.” In private
suits, standing is governed by the “real-parties-in interest” rule. It provides that “every action
4. Court finds difficulty in upholding the constitutionality of Executive Order No. 1 in view of
must be prosecuted or defended in the name of the real party in interest.” Real-party-in
its apparent transgression of the equal protection clause enshrined in Section 1, Article III
interest is “the party who stands to be benefited or injured by the judgment in the suit or the
(Bill of Rights) of the 1987 Constitution.
party entitled to the avails of the suit.”
Equal protection requires that all persons or things similarly situated should be treated alike,
Difficulty of determining locus standi arises in public suits. Here, the plaintiff who asserts a
both as to rights conferred and responsibilities imposed. It requires public bodies and
“public right” in assailing an allegedly illegal official action, does so as a representative of the
institutions to treat similarly situated individuals in a similar manner. The purpose of the
general public. He has to show that he is entitled to seek judicial protection. He has to make
equal protection clause is to secure every person within a state’s jurisdiction against
out a sufficient interest in the vindication of the public order and the securing of relief as a
intentional and arbitrary discrimination, whether occasioned by the express terms of a statue
“citizen” or “taxpayer.
or by its improper execution through the state’s duly constituted authorities.
The person who impugns the validity of a statute must have “a personal and substantial
There must be equality among equals as determined according to a valid classification. Equal
interest in the case such that he has sustained, or will sustain direct injury as a result.” The
protection clause permits classification. Such classification, however, to be valid must pass
Court, however, finds reason in Biraogo’s assertion that the petition covers matters of
the test of reasonableness. The test has four requisites: (1) The classification rests on
transcendental importance to justify the exercise of jurisdiction by the Court. There are
substantial distinctions; (2) It is germane to the purpose of the law; (3) It is not limited to
constitutional issues in the petition which deserve the attention of this Court in view of their
existing conditions only; and (4) It applies equally to all members of the same class.
seriousness, novelty and weight as precedents
The classification will be regarded as invalid if all the members of the class are not similarly
The Executive is given much leeway in ensuring that our laws are faithfully executed. The
treated, both as to rights conferred and obligations imposed.
powers of the President are not limited to those specific powers under the Constitution. One
186

Executive Order No. 1 should be struck down as violative of the equal protection clause. The The provisions of Book III, Chapter 10, Section 31 of the Administrative Code of 1987 cannot
clear mandate of truth commission is to investigate and find out the truth concerning the legitimize E.O. No. 1 because the delegated authority of the President to structurally
reported cases of graft and corruption during the previous administration only. The intent to reorganize the Office of the President to achieve economy, simplicity, and efficiency does not
single out the previous administration is plain, patent and manifest. include the power to create an entirely new office was inexistent like the Truth Commission;

Arroyo administration is but just a member of a class, that is, a class of past administrations. The E.O illegally amended the Constitution when it made the Truth Commission and vesting
It is not a class of its own. Not to include past administrations similarly situated constitutes it the power duplicating and even exceeding those of the Office of the Ombudsman and the
arbitrariness which the equal protection clause cannot sanction. Such discriminating DOJ.
differentiation clearly reverberates to label the commission as a vehicle for vindictiveness
and selective retribution. Superficial differences do not make for a valid classification. It violates the equal protection clause

The PTC must not exclude the other past administrations. The PTC must, at least, have the ISSUE: WHETHER OR NOT the said E.O is unconstitutional.
authority to investigate all past administrations.
RULING:
The Constitution is the fundamental and paramount law of the nation to which all other laws
Yes, E.O No. 1 should be struck down as it is violative of the equal protection clause. The
must conform and in accordance with which all private rights determined and all public
Chief Executive’s power to create the Ad hoc Investigating Committee cannot be doubted.
authority administered. Laws that do not conform to the Constitution should be stricken
Having been constitutionally granted full control of the Executive Department, to which
down for being unconstitutional.
respondents belong, the President has the obligation to ensure that all executive officials and
WHEREFORE, the petitions are GRANTED. Executive Order No. 1 is hereby declared employees faithfully comply with the law. With AO 298 as mandate, the legality of the
UNCONSTITUTIONAL insofar as it is violative of the equal protection clause of the investigation is sustained. Such validity is not affected by the fact that the investigating team
Constitution. and the PCAGC had the same composition, or that the former used the offices and facilities
of the latter in conducting the inquiry.
FACT:

E.O No. 1 establishing the Philippine Truth Commission (PTC) of 2010 was signed by President
Aquino. The said PTC is a mere branch formed under the Office of the President tasked to
investigate reports of graft and corruption committed by third-level public officers and
employees, their co-principals, accomplices and accessories during the previous
administration and submit their findings and recommendations to the President, Congress
and the Ombudsman. However, PTC is not a quasi-judicial body, it cannot adjudicate,
arbitrate, resolve, settle or render awards in disputes between parties. Its job is to
investigate, collect and asses evidences gathered and make recommendations. It has
subpoena powers but it has no power to cite people in contempt or even arrest. It cannot
determine for such facts if probable cause exist as to warrant the filing of an information in
our courts of law.

Petitioners contends the Constitutionality of the E.O. on the grounds that.

It violates separation of powers as it arrogates the power of Congress to create a public office
and appropriate funds for its operation;
187

EN BANC The authority of the National Artists Awards Committee to administer the conferment of the
National Artist Award was again reiterated in Presidential Decree No. 2086 issued on June 7,
G.R. No. 189028 July 16, 2013 1973.

NATIONAL ARTIST FOR LITERATURE VIRGILIO ALMARIO, NATIONAL ARTIST FOR On April 3, 1992, Republic Act No. 7356, otherwise known as the Law Creating the National
LITERATURE BIENVENIDO LUMBERA, NATIONAL ARTIST FOR VISUAL ARTS Commission for Culture and the Arts, was signed into law. It established the National
(PAINTING) Petitioners, Commission for Culture and the Arts (NCCA) and gave it an extensive mandate over the
vs. development, promotion and preservation of the Filipino national culture and arts and the
THE EXECUTIVE SECRETARY, THE SECRETARY OF THE DEPARTMENT OF BUDGET AND Filipino cultural heritage. The NCCA was tasked with the following:
MANAGEMENT, THE CULTURAL CENTER OF THE PHILIPPINES, THE NATIONAL COMMISSION
ON CULTURE AND THE ARTS Respondents. Sec. 8. The Commission. – A National Commission for Culture and Arts is hereby created to
formulate policies for the development of culture and arts; implement these policies in
DECISION coordination with affiliated cultural agencies; coordinate the implementation of programs of
these affiliated agencies; administer the National Endowment Fund for Culture and Arts
(NEFCA); encourage artistic creation within a climate of artistic freedom; develop and
LEONARDO-DE CASTRO, J.:
promote the Filipino national culture and arts; and preserve Filipino cultural heritage. The
Commission shall be an independent agency. It shall render an annual report of its activities
Art has traditionally been viewed as the expression of everything that is true, good and and achievements to the President and to Congress.
beautiful. As such, it is perceived to evoke and produce a spirit of harmony. Art is also
considered as a civilizing force, a catalyst of nation-building. The notion of art and artists as
Among the specific mandates of the NCCA under Republic Act No. 7356 is to "extend
privileged expressions of national culture helped shape the grand narratives of the nation
recognition of artistic achievement through awards, grants and services to artists and cultural
and shared symbols of the people. The artist does not simply express his/her own individual
groups which contribute significantly to the Filipino’s cultural legacy."7 In connection with
inspiration but articulates the deeper aspirations of history and the soul of the people.2 The
this mandate, the NCCA is vested with the power to "advise the President on matters
law recognizes this role and views art as something that "reflects and shapes values, beliefs,
pertaining to culture and the arts, including the creation of a special decoration or award, for
aspirations, thereby defining a people’s national identity."3If unduly politicized, however, art
persons who have significantly contributed to the development and promotion of Philippine
and artists could stir controversy and may even cause discord, as what happened in this case.
culture and arts."8

The Antecedents
As both the CCP Board of Trustees and the NCCA have been mandated by law to promote,
develop and protect the Philippine national culture and the arts, and authorized to give
History of the Order of National Artists awards to deserving Filipino artists, the two bodies decided to team up and jointly administer
the National Artists Award.9 Thereafter, they reviewed the guidelines for the nomination,
On April 27, 1972, former President Ferdinand E. Marcos issued Proclamation No. 10014 and, selection and administration of the National Artists Award. Pursuant to their respective
upon recommendation of the Board of Trustees of the Cultural Center of the Philippines powers to draft and promulgate rules, regulations and measures to guide them in their
(CCP), created the category of Award and Decoration of National Artist to be awarded to deliberations in the choice of National Artists, the CCP and NCCA adopted the following
Filipinos who have made distinct contributions to arts and letters. In the same issuance, revised guidelines in September 200710:
Fernando Amorsolo was declared as the first National Artist.
4. ADMINISTRATION OF THE AWARD
On May 15, 1973, Proclamation No. 11445 was issued. It amended Proclamation No. 1001 "by
creating a National Artists Awards Committee" that would "administer the conferment of the 4.1. The National Commission for Culture and the Arts (NCCA) shall plan, organize
category of National Artist" upon deserving Filipino artists. The Committee, composed of and implement the Order of National Artists in coordination with the Cultural
members of the Board of Trustees of the CCP, was tasked to "draft the rules to guide its Center of the Philippines (CCP).
deliberations in the choice of National Artists, to the end that those who have created a body
of work in the arts and letters capable of withstanding the test of time will be so recognized."
188

4.2. It shall enlist the support and cooperation of private sector experts from the 4.7. The National Artist Award Council of Experts shall be composed of a maximum
various fields of art to ensure that the awards are implemented in a successful and of seven (7) members each of the seven (7) areas/disciplines. The living National
impartial manner. Artists will automatically become members in addition to the forty-nine (49)
selected members. These members will constitute the first deliberation panel and
4.3. The National Artist Award Secretariat shall commission art experts to form a will be invited to evaluate the nominations and materials submitted by the Special
Special Research Group who shall verify information submitted on nominees and Research Group.
provide essential data.
4.8. Any member of the Council of Experts who is nominated or related to a
They shall be selected for their specialization and familiarity with the works and nominee up to the fourth degree of consanguinity or affinity shall inhibit
accomplishments of nominated artists. himself/herself from the deliberation process. Likewise, any member may decline
to participate in the deliberation for any reason or may be removed for just cause
upon recommendation to the NCCA Board by at least two thirds (2/3) of the
4.4. The Special Research Group shall be composed of ten (10) to twenty (20)
members; in which case, the National Artist Award Secretariat shall again select the
members who have expertise in one or more fields or disciplines.
replacements for those who decline or resigned until the first deliberation panel is
completed.
4.5. The National Artist Award Council of Experts shall be created before or during
the nomination period. It is tasked to screen nominees and recommend to the
4.9. The list of nominated members of the National Artist Award Council of Experts
NCCA and CCP Boards the candidates for the Order of National Artists. It shall be
shall be reviewed by the National Artist Award Secretariat as needed, for purposes
composed of highly regarded peers, scholars, (including cultural philosophers and
of adding new members or replacements.
historians), academicians, researchers, art critics, and other knowledgeable
individuals. A wider age-range of experts who would have first-hand knowledge of
achievements of nominees shall be considered. 4.10. The members of the National Artist Award Council of Experts shall serve for a
fixed term of three (3) years.
4.6. The selection of the members of the National Artist Award Council of Experts
shall be based on the following criteria: 5. CRITERIA FOR SELECTION

(a) should have achieved authority, credibility and track record in his The Order of National Artists shall be given to:
field(s) of expertise;
5.1 Living artists who are Filipino citizens at the time of nomination, as well as
(b) should have extensive knowledge in his field(s) and his views on those who died after the establishment of the award in 1972 but were Filipino
Philippine art and culture must be national in perspective; citizens at the time of their death.

(c) should be a recognized authority in the study or research of Philippine 5.2 Artists who through the content and form of their works have contributed in
art and culture; building a Filipino sense of nationhood.

(d) must be willing to devote sufficient time and effort to the work of the 5.3. Artists who have pioneered in a mode of creative expression or style, thus,
Council; earning distinction and making an impact on succeeding generations of artists.

(e) must be willing to sign a non-disclosure statement in order to 5.4. Artists who have created a substantial and significant body of works and/or
safeguard the confidentiality of the deliberations; consistently displayed excellence in the practice of their art form thus enriching
artistic expression or style.
(f) must not have been convicted with finality of any crime by a court of
justice or dismissed for cause by any organization, whether public or 5.5 Artists who enjoy broad acceptance through:
private.
189

5.5.1. prestigious national and/or international recognition, 6.5. NCCA and CCP Board members and consultants and NCCA and CCP officers and
such as the Gawad CCP Para sa Sining, CCP Thirteen Artists staff are automatically disqualified from being nominated.
Award and NCCA Alab ng Haraya
6.6. Nominations shall be accepted only when these are submitted in writing and
5.5.2. critical acclaim and/or reviews of their works with proper supporting documentation, as follows:

5.5.3. respect and esteem from peers. 6.6.1. A cover letter signed by the head or designated representative of
the nominating organization.
6. NOMINATION PROCEDURE
The cover letter shall be accompanied by a Board Resolution approving the nominee
6.1. The National Artist Award Secretariat shall announce the opening of concerned with the said resolution signed by the organization President and duly certified by
nominations through media releases and letters to qualified organizations. the Board Secretary.

6.2. Candidates may be nominated under one or more of the following categories: 6.6.2. A duly accomplished nomination form;

6.2.1. Dance – choreography, direction and/or performance. 6.6.3. A detailed curriculum vitae of the nominee;

6.2.2. Music – composition, direction, and/or performance. 6.6.4. A list of the nominee’s significant works categorized according to
the criteria;
6.2.3. Theater – direction, performance and/or production design.
6.6.5. The latest photograph (color or black and white) of the nominee,
either 5" x 7" or 8" x 11";
6.2.4. Visual Arts – painting, sculpture, printmaking, photography, installation art,
mixed media works, illustration, comics/komiks, graphic arts, performance art
and/or imaging. 6.6.6. Pertinent information materials on the nominee’s significant works
(on CDs, VCDs and DVDs);
6.2.5. Literature – poetry, fiction (short story, novel and play); non-fiction (essay,
journalism, literary criticism and historical literature). 6.6.7. Copies of published reviews;

6.2.6. Film and Broadcast Arts – direction, writing, production design, 6.6.8. Any other document that may be required.
cinematography, editing, camera work, and/or performance.
6.7. Nominations received beyond the announced deadline for the submission of
6.2.7. Architecture, Design and Allied Arts – architecture design, interior design, nominations shall not be considered.
industrial arts design, landscape architecture and fashion design.
6.8. The National Artist Award Secretariat shall announce the opening of
6.3. Nominations for the Order of National Artists may be submitted by nominations through media releases.
government and non-government cultural organizations and educational
institutions, as well as private foundations and councils. 6.9. All inquiries and nominations shall be submitted to

6.4. Members of the Special Research Group, as well as agencies attached to the The NATIONAL ARTIST AWARD SECRETARIAT
NCCA and CCP shall not submit nominations.
190

Office of the Artistic Director Cultural Center of the Philippines Roxas Boulevard, 1300 Pasay across disciplines. The National Artists will be given the option whether to vote on
City or The NATIONAL ARTIST AWARD SECRETARIAT Office of the Deputy Executive Director all categories or on his/her particular discipline.
National Commission for Culture and the Arts 633 General Luna Street, Intramuros, Manila
7.8. Proxy votes will not be allowed in the Selection Process. Designation of
7. SCREENING AND SELECTION PROCESS permanent representatives of agencies should be made at the outset to make
them regular Board members of NCCA and thus, may be allowed to cast votes.
7.1. The National Artist Award Secretariat shall pre-screen the nominees based on
technical guideline items 5.1, 6.2, 6.3, 6.4, 6.5 and 6.6. The pre-screening shall not 7.9. The list of awardees shall be submitted to the President of the Republic of the
be based on the accomplishments and merits of the nominee. Philippines for confirmation, proclamation and conferral.

7.2. The Special Research Group shall accomplish its task within six (6) months. The 8. PRESENTATION OF THE AWARDS
main objective is to verify the validity of the data, and evaluate the quality, true
value and significance of works according to the criteria. It shall come up with the 8.1. The Order of National Artists shall not be conferred more frequently than
updated and comprehensive profiles of nominees reflecting their most outstanding every three (3) years.
achievements.
8.2. The Order of National Artists shall be conferred by the President of the
7.3. The National Artist Award Secretariat will meet to review the list of nominees Philippines on June 11 or any appropriate date in fitting ceremonies to be
for oversights. Consequently, deserving nominees shall be added to the list. organized by the National Artist Secretariat.

7.4. The first deliberation panel (Council of Experts) shall be intra-disciplinary. The 8.3. The medallion of the Order of National Artists and citation shall be given to the
panelists shall be grouped according to their respective fields of expertise or honoree during the conferment ceremony. The cash award of ₱100,000.00 in
disciplines to shortlist the nominees in their disciplines or categories for cheque shall be given immediately after the ceremony or at another time and place
presentation to the second deliberation panel. as requested by the honoree.

7.5. The second deliberation panel shall be composed of a different set of experts 8.4. A posthumous conferral consisting of the medallion and citation shall be given
from the first deliberation panel three (3) experts each of the seven (7) to the family or legal heir/s of the honoree. The cash award of ₱75,000.00 in
areas/discipline and may include members from varying backgrounds such as critics cheque shall be given to the honoree’s legal heir/s or a representative designated
and academicians. The achievements of each shortlisted nominee shall be by the family immediately after the ceremony or at another time and place as
presented by one designated member of Council of Experts. Then panel deliberates requested by the family. (Emphases supplied.)
and ranks the shortlisted nominees according to the order of precedence following
the set criteria of the Order of National Artists. In extreme cases, the Second
In 1996, the NCCA and the CCP created a National Artist Award Secretariat composed of the
Deliberation may add new names to the lists.
NCCA Executive Director as Chairperson, the CCP President as Vice-Chairperson, and the
NCCA Deputy Executive Director, the CCP Vice-President/Artistic Director, the NCCA National
7.6. The second deliberation panel may recommend not to give award in any Artist Award Officer and the CCP National Artist Award Officer as members. They also
category if no nominee is found deserving. The number of awardees shall also centralized with the NCCA all financial resources and management for the administration of
depend on the availability of funds. All decisions and recommendations shall be in the National Artists Award. They added another layer to the selection process to involve and
writing. allow the participation of more members of the arts and culture sector of the Philippines in
the selection of who may be proclaimed a National Artist.
7.7. The recommendations from the Second Deliberation Panel of the National
Artist Award Council of Experts shall then be presented to the joint boards of NCCA On September 19, 2003, Executive Order No. 236, s. 2003, entitled Establishing the Honors
and CCP for final selection. The presentors shall prepare their presentation in Code of the Philippines to Create an Order of Precedence of Honors Conferred and for Other
writing together with an audio-visual presentation or powerpoint presentation. Purposes, was issued. The National Artists Award was renamed the Order of National Artists
Written interpellations/opinions will be accepted from selected critics. The review and raised to the level of a Cultural Order, fourth in precedence among the orders and
shall be based on the ranking done by the Second Deliberation. The voting shall be decorations that comprise the Honors of the Philippines. Executive Order No. 236, s. 2003,
191

recognizes the vital role of the NCCA and the CCP in identifying Filipinos who have made The President of the Philippines takes the recommendations of the Committee on Honors in
distinct contributions to arts and letters and states that the National Artist recognition is the highest consideration when making the final decision on the conferment of awards.
conferred "upon the recommendation of the Cultural Center of the Philippines and the (Emphasis supplied.)
National Commission for Culture and the Arts."12 Executive Order No. 236, s. 2003, further
created a Committee on Honors to "assist the President in evaluating nominations for Executive Order No. 435, s. 2005, entitled Amending Section 5(IV) of Executive Order No. 236
recipients of Honors,"13 including the Order of National Artists, and presidential awards. The Entitled "Establishing the Honors Code of the Philippines to Create an Order of Precedence of
Committee on Honors has been allowed to "authorize relevant department or government Honors Conferred and for Other Purposes" was subsequently issued on June 8, 2005. It
agencies to maintain Honors and/or Awards Committees to process nominations for Honors amended the wording of Executive Order No. 236, s. 2003, on the Order of National Artists
and/or Presidential Awards."14 In this connection, Section 2.4(A) of the Implementing Rules and clarified that the NCCA and the CCP "shall advise the President on the conferment of the
and Regulations15 of Executive Order No. 236, s. 2003, states: Order of National Artists."

2.4: Awards Committees Controversy Surrounding the 2009


Order of National Artists
There shall be two types of awards committees: the Committee on Honors and the various
awards committees in the various units of the government service. Petitioners alleged that on January 30, 2007, a joint meeting of the NCCA Board of
Commissioners and the CCP Board of Trustees was held to discuss, among others, the
A. The Committee on Honors evaluation of the 2009 Order of National Artists and the convening of the National Artist
Award Secretariat. The nomination period was set for September 2007 to December 31,
The Committee on Honors serves as a National Awards Committee. It is composed of the 2007, which was later extended to February 28, 2008. The pre-screening of nominations was
following: held from January to March 2008.16

The Executive Secretary, Chairman On April 3, 2009, the First Deliberation Panel met.17 A total of 87 nominees18 were
considered during the deliberation and a preliminary shortlist19 of 32 names was compiled.
The Secretary of Foreign Affairs, Vice-Chairman
On April 23, 2009, the Second Deliberation Panel purportedly composed of an entirely new
set of Council of Experts met and shortlisted 13 out of the 32 names in the preliminary
Head, Presidential Management Staff, member
shortlist.20 On May 6, 2009, the final deliberation was conducted by the 30-member Final
Deliberation Panel comprised of the CCP Board of Trustees and the NCCA Board of
Presidential Assistant for Historical Affairs, member Commissioners and the living National Artists.21 From the 13 names in the second shortlist, a
final list of four names was agreed upon.22 The final list, according to rank, follows:
Chief of Presidential Protocol, member
Name Art Field/Category Number of Votes
Chief of Protocol, DFA, member
Manuel Conde (+) Film and Broadcast Arts (Film) 26
All nominations from the various awards committees must be submitted to the Committee Ramon Santos Music 19
on Honors via the Chancellery of Philippine Orders and State Decorations. The Chancellery
shall process nominations for the consideration of the Committee on Honors. The Committee Lazaro Francisco (+) Literature 15
on Honors shall screen and recommend these nominations to the President.
Federico Aguilar-Alcuaz Visual Arts 15
The Committee on Honors shall, as a general rule, serve as a screening committee to ensure
that nominations received from the various awards committees meet two tests: that there On May 6, 2009, a letter, signed jointly by the Chairperson of the NCCA, Undersecretary
has not been an abuse of discretion in making the nomination, and that the nominee is in Vilma Labrador, and the President and Artistic Director of the CCP, Mr. Nestor Jardin, was
good standing. Should a nomination meet these criteria, a recommendation to the President sent to the President.23 The letter stated, thus:
for conferment shall be made.
192

May 6, 2009 (Sgd.)


NESTOR O. JARDIN
Her Excellency GLORIA MACAPAGAL-ARROYO President and Artistic Director
President of the Philippines Cultural Center of the Philippines24
Malacañan Palace, Manila
According to respondents, the aforementioned letter was referred by the Office of the
Subject: 2009 Order of National Artist Awardees President to the Committee on Honors. Meanwhile, the Office of the President allegedly
received nominations from various sectors, cultural groups and individuals strongly endorsing
private respondents Cecile Guidote-Alvarez, Carlo Magno Jose Caparas, Francisco Mañosa
Dear President Arroyo:
and Jose Moreno. The Committee on Honors purportedly processed these nominations and
invited resource persons to validate the qualifications and credentials of the nominees. 25
We are respectfully submitting a recommendation of the NCCA Board of Trustees and CCP
Board of Trustees for the Proclamation of the following as 2009 Order of National Artists:
The Committee on Honors thereafter submitted a memorandum to then President Gloria
Macapagal-Arroyo recommending the conferment of the Order of National Artists on the
1. Mr. MANUEL CONDE+ (Posthumous) – Film and Broadcast Arts four recommendees of the NCCA and the CCP Boards, as well as on private respondents
Guidote-Alvarez, Caparas, Mañosa and Moreno. Acting on this recommendation,
2. Dr. RAMON SANTOS – Music Proclamation No. 1823 declaring Manuel Conde a National Artist was issued on June 30,
2009. Subsequently, on July 6, 2009, Proclamation Nos. 1824 to 1829 were issued declaring
3. Mr. LAZARO FRANCISCO+ (Posthumous) – Literature Lazaro Francisco, Federico AguilarAlcuaz and private respondents Guidote-Alvarez, Caparas,
Mañosa and Moreno, respectively, as National Artists. This was subsequently announced to
the public by then Executive Secretary Eduardo Ermita on July 29, 2009.26
4. Mr. FEDERICO AGUILAR-ALCUAZ – Visual Arts
Convinced that, by law, it is the exclusive province of the NCCA Board of Commissioners and
The above persons were identified by experts in the various fields of arts and culture, the CCP Board of Trustees to select those who will be conferred the Order of National Artists
including living National Artists. An intensive selection process was observed following and to set the standard for entry into that select group, petitioners instituted this petition for
established practice. In the past, awards were presented by the President at a Ceremony prohibition, certiorari and injunction (with prayer for restraining order) praying that the
held at the Malacañan Palace followed by a program called "Parangal" at the Cultural Center Order of National Artists be conferred on Dr. Santos and that the conferment of the Order of
of the Philippines. We also propose to continue with past practice of celebrating the life and National Artists on respondents Guidote-Alvarez, Caparas, Mañosa and Moreno be enjoined
works of the four (4) Order of National Artists through an exhibit that will open and a and declared to have been rendered in grave abuse of discretion.27
commemorative publication that will be released on the day of the proclamation.
In a Resolution28 dated August 25, 2009, the Court issued a status quo order29 enjoining
We respectfully suggest, subject to Her Excellency’s availability, that the Proclamation be on "public respondents" "from conferring the rank and title of the Order of National Artists on
June 11, 2009, if possible at the Malacañan Palace. private respondents; from releasing the cash awards that accompany such conferment and
recognition; and from holding the acknowledgment ceremonies for recognition of the private
Thank you for your kind attention. respondents as National Artists."

Very respectfully yours, What is the nature and scope of the power of the President to confer the Order of the
National Artists and how should it be exercised? This is the essential issue presented in this
(Sgd.) case. It will determine whether the proclamation of respondents as National Artists is valid.
VILMA L. LABRADOR Preliminary procedural issues on the standing of the petitioners and the propriety of the
Chairman remedies taken,30 however, call for resolution as a prerequisite to the discussion of the main
National Commission for Culture and the Arts question.

Contention of the Parties


193

A perusal of the pleadings submitted by the petitioners reveals that they are an aggrupation Alvarez. This was clearly grave abuse of discretion for being manifest and undue bias violative
of at least three groups, the National Artists, cultural workers and academics, and the of the equal protection clause.36
Concerned Artists of the Philippines (CAP). The National Artists assert an "actual as well as
legal interest in maintaining the reputation of the Order of National Artists."31 In particular, Respondent Caparas refutes the contention of the petitioning National Artists and insists that
they invoke their right to due process not to have the honor they have been conferred with there could be no prejudice to the latter. They remain to be National Artists and continue to
diminished by the irregular and questionable conferment of the award on respondents receive the emoluments, benefits and other privileges pertaining to them by virtue of that
Guidote-Alvarez, Caparas, Mañosa and Moreno. For petitioners, this would adversely affect honor. On the other hand, all the other petitioners failed to show any material and personal
their right to live a meaningful life as it detracts not only from their right to enjoy their honor injury or harm caused to them by the conferment of the Order of National Artists on
as a fruit of their lifelong labor but also from the respect of their peers.32 respondents Guidote-Alvarez, Caparas, Mañosa and Moreno. The rule on standing may not
be relaxed in favor of the petitioners as no question of constitutionality has been raised and
The cultural workers, academics and CAP claim to be Filipinos who are deeply concerned no issue of transcendental importance is involved.37
with the preservation of the country’s rich cultural and artistic heritage. As taxpayers, they
are concerned about the use of public monies for illegal appointments or spurious acts of Respondent Caparas further argues that the remedies of prohibition and injunction are
discretion.33 improper as the act sought to be enjoined – the declaration of respondents Guidote-Alvarez,
Caparas, Mañosa and Moreno as National Artists – had already been consummated. In
All of the petitioners claim that former President Macapagal-Arroyo gravely abused her particular, respondent Caparas was already proclaimed National Artist through Proclamation
discretion in disregarding the results of the rigorous screening and selection process for the No. 1827 issued on July 6, 2009.38
Order of National Artists and in substituting her own choice for those of the Deliberation
Panels. According to petitioners, the President’s discretion to name National Artists is not On the merits, respondent Caparas contends that no grave abuse of discretion attended his
absolute but limited. In particular, her discretion on the matter cannot be exercised in the proclamation as National Artist. The former President considered the respective
absence of or against the recommendation of the NCCA and the CCP. In adding the names of recommendations of the NCCA and the CCP Boards and of the Committee on Honors in
respondents Caparas, Guidote-Alvarez, Mañosa and Moreno while dropping Dr. Santos from eventually declaring him (Caparas) as National Artist. The function of the NCCA and the CCP
the list of conferees, the President’s own choices constituted the majority of the awardees in Boards is simply to advise the President. The award of the Order of National Artists is the
utter disregard of the choices of the NCCA and the CCP and the arts and culture community exclusive prerogative of the President who is not bound in any way by the recommendation
which were arrived at after a long and rigorous process of screening and deliberation. of the NCCA and the CCP Boards. The implementing rules and regulations or guidelines of the
Moreover, the name of Dr. Santos as National Artist for Music was deleted from the final list NCCA cannot restrict or limit the exclusive power of the President to select the recipients of
submitted by the NCCA and the CCP Boards without clearly indicating the basis thereof. For the Order of National Artists.39
petitioners, the President’s discretion to name National Artists cannot be exercised to defeat
the recommendations made by the CCP and NCCA Boards after a long and rigorous screening
For her part, in a letter40 dated March 11, 2010, respondent Guidote-Alvarez manifested that
process and with the benefit of expertise and experience. The addition of four names to the
she was waiving her right to file her comment on the petition and submitted herself to the
final list submitted by the Boards of the CCP and the NCCA and the deletion of one name
Court’s discretion and wisdom.
from the said list constituted a substitution of judgment by the President and a unilateral
reconsideration without clear justification of the decision of the First, Second and Final
Deliberation Panels composed of experts.34 Respondent Mañosa manifested that his creations speak for themselves as his contribution
to Filipino cultural heritage and his worthiness to receive the award. Nonetheless, he
expressed his conviction that the Order of National Artists is not a right but a privilege that
Petitioners further argue that the choice of respondent GuidoteAlvarez was illegal and
he would willingly relinquish should he be found not worthy of it.41
unethical because, as the then Executive Director of the NCCA and presidential adviser on
culture and arts, she was disqualified from even being nominated.35 Moreover, such action
on the part of the former President constituted grave abuse of discretion as it gave Respondent Moreno did not file any pleading despite being given several opportunities to do
preferential treatment to respondent Guidote-Alvarez by naming the latter a National Artist so. Hence, the Court dispensed with his pleadings.42
despite her not having been nominated and, thus, not subjected to the screening process
provided by the rules for selection to the Order of National Artists. Her inclusion in the list by In a Resolution dated July 12, 2011, this Court gave due course to the petition and required
the President represented a clear and manifest favor given by the President in that she was the parties to file their respective memoranda.43 Respondent Caparas filed his memorandum
exempted from the process that all other artists have to undergo. According to petitioners, it on September 8, 2011,44 the CCP filed its memorandum on September 19,
may be said that the President used a different procedure to qualify respondent Guidote- 2011,45 respondent Mañosa on September 20, 2011,46 and the Office of the Solicitor General
194

filed a manifestation stating that it is adopting its comment as its memorandum on adverseness which sharpens the presentation of issues upon which the court depends for
September 21, 2011.47 Respondent Moreno failed to file a Memorandum, hence, the Court illumination of difficult constitutional questions.57
resolved to dispense with the same.48Petitioners filed their Memorandum on May 14, 2012.49
The parties who assail the constitutionality or legality of a statute or an official act must have
On the other hand, the original position of the Office of the Solicitor General (OSG) was a direct and personal interest. They must show not only that the law or any governmental act
similar to that of respondent Caparas.50 In a subsequent manifestation,51 however, the OSG is invalid, but also that they sustained or are in immediate danger of sustaining some direct
stated that the current Board of Commissioners of the NCCA agree with the petitioners that injury as a result of its enforcement, and not merely that they suffer thereby in some
the President cannot honor as a National Artist one who was not recommended by the joint indefinite way. They must show that they have been or are about to be denied some right or
Boards of the NCCA and the CCP. The implementing rules and regulations of Executive Order privilege to which they are lawfully entitled or that they are about to be subjected to some
No. 236, s. 2003, recognized the binding character of the recommendation of the NCCA and burdens or penalties by reason of the statute or act complained of.58
the CCP Boards and limited the authority of the Committee on Honors to the determination
that (1) there has been no grave abuse of discretion on the part of the NCCA and the CCP In this case, we find that the petitioning National Artists will be denied some right or privilege
Boards in making the nomination, and (2) the nominee is in good standing. Where a to which they are entitled as members of the Order of National Artists as a result of the
nomination meets the said two criteria, a recommendation to the President to confer the conferment of the award on respondents Guidote-Alvarez, Caparas, Mañosa and Moreno. In
award shall be made.52 particular, they will be denied the privilege of exclusive membership in the Order of National
Artists.
The OSG further argued that, while the President exercises control over the NCCA and the
CCP, the President has the duty to faithfully execute the laws, including the NCCA-CCP In accordance with Section 2(a)59 of Executive Order No. 236, s. 2003, the Order of National
guidelines for selection of National Artists and the implementing rules of Executive Order No. Artists is "an exclusive association of honored individuals." To ensure the exclusivity of the
236, s. 2003. Moreover, the laws recognize the expertise of the NCCA and the CCP in the arts membership in the Order, a rigid nomination and screening process has been established
and tasked them to screen and select the artists to be conferred the Order of National with different sets of renowned artists and respected art critics invited to sit as the Council of
Artists. Their mandate is clear and exclusive as no other agency possesses such expertise.53 Experts for the First and Second Deliberation Panels. Moreover, all living National Artists are
given a voice on who should be included in their exclusive club as they automatically become
The OSG also assailed the former President’s choice of respondent Guidote-Alvarez for being members of the Final Deliberation Panel that will vote on who should be included in the final
contrary to Republic Act No. 7356.54 Section 11 of the said law provides: list to be submitted to the President for conferment of the Order of National Artists. To allow
the untrammeled discretion and authority of the President to confer the Order of National
Sec. 11. Membership Restrictions. – During his/her term as member of the Commission, a Artists without regard to the stringent screening and rigorous selection process established
Commissioner shall not be eligible for any grant, or such other financial aid from the by the NCCA and the CCP will diminish, if not negate, the exclusive nature of the said Order.
Commission as an individual: Provided, however, That he/she may compete for grants and It will unduly subject the selection and conferment of the Order of National Artists to politics
awards on the same level as other artists one (1) year after his/her term shall have expired. rather than to principles and procedures. It will subvert the transparent and rigorous process
and allow entry to the exclusive Order of National Artists through a secret backdoor of
lobbying, back channeling and political accommodation.
The omission of the word "award" in the first portion of the above provision appears to be
unintentional as shown by the proviso which states that a member may compete for grants
and awards only one year after his or her term shall have expired. As such, respondent Among the other petitioners, Prof. Gemino Abad presents a unique valid personal and
Guidote-Alvarez is restricted and disqualified from being conferred the 2009 Order of substantial interest. Like respondents Caparas, Mañosa and Moreno, he was among the 87
National Artists.55 nominees for the 2009 Order of National Artists. Like respondent Moreno, he made it to the
preliminary shortlist. As he did not make it to the second shortlist, he was not considered by
the Final Deliberation Panel, more so by the former President.
The Court’s Ruling

It should be recalled too that respondent Guidote-Alvarez was disqualified to be nominated


Standing of the Petitioners
for being the Executive Director of the NCCA at that time while respondents Mañosa and
Caparas did not make it to the preliminary shortlist and respondent Moreno was not
Standing is the determination of whether a specific person is the proper party to bring a included in the second shortlist. Yet, the four of them were treated differently and
matter to the court for adjudication.56 The gist of the question of standing is whether a party considered favorably when they were exempted from the rigorous screening process of the
alleges such personal stake in the outcome of the controversy as to assure that concrete NCCA and the CCP and conferred the Order of National Artists. The Committee on Honors
195

and the former President effectively treated respondents Guidote-Alvarez, Caparas, Mañosa this is not the first time that the Order of National Artists was conferred in the manner that is
and Moreno as a preferred class. The special treatment accorded to respondents Guidote- being assailed in this case.72 If not addressed here and now, there is great probability that the
Alvarez, Caparas, Mañosa and Moreno fails to pass rational scrutiny.60 No real and central question involved in this case will haunt us again in the future. Every President may
substantial distinction between respondents and petitioner Abad has been shown that would invoke absolute presidential prerogative and thrust upon us National Artists after his or her
justify deviating from the laws, guidelines and established procedures, and placing own heart, in total disregard of the advise of the CCP and the NCCA and the voice of the
respondents in an exceptional position. The undue classification was not germane to the community of artists, resulting to repeated episodes of indignation and uproar from the
purpose of the law. Instead, it contradicted the law and well-established guidelines, rules and artists and the public.
regulations meant to carry the law into effect. While petitioner Abad cannot claim
entitlement to the Order of National Artists,61 he is entitled to be given an equal opportunity Furthermore, if not corrected, such an act would give rise to mischief and dangerous
to vie for that honor. In view of the foregoing, there was a violation of petitioner Abad’s right precedent whereby those in the corridors of power could avoid judicial intervention and
to equal protection, an interest that is substantial enough to confer him standing in this case. review by merely speedily and stealthily completing the commission of an illegality. 73

As regards the other concerned artists and academics as well as the CAP, their claim of deep In any event, the present petition is also for certiorari and there is no procedural bar for the
concern for the preservation of the country’s rich cultural and artistic heritage, while Court to pass upon the question of whether the proclamations of respondents Guidote-
laudable, falls short of the injury in fact requirement of standing. Their assertion constitutes a Alvarez, Caparas, Mañosa and Moreno as National Artists were attended by grave abuse of
generalized grievance shared in a substantially equal measure by all or a large class of presidential discretion.
citizens.62 Nor can they take refuge in their status as taxpayers as the case does not involve
any illegal appropriation or taxation. A taxpayer’s suit is proper only when there is an
Limits of the President’s Discretion
exercise of the spending or taxing power of the Congress.63

The respective powers of the CCP Board of Trustees and of the NCCA Board of
Nonetheless, as a reading of the petition shows that it has advanced an issue which deserves
Commissioners with respect to the conferment of the Order of National Artists are clear.
the attention of this Court in view of its seriousness, novelty and weight as precedent, it
They jointly administer the said award and, upon their recommendation or advice, the
behooves the Court to relax the rules on standing and to resolve the issue presented before
President confers the Order of National Artists.
it.64 Moreover, this issue is of paramount interest,65 which further justifies a liberal stance on
standing.
To "recommend" and to "advise" are synonymous. To "recommend" is "to advise or
counsel."74 To "advise" is "to give an opinion or counsel, or recommend a plan or course of
Propriety of the Remedies
action; also to give notice. To encourage, inform or acquaint." 75 "Advise" imports that it is
discretionary or optional with the person addressed whether he will act on such advice or
The present action is a petition for prohibition, certiorari, injunction, restraining order and all not.76 This has been clearly explained in Cojuangco, Jr. v. Atty. Palma77:
other legal, just and equitable reliefs.
The "power to recommend" includes the power to give "advice, exhortation or indorsement,
It has been held that the remedies of prohibition and injunction are preventive and, as such, which is essentially persuasive in character, not binding upon the party to whom it is made."
cannot be availed of to restrain an act that is already fait accompli.66 Where the act sought to (Emphasis supplied.)
be prohibited or enjoined has already been accomplished or consummated, prohibition or
injunction becomes moot.67
Thus, in the matter of the conferment of the Order of National Artists, the President may or
may not adopt the recommendation or advice of the NCCA and the CCP Boards. In other
Nevertheless, even if the principal issue is already moot, this Court may still resolve its merits words, the advice of the NCCA and the CCP is subject to the President’s discretion.
for the future guidance of both bench and bar. Courts will decide a question otherwise moot
and academic if it is "capable of repetition, yet evading review."68
Nevertheless, the President’s discretion on the matter is not totally unfettered, nor the role
of the NCCA and the CCP Boards meaningless.
It is an opportune time for the Court to assert its role as republican schoolmaster,69 a teacher
in a vital national seminar.70 There are times when the controversy is of such character that,
Discretion is not a free-spirited stallion that runs and roams wherever it pleases but is reined
to prevent its recurrence and to assure respect for constitutional limitations, this Court must
in to keep it from straying. In its classic formulation, "discretion is not unconfined and
pass on the merits of a case.71 This is one such case. More than being a teaching moment,
vagrant" but "canalized within banks that keep it from overflowing."78
196

The President’s power must be exercised in accordance with existing laws. Section 17, Article For this purpose, these agencies shall submit periodic reports, including recommendations to
VII of the Constitution prescribes faithful execution of the laws by the President: the [NCCA]. (Emphasis supplied.)

Sec. 17. The President shall have control of all the executive departments, bureaus and On the other hand, the NCCA has been given the following mandate in connection with the
offices. He shall ensure that the laws be faithfully executed. (Emphasis supplied.) conferment of cultural or arts awards:

The President’s discretion in the conferment of the Order of National Artists should be Sec. 12. Mandate. – The Commission is hereby mandated to formulate and implement
exercised in accordance with the duty to faithfully execute the relevant laws. The faithful policies and plans in accordance with the principles stated in Title 1 of this Act.
execution clause is best construed as an obligation imposed on the President, not a separate
grant of power.79 It simply underscores the rule of law and, corollarily, the cardinal principle (a) To encourage the continuing and balanced development of a pluralistic culture
that the President is not above the laws but is obliged to obey and execute them.80 This is by the people themselves, it shall:
precisely why the law provides that "administrative or executive acts, orders and regulations
shall be valid only when they are not contrary to the laws or the Constitution."81
(4) extend recognition of artistic achievement through awards, grants and services
to artists and cultural groups which contribute significantly to the Filipino’s cultural
In this connection, the powers granted to the NCCA and the CCP Boards in connection with legacy;
the conferment of the Order of National Artists by executive issuances were institutionalized
by two laws, namely, Presidential Decree No. 208 dated June 7, 1973 and Republic Act No.
Sec. 13. Powers and Functions. – To carry out its mandate, the Commission shall
7356. In particular, Proclamation No. 1144 dated May 15, 1973 constituted the CCP Board as
exercise the following powers and functions:
the National Artists Awards Committee and tasked it to "administer the conferment of the
category of National Artist" upon deserving Filipino artists with the mandate to "draft the
rules to guide its deliberations in the choice of National Artists": (j) advise the President on matters pertaining to culture and the arts, including the
creation of a special decoration or award, for persons who have significantly
contributed to the development and promotion of Philippine culture and arts;
Proclamation No. 1001 dated April 27, 1972, creating the Award and Decoration of National
Artist, is hereby amended by creating a National Artists Awards Committee, hereinafter to
administer the conferment of the category of National Artist upon those deserving thereof. (k) promulgate rules, regulations and undertake any and all measures as may be
The Committee, which shall be composed of members of the Board of Trustees of the necessary to implement this Act. (Emphases supplied.)
Cultural Center of the Philippines, shall organize itself immediately and shall draft the rules to
guide its deliberations in the choice of National Artists, to the end that those who have By virtue of their respective statutory mandates in connection with the conferment of the
created a body of work in the arts and in letters capable of withstanding the test of time will National Artist Award, the NCCA and the CCP decided to work together and jointly administer
be so recognized. (Emphases supplied.) the National Artist Award. They reviewed the guidelines for the nomination, selection and
administration of the National Artist Award, created a National Artist Award Secretariat,
The authority of the CCP Board of Trustees as National Artists Awards Committee was centralized all financial resources and management for the administration of the National
reiterated in Presidential Decree No. 208 dated June 7, 1973. Artist Award, and added another layer to the selection process so that more members of the
arts and culture sector of the Philippines may be involved and participate in the selection of
National Artists.
The function of the CCP Board of Trustees as National Artists Awards Committee has been
recognized under Republic Act No. 7356:
We have held that an administrative regulation adopted pursuant to law has the force and
effect of law.82 Thus, the rules, guidelines and policies regarding the Order of National Artists
Sec. 18. The National Cultural Agencies. – The [NCCA] shall coordinate with the national
jointly issued by the CCP Board of Trustees and the NCCA pursuant to their respective
cultural agencies including but not limited to the Cultural Center of the Philippines, the
statutory mandates have the force and effect of law. Until set aside, they are binding upon
Institute of Philippine Languages, the National Historical Institute, the National Library, the
executive and administrative agencies,83 including the President himself/herself as chief
National Museum, the Records Management and Archives Office. However, they shall
executor of laws. In this connection, Section 2.5(A) of the Implementing Rules and
continue operating under their respective charters or as provided by law where provisions
Regulations84 of Executive Order No. 236, s. 2003 provides:
therein are not inconsistent with the provisions of this Act. They shall serve as the national
repository and/or showcase, as the case may be, of the best of Philippine culture and arts.
197

2.5: General Guidelines for Awards Committees The President of the Philippines takes the recommendations of the Committee on Honors in
the highest consideration when making the final decision on the conferment of awards.
A. National Orders of Cultural and Scientific Merit (Emphasis supplied.)

The existing modalities of the NCCA for selecting recipients for the Order of National Artists, Pursuant to the above provision of the implementing rules of Executive Order No. 236, s.
and the Gawad sa Manlilikha ng Bayan, and of the NAST for selecting recipients of the Order 2003, the authority of the Committee on Honors is limited to determining whether the
of National Scientists, shall remain in force. (Emphases supplied.) nominations submitted by a particular awards committee, in this case, the joint NCCA and
CCP Boards, have been tainted by abuse of discretion, and whether the nominees are in good
standing. Should the nominations meet these two criteria, the Committee on Honors shall
Section 2.4(A) of the same implementing rules further states:
make a recommendation to the President for conferment of the Order of National Artists.

2.4: Awards Committees


In view of the various stages of deliberation in the selection process and as a consequence of
his/her duty to faithfully enforce the relevant laws, the discretion of the President in the
There shall be two types of awards committees: the Committee on Honors and the various matter of the Order of National Artists is confined to the names submitted to him/her by the
awards committees in the various units of the government service. NCCA and the CCP Boards. This means that the President could not have considered
conferment of the Order of National Artists on any person not considered and recommended
A. The Committee on Honors by the NCCA and the CCP Boards. That is the proper import of the provision of Executive
Order No. 435, s. 2005, that the NCCA and the CCP "shall advise the President on the
The Committee on Honors serves as a National Awards Committee. It is composed of the conferment of the Order of National Artists." Applying this to the instant case, the former
following: President could not have properly considered respondents Guidote-Alvarez, Caparas,
Mañosa and Moreno, as their names were not recommended by the NCCA and the CCP
Boards. Otherwise, not only will the stringent selection and meticulous screening process be
The Executive Secretary, Chairman rendered futile, the respective mandates of the NCCA and the CCP Board of Trustees under
relevant laws to administer the conferment of Order of National Artists, draft the rules and
The Secretary of Foreign Affairs, Vice-Chairman regulations to guide its deliberations, formulate and implement policies and plans, and
undertake any and all necessary measures in that regard will also become meaningless.
Head, Presidential Management Staff, member
Furthermore, with respect to respondent Guidote-Alvarez who was the Executive Director of
Presidential Assistant for Historical Affairs, member the NCCA at that time, the Guidelines expressly provides:

Chief of Presidential Protocol, member 6.5 NCCA and CCP Board members and consultants and NCCA and CCP officers and staff are
automatically disqualified from being nominated.85
Chief of Protocol, DFA, member
Respondent Guidote-Alvarez could not have even been nominated, hence, she was not
qualified to be considered and conferred the Order of National Artists at that time. The
All nominations from the various awards committees must be submitted to the Committee
President’s discretion on the matter does not extend to removing a legal impediment or
on Honors via the Chancellery of Philippine Orders and State Decorations. The Chancellery
overriding a legal restriction.
shall process nominations for the consideration of the Committee on Honors. The Committee
on Honors shall screen and recommend these nominations to the President.
From the foregoing, the advice or recommendation of the NCCA and the CCP Boards as to the
conferment of the Order of National Artists on Conde, Dr. Santos, Francisco and Alcuaz was
The Committee on Honors shall, as a general rule, serve as a screening committee to ensure
not binding on the former President but only discretionary or optional for her whether or not
that nominations received from the various awards committees meet two tests: that there
to act on such advice or recommendation. Also, by virtue of the power of control, the
has not been an abuse of discretion in making the nomination, and that the nominee is in
President had the authority to alter or modify or nullify or set aside such recommendation or
good standing. Should a nomination meet these criteria, a recommendation to the President
advice. It was well within the President’s power and discretion to proclaim all, or some or
for conferment shall be made.
even none of the recommendees of the CCP and the NCCA Boards, without having to justify
198

his or her action. Thus, the exclusion of Santos did not constitute grave abuse of discretion FACTS:
on the part of the former President.
The National Artists Awards Committee. and the NCCA decided to team up and jointly
The conferment of the Order of National Artists on respondents Guidote-Alvarez, Caparas, administer the National Artists Award. There were three deliberations for determining the
Mañosa and Moreno was an entirely different matter. nominees and on the final deliberation, a final list of four names was agreed upon namely:
Manuel Conde, Ramon Santos, Lazaro Francisco and Federico Aguilar-Alcuaz.
There is grave abuse of discretion when an act is (1) done contrary to the Constitution, the
law or jurisprudence or (2) executed whimsically, capriciously or arbitrarily, out of malice, ill They submitted this recommendation to the President. According to respondents, the
will or personal bias.86 aforementioned letter was referred by the Office of the President to the Committee on
Honors. Meanwhile, the Office of the President allegedly received nominations from various
There was a violation of the equal protection clause of the Constitution87 when the former sectors, cultural groups and individuals strongly endorsing private respondents.
President gave preferential treatment to respondents Guidote-Alvarez, Caparas, Mañosa and
Moreno.1âwphi1 The former President’s constitutional duty to faithfully execute the laws Acting on this recommendation, a series of Proclamations were issued declaring Lazaro
and observe the rules, guidelines and policies of the NCCA and the CCP as to the selection of
Francisco, Federico Aguilar-Alcuaz and private respondents, Guidote-Alvarez, Caparas, Masa
the nominees for conferment of the Order of National Artists proscribed her from having a
and Moreno, respectively, as National Artists.
free and uninhibited hand in the conferment of the said award. The manifest disregard of the
rules, guidelines and processes of the NCCA and the CCP was an arbitrary act that unduly
favored respondents Guidote-Alvarez, Caparas, Mañosa and Moreno. The conferment of the Hence, the petition. All of the petitioners claim that former President Macapagal-Arroyo
Order of National Artists on said respondents was therefore made with grave abuse of gravely abused her discretion in disregarding the results of the rigorous screening and
discretion and should be set aside. selection process for the Order of National Artists and in substituting her own choice for
those of the Deliberation Panels.
While the Court invalidates today the proclamation of respondents Guidote-Alvarez, Caparas,
Mañosa and Moreno as National Artists, such action should not be taken as a ISSUE: Whether or not the act of the President amounted to grave abuse of discretion with
pronouncement on whether they are worthy to be conferred that honor. Only the President, regards to the violation of the right to equal protection
upon the advise of the NCCA and the CCP Boards, may determine that. The Court simply
declares that, as the former President committed grave abuse of discretion in issuing RULING: Yes. It should be recalled that one of the respondents was disqualified to be
Proclamation Nos. 1826 to 1829 dated July 6, 2009, the said proclamations are invalid. nominated for being the Executive Director of the NCCA at that time while respondents Masa
However, nothing in this Decision should be read as a disqualification on the part of and Caparas did not make it to the preliminary shortlist and respondent Moreno was not
respondents Guidote-Alvarez, Caparas, Mañosa and Moreno to be considered for the honor
included in the second shortlist.
of National Artist in the future, subject to compliance with the laws, rules and regulations
governing said award.
Yet, the four of them were treated differently and considered favorably when they were
exempted from the rigorous screening process of the NCCA and the CCP and conferred the
WHEREFORE, the petition is hereby GRANTED in PART. Proclamation Nos. 1826 to 1829
dated July 6, 2009 proclaiming respondents Cecile Guidote-Alvarez, Carlo Magno Jose Order of National Artists.
Caparas, Francisco Mañosa, and Jose Moreno, respectively, as National Artists are declared
INVALID and The special treatment accorded to respondents Guidote-Alvarez, Caparas, Masa and Moreno
fails to pass rational scrutiny. No real and substantial distinction between respondents and
SET ASIDE for having been issued with grave abuse of discretion. petitioner Abad has been shown that would justify deviating from the laws, guidelines and
established procedures, and placing respondents in an exceptional position.
SO ORDERED.
In view of the foregoing, there was a violation of petitioner Abads right to equal protection,
an interest that is substantial enough to confer him standing in this case.
199

FACTS:

The National Artists Awards Committee and the NCCA decided to team up and jointly
administer the National Artists Award. There were three deliberations for determining the
nominees and on the final deliberation, a final list of four names was agreed upon namely:
Manuel Conde, Ramon Santos, Lazaro Francisco and Federico Aguilar-Alcuaz.

They submitted this recommendation to the President. According to respondents, the


aforementioned letter was referred by the Office of the President to the Committee on
Honors. Meanwhile, the Office of the President allegedly received nominations from various
sectors, cultural groups and individuals strongly endorsing private respondents.

Acting on this recommendation, a series of Proclamations were issued declaring Lazaro


Francisco, Federico Aguilar-Alcuaz and private respondents, Guidote-Alvarez, Caparas, Masa
and Moreno, respectively, as National Artists.

Hence, the petition. All of the petitioners claim that former President Macapagal-Arroyo
gravely abused her discretion in disregarding the results of the rigorous screening and
selection process for the Order of National Artists and in substituting her own choice for
those of the Deliberation Panels.

ISSUE: Whether or not the act of the President amounted to grave abuse of discretion with
regards to the violation of the right to equal protection

RULING:

Yes. It should be recalled that one of the respondents was disqualified to be nominated for
being the Executive Director of the NCCA at that time while respondents Masa and Caparas
did not make it to the preliminary shortlist and respondent Moreno was not included in the
second shortlist.

Yet, the four of them were treated differently and considered favorably when they were
exempted from the rigorous screening process of the NCCA and the CCP and conferred the
Order of National Artists.

The special treatment accorded to respondents Guidote-Alvarez, Caparas, Masa and Moreno
fails to pass rational scrutiny. No real and substantial distinction between respondents and
petitioner Abad has been shown that would justify deviating from the laws, guidelines and
established procedures, and placing respondents in an exceptional position. In view of the
foregoing, there was a violation of petitioner Abads right to equal protection, an interest that
is substantial enough to confer him standing in this case.
200

EN BANC mitigating circumstances, however, the lesser penalty shall be applied in accordance with
Article 63 of the Revised Penal Code. Accordingly, the accused Former President Joseph
G.R. No. 206666 January 21, 2015 Ejercito Estrada is hereby sentenced to suffer the penalty of Reclusion Perpetua and the
accessory penalties of civil interdiction during the period of sentence and perpetual absolute
disqualification.
ATTY. ALICIA RISOS-VIDAL, Petitioner,
ALFREDO S. LIM Petitioner-Intervenor,
vs. The period within which accused Former President Joseph Ejercito Estrada has been under
COMMISSION ON ELECTIONS and JOSEPH EJERCITO ESTRADA, Respondents. detention shall be credited to him in full as long as he agrees voluntarily in writing to abide
by the same disciplinary rules imposed upon convicted prisoners.
DECISION
Moreover, in accordance with Section 2 of Republic Act No. 7080, as amended by Republic
Act No. 7659, the Court hereby declares the forfeiture in favor of the government of the
LEONARDO-DE CASTRO, J.:
following:

Before the Court are (1) a Petition for Certiorari filed under Rule 64, in relation to Rule 65,
(1) The total amount of Five Hundred Forty[-]Two Million Seven Hundred Ninety[-
both of the Revised Rules of Court, by Atty. Alicia Risos-Vidal (Risos-Vidal), which essentially
]One Thousand Pesos (₱545,291,000.00), with interest and income earned,
prays for the issuance of the writ of certiorari annulling and setting aside the April 1,
inclusive of the amount of Two Hundred Million Pesos (₱200,000,000.00),
20131 and April 23, 20132 Resolutions of the Commission on Elections (COMELEC), Second
deposited in the name and account of the Erap Muslim Youth Foundation.
Division and En bane, respectively, in SPA No. 13-211 (DC), entitled "Atty. Alicia Risos-Vidal v.
Joseph Ejercito Estrada" for having been rendered with grave abuse of discretion amounting
to lack or excess of jurisdiction; and (2) a Petition-in-Intervention3 filed by Alfredo S. Lim (2) The amount of One Hundred Eighty[-]Nine Million Pesos (₱189,000,000.00),
(Lim), wherein he prays to be declared the 2013 winning candidate for Mayor of the City of inclusive of interests and income earned, deposited in the Jose Velarde account.
Manila in view of private respondent former President Joseph Ejercito Estrada’s (former
President Estrada) disqualification to run for and hold public office. (3) The real property consisting of a house and lot dubbed as "Boracay Mansion"
located at #100 11th Street, New Manila, Quezon City.
The Facts
The cash bonds posted by accused Jose "Jinggoy" Estrada and Atty. Edward S. Serapio are
The salient facts of the case are as follows: hereby ordered cancelled and released to the said accused or their duly authorized
representatives upon presentation of the original receipt evidencing payment thereof and
subject to the usual accounting and auditing procedures. Likewise, the hold-departure orders
On September 12, 2007, the Sandiganbayan convicted former President Estrada, a former
issued against the said accused are hereby recalled and declared functus oficio.4
President of the Republic of the Philippines, for the crime of plunder in Criminal Case No.
26558, entitled "People of the Philippines v. Joseph Ejercito Estrada, et al." The dispositive
part of the graft court’s decision reads: On October 25, 2007, however, former President Gloria Macapagal Arroyo (former President
Arroyo) extended executive clemency, by way of pardon, to former President Estrada. The
full text of said pardon states:
WHEREFORE, in view of all the foregoing, judgment is hereby rendered in Criminal Case No.
26558 finding the accused, Former President Joseph Ejercito Estrada, GUILTY beyond
reasonable doubt of the crime of PLUNDER, defined in and penalized by Republic Act No. MALACAÑAN PALACE
7080, as amended. On the other hand, for failure of the prosecution to prove and establish MANILA
their guilt beyond reasonable doubt, the Court finds the accused Jose "Jinggoy" Estrada and
Atty. Edward S. Serapio NOT GUILTY of the crime of plunder, and accordingly, the Court By the President of the Philippines
hereby orders their ACQUITTAL.
PARDON
The penalty imposable for the crime of plunder under Republic Act No. 7080, as amended by
Republic Act No. 7659, is Reclusion Perpetua to Death. There being no aggravating or
201

WHEREAS, this Administration has a policy of releasing inmates who have reached the age of former President Arroyo restored the former’s right to vote and be voted for a public office.
seventy (70), The subsequent motions for reconsideration thereto were denied by the COMELEC En banc.

WHEREAS, Joseph Ejercito Estrada has been under detention for six and a half years, After the conduct of the May 10, 2010 synchronized elections, however, former President
Estrada only managed to garner the second highest number of votes.
WHEREAS, Joseph Ejercito Estrada has publicly committed to no longer seek any elective
position or office, Of the three petitioners above-mentioned, only Pormento sought recourse to this Court and
filed a petition for certiorari, which was docketed as G.R. No. 191988, entitled "Atty. Evilio C.
IN VIEW HEREOF and pursuant to the authority conferred upon me by the Constitution, I Pormento v. Joseph ‘ERAP’ Ejercito Estrada and Commission on Elections." But in a
hereby grant executive clemency to JOSEPH EJERCITO ESTRADA, convicted by the Resolution9 dated August 31, 2010, the Court dismissed the aforementioned petition on the
Sandiganbayan of Plunder and imposed a penalty of Reclusion Perpetua. He is hereby ground of mootness considering that former President Estrada lost his presidential bid.
restored to his civil and political rights.
On October 2, 2012, former President Estrada once more ventured into the political arena,
The forfeitures imposed by the Sandiganbayan remain in force and in full, including all writs and filed a Certificate of Candidacy,10 this time vying for a local elective post, that ofthe
and processes issued by the Sandiganbayan in pursuance hereof, except for the bank Mayor of the City of Manila.
account(s) he owned before his tenure as President.
On January 24, 2013, Risos-Vidal, the petitioner in this case, filed a Petition for
Upon acceptance of this pardon by JOSEPH EJERCITO ESTRADA, this pardon shall take effect. Disqualification against former President Estrada before the COMELEC. The petition was
docketed as SPA No. 13-211 (DC). Risos Vidal anchored her petition on the theory that
"[Former President Estrada] is Disqualified to Run for Public Office because of his Conviction
Given under my hand at the City of Manila, this 25th Day of October, in the year of Our Lord,
for Plunder by the Sandiganbayan in Criminal Case No. 26558 entitled ‘People of the
two thousand and seven.
Philippines vs. Joseph Ejercito Estrada’ Sentencing Him to Suffer the Penalty of Reclusion
Perpetuawith Perpetual Absolute Disqualification."11 She relied on Section 40 of the Local
Gloria M. Arroyo (sgd.) Government Code (LGC), in relation to Section 12 of the Omnibus Election Code (OEC), which
state respectively, that:
By the President:
Sec. 40, Local Government Code:
IGNACIO R. BUNYE (sgd.)
Acting Executive Secretary5 SECTION 40. Disqualifications.- The following persons are disqualified from running for any
elective local position:
On October 26, 2007, at 3:35 p.m., former President Estrada "received and accepted"6 the
pardon by affixing his signature beside his handwritten notation thereon. (a) Those sentenced by final judgment for an offense involving moral turpitude or
for an offense punishable by one (1) year or more of imprisonment, within two (2)
On November 30, 2009, former President Estrada filed a Certificate of Candidacy7 for the years after serving sentence; (b) Those removed from office as a result of an
position of President. During that time, his candidacy earned three oppositions in the administrative case;
COMELEC: (1) SPA No. 09-024 (DC), a "Petition to Deny Due Course and Cancel Certificate of
Candidacy" filed by Rev. Elly Velez B. Lao Pamatong, ESQ; (2) SPA No. 09-028 (DC), a petition (c) Those convicted by final judgment for violating the oath of allegiance to the
for "Disqualification as Presidential Candidate" filed by Evilio C. Pormento (Pormento); and Republic;
(3) SPA No. 09-104 (DC), a "Petition to Disqualify Estrada Ejercito, Joseph M.from Running as
President due to Constitutional Disqualification and Creating Confusion to the Prejudice of
(d) Those with dual citizenship;
Estrada, Mary Lou B" filed by Mary Lou Estrada. In separate Resolutions8 dated January 20,
2010 by the COMELEC, Second Division, however, all three petitions were effectively
dismissed on the uniform grounds that (i) the Constitutional proscription on reelection (e) Fugitives from justice in criminal or nonpolitical cases here or abroad;
applies to a sitting president; and (ii) the pardon granted to former President Estrada by
202

(f) Permanent residents in a foreign country or those who have acquired the right RESPONDENT ESTRADA IS DISQUALIFIED TO RUN AS MAYOR OF MANILA UNDER
to reside abroad and continue to avail of the same right after the effectivity of this SEC. 40 OF THE LOCAL GOVERNMENTCODE OF 1991 FOR HAVING BEEN CONVICTED
Code; and OF PLUNDER, AN OFFENSE INVOLVING MORAL TURPITUDE;

(g) The insane or feeble minded. (Emphasis supplied.) III. RESPONDENT COMELEC COMMITTED GRAVE ABUSE OF DISCRETION
AMOUNTING TO LACK OR EXCESS OF JURISDICTION IN DISMISSING THE PETITION
Sec. 12, Omnibus Election Code: FOR DISQUALIFICATION ON THE GROUND THAT THE CASE INVOLVES THE SAME OR
SIMILAR ISSUES IT ALREADY RESOLVED IN THE CASES OF "PORMENTO VS.
ESTRADA", SPA NO. 09-028 (DC) AND IN "RE: PETITION TO DISQUALIFY ESTRADA
Section 12. Disqualifications. - Any person who has been declared by competent authority
EJERCITO, JOSEPH M. FROM RUNNING AS PRESIDENT, ETC.," SPA NO. 09-104 (DC);
insane or incompetent, or has been sentenced by final judgmentfor subversion, insurrection,
rebellion, or for any offense for which he has been sentenced to a penalty of more than
eighteen months or for a crime involving moral turpitude, shall be disqualified to be a IV. RESPONDENT COMELEC COMMITTED GRAVE ABUSE OF DISCRETION
candidate and to hold any public office, unless he has been given plenary pardon or granted AMOUNTING TO LACK OR EXCESS OF JURISDICTION IN NOT RULING THAT
amnesty. (Emphases supplied.) RESPONDENT ESTRADA’S PARDON NEITHER RESTORED HIS RIGHT OF SUFFRAGE
NOR REMITTED HIS PERPETUAL ABSOLUTE DISQUALIFICATION FROM SEEKING
PUBLIC OFFICE; and
In a Resolution dated April 1, 2013,the COMELEC, Second Division, dismissed the petition for
disqualification, the fallo of which reads:
V. RESPONDENT COMELEC COMMITTED GRAVE ABUSE OF DISCRETION
AMOUNTING TO LACK OR EXCESS OF JURISDICTION IN NOT HAVING EXERCISED ITS
WHEREFORE, premises considered, the instant petition is hereby DISMISSED for utter lack of
POWER TO MOTU PROPRIO DISQUALIFY RESPONDENT ESTRADA IN THE FACE OF
merit.12
HIS PATENT DISQUALIFICATION TO RUN FOR PUBLIC OFFICE BECAUSE OF HIS
PERPETUAL AND ABSOLUTE DISQUALIFICATION TO SEEK PUBLIC OFFICE AND TO
The COMELEC, Second Division, opined that "[h]aving taken judicial cognizance of the VOTE RESULTING FROM HIS CRIMINAL CONVICTION FOR PLUNDER.14
consolidated resolution for SPA No. 09-028 (DC) and SPA No. 09-104 (DC) and the 10 May
2010 En Banc resolution affirming it, this Commission will not be labor the controversy
While this case was pending beforethe Court, or on May 13, 2013, the elections were
further. Moreso, [Risos-Vidal] failed to present cogent proof sufficient to reverse the
conducted as scheduled and former President Estrada was voted into office with 349,770
standing pronouncement of this Commission declaring categorically that [former President
votes cast in his favor. The next day, the local board of canvassers proclaimed him as the duly
Estrada’s] right to seek public office has been effectively restored by the pardon vested upon
elected Mayor of the City of Manila.
him by former President Gloria M. Arroyo. Since this Commission has already spoken, it will
no longer engage in disquisitions of a settled matter lest indulged in wastage of government
resources."13 On June 7, 2013, Lim, one of former President Estrada’s opponents for the position of Mayor,
moved for leave to intervene in this case. His motion was granted by the Court in a
Resolution15 dated June 25, 2013. Lim subscribed to Risos-Vidal’s theory that former
The subsequent motion for reconsideration filed by Risos-Vidal was denied in a Resolution
President Estrada is disqualified to run for and hold public office as the pardon granted to the
dated April 23, 2013.
latter failed to expressly remit his perpetual disqualification. Further, given that former
President Estrada is disqualified to run for and hold public office, all the votes obtained by
On April 30, 2013, Risos-Vidal invoked the Court’s jurisdiction by filing the present petition. the latter should be declared stray, and, being the second placer with 313,764 votes to his
She presented five issues for the Court’s resolution, to wit: name, he (Lim) should be declared the rightful winning candidate for the position of Mayor
of the City of Manila.
I. RESPONDENT COMELEC COMMITTED GRAVE ABUSE OF DISCRETION
AMOUNTING TO LACK OR EXCESS OF JURISDICTION IN HOLDING THAT The Issue
RESPONDENT ESTRADA’S PARDON WAS NOT CONDITIONAL;
Though raising five seemingly separate issues for resolution, the petition filed by Risos-Vidal
II. RESPONDENT COMELEC COMMITTED GRAVE ABUSE OF DISCRETION actually presents only one essential question for resolution by the Court, that is, whether or
AMOUNTING TO LACK OR EXCESS OF JURISDICTION IN NOT FINDING THAT not the COMELEC committed grave abuse of discretion amounting to lack or excess of
203

jurisdiction in ruling that former President Estrada is qualified to vote and be voted for in She avers that in view of the foregoing provisions of law, it is not enough that a pardon
public office as a result of the pardon granted to him by former President Arroyo. makes a general statement that such pardon carries with it the restoration of civil and
political rights. By virtue of Articles 36 and 41, a pardon restoring civil and political rights
In her petition, Risos-Vidal starts her discussion by pointing out that the pardon granted to without categorically making mention what specific civil and political rights are restored
former President Estrada was conditional as evidenced by the latter’s express acceptance "shall not work to restore the right to hold public office, or the right of suffrage; nor shall it
thereof. The "acceptance," she claims, is an indication of the conditional natureof the remit the accessory penalties of civil interdiction and perpetual absolute disqualification for
pardon, with the condition being embodied in the third Whereas Clause of the pardon, i.e., the principal penalties of reclusion perpetua and reclusion temporal."17 In other words, she
"WHEREAS, Joseph Ejercito Estrada has publicly committed to no longer seek any elective considers the above constraints as mandatory requirements that shun a general or implied
position or office." She explains that the aforementioned commitment was what impelled restoration of civil and political rights in pardons.
former President Arroyo to pardon former President Estrada, without it, the clemency would
not have been extended. And any breach thereof, that is, whenformer President Estrada filed Risos-Vidal cites the concurring opinions of Associate Justices Teodoro R. Padilla and
his Certificate of Candidacy for President and Mayor of the City of Manila, he breached the Florentino P. Feliciano in Monsanto v. Factoran, Jr.18 to endorse her position that "[t]he
condition of the pardon; hence, "he ought to be recommitted to prison to serve the restoration of the right to hold public office to one who has lost such right by reason of
unexpired portion of his sentence x x x and disqualifies him as a candidate for the mayoralty conviction in a criminal case, but subsequently pardoned, cannot be left to inference, no
[position] of Manila."16 matter how intensely arguable, but must be statedin express, explicit, positive and specific
language."
Nonetheless, Risos-Vidal clarifies that the fundamental basis upon which former President
Estrada mustbe disqualified from running for and holding public elective office is actually the Applying Monsantoto former President Estrada’s case, Risos-Vidal reckons that "such express
proscription found in Section 40 of the LGC, in relation to Section 12 ofthe OEC. She argues restoration is further demanded by the existence of the condition in the [third] [W]hereas
that the crime of plunder is both an offense punishable by imprisonment of one year or more [C]lause of the pardon x x x indubitably indicating that the privilege to hold public office was
and involving moral turpitude; such that former President Estrada must be disqualified to run not restored to him."19
for and hold public elective office.
On the other hand, the Office ofthe Solicitor General (OSG) for public respondent COMELEC,
Even with the pardon granted to former President Estrada, however, Risos-Vidal insists that maintains that "the issue of whether or not the pardon extended to [former President
the same did not operate to make available to former President Estrada the exception Estrada] restored his right to run for public office had already been passed upon by public
provided under Section 12 of the OEC, the pardon being merely conditional and not absolute respondent COMELEC way back in 2010 via its rulings in SPA Nos. 09-024, 09-028 and 09-104,
or plenary. Moreover, Risos-Vidal puts a premium on the ostensible requirements provided there is no cogent reason for it to reverse its standing pronouncement and declare [former
under Articles 36 and 41 of the Revised Penal Code, to wit: President Estrada] disqualified to run and be voted as mayor of the City of Manila in the
absence of any new argument that would warrant its reversal. To be sure, public respondent
ART. 36. Pardon; its effects.– A pardon shall not work the restoration of the right to hold COMELEC correctly exercised its discretion in taking judicial cognizance of the aforesaid
publicoffice, or the right of suffrage, unless such rights be expressly restored by the terms of rulings which are known toit and which can be verified from its own records, in accordance
the pardon. with Section 2, Rule 129 of the Rules of Court on the courts’ discretionary power to take
judicial notice of matters which are of public knowledge, orare capable of unquestionable
demonstration, or ought to be known to them because of their judicial functions."20
A pardon shall in no case exempt the culprit from the payment of the civil indemnity imposed
upon him by the sentence.
Further, the OSG contends that "[w]hile at first glance, it is apparent that [former President
Estrada’s] conviction for plunder disqualifies him from running as mayor of Manila under
ART. 41. Reclusion perpetua and reclusion temporal – Their accessory penalties.– The
Section 40 of the [LGC], the subsequent grant of pardon to him, however, effectively
penalties of reclusion perpetua and reclusion temporal shall carry with them that of civil
restored his right to run for any public office."21 The restoration of his right to run for any
interdiction for life or during the period of the sentence as the case may be, and that of
public office is the exception to the prohibition under Section 40 of the LGC, as provided
perpetual absolute disqualification which the offender shall suffer even though pardoned as
under Section 12 of the OEC. As to the seeming requirement of Articles 36 and 41 of the
to the principal penalty, unless the same shall have been expressly remitted in the pardon.
Revised Penal Code, i.e., the express restoration/remission of a particular right to be stated
(Emphases supplied.)
in the pardon, the OSG asserts that "an airtight and rigid interpretation of Article 36 and
Article 41 of the [RPC] x x x would be stretching too much the clear and plain meaning of the
aforesaid provisions."22 Lastly, taking into consideration the third Whereas Clause of the
204

pardon granted to former President Estrada, the OSG supports the position that it "is not an former President Estrada, docketed as SPA No. 13-211 (DC), was anchored on Section 40 of
integral part of the decree of the pardon and cannot therefore serve to restrict its the LGC, in relation to Section 12 of the OEC, that is, having been convicted of a crime
effectivity."23 punishable by imprisonment of one year or more, and involving moral turpitude, former
President Estrada must be disqualified to run for and hold public elective office
Thus, the OSG concludes that the "COMELEC did not commit grave abuse of discretion notwithstanding the fact that he is a grantee of a pardon that includes a statement
amounting to lack or excess of jurisdiction in issuing the assailed Resolutions." 24 expressing "[h]e is hereby restored to his civil and political rights." Risos-Vidal theorizes that
former President Estrada is disqualified from running for Mayor of Manila inthe May 13,
2013 Elections, and remains disqualified to hold any local elective post despite the
For his part, former President Estrada presents the following significant arguments to defend
presidential pardon extended to him in 2007 by former President Arroyo for the reason that
his stay in office: that "the factual findings of public respondent COMELEC, the Constitutional
it (pardon) did not expressly provide for the remission of the penalty of perpetual absolute
body mandated to administer and enforce all laws relative to the conduct of the elections,
disqualification, particularly the restoration of his (former President Estrada) right to vote
[relative to the absoluteness of the pardon, the effects thereof, and the eligibility of former
and bevoted upon for public office. She invokes Articles 36 and 41 of the Revised Penal Code
President Estrada to seek public elective office] are binding [and conclusive] on this
as the foundations of her theory.
Honorable Supreme Court;" that he "was granted an absolute pardon and thereby restored
to his full civil and political rights, including the right to seek public elective office such as the
mayoral (sic) position in the City of Manila;" that "the majority decision in the case of It is insisted that, since a textual examination of the pardon given to and accepted by former
Salvacion A. Monsanto v. Fulgencio S. Factoran, Jr.,which was erroneously cited by both Vidal President Estrada does not actually specify which political right is restored, it could be
and Lim as authority for their respective claims, x x x reveal that there was no discussion inferred that former President Arroyo did not deliberately intend to restore former President
whatsoever in the ratio decidendi of the Monsanto case as to the alleged necessity for an Estrada’s rights of suffrage and to hold public office, orto otherwise remit the penalty of
expressed restoration of the ‘right to hold public office in the pardon’ as a legal prerequisite perpetual absolute disqualification. Even if her intention was the contrary, the same cannot
to remove the subject perpetual special disqualification;" that moreover, the "principal be upheld based on the pardon’s text.
question raised in this Monsanto case is whether or not a public officer, who has been
granted an absolute pardon by the Chief Executive, is entitled to reinstatement toher former The pardoning power of the President cannot be limited by legislative action.
position without need of a new appointment;" that his "expressed acceptance [of the
pardon] is not proof that the pardon extended to [him] is conditional and not absolute;" that The 1987 Constitution, specifically Section 19 of Article VII and Section 5 of Article IX-C,
this case is a mere rehash of the casesfiled against him during his candidacy for President provides that the President of the Philippines possesses the power to grant pardons, along
back in 2009-2010; that Articles 36 and 41 of the Revised Penal Code "cannot abridge or with other acts of executive clemency, to wit:
diminish the pardoning power of the President expressly granted by the Constitution;" that
the text of the pardon granted to him substantially, if not fully, complied with the
Section 19. Except in cases of impeachment, or as otherwise provided in this Constitution,
requirement posed by Article 36 of the Revised Penal Code as it was categorically stated in
the President may grant reprieves, commutations, and pardons, and remit fines and
the said document that he was "restored to his civil and political rights;" that since pardon is
forfeitures, after conviction by final judgment.
an act of grace, it must be construed favorably in favor of the grantee;25 and that his
disqualification will result in massive disenfranchisement of the hundreds of thousands of
Manileños who voted for him.26 He shall also have the power to grant amnesty with the concurrence of a majority of all the
Members of the Congress.
The Court's Ruling
Section 5. No pardon, amnesty, parole, or suspension of sentence for violation of election
laws, rules, and regulations shall be granted by the President without the favorable
The petition for certiorari lacks merit.
recommendation of the Commission.

Former President Estrada was granted an absolute pardon that fully restored allhis civil and
It is apparent from the foregoing constitutional provisions that the only instances in which
political rights, which naturally includes the right to seek public elective office, the focal point
the President may not extend pardon remain to be in: (1) impeachment cases; (2) cases that
of this controversy. The wording of the pardon extended to former President Estrada is
have not yet resulted in a final conviction; and (3) cases involving violations of election laws,
complete, unambiguous, and unqualified. It is likewise unfettered by Articles 36 and 41 of
rules and regulations in which there was no favorable recommendation coming from the
the Revised Penal Code. The only reasonable, objective, and constitutional interpretation of
COMELEC. Therefore, it can be argued that any act of Congress by way of statute cannot
the language of the pardon is that the same in fact conforms to Articles 36 and 41 of the
operate to delimit the pardoning power of the President.
Revised Penal Code. Recall that the petition for disqualification filed by Risos-Vidal against
205

In Cristobal v. Labrador27 and Pelobello v. Palatino,28 which were decided under the 1935 of that Commission is required before executive clemency isgranted because violations of the
Constitution,wherein the provision granting pardoning power to the President shared similar election laws go into the very political life of the country.
phraseology with what is found in the present 1987 Constitution, the Court then
unequivocally declared that "subject to the limitations imposed by the Constitution, the With respect to violations of our Corrupt Practices Law, we felt that it is also necessary to
pardoning power cannot be restricted or controlled by legislative action." The Court have that subjected to the same condition because violation of our Corrupt Practices Law
reiterated this pronouncement in Monsanto v. Factoran, Jr.29 thereby establishing that, may be of such magnitude as to affect the very economic systemof the country.
under the present Constitution, "a pardon, being a presidential prerogative, should not be Nevertheless, as a compromise, we provided here that it will be the Congress that will
circumscribed by legislative action." Thus, it is unmistakably the long-standing position of this provide for the classification as to which convictions will still require prior recommendation;
Court that the exercise of the pardoning power is discretionary in the President and may not after all, the Congress could take into account whether or not the violation of the Corrupt
be interfered with by Congress or the Court, except only when it exceeds the limits provided Practices Law is of such magnitude as to affect the economic life of the country, if it is in the
for by the Constitution. millions or billions of dollars. But I assume the Congress in its collective wisdom will exclude
those petty crimes of corruption as not to require any further stricture on the exercise of
This doctrine of non-diminution or non-impairment of the President’s power of pardon by executive clemency because, of course, there is a whale of a difference if we consider a lowly
acts of Congress, specifically through legislation, was strongly adhered to by an clerk committing malversation of government property or funds involving one hundred
overwhelming majority of the framers of the 1987 Constitution when they flatly rejected a pesos. But then, we also anticipate the possibility that the corrupt practice of a public officer
proposal to carve out an exception from the pardoning power of the President in the form of is of such magnitude as to have virtually drained a substantial portion of the treasury, and
"offenses involving graft and corruption" that would be enumerated and defined by Congress then he goes through all the judicial processes and later on, a President who may have close
through the enactment of a law. The following is the pertinent portion lifted from the Record connections with him or out of improvident compassion may grant clemency under such
of the Commission (Vol. II): conditions. That is why we left it to Congress to provide and make a classification based on
substantial distinctions between a minor act of corruption or an act of substantial
MR. ROMULO. I ask that Commissioner Tan be recognized to introduce an amendment on proportions. SR. TAN. So, why do we not just insert the word GROSS or GRAVE before the
the same section. word "violations"?

THE PRESIDENT. Commissioner Tan is recognized. MR. REGALADO. We feel that Congress can make a better distinction because "GRAVE" or
"GROSS" can be misconstrued by putting it purely as a policy.
SR. TAN. Madam President, lines 7 to 9 state:
MR. RODRIGO. Madam President.
However, the power to grant executive clemency for violations of corrupt practices laws may
be limited by legislation. THE PRESIDENT. Commissioner Rodrigo is recognized.

I suggest that this be deletedon the grounds that, first, violations of corrupt practices may MR. RODRIGO. May I speak in favor of the proposed amendment?
include a very little offense like stealing ₱10; second, which I think is more important, I get
the impression, rightly or wrongly, that subconsciously we are drafting a constitution on the THE PRESIDENT. Please proceed.
premise that all our future Presidents will bebad and dishonest and, consequently, their acts
will be lacking in wisdom. Therefore, this Article seems to contribute towards the creation of MR. RODRIGO. The power to grant executive clemency is essentially an executive power, and
an anti-President Constitution or a President with vast responsibilities but no corresponding that is precisely why it is called executive clemency. In this sentence, which the amendment
power except to declare martial law. Therefore, I request that these lines be deleted. seeks to delete, an exception is being made. Congress, which is the legislative arm, is allowed
to intrude into this prerogative of the executive. Then it limits the power of Congress to
MR. REGALADO. Madam President,may the Committee react to that? subtract from this prerogative of the President to grant executive clemency by limiting the
power of Congress to only corrupt practices laws. There are many other crimes more serious
THE PRESIDENT. Yes, please. than these. Under this amendment, Congress cannot limit the power of executive clemency
in cases of drug addiction and drug pushing which are very, very serious crimes that can
endanger the State; also, rape with murder, kidnapping and treason. Aside from the fact that
MR. REGALADO. This was inserted here on the resolution of Commissioner Davide because of
it is a derogation of the power of the President to grant executive clemency, it is also
the fact that similar to the provisions on the Commission on Elections, the recommendation
206

defective in that it singles out just one kind of crime. There are far more serious crimes which Madam President, over and over again, we have been saying and arguing before this
are not included. Constitutional Commission that we are emasculating the powers of the presidency, and this
provision to me is another clear example of that. So, I speak against this provision. Even the
MR. REGALADO. I will just make one observation on that. We admit that the pardoning 1935 and the 1973 Constitutions do not provide for this kind of provision.
power is anexecutive power. But even in the provisions on the COMELEC, one will notice that
constitutionally, it is required that there be a favorable recommendation by the Commission I am supporting the amendment by deletion of Commissioner Tan.
on Elections for any violation of election laws.
MR. ROMULO. Commissioner Tingson would like to be recognized.
At any rate, Commissioner Davide, as the principal proponent of that and as a member of the
Committee, has explained in the committee meetings we had why he sought the inclusion of THE PRESIDENT. Commissioner Tingson is recognized.
this particular provision. May we call on Commissioner Davide to state his position.
MR. TINGSON. Madam President, I am also in favor of the amendment by deletion because I
MR. DAVIDE. Madam President. am in sympathy with the stand of Commissioner Francisco "Soc" Rodrigo. I do believe and we
should remember that above all the elected or appointed officers of our Republic, the leader
THE PRESIDENT. Commissioner Davide is recognized. is the President. I believe that the country will be as the President is, and if we systematically
emasculate the power of this presidency, the time may come whenhe will be also handcuffed
MR. DAVIDE. I am constrained to rise to object to the proposal. We have just approved the that he will no longer be able to act like he should be acting.
Article on Accountability of Public Officers. Under it, it is mandated that a public office is a
public trust, and all government officers are under obligation to observe the utmost of So, Madam President, I am in favor of the deletion of this particular line.
responsibility, integrity, loyalty and efficiency, to lead modest lives and to act with patriotism
and justice. MR. ROMULO. Commissioner Colayco would like to be recognized.

In all cases, therefore, which would go into the verycore of the concept that a public office is THE PRESIDENT. Commissioner Colayco is recognized.
a public trust, the violation is itself a violation not only of the economy but the moral fabric
of public officials. And that is the reason we now want that if there is any conviction for the
MR. COLAYCO. Thank you very much, Madam President.
violation of the Anti-Graft and Corrupt Practices Act, which, in effect, is a violation of the
public trust character of the public office, no pardon shall be extended to the offender,
unless some limitations are imposed. I seldom rise here to object to or to commend or to recommend the approval of proposals,
but now I find that the proposal of Commissioner Tan is worthy of approval of this body.
Originally, my limitation was, it should be with the concurrence of the convicting court, but
the Committee left it entirely to the legislature to formulate the mechanics at trying, Why are we singling out this particular offense? There are other crimes which cast a bigger
probably, to distinguish between grave and less grave or serious cases of violation of the blot on the moral character of the public officials.
Anti-Graft and Corrupt Practices Act. Perhaps this is now the best time, since we have
strengthened the Article on Accountability of Public Officers, to accompany it with a mandate Finally, this body should not be the first one to limit the almost absolute power of our Chief
that the President’s right to grant executive clemency for offenders or violators of laws Executive in deciding whether to pardon, to reprieve or to commute the sentence rendered
relating to the concept of a public office may be limited by Congress itself. by the court.

MR. SARMIENTO. Madam President. I thank you.

THE PRESIDENT. Commissioner Sarmiento is recognized. THE PRESIDENT. Are we ready to vote now?

MR. SARMIENTO. May I briefly speak in favor of the amendment by deletion. MR. ROMULO. Commissioner Padilla would like to be recognized, and after him will be
Commissioner Natividad.
207

THE PRESIDENT. Commissioner Padilla is recognized. As many as are against, please raise their hand. (Few Members raised their hand.)

MR. PADILLA. Only one sentence, Madam President. The Sandiganbayan has been called the The results show 34 votes in favor and 4 votes against; the amendment is
Anti-Graft Court, so if this is allowed to stay, it would mean that the President’s power approved.30 (Emphases supplied.)
togrant pardon or reprieve will be limited to the cases decided by the Anti-Graft Court, when
as already stated, there are many provisions inthe Revised Penal Code that penalize more The proper interpretation of Articles
serious offenses.
36 and 41 of the Revised Penal Code.
Moreover, when there is a judgment of conviction and the case merits the consideration of
the exercise of executive clemency, usually under Article V of the Revised Penal Code the
The foregoing pronouncements solidify the thesis that Articles 36 and 41 of the Revised
judge will recommend such exercise of clemency. And so, I am in favor of the amendment
Penal Code cannot, in any way, serve to abridge or diminish the exclusive power and
proposed by Commissioner Tan for the deletion of this last sentence in Section 17.
prerogative of the President to pardon persons convicted of violating penal statutes.

THE PRESIDENT. Are we ready to vote now, Mr. Floor Leader?


The Court cannot subscribe to Risos-Vidal’s interpretation that the said Articles contain
specific textual commands which must be strictly followed in order to free the beneficiary of
MR. NATIVIDAD. Just one more. presidential grace from the disqualifications specifically prescribed by them.

THE PRESIDENT. Commissioner Natividad is recognized. Again, Articles 36 and 41 of the Revised Penal Code provides:

MR. NATIVIDAD. I am also against this provision which will again chip more powers from the ART. 36. Pardon; its effects.– A pardon shall not work the restoration of the right to hold
President. In case of other criminals convicted in our society, we extend probation to them publicoffice, or the right of suffrage, unless such rights be expressly restored by the terms of
while in this case, they have already been convicted and we offer mercy. The only way we the pardon.
can offer mercy to them is through this executive clemency extended to them by the
President. If we still close this avenue to them, they would be prejudiced even worse than
A pardon shall in no case exempt the culprit from the payment of the civil indemnity imposed
the murderers and the more vicious killers in our society. I do not think they deserve this
upon him by the sentence.
opprobrium and punishment under the new Constitution.

ART. 41. Reclusion perpetua and reclusion temporal – Their accessory penalties.– The
I am in favor of the proposed amendment of Commissioner Tan.
penalties of reclusion perpetua and reclusion temporal shall carry with them that of civil
interdiction for life or during the period of the sentence as the case may be, and that of
MR. ROMULO. We are ready tovote, Madam President. perpetual absolute disqualification which the offender shall suffer even though pardoned as
to the principal penalty, unless the same shall have been expressly remitted in the pardon.
THE PRESIDENT. Is this accepted by the Committee? (Emphases supplied.)

MR. REGALADO. The Committee, Madam President, prefers to submit this to the floor and A rigid and inflexible reading of the above provisions of law, as proposed by Risos-Vidal, is
also because of the objection of the main proponent, Commissioner Davide. So we feel that unwarranted, especially so if it will defeat or unduly restrict the power of the President to
the Commissioners should vote on this question. grant executive clemency.

VOTING It is well-entrenched in this jurisdiction that where the words of a statute are clear, plain, and
free from ambiguity, it must be given its literal meaning and applied without attempted
THE PRESIDENT. As many as are in favor of the proposed amendment of Commissioner Tan interpretation. Verba legis non est recedendum. From the words of a statute there should be
to delete the last sentence of Section 17 appearing on lines 7, 8 and 9, please raise their no departure.31 It is this Court’s firm view that the phrase in the presidential pardon at issue
hand. (Several Members raised their hand.) which declares that former President Estrada "is hereby restored to his civil and political
rights" substantially complies with the requirement of express restoration.
208

The Dissent of Justice Marvic M.V.F. Leonen agreed with Risos Vidal that there was no both. Thus, Articles 36 and 41 only clarify the effect of the pardon so decided upon by the
express remission and/or restoration of the rights of suffrage and/or to hold public office in President on the penalties imposedin accordance with law.
the pardon granted to former President Estrada, as required by Articles 36 and 41 of the
Revised Penal Code. A close scrutiny of the text of the pardon extended to former President Estrada shows that
both the principal penalty of reclusion perpetua and its accessory penalties are included in
Justice Leonen posits in his Dissent that the aforementioned codal provisions must be the pardon. The first sentence refers to the executive clemency extended to former
followed by the President, as they do not abridge or diminish the President’s power to President Estrada who was convicted by the Sandiganbayan of plunder and imposed a
extend clemency. He opines that they do not reduce the coverage of the President’s penalty of reclusion perpetua. The latter is the principal penalty pardoned which relieved him
pardoning power. Particularly, he states: of imprisonment. The sentence that followed, which states that "(h)e is hereby restored to
his civil and political rights," expressly remitted the accessory penalties that attached to the
Articles 36 and 41 refer only to requirements of convention or form. They only provide a principal penalty of reclusion perpetua. Hence, even if we apply Articles 36 and 41 of the
procedural prescription. They are not concerned with areas where or the instances when the Revised Penal Code, it is indubitable from the textof the pardon that the accessory penalties
President may grant pardon; they are only concerned with how he or she is to exercise such of civil interdiction and perpetual absolute disqualification were expressly remitted together
power so that no other governmental instrumentality needs to intervene to give it full effect. with the principal penalty of reclusion perpetua.

All that Articles 36 and 41 do is prescribe that, if the President wishes to include in the In this jurisdiction, the right toseek public elective office is recognized by law as falling under
pardon the restoration of the rights of suffrage and to hold public office, or the remission of the whole gamut of civil and political rights.
the accessory penalty of perpetual absolute disqualification,he or she should do so expressly.
Articles 36 and 41 only ask that the President state his or her intentions clearly, directly, Section 5 of Republic Act No. 9225,34 otherwise known as the "Citizenship Retention and
firmly, precisely, and unmistakably. To belabor the point, the President retains the power to Reacquisition Act of 2003," reads as follows:
make such restoration or remission, subject to a prescription on the manner by which he or
she is to state it.32 Section 5. Civil and Political Rights and Liabilities.– Those who retain or reacquire Philippine
citizenship under this Act shall enjoy full civil and political rights and be subject to all
With due respect, I disagree with the overbroad statement that Congress may dictate as to attendant liabilities and responsibilities under existing laws of the Philippines and the
how the President may exercise his/her power of executive clemency. The form or manner following conditions: (1) Those intending to exercise their right of suffrage must meet the
by which the President, or Congress for that matter, should exercise their respective requirements under Section 1, Article V of the Constitution, Republic Act No. 9189, otherwise
Constitutional powers or prerogatives cannot be interfered with unless it is so provided in known as "The Overseas Absentee Voting Act of 2003" and other existing laws;
the Constitution. This is the essence of the principle of separation of powers deeply ingrained
in our system of government which "ordains that each of the three great branches of (2) Those seeking elective public office in the Philippines shall meet the
government has exclusive cognizance of and is supreme in matters falling within its own qualifications for holding such public office as required by the Constitution and
constitutionally allocated sphere."33 Moreso, this fundamental principle must be observed if existing laws and, at the time of the filing of the certificate of candidacy, make a
noncompliance with the form imposed by one branch on a co-equal and coordinate branch personal and sworn renunciation of any and all foreign citizenship before any
will result into the diminution of an exclusive Constitutional prerogative. public officer authorized to administer an oath;

For this reason, Articles 36 and 41 of the Revised Penal Code should be construed in a way (3) Those appointed to any public office shall subscribe and swear an oath of
that will give full effect to the executive clemency granted by the President, instead of allegiance to the Republic of the Philippines and its duly constituted authorities
indulging in an overly strict interpretation that may serve to impair or diminish the import of prior to their assumption of office: Provided, That they renounce their oath of
the pardon which emanated from the Office of the President and duly signed by the Chief allegiance to the country where they took that oath; (4) Those intending to practice
Executive himself/herself. The said codal provisions must be construed to harmonize the their profession in the Philippines shall apply with the proper authority for a license
power of Congress to define crimes and prescribe the penalties for such crimes and the or permit to engage in such practice; and
power of the President to grant executive clemency. All that the said provisions impart is that
the pardon of the principal penalty does notcarry with it the remission of the accessory
(5) That right to vote or be elected or appointed to any public office in the
penalties unless the President expressly includes said accessory penalties in the pardon. It
Philippines cannot be exercised by, or extended to, those who:
still recognizes the Presidential prerogative to grant executive clemency and, specifically, to
decide to pardon the principal penalty while excluding its accessory penalties or to pardon
209

(a) are candidates for or are occupying any public office in the country of hold public office has been restored, and that the penalty of perpetual absolute
which theyare naturalized citizens; and/or disqualification has been remitted.

(b) are in active service as commissioned or non commissioned officers in This is incorrect.
the armed forces of the country which they are naturalized citizens.
(Emphases supplied.) Her reliance on said opinions is utterly misplaced. Although the learned views of Justices
Teodoro R. Padilla and Florentino P. Feliciano are to be respected, they do not form partof
No less than the International Covenant on Civil and Political Rights, to which the Philippines the controlling doctrine nor to be considered part of the law of the land. On the contrary, a
is a signatory, acknowledges the existence of said right. Article 25(b) of the Convention careful reading of the majority opinion in Monsanto, penned by no less than Chief Justice
states: Article 25 Marcelo B. Fernan, reveals no statement that denotes adherence to a stringent and overly
nuanced application of Articles 36 and 41 of the Revised Penal Code that will in effect require
Every citizen shall have the right and the opportunity, without any of the distinctions the President to use a statutorily prescribed language in extending executive clemency, even
mentioned in Article 2 and without unreasonable restrictions: if the intent of the President can otherwise be deduced from the text or words used in the
pardon. Furthermore, as explained above, the pardon here is consistent with, and not
contrary to, the provisions of Articles 36 and 41.
(b) To vote and to be electedat genuine periodic elections which shall be by universal and
equal suffrage and shall be held by secret ballot, guaranteeing the free expression of the will
of the electors[.] (Emphasis supplied.) The disqualification of former President Estrada under Section 40 of the LGC in relation to
Section 12 of the OEC was removed by his acceptance of the absolute pardon granted to him.
Recently, in Sobejana-Condon v. Commission on Elections,35 the Court unequivocally referred
to the right to seek public elective office as a political right, to wit: Section 40 of the LGC identifies who are disqualified from running for any elective local
position. Risos-Vidal argues that former President Estrada is disqualified under item (a), to
wit:
Stated differently, it is an additional qualification for elective office specific only to Filipino
citizens who re-acquire their citizenship under Section 3 of R.A. No. 9225. It is the operative
act that restores their right to run for public office. The petitioner’s failure to comply there (a) Those sentenced by final judgment for an offense involving moral turpitude or for an
with in accordance with the exact tenor of the law, rendered ineffectual the Declaration of offense punishable by one (1) year or more of imprisonment, within two (2) years after
Renunciation of Australian Citizenship she executed on September 18, 2006. As such, she is serving sentence[.] (Emphasis supplied.)
yet to regain her political right to seek elective office. Unless she executes a sworn
renunciation of her Australian citizenship, she is ineligible to run for and hold any elective Likewise, Section 12 of the OEC provides for similar prohibitions, but it provides for an
office in the Philippines. (Emphasis supplied.) exception, to wit:

Thus, from both law and jurisprudence, the right to seek public elective office is Section 12. Disqualifications. – x x x unless he has been given plenary pardon or granted
unequivocally considered as a political right. Hence, the Court reiterates its earlier statement amnesty. (Emphasis supplied.)
that the pardon granted to former President Estrada admits no other interpretation other
than to mean that, upon acceptance of the pardon granted tohim, he regained his FULL civil As earlier stated, Risos-Vidal maintains that former President Estrada’s conviction for plunder
and political rights – including the right to seek elective office. disqualifies him from running for the elective local position of Mayor of the City of Manila
under Section 40(a) of the LGC. However, the subsequent absolute pardon granted to former
On the other hand, the theory of Risos-Vidal goes beyond the plain meaning of said penal President Estrada effectively restored his right to seek public elective office. This is made
provisions; and prescribes a formal requirement that is not only unnecessary but, if insisted possible by reading Section 40(a) of the LGC in relation to Section 12 of the OEC.
upon, could be in derogation of the constitutional prohibition relative to the principle that
the exercise of presidential pardon cannot be affected by legislative action. While it may be apparent that the proscription in Section 40(a) of the LGC is worded in
absolute terms, Section 12 of the OEC provides a legal escape from the prohibition – a
Risos-Vidal relied heavily on the separate concurring opinions in Monsanto v. Factoran, plenary pardon or amnesty. In other words, the latter provision allows any person who has
Jr.36 to justify her argument that an absolute pardon must expressly state that the right to been granted plenary pardon or amnesty after conviction by final judgment of an offense
210

involving moral turpitude, inter alia, to run for and hold any public office, whether local or The present dispute does not raise anything which the 20 January 2010 Resolution did not
national position. conclude upon. Here, Petitioner Risos-Vidal raised the same argument with respect to the
3rd "whereas clause" or preambular paragraph of the decree of pardon. It states that
Take notice that the applicability of Section 12 of the OEC to candidates running for local "Joseph Ejercito Estrada has publicly committed to no longer seek any elective position or
elective positions is not unprecedented. In Jalosjos, Jr. v. Commission on Elections,37 the office." On this contention, the undersigned reiterates the ruling of the Commission that the
Court acknowledged the aforementioned provision as one of the legal remedies that may be 3rd preambular paragraph does not have any legal or binding effect on the absolute nature
availed of to disqualify a candidate in a local election filed any day after the last day for filing of the pardon extended by former President Arroyo to herein Respondent. This ruling is
of certificates of candidacy, but not later than the date of proclamation.38 The pertinent consistent with the traditional and customary usage of preambular paragraphs. In the case of
ruling in the Jalosjos case is quoted as follows: Echegaray v. Secretary of Justice, the Supreme Court ruled on the legal effect of preambular
paragraphs or whereas clauses on statutes. The Court stated, viz.:
What is indisputably clear is that false material representation of Jalosjos is a ground for a
petition under Section 78. However, since the false material representation arises from a Besides, a preamble is really not an integral part of a law. It is merely an introduction to show
crime penalized by prision mayor, a petition under Section 12 ofthe Omnibus Election Code its intent or purposes. It cannot be the origin of rights and obligations. Where the meaning of
or Section 40 of the Local Government Code can also be properly filed. The petitioner has a a statute is clear and unambiguous, the preamble can neither expand nor restrict its
choice whether to anchor his petition on Section 12 or Section 78 of the Omnibus Election operation much less prevail over its text.
Code, or on Section 40 of the Local Government Code. The law expressly provides multiple
remedies and the choice of which remedy to adopt belongs to petitioner.39 (Emphasis If former President Arroyo intended for the pardon to be conditional on Respondent’s
supplied.) promise never to seek a public office again, the former ought to have explicitly stated the
same in the text of the pardon itself. Since former President Arroyo did not make this an
The third preambular clause of the pardon did not operate to make the pardon conditional. integral part of the decree of pardon, the Commission is constrained to rule that the 3rd
preambular clause cannot be interpreted as a condition to the pardon extended to former
President Estrada.42 (Emphasis supplied.)
Contrary to Risos-Vidal’s declaration, the third preambular clause of the pardon, i.e.,
"[w]hereas, Joseph Ejercito Estrada has publicly committed to no longer seek any elective
position or office," neither makes the pardon conditional, nor militate against the conclusion Absent any contrary evidence, former President Arroyo’s silence on former President
that former President Estrada’s rights to suffrage and to seek public elective office have been Estrada’s decision torun for President in the May 2010 elections against, among others, the
restored. candidate of the political party of former President Arroyo, after the latter’s receipt and
acceptance of the pardon speaks volume of her intention to restore him to his rights to
suffrage and to hold public office.
This is especially true as the pardon itself does not explicitly impose a condition or limitation,
considering the unqualified use of the term "civil and political rights"as being restored.
Jurisprudence educates that a preamble is not an essential part of an act as it is an Where the scope and import of the executive clemency extended by the President is in issue,
introductory or preparatory clause that explains the reasons for the enactment, usually the Court must turn to the only evidence available to it, and that is the pardon itself. From a
introduced by the word "whereas."40 Whereas clauses do not form part of a statute because, detailed review ofthe four corners of said document, nothing therein gives an iota of
strictly speaking, they are not part of the operative language of the statute. 41 In this case, the intimation that the third Whereas Clause is actually a limitation, proviso, stipulation or
whereas clause at issue is not an integral part of the decree of the pardon, and therefore, condition on the grant of the pardon, such that the breach of the mentioned commitment
does not by itself alone operate to make the pardon conditional or to make its effectivity not to seek public office will result ina revocation or cancellation of said pardon. To the
contingent upon the fulfilment of the aforementioned commitment nor to limit the scope of Court, what it is simply is a statement of fact or the prevailing situation at the time the
the pardon. executive clemency was granted. It was not used as a condition to the efficacy orto delimit
the scope of the pardon.
On this matter, the Court quotes with approval a relevant excerpt of COMELEC Commissioner
Maria Gracia Padaca’s separate concurring opinion in the assailed April 1, 2013 Resolution of Even if the Court were to subscribe to the view that the third Whereas Clausewas one of the
the COMELEC in SPA No. 13-211 (DC), which captured the essence of the legal effect of reasons to grant the pardon, the pardon itself does not provide for the attendant
preambular paragraphs/whereas clauses, viz: consequence of the breach thereof. This Court will be hard put to discern the resultant effect
of an eventual infringement. Just like it will be hard put to determine which civil or political
rights were restored if the Court were to take the road suggested by Risos-Vidal that the
statement "[h]e is hereby restored to his civil and political rights" excludes the restoration of
211

former President Estrada’s rights to suffrage and to hold public office. The aforequoted text In light of the foregoing, contrary to the assertions of Risos-Vidal, the COMELEC did not
ofthe executive clemency granted does not provide the Court with any guide asto how and commit grave abuse of discretion amounting to lack or excess of jurisdiction in issuing the
where to draw the line between the included and excluded political rights. assailed Resolutions.

Justice Leonen emphasizes the point that the ultimate issue for resolution is not whether the The Court has consistently held that a petition for certiorariagainst actions of the COMELEC is
pardon is contingent on the condition that former President Estrada will not seek janother confined only to instances of grave abuse of discretion amounting to patentand substantial
elective public office, but it actually concerns the coverage of the pardon – whether the denial of due process, because the COMELEC is presumed to be most competent in matters
pardon granted to former President Estrada was so expansive as to have restored all his falling within its domain.43
political rights, inclusive of the rights of suffrage and to hold public office. Justice Leonen is of
the view that the pardon in question is not absolute nor plenary in scope despite the As settled in jurisprudence, grave abuse of discretion is the arbitrary exercise of power due
statement that former President Estrada is "hereby restored to his civil and political rights," to passion, prejudice or personal hostility; or the whimsical, arbitrary, or capricious exercise
that is, the foregoing statement restored to former President Estrada all his civil and political of power that amounts to an evasion or refusal to perform a positive duty enjoined by law or
rights except the rights denied to him by the unremitted penalty of perpetual absolute to act at all in contemplation of law. For an act to be condemned as having been done with
disqualification made up of, among others, the rights of suffrage and to hold public office. He grave abuse of discretion, such an abuse must be patent and gross.44
adds that had the President chosen to be so expansive as to include the rights of suffrage and
to hold public office, she should have been more clear on her intentions.
The arguments forwarded by Risos-Vidal fail to adequately demonstrate any factual or legal
bases to prove that the assailed COMELEC Resolutions were issued in a "whimsical, arbitrary
However, the statement "[h]e is hereby restored to his civil and political rights," to the mind or capricious exercise of power that amounts to an evasion orrefusal to perform a positive
of the Court, iscrystal clear – the pardon granted to former President Estrada was absolute, duty enjoined by law" or were so "patent and gross" as to constitute grave abuse of
meaning, it was not only unconditional, it was unrestricted in scope, complete and plenary in discretion.
character, as the term "political rights"adverted to has a settled meaning in law and
jurisprudence.
On the foregoing premises and conclusions, this Court finds it unnecessary to separately
discuss Lim's petition-in-intervention, which substantially presented the same arguments as
With due respect, I disagree too with Justice Leonen that the omission of the qualifying word Risos-Vidal's petition.
"full" can be construed as excluding the restoration of the rights of suffrage and to hold
public office. There appears to be no distinction as to the coverage of the term "full political
WHEREFORE, the petition for certiorari and petition-inintervention are DISMISSED. The
rights" and the term "political rights" used alone without any qualification. How to ascribe to
Resolution dated April 1, 2013 of the Commission on Elections, Second Division, and the
the latter term the meaning that it is "partial" and not "full" defies one’s understanding.
Resolution dated April 23, 2013 of the Commission on Elections, En bane, both in SPA No. 13-
More so, it will be extremely difficult to identify which of the political rights are restored by
211 (DC), are AFFIRMED.
the pardon, when the text of the latter is silent on this matter. Exceptions to the grant of
pardon cannot be presumed from the absence of the qualifying word "full" when the pardon
restored the "political rights" of former President Estrada without any exclusion or SO ORDERED.
reservation.
Facts:
Therefore, there can be no other conclusion but to say that the pardon granted to former
President Estrada was absolute in the absence of a clear, unequivocal and concrete factual Former President Estrada was impeached and removed from presidency. He was convicted of
basis upon which to anchor or support the Presidential intent to grant a limited pardon. the crime of plunder. During the term of President Gloria Macapagal-Arroyo, she extended
an absolute pardon to herein private respondent. Estrada filed a certificate of candidacy for
To reiterate, insofar as its coverageis concerned, the text of the pardon can withstand close the position of City Mayor of Manila which was questioned by petitioner Atty. Risos-Vidal
scrutiny even under the provisions of Articles 36 and 41 of the Revised Penal Code. alleging that his conviction disqualified him from running for public office. The COMELEC took
discretionary judicial notice on Estrada’s pardon.
The COMELEC did not commit grave abuse of discretion amounting to lack or excess of
jurisdiction in issuing the assailed Resolutions.
212

Issue: Whether or not the court properly took judicial notice on the pardon of former In 2009, Estrada filed a Certificate of Candidacy for the position of President. None of the
President Joseph Ejercito Estrada. disqualification cases against him prospered but he only placed second in the results.

Ruling: In 2012, Estrada once more ventured into the political arena, and filed a Certificate of
Candidacy, this time vying for a local elective post, that of the Mayor of the City of Manila.
Yes. On the other hand, the Office of the Solicitor General (OSG) for public respondent
COMELEC, maintains that “the issue of whether or not the pardon extended to [former Petitioner Risos-Vidal filed a Petition for Disqualification against Estrada before the Comelec
President Estrada] restored his right to run for public office had already been passed upon by stating that Estrada is disqualified to run for public office because of his conviction for
public respondent COMELEC way back in 2010 via its rulings in SPA Nos. 09-024, 09-028 and plunder sentencing him to suffer the penalty of reclusion perpetua with perpetual absolute
09-104, there is no cogent reason for it to reverse its standing pronouncement and declare disqualification. Petitioner relied on Section 40 of the Local Government Code (LGC), in
[former President Estrada] disqualified to run and be voted as mayor of the City of Manila in relation to Section 12 of the Omnibus Election Code (OEC).
the absence of any new argument that would warrant its reversal. To be sure, public
respondent COMELEC correctly exercised its discretion in taking judicial cognizance of the The Comelec dismissed the petition for disqualification holding that President Estrada’s right
aforesaid rulings which are known toit and which can be verified from its own records, in to seek public office has been effectively restored by the pardon vested upon him by former
accordance with Section 2, Rule 129 of the Rules of Court on the courts’ discretionary power President Gloria M. Arroyo.
to take judicial notice of matters which are of public knowledge, or are capable of
Estrada won the mayoralty race in May 13, 2013 elections. Alfredo Lim, who garnered the
unquestionable demonstration, or ought to be known to them because of their judicial
second highest votes, intervened and sought to disqualify Estrada for the same ground as the
functions.”
contention of Risos-Vidal and praying that he be proclaimed as Mayor of Manila.
Further, the OSG contends that “[w]hile at first glance, it is apparent that [former President
Issue: May former President Joseph Estrada run for public office despite having been
Estrada’s] conviction for plunder disqualifies him from running as mayor of Manila under
convicted of the crime of plunder which carried an accessory penalty of perpetual
Section 40 of the [LGC], the subsequent grant of pardon to him, however, effectively
disqualification to hold public office?
restored his right to run for any public office.” The restoration of his right to run for any
public office is the exception to the prohibition under Section 40 of the LGC, as provided
Held:
under Section 12 of the OEC. As to the seeming requirement of Articles 36 and 41 of the
Revised Penal Code, i.e., the express restoration/remission of a particular right to be stated Yes. Estrada was granted an absolute pardon that fully restored all his civil and political
in the pardon, the OSG asserts that “an airtight and rigid interpretation of Article 36 and rights, which naturally includes the right to seek public elective office, the focal point of this
Article 41 of the [RPC] x x x would be stretching too much the clear and plain meaning of the controversy. The wording of the pardon extended to former President Estrada is complete,
aforesaid provisions.” Lastly, taking into consideration the third Whereas Clause of the unambiguous, and unqualified. It is likewise unfettered by Articles 36 and 41 of the Revised
pardon granted to former President Estrada, the OSG supports the position that it “is not an Penal Code. The only reasonable, objective, and constitutional interpretation of the language
integral part of the decree of the pardon and cannot therefore serve to restrict its of the pardon is that the same in fact conforms to Articles 36 and 41 of the Revised Penal
effectivity.” Code.

Facts: It is insisted that, since a textual examination of the pardon given to and accepted by former
President Estrada does not actually specify which political right is restored, it could be
In September 12, 2007, the Sandiganbayan convicted former President Estrada for the crime
inferred that former President Arroyo did not deliberately intend to restore former President
of plunder and was sentenced to suffer the penalty of Reclusion Perpetua and the accessory
Estrada’s rights of suffrage and to hold public office, orto otherwise remit the penalty of
penalties of civil interdiction during the period of sentence and perpetual absolute
perpetual absolute disqualification. Even if her intention was the contrary, the same cannot
disqualification. On October 25, 2007, however, former President Gloria Macapagal Arroyo
be upheld based on the pardon’s text.
extended executive clemency, by way of pardon, to former President Estrada, explicitly
stating that he is restored to his civil and political rights.
213

The pardoning power of the President cannot be limited by legislative action. involving moral turpitude, inter alia, to run for and hold any public office, whether local or
national position.
The 1987 Constitution, specifically Section 19 of Article VII and Section 5 of Article IX-C,
provides that the President of the Philippines possesses the power to grant pardons, along The third preambular clause of the pardon did not operate to make the pardon conditional.
with other acts of executive clemency, to wit:
Contrary to Risos-Vidal’s declaration, the third preambular clause of the pardon, i.e.,
Section 19. Except in cases of impeachment, or as otherwise provided in this Constitution, "[w]hereas, Joseph Ejercito Estrada has publicly committed to no longer seek any elective
the President may grant reprieves, commutations, and pardons, and remit fines and position or office," neither makes the pardon conditional, nor militate against the conclusion
forfeitures, after conviction by final judgment. that former President Estrada’s rights to suffrage and to seek public elective office have been
restored.
He shall also have the power to grant amnesty with the concurrence of a majority of all the
Members of the Congress. This is especially true as the pardon itself does not explicitly impose a condition or limitation,
considering the unqualified use of the term "civil and political rights"as being restored.
Section 5. No pardon, amnesty, parole, or suspension of sentence for violation of election Jurisprudence educates that a preamble is not an essential part of an act as it is an
laws, rules, and regulations shall be granted by the President without the favorable introductory or preparatory clause that explains the reasons for the enactment, usually
recommendation of the Commission. introduced by the word "whereas." Whereas clauses do not form part of a statute because,
strictly speaking, they are not part of the operative language of the statute. In this case, the
It is apparent from the foregoing constitutional provisions that the only instances in which
whereas clause at issue is not an integral part of the decree of the pardon, and therefore,
the President may not extend pardon remain to be in: (1) impeachment cases; (2) cases that
does not by itself alone operate to make the pardon conditional or to make its effectivity
have not yet resulted in a final conviction; and (3) cases involving violations of election laws,
contingent upon the fulfilment of the aforementioned commitment nor to limit the scope of
rules and regulations in which there was no favorable recommendation coming from the
the pardon.
COMELEC. Therefore, it can be argued that any act of Congress by way of statute cannot
operate to delimit the pardoning power of the President. Besides, a preamble is really not an integral part of a law. It is merely an introduction to show
its intent or purposes. It cannot be the origin of rights and obligations. Where the meaning of
The proper interpretation of Articles 36 and 41 of the Revised Penal Code.
a statute is clear and unambiguous, the preamble can neither expand nor restrict its
operation much less prevail over its text.
A close scrutiny of the text of the pardon extended to former President Estrada shows that
both the principal penalty of reclusion perpetua and its accessory penalties are included in
If former President Arroyo intended for the pardon to be conditional on Respondent’s
the pardon. The sentence which states that “(h)e is hereby restored to his civil and political
promise never to seek a public office again, the former ought to have explicitly stated the
rights,” expressly remitted the accessory penalties that attached to the principal penalty of
same in the text of the pardon itself. Since former President Arroyo did not make this an
reclusion perpetua. Hence, even if we apply Articles 36 and 41 of the Revised Penal Code, it is
integral part of the decree of pardon, the Commission is constrained to rule that the 3rd
indubitable from the text of the pardon that the accessory penalties of civil interdiction and
preambular clause cannot be interpreted as a condition to the pardon extended to former
perpetual absolute disqualification were expressly remitted together with the principal
President Estrada.
penalty of reclusion perpetua.
FACTS:
The disqualification of former President Estrada under Section 40 of the LGC in relation to
Section 12 of the OEC was removed by his acceptance of the absolute pardon granted to him September 12, 2007, the Sandiganbayan convicted former President Estrada, a former
President of the Republic of the Philippines, for the crime of plunder.
While it may be apparent that the proscription in Section 40(a) of the LGC is worded in
absolute terms, Section 12 of the OEC provides a legal escape from the prohibition – a October 25, 2007, however, former President Gloria Macapagal Arroyo (former President
plenary pardon or amnesty. In other words, the latter provision allows any person who has Arroyo) extended executive clemency, by way of pardon, to former President Estrada.
been granted plenary pardon or amnesty after conviction by final judgment of an offense
214

October 2, 2012, former President Estrada once more ventured into the political arena, and
filed a Certificate of Candidacy, this time vying for a local elective post, that of the Mayor of
the City of Manila.

January 24, 2013, Risos-Vidal, the petitioner in this case, filed a Petition for Disqualification
against former President Estrada before the COMELEC. Risos Vidal anchored her petition on
the theory that "Former President Estrada is Disqualified to Run for Public Office because of
his Conviction for Plunder by the Sandiganbayan Sentencing Him to Suffer the Penalty of
Reclusion Perpetua with Perpetual Absolute Disqualification."

ISSUE: Whether or not former President Estrada is qualified to vote and be voted for in public
office as a result of the pardon granted to him by former President Arroyo.

HELD:

Yes, former President Estrada is qualified to vote and be voted for in public office as a result
of the pardon granted to him by former President Arroyo. It is well-entrenched that where
the words of a statute are clear, plain, and free from ambiguity, it must be given its literal
meaning and applied without attempted interpretation. Verba legis non est recedendum.
From the words of a statute there should be no departure. It is this Court’s firm view that the
phrase in the presidential pardon at issue which declares that former President Estrada "is
hereby restored to his civil and political rights" substantially complies with the requirement
of express restoration.
215

FIRST DIVISION Conditions was not signed by PGMA; (b) consequently, the documents relative to petitioner's
case were returned to the BPP; and (c) the BPP had resolved to defer action thereon pending
G.R. No. 211269, June 15, 2016 compliance with all the basic requirements for executive clemency.

RUBEN E. TIU, Petitioner, v. HON. NATIVIDAD G. DIZON, ACTING CHAIRPERSON OF THE In the meantime, President Benigno Simeon C. Aquino III signed into law Republic Act No.
BOARD OF PARDONS AND PAROLE, HON. FRANKLIN JESUS BUCAYU, DIRECTOR OF THE (RA) 10592,18 which, subject to its provisions, would substantially increase the Good
BUREAU OF CORRECTIONS, HON. SECRETARY LEILA M. DE LIMA OF THE DEPARTMENT OF Conduct Time Allowance (GCTA) of qualified inmates. Thus, on July 27, 2013, petitioner's
JUSTICE, HON. PAQUITO N. OCHOA JR., THE EXECUTIVE SECRETARY, Respondents. carpeta was returned to the Bureau of Corrections in Muntinlupa City for the re-computation
of his time served.
DECISION
On July 7, 2014, petitioner filed the instant Amended Petition for Habeas Corpus,20 insisting
PERLAS-BERNABE, J.:
on the efficacy and enforceability of his conditional pardon without parole conditions, which
allegedly necessitates his release from prison. Further, he claims that he is entitled to
Before the Court is a petition for habeas corpus1 filed by petitioner Ruben E. Tiu (petitioner),
nineteen (19) years and seven (7) months of GCTA, computed hereafter, which, when tacked
who is detained at the Sablayan Prison and Penal Farm in Sablayan, Occidental Mindoro,
to his actual service of fourteen (14) years and nine (9) months, would add up to thirty-four
seeking his immediate release from prison on the strength of his conditional pardon without
(34) years and four (4) months, or more than his alleged reduced sentence of thirty (30)
parole conditions, as well as the automatic reduction of his sentence by virtue of his status as
years:
a penal colonist.
MONTHS
The Facts
DAYS GCTA
On June 16, 2000, petitioner and two others3 were found guilty beyond reasonable doubt by
the Regional Trial Court of Makati City, Branch 143, of selling, delivering, and giving away to a
MONTHLY GCTA
poseur-buyer 1,977 grams of methamphetamine hydrochloride, commonly known as
"shabu" a regulated drug, without authority of law or corresponding license therefor.4 01 October 1999 - 01 October 2001
Consequently, they were sentenced to suffer the penalty of reclusion perpetua and to pay
the fine of P10,000,000.00 each.5 Their conviction, which was affirmed by the Court in a 20 days
Decision6 dated March 10, 2004, became final and executory on July 29, 2004.
24 months
On March 24, 2009, the Board of Pardons and Parole (BPP) issued Resolution No. 022-3-098
recommending the grant of executive clemency to petitioner, among many others. On June 01 October 2002 - 01 October 2005
3, 2010, acting on said recommendation, then President Gloria Macapagal-Arroyo (PGMA)
23 days
granted9 him "conditional pardon without parole conditions,"10 but was, nonetheless, still
"subject to the conditions indicated in [the individual pardon papers]."11 It turned out,
36 months
however, that no such papers were issued in petitioner's favor. Thus, petitioner repeatedly
requested12 for a certificate of conditional pardon without parole conditions from the Legal 01 October 2006 - 01 October 2010
Affairs Office of the Office of the President (OP), but said requests were denied by Deputy
Executive Secretary for Legal Affairs Michael G. Aguinaldo (Deputy Executive Secretary 25 days
Aguinaldo) in three (3) separate letters dated March 13, 2013,13 August 12, 2013,14 and
August 14, 2013,15 informing petitioner that the records of his case were referred back to 178 months
the BPP. Respondent Natividad G. Dizon, Chairman of the BPP, confirmed in a letter16 dated
01 October 2011 - 01 July 2014
September 5, 2013 that: (a) petitioner's Certificate of Conditional Pardon without Parole
216

30 days prisoner shall receive the benefit of this section during the first two years of imprisonment
unless authorized by the Director of Prisons [(now Director of Corrections)] for special
44 months reasons. (Emphases and underscoring supplied)

He argues that, since he was granted a "colonist status" by then Director of Corrections Section 6, Chapter 3, Part II, Book I of the BuCor-OM
Gaudencio S. Pangilinan (Director of Corrections Pangilinan) on December 21, 2011, as
contained in Correction's Order No. 015-5-2012,22 his sentence was automatically reduced Section 6. Colonist. - The Director may, upon the recommendation of the Classification
to thirty (30) years23 pursuant to Section 7 (b), Chapter 3, Part II, Book I of the Bureau of Board, classify an inmate who has the following qualifications as a colonist:
Corrections Operating Manual (BuCor-OM), the pertinent portions of which read as follows:
be at least a first class inmate and has served one (1) year immediately preceding the
SECTION 7. Privileges of a colonist. - A colonist shall have the following privileges: completion of the period specified in the following qualifications;

credit of an additional GCTA of five (5) days for each calendar month while he retains said has served imprisonment with good conduct for a period equivalent to one fifth (1/5) of the
classification aside from the regular GCTA authorized under Article 97 of the Revised Penal maximum term of his prison sentence, or seven (7) years in the case of a life sentence.
Code;
Finally, petitioner invokes Section 527 of RA 10592, which provides that the time allowances
automatic reduction of the life sentence imposed on the colonist to a sentence of thirty (30) for good conduct once granted shall not be revoked.28 He further proposes that RA 10592 be
years; given retroactive effect in light of the liberal construction provided for in the rules to favor
detained or convicted prisoners like him.
To bolster his claim of reduction of sentence, petitioner cites24 Sections 5 and 7 of Act No.
2489,25cralawred which provide for automatic modification of sentence from life On the other hand, herein respondents, through the Office of the Solicitor General (OSG),
imprisonment to thirty (30) years for prisoners receiving and retaining the classification of maintain30 that a prisoner serving a sentence of life imprisonment receiving and retaining
penal colonists or trusties. He theorizes26 that, although said law requires executive classification as a penal colonist will automatically have his sentence modified to thirty (30)
approval for such classification, his colonist status was nonetheless "regularly awarded" by years of imprisonment only "when receiving the executive approval for this classification."31
the Director of Corrections whose authority to so classify him as such is derived from Section However, petitioner failed to obtain such executive approval. They argue further against
6, Chapter 3, Part II, Book I of the BuCor-OM. The aforementioned provisions read: petitioner's reliance on the BuCor-OM, which is a mere administrative rule or regulation that
cannot amend Act No. 2489 by abridging or expanding its scope.32 Petitioner's colonist
Provisions in Act No. 2489 status granted merely by the Director of Corrections, without executive approval, did not
modify his sentence.33 Hence, there being no unlawful restraint, no writ of habeas corpus
Section 5. Prisoners serving sentences of life imprisonment receiving and retaining the
should be issued in his favor.
classification of penal colonists or trusties will automatically have the sentence of life
imprisonment modified to a sentence of thirty years when receiving the executive approval The Issue Before the Court
for this classification upon which the regular credit now authorized by law and special credit
authorized in the preceding paragraph, for good conduct, may be made. The essential issue for the Court's resolution is whether or not a writ of habeas corpus should
be issued in favor of petitioner.
Section 7. The provisions of this Act as applied in the case of penal colonists and trusties may,
by executive approval and upon recommendation of the Director of Prisons [(now Director of The Court's Ruling
Corrections)], be made applicable to all first-class workmen confined in Bilibid Prison who
have earned the privilege of classification as penal colonists or trusties by serving one-fifth of The petition lacks merit.
the time sentence as imposed by the court, or seven years in the case of a life-sentenced
The object of the writ of habeas corpus is to inquire into the legality of the detention, and, if
prisoner, in addition to the compensation allowed, if any of such first-class workmen shall by
the detention is found to be illegal, to require the release of the detainee. Well-settled is the
written petition elect to remain in the industrial division at Bilibid Prison: Provided, That no
rule that the writ will not issue where the person in whose behalf the writ is sought is in the
217

custody of an officer under process issued by a court or judge with jurisdiction or by virtue of not, and could not, resolve here. This is because the grant of pardon and the determination
a judgment or order of a court of record.34 The writ is denied if the petitioner fails to show of the terms and conditions of a conditional pardon are purely executive acts which are not
facts that he is entitled thereto ex merito justicias. subject to judicial scrutiny.

In this case, petitioner is serving sentence by virtue of a final judgment convicting him of the Second. As correctly argued by the OSG, the conferment by the Director of Corrections of a
offense of selling and delivering prohibited drugs defined and penalized under Section 15, colonist status to petitioner did not operate to reduce the latter's sentence. Section 5 of Act
Article III of RA 6425,36 as amended by RA 7659.37 He failed to show, however, that his No. 2489 is clear and unambiguous: "[p]risoners serving sentences of life imprisonment
further incarceration is no longer lawful and that he is entitled to relief under a writ of receiving and retaining the classification of penal colonists or trusties will automatically have
habeas corpus. the sentence of life imprisonment modified to a sentence of thirty years when receiving the
executive approval for this classification upon which the regular credit now authorized by law
First. Petitioner's insistence on the efficacy and enforceability of the conditional pardon and special credit authorized in the preceding paragraph, for good conduct, may be made."
without parole conditions granted to him by PGMA on June 3, 2010 deserves scant
consideration. The wording of the law is such that the act of classification as a penal colonist or trustie is
separate from and necessarily precedes the act of approval by the Executive. Under Section
It must be emphasized that pardon is an act of grace, proceeding from the power entrusted 6, Chapter 3, Part II, Book I of the BuCor-OM quoted earlier, the Director of Corrections may,
with the execution of the laws, which exempts the individual, on whom it is bestowed, from upon the recommendation of the Classification Board45 of the Bureau of Corrections, classify
the punishment the law inflicts for a crime he has committed. It is the private, though official an inmate as a colonist. It is crucial, however, that the prisoner not only receives, but retains
act of the executive magistrate, delivered to the individual for whose benefit it is intended such classification, because the grant of a colonist status may, for cause, be revoked at any
and not communicated officially to the court. A pardon is a deed, to the validity of which time by the Superintendent with the approval of the Director of Corrections pursuant to
delivery is essential. Section 946 of the same Chapter. It is the classification of the penal colonist and trustie of the
Director of Corrections which subsequently receives executive approval.
The executive clemency extended by PGMA on June 3, 2010 to a number of prisoners
including petitioner was made "subject to the conditions indicated in the corresponding The foregoing is bolstered by the fact that the reduction of a prisoner's sentence is a partial
documents."39 It is undisputed, however, that no individual pardon papers were issued in pardon,47and our Constitution reposes in the President the power and the exclusive
petitioner's favour, thereby rendering the grant of executive clemency to him as incomplete prerogative to extend the same.48 The 1987 Constitution, specifically under Section 19,
and ineffective, as clarified by Deputy Executive Secretary Aguinaldo.40 The necessity for the Article VII thereof, provides that the President possesses the power to grant pardons, along
individual pardon papers is best explained by the nature of a conditional pardon, which is "a with other acts of executive clemency,49 which petitioner explicitly recognized by applying
contract between the sovereign power or the Chief Executive and the convicted criminal to for commutation of sentence even during the pendency of his request for the
the effect that the former will release the latter subject to the condition that if he does not implementation of the conditional pardon.50 Section 19, Article VII of the 1987 Constitution
comply with the terms of the pardon, he will be recommitted to prison to serve the reads:
unexpired portion of the sentence or an additional one. By the pardonee's consent to the
terms stipulated in this contract, the pardonee has thereby placed himself under the Section 19. Except in cases of impeachment, or as otherwise provided in this Constitution,
supervision of the Chief Executive or his delegate who is duty-bound to see to it that the the President may grant reprieves, commutations, and pardons, and remit fines and
pardonee complies with the terms and conditions of the pardon."41 The individual pardon forfeitures, after conviction by final judgment.
papers, therefore, contain the terms and conditions of the contract of pardon, the
compliance of which is essential to the pardonee's freedom from recommitment to prison. He shall also have the power to grant amnesty with the concurrence of a majority of all the
Members of the Congress.
Notably, when the records of petitioner's case were referred back to the BPP, it required
compliance first with all the basic requirements for executive clemency before acting It has long been recognized that the exercise of the pardoning power, notwithstanding the
thereon.42 This is not to say, however, that petitioner's pardon papers may not have been judicial determination of guilt of the accused, demands the exclusive exercise by the
issued due to non-compliance with the requirements, which is a matter that the Court shall President of the constitutionally vested power.51 Stated otherwise, since the Chief Executive
218

is required by the Constitution to act in person, he may not delegate the authority to pardon Petitioners challenged the constitutionality of EO 97-A for allegedly being violative of their
prisoners under the doctrine of qualified political agency, which "essentially postulates that right to equal protection of the laws. This was due to the limitation of tax incentives to Subic
the heads of the various executive departments are the alter egos of the President, and, and not to the entire area of Olongapo. The case was referred to the Court of Appeals.
thus, the actions taken by such heads in the performance of their official duties are deemed
the acts of the President unless the President himself should disapprove such acts." The appellate court concluded that such being the case, petitioners could not claim that EO
97-A is unconstitutional, while at the same time maintaining the validity of RA 7227.
In sum, there being no unlawful restraint on petitioner's liberty, no relief under a writ of
habeas corpus can be granted to him. The court a quo also explained that the intention of Congress was to confine the coverage of
the SSEZ to the "secured area" and not to include the "entire Olongapo City and other areas
WHEREFORE, the petition is DISMISSED. mentioned in Section 12 of the law.

SO ORDERED. Hence, this was a petition for review under Rule 45 of the Rules of Court.

Facts: Issue: Whether the provisions of Executive Order No. 97-A confining the application of R.A.
7227 within the secured area and excluding the residents of the zone outside of the secured
On March 13, 1992, Congress, with the approval of the President, passed into law RA 7227. area is discriminatory or not owing to a violation of the equal protection clause.
This was for the conversion of former military bases into industrial and commercial uses.
Subic was one of these areas. It was made into a special economic zone.

In the zone, there were no exchange controls. Such were liberalized. There was also tax Held:
incentives and duty free importation policies under this law.
No. Petition dismissed.
On June 10, 1993, then President Fidel V. Ramos issued Executive Order No. 97 (EO 97),
clarifying the application of the tax and duty incentives. It said that Ratio:

On Import Taxes and Duties. — Tax and duty-free importations shall apply only to raw Citing Section 12 of RA 7227, petitioners contend that the SSEZ encompasses (1) the City of
materials, capital goods and equipment brought in by business enterprises into the SSEZ Olongapo, (2) the Municipality of Subic in Zambales, and (3) the area formerly occupied by
the Subic Naval Base. However, they claimed that the E.O. narrowed the application to the
On All Other Taxes. — In lieu of all local and national taxes (except import taxes and duties), naval base only.
all business enterprises in the SSEZ shall be required to pay the tax specified in Section 12(c)
of R.A. No. 7227. OSG- The E.O. Was a valid classification.

Nine days after, on June 19, 1993, the President issued Executive Order No. 97-A (EO 97-A), Court- The fundamental right of equal protection of the laws is not absolute, but is subject to
specifying the area within which the tax-and-duty-free privilege was operative. reasonable classification. If the groupings are characterized by substantial distinctions that
make real differences, one class may be treated and regulated differently from another. The
Section 1.1. The Secured Area consisting of the presently fenced-in former Subic Naval classification must also be germane to the purpose of the law and must apply to all those
Base shall be the only completely tax and duty-free area in the SSEFPZ. Business enterprises belonging to the same class.
and individuals (Filipinos and foreigners) residing within the Secured Area are free to import
raw materials, capital goods, equipment, and consumer items tax and duty-free. Inchong v Hernandez- Equal protection does not demand absolute equality among residents;
it merely requires that all persons shall be treated alike, under like circumstances and
conditions both as to privileges conferred and liabilities enforced.
219

Classification, to be valid, must (1) rest on substantial distinctions, (2) be germane to the
purpose of the law, (3) not be limited to existing conditions only, and (4) apply equally to all
members of the same class.

RA 7227 aims primarily to accelerate the conversion of military reservations into productive
uses. This was really limited to the military bases as the law's intent provides. Moreover, the
law tasked the BCDA to specifically develop the areas the bases occupied.

Among such enticements are: (1) a separate customs territory within the zone, (2) tax-and-
duty-free importations, (3) restructured income tax rates on business enterprises within the
zone, (4) no foreign exchange control, (5) liberalized regulations on banking and finance, and
(6) the grant of resident status to certain investors and of working visas to certain foreign
executives and workers. The target of the law was the big investor who can pour in capital.

Even more important, at this time the business activities outside the "secured area" are not
likely to have any impact in achieving the purpose of the law, which is to turn the former
military base to productive use for the benefit of the Philippine economy. Hence, there was
no reasonable basis to extend the tax incentives in RA 7227.

It is well-settled that the equal-protection guarantee does not require territorial uniformity
of laws. As long as there are actual and material differences between territories, there is no
violation of the constitutional clause.

Besides, the businessmen outside the zone can always channel their capital into it.

RA 7227, the objective is to establish a "self-sustaining, industrial, commercial, financial and


investment center”. There will really be differences between it and the outside zone of
Olongapo.

The classification of the law also applies equally to the residents and businesses in the zone.
They are similarly treated to contribute to the end gaol of the law.
220

EN BANC | August 15, 2006 Instead, the Court has to resolve whether petitioners may be subjected to military discipline
on account of their defiance of a direct order of the AFP Chief of Staff.
B/GEN. (RET.) FRANCISCO V. G.R. No. 170165
The solicited writs of certiorari and prohibition do not avail; the petition must be denied.
GUDANI AND LT. COL.Petitioners.
I.
- versus -
The petitioners are high-ranking officers of the Armed Forces of the Philippines (AFP). Both
LT./GEN. GENEROSO S. SENGA CORONA, petitioners, Brigadier General Francisco Gudani (Gen. Gudani) and Lieutenant Colonel
Alexander Balutan (Col. Balutan), belonged to the Philippine Marines. At the time of the
Respondents.
subject incidents, both Gen. Gudani and Col. Balutan were assigned to the Philippine Military
Academy (PMA) in Baguio City, the former as the PMA Assistant Superintendent, and the
DECISION
latter as the Assistant Commandant of Cadets.[2]
TINGA, J.:
On 22 September 2005, Senator Rodolfo Biazon (Sen. Biazon) invited several senior officers
A most dangerous general proposition is foisted on the Court that soldiers who defy orders of of the AFP to appear at a public hearing before the Senate Committee on National Defense
their superior officers are exempt from the strictures of military law and discipline if such and Security (Senate Committee) scheduled on 28 September 2005. The hearing was
defiance is predicated on an act otherwise valid under civilian law. Obedience and deference scheduled after topics concerning the conduct of the 2004 elections emerged in the public
to the military chain of command and the President as commander-in-chief are the eye, particularly allegations of massive cheating and the surfacing of copies of an audio
cornerstones of a professional military in the firm cusp of civilian control. These values of excerpt purportedly of a phone conversation between President Gloria Macapagal Arroyo
obedience and deference expected of military officers are content-neutral, beyond the sway and an official of the Commission on Elections (COMELEC) widely reputed as then COMELEC
of the officers own sense of what is prudent or rash, or more elementally, of right or wrong. Commissioner Virgilio Garcillano. At the time of the 2004 elections, Gen. Gudani had been
A self-righteous military invites itself as the scoundrels activist solution to the ills of designated as commander, and Col. Balutan a member, of Joint Task Force Ranao by the AFP
participatory democracy. Southern Command. Joint Task Force Ranao was tasked with the maintenance of peace and
order during the 2004 elections in the provinces of Lanao del Norte and Lanao del Sur.[3] `
Petitioners seek the annulment of a directive from President Gloria Macapagal-Arroyo[1]
enjoining them and other military officers from testifying before Congress without the Gen. Gudani, Col. Balutan, and AFP Chief of Staff Lieutenant General Generoso Senga (Gen.
Presidents consent. Petitioners also pray for injunctive relief against a pending preliminary Senga) were among the several AFP officers who received a letter invitation from Sen. Biazon
investigation against them, in preparation for possible court-martial proceedings, initiated to attend the 28 September 2005 hearing. On 23 September 2005, Gen. Senga replied
within the military justice system in connection with petitioners violation of the through a letter to Sen. Biazon that he would be unable to attend the hearing due to a
aforementioned directive. previous commitment in Brunei, but he nonetheless directed other officers from the AFP
who were invited to attend the hearing.[4]
Court is cognizant that petitioners, in their defense, invoke weighty constitutional principles
that center on fundamental freedoms enshrined in the Bill of Rights. Although these On 26 September 2005, the Office of the Chief of Staff of the AFP issued a Memorandum
concerns will not be addressed to the Superintendent of the PMA Gen. Cristolito P. Baloing (Gen. Baloing). It was
signed by Lt. Col. Hernando DCA Iriberri in behalf of Gen. Senga.[5] Noting that Gen. Gudani
addressed to the satisfaction of petitioners, the Court recognizes these values as of and Col. Balutan had been invited to attend the Senate Committee hearing on 28 September
paramount importance to our civil society, even if not determinative of the resolution of this 2005, the Memorandum directed the two officers to attend the hearing.[6] Conformably,
petition. Had the relevant issue before us been the right of the Senate to compel the Gen. Gudani and Col. Balutan filed their respective requests for travel authority addressed to
testimony of petitioners, the constitutional questions raised by them would have come to the PMA Superintendent.
fore. Such a scenario could have very well been presented to the Court in such manner,
without the petitioners having had to violate a direct order from their commanding officer.
221

On 27 September 2005, Gen. Senga wrote a letter to Sen. Biazon, requesting the On the very day of the hearing, 28 September 2005, President Gloria-Macapagal-Arroyo
postponement of the hearing scheduled for the following day, since the AFP Chief of Staff issued Executive Order No. 464 (E.O. 464). The OSG notes that the E.O. enjoined officials of
was himself unable to attend said hearing, and that some of the invited officers also could the executive department including the military establishment from appearing in any
not attend as they were attending to other urgent operational matters. By this time, both legislative inquiry without her approval.[10] This Court subsequently ruled on the
Gen. Gudani and Col. Balutan had already departed Baguio for Manila to attend the hearing. constitutionality of the said executive order in Senate v. Ermita.[11] The relevance of E.O.
464 and Senate to the present petition shall be discussed forthwith.
Then on the evening of 27 September 2005, at around 10:10 p.m., a message was
transmitted to the PMA Superintendent from the office of Gen. Senga, stating as follows: In the meantime, on 30 September 2005, petitioners were directed by General Senga,
through Col. Henry A. Galarpe of the AFP Provost Marshal General, to appear before the
PER INSTRUCTION OF HER EXCELLENCY PGMA, NO AFP PERSONNEL SHALL APPEAR BEFORE Office of the Provost Marshal General (OPMG) on 3 October 2005 for investigation. During
ANY CONGRESSIONAL OR SENATE HEARING WITHOUT HER APPROVAL. INFORM BGEN their appearance before Col. Galarpe, both petitioners invoked their right to remain
FRANCISCO F GUDANI AFP AND LTC ALEXANDER BALUTAN PA (GSC) ACCORDINGLY.[7] silent.[12] The following day, Gen. Gudani was compulsorily retired from military service,
having reached the age of 56.[13]
The following day, Gen. Senga sent another letter to Sen. Biazon, this time informing the
senator that no approval has been granted by the President to any AFP officer to appear In an Investigation Report dated 6 October 2005, the OPMG recommended that petitioners
before the hearing scheduled on that day. Nonetheless, both Gen. Gudani and Col. Balutan be charged with violation of Article of War 65, on willfully disobeying a superior officer, in
were present as the hearing started, and they both testified as to the conduct of the 2004 relation to Article of War 97, on conduct prejudicial to the good order and military
elections. discipline.[14] As recommended, the case was referred to a Pre-Trial Investigation Officer
(PTIO) preparatory to trial by the General Court Martial (GCM).[15] Consequently, on 24
The Office of the Solicitor General (OSG), representing the respondents before this Court, has
October 2005, petitioners were separately served with Orders respectively addressed to
offered additional information surrounding the testimony of Gen. Gudani and Col. Balutan.
them and signed by respondent Col. Gilbert Jose C. Roa, the Pre-Trial Investigating Officer of
The OSG manifests that the couriers of the AFP Command Center had attempted to deliver
the PTIO. The Orders directed petitioners to appear in person before Col. Roa at the Pre-Trial
the radio message to Gen. Gudanis residence in a subdivision in Paraaque City late in the
Investigation of the Charges for violation of Articles 65[16] and 97[17] of Commonwealth Act
night of 27 September 2005, but they were not permitted entry by the subdivision guards.
No. 408,[18] and to submit their counter-affidavits and affidavits of witnesses at the Office of
The next day, 28 September 2005, shortly before the start of the hearing, a copy of Gen.
the Judge Advocate General.[19] The Orders were accompanied by respective charge sheets
Sengas letter to Sen. Biazon sent earlier that day was handed at the Senate by Commodore
against petitioners, accusing them of violating Articles of War 65 and 97.
Amable B. Tolentino of the AFP Office for Legislative Affairs to Gen. Gudani, who replied that
he already had a copy. Further, Gen. Senga called Commodore Tolentino on the latters cell It was from these premises that the present petition for certiorari and prohibition was filed,
phone and asked to talk to Gen. Gudani, but Gen. Gudani refused. In response, Gen. Senga particularly seeking that (1) the order of President Arroyo coursed through Gen. Senga
instructed Commodore Tolentino to inform Gen. Gudani that it was an order, yet Gen. preventing petitioners from testifying before Congress without her prior approval be
Gudani still refused to take Gen. Sengas call.[8] declared unconstitutional; (2) the charges stated in the charge sheets against petitioners be
quashed; and (3) Gen. Senga, Col. Galarpe, Col. Roa, and their successors-in-interest or
A few hours after Gen. Gudani and Col. Balutan had concluded their testimony, the office of
persons acting for and on their behalf or orders, be permanently enjoined from proceeding
Gen. Senga issued a statement which noted that the two had appeared before the Senate
against petitioners, as a consequence of their having testified before the Senate on 28
Committee in spite of the fact that a guidance has been given that a Presidential approval
September 2005.[20]
should be sought prior to such an appearance; that such directive was in keeping with the
time[-]honored principle of the Chain of Command; and that the two officers disobeyed a Petitioners characterize the directive from President Arroyo requiring her prior approval
legal order, in violation of A[rticles of] W[ar] 65 (Willfully Disobeying Superior Officer), hence before any AFP personnel appear before Congress as a gag order, which violates the principle
they will be subjected to General Court Martial proceedings x x x Both Gen. Gudani and Col. of separation of powers in government as it interferes with the investigation of the Senate
Balutan were likewise relieved of their assignments then.[9] Committee conducted in aid of legislation. They also equate the gag order with culpable
violation of the Constitution, particularly in relation to the publics constitutional right to
222

information and transparency in matters of public concern. Plaintively, petitioners claim that personnel of whatever rank are liable under military law for violating a direct order of an
the Filipino people have every right to hear the [petitioners] testimonies, and even if the gag officer superior in rank. Whether petitioners did violate such an order is not for the Court to
order were unconstitutional, it still was tantamount to the crime of obstruction of justice. decide, but it will be necessary to assume, for the purposes of this petition, that petitioners
Petitioners further argue that there was no law prohibiting them from testifying before the did so.
Senate, and in fact, they were appearing in obeisance to the authority of Congress to conduct
inquiries in aid of legislation. III.

Finally, it is stressed in the petition that Gen. Gudani was no longer subject to military Preliminarily, we must discuss the effect of E.O. 464 and the Courts ruling in Senate on the
jurisdiction on account of his compulsory retirement on 4 October 2005. It is pointed out that present petition. Notably, it is not alleged that petitioners were in any way called to task for
Article 2, Title I of the Articles of War defines persons subject to military law as all officers violating E.O. 464, but instead, they were charged for violating the direct order of Gen. Senga
and soldiers in the active service of the AFP. not to appear before the Senate Committee, an order that stands independent of the
executive order. Distinctions are called for, since Section 2(b) of E.O. 464 listed generals and
II. flag officers of the Armed Forces of the Philippines and such other officers who in the
judgment of the Chief of Staff are covered by the executive privilege, as among those public
We first proceed to define the proper litigable issues. Notably, the guilt or innocence of officials required in Section 3 of E.O. 464 to secure prior consent of the President prior to
petitioners in violating Articles 65 and 97 of the Articles of War is not an issue before this appearing before either House of Congress. The Court in Senate declared both Section 2(b)
Court, especially considering that per records, petitioners have not yet been subjected to and Section 3 void,[24] and the impression may have been left following Senate that it
court martial proceedings. Owing to the absence of such proceedings, the correct inquiry settled as doctrine, that the President is prohibited from requiring military personnel from
should be limited to whether respondents could properly initiate such proceedings attending congressional hearings without having first secured prior presidential consent.
preparatory to a formal court-martial, such as the aforementioned preliminary investigation, That impression is wrong.
on the basis of petitioners acts surrounding their testimony before the Senate on 28
September 2005. Yet this Court, consistent with the principle that it is not a trier of facts at Senate turned on the nature of executive privilege, a presidential prerogative which is
first instance,[21] is averse to making any authoritative findings of fact, for that function is encumbered by significant limitations. Insofar as E.O. 464 compelled officials of the executive
first for the court-martial court to fulfill. branch to seek prior presidential approval before appearing before Congress, the notion of
executive control also comes into consideration.[25] However, the ability of the President to
Thus, we limit ourselves to those facts that are not controverted before the Court, having require a military official to secure prior consent before appearing before Congress pertains
been commonly alleged by petitioners and the OSG (for respondents). Petitioners were to a wholly different and independent specie of presidential authoritythe commander-in-
called by the Senate Committee to testify in its 28 September 2005 hearing. Petitioners chief powers of the President. By tradition and jurisprudence, the commander-in-chief
attended such hearing and testified before the Committee, despite the fact that the day powers of the President are not encumbered by the same degree of restriction as that which
before, there was an order from Gen. Senga (which in turn was sourced per instruction from may attach to executive privilege or executive control.
President Arroyo) prohibiting them from testifying without the prior approval of the
President. Petitioners do not precisely admit before this Court that they had learned of such During the deliberations in Senate, the Court was very well aware of the pendency of this
order prior to their testimony, although the OSG asserts that at the very least, Gen. Gudani petition as well as the issues raised herein. The decision in Senate was rendered with the
already knew of such order before he testified.[22] Yet while this fact may be ultimately comfort that the nullification of portions of E.O. 464 would bear no impact on the present
material in the court-martial proceedings, it is not determinative of this petition, which as petition since petitioners herein were not called to task for violating the executive order.
stated earlier, does not proffer as an issue whether petitioners are guilty of violating the Moreover, the Court was then cognizant that Senate and this case would ultimately hinge on
Articles of War. disparate legal issues. Relevantly, Senate purposely did not touch upon or rule on the faculty
of the President, under the aegis of the commander-in-chief powers[26] to require military
What the Court has to consider though is whether the violation of the aforementioned order officials from securing prior consent before appearing before Congress. The pertinent factors
of Gen. Senga, which emanated from the President, could lead to any investigation for court- in considering that question are markedly outside of those which did become relevant in
martial of petitioners. It has to be acknowledged as a general principle[23] that AFP adjudicating the issues raised in Senate. It is in this petition that those factors come into play.
223

At this point, we wish to dispose of another peripheral issue before we strike at the heart of [a]n officer or enlisted man carried in the retired list [of the Armed Forces of the Philippines]
the matter. General Gudani argues that he can no longer fall within the jurisdiction of the shall be subject to the Articles of War x x x[30] To this citation, petitioners do not offer any
court-martial, considering his retirement last 4 October 2005. He cites Article 2, Title I of response, and in fact have excluded the matter of Gen. Gudanis retirement as an issue in
Commonwealth Act No. 408, which defines persons subject to military law as, among others, their subsequent memorandum.
all officers and soldiers in the active service of the [AFP], and points out that he is no longer
in the active service. IV.

This point was settled against Gen. Gudanis position in Abadilla v. Ramos,[27] where the We now turn to the central issues.
Court declared that an officer whose name was dropped from the roll of officers cannot be
Petitioners wish to see annulled the gag order that required them to secure presidential
considered to be outside the jurisdiction of military authorities when military justice
consent prior to their appearance before the Senate, claiming that it violates the
proceedings were initiated against him before the termination of his service. Once
constitutional right to information and transparency in matters of public concern; or if not, is
jurisdiction has been acquired over the officer, it continues until his case is terminated. Thus,
tantamount at least to the criminal acts of obstruction of justice and grave coercion.
the Court held:
However, the proper perspective from which to consider this issue entails the examination of
The military authorities had jurisdiction over the person of Colonel Abadilla at the time of the the basis and authority of the President to issue such an order in the first place to members
alleged offenses. This jurisdiction having been vested in the military authorities, it is retained of the AFP and the determination of whether such an order is subject to any limitations.
up to the end of the proceedings against Colonel Abadilla. Well-settled is the rule that
The vitality of the tenet that the President is the commander-in-chief of the Armed Forces is
jurisdiction once acquired is not lost upon the instance of the parties but continues until the
most crucial to the democratic way of life, to civilian supremacy over the military, and to the
case is terminated.[28]
general stability of our representative system of government. The Constitution reposes final
Citing Colonel Winthrops treatise on Military Law, the Court further stated: authority, control and supervision of the AFP to the President, a civilian who is not a member
of the armed forces, and whose duties as commander-in-chief represent only a part of the
We have gone through the treatise of Colonel Winthrop and We find the following passage organic duties imposed upon the office, the other functions being clearly civil in nature.[31]
which goes against the contention of the petitioners, viz Civilian supremacy over the military also countermands the notion that the military may
bypass civilian authorities, such as civil courts, on matters such as conducting warrantless
3. Offenders in general Attaching of jurisdiction. It has further been held, and is now settled searches and seizures.[32]
law, in regard to military offenders in general, that if the military jurisdiction has once duly
attached to them previous to the date of the termination of their legal period of service, they Pursuant to the maintenance of civilian supremacy over the military, the Constitution has
may be brought to trial by court-martial after that date, their discharge being meanwhile allocated specific roles to the legislative and executive branches of government in relation to
withheld. This principle has mostly been applied to cases where the offense was committed military affairs. Military appropriations, as with all other appropriations, are determined by
just prior to the end of the term. In such cases the interests of discipline clearly forbid that Congress, as is the power to declare the existence of a state of war.[33] Congress is also
the offender should go unpunished. It is held therefore that if before the day on which his empowered to revoke a proclamation of martial law or the suspension of the writ of habeas
service legally terminates and his right to a discharge is complete, proceedings with a view to corpus.[34] The approval of the Commission on Appointments is also required before the
trial are commenced against him as by arrest or the service of charges, the military President can promote military officers from the rank of colonel or naval captain.[35]
jurisdiction will fully attach and once attached may be continued by a trial by court-martial Otherwise, on the particulars of civilian dominance and administration over the military, the
ordered and held after the end of the term of the enlistment of the accused x x x [29] Constitution is silent, except for the commander-in-chief clause which is fertile in meaning
and implication as to whatever inherent martial authority the President may possess.[36]
Thus, military jurisdiction has fully attached to Gen. Gudani inasmuch as both the acts
complained of and the initiation of the proceedings against him occurred before he The commander-in-chief provision in the Constitution is denominated as Section 18, Article
compulsorily retired on 4 October 2005. We see no reason to unsettle the Abadilla doctrine. VII, which begins with the simple declaration that [t]he President shall be the Commander-in-
The OSG also points out that under Section 28 of Presidential Decree No. 1638, as amended, Chief of all armed forces of the Philippines x x x[37] Outside explicit constitutional limitations,
224

such as those found in Section 5, Article XVI, the commander-in-chief clause vests on the separate from civilian society. [43] In the elegant prose of the eminent British military
President, as commander-in-chief, absolute authority over the persons and actions of the historian, John Keegan:
members of the armed forces. Such authority includes the ability of the President to restrict
the travel, movement and speech of military officers, activities which may otherwise be [Warriors who fight wars have] values and skills [which] are not those of politicians and
sanctioned under civilian law. diplomats. They are those of a world apart, a very ancient world, which exists in parallel with
the everyday world but does not belong to it. Both worlds change over time, and the warrior
Reference to Kapunan, Jr. v. De Villa[38] is useful in this regard. Lt. Col. Kapunan was ordered world adopts in step to the civilian. It follows it, however, at a distance. The distance can
confined under house arrest by then Chief of Staff (later President) Gen. Fidel Ramos. never be closed, for the culture of the warrior can never be that of civilization itself.[44]
Kapunan was also ordered, as a condition for his house arrest, that he may not issue any
press statements or give any press conference during his period of detention. The Court Critical to military discipline is obeisance to the military chain of command. Willful
unanimously upheld such restrictions, noting: disobedience of a superior officer is punishable by court-martial under Article 65 of the
Articles of War.[45] An individual soldier is not free to ignore the lawful orders or duties
[T]he Court is of the view that such is justified by the requirements of military discipline. It assigned by his immediate superiors. For there would be an end of all discipline if the seaman
cannot be gainsaid that certain liberties of persons in the military service, including the and marines on board a ship of war [or soldiers deployed in the field], on a distant service,
freedom of speech, may be circumscribed by rules of military discipline. Thus, to a certain were permitted to act upon their own opinion of their rights [or their opinion of the
degree, individual rights may be curtailed, because the effectiveness of the military in
fulfilling its duties under the law depends to a large extent on the maintenance of discipline Presidents intent], and to throw off the authority of the commander whenever they
within its ranks. Hence, lawful orders must be followed without question and rules must be supposed it to be unlawfully exercised.[46]
faithfully complied with, irrespective of a soldier's personal views on the matter. It is from
Further traditional restrictions on members of the armed forces are those imposed on free
this viewpoint that the restrictions imposed on petitioner Kapunan, an officer in the AFP,
speech and mobility. Kapunan is ample precedent in justifying that a soldier may be
have to be considered.[39]
restrained by a superior officer from speaking out on certain matters. As a general rule, the
Any good soldier, or indeed any ROTC cadet, can attest to the fact that the military way of life discretion of a military officer to restrain the speech of a soldier under his/her command will
circumscribes several of the cherished freedoms of civilian life. It is part and parcel of the be accorded deference, with minimal regard if at all to the reason for such restraint. It is
military package. Those who cannot abide by these limitations normally do not pursue a integral to military discipline that the soldiers speech be with the consent and approval of
military career and instead find satisfaction in other fields; and in fact many of those the military commander.
discharged from the service are inspired in their later careers precisely by their rebellion
The necessity of upholding the ability to restrain speech becomes even more imperative if
against the regimentation of military life. Inability or unwillingness to cope with military
the soldier desires to speak freely on political matters. The Constitution requires that [t]he
discipline is not a stain on character, for the military mode is a highly idiosyncratic path which
armed forces shall be insulated from partisan politics, and that [n]o member of the military
persons are not generally conscripted into, but volunteer themselves to be part of. But for
shall engage directly or indirectly in any partisan political activity, except to vote.[47]
those who do make the choice to be a soldier, significant concessions to personal freedoms
Certainly, no constitutional provision or military indoctrination will eliminate a soldiers ability
are expected. After all, if need be, the men and women of the armed forces may be
to form a personal political opinion, yet it is vital that such opinions be kept out of the public
commanded upon to die for country, even against their personal inclinations.
eye. For one, political belief is a potential source of discord among people, and a military torn
It may be so that military culture is a remnant of a less democratic era, yet it has been fully by political strife is incapable of fulfilling its constitutional function as protectors of the
integrated into the democratic system of governance. The constitutional role of the armed people and of the State. For another, it is ruinous to military discipline to foment an
forces is as protector of the people and of the State.[40] Towards this end, the military must atmosphere that promotes an active dislike of or dissent against the President, the
insist upon a respect for duty and a discipline without counterpart in civilian life.[41] The commander-in-chief of the armed forces. Soldiers are constitutionally obliged to obey a
laws and traditions governing that discipline have a long history; but they are founded on President they may dislike or distrust. This fundamental principle averts the country from
unique military exigencies as powerful now as in the past.[42] In the end, it must be borne in going the way of banana republics.
mind that the armed forces has a distinct subculture with unique needs, a specialized society
225

Parenthetically, it must be said that the Court is well aware that our countrys recent past is V.
marked by regime changes wherein active military dissent from the chain of command
formed a key, though not exclusive, element. The Court is not blind to history, yet it is a Still, it would be highly myopic on our part to resolve the issue solely on generalities
judge not of history but of the Constitution. The Constitution, and indeed our modern surrounding military discipline. After all, petitioners seek to impress on us that their acts are
democratic order, frown in no uncertain terms on a politicized military, informed as they are justified as they were responding to an invitation from the Philippine Senate, a component of
on the trauma of absolute martial rule. Our history might imply that a political military is part the legislative branch of government. At the same time, the order for them not to testify
of the natural order, but this view cannot be affirmed by the legal order. The evolutionary ultimately came from the President, the head of the executive branch of government and the
path of our young democracy necessitates a reorientation from this view, reliant as our commander-in-chief of the armed forces.
socio-political culture has become on it. At the same time, evolution mandates a similar
Thus, we have to consider the question: may the President prevent a member of the armed
demand that our system of governance be more responsive to the needs and aspirations of
forces from testifying before a legislative inquiry? We hold that the President has
the citizenry, so as to avoid an environment vulnerable to a military apparatus able at will to
constitutional authority to do so, by virtue of her power as commander-in-chief, and that as a
exert an undue influence in our polity.
consequence a military officer who defies such injunction is liable under military justice. At
Of possibly less gravitas, but of equal importance, is the principle that mobility of travel is the same time, we also hold that any chamber of Congress which seeks the appearance
another necessary restriction on members of the military. A soldier cannot leave his/her post before it of a military officer against the consent of the President has adequate remedies
without the consent of the commanding officer. The reasons are self-evident. The under law to compel such attendance. Any military official whom Congress summons to
commanding officer has to be aware at all times of the location of the troops under testify before it may be compelled to do so by the President. If the President is not so
command, so as to be able to appropriately respond to any exigencies. For the same reason, inclined, the President may be commanded by judicial order to compel the attendance of the
commanding officers have to be able to restrict the movement or travel of their soldiers, if in military officer. Final judicial orders have the force of the law of the land which the President
their judgment, their presence at place of call of duty is necessary. At times, this may lead to has the duty to faithfully execute.[50]
unsentimental, painful consequences, such as a soldier being denied permission to witness
Explication of these principles is in order.
the birth of his first-born, or to attend the funeral of a parent. Yet again, military life calls for
considerable personal sacrifices during the period of conscription, wherein the higher duty is
As earlier noted, we ruled in Senate that the President may not issue a blanket requirement
not to self but to country.
of prior consent on executive officials summoned by the legislature to attend a congressional
hearing. In doing so, the Court recognized the considerable limitations on executive privilege,
Indeed, the military practice is to require a soldier to obtain permission from the
and affirmed that the privilege must be formally invoked on specified grounds. However, the
commanding officer before he/she may leave his destination. A soldier who goes from the
ability of the President to prevent military officers from testifying before Congress does not
properly appointed place of duty or absents from his/her command, guard, quarters, station,
turn on executive privilege, but on the Chief Executives power as commander-in-chief to
or camp without proper leave is subject to punishment by court-martial.[48] It is even clear
control the actions and speech of members of the armed forces. The Presidents prerogatives
from the record that petitioners had actually requested for travel authority from the PMA in
as commander-in-chief are not hampered by the same limitations as in executive privilege.
Baguio City to Manila, to attend the Senate Hearing.[49] Even petitioners are well aware that
it was necessary for them to obtain permission from their superiors before they could travel
Our ruling that the President could, as a general rule, require military officers to seek
to Manila to attend the Senate Hearing.
presidential approval before appearing before Congress is based foremost on the notion that
a contrary rule unduly diminishes the prerogatives of the President as commander-in-chief.
It is clear that the basic position of petitioners impinges on these fundamental principles we
Congress holds significant control over the armed forces in matters such as budget
have discussed. They seek to be exempted from military justice for having traveled to the
appropriations and the approval of higher-rank promotions,[51] yet it is on the President that
Senate to testify before the Senate Committee against the express orders of Gen. Senga, the
the Constitution vests the title as commander-in-chief and all the prerogatives and functions
AFP Chief of Staff. If petitioners position is affirmed, a considerable exception would be
appertaining to the position. Again, the exigencies of military discipline and the chain of
carved from the unimpeachable right of military officers to restrict the speech and
command mandate that the Presidents ability to control the individual members of the
movement of their juniors. The ruinous consequences to the chain of command and military
armed forces be accorded the utmost respect. Where a military officer is torn between
discipline simply cannot warrant the Courts imprimatur on petitioners position.
226

obeying the President and obeying the Senate, the Court will without hesitation affirm that Senate affirmed both the Arnault and Bengzon rulings. It elucidated on the constitutional
the officer has to choose the President. After all, the Constitution prescribes that it is the scope and limitations on the constitutional power of congressional inquiry. Thus:
President, and not the Senate, who is the commander-in-chief of the armed forces.[52]
As discussed in Arnault, the power of inquiry, with process to enforce it, is grounded on the
At the same time, the refusal of the President to allow members of the military to appear necessity of information in the legislative process. If the information possessed by executive
before Congress is still subject to judicial relief. The Constitution itself recognizes as one of officials on the operation of their offices is necessary for wise legislation on that subject, by
the legislatures functions is the conduct of inquiries in aid of legislation.[53] Inasmuch as it is parity of reasoning, Congress has the right to that information and the power to compel the
ill-advised for Congress to interfere with the Presidents power as commander-in-chief, it is disclosure thereof.
similarly detrimental for the President to unduly interfere with Congresss right to conduct
legislative inquiries. The impasse did not come to pass in this petition, since petitioners As evidenced by the American experience during the so-called McCarthy era, however, the
testified anyway despite the presidential prohibition. Yet the Court is aware that with its right of Congress to conduct inquirites in aid of legislation is, in theory, no less susceptible to
pronouncement today that the President has the right to require prior consent from abuse than executive or judicial power. It may thus be subjected to judicial review pursuant
members of the armed forces, the clash may soon loom or actualize. to the Courts certiorari powers under Section 1, Article VIII of the Constitution.

We believe and hold that our constitutional and legal order sanctions a modality by which For one, as noted in Bengzon v. Senate Blue Ribbon Committee, the inquiry itself might not
members of the military may be compelled to attend legislative inquiries even if the properly be in aid of legislation, and thus beyond the constitutional power of Congress. Such
President desires otherwise, a modality which does not offend the Chief Executives inquiry could not usurp judicial functions. Parenthetically, one possible way for Congress to
prerogatives as commander-in-chief. The remedy lies with the courts. avoid such result as occurred in Bengzon is to indicate in its invitations to the public officials
concerned, or to any person for that matter, the possible needed statute which prompted
The fact that the executive branch is an equal, coordinate branch of government to the the need for the inquiry. Given such statement in its invitations, along with the usual
legislative creates a wrinkle to any basic rule that persons summoned to testify before indication of the subject of inquiry and the questions relative to and in furtherance thereof,
Congress must do so. There is considerable interplay between the legislative and executive there would be less room for speculation on the part of the person invited on whether the
branches, informed by due deference and respect as to their various constitutional functions. inquiry is in aid of legislation.
Reciprocal courtesy idealizes this relationship; hence, it is only as a last resort that one
branch seeks to compel the other to a particular mode of behavior. The judiciary, the third Section 21, Article VI likewise establishes critical safeguards that proscribe the legislative
coordinate branch of government, does not enjoy a similar dynamic with either the power of inquiry. The provision requires that the inquiry be done in accordance with the
legislative or executive branches. Whatever weakness inheres on judicial power due to its Senate or Houses duly published rules of procedure, necessarily implying the constitutional
inability to originate national policies and legislation, such is balanced by the fact that it is the infirmity of an inquiry conducted without duly published rules of procedure. Section 21 also
branch empowered by the Constitution to compel obeisance to its rulings by the other mandates that the rights of persons appearing in or affected by such inquiries be respected,
branches of government. an imposition that obligates Congress to adhere to the guarantees in the Bill of Rights.

As evidenced by Arnault v. Nazareno[54] and Bengzon v. Senate Blue Ribbon Committee,[55] These abuses are, of course, remediable before the courts, upon the proper suit filed by the
among others, the Court has not shirked from reviewing the exercise by Congress of its persons affected, even if they belong to the executive branch. Nonetheless, there may be
power of legislative inquiry.[56] Arnault recognized that the legislative power of inquiry and exceptional circumstances wherein a clear pattern of abuse of the legislative power of
the process to enforce it, is an essential and appropriate auxiliary to the legislative inquiry might be established, resulting in palpable violations of the rights guaranteed to
function.[57] On the other hand, Bengzon acknowledged that the power of both houses of members of the executive department under the Bill of Rights. In such instances, depending
Congress to conduct inquiries in aid of legislation is not absolute or unlimited, and its on the particulars of each case, attempts by the Executive Branch to forestall these abuses
exercise is circumscribed by Section 21, Article VI of the Constitution.[58] From these may be accorded judicial sanction[59].
premises, the Court enjoined the Senate Blue Ribbon Committee from requiring the
In Senate, the Court ruled that the President could not impose a blanket prohibition barring
petitioners in Bengzon from testifying and producing evidence before the committee, holding
executive officials from testifying before Congress without the Presidents consent
that the inquiry in question did not involve any intended legislation.
227

notwithstanding the invocation of executive privilege to justify such prohibition. The Court Petitioners have presented several issues relating to the tenability or wisdom of the
did not rule that the power to conduct legislative inquiry ipso facto superseded the claim of Presidents order on them and other military officers not to testify before Congress without
executive privilege, acknowledging instead that the viability of executive privilege stood on a the Presidents consent. Yet these issues ultimately detract from the main point that they
case to case basis. Should neither branch yield to the other branchs assertion, the testified before the Senate despite an order from their commanding officer and their
constitutional recourse is to the courts, as the final arbiter if the dispute. It is only the courts commander-in-chief for them not to do so,[61] in contravention of the traditions of military
that can compel, with conclusiveness, attendance or non-attendance in legislative inquiries. discipline which we affirm today. The issues raised by petitioners could have very well been
raised and properly adjudicated if the proper procedure was observed. Petitioners could
Following these principles, it is clear that if the President or the Chief of Staff refuses to allow have been appropriately allowed to testify before the Senate without having to countermand
a member of the AFP to appear before Congress, the legislative body seeking such testimony their Commander-in-chief and superior officer under the setup we have prescribed.
may seek judicial relief to compel the attendance. Such judicial action should be directed at
the heads of the executive branch or the armed forces, the persons who wield authority and We consider the other issues raised by petitioners unnecessary to the resolution of this
control over the actions of the officers concerned. The legislative purpose of such testimony, petition.
as well as any defenses against the same whether grounded on executive privilege, national
security or similar concerns would be accorded due judicial evaluation. All the constitutional Petitioners may have been of the honest belief that they were defying a direct order of their
considerations pertinent to either branch of government may be raised, assessed, and Commander-in-Chief and Commanding General in obeisance to a paramount idea formed
ultimately weighed against each other. And once the courts speak with finality, both within their consciences, which could not be lightly ignored. Still, the Court, in turn, is guided
branches of government have no option but to comply with the decision of the courts, by the superlative principle that is the Constitution, the embodiment of the national
whether the effect of the decision is to their liking or disfavor. conscience. The Constitution simply does not permit the infraction which petitioners have
allegedly committed, and moreover, provides for an orderly manner by which the same
Courts are empowered, under the constitutional principle of judicial review, to arbitrate result could have been achieved without offending constitutional principles.
disputes between the legislative and executive branches of government on the proper
constitutional parameters of power.[60] This is the fair and workable solution implicit in the WHEREFORE, the petition is DENIED. No pronouncement as to costs.
constitutional allocation of powers among the three branches of government. The judicial
SO ORDERED.
filter helps assure that the particularities of each case would ultimately govern, rather than
any overarching principle unduly inclined towards one branch of government at the expense
Facts of the Case:
of the other. The procedure may not move as expeditiously as some may desire, yet it
ensures thorough deliberation of all relevant and cognizable issues before one branch is Senator Biazon invited senior officers of the Armed Forces of the Philippines (AFP) including
compelled to yield to the other. Moreover, judicial review does not preclude the legislative General Gudani to appear before a public hearing in the Senate Committee on National
and executive branches from negotiating a mutually acceptable solution to the impasse. Defense and Security wherein Hello Garci controversy of President Gloria Macapagal Arroyo
After all, the two branches, exercising as they do functions and responsibilities that are emerged. Upon the discretion of the President, AFP Chief of Staff Senga issued a
political in nature, are free to smooth over the thorns in their relationship with a salve of memorandum prohibiting General Gudani and company from appearing before the Senate
their own choosing. Committee without Presidential approval. However, General Gudani and Colonel Batulan still
attended the said committee in compliance with Senator Biazon.
And if emphasis be needed, if the courts so rule, the duty falls on the shoulders of the
President, as commander-in-chief, to authorize the appearance of the military officers before Issue: Can the President can prevent military officers from testifying a legislative inquiry?
Congress. Even if the President has earlier disagreed with the notion of officers appearing
before the legislature to testify, the Chief Executive is nonetheless obliged to comply with the
final orders of the courts.
228

Court Ruling: and affirmed that the privilege must be formally invoked on specified grounds. However, the
ability of the President to prevent military officers from testifying before Congress does not
YES. By virtue of her power as a commander-in-chief of the Armed Forces of the Philippines, turn on executive privilege, but on the Chief Executive’s power as commander-in-chief to
President Gloria Macapagal Arroyo has the constitutional authority to prohibit members of control the actions and speech of members of the armed forces. The President’s prerogatives
the AFP from attending a Senate hearing. as commander-in-chief are not hampered by the same limitations as in executive privilege.

This is also under her prerogative as the highest official of the AFP. Note that it is not an At the same time, the refusal of the President to allow members of the military to appear
invocation of her executive privilege, but on the Chief Executive's power to control the before Congress is still subject to judicial relief. The Constitution itself recognizes as one of
actions and speech of the members of the AFP. Non-compliance of the military subordinates the legislature’s functions is the conduct of inquiries in aid of legislation. Inasmuch as it is ill-
would violate the principle that 'the civilian authority is supreme over the military authority'. advised for Congress to interfere with the President’s power as commander-in-chief, it is
similarly detrimental for the President to unduly interfere with Congress’s right to conduct
FACTS:
legislative inquiries. The impasse did not come to pass in this petition, since petitioners
testified anyway despite the presidential prohibition. Yet the Court is aware that with its
The Senate invited Gen. Gudani and Lt. Col. Balutan to clarify allegations of 2004 election
pronouncement today that the President has the right to require prior consent from
fraud and the surfacing of the “Hello Garci” tapes. PGMA issued EO 464 enjoining officials of
members of the armed forces, the clash may soon loom or actualize.
the executive department including the military establishment from appearing in any
legislative inquiry without her consent. AFP Chief of Staff Gen. Senga issued a Memorandum,
The duty falls on the shoulders of the President, as commander-in-chief, to authorize the
prohibiting Gen. Gudani, Col. Balutan et al from appearing before the Senate Committee
appearance of the military officers before Congress. Even if the President has earlier
without Presidential approval. However, the two appeared before the Senate in spite the fact
disagreed with the notion of officers appearing before the legislature to testify, the Chief
that a directive has been given to them. As a result, the two were relieved of their
Executive is nonetheless obliged to comply with the final orders of the courts.
assignments for allegedly violating the Articles of War and the time honoured principle of the
“Chain of Command.” Gen. Senga ordered them to be subjected before the General Court FACTS:
Martial proceedings for willfuly violating an order of a superior officer.
Petitioners Gen. Gudani and Lieutenant Colonel Balutan are high-ranking officers of
ISSUE: Whether or not the President has the authority to issue an order to the members of Philippine Marines assigned to the Philippine Military Academy (PMA) in Baguio City. Senator
the AFP preventing them from testifying before a legislative inquiry. Biazon invited several senior officers of the military to appear at a public hearing before a
Senate Committee to clarify allegations of massive cheating and the surfacing of copies of an
RULING:
audio excerpt purportedly of a phone conversation between the President and then
Commission on Elections Commissioner Garcillano. At the time of the 2004 elections, Gen.
Yes. The SC hold that President has constitutional authority to do so, by virtue of her power
Gudani had been designated as commander, and Col. Balutan a member, of “Joint Task Force
as commander-in-chief, and that as a consequence a military officer who defies such
Ranao” by the AFP Southern Command. Armed Forces of the Philippines (AFP) Chief of Staff
injunction is liable under military justice. At the same time, any chamber of Congress which
Lt . Gen. Senga were among the several AFP officers also received a letter invitation from
seeks the appearance before it of a military officer against the consent of the President has
Sen. Biazon to attend the hearing. But only Gen. Gudani, and Col. Balutan attended the
adequate remedies under law to compel such attendance. Any military official whom
invitation from Sen. Biazon.
Congress summons to testify before it may be compelled to do so by the President. If the
President is not so inclined, the President may be commanded by judicial order to compel
Thereafter, the Office of the Chief of Staff of the AFP issued a Memorandum addressed to
the attendance of the military officer. Final judicial orders have the force of the law of the
Gen. Baloing. It was signed by Lt. Col. Hernando DCA Iriberri in behalf of Gen. Senga. Noting
land which the President has the duty to faithfully execute.
that Gen. Gudani and Col. Balutan had been invited to attend the Senate Committee hearing,
the Memorandum directed the two officers to attend the hearing. Conformably, Gen. Gudani
SC ruled in Senate v. Ermita that the President may not issue a blanket requirement of prior
and Col. Balutan filed their respective requests for travel authority addressed to the PMA
consent on executive officials summoned by the legislature to attend a congressional
Superintendent.
hearing. In doing so, the Court recognized the considerable limitations on executive privilege,
229

However, Gen. Senga did not attend to the requested hearing as per instruction from the are not encumbered by the same degree of restriction as that which may attach to executive
President that NO AFP PERSONNEL SHALL APPEAR BEFORE ANY CONGRESSIONAL OR SENATE privilege or executive control.
HEARING WITHOUT HER APPROVAL. `
We hold that the President has constitutional authority to do so, by virtue of her power as
While Gen. Gudani and Col. Balutan had concluded their testimony, the office of Gen. Senga commander-in-chief, and that as a consequence a military officer who defies such injunction
issued a statement which noted that the two had appeared before the Senate Committee “in is liable under military justice. At the same time, we also hold that any chamber of Congress
spite of the fact that a guidance has been given that a Presidential approval should be sought which seeks to appear before it a military officer against the consent of the President has
prior to such an appearance;” that such directive was “in keeping with the time[-]honored adequate remedies under law to compel such attendance. Any military official whom
principle of the Chain of Command;” and that the two officers “disobeyed a legal order, in Congress summons to testify before it may be compelled to do so by the President. If the
violation of A[rticles of] W[ar] 65 (Willfully Disobeying Superior Officer), hence they will be President is not so inclined, the President may be commanded by judicial order to compel
subjected to General Court Martial proceedings x x x” Both Gen. Gudani and Col. Balutan the attendance of the military officer. Final judicial orders have the force of the law of the
were likewise relieved of their assignments then. land which the President has the duty to faithfully execute.

On the very day of the hearing, the President issued Executive Order (E.O.) 464. The Office of Again, let it be emphasized that the ability of the President to prevent military officers from
the Solicitor General notes that the E.O. “enjoined officials of the executive department testifying before Congress does not turn on executive privilege, but on the Chief Executive’s
including the military establishment from appearing in any legislative inquiry without her power as commander-in-chief to control the actions and speech of members of the armed
approval. forces. The President’s prerogatives as commander-in-chief are not hampered by the same
limitations as in executive privilege. The commander-in-chief provision in the Constitution is
Now, petitioners seek the annulment of a directive from the President enjoining them and denominated as Section 18, Article VII, which begins with the simple declaration that “[t]he
other military officers from testifying before Congress without the President’s consent. President shall be the Commander-in-Chief of all armed forces of the Philippines x x x Outside
Petitioners also pray for injunctive relief against a pending preliminary investigation against explicit constitutional limitations, such as those found in Section 5, Article XVI, the
them, in preparation for possible court-martial proceedings, initiated within the military commander-in-chief clause vests on the President, as commander-in-chief, absolute
justice system in connection with petitioners’ violation of the aforementioned directive. authority over the persons and actions of the members of the armed forces. Such authority
includes the ability of the President to restrict the travel, movement and speech of military
The Court has to resolve whether petitioners may be subjected to military discipline on
officers, activities which may otherwise be sanctioned under civilian law.
account of their defiance of a direct order of the AFP Chief of Staff.
Reference to Kapunan, Jr. v. De Villa is useful in this regard. Lt. Col. Kapunan was ordered
ISSUE: Whether or not E.O. 464 which provides among others that NO AFP PERSONNEL
confined under “house arrest” by then Chief of Staff (later President) Gen. Fidel Ramos.
SHALL APPEAR BEFORE ANY CONGRESSIONAL OR SENATE HEARING WITHOUT HER
Kapunan was also ordered, as a condition for his house arrest, that he may not issue any
APPROVAL is unconstitutional?
press statements or give any press conference during his period of detention. The Court
unanimously upheld such restrictions, noting:
RULING:
“… to a certain degree, individual rights may be curtailed, because the effectiveness of the
The Petition is dismissed. Is EO 464 constitutional or not, or may the President prevent a
military in fulfilling its duties under the law depends to a large extent on the maintenance of
member of the armed forces from testifying before a legislative inquiry?
discipline within its ranks. Hence, lawful orders must be followed without question and rules
Insofar as E.O. 464 compelled officials of the executive branch to seek prior presidential must be faithfully complied with, irrespective of a soldier's personal views on the matter. It is
approval before appearing before Congress, the notion of executive control also comes into from this viewpoint that the restrictions imposed on petitioner Kapunan, an officer in the
consideration. The impression is wrong. The ability of the President to require a military AFP, have to be considered.”
official to secure prior consent before appearing in Congress pertains to wholly different and
As a general rule, it is integral to military discipline that the soldier’s speech be with the
independent specie of presidential authority—the commander-in-chief powers of the
consent and approval of the military commander. The necessity of upholding the ability to
President. By tradition and jurisprudence, the commander-in-chief powers of the President
230

restrain speech becomes even more imperative if the soldier desires to speak freely on needed statute which prompted the need for the inquiry. Section 21, Article VI likewise
political matters. For there is no constitutional provision or military indoctrination will establishes critical safeguards that proscribe the legislative power of inquiry. The provision
eliminate a soldier’s ability to form a personal political opinion, yet it is vital that such requires that the inquiry be done in accordance with the Senate or House’s duly published
opinions be kept out of the public eye. For one, political belief is a potential source of discord rules of procedure, necessarily implying the constitutional infirmity of an inquiry conducted
among people, and a military torn by political strife is incapable of fulfilling its constitutional without duly published rules of procedure. Section 21 also mandates that the rights of
function as protectors of the people and of the State. For another, it is ruinous to military persons appearing in or affected by such inquiries be respected, an imposition that obligates
discipline to foment an atmosphere that promotes an active dislike of or dissent against the Congress to adhere to the guarantees in the Bill of Rights.
President, the commander-in-chief of the armed forces. Soldiers are constitutionally obliged
to obey a President they may dislike or distrust. Even petitioners are well aware that it was In Senate, the Court ruled that the President could not impose a blanket prohibition barring
necessary for them to obtain permission from their superiors before they could travel to executive officials from testifying before Congress without the President’s consent
Manila to attend the Senate Hearing. notwithstanding the invocation of executive privilege to justify such prohibition. Should
neither branch yield to the other branch’s assertion, the constitutional recourse is to the
Congress holds significant control over the armed forces in matters such as budget courts, as the final arbiter if the dispute. It is only the courts that can compel, with
appropriations and the approval of higher-rank promotions, yet it is on the President that the conclusiveness, attendance or non-attendance in legislative inquiries.
Constitution vests the title as commander-in-chief and all the prerogatives and functions
appertaining to the position. Again, the exigencies of military discipline and the chain of Courts are empowered, under the constitutional principle of judicial review, to arbitrate
command mandate that the President’s ability to control the individual members of the disputes between the legislative and executive branches of government on the proper
armed forces be accorded the utmost respect. Where a military officer is torn between constitutional parameters of power. By this and, if the courts so rule, the duty falls on the
obeying the President and obeying the Senate, the Court will without hesitation affirm that shoulders of the President, as commander-in-chief, to authorize the appearance of the
the officer has to choose the President. After all, the Constitution prescribes that it is the military officers before Congress. Even if the President has earlier disagreed with the notion
President, and not the Senate, who is the commander-in-chief of the armed forces. of officers appearing before the legislature to testify, the Chief Executive is nonetheless
obliged to comply with the final orders of the courts.
Judicial relief as remedy:
Lastly, General Gudani argues that he can no longer fall within the jurisdiction of the court-
The refusal of the President to allow members of the military to appear before Congress is martial, considering his retirement last 4 October 2005. He cites Article 2, Title I of
not absolute. Inasmuch as it is ill-advised for Congress to interfere with the President’s power Commonwealth Act No. 408, which defines persons subject to military law as, among others,
as commander-in-chief, it is similarly detrimental for the President to unduly interfere with “all officers and soldiers in the active service of the [AFP],” and points out that he is no longer
Congress’s right to conduct legislative inquiries. The impasse did not come to pass in this in the active service. However, an officer whose name was dropped from the roll of officers
petition, since petitioners testified anyway despite the presidential prohibition. The remedy cannot be considered to be outside the jurisdiction of military authorities when military
lies with the courts. justice proceedings were initiated against him before the termination of his service. Once
jurisdiction has been acquired over the officer, it continues until his case is terminated.
Senate affirmed both the Arnault and Bengzon rulings. It elucidated on the constitutional
scope and limitations on the constitutional power of congressional inquiry. Thus, the power FACTS:
of inquiry, “with process to enforce it,” is grounded on the necessity of information in the
legislative process. If the information possessed by executive officials on the operation of On September 22, 2005, Senator Rodolfo Biazon invited several senior officers of the Armed
their offices is necessary for wise legislation on that subject, by parity of reasoning, Congress Forces of the Philippines (AFP) to appear at a public hearing before the Senate Committee on
has the right to that information and the power to compel the disclosure thereof. National Defense and Security scheduled on September 28, 2005. The hearing was scheduled
after topics concerning the conduct of the 2004 elections emerged in the public eye,
It may thus be subjected to judicial review pursuant to the Court’s certiorari powers under particularly allegations of massive cheating and the surfacing of copies of an audio excerpt
Section 1, Article VIII of the Constitution. To avoid conflict, Congress must indicate in its purportedly of a phone conversation between President Gloria Macapagal Arroyo and an
invitations to the public officials concerned, or to any person for that matter, the possible official of the Commission on Elections (COMELEC) widely reputed as then COMELEC
231

Commissioner Virgilio Garcillano. Brigadier General Francisco Gudani and Lieutenant Colonel armed forces. Such authority includes the ability of the President to restrict the travel,
Alexander Balutan, both assigned to the Philippine Military Academy as PMA Assistant movement and speech of military officers, activities which may otherwise be sanctioned
Superintendent and Assistant Commandant of Cadets, respectively, as well as AFP Chief of under civilian law.
Staff Lieutenant General Generoso Senga were among the AFP officers who received a letter
of invitation from Sen. Biazon to attend the hearing. On the evening of September 27, 2005,
a message was transmitted to the PMA Superintendent from Gen. Senga stating that “Per
instruction of Her Excellency PGMA, No AFP personnel shall appear before any congressional
or senate hearing without her approval” and to inform Gen. Gudani and Lt. Col. Balutan of
the same. But the two were already en route to Manila from Baguio and did not receive the
message such that the following day, both were present at the hearing and testified as to the
conduct of the 2004 elections. The office of Gen. Senga issued a statement noting that
Gudani and Balutan had appeared before the Senate Committee in spite of the fact that a
guidance had been given that a Presidential approval should be sought prior to such an
appearance; that such directive was in keeping with the time-honored principle of the Chain
of Command; and that the two officers disobeyed a legal order, in violation of Articles of War
65 (Willfully Disobeying Superior Officer) such that the two will be subjected to General
Court Martial proceedings and were likewise relieved of their assignments. The Office of the
Provost Marshal General (OPMG) recommended that petitioners be charged with violation of
Article of War 65. The case was referred to a Pre-Trial Investigation Officer (PTIO)
preparatory to trial by the General Court Martial.

ISSUE:

Does the President have the authority to issue an order to the members of the AFP
preventing them from testifying before a legislative inquiry?

HELD:

Yes. The vitality of the tenet that the President is the commander-in-chief of the Armed
Forces is most crucial to the democratic way of life, to civilian supremacy over the military,
and to the general stability of our representative system of government. The Constitution
reposes final authority, control and supervision of the AFP to the President, a civilian who is
not a member of the armed forces, and whose duties as commander-in-chief represent only
a part of the organic duties imposed upon the office, the other functions being clearly civil in
nature. Civilian supremacy over the military also countermands the notion that the military
may bypass civilian authorities, such as civil courts, on matters such as conducting
warrantless searches and seizures. The commander-in-chief provision in the Constitution is
denominated as Section 18, Article VII, which begins with the simple declaration that the
President shall be the Commander-in-Chief of all armed forces of the Philippines. Outside
explicit constitutional limitations, the commander-in-chief clause vests on the President, as
commander-in-chief, absolute authority over the persons and actions of the members of the
232

EN BANC mayors to offer their services in order that "the early and safe rescue of the hostages may be
achieved."7
G.R. No. 187298 July 03, 2012
This Memorandum, which was labeled ‘secret’ on its all pages, also outlined the
JAMAR M. KULAYAN, TEMOGEN S. TULAWIE, HJI. MOH. YUSOP ISMI, JULHAJAN AWADI, responsibilities of each of the party signatories, as follows:
and SPO1 SATTAL H. JADJULI, Petitioners,
vs. Responsibilities of the Provincial Government:
GOV. ABDUSAKUR M. TAN, in his capacity as Governor of Sulu; GEN. JUANCHO SABAN,
COL. EUGENIO CLEMEN PN, P/SUPT. JULASIRIM KASIM and P/SUPT. BIENVENIDO G. LATAG, 1) The Provincial Government shall source the funds and logistics needed for the
in their capacity as officers of the Phil. Marines and Phil. National Police, activation of the CEF;
respectively, Respondents.
2) The Provincial Government shall identify the Local Government Units which shall
DECISION participate in the operations and to propose them for the approval of the parties to
this agreement;
SERENO, J.:
3) The Provincial Government shall ensure that there will be no unilateral action(s)
On 15 January 2009, three members from the International Committee of the Red Cross by the CEF without the knowledge and approval by both parties.
(ICRC) were kidnapped in the vicinity of the Provincial Capitol in Patikul, Sulu.1 Andres Notter,
a Swiss national and head of the ICRC in Zamboanga City, Eugenio Vagni, an Italian national Responsibilities of AFP/PNP/ TF ICRC (Task Force ICRC):
and ICRC delegate, and Marie Jean Lacaba, a Filipino engineer, were purportedly inspecting a
water and sanitation project for the Sulu Provincial Jail when inspecting a water and
1) The AFP/PNP shall remain the authority as prescribed by law in military
sanitation project for the Sulu Provincial Jail when they were seized by three armed men who
operations and law enforcement;
were later confirmed to be members of the Abu Sayyaf Group (ASG).2 The leader of the
alleged kidnappers was identified as Raden Abu, a former guard at the Sulu Provincial Jail.
News reports linked Abu to Albader Parad, one of the known leaders of the Abu Sayyaf. 2) The AFP/PNP shall ensure the orderly deployment of the CEF in the performance
of their assigned task(s);
On 21 January 2009, a task force was created by the ICRC and the Philippine National Police
(PNP), which then organized a parallel local group known as the Local Crisis Committee. 3 The 3) The AFP/PNP shall ensure the safe movements of the CEF in identified areas of
local group, later renamed Sulu Crisis Management Committee, convened under the operation(s);
leadership of respondent Abdusakur Mahail Tan, the Provincial Governor of Sulu. Its armed
forces component was headed by respondents General Juancho Saban, and his deputy, 4) The AFP/PNP shall provide the necessary support and/or assistance as called for
Colonel Eugenio Clemen. The PNP component was headed by respondent Police in the course of operation(s)/movements of the CEF.8
Superintendent Bienvenido G. Latag, the Police Deputy Director for Operations of the
Autonomous Region of Muslim Mindanao (ARMM).4 Meanwhile, Ronaldo Puno, then Secretary of the Department of Interior and Local
Government, announced to the media that government troops had cornered some one
Governor Tan organized the Civilian Emergency Force (CEF), a group of armed male civilians hundred and twenty (120) Abu Sayyaf members along with the three (3) hostages.9 However,
coming from different municipalities, who were redeployed to surrounding areas of the ASG made contact with the authorities and demanded that the military pull its troops
Patikul.5 The organization of the CEF was embodied in a "Memorandum of back from the jungle area.10 The government troops yielded and went back to their barracks;
Understanding"6 entered into the Philippine Marines withdrew to their camp, while police and civilian forces pulled back
from the terrorists’ stronghold by ten (10) to fifteen (15) kilometers. Threatening that one of
between three parties: the provincial government of Sulu, represented by Governor Tan; the the hostages will be beheaded, the ASG further demanded the evacuation of the military
Armed Forces of the Philippines, represented by Gen. Saban; and the Philippine National camps and bases in the different barangays in Jolo.11 The authorities were given no later than
Police, represented by P/SUPT. Latag. The Whereas clauses of the Memorandum alluded to 2:00 o’clock in the afternoon of 31 March 2009 to comply.12
the extraordinary situation in Sulu, and the willingness of civilian supporters of the municipal
233

On 31 March 2009, Governor Tan issued Proclamation No. 1, Series of 2009 (Proclamation 1- apprehending officer alleged that they were suspected ASG supporters and were being
09), declaring a state of emergency in the province of Sulu.13 It cited the kidnapping incident arrested under Proclamation 1-09. The following day, 2 April 2009, the hostage Mary Jane
as a ground for the said declaration, describing it as a terrorist act pursuant to the Human Lacaba was released by the ASG.
Security
On 4 April 2009, the office of Governor Tan distributed to civic organizations, copies of the
Act (R.A. 9372). It also invoked Section 465 of the Local Government Code of 1991 (R.A. "Guidelines for the Implementation of Proclamation No. 1, Series of 2009 Declaring a State of
7160), which bestows on the Provincial Governor the power to carry out emergency Emergency in the Province of Sulu."18These Guidelines suspended all Permits to Carry
measures during man-made and natural disasters and calamities, and to call upon the
appropriate national law enforcement agencies to suppress disorder and lawless violence. Firearms Outside of Residence (PTCFORs) issued by the Chief of the PNP, and allowed
civilians to seek exemption from the gun ban only by applying to the Office of the Governor
In the same Proclamation, respondent Tan called upon the PNP and the CEF to set up and obtaining the appropriate identification cards. The said guidelines also allowed general
checkpoints and chokepoints, conduct general search and seizures including arrests, and searches and seizures in designated checkpoints and chokepoints.
other actions necessary to ensure public safety. The pertinent portion of the proclamation
states: On 16 April 2009, Jamar M. Kulayan, Temogen S. Tulawie, Hadji Mohammad Yusop Ismi,
Ahajan Awadi, and SPO1 Sattal H. Jadjuli, residents of Patikul, Sulu, filed the present Petition
NOW, THEREFORE, BY VIRTUE OF THE POWERS VESTED IN ME BY LAW, I, ABDUSAKUR for Certiorari and Prohibition,19 claiming that Proclamation 1-09 was issued with grave abuse
MAHAIL TAN, GOVERNOR OF THE PROVINCE OF SULU, DO HEREBY DECLARE A STATE OF of discretion amounting to lack or excess of jurisdiction, as it threatened fundamental
EMERGENCY IN THE PROVINCE OF SULU, AND CALL ON THE PHILIPPINE NATIONAL POLICE freedoms guaranteed under Article III of the 1987 Constitution.
WITH THE ASSISTANCE OF THE ARMED FORCES OF THE PHILIPPINES AND THE CIVILIAN
EMERGENCY FORCE TO IMPLEMENT THE FOLLOWING: Petitioners contend that Proclamation No. 1 and its Implementing Guidelines were issued
ultra vires, and thus null and void, for violating Sections 1 and 18, Article VII of the
1. The setting-up of checkpoints and chokepoints in the province; Constitution, which grants the President sole authority to exercise emergency powers and
calling-out powers as the chief executive of the Republic and commander-in-chief of the
2. The imposition of curfew for the entire province subject to such Guidelines as armed forces.20 Additionally, petitioners claim that the Provincial Governor is not authorized
may be issued by proper authorities; by any law to create civilian armed forces under his command, nor regulate and limit the
issuances of PTCFORs to his own private army.
3. The conduct of General Search and Seizure including arrests in the pursuit of the
kidnappers and their supporters; and In his Comment, Governor Tan contended that petitioners violated the doctrine on hierarchy
of courts when they filed the instant petition directly in the court of last resort, even if both
the Court of Appeals (CA) and the Regional Trial Courts (RTC) possessed concurrent
4. To conduct such other actions or police operations as may be necessary to
jurisdiction with the
ensure public safety.

Supreme Court under Rule 65.21 This is the only procedural defense raised by respondent
DONE AT THE PROVINCIAL CAPITOL, PROVINCE OF SULU THIS
Tan. Respondents Gen. Juancho Saban, Col. Eugenio Clemen, P/SUPT. Julasirim Kasim, and
P/SUPT. Bienvenido Latag did not file their respective Comments.1âwphi1
31STDAY OF MARCH 2009. Sgd. Abdusakur M. Tan Governor.14
On the substantive issues, respondents deny that Proclamation 1-09 was issued ultra vires, as
On 1 April 2009, SPO1 Sattal Jadjuli was instructed by his superior to report to respondent Governor Tan allegedly acted pursuant to Sections 16 and 465 of the Local Government
P/SUPT. Julasirim Kasim.15 Upon arriving at the police station, he was booked, and Code, which empowers the Provincial Governor to carry out emergency measures during
interviewed about his relationship to Musin, Jaiton, and Julamin, who were all his deceased calamities and disasters, and to call upon the appropriate national law enforcement agencies
relatives. Upon admitting that he was indeed related to the three, he was detained. After a to suppress disorder, riot, lawless violence, rebellion or sedition. 22 Furthermore, the
few hours, former Punong Barangay Juljahan Awadi, Hadji Hadjirul Bambra, Abdugajir Sangguniang Panlalawigan of Sulu authorized the declaration of a state of emergency as
Hadjirul, as well as PO2 Marcial Hajan, SPO3 Muhilmi Ismula, Punong Barangay Alano evidenced by Resolution No. 4, Series of 2009 issued on 31 March 2009 during its regular
Mohammad and jeepney driver Abduhadi Sabdani, were also arrested.16 The affidavit17 of the session.23
234

The threshold issue in the present case is whether or not Section 465, in relation to Section In David v. Macapagal-Arroyo,29 the Court highlighted the transcendental public importance
16, of the Local Government Code authorizes the respondent governor to declare a state of involved in cases that concern restrictive custody, because judicial review in these cases
emergency, and exercise the powers enumerated under Proclamation 1-09, specifically the serves as "a manifestation of the crucial defense of civilians ‘in police power’ cases due to the
conduct of general searches and seizures. Subsumed herein is the secondary question of diminution of their basic liberties under the guise of a state of emergency."30 Otherwise, the
whether or not the provincial governor is similarly clothed with authority to convene the CEF importance of the high tribunal as the court of last resort would be put to naught,
under the said provisions. considering the nature of "emergency" cases, wherein the proclamations and issuances are
inherently short-lived. In finally disposing of the claim that the issue had become moot and
We grant the petition. academic, the Court also cited transcendental public importance as an exception, stating:

I. Transcendental public Importance warrants a relaxation of the Doctrine of Hierarchy of Sa kabila ng pagiging akademiko na lamang ng mga isyu tungkol sa mahigpit na
Courts pangangalaga (restrictive custody) at pagmonitor ng galaw (monitoring of movements) ng
nagpepetisyon, dedesisyunan namin ito (a) dahil sa nangingibabaw na interes ng madla na
nakapaloob dito,
We first dispose of respondents’ invocation of the doctrine of hierarchy of courts which
allegedly prevents judicial review by this Court in the present case, citing for this specific
purpose, Montes v. Court of Appeals and Purok Bagong Silang Association, Inc. v. (b) dahil sa posibilidad na maaaring maulit ang pangyayari at (c) dahil kailangang maturuan
Yuipco.24 Simply put, the ang kapulisan tungkol dito.

doctrine provides that where the issuance of an extraordinary writ is also within the The moot and academic principle is not a magical formula that can automatically dissuade
competence of the CA or the RTC, it is in either of these courts and not in the Supreme Court, the courts in resolving a case. Courts will decide cases, otherwise moot and academic, if: first,
that the specific action for the issuance of such writ must be sought unless special and there is a grave violation of the Constitution; second, the exceptional character of the
important laws are clearly and specifically set forth in the petition. The reason for this is that situation and the paramount public interest is involved; third, when [the] constitutional issue
this Court is a court of last resort and must so remain if it is to perform the functions assigned raised requires formulation of controlling principles to guide the bench, the bar, and the
to it by the Constitution and immemorial tradition. It cannot be burdened with deciding public; and fourth, the case is capable of repetition yet evading review.
cases in the first instance.25
…There is no question that the issues being raised affect the public interest, involving as they
The said rule, however, is not without exception. In Chavez v. PEA-Amari,26 the Court stated: do the people’s basic rights to freedom of expression, of assembly and of the press.
Moreover, the
PEA and AMARI claim petitioner ignored the judicial hierarchy by seeking relief directly from
the Court. The principle of hierarchy of courts applies generally to cases involving factual Court has the duty to formulate guiding and controlling constitutional precepts, doctrines or
questions. As it is not a trier of facts, the Court cannot entertain cases involving factual rules. It has the symbolic function of educating the bench and the bar, and in the present
issues. The instant case, however, raises constitutional questions of transcendental petitions, the military and the police, on the extent of the protection given by constitutional
importance to the public. The Court can resolve this case without determining any factual guarantees. And lastly, respondents contested actions are capable of repetition. Certainly,
issue related to the case. Also, the instant case is a petition for mandamus which falls under the petitions are subject to judicial review.
the original jurisdiction of the Court under Section 5, Article VIII of the Constitution. We
resolve to exercise primary jurisdiction over the instant case.27 Evidently, the triple reasons We advanced at the start of Our ruling are justified under the
foregoing exceptions. Every bad, unusual incident where police officers figure in generates
The instant case stems from a petition for certiorari and prohibition, over which the Supreme public interest and people watch what will be done or not done to them. Lack of disciplinary
Court possesses original jurisdiction.28 More crucially, this case involves acts of a public steps taken against them erode public confidence in the police institution. As petitioners
official which pertain to restrictive custody, and is thus impressed with transcendental public themselves assert, the restrictive custody of policemen under investigation is an existing
importance that would warrant the relaxation of the general rule. The Court would be remiss practice, hence, the issue is bound to crop up every now and then. The matter is capable of
in its constitutional duties were it to dismiss the present petition solely due to claims of repetition or susceptible of recurrence. It better be resolved now for the education and
judicial hierarchy. guidance of all concerned.
235

Hence, the instant petition is given due course, impressed as it is with transcendental public prevent or suppress lawless violence, invasion or rebellion. In case of invasion or rebellion,
importance. when the public safety requires it, he may, for a period not exceeding sixty days, suspend the
privilege of the writ of habeas corpus or place the Philippines or any part thereof under
II. Only the President is vested with calling-out powers, as the commander-in-chief of the martial law. Within forty-eight hours from the proclamation of martial law or the suspension
Republic of the privilege of the writ of habeas corpus, the President shall submit a report in person or
in writing to the Congress. The Congress, voting jointly, by a vote of at least a majority of all
its Members in regular or special session, may revoke such proclamation or suspension,
i. One executive, one commander-in-chief
which revocation shall not be set aside by the President. Upon the initiative of the President,
the Congress may, in the same manner, extend such proclamation or suspension for a period
As early as Villena v. Secretary of Interior,32 it has already been established that there is one to be determined by the Congress, if the invasion or rebellion shall persist and public safety
repository of executive powers, and that is the President of the Republic. This means that requires it.
when Section 1, Article VII of the Constitution speaks of executive power, it is granted to the
President and no one else.33 As emphasized by Justice Jose P. Laurel, in his ponencia in
The Congress, if not in session, shall, within twenty-four hours following such proclamation
Villena:
or suspension, convene in accordance with its rules without need of a call.36

With reference to the Executive Department of the government, there is one purpose which
The power to declare a state of martial law is subject to the Supreme Court’s authority to
is crystal-clear and is readily visible without the projection of judicial searchlight, and that is
review the factual basis thereof. 37 By constitutional fiat, the calling-out powers, which is of
the establishment of a single, not plural, Executive. The first section of Article VII of the
lesser gravity than the power to declare martial law, is bestowed upon the President alone.
Constitution, dealing with the Executive Department, begins with the enunciation of the
As noted in Villena, "(t)here are certain constitutional powers and prerogatives of the Chief
principle that "The executive power shall be vested in a President of the Philippines." This
Executive of the Nation which must be exercised by him in person and no amount of
means that the President of the Philippines is the Executive of the Government of the
approval or ratification will validate the exercise of any of those powers by any other person.
Philippines, and no other.34
Such, for instance, is his power to suspend the writ of habeas corpus and proclaim martial
law x x x.38
Corollarily, it is only the President, as Executive, who is authorized to exercise emergency
powers as provided under Section 23, Article VI, of the Constitution, as well as what became
Indeed, while the President is still a civilian, Article II, Section 339 of the Constitution
known as the calling-out powers under Section 7, Article VII thereof.
mandates that civilian authority is, at all times, supreme over the military, making the civilian
president the nation’s supreme military leader. The net effect of Article II, Section 3, when
ii. The exceptional character of Commander-in-Chief powers dictate that they are exercised by read with Article VII,
one president
Section 18, is that a civilian President is the ceremonial, legal and administrative head of the
Springing from the well-entrenched constitutional precept of One President is the notion that armed forces. The Constitution does not require that the President must be possessed of
there are certain acts which, by their very nature, may only be performed by the president as military training and talents, but as Commander-in-Chief, he has the power to direct military
the Head of the State. One of these acts or prerogatives is the bundle of Commander-in-Chief operations and to determine military strategy. Normally, he would be expected to delegate
powers to which the "calling-out" powers constitutes a portion. The President’s Emergency the actual command of the armed forces to military experts; but the ultimate power is
Powers, on the other hand, is balanced only by the legislative act of Congress, as embodied in his.40 As Commander-in-Chief, he is authorized to direct the movements of the naval and
the second paragraph of Section 23, Article 6 of the Constitution: military forces placed by law at his command, and to employ them in the manner he may
deem most effectual.41
Article 6, Sec 23(2). In times of war or other national emergency, the Congress may, by law,
authorize the President, for a limited period and subject to such restrictions as it may In the case of Integrated Bar of the Philippines v. Zamora,42 the Court had occasion to rule
prescribe, to exercise powers necessary and proper to carry out a declared national policy. that the calling-out powers belong solely to the President as commander-in-chief:
Unless sooner withdrawn by resolution of the Congress, such powers shall cease upon the
next adjournment thereof.35
When the President calls the armed forces to prevent or suppress lawless violence, invasion
or rebellion, he necessarily exercises a discretionary power solely vested in his wisdom. This
Article 7, Sec 18. The President shall be the Commander-in-Chief of all armed forces of the is clear from the intent of the framers and from the text of the Constitution itself. The Court,
Philippines and whenever it becomes necessary, he may call out such armed forces to thus, cannot be called upon to overrule the President’s wisdom or substitute its own.
236

However, this does not prevent an examination of whether such power was exercised within x x x Thus, it is the unclouded intent of the Constitution to vest upon the President, as
permissible constitutional limits or whether it was exercised in a manner constituting grave Commander-in-Chief of the Armed Forces, full discretion to call forth the military when in his
abuse of discretion. In view of the constitutional intent to give the President full discretionary judgment it is necessary to do so in order to prevent or suppress lawless violence, invasion or
power to determine the necessity of calling out the armed forces, it is incumbent upon the rebellion.45 (Emphasis Supplied)
petitioner to show that the President’s decision is totally bereft of factual basis.
In the more recent case of Constantino, Jr. v. Cuisia,46 the Court characterized these powers
There is a clear textual commitment under the Constitution to bestow on the President full as exclusive to the President, precisely because they are of exceptional import:
discretionary power to call out the armed forces and to determine the necessity for the
exercise of such power.43(Emphasis supplied) These distinctions hold true to this day as they remain embodied in our fundamental law.
There are certain presidential powers which arise out of exceptional circumstances, and if
Under the foregoing provisions, Congress may revoke such proclamation or suspension and exercised, would involve the suspension of fundamental freedoms, or at least call for the
the Court may review the sufficiency of the factual basis thereof. However, there is no such supersedence of executive prerogatives over those exercised by co-equal branches of
equivalent provision dealing with the revocation or review of the President’s action to call government. The declaration of martial law, the suspension of the writ of habeas corpus, and
out the armed forces. The distinction places the calling out power in a different category the exercise of the pardoning power, notwithstanding the judicial determination of guilt of
from the power to declare martial law and the power to suspend the privilege of the writ of the accused, all fall within this special class that demands the exclusive exercise by the
habeas corpus, otherwise, the framers of the Constitution would have simply lumped President of the constitutionally vested power. The list is by no means exclusive, but there
together the three powers and provided for their revocation and review without any must be a showing that the executive power in question is of similar gravitas and exceptional
qualification.44 import.47

That the power to call upon the armed forces is discretionary on the president is clear from In addition to being the commander-in-chief of the armed forces, the President also acts as
the deliberation of the Constitutional Commission: the leader of the country’s police forces, under the mandate of Section 17, Article VII of the
Constitution, which provides that, "The President shall have control of all the executive
FR. BERNAS. It will not make any difference. I may add that there is a graduated power of the departments, bureaus, and offices. He shall ensure that the laws be faithfully executed."
President as Commander-in-Chief. First, he can call out such Armed Forces as may be During the deliberations of the Constitutional Commission on the framing of this provision,
necessary to suppress lawless violence; then he can suspend the privilege of the writ of Fr. Bernas defended the retention of the word "control," employing the same rationale of
habeas corpus, then he can impose martial law. This is a graduated sequence. singularity of the office of the president, as the only Executive under the presidential form of
government.48
When he judges that it is necessary to impose martial law or suspend the privilege of the writ
of habeas corpus, his judgment is subject to review. We are making it subject to review by Regarding the country’s police force, Section 6, Article XVI of the Constitution states that:
the Supreme Court and subject to concurrence by the National Assembly. But when he "The State shall establish and maintain one police force, which shall be national in scope and
exercises this lesser power of calling on the Armed Forces, when he says it is necessary, it is civilian in character, to be administered and controlled by a national police commission. The
my opinion that his judgment cannot be reviewed by anybody. authority of local executives over the police units in their jurisdiction shall be provided by
law."49
MR. REGALADO. That does not require any concurrence by the legislature nor is it subject to
judicial review. A local chief executive, such as the provincial governor, exercises operational supervision
over the police,50 and may exercise control only in day-to-day operations, viz:
The reason for the difference in the treatment of the aforementioned powers highlights the
intent to grant the President the widest leeway and broadest discretion in using the power to Mr. Natividad: By experience, it is not advisable to provide either in our Constitution or by
call out because it is considered as the lesser and more benign power compared to the law full control of the police by the local chief executive and local executives, the mayors. By
power to suspend the privilege of the writ of habeas corpus and the power to impose martial our experience, this has spawned warlordism, bossism and sanctuaries for vices and abuses.
law, both of which involve the curtailment and suppression of certain basic civil rights and If the national government does not have a mechanism to supervise these 1,500 legally,
individual freedoms, and thus necessitating safeguards by Congress and review by this Court. technically separate police forces, plus 61 city police forces, fragmented police system, we
will have a lot of difficulty in presenting a modern professional police force. So that a certain
amount of supervision and control will have to be exercised by the national government.
237

For example, if a local government, a town cannot handle its peace and order problems or Mr. Natividad: No, not under the Commander-in-Chief provision.
police problems, such as riots, conflagrations or organized crime, the national government
may come in, especially if requested by the local executives. Under that situation, if they Mr. Rodrigo: There are two other powers of the President. The
come in under such an extraordinary situation, they will be in control. But if the day-to-day
business of police investigation of crime, crime prevention, activities, traffic control, is all
President has control over ministries, bureaus and offices, and supervision over local
lodged in the mayors, and if they are in complete operational control of the day-to-day
governments. Under which does the police fall, under control or under supervision?
business of police service, what the national government would control would be the
administrative aspect.
Mr. Natividad: Both, Madam President.
Mr. de los Reyes: so the operational control on a day-to-day basis, meaning, the usual duties
being performed by the ordinary policemen, will be under the supervision of the local Mr. Rodrigo: Control and supervision.
executives?
Mr. Natividad: Yes, in fact, the National Police Commission is under the Office of the
Mr. Natividad: Yes, Madam President. President.52

Mr. de los Reyes: But in exceptional cases, even the operational control can be taken over by In the discussions of the Constitutional Commission regarding the above provision it is clear
the National Police Commission? that the framers never intended for local chief executives to exercise unbridled control over
the police in emergency situations. This is without prejudice to their authority over police
units in their jurisdiction as provided by law, and their prerogative to seek assistance from
Mr. Natividad: If the situation is beyond the capacity of the local governments. 51 (Emphases
the police in day to day situations, as contemplated by the Constitutional Commission. But as
supplied)
a civilian agency of the government, the police, through the NAPOLCOM, properly comes
within, and is subject to, the exercise by the President of the power of executive control.53
Furthermore according to the framers, it is still the President who is authorized to exercise
supervision and control over the police, through the National Police Commission:
iii. The provincial governor does not possess the same calling-out powers as the President

Mr. Rodrigo: Just a few questions. The President of the Philippines is the Commander-in-
Given the foregoing, respondent provincial governor is not endowed with the power to call
Chief of all the armed forces.
upon the armed forces at his own bidding. In issuing the assailed proclamation, Governor Tan
exceeded his authority when he declared a state of emergency and called upon the Armed
Mr. Natividad: Yes, Madam President. Forces, the police, and his own Civilian Emergency Force. The calling-out powers
contemplated under the Constitution is exclusive to the President. An exercise by another
Mr. Rodrigo: Since the national police is not integrated with the armed forces, I do not official, even if he is the local chief executive, is ultra vires, and may not be justified by the
suppose they come under the Commander-in-Chief powers of the President of the invocation of Section 465 of the Local Government Code, as will be discussed subsequently.
Philippines.
Respondents, however, justify this stance by stating that nowhere in the seminal case of
Mr. Natividad: They do, Madam President. By law, they are under the supervision and control David v. Arroyo, which dealt squarely with the issue of the declaration of a state of
of the President of the Philippines. emergency, does it limit the said authority to the President alone. Respondents contend that
the ruling in David expressly limits the authority to declare a national emergency, a condition
Mr. Rodrigo: Yes, but the President is not the Commander-in-Chief of the national police. which covers the entire country, and does not include emergency situations in local
government units.54 This claim is belied by the clear intent of the framers that in all situations
involving threats to security, such as lawless violence, invasion or rebellion, even in localized
Mr. Natividad: He is the President.
areas, it is still the President who possesses the sole authority to exercise calling-out powers.
As reflected in the Journal of the Constitutional Commission:
Mr. Rodrigo: Yes, the Executive. But they do not come under that specific provision that the
President is the Commander-in-Chief of all the armed forces.
238

Thereafter, Mr. Padilla proposed on line 29 to insert the phrase OR PUBLIC DISORDER in lieu complainant and the witnesses he may produce, and particularly describing the place to be
of "invasion or rebellion." Mr. Sumulong stated that the committee could not accept the searched and the persons or things to be seized.58
amendment because under the first section of Section 15, the President may call out and
make use of the armed forces to prevent or suppress not only lawless violence but even In fact, respondent governor has arrogated unto himself powers exceeding even the martial
invasion or rebellion without declaring martial law. He observed that by deleting "invasion or law powers of the President, because as the Constitution itself declares, "A state of martial
rebellion" and substituting PUBLIC DISORDER, the President would have to declare martial law does not suspend the operation of the Constitution, nor supplant the functioning of the
law before he can make use of the armed forces to prevent or suppress lawless invasion or civil courts or legislative assemblies, nor authorize the conferment of the jurisdiction on
rebellion. military courts and agencies over civilians where civil courts are able to function, nor
automatically suspend the privilege of the writ."59
Mr. Padilla, in reply thereto, stated that the first sentence contemplates a lighter situation
where there is some lawless violence in a small portion of the country or public disorder in We find, and so hold, that there is nothing in the Local Government Code which justifies the
another at which times, the armed forces can be called to prevent or suppress these acts sanctioned under the said Proclamation. Not even Section 465 of the said Code, in
incidents. He noted that the Commander-in-Chief can do so in a minor degree but he can also relation to Section 16, which states:
exercise such powers should the situation worsen. The words "invasion or rebellion" to be
eliminated on line 14 are covered by the following sentence which provides for "invasion or
Section 465. The Chief Executive: Powers, Duties, Functions, and Compensation.
rebellion." He maintained that the proposed amendment does not mean that under such
circumstances, the President cannot call on the armed forces to prevent or suppress the
same.55 (Emphasis supplied) (b) For efficient, effective and economical governance the purpose of which is the general
welfare of the province and its inhabitants pursuant to Section 16 of this Code, the provincial
governor shall:
III. Section 465 of the Local

(1) Exercise general supervision and control over all programs, projects, services, and
Government Code cannot be invoked to justify the powers enumerated under Proclamation 1-
activities of the provincial government, and in this connection, shall:
09

(vii) Carry out such emergency measures as may be necessary during and in the aftermath of
Respondent governor characterized the kidnapping of the three ICRC workers as a terroristic
man-made and natural disasters and calamities;
act, and used this incident to justify the exercise of the powers enumerated under
Proclamation 1-09.56 He invokes Section 465, in relation to Section 16, of the Local
Government Code, which purportedly allows the governor to carry out emergency measures (2) Enforce all laws and ordinances relative to the governance of the province and the
and call upon the appropriate national law enforcement agencies for assistance. But a closer exercise of the appropriate corporate powers provided for under Section 22 of this Code,
look at the said proclamation shows that there is no provision in the Local Government Code implement all approved policies, programs, projects, services and activities of the province
nor in any law on which the broad and unwarranted powers granted to the Governor may be and, in addition to the foregoing, shall:
based.
(vi) Call upon the appropriate national law enforcement agencies to suppress disorder, riot,
Petitioners cite the implementation of "General Search and Seizure including arrests in the lawless violence, rebellion or sedition or to apprehend violators of the law when public
pursuit of the kidnappers and their supporters,"57 as being violative of the constitutional interest so requires and the police forces of the component city or municipality where the
proscription on general search warrants and general seizures. Petitioners rightly assert that disorder or violation is happening are inadequate to cope with the situation or the violators.
this alone would be sufficient to render the proclamation void, as general searches and
seizures are proscribed, for being violative of the rights enshrined in the Bill of Rights, Section 16. General Welfare. - Every local government unit shall exercise the powers
particularly: expressly granted, those necessarily implied therefrom, as well as powers necessary,
appropriate, or incidental for its efficient and effective governance, and those which are
The right of the people to be secure in their persons, houses, papers, and effects against essential to the promotion of the general welfare. Within their respective territorial
unreasonable searches and seizures of whatever nature and for any purpose shall be jurisdictions, local government units shall ensure and support, among other things, the
inviolable, and no search warrant or warrant of arrest shall issue except upon probable cause preservation and enrichment of culture, promote health and safety, enhance the right of the
to be determined personally by the judge after examination under oath or affirmation of the people to a balanced ecology, encourage and support the development of appropriate and
self-reliant scientific and technological capabilities, improve public morals, enhance
239

economic prosperity and social justice, promote full employment among their residents, diminution of central office powers and responsibilities. Certain government agencies,
maintain peace and order, and preserve the comfort and convenience of their inhabitants. including the police force, are exempted from the decentralization process because their
(Emphases supplied) functions are not inherent in local government units."63

Respondents cannot rely on paragraph 1, subparagraph (vii) of Article 465 above, as the said IV. Provincial governor is not authorized to convene CEF
provision expressly refers to calamities and disasters, whether man-made or natural. The
governor, as local chief executive of the province, is certainly empowered to enact and Pursuant to the national policy to establish one police force, the organization of private
implement emergency measures during these occurrences. But the kidnapping incident in citizen armies is proscribed. Section 24 of Article XVIII of the Constitution mandates that:
the case at bar cannot be considered as a calamity or a disaster. Respondents cannot find any
legal mooring under this provision to justify their actions.
Private armies and other armed groups not recognized by duly constituted authority shall be
dismantled. All paramilitary forces including Civilian Home Defense Forces (CHDF) not
Paragraph 2, subparagraph (vi) of the same provision is equally inapplicable for two reasons. consistent with the citizen armed force established in this Constitution, shall be dissolved or,
First, the Armed Forces of the Philippines does not fall under the category of a "national law where appropriate, converted into the regular force.
enforcement agency," to which the National Police Commission (NAPOLCOM) and its
departments belong.
Additionally, Section 21of Article XI states that, "The preservation of peace and order within
the regions shall be the responsibility of the local police agencies which shall be organized,
Its mandate is to uphold the sovereignty of the Philippines, support the Constitution, and maintained, supervised, and utilized in accordance with applicable laws. The defense and
defend the Republic against all enemies, foreign and domestic. Its aim is also to secure the security of the regions shall be the responsibility of the National Government."
integrity of the national territory.60
Taken in conjunction with each other, it becomes clear that the Constitution does not
Second, there was no evidence or even an allegation on record that the local police forces authorize the organization of private armed groups similar to the CEF convened by the
were inadequate to cope with the situation or apprehend the violators. If they were respondent Governor. The framers of the Constitution were themselves wary of armed
inadequate, the recourse of the provincial governor was to ask the assistance of the citizens’ groups, as shown in the following proceedings:
Secretary of Interior and Local Government, or such other authorized officials, for the
assistance of national law enforcement agencies.
MR. GARCIA: I think it is very clear that the problem we have here is a paramilitary force
operating under the cloak, under the mantle of legality is creating a lot of problems precisely
The Local Government Code does not involve the diminution of central powers inherently by being able to operate as an independent private army for many regional warlords. And at
vested in the National Government, especially not the prerogatives solely granted by the the same time, this I think has been the thrust, the intent of many of the discussions and
Constitution to the President in matters of security and defense. objections to the paramilitary units and the armed groups.

The intent behind the powers granted to local government units is fiscal, economic, and MR. PADILLA: My proposal covers two parts: the private armies of political warlords and
administrative in nature.1âwphi1The Code is concerned only with powers that would make other armed torces not recognized by constituted authority which shall be dismantled and
the delivery of basic services more effective to the constituents,61 and should not be unduly dissolved. In my trips to the provinces, I heard of many abuses committed by the CHDF
stretched to confer calling-out powers on local executives. (Civilian Home Defense Forces), specially in Escalante, Negros Occidental. But I do not know
whether a particular CHDF is approved or authorized by competent authority. If it is not
In the sponsorship remarks for Republic Act 7160, it was stated that the devolution of authorized, then the CHDF will have to be dismantled. If some CHDFs, say in other provinces,
powers is a step towards the autonomy of local government units (LGUs), and is actually an are authorized by constituted authority, by the Armed Forces of the Philippines, through the
experiment whose success heavily relies on the power of taxation of the LGUs. The Chief of Staff or the Minister of National Defense, if they are recognized and authorized, then
underpinnings of the Code can be found in Section 5, Article II of the 1973 Constitution, they will not be dismantled. But I cannot give a categorical answer to any specific CHDF unit,
which allowed LGUs to create their own sources of revenue.62 During the interpellation made only the principle that if they are armed forces which are not authorized, then they should be
by Mr. Tirol addressed to Mr. de Pedro, the latter emphasized that "Decentralization is an dismantled. 64 (Emphasis supplied)
administrative concept and the process of shifting and delegating power from a central point
to subordinate levels to promote independence, responsibility, and quicker decision-making. Thus, with the discussions in the Constitutional Commission as guide, the creation of the
… (I)t does not involve any transfer of final authority from the national to field levels, nor Civilian Emergency Force (CEF) in the present case, is also invalid.
240

WHEREFORE, the instant petition is GRANTED. Judgment is rendered commanding Responsibilities of the Provincial Government:
respondents to desist from further proceedings m implementing Proclamation No. 1, Series
of 2009, and its Implementing Guidelines. The said proclamation and guidelines are hereby 1) The Provincial Government shall source the funds and logistics needed for the
declared NULL and VOID for having been issued in grave abuse of discretion, amounting to activation of the CEF;
lack or excess of jurisdiction.
2) The Provincial Government shall identify the Local Government Units which shall
SO ORDERED. participate in the operations and to propose them for the approval of the parties to
this agreement;
G.R. No. 187298
3) The Provincial Government shall ensure that there will be no unilateral action(s)
FACTS: by the CEF without the knowledge and approval by both parties.

On 15 January 2009, three members from the International Committee of the Red Cross
Responsibilities of AFP/PNP/ TF ICRC (Task Force ICRC):
(ICRC) were kidnapped in the vicinity of the Provincial Capitol in Patikul, Sulu. Andres Notter,
a Swiss national and head of the ICRC in Zamboanga City, Eugenio Vagni, an Italian national 1) The AFP/PNP shall remain the authority as prescribed by law in military
and ICRC delegate, and Marie Jean Lacaba, a Filipino engineer, were purportedly inspecting a operations and law enforcement;
water and sanitation project for the Sulu Provincial Jail when inspecting a water and
sanitation project for the Sulu Provincial Jail when they were seized by three armed men who 2) The AFP/PNP shall ensure the orderly deployment of the CEF in the performance
were later confirmed to be members of the Abu Sayyaf Group (ASG). The leader of the of their assigned task(s);
alleged kidnappers was identified as Raden Abu, a former guard at the Sulu Provincial Jail.
News reports linked Abu to Albader Parad, one of the known leaders of the Abu Sayyaf. 3) The AFP/PNP shall ensure the safe movements of the CEF in identified areas of
operation(s);
On 21 January 2009, a task force was created by the ICRC and the Philippine National Police
(PNP), which then organized a parallel local group known as the Local Crisis Committee. The 4) The AFP/PNP shall provide the necessary support and/or assistance as called for
local group, later renamed Sulu Crisis Management Committee, convened under the in the course of operation(s)/movements of the CEF.
leadership of respondent Abdusakur Mahail Tan, the Provincial Governor of Sulu. Its armed
forces component was headed by respondents General Juancho Saban, and his deputy, Ronaldo Puno, then Secretary of the Department of Interior and Local Government,
Colonel Eugenio Clemen. The PNP component was headed by respondent Police announced to the media that government troops had cornered some one hundred and
Superintendent Bienvenido G. Latag, the Police Deputy Director for Operations of the twenty (120) Abu Sayyaf members along with the three (3) hostages.
Autonomous Region of Muslim Mindanao (ARMM).
On 31 March 2009, Governor Tan issued Proclamation No. 1, Series of 2009 (Proclamation
Governor Tan organized the Civilian Emergency Force (CEF), a group of armed male civilians 1-09), declaring a state of emergency in the province of Sulu. It cited the kidnapping
coming from different municipalities, who were redeployed to surrounding areas of incident as a ground for the said declaration, describing it as a terrorist act pursuant to the
Patikul. The organization of the CEF was embodied in a "Memorandum of Understanding" Human Security.
entered into between three parties: the provincial government of Sulu, represented by
Governor Tan; the Armed Forces of the Philippines, represented by Gen. Saban; and the
Act (R.A. 9372). It also invoked Section 465 of the Local Government Code of 1991 (R.A.
Philippine National Police, represented by P/SUPT. Latag. The Whereas clauses of the
7160), which bestows on the Provincial Governor the power to carry out emergency
Memorandum alluded to the extraordinary situation in Sulu, and the willingness of civilian
measures during man-made and natural disasters and calamities, and to call upon the
supporters of the municipal mayors to offer their services in order that "the early and safe
appropriate national law enforcement agencies to suppress disorder and lawless violence.
rescue of the hostages may be achieved.".

The office of Governor Tan distributed to civic organizations, copies of the "Guidelines for the
This Memorandum, which was labeled ‘secret’ on its all pages, also outlined the
Implementation of Proclamation No. 1, Series of 2009 Declaring a State of Emergency in the
responsibilities of each of the party signatories, as follows:
Province of Sulu."These Guidelines suspended all Permits to Carry.
241

Petitioners contention: That Proclamation No. 1 and its Implementing Guidelines were Springing from the well-entrenched constitutional precept of One President is the notion that
issued ultra vires, and thus null and void, for violating Sections 1 and 18, Article VII of the there are certain acts which, by their very nature, may only be performed by the president as
Constitution, which grants the President sole authority to exercise emergency powers and the Head of the State. One of these acts or prerogatives is the bundle of Commander-in-Chief
calling-out powers as the chief executive of the Republic and commander-in-chief of the powers to which the "calling-out" powers constitutes a portion. The President’s Emergency
armed forces. Additionally, petitioners claim that the Provincial Governor is not authorized by Powers, on the other hand, is balanced only by the legislative act of Congress, as embodied in
any law to create civilian armed forces under his command, nor regulate and limit the the second paragraph of Section 23, Article 6 of the Constitution.
issuances of PTCFORs to his own private army.
Article 6, Sec 23(2). In times of war or other national emergency, the Congress may, by law,
Governor Tan contended that petitioners violated the doctrine on hierarchy of courts when authorize the President, for a limited period and subject to such restrictions as it may
they filed the instant petition directly in the court of last resort, even if both the Court of prescribe, to exercise powers necessary and proper to carry out a declared national policy.
Appeals (CA) and the Regional Trial Courts (RTC) possessed concurrent jurisdiction with the Unless sooner withdrawn by resolution of the Congress, such powers shall cease upon the
Supreme Court under Rule 65. next adjournment thereof.

ISSUE: Whether Governor Tan has the power to declare a state of emergency, and exercise Article 7, Sec 18. The President shall be the Commander-in-Chief of all armed forces of the
the powers enumerated under Proclamation 1-09, specifically the conduct of general Philippines and whenever it becomes necessary, he may call out such armed forces to
searches and seizures. prevent or suppress lawless violence, invasion or rebellion. In case of invasion or rebellion,
when the public safety requires it, he may, for a period not exceeding sixty days, suspend the
RULING: privilege of the writ of habeas corpus or place the Philippines or any part thereof under
martial law. Within forty-eight hours from the proclamation of martial law or the suspension
of the privilege of the writ of habeas corpus, the President shall submit a report in person or
No.Only the President is vested with calling-out powers, as the commander-in-chief of the
in writing to the Congress. The Congress, voting jointly, by a vote of at least a majority of all
Republic
its Members in regular or special session, may revoke such proclamation or suspension,
which revocation shall not be set aside by the President. Upon the initiative of the President,
i. One executive, one commander-in-chief the Congress may, in the same manner, extend such proclamation or suspension for a period
to be determined by the Congress, if the invasion or rebellion shall persist and public safety
As early as Villena v. Secretary of Interior, it has already been established that there is one requires it.
repository of executive powers, and that is the President of the Republic. This means that
when Section 1, Article VII of the Constitution speaks of executive power, it is granted to the The Congress, if not in session, shall, within twenty-four hours following such proclamation
President and no one else. As emphasized by Justice Jose P. Laurel, in his ponencia in Villena: or suspension, convene in accordance with its rules without need of a call.

With reference to the Executive Department of the government, there is one purpose which The power to declare a state of martial law is subject to the Supreme Court’s authority to
is crystal-clear and is readily visible without the projection of judicial searchlight, and that is review the factual basis thereof. By constitutional fiat, the calling-out powers, which is of
the establishment of a single, not plural, Executive. The first section of Article VII of the lesser gravity than the power to declare martial law, is bestowed upon the President alone.
Constitution, dealing with the Executive Department, begins with the enunciation of the As noted in Villena, "(t)here are certain constitutional powers and prerogatives of the Chief
principle that "The executive power shall be vested in a President of the Philippines." This Executive of the Nation which must be exercised by him in person and no amount of
means that the President of the Philippines is the Executive of the Government of the approval or ratification will validate the exercise of any of those powers by any other person.
Philippines, and no other. Such, for instance, is his power to suspend the writ of habeas corpus and proclaim martial
law x x x.
Corollarily, it is only the President, as Executive, who is authorized to exercise emergency
powers as provided under Section 23, Article VI, of the Constitution, as well as what Indeed, while the President is still a civilian, Article II, Section 3of the Constitution mandates
became known as the calling-out powers under Section 7, Article VII thereof. that civilian authority is, at all times, supreme over the military, making the civilian president
the nation’s supreme military leader. The net effect of Article II, Section 3, when read with
ii. The exceptional character of Commander-in-Chief powers dictate that they are exercised Article VII, Section 18, is that a civilian President is the ceremonial, legal and administrative
by one president head of the armed forces. The Constitution does not require that the President must be
possessed of military training and talents, but as Commander-in-Chief, he has the power to
242

direct military operations and to determine military strategy. Normally, he would be


expected to delegate the actual command of the armed forces to military experts; but the
ultimate power is his. As Commander-in-Chief, he is authorized to direct the movements of
the naval and military forces placed by law at his command, and to employ them in the
manner he may deem most effectual.

In the case of Integrated Bar of the Philippines v. Zamora, the Court had occasion to rule that
the calling-out powers belong solely to the President as commander-in-chief:

When the President calls the armed forces to prevent or suppress lawless violence, invasion
or rebellion, he necessarily exercises a discretionary power solely vested in his wisdom. This
is clear from the intent of the framers and from the text of the Constitution itself. The Court,
thus, cannot be called upon to overrule the President’s wisdom or substitute its own.
However, this does not prevent an examination of whether such power was exercised within
permissible constitutional limits or whether it was exercised in a manner constituting grave
abuse of discretion. In view of the constitutional intent to give the President full discretionary
power to determine the necessity of calling out the armed forces, it is incumbent upon the
petitioner to show that the President’s decision is totally bereft of factual basis.

There is a clear textual commitment under the Constitution to bestow on the President full
discretionary power to call out the armed forces and to determine the necessity for the
exercise of such power.
243

EN BANC weaponry, the movement of their heavily armed units in strategic positions, the closure of
the Maguindanao Provincial Capitol, Ampatuan Municipal Hall, Datu Unsay Municipal Hall,
G.R. No. 190293 March 20, 2012 and 14 other municipal halls, and the use of armored vehicles, tanks, and patrol cars with
unauthorized "PNP/Police" markings.
PHILIP SIGFRID A. FORTUN and ALBERT LEE G. ANGELES, Petitioners,
vs. On December 9, 2009 Congress, in joint session, convened pursuant to Section 18, Article VII
GLORIA MACAPAGAL-ARROYO, as Commander-in-Chief and President of the Republic of of the 1987 Constitution to review the validity of the President’s action. But, two days later
the Philippines, EDUARDO ERMITA, Executive Secretary, ARMED FORCES OF THE or on December 12 before Congress could act, the President issued Presidential Proclamation
PHILIPPINES (AFP), or any of their units, PHILIPPINE NATIONAL POLICE (PNP), or any of 1963, lifting martial law and restoring the privilege of the writ of habeas corpus in
their units, JOHN DOES and JANE DOES acting under their direction and Maguindanao.
control, Respondents.
Petitioners Philip Sigfrid A. Fortun and the other petitioners in G.R. 190293, 190294,
DECISION 190301,190302, 190307, 190356, and 190380 brought the present actions to challenge the
constitutionality of President Arroyo’s Proclamation 1959 affecting Maguindanao. But, given
the prompt lifting of that proclamation before Congress could review it and before any
ABAD, J.:
serious question affecting the rights and liberties of Maguindanao’s inhabitants could arise,
the Court deems any review of its constitutionality the equivalent of beating a dead horse.
These cases concern the constitutionality of a presidential proclamation of martial law and
suspension of the privilege of habeas corpus in 2009 in a province in Mindanao which were
Prudence and respect for the co-equal departments of the government dictate that the Court
withdrawn after just eight days.
should be cautious in entertaining actions that assail the constitutionality of the acts of the
Executive or the Legislative department. The issue of constitutionality, said the Court in
The Facts and the Case Biraogo v. Philippine Truth Commission of 2010,1 must be the very issue of the case, that the
resolution of such issue is unavoidable.
The essential background facts are not in dispute. On November 23, 2009 heavily armed
men, believed led by the ruling Ampatuan family, gunned down and buried under shoveled The issue of the constitutionality of Proclamation 1959 is not unavoidable for two reasons:
dirt 57 innocent civilians on a highway in Maguindanao. In response to this carnage, on
November 24 President Arroyo issued Presidential Proclamation 1946, declaring a state of
One. President Arroyo withdrew her proclamation of martial law and suspension of the
emergency in Maguindanao, Sultan Kudarat, and Cotabato City to prevent and suppress
privilege of the writ of habeas corpus before the joint houses of Congress could fulfill their
similar lawless violence in Central Mindanao.
automatic duty to review and validate or invalidate the same. The pertinent provisions of
Section 18, Article VII of the 1987 Constitution state:
Believing that she needed greater authority to put order in Maguindanao and secure it from
large groups of persons that have taken up arms against the constituted authorities in the
Sec. 18. The President shall be the Commander-in-Chief of all armed forces of the Philippines
province, on December 4, 2009 President Arroyo issued Presidential Proclamation 1959
and whenever it becomes necessary, he may call out such armed forces to prevent or
declaring martial law and suspending the privilege of the writ of habeas corpus in that
suppress lawless violence, invasion or rebellion. In case of invasion or rebellion, when the
province except for identified areas of the Moro Islamic Liberation Front.
public safety requires it, he may, for a period not exceeding sixty days, suspend the privilege
of the writ of habeas corpus or place the Philippines or any part thereof under martial law.
Two days later or on December 6, 2009 President Arroyo submitted her report to Congress in Within forty-eight hours from the proclamation of martial law or the suspension of the
accordance with Section 18, Article VII of the 1987 Constitution which required her, within 48 privilege of writ of habeas corpus, the President shall submit a report in person or in writing
hours from the proclamation of martial law or the suspension of the privilege of the writ of to the Congress. The Congress, voting jointly, by a vote of at least a majority of all its
habeas corpus, to submit to that body a report in person or in writing of her action. Members in regular or special session, may revoke such proclamation or suspension, which
revocation shall not be set aside by the President. Upon the initiative of the President, the
In her report, President Arroyo said that she acted based on her finding that lawless men Congress may, in the same manner, extend such proclamation or suspension for a period to
have taken up arms in Maguindanao and risen against the government. The President be determined by the Congress, if the invasion or rebellion shall persist and public safety
described the scope of the uprising, the nature, quantity, and quality of the rebels’ requires it.
244

The Congress, if not in session, shall, within twenty-four hours following such proclamation Two. Since President Arroyo withdrew her proclamation of martial law and suspension of the
or suspension, convene in accordance with its rules without any need of a call. privilege of the writ of habeas corpus in just eight days, they have not been meaningfully
implemented. The military did not take over the operation and control of local government
xxxx units in Maguindanao. The President did not issue any law or decree affecting Maguindanao
that should ordinarily be enacted by Congress. No indiscriminate mass arrest had been
reported. Those who were arrested during the period were either released or promptly
Although the above vests in the President the power to proclaim martial law or suspend the
charged in court. Indeed, no petition for habeas corpus had been filed with the Court
privilege of the writ of habeas corpus, he shares such power with the Congress. Thus:
respecting arrests made in those eight days. The point is that the President intended by her
action to address an uprising in a relatively small and sparsely populated province. In her
1. The President’s proclamation or suspension is temporary, good for only 60 days; judgment, the rebellion was localized and swiftly disintegrated in the face of a determined
and amply armed government presence.
2. He must, within 48 hours of the proclamation or suspension, report his action in
person or in writing to Congress; In Lansang v. Garcia,3 the Court received evidence in executive session to determine if
President Marcos’ suspension of the privilege of the writ of habeas corpus in 1971 had
3. Both houses of Congress, if not in session must jointly convene within 24 hours sufficient factual basis. In Aquino, Jr. v. Enrile,4while the Court took judicial notice of the
of the proclamation or suspension for the purpose of reviewing its validity; and factual bases for President Marcos’ proclamation of martial law in 1972, it still held hearings
on the petitions for habeas corpus to determine the constitutionality of the arrest and
4. The Congress, voting jointly, may revoke or affirm the President’s proclamation detention of the petitioners. Here, however, the Court has not bothered to examine the
or suspension, allow their limited effectivity to lapse, or extend the same if evidence upon which President Arroyo acted in issuing Proclamation 1959, precisely because
Congress deems warranted. it felt no need to, the proclamation having been withdrawn within a few days of its issuance.

It is evident that under the 1987 Constitution the President and the Congress act in tandem Justice Antonio T. Carpio points out in his dissenting opinion the finding of the Regional Trial
in exercising the power to proclaim martial law or suspend the privilege of the writ of habeas Court (RTC) of Quezon City that no probable cause exist that the accused before it committed
corpus. They exercise the power, not only sequentially, but in a sense jointly since, after the rebellion in Maguindanao since the prosecution failed to establish the elements of the crime.
President has initiated the proclamation or the suspension, only the Congress can maintain But the Court cannot use such finding as basis for striking down the President’s proclamation
the same based on its own evaluation of the situation on the ground, a power that the and suspension. For, firstly, the Court did not delegate and could not delegate to the RTC of
President does not have. Quezon City its power to determine the factual basis for the presidential proclamation and
suspension. Secondly, there is no showing that the RTC of Quezon City passed upon the same
evidence that the President, as Commander-in-Chief of the Armed Forces, had in her
Consequently, although the Constitution reserves to the Supreme Court the power to review possession when she issued the proclamation and suspension.
the sufficiency of the factual basis of the proclamation or suspension in a proper suit, it is
implicit that the Court must allow Congress to exercise its own review powers, which is
automatic rather than initiated. Only when Congress defaults in its express duty to defend The Court does not resolve purely academic questions to satisfy scholarly interest, however
the Constitution through such review should the Supreme Court step in as its final rampart. intellectually challenging these are.5 This is especially true, said the Court in Philippine
The constitutional validity of the President’s proclamation of martial law or suspension of the Association of Colleges and Universities v. Secretary of Education,6 where the issues "reach
writ of habeas corpus is first a political question in the hands of Congress before it becomes a constitutional dimensions, for then there comes into play regard for the court’s duty to avoid
justiciable one in the hands of the Court. decision of constitutional issues unless avoidance becomes evasion." The Court’s duty is to
steer clear of declaring unconstitutional the acts of the Executive or the Legislative
department, given the assumption that it carefully studied those acts and found them
Here, President Arroyo withdrew Proclamation 1959 before the joint houses of Congress, consistent with the fundamental law before taking them. "To doubt is to sustain." 7
which had in fact convened, could act on the same. Consequently, the petitions in these
cases have become moot and the Court has nothing to review. The lifting of martial law and
restoration of the privilege of the writ of habeas corpus in Maguindanao was a supervening Notably, under Section 18, Article VII of the 1987 Constitution, the Court has only 30 days
event that obliterated any justiciable controversy.2 from the filing of an appropriate proceeding to review the sufficiency of the factual basis of
the proclamation of martial law or the suspension of the privilege of the writ of habeas
corpus. Thus –
245

The Supreme Court may review, in an appropriate proceeding filed by any citizen, the WHEREFORE, the Court DISMISSES the consolidated petitions on the ground that the same
sufficiency of the factual basis of the proclamation of martial law or the suspension of the have become moot and academic.
privilege of the writ of habeas corpus or the extension thereof, and must promulgate its
decision thereon within thirty days from its filing. (Emphasis supplied) SO ORDERED.

More than two years have passed since petitioners filed the present actions to annul ROBERTO A. ABAD
Proclamation 1959.1âwphi1 When the Court did not decide it then, it actually opted for a
default as was its duty, the question having become moot and academic.
CERTIFICATION
Justice Carpio of course points out that should the Court regard the powers of the President
and Congress respecting the proclamation of martial law or the suspension of the privilege of
Pursuant to Section 13, Article VIII of the Constitution, it is hereby certified that the
the writ of habeas corpus as sequential or joint, it would be impossible for the Court to
conclusions in the above Decision had been reached in consultation before the case was
exercise its power of review within the 30 days given it.
assigned to the writer of the opinion of the Court.

But those 30 days, fixed by the Constitution, should be enough for the Court to fulfill its duty
RENATO C. CORONA
without pre-empting congressional action. Section 18, Article VII, requires the President to
Chief Justice
report his actions to Congress, in person or in writing, within 48 hours of such proclamation
or suspension. In turn, the Congress is required to convene without need of a call within 24
hours following the President’s proclamation or suspension. Clearly, the Constitution calls for DISSENTING OPINION
quick action on the part of the Congress. Whatever form that action takes, therefore, should
give the Court sufficient time to fulfill its own mandate to review the factual basis of the CARPIO, J.:
proclamation or suspension within 30 days of its issuance.
I dissent.
If the Congress procrastinates or altogether fails to fulfill its duty respecting the proclamation
or suspension within the short time expected of it, then the Court can step in, hear the The Cases
petitions challenging the President’s action, and ascertain if it has a factual basis. If the Court
finds none, then it can annul the proclamation or the suspension. But what if the 30 days
given it by the Constitution proves inadequate? Justice Carpio himself offers the answer in These are consolidated petitions for the writs of certiorari and prohibition challenging the
his dissent: that 30-day period does not operate to divest this Court of its jurisdiction over constitutionality of Presidential Proclamation No. 1959, which declared a state of martial law
the case. The settled rule is that jurisdiction once acquired is not lost until the case has been and suspended the privilege of the writ of habeas corpus in the Province of Maguindanao,
terminated. except for identified areas of the Moro Islamic Liberation Front.

The problem in this case is that the President aborted the proclamation of martial law and The Antecedents
the suspension of the privilege of the writ of habeas corpus in Maguindanao in just eight
days. In a real sense, the proclamation and the suspension never took off. The Congress itself In the morning of 23 November 2009, fifty-seven (57) innocent civilians met their tragic and
adjourned without touching the matter, it having become moot and academic. untimely death in a gruesome massacre unequaled in recent history,1 considered to be the
Philippines’ worst case of election-related violence. Brutally killed were female family
Of course, the Court has in exceptional cases passed upon issues that ordinarily would have members of then Buluan Vice Mayor Esmael "Toto" Mangudadatu (Mangudadatu), including
been regarded as moot. But the present cases do not present sufficient basis for the exercise his wife and sisters, and members of the press who were part of a convoy on the way to
of the power of judicial review. The proclamation of martial law and the suspension of the Shariff Aguak in Maguindanao. Mangudadatu’s wife was bringing with her Mangudadatu’s
privilege of the writ of habeas corpus in this case, unlike similar Presidential acts in the late certificate of candidacy for Governor of Maguindanao for filing with the Provincial Office of
60s and early 70s, appear more like saber-rattling than an actual deployment and arbitrary the Commission on Elections in Shariff Aguak. Five of the victims were not part of the convoy
use of political power. but happened to be traveling on the same highway.2
246

In its Consolidated Comment dated 14 December 2009, the Office of the Solicitor General 7. Upon reaching Barangay Masalay, Ampatuan, the soldiers on foot patrol found
(OSG), representing public respondents, narrated the harrowing events which unfolded on dead bodies, bloodied and scattered on the ground and inside the four (4) vehicles
that fateful day of 23 November 2009, to wit: used by the convoy. Three (3) newly covered graves and a back hoe belonging to
the Maguindanao Provincial Government parked nearby with its engine still
3. Vice Mayor Mangudadatu confirmed having received reports that his political running were found at the site. When the graves were dug up by the soldiers,
rivals (Ampatuans) were planning to kill him upon his filing of a certificate of twenty four (24) dead bodies were found in the first grave; six (6) dead bodies with
candidacy (COC) for the gubernatorial seat in Maguindanao. Believing that the three (3) vehicles, particularly a Toyota Vios with the seal of the Tacurong City
presence of women and media personalities would deter any violent assault, he Government, a Tamaraw FX and an L300 owned by the media outfit UNTV were
asked his wife and female relatives to file his COC and invited several media found in the second grave; and five (5) more dead bodies were recovered from the
reporters to cover the event. third grave, yielding 35 buried dead bodies and, together with other cadavers,
resulted in a total of fifty seven (57) fatalities.
4. At around 10 a.m., the convoy stopped at a designated PNP checkpoint along the
highway of Ampatuan, Maguindanao manned by the Maguindanao 1508th 9. Examination of the bodies revealed that most, if not all, of the female victims’
Provincial Mobile Group, particularly, Eshmail Canapia and Takpan Dilon. While at a pants were found unzipped, and their sexual organs mutilated and mangled. Five
stop, they were approached by about one hundred (100) armed men. The armed (5) of them were tested positive for traces of semen, indicative of sexual abuse
men pointed their weapons at the members of the 1508th Provincial Mobile Group while some of the victims were shot in the genital area. The genitalia of Genelyn
manning the check point, and threatened them to refrain from interfering. The Mangudadatu was lacerated four (4) times, and blown off by a gun fire, and her
members of the convoy were then ordered to alight from their vehicles and to lie body horrifyingly mutilated. Two of the women killed were pregnant, while another
face down on the ground, as the armed men forcibly took their personal two were lawyers. Twenty-nine (29) of the casualties were media personnel.
belongings. Subsequently, all members of the convoy were ordered to board their Almost all gun shot injuries were on the heads of the victims, rendering them
vehicles. They were eventually brought by the armed men to the hills in Barangay unrecognizable albeit two (2) bodies remain unidentified. Those found in the
Masalay, Ampatuan, about 2.5 kilometers from the checkpoint. graves were coarsely lumped like trash, and some of the victims were found
hogtied. All the dead bodies bear marks of despicable torture, contempt and
outrageous torment.3
5. At about the same time, Vice Mayor Mangudadatu received a call from his wife
Genelyn who, in a trembling voice, told him that a group of more or less 100 armed
men stopped their convoy, and that Datu Unsay Mayor Andal Ampatuan, Jr. was A day after the carnage, on 24 November 2009, former President Gloria Macapagal-Arroyo
walking towards her, and was about to slap her face. After those last words were (President Arroyo) issued Proclamation No. 1946, declaring a state of emergency in the
uttered, the phone line went dead and her cellphone could not be contacted any provinces of Maguindanao and Sultan Kudarat, and in the City of Cotabato, "to prevent and
longer. Alarmed that his wife and relatives, as well as the media personalities were suppress the occurrence of similar other incidents of lawless violence in Central Mindanao."
in grave danger, Vice Mayor Mangudadatu immediately reported the incident to The full text of Proclamation No. 1946 reads:
the Armed Forces of the Philippines.
DECLARING A STATE OF EMERGENCY IN CENTRAL MINDANAO
6. In the afternoon of the same day, soldiers – aboard two army trucks led by Lt.
Col. Rolando Nerona, Head of the Philippine Army’s 64th Infantry Battalion – went WHEREAS, on November 23, 2009, several persons, including women and members of media
to the town of Ampatuan to confirm the report. At around 3 p.m., they passed by were killed in a violent incident which took place in Central Mindanao;
the checkpoint along the highway in Ampatuan manned by the 1508th Provincial
Mobile Group and asked whether they were aware of the reported abduction. WHEREAS, there is an urgent need to prevent and suppress the occurrence of similar other
Members of the 1508th Provincial Mobile Group denied having knowledge of what incidents of lawless violence in Central Mindanao;
they have witnessed at around 10 in the morning purportedly out of fear of
retaliation from the powerful Ampatuan clan. Nevertheless, P/CI Sukarno Adil
NOW, THEREFORE, I, GLORIA MACAPAGAL-ARROYO, President of the Republic of the
Dicay, the head of the Mobile Group, instructed P/INSP Diongon to accompany the
Philippines, by virtue of the powers vested in me by the Constitution and by law, do hereby
military on foot patrol as they conduct their operation relative to the reported
proclaim, as follows:
abduction.
247

SECTION 1. The Provinces of Maguindanao and Sultan Kudarat and the City of WHEREAS, the condition of peace and order in the province of Maguindanao has
Cotabato are hereby placed under a state of emergency for the purpose of deteriorated to the extent that the local judicial system and other government mechanisms
preventing and suppressing lawless violence in the aforesaid jurisdiction. in the province are not functioning, thus endangering public safety;

SECTION 2. The Armed Forces of the Philippines (AFP) and the Philippine National WHEREAS, the Implementing Operational Guidelines of the GRP-MILF Agreement on the
Police (PNP) are hereby ordered to undertake such measures as may be allowed by General Cessation of Hostilities dated 14 November 1997 provides that the following is
the Constitution and by law to prevent and suppress all incidents of lawless considered a prohibited hostile act: "x x x establishment of checkpoints except those
violence in the said jurisdiction. necessary for the GRP’s enforcement and maintenance of peace and order; and, for the
defense and security of the MILF in their identified areas, as jointly determined by the GRP
SECTION 3. The state of emergency covering the Provinces of Maguindanao and and MILF. x x x"
Sultan Kudarat and the City of Cotabato shall remain in force and effect until lifted
or withdrawn by the President.4 NOW, THEREFORE, I, GLORIA MACAPAGAL-ARROYO, President of the Republic of the
Philippines, by virtue of the powers vested in me by the Constitution and by law, do hereby
On 4 December 2009, President Arroyo issued Proclamation No. 1959, declaring martial law proclaim, as follows:
and suspending the privilege of the writ of habeas corpus (writ) in the Province of
Maguindanao, except for the identified areas of the Moro Islamic Liberation Front (MILF). SECTION 1. There is hereby declared a state of martial law in the province of
The full text of Proclamation No. 1959, signed by President Arroyo and attested by Executive Maguindanao, except for the identified areas of the Moro Islamic Liberation Front
Secretary Eduardo Ermita, reads: as referred to in the Implementing Operational Guidelines of the GRP-MILF
Agreement on the General Cessation of Hostilities.
PROCLAMATION NO. 1959
SECTION 2. The privilege of the writ of habeas corpus shall likewise be suspended
PROCLAIMING A STATE OF MARTIAL LAW AND SUSPENDING THE PRIVILEGE OF THE WRIT in the aforesaid area for the duration of the state of martial law.6
OF HABEAS CORPUS IN THE PROVINCE OF MAGUINDANAO, EXCEPT FOR CERTAIN AREAS
On 6 December 2009, President Arroyo submitted her Report to Congress in accordance with
WHEREAS, Proclamation No. 1946 was issued on 24 November 2009 declaring a state of the provision in Section 18, Article VII of the 1987 Constitution, which states that "within
emergency in the provinces of Maguindanao, Sultan Kudarat and the City of Cotabato for the forty-eight hours from the proclamation of martial law or the suspension of the privilege of
purpose of preventing and suppressing lawless violence in the aforesaid areas; the writ of habeas corpus, the President shall submit a report in person or in writing to the
Congress." In her Report, President Arroyo presented the following justifications for imposing
martial law and suspending the writ in Maguindanao, to wit:
WHEREAS, Section 18, Art.VII of the Constitution provides that "x x x In case of invasion or
rebellion, when the public safety requires it, (the President) may, for a period not exceeding
sixty days, suspend the privilege of the writ of habeas corpus or place the Philippines or any Pursuant to the provision of Section 18, Article VII of the 1987 Constitution, the President of
part thereof under martial law. x x x" the Republic of the Philippines is submitting the hereunder Report relative to Proclamation
No. 1959 "Proclaiming a State of Martial Law and Suspending the Privilege of the Writ of
Habeas Corpus in the Province of Maguindanao, except for Certain Areas," which she issued
WHEREAS, R.A. No. 69865 provides that the crime of rebellion or insurrection is committed
on 04 December 2009, as required by public safety, after finding that lawless elements have
by rising publicly and taking arms against the Government for the purpose of x x x depriving
taken up arms and committed public uprising against the duly constituted government and
the Chief Executive or the Legislature, wholly or partially, of any of their powers or
against the people of Maguindanao, for the purpose of removing from the allegiance to the
prerogatives."
Government or its laws, the Province of Maguindanao, and likewise depriving the Chief
Executive of her powers and prerogatives to enforce the laws of the land and to maintain
WHEREAS, heavily armed groups in the province of Maguindanao have established positions public order and safety, to the great damage, prejudice and detriment of the people in
to resist government troops, thereby depriving the Executive of its powers and prerogatives Maguindanao and the nation as a whole.
to enforce the laws of the land and to maintain public order and safety;
The capture of identified leader Mayor Andal Ampatuan, Jr. would have resulted in the
expeditious apprehension and prosecution of all others involved in the gruesome massacre,
248

but the situation proved the contrary. The Ampatuan group backed by formidable group of II. A group with more or less 200 armed rebels has moved from Old
armed followers, have since used their strength and political position to deprive the Chief Maganoy into an offensive position.
Executive of her power to enforce the law and to maintain public order and safety. More
importantly, a separatist group based in Maguindanao has joined forces with the Ampatuans III. More or less 80 fully armed rebels remain in Tuka, Mamasapano.
for this purpose. These are the facts:
IV. More or less 50 fully armed rebels led by a former MNLF Commander
1. Local government offices in the province of Maguindanao were closed and are in offensive position in Barangay Baital, Rajah Buayan.
ranking local government officials refused to discharge their functions, which
hindered the investigation and prosecution team from performing their tasks;
V. More or less 70 fully armed rebels with two (2) M60 LMG remain in
offensive position in the vicinity of Barangay Kagwaran, Barangay
2. The Local Civil Registrar of Maguindanao refused to accept the registration of the Iginampong, Datu Unsay (right side of Salvo-General Santos City national
death certificates of the victims purportedly upon the orders of Andal Ampatuan highway).
Sr.;
VI. More or less 60 fully armed rebels with four (4) M60 LMG remain in
3. The local judicial system has been crippled by the absence or non-appearance of offensive position in the vicinity of Kinugitan, the upper portion of
judges of local courts, thereby depriving the government of legal remedies in their Barangay Maitumaig, Datu Unsay.
prosecutorial responsibilities (i.e. issuance of warrants of searches, seizure and
arrest). While the Supreme Court has designated an Acting Presiding Judge from
VII. Kagui Akmad Ampatuan was sighted in Sultan Sa Barongis with 400
another province, the normal judicial proceedings could not be carried out in view
armed rebels. Locals heard him uttered "PATAYAN NA KUNG PATAYAN."
of threats to their lives or safety, prompting government to seek a change of venue
of the criminal cases after informations have been filed.
VIII. More or less 100 armed rebels led by one of the identified leaders in
the massacre have been sighted at the quarry of Barangay Lagpan,
Duly verified information disclosed that the Ampatuan group is behind the closing down of
boundary of Rajah Buayan and Sultan Sa Barongis. The group is armed
government offices, the refusal of local officials to discharge their functions and the
with one (1) 90RR, one (1) cal 50 LMG, two (2) cal 30 LMG, two (2) 60 mm
simultaneous absence or non-appearance of judges in local courts.
mortar and assorted rifles.

Detailed accounts pertaining to the rebel armed groups and their active movements in
The strength of the rebels is itself estimated to be around 800 with about 2,000 firearms
Maguindanao have been confirmed:
(Fas). These forces are concentrated in the following areas in Maguindanao which are
apparently also their political stronghold:
I. As of November 29, 2009, it is estimated that there are about 2,413 armed
combatants coming from the municipalities of Shariff Aguak, Datu Unsay, Datu
The existence of this armed rebellion is further highlighted by the recent recovery of high
Salibo, Mamasapano, Datu Saudi Ampatuan (Dikalungan), Sultan Sa Barungis, Datu
powered firearms and ammunitions from the 400 security escorts of Datu Andal Ampatuan
Piang, Guindulungan, and Talayan, who are in possession of around 2,000
Sr.
firearms/armaments.

Indeed, the nature, quantity and quality of their weaponry, the movement of heavily
II. The Ampatuan group has consolidated a group of rebels consisting of 2,413
armed rebels in strategic positions, the closure of the Maguindanao Provincial Capitol,
heavily armed men, with 1,160 of them having been strategically deployed in
Ampatuan Municipal Hall, Datu Unsay Municipal Hall, and fourteen other municipal halls,
Maguindanao. Validated information on the deployment of rebels are as follows:
and the use of armored vehicles, tanks and patrol cars with unauthorized "PNP/Police"
markings, all together confirm the existence of armed public uprising for the political
I. Around five hundred (500) armed rebels with 2 "Sanguko" armored purpose of:
vehicles are in offensive position in the vicinity of Kakal, Ampatuan,
Dimampao, Mamasapano and Sampao Ampatuan.
1. removing allegiance from the national government of the Province of Maguindanao;
and,
249

2. depriving the Chief Executive of her powers and prerogatives to enforce the laws of the 6. G.R. No. 190356 is a petition for prohibition, with an application for the issuance
land and to maintain public order and safety. of a temporary restraining order and/or a writ of preliminary injunction, assailing
the constitutionality and the sufficiency of the factual basis of Proclamation No.
While the government is at present conducting legitimate operations to address the on- 1959, declaring a state of martial law in the province of Maguindanao (except for
going rebellion, public safety still requires the continued implementation of martial law identified areas of the MILF) and suspending the writ in the same area.
and the suspension of the privilege of the writ of habeas corpus in the Province of
Maguindanao until the time that such rebellion is completely quelled.7 (Emphasis supplied) 7. G.R. No. 190380 is a petition for certiorari assailing the validity of Proclamation
No. 1959, declaring a state of martial law in the province of Maguindanao, except
In the meantime, the present petitions were filed impugning the constitutionality of for the identified areas of the MILF, and suspending the writ in the same area.
Proclamation No. 1959.
On 9 December 2009, Congress convened in joint session pursuant to Section 18, Article VII
1. G.R. No. 190293 is a petition "for the issuance of a temporary restraining order of the 1987 Constitution, which provides, "The Congress, if not in session, shall, within
and writs of prohibition and preliminary prohibitory injunction (1) to declare twenty-four hours following such proclamation [of martial law] or suspension [of the writ],
Proclamation No. 1959 or any act, directive or order arising from or connected to it convene in accordance with its rules without need of a call."
as unconstitutional, and (2) to enjoin public respondents from further enforcing the
same." Meanwhile, eight days after the declaration of martial law, on 12 December 2009, President
Arroyo issued Proclamation No. 1963 lifting martial law and restoring the writ in
2. G.R. No. 190294 is a petition for certiorari assailing the constitutionality of Maguindanao. The full text of Proclamation No. 1963, signed by President Arroyo and
Proclamation No. 1959 "for gross insufficiency of the factual basis in proclaiming a attested by Executive Secretary Eduardo Ermita, reads:
state of martial law and suspending the [writ] in the Province of Maguindanao." It
prayed for the issuance of a writ of prohibition under Section 2 of Rule 65 to enjoin PROCLAMATION NO. 1963
and prohibit respondents from enforcing Proclamation No. 1959.
PROCLAIMING THE TERMINATION OF THE STATE OF MARTIAL LAW AND THE RESTORATION
3. G.R. No. 190301 is a petition seeking "the nullification of Proclamation No. 1959, OF THE PRIVILEGE OF THE WRIT OF HABEAS CORPUS IN THE PROVINCE OF MAGUINDANAO
proclaiming a state of martial law and suspending the [writ] in the province of
Maguindanao, except for certain areas, as it is patently illegal and unconstitutional WHEREAS, Proclamation No. 1946 was issued on 24 November 2009 declaring a state of
for lack of any factual basis." emergency in the provinces of Maguindanao, Sultan Kudarat and the City of Cotabato for the
purpose of preventing and suppressing lawless violence in the aforesaid areas;
4. G.R. No. 190302 is a petition for certiorari to declare Proclamation No. 1959 as
null and void for being unconstitutional, and for prohibition to enjoin respondents WHEREAS, by virtue of the powers granted under Section 18, Article VII of the Constitution,
from further actions or proceedings in enforcing or implementing Proclamation No. the President of the Philippines promulgated Proclamation No. 1959 on December 4, 2009,
1959. proclaiming a state of martial law and suspending the privilege of the writ of Habeas Corpus
in the province of Maguindanao, except for certain areas;
5. G.R. No. 190307 is a petition for certiorari, prohibition, and mandamus with a
prayer for a preliminary prohibitory injunction and/or a temporary restraining WHEREAS, the Armed Forces of the Philippines and the Philippine National Police have
order, and/or a petition for review pursuant to Article VII, Section 18, paragraph 3 reported that over six hundred (600) persons who allegedly rose publicly and took up arms
of the 1987 Constitution, asking the Court to declare that then Executive Secretary against the Government have surrendered or have been arrested or detained;
Eduardo Ermita committed grave abuse of discretion amounting to lack or excess
of jurisdiction when he signed, in the name of President Arroyo, Proclamation No.
WHEREAS, the Armed Forces of the Philippines and the Philippine National Police have
1959. The petition also prayed for the issuance of a Temporary Restraining Order
reported that the areas where heavily armed groups in the province of Maguindanao
and/or preliminary prohibitory injunction, prohibiting respondents, and anyone
established positions to resist government troops have been cleared;
acting under their authority, stead, or behalf, from implementing Proclamation No.
1959 during the pendency of the case.
250

WHEREAS, the court and prosecutors’ offices of Cotabato City have resumed normal working with their employees going on mass leave and their respective offices were closed and not
hours, paving the way for the criminal justice system in Maguindanao to be restored to functioning. The Regional Trial Courts of the area are not functioning, refused to accept the
normalcy; application for search warrants for violation of PD 1866 to authorize the search of the
properties of the heads of the rebellion; and that there was undue delay in the issuance of
WHEREAS, the Vice-Governor of the Autonomous Region of Muslim Mindanao has assumed court processes despite the exigency of the situation.
as Acting Governor, paving the way for the restoration of the functioning of government
mechanisms in the province of Maguindanao; CONTRARY TO LAW.12

NOW, THEREFORE, I, GLORIA MACAPAGAL-ARROYO, President of the Republic of the On the next day, 10 December 2009, accused Ampatuan, et al. filed an Urgent Omnibus
Philippines, by virtue of the powers vested in me by the Constitution and by law, do hereby Motion, which included a motion for judicial determination of probable cause for the offense
revoke Proclamation No. 1959 and proclaim the termination of the state of martial law and charged. On the same day, the Acting Presiding Judge of RTC-Cotabato issued an Order,
the restoration of the privilege of the writ of habeas corpus in the province of Maguindanao; stating that "the Court needs time to go over the resolution finding probable cause against
provided that Proclamation No. 1946 shall continue to be in force and effect.8 the accused Datu Andal Ampatuan, Sr., [et al.]."

In the Resolutions dated 8 and 15 December 2009,9 the Court consolidated the petitions and On 1 February 2010, the Regional Trial Court of Quezon City received the records of the case,
required the Office of the Solicitor General and the respondents to comment on the pursuant to the Supreme Court’s En Banc Resolution, dated 12 January 2010, which ordered
petitions. the transfer of venue of the rebellion case to Quezon City. The case, docketed as Criminal
Case No. Q-10-162667 and entitled People of the Philippines v. Datu Andal Ampatuan, Sr., et
In a Resolution dated 12 January 2010, the Court resolved "to appoint as amici curiae Justice al., was raffled to Branch 77 of the Regional Trial Court of Quezon City (RTC-Quezon City) on
Vicente Mendoza, Senator Joker Arroyo, and Father Joaquin Bernas, [S.J.] and request them 2 February 2010.
to submit their respective Amicus Brief on the questions to be addressed by the parties."10
On 3 February 2010, the accused filed an Urgent Motion praying for the issuance of an order
Meanwhile, on 9 December 2009, an Information for rebellion was filed before the Regional suspending the transfer of custody of all the accused pending the resolution of their motion
Trial Court, Branch 15, Cotabato City (RTC-Cotabato), against Ampatuan, et al.11 The for judicial determination of probable cause.
information reads:
On 26 March 2010, the RTC-Quezon City dismissed the charge of rebellion for lack of
That on or about 27th day of November, 2009, and continuously thereafter, until the present probable cause, to wit:
time, in Maguindanao Province and within the jurisdiction of this Honorable Court, accused
Datu Andal Ampatuan, Sr., Datu Zaldy Uy Ampatuan, Datu Akmad Tato Ampatuan, Datu After a careful and judicious scrutiny of the evidence forming part of the records and those
Anwar Ampatuan and Datu Sajid Islam Uy Ampatuan as heads of the rebellion, conspiring, adduced by the prosecution during the hearing on the motion for judicial determination of
confederating and cooperating with each other, as well as with the other accused as probable cause, the Court is convinced that there exist[s] no probable cause to hold under
participants or executing the commands of others in the rebellion and also with other John detention and to indict the accused for rebellion.
Does whose whereabouts and identities are still unknown, the said accused, who are heads
of the rebellion, did then and there willfully, unlawfully and feloniously help, support, Rebellion under Article 134 of the Revised Penal Code is committed –
promote, maintain, cause, direct and/or command their co-accused who are their followers
to rise publicly and take arms against the Republic of the Philippines, or otherwise participate
[B]y rising publicly and taking arms against the Government for the purpose of removing
in such armed public uprising, for the purpose of removing allegiance to the government or
from the allegiance to said Government or its laws, the territory of the Republic of the
its laws, the territory of the Republic of the Philippines or any part thereof or depriving the
Philippines or any part thereof, or any body of land, naval, or other armed forces or depriving
Chief Executive of any of her powers or prerogatives as in fact they have been massing up
the Chief Executive or the Legislature, wholly or partially, of any of their powers or
armed men and organizing assemblies, as a necessary means to commit the crime of
prerogatives.
rebellion, and in furtherance thereof, have then and there committed acts preventing public
prosecutors from being available to conduct inquest and preliminary investigations. There
were massive formations of numerous armed civilians supported by armored vehicles and The elements of the offense are:
under the command of the Ampatuans who have formed a private army to resist
government troops; that the local provincial government of Maguindanao could not function
251

1. That there be a (a) public uprising and (b) taking arms against the Government; The statements of prosecution witnesses Mangacop and Dingcong are general allegations.
and Their statements do not show that the accused were responsible for the mass leave of
officials and employees of the local government units. There is no evidence to show that the
2. That the purpose of the uprising or movement is either – accused actually prevented the local officials and employees from reporting to their offices.

(a) to remove from the allegiance to said Government or its laws: The evidence will show that the Department of Interior and Local Government and the
Philippine National Police closed down these offices, without any justifiable reasons. In fact,
there were news footages which showed that many employees were caught by surprise on
(1) the territory of the Philippines or any part thereof; or
the unexpected closure of their offices.

(2) any body of land, naval, or other armed forces; or


It is alleged in the Information that the courts were no longer functioning in Cotabato City
and in Maguindanao province, which have jurisdiction over the place of the commission of
(b) to deprive the Chief Executive or Congress, wholly or partially, of any the massacre. The factual circumstances, however, belie said allegation. This Court takes
of their powers and prerogatives. judicial notice of the fact that no less than the Supreme Court of the Republic of the
Philippines had denied the allegation that civilian courts were or are no longer functioning in
The essential element of public armed uprising against the government is lacking. There were Maguindanao.
no masses or multitudes involving crowd action done in furtherance of a political end. So,
even assuming that there was uprising, there is no showing that the purpose of the uprising WHEREFORE, premises considered, the Court finds that there exists no probable cause to
is political, that is, to overthrow the duly constituted government in order to establish indict and hold under detention the accused for rebellion. Accordingly, the instant case is
another form of government. In other words, the second element is also absent. hereby dismissed and the accused-movants are hereby ordered released from further
detention, unless they are held by a court of law for other lawful cause/s.
x x x It is quite interesting that the prosecution failed to present any particular instance
where the accused had directly or indirectly prevented government prosecutors from Let this Order be served personally upon the accused-movants, through the responsible
performing their job relative to the prosecution of the suspects in the infamous officers of the law having custody over them, who are hereby directed to release the accused
Maguindanao massacre. from detention immediately upon receipt hereof.

On the contrary, documentary evidence on record shows that the alleged principal suspect in SO ORDERED.13
the mass killings, Datu Andal Ampatuan, Jr., was made to undergo inquest proceedings at
General Santos City, immediately after he was taken into custody by law enforcement
In an Order dated 28 May 2010, the RTC-Quezon City denied the prosecution’s motion for
authorities. This alone belies the prosecution’s theory that the prosecutors were not
reconsideration of the Order dated 26 March 2010.
available to conduct inquest and preliminary investigations relative to the mass killings in the
Municipality of Ampatuan, Province of Maguindanao.
The DOJ filed a petition for certiorari14 before the Court of Appeals assailing the dismissal of
the rebellion charges against accused Ampatuan, et al.
x x x [T]he intelligence reports presented by the military and police are unfounded. The
reports do not suggest that the alleged armed groups loyal to the accused are initiating
violent and hostile actions, whether directly or indirectly, against government security forces. In a Decision promulgated on 15 December 2011,15 the Court of Appeals denied the petition
Even the discovery and confiscation of large cache of firearm and ammunitions, allegedly for certiorari. Quoting the findings of the RTC-Quezon City, the Court of Appeals held that
belonging to the Ampatuans, cannot be considered as an act of rebellion. In fact, the firearms there is no probable cause as there is no showing that all the elements of the crime of
and ammunitions were subsequently unearthed, recovered and confiscated from different rebellion are present. The Court of Appeals stated that "a review of its own narration of
places. The government security forces should have been able to engage and neutralize the events only lends to the belief that the rebellion existed only in the minds of the
reported armed groups on the basis of its intelligence reports confirming their size, strength complainants." The Court of Appeals ruled that there was no armed public uprising, finding
and whereabouts. "no proof that armed groups were massing up and were planning to instigate civil
disobedience and to challenge the government authorities for political ends."
252

The Issues Discussion

The crux of the present controversy is the constitutionality of Proclamation No. 1959, I dissent from the majority's dismissal of the petitions as moot. I find Proclamation No. 1959
declaring martial law and suspending the writ in Maguindanao. The threshold issue before unconstitutional for lack of factual basis as required in Section 18, Article VII of the 1987
this Court is whether there is sufficient factual basis for the issuance of Proclamation No. Constitution for the declaration of martial law and suspension of the writ. The majority in
1959 based on the stringent requirements set forth in Section 18, Article VII of the 1987 effect refuses to exercise this Court’s constitutional power in Section 18 of Article VII, to
Constitution. "review, in an appropriate proceeding filed by any citizen, the sufficiency of the factual basis
of the proclamation of martial law or the suspension of the privilege of the writ or the
In its 15 December 2009 Resolution, the Court additionally posed the following questions for extension thereof."
resolution:
Before proceeding to the substantive issues, I shall first discuss the issue on locus standi.
1. Whether the issuance of Proclamation No. 1963, lifting martial law and restoring
the writ in Maguindanao, rendered the issues raised in the present petitions moot In its Comment Re: Resolution dated 15 December 2009, the OSG questioned the legal
and academic; standing of petitioners in challenging the constitutionality of Proclamation No. 1959. The
OSG argued that the phrase "any citizen" in Section 18, Article VII of the 1987 Constitution
2. Whether the term "rebellion" in Section 18, Article VII of the 1987 Constitution must be read in conjunction with the phrase "appropriate proceeding." Since petitioners
has the same meaning as the term "rebellion" that is defined in Article 134 of the deemed the original actions for certiorari and prohibition as the appropriate proceeding
Revised Penal Code; referred to in Section 18, Article VII of the Constitution, petitioners must satisfy the
requirements under Rule 65 of the Rules of Court, one of which is the institution of the
action by the aggrieved party. The OSG pointed out that none of the petitioners qualify as an
3. Whether the declaration of martial law or the suspension of the writ authorizes
aggrieved party.
warrantless arrests, searches and seizures;

This is error.
4. Whether the declaration of martial law or the suspension of the writ is a joint
and sequential function of the President and Congress such that, without
Congressional action on the proclamation either affirming or revoking it, the "Legal standing" or locus standi has been defined as a personal and substantial interest in the
President having in the meantime lifted the declaration and restored the writ, this case such that the party has sustained or will sustain direct injury as a result of the
Court has nothing to review; governmental act that is being challenged.16 In case of a suit questioning the sufficiency of
the factual basis of the proclamation of martial law or suspension of the writ, such as here,
Section 18, Article VII of the Constitution expressly provides:
5. If the constitutional power of this Court to review the factual basis of the
declaration of martial law or suspension of the writ can be exercised
simultaneously with the constitutional power of Congress to revoke the declaration The Supreme Court may review, in an appropriate proceeding filed by any citizen, the
or suspension, and the decision of this Court conflicts with the decision of sufficiency of the factual basis of the proclamation of martial law or the suspension of the
Congress, which decision shall prevail; and privilege of the writ of habeas corpus or the extension thereof, and must promulgate its
decision thereon within thirty days from its filing. (Emphasis supplied)
6. Whether this Court’s determination of the sufficiency of the factual basis of the
declaration of martial law or suspension of the writ, which in the meantime has It is clear that the Constitution explicitly clothes "any citizen" with the legal standing to
been lifted and restored, respectively, would be essential to the resolution of issues challenge the constitutionality of the declaration of martial law or suspension of the writ. The
concerning the validity of related acts that the government committed during the Constitution does not make any distinction as to who can bring such an action. As discussed
time martial law was in force. in the deliberations of the Constitutional Commission, the "citizen" who can challenge the
declaration of martial law or suspension of the writ need not even be a taxpayer.17 This was
deliberately designed to arrest, without further delay, the grave effects of an illegal
In its Comment Re: Resolution dated 15 December 2009, the OSG raised the issue of whether
declaration of martial law or suspension of the writ, and to provide immediate relief to those
petitioners possess legal standing to challenge the constitutionality of Proclamation No.
aggrieved by the same. Accordingly, petitioners, being Filipino citizens, possess legal standing
1959.
253

to file the present petitions assailing the sufficiency of the factual basis of Proclamation No. In David v. Arroyo,22 this Court held that the "moot and academic" principle is not a magical
1959. formula that automatically dissuades courts in resolving a case. Courts are not prevented
from deciding cases, otherwise moot and academic, if (1) there is a grave violation of the
Moreover, given the transcendental importance of the issues raised in the present petitions, Constitution;23 (2) the situation is of exceptional character and of paramount public
the Court may relax the standing requirement and allow a suit to prosper even where there interest;24 (3) the constitutional issue raised requires formulation of controlling principles to
is no direct injury to the party claiming the right of judicial review.18 The Court has held: guide the bench, the bar, and the public;25 and (4) the case is capable of repetition yet
evading review.26
Notwithstanding, in view of the paramount importance and the constitutional significance of
the issues raised in the petitions, this Court, in the exercise of its sound discretion, brushes In Province of North Cotabato v. Government of the Republic of the Philippines Peace Panel
aside the procedural barrier and takes cognizance of the petitions, as we have done in the on Ancestral Domain (GRP),27 the Court ruled that once a suit is filed, the Court cannot
early Emergency Powers Cases, where we had occasion to rule: automatically be deprived of its jurisdiction over a case by the mere expedient of the doer
voluntarily ceasing to perform the challenged conduct. Otherwise, the doer would be
dictating when this Court should relinquish its jurisdiction over a case. Further, a case is not
‘x x x ordinary citizens and taxpayers were allowed to question the constitutionality of
mooted when the plaintiff seeks damages or prays for injunctive relief against the possible
several executive orders issued by President Quirino although they [involved] only an indirect
recurrence of the violation.28
and general interest shared in common with the public. The Court dismissed the objection
that they were not proper parties and ruled that ‘transcendental importance to the public of
these cases demands that they be settled promptly and definitely, brushing aside, if we Contrary to the majority opinion, the present petitions fall squarely under these exceptions,
must, technicalities of procedure.’ We have since then applied the exception in many other justifying this Court’s exercise of its review power.
cases.19 (Emphasis supplied)
First, whether Proclamation No. 1959 complied with the requirements under
I. Section 18, Article VII of the Constitution is without doubt an extremely serious
constitutional question. In order to forestall any form of abuse in the exercise of
the President’s extraordinary emergency powers, as what happened during the
Whether the issuance of Proclamation No. 1963,
Martial Law regime under former President Ferdinand Marcos (President Marcos),
lifting martial law and restoring the writ in the province of Maguindanao,
the 1987 Constitution has carefully put in place specific safeguards, which the
rendered the issues raised in the petitions moot and academic.
President must strictly observe. Any declaration of martial law or suspension of the
writ falling short of the constitutional requirements must be stricken down as a
The majority dismisses the petitions on mootness, agreeing with respondents' contention matter of constitutional duty by this Court.
that the issuance of Proclamation No. 1963, lifting martial law and restoring the writ in the
province of Maguindanao, rendered the issues raised in the present petitions moot and
Second, whether the President exercised her Commander-in-Chief powers in
academic. Respondents maintain that the petitions have ceased to present an "actual case or
accordance with the Constitution indisputably presents a transcendental issue fully
controversy" with the lifting of martial law and the restoration of the writ, the sufficiency of
imbued with public interest. I agree with amicus curiae Father Joaquin Bernas’
the factual basis of which is the subject of these petitions. Proclamation No. 1963 is allegedly
opinion: "The practice of martial rule can have a profoundly disturbing effect on
a "supervening event" that rendered of no practical use or value the consolidated petitions.
the life, liberty and fortunes of people. Likewise, the actions taken by the police
and military during the period when martial law is in effect can have serious
As a rule, courts may exercise their review power only when there is an actual case or consequences on fundamental rights."29
controversy, which involves a conflict of legal claims susceptible of judicial resolution. Such a
case must be "definite and concrete, touching the legal relations of parties having conflicting
Third, the issue on the constitutionality of Proclamation No. 1959 unquestionably
legal interests;" a real, as opposed to an imagined, controversy calling for a specific relief.20
requires formulation of controlling principles to guide the Executive, Legislature,
and the public.
Corollarily, courts generally decline jurisdiction over a moot and academic case or outrightly
dismiss it on the ground of mootness. A moot and academic case is one that ceases to
The President’s issuance of Proclamation No. 1959 generated strong reactions from various
present a justiciable controversy by virtue of supervening events, so that assuming
sectors of society. This, of course, is an expected response from a nation whose painful
jurisdiction over the same, and eventually deciding it, would be of no practical use or value.21
memory of the dark past remains fresh. The nation remembers that martial law was the
vehicle of President Marcos to seize unlimited State power, which resulted in gross and
254

wanton violations of fundamental human rights of the people. That era saw the collapse of II.
the rule of law and what reigned supreme was a one man-rule for the dictator’s own
personal benefit. Whether the term "rebellion" in Section 18, Article VII of the 1987 Constitution has the same
meaning as the term "rebellion" that is
The present controversy, being the first case under the 1987 Constitution involving the defined in Article 134 of the Revised Penal Code.
President’s exercise of the power to declare martial law and suspend the writ, provides this
Court with a rare opportunity,30 which it must forthwith seize, to formulate controlling Article 134 of the Revised Penal Code, as amended by Republic Act No. 6968,32 defines the
principles for the guidance of all sectors concerned, most specially the Executive which is in crime of rebellion, thus:
charge of enforcing the emergency measures. Dismissing the petitions on the ground of
mootness will most certainly deprive the entire nation of instructive and valuable principles
Art. 134. Rebellion or insurrection; How committed. — The crime of rebellion or insurrection
on this extremely crucial national issue.
is committed by rising publicly and taking arms against the Government for the purpose of
removing from the allegiance to said Government or its laws, the territory of the Philippine
Fourth, the present case is capable of repetition yet evading review. I agree with Father Islands or any part thereof, of any body of land, naval or other armed forces, depriving the
Bernas’ view: "[H]istory clearly attests that the events that can lead to martial law, as well as Chief Executive or the Legislature, wholly or partially, of any of their powers or prerogatives.
the imposition of martial law itself, and the suspension of the privilege together with actions
taken by military and police during a period of martial law are capable of repetition and are
The Constitution, however, does not provide any definition of the term "rebellion." Portions
too important to allow to escape review through the simple expedient of the President lifting
of the first paragraph of Section 18, Article VII of the Constitution, where the term "rebellion"
a challenged proclamation."31
appears, read:

Fifth, the respondent’s or doer’s voluntary cessation of the questioned act does not by itself
Section 18. The President shall be the Commander-in-Chief of all armed forces of the
deprive the Court of its jurisdiction once the suit is filed. In this case, President Arroyo, after
Philippines and whenever it becomes necessary, he may call out such armed forces to
eight days from the issuance of Proclamation No. 1959, issued Proclamation No. 1963
prevent or suppress lawless violence, invasion or rebellion. In case of invasion or rebellion,
revoking Proclamation No. 1959. President Arroyo’s lifting of martial law and restoration of
when the public safety requires it, he may, for a period not exceeding sixty days, suspend the
the writ translate to a voluntary cessation of the very acts complained of in the present
privilege of the writ of habeas corpus or place the Philippines or any part thereof under
petitions. However, the present petitions were filed with this Court while Proclamation No.
martial law.
1959 was still in effect and before Proclamation No. 1963 was issued, thus foreclosing any
legal strategy to divest this Court of its jurisdiction by the mere cessation or withdrawal of
the challenged act. Respondents submit that the term "rebellion" must, "for constitutional law purposes, be
applied in such manner as to be amply responsive to the call of the times." Respondents
point out that the deliberations of the 1986 Constitutional Commission reveal that the
Moreover, the fact that every declaration of martial law or suspension of the writ will involve
concept of the term "rebellion" depends much on its magnitude and scope, as determined by
its own set of circumstances peculiar to the necessity of time, events or participants should
the President based on prevailing circumstances.33
not preclude this Court from reviewing the President’s use of such emergency powers.
Whatever are the circumstances surrounding each declaration of martial law or suspension
of the writ, the declaration or suspension will always be governed by the same safeguards I disagree. The term "rebellion" in Section 18, Article VII of the 1987 Constitution must be
and limitations prescribed in the same provisions of the Constitution. Failing to determine understood as having the same meaning as the crime of "rebellion" that is defined in Article
the constitutionality of Proclamation No. 1959 by dismissing the cases on the ground of 134 of the Revised Penal Code, as amended.
mootness sets a very dangerous precedent to the leaders of this country that they could
easily impose martial law or suspend the writ without any factual or legal basis at all, and First, this is the clear import of the last two paragraphs of Section 18, Article VII of the
before this Court could review such declaration, they would simply lift the same and escape Constitution, which explicitly state:
possible judicial rebuke.
The suspension of the privilege of the writ of habeas corpus shall apply only to persons
judicially charged for rebellion or offenses inherent in, or directly connected with, invasion.
255

During the suspension of the privilege of the writ of habeas corpus, any person thus arrested In exercising the Commander-in-Chief powers under the Constitution, every President must
or detained shall be judicially charged within three days, otherwise he shall be released. insure the existence of the elements of the crime of rebellion, which are: (1) there is a (a)
public uprising and (b) taking arms against the Government; and (2) the purpose of the
For a person to be judicially charged for rebellion, there must necessarily be a statute uprising or movement is either (a) to remove from the allegiance to the Government or its
defining rebellion. There is no statute defining rebellion other than the Revised Penal Code. laws: (1) the territory of the Philippines or any part thereof; or (2) any body of land, naval, or
Hence, "one can be ‘judicially charged’ with rebellion only if one is suspected of having other armed forces; or (b) to deprive the Chief Executive or Congress, wholly or partially, of
committed acts defined as rebellion in Article 134 of the Revised Penal Code."34 any of their powers and prerogatives.38

Second, the Revised Penal Code definition of rebellion is the only legal definition of rebellion To repeat, the term "rebellion" in Section 18, Article VII of the Constitution must be
known and understood by the Filipino people when they ratified the 1987 Constitution. understood to have the same meaning as the crime of rebellion defined in Article 134 of the
Indisputably, the Filipino people recognize and are familiar with only one meaning of Revised Penal Code. Ascribing another meaning to the term "rebellion" for constitutional law
rebellion, that is, the definition provided in Article 134 of the Revised Penal Code. To depart purposes, more specifically in imposing martial law and suspending the writ, different from
from such meaning is to betray the Filipino people’s understanding of the term "rebellion" the definition in Article 134 of the Revised Penal Code, overstretches its definition without
when they ratified the Constitution. There can be no question that "the Constitution does any standards, invites unnecessary confusion, and undeniably defeats the intention of the
not derive its force from the convention which framed it, but from the people who ratified Constitution to restrain the extraordinary Commander-in-Chief powers of the President.
it."35
Since the term "rebellion" in Section 18, Article VII of the Constitution pertains to the crime
Third, one of the Whereas clauses of Proclamation No. 1959 expressly cites the Revised Penal of rebellion as defined in Article 134 of the Revised Penal Code, the next question turns on
Code definition of rebellion, belying the government’s claim that the Revised Penal Code the kind of proof required for a valid declaration of martial law and suspension of the writ.
definition of rebellion merely guided the President in issuing Proclamation No. 1959.
While the Constitution expressly provides strict safeguards against any potential abuse of the
In SANLAKAS v. Executive Secretary,36 where the Court regarded President Arroyo’s President’s emergency powers, the Constitution does not compel the President to produce
declaration of a state of rebellion in Proclamation No. 427 a superfluity,37 the term such amount of proof as to unduly burden and effectively incapacitate her from exercising
"rebellion" in said proclamation referred to the crime of rebellion as defined in Article 134 of such powers.
the Revised Penal Code. Proclamation No. 427 pertinently reads:
Definitely, the President need not gather proof beyond reasonable doubt, which is the
DECLARING A STATE OF REBELLION standard of proof required for convicting an accused charged with a criminal offense. Section
2, Rule 133 of the Rules of Court defines proof beyond reasonable doubt as follows:
WHEREAS, certain elements of the Armed Forces of the Philippines, armed with high-
powered firearms and explosives, acting upon the instigation and command and direction of Proof beyond reasonable doubt does not mean such a degree of proof as, excluding
known and unknown leaders, have seized a building in Makati City, put bombs in the area, possibility of error, produces absolute certainty. Moral certainty only is required, or that
publicly declared withdrawal of support for, and took arms against the duly constituted degree of proof which produces conviction in an unprejudiced mind.
Government, and continue to rise publicly and show open hostility, for the purpose of
removing allegiance to the Government certain bodies of the Armed Forces of the Proof beyond reasonable doubt is the highest quantum of evidence, and to require the
Philippines and the Philippine National Police, and depriving the President of the Republic President to establish the existence of rebellion or invasion with such amount of proof before
of the Philippines, wholly or partially, of her powers and prerogatives which constitute the declaring martial law or suspending the writ amounts to an excessive restriction on "the
crime of rebellion punishable under Article 134 of the Revised Penal Code, as amended; x x President’s power to act as to practically tie her hands and disable her from effectively
x (Emphasis supplied) protecting the nation against threats to public safety."39

In issuing Proclamation No. 427, President Arroyo relied on the Revised Penal Code definition Neither clear and convincing evidence, which is employed in either criminal or civil cases, is
of rebellion in declaring a state of rebellion. In other words, President Arroyo understood indispensable for a lawful declaration of martial law or suspension of the writ. This amount of
that, for purposes of declaring a state of rebellion, the term "rebellion" found in the proof likewise unduly restrains the President in exercising her emergency powers, as it
Constitution refers to the crime of rebellion defined in Article 134 of the Revised Penal Code. requires proof greater than preponderance of evidence although not beyond reasonable
doubt.40
256

Not even preponderance of evidence,41 which is the degree of proof necessary in civil cases, of probable cause that a person can be "judicially charged" under the last two paragraphs of
is demanded for a lawful declaration of martial law. Section 18, Article VII, to wit:

By preponderance of evidence is meant that the evidence as a whole adduced by one side is The suspension of the privilege of the writ of habeas corpus shall apply only to
superior to that of the other. It refers to the weight, credit and value of the aggregate persons judicially charged for rebellion or offenses inherent in, or directly connected with,
evidence on either side and is usually considered to be synonymous with the term "greater invasion.
weight of evidence" or "greater weight of the credible evidence". It is evidence which is more
convincing to the court as worthy of belief than that which is offered in opposition thereto.42 During the suspension of the privilege of the writ of habeas corpus, any person thus arrested
or detained shall be judicially charged within three days, otherwise he shall be released.
Weighing the superiority of the evidence on hand, from at least two opposing sides, before (Emphasis supplied)
she can act and impose martial law or suspend the writ unreasonably curtails the President’s
emergency powers. III.

Similarly, substantial evidence constitutes an unnecessary restriction on the President’s use Whether the declaration of martial law or the suspension
of her emergency powers. Substantial evidence is the amount of proof required in of the writ authorizes warrantless arrests, searches and seizures.
administrative or quasi-judicial cases, or that amount of relevant evidence which a
reasonable mind might accept as adequate to justify a conclusion.43
Section 18, Article VII of the Constitution partially states:

I am of the view that probable cause of the existence of either invasion or rebellion suffices
A state of martial law does not suspend the operation of the Constitution, nor supplant the
and satisfies the standard of proof for a valid declaration of martial law and suspension of
functioning of the civil courts or legislative assemblies, nor authorize the conferment of
the writ.
jurisdiction on military courts and agencies over civilians where civil courts are able to
function, nor automatically suspend the privilege of the writ of habeas corpus.
Probable cause is the same amount of proof required for the filing of a criminal information
by the prosecutor and for the issuance of an arrest warrant by a judge. Probable cause has
The suspension of the privilege of the writ of habeas corpus shall apply only to persons
been defined as a "set of facts and circumstances as would lead a reasonably discreet and
judicially charged for rebellion or offenses inherent in, or directly connected with, invasion.
prudent man to believe that the offense charged in the Information or any offense included
therein has been committed by the person sought to be arrested."44
The 1935 and 1973 Constitutions did not contain a similar provision. Obviously, this new
provision in the 1987 Constitution was envisioned by the framers of the Constitution to serve
In determining probable cause, the average man weighs the facts and circumstances
as an essential safeguard against potential abuses in the exercise of the President’s
without resorting to the calibrations of the rules of evidence of which he has no technical
emergency powers.
knowledge. He relies on common sense. A finding of probable cause needs only to rest on
evidence showing that, more likely than not, a crime has been committed and that it was
committed by the accused. Probable cause demands more than suspicion; it requires less The Constitution now expressly declares, "A state of martial law does not suspend the
than evidence that would justify conviction.45 (Emphasis supplied) operation of the Constitution." Neither does a state of martial law supplant the functioning
of the civil courts or legislative assemblies. Nor does it authorize the conferment of
jurisdiction on military courts and agencies over civilians where civil courts are able to
Probable cause, basically premised on common sense, is the most reasonable, most practical,
function, or automatically suspend the writ. There is therefore no dispute that the
and most expedient standard by which the President can fully ascertain the existence or non-
constitutional guarantees under the Bill of Rights remain fully operative and continue to
existence of rebellion, necessary for a declaration of martial law or suspension of the writ.
accord the people its mantle of protection during a state of martial law. In case the writ is
Therefore, lacking probable cause of the existence of rebellion, a declaration of martial law
also suspended, the suspension applies only to those judicially charged for rebellion or
or suspension of the writ is without any basis and thus, unconstitutional.
offenses directly connected with invasion.

The requirement of probable cause for the declaration of martial law or suspension of the
writ is consistent with Section 18, Article VII of the Constitution. It is only upon the existence
257

Considering the non-suspension of the operation of the Constitution during a state of martial of Chief Justice Claudio Teehankee — to the law of force rather than the force of law, it is
law, a declaration of martial law does not authorize warrantless arrests, searches and necessary to remind ourselves that certain basic rights and liberties are immutable and
seizures, in derogation of Section 2, Article III of the Constitution, which provides: cannot be sacrificed to the transient needs or imperious demands of the ruling power. The
rule of law must prevail, or else liberty will perish. x x x
Section 2. The right of the people to be secure in their persons, houses, papers, and effects
against unreasonable searches and seizures of whatever nature and for any purpose shall be It may be that the respondents, as members of the Armed Forces of the Philippines, were
inviolable, and no search warrant or warrant of arrest shall issue except upon probable cause merely responding to their duty, as they claim, "to prevent or suppress lawless violence,
to be determined personally by the judge after examination under oath or affirmation of the insurrection, rebellion and subversion" in accordance with Proclamation No. 2054 of
complainant and the witnesses he may produce, and particularly describing the place to be President Marcos, despite the lifting of martial law on January 27, 1981, and in pursuance of
searched and the persons or things to be seized. such objective, to launch pre-emptive strikes against alleged communist terrorist
underground houses. But this cannot be construed as a blanket license or a roving
Warrantless arrests, search and seizure are valid only in instances where such acts are commission untramelled by any constitutional restraint, to disregard or transgress upon
justified, i.e., those enumerated in Section 5, Rule 113 of the Rules of Court.46 the rights and liberties of the individual citizen enshrined in and protected by the
Constitution. The Constitution remains the supreme law of the land to which all officials,
high or low, civilian or military, owe obedience and allegiance at all times.
In Pequet v. Tangonan,47 decided during the Martial Law regime under former President
Marcos, the Court stressed that military personnel, in effecting arrests, must strictly observe
the applicable Rules of Court and settled jurisprudence, thus: This is not to say that military authorities are restrained from pursuing their assigned task or
carrying out their mission with vigor. We have no quarrel with their duty to protect the
Republic from its enemies, whether of the left or of the right, or from within or without,
Martial law has precisely been provided in both the 1935 Charter and the present
seeking to destroy or subvert our democratic institutions and imperil their very
Constitution to assure that the State is not powerless to cope with invasion, insurrection or
existence. What we are merely trying to say is that in carrying out this task and mission,
rebellion or any imminent danger of its occurrence. When resort to it is therefore justified, it
constitutional and legal safeguards must be observed, otherwise, the very fabric of our
is precisely in accordance with and not in defiance of the fundamental law. There is all the
faith will start to unravel. x x x
more reason then for the rule of law to be followed. For as was so eloquently proclaimed
in Ex parte Milligan: "The Constitution is a "law for rulers and for people equally in war and in
peace and covers with the shield of its protection all classes of men at all times and under all We do not agree. We find merit in petitioners’ contention that the suspension of the
circumstances." It is true, of course, as admitted by Willoughby, who would limit the scope of privilege of the writ of habeas corpus does not destroy petitioners’ right and cause of
martial law power, that the military personnel are called upon to assist in the maintenance of action for damages for illegal arrest and detention and other violations of their
peace and order and the enforcement of legal norms. They can therefore act like ordinary constitutional rights. The suspension does not render valid an otherwise illegal arrest or
peace officers. In effecting arrests, however, they are not free to ignore, but are precisely detention. What is suspended is merely the right of the individual to seek release from
bound by, the applicable Rules of Court and doctrinal pronouncements. (Emphasis detention through the writ of habeas corpus as a speedy means of obtaining his
supplied) liberty.49 (Emphasis supplied)

In Aberca v. Ver,48 the Court emphasized that the suspension of the writ does not give IV.
imprimatur to warrantless arrests in violation of the Constitution. In that case, which
involved the issue of whether the suspension of the writ bars a civil action for damages for Whether the declaration of martial law or suspension of
illegal searches and for other human rights violations committed by the military, the Court the writ is a joint and sequential function of the
held: President and Congress such that, without Congressional action
on the proclamation or suspension either affirming or revoking it,
At the heart of petitioners’ complaint is Article 32 of the Civil Code which provides: the President having in the meantime lifted the same,
this Court has nothing to review.
It is obvious that the purpose of the above codal provision is to provide a sanction to the
deeply cherished rights and freedoms enshrined in the Constitution. Its message is clear; no Section 18, Article VII of the 1987 Constitution provides:
man may seek to violate those sacred rights with impunity. In times of great upheaval or of
social and political stress, when the temptation is strongest to yield — borrowing the words
258

Section 18. The President shall be the Commander-in-Chief of all armed forces of the suspension, the Court may review the declaration or suspension in an appropriate
Philippines and whenever it becomes necessary, he may call out such armed forces to proceeding filed by any citizen. Otherwise stated, Congress’ inaction on the declaration or
prevent or suppress lawless violence, invasion or rebellion. In case of invasion or rebellion, suspension is not determinative of the Court’s exercise of its review power under Section 18,
when the public safety requires it, he may, for a period not exceeding sixty days, suspend the Article VII of the Constitution.
privilege of the writ of habeas corpus or place the Philippines or any part thereof under
martial law. Within forty-eight hours from the proclamation of martial law or the suspension To hold that the power of this Court to review the President’s declaration of martial law or
of the privilege of the writ of habeas corpus, the President shall submit a report in person or suspension of the writ is sequential, or joint, with the review power of Congress is to make it
in writing to the Congress. The Congress, voting jointly, by a vote of at least a majority of all impossible for this Court to decide a case challenging the declaration or suspension "within
its Members in regular or special session, may revoke such proclamation or suspension, thirty days from its filing," as mandated by the Constitution. Congress has no deadline when
which revocation shall not be set aside by the President. Upon the initiative of the President, to revoke the President’s declaration or suspension. Congress may not even do anything with
the Congress may, in the same manner, extend such proclamation or suspension for a period the President’s declaration or suspension and merely allow it to lapse after 60 days. On the
to be determined by the Congress, if the invasion or rebellion shall persist and public safety other hand, the Constitution mandates that this Court "must promulgate its decision
requires it. thereon within thirty days from [the] filing" of the case. Clearly, the Court’s review power is
neither sequential nor joint with the review power of Congress.
The Congress, if not in session, shall, within twenty-four hours following such proclamation
or suspension, convene in accordance with its rules without need of a call. Moreover, the President’s lifting of the declaration or suspension before this Court could
decide the case within the 30-day period does not operate to divest this Court of its
The Supreme Court may review, in an appropriate proceeding filed by any citizen, the jurisdiction over the case. A party cannot simply oust the Court’s jurisdiction, already
sufficiency of the factual basis of the proclamation of martial law or the suspension of the acquired, by a party’s own unilateral act. The President’s lifting of the declaration or
privilege of the writ of habeas corpus or the extension thereof, and must promulgate its suspension merely means that this Court does not have to decide the case within the 30-day
decision thereon within thirty days from its filing. period, as the urgency of deciding has ceased. Certainly, the Court is not divested of its
jurisdiction simply because the urgency of deciding a case has ceased.
The Constitution vests exclusively in the President, as Commander-in-Chief, the emergency
powers to declare martial law or suspend the writ in cases of rebellion or invasion, when the V.
public safety requires it. The imposition of martial law or suspension of the writ takes effect
the moment it is declared by the President. No other act is needed for the perfection of the If the constitutional power of this Court to review the factual basis
declaration of martial law or the suspension of the writ. As amicus curiae retired Justice of the declaration of martial law or suspension of the writ can be exercised simultaneously
Mendoza states: with the constitutional power of Congress to revoke the declaration or suspension, and the
decision of this Court conflicts with the decision of Congress, which decision shall prevail.
A declaration of martial law by the President alone is complete by itself and does not require
for its validity the approval or concurrence of Congress. It is a power placed solely in the The President has the sole and exclusive power to declare martial law or suspend the writ.
keeping of the President to enable him to secure the people from harm and restore the This power of the President is subject to review separately by Congress and the Supreme
public order so that they can enjoy their freedoms. Because it is liable to abuse, it is made Court. Justice Mendoza stresses, "Thus, Congress and this Court have separate spheres of
subject to check by Congress and/or the [Supreme Court]. competence. They do not act ‘jointly and sequentially’ but independently of each
other."50 Father Bernas points out, "Since the powers of Congress and the Court are
The power of Congress is to revoke – not to confirm or ratify, much less to approve, – the independent of each other, there is nothing to prevent Congress and the Court from
President’s action declaring martial law or suspending the privilege of the writ of habeas simultaneously exercising their separate powers."51
corpus. It is a veto power, just as the power of the judiciary to review the President’s action is
a veto power on the Executive’s action. In the exercise by the Court and Congress of their separate "review powers" under Section
18, Article VII of the Constitution, three possible scenarios may arise.
It is clear, therefore, that the President’s power to declare martial law or suspend the writ is
independent, separate, and distinct from any constitutionally mandated act to be performed First, the President’s martial law declaration or suspension of the writ is questioned
by either the Legislature or the Judiciary. It is neither joint nor sequential with Congress’ in the Supreme Court without Congress acting on the same. Such a situation
power to revoke the declaration or suspension or to extend it upon the initiative of the generates no conflict between the Supreme Court and Congress. There is no
President. Accordingly, even if Congress has not acted upon the President’s declaration or
259

question that the Supreme Court can annul such declaration or suspension if it Various acts carried out by government forces during martial law or suspension of the writ in
lacks factual basis. Congress, whose only power under Section 18, Article VII of the the guise of protecting public safety may in reality amount to serious abuses of power and
Constitution is to revoke the declaration or suspension on any ground, is left with authority. Whatever the Court’s decision will be on the sufficiency of the factual basis of the
nothing to revoke if the Court has already annulled the declaration or suspension. President’s declaration or suspension does not preclude those aggrieved by such illegal acts
from pursuing any course of legal action available to them. Therefore, the determination by
Second, Congress decides first to revoke the martial law declaration or suspension this Court of the sufficiency of the factual basis of the declaration or suspension is not
of the writ. Since the Constitution does not limit the grounds for congressional essential to the resolution of issues concerning the validity of related acts that government
revocation, Congress can revoke the declaration or suspension for policy reasons, forces may have committed during the emergency.
or plainly for being insignificant, as for instance it involves only
one barangayrebelling, or if it finds no actual rebellion. In this case, the Supreme VII.
Court is left with nothing to act on as the revocation by Congress takes effect
immediately. The Supreme Court must respect the revocation by Congress even if Whether Proclamation No. 1959 has sufficient factual basis.
the Court believes a rebellion exists because Congress has the unlimited power to
revoke the declaration or suspension.
The full text of Section 18, Article VII of the 1987 Constitution reads:

Third, the Supreme Court decides first and rules that there is factual basis for the
Section 18. The President shall be the Commander-in-Chief of all armed forces of the
declaration of martial law or suspension of the writ. In such a situation, Congress
Philippines and whenever it becomes necessary, he may call out such armed forces to
can still revoke the declaration or suspension as its power under the Constitution is
prevent or suppress lawless violence, invasion or rebellion. In case of invasion or rebellion,
broader insofar as the declaration or suspension is concerned. "Congress cannot be
when the public safety requires it, he may, for a period not exceeding sixty days, suspend the
prevented by the Court from revoking the President’s decision because it is not for
privilege of the writ of habeas corpus or place the Philippines or any part thereof under
the Court to determine what to do with an existing factual situation. x x x Congress
martial law. Within forty-eight hours from the proclamation of martial law or the suspension
has been given unlimited power to revoke the President’s decision."52 In short,
of the privilege of the writ of habeas corpus, the President shall submit a report in person or
even if there is an actual rebellion, whether affirmed or not by the Supreme Court,
in writing to the Congress. The Congress, voting jointly, by a vote of at least a majority of all
Congress has the power to revoke the President’s declaration or suspension.
its Members in regular or special session, may revoke such proclamation or suspension,
which revocation shall not be set aside by the President. Upon the initiative of the President,
In the present controversy, Congress failed to act on Proclamation No. 1959 when it the Congress may, in the same manner, extend such proclamation or suspension for a period
commenced its Joint Session on 9 December 2009 until the lifting of the martial law to be determined by the Congress, if the invasion or rebellion shall persist and public safety
declaration and restoration of the writ on 12 December 2009. Congress’ non-revocation of requires it.
Proclamation No. 1959 categorizes the present case under the first scenario. In such a
situation, where no conflict ensues, Congress’ inaction on Proclamation No. 1959 does not
The Congress, if not in session, shall, within twenty-four hours following such proclamation
preclude this Court from ruling on the sufficiency of the factual basis of the declaration of
or suspension, convene in accordance with its rules without need of a call.
martial law and suspension of the writ.

The Supreme Court may review, in an appropriate proceeding filed by any citizen, the
VI.
sufficiency of the factual basis of the proclamation of martial law or the suspension of the
privilege of the writ of habeas corpus or the extension thereof, and must promulgate its
Whether this Court’s determination of the sufficiency of the factual basis decision thereon within thirty days from its filing.
of the declaration of martial law and suspension of the writ,
which in the meantime have been lifted, would be essential
A state of martial law does not suspend the operation of the Constitution, nor supplant the
to the resolution of issues concerning the validity of related acts
functioning of the civil courts or legislative assemblies, nor authorize the conferment of
that the government committed during the time
jurisdiction on military courts and agencies over civilians where civil courts are able to
that martial law and the suspension of the writ were in force.
function, nor automatically suspend the privilege of the writ of habeas corpus.

Indisputably, unlawful acts may be committed during martial law or suspension of the writ,
The suspension of the privilege of the writ of habeas corpus shall apply only to persons
not only by the rebels, but also by government forces who are duty bound to enforce the
judicially charged for rebellion or offenses inherent in, or directly connected with, invasion.
declaration or suspension and immediately put an end to the root cause of the emergency.
260

During the suspension of the privilege of the writ of habeas corpus, any person thus arrested The Commander-in-Chief provisions of the 1935 Constitution had enabled President
or detained shall be judicially charged within three days, otherwise he shall be released. Ferdinand Marcos to impose authoritarian rule on the Philippines from 1972 to 1986.
Supreme Court decisions during that period upholding the actions taken by Mr. Marcos made
The Commander-in-Chief provisions of the 1935 and 1973 Constitutions, on the other hand, authoritarian rule part of Philippine constitutional jurisprudence. The members of the
respectively state: Constitutional Commission, very much aware of these facts, went about reformulating the
Commander-in-Chief powers with a view to dismantling what had been constructed during
the authoritarian years. The new formula included revised grounds for the activation of
Section 10(2), Article VII of the 1935 Constitution
emergency powers, the manner of activating them, the scope of the powers, and review of
presidential action.54
2. The President shall be commander-in-chief of all armed forces of the Philippines, and,
whenever it becomes necessary, he may call out such armed forces to prevent or suppress
Consistent with the framers’ intent to reformulate the Commander-in-Chief powers of the
lawless violence, invasion, insurrection, or rebellion. In case of invasion, insurrection, or
President, the 1987 Constitution requires the concurrence of two conditions in declaring
rebellion or imminent danger thereof, when the public safety requires it, he may suspend the
martial law or suspending the writ, namely, (1) an actual invasion or rebellion, and (2) public
privilege of the writ of habeas corpus, or place the Philippines or any part thereof under
safety requires the exercise of such power.55 The Constitution no longer allows imminent
Martial Law.
danger of rebellion or invasion as a ground for the declaration or suspension, which the 1935
and 1973 Constitutions expressly permitted.
Section 12, Article IX of the 1973 Constitution
In the present case, President Arroyo grounded the declaration of martial law and
SEC. 12. The Prime Minister shall be commander-in-chief of all armed forces of the suspension of the writ on the existence of rebellion in Maguindanao. In her Report submitted
Philippines, and, whenever it becomes necessary, he may call out such armed forces to to Congress, President Arroyo cited the following instances as constitutive of rebellion:
prevent or suppress lawless violence, invasion, insurrection, or rebellion or imminent danger
thereof, when the public safety requires it, he may suspend the privilege of the writ
1. Local government offices in the province of Maguindanao were closed and
of habeas corpus, or place the Philippines or any part thereof under Martial Law.
ranking local government officials refused to discharge their functions, which
hindered the investigation and prosecution team from performing their tasks;
Notably, the 1935 and 1973 Constitutions only specify the instances when martial law may
be declared or when the writ may be suspended.
2. The Local Civil Registrar of Maguindanao refused to accept the registration of the
death certificates of the victims purportedly upon the orders of Andal Ampatuan
The 1987 Constitution, on the other hand, not only explicitly includes the specific grounds for Sr.;
the activation of such emergency powers, but also imposes express limitations on the
exercise of such powers. Upon the President’s declaration of martial law or suspension of the
3. The local judicial system has been crippled by the absence or non-appearance of
writ, the following safeguards are automatically set into motion: (1) the duration of martial
judges of local courts, thereby depriving the government of legal remedies in their
law or suspension of the writ is limited to a period not exceeding sixty days; (2) the President
prosecutorial responsibilities (i.e. issuance of warrants of searches, seizure and
is mandated to submit a report to Congress within forty-eight hours from the declaration or
arrest). While the Supreme Court has designated an Acting Presiding Judge from
suspension; and (3) the declaration or suspension is subject to review by Congress, which
another province, the normal judicial proceedings could not be carried out in view
may revoke such declaration or suspension. If Congress is not in session, it shall convene
of threats to their lives or safety, prompting government to seek a change of venue
within 24 hours without need for call.53 In addition, the sufficiency of the factual basis of the
of the criminal cases after informations have been filed.
declaration, suspension, or their extension is subject to review by the Supreme Court in an
appropriate proceeding.
Indeed, the nature, quantity and quality of their weaponry, the movement of heavily
armed rebels in strategic positions, the closure of the Maguindanao Provincial Capitol,
The mechanism and limitations laid down in Section 18, Article VII of the Constitution in
Ampatuan Municipal Hall, Datu Unsay Municipal Hall, and fourteen other municipal halls,
declaring martial law or suspending the writ were introduced precisely to preclude a
and the use of armored vehicles, tanks and patrol cars with unauthorized "PNP/Police"
repetition of the kind of martial law imposed by President Marcos, which ushered in a
markings, all together confirm the existence of armed public uprising for the political
permanent authoritarian regime. As Father Bernas wrote in his book:
purpose of:
261

(1) removing allegiance from the national government of the Province of Also, during the Joint Session, then Senator (now President) Benigno S. Aquino III pointed out
Maguindanao; and, the public statements made by former Department of Interior and Local Government
Secretary Ronaldo V. Puno, then Armed Forces of the Philippines spokesperson Lt. Col.
(2) depriving the Chief Executive of her powers and prerogatives to enforce the Romeo Brawner, and former Defense Secretary Norberto Gonzales admitting there was no
laws of the land and to maintain public order and safety. need for martial law:

While the government is at present conducting legitimate operations to address the on- THE SENATE PRESIDENT. With the indulgence of the Chamber and the Speaker, may we
going rebellion, public safety still requires the continued implementation of martial law request now to allow the distinguished Gentleman from Tarlac, Senator Benigno "Noynoy"
and the suspension of the privilege of the writ of habeas corpus in the Province of Aquino III the floor.
Maguindanao until the time that such rebellion is completely quelled.56 (Emphasis supplied)
SEN. AQUINO. Thank you, Mr. President. May I direct my first question to Secretary Puno.
The question now is whether there was probable cause, which is the required quantum of And this is to lay the proper predicate for our first question. The newspaper has been quoting
proof, to declare the existence of rebellion justifying the President’s declaration of martial Secretary Puno as not having recommended the imposition of martial law prior to its
law and suspension of the writ. imposition in Maguindanao. May we know if this was a correct attribution to the Honorable
Secretary.
The answer is in the negative.
MR. PUNO. Until, Your Honor, Mr. Speaker, Mr. Senate President, until the situation
developed where police officers went absent on leave and joined the rebel forces, and a
The contemporaneous public statements made by the President’s alter egos explaining the
significant segment of the civilian armed volunteers of the local governments constituted
grounds for the issuance of Proclamation No. 1959 negate rather than establish the existence
themselves into a rebel group, until that time I did not believe that it was necessary that
of an actual rebellion in Maguindanao.
martial law be declared. But upon receipt of a report from the Armed Forces of the
Philippines and the briefing conducted with the National Security Council, where it was made
During the interpellations in the Joint Session of Congress, convened pursuant to the clear that a separate rebel armed group had already been organized, we concurred, Your
provisions of Section 18, Article VII of the Constitution, then Executive Secretary Eduardo Honor, with the recommendation on martial law.
Ermita admitted the absence of an actual rebellion in Maguindanao, to wit:
SEN. AQUINO. For the record, Mr. Senate President and Mr. Speaker, the AFP, we
REP. LAGMAN. Mr. Speaker, Mr. President, a perusal of the text of Proclamation No. 1959 understand, through the spokesperson, Lt. Col. Romeo Brawner, declared on 13 November
would show the absence of a clear and categorical finding or determination that actual 2009 that there is no need for the declaration of martial law in Maguindanao or elsewhere in
rebellion is occurring in Maguindanao. Would that be an accurate observation of a reading of the country because the AFP and PNP are on top of the situation. He was quoted as saying,
the text of Proclamation No. 1959? and we quote: "We now have a level of normalcy in the Province of Maguindanao, primarily
because of the occupation by our government forces and our law enforcement agencies of
MR. ERMITA. Your Honor, you may be correct that there was no actual rebellion going the seats of government." Secretary Norberto Gonzales, who unfortunately is not present,
on. However, all the indicators that rebellion is, indeed, being committed and happening on declared on December 1, 2009 that the government’s effort to contain the tension in the
the ground is because of the presence of the armed groups that prevent authorities from province is holding ground. We also have now the admission by the honorable Secretary
being able to do its duty of even effecting the arrest of those who should be arrested in spite Puno that prior to the undated national security briefing, he was also of the opinion that
of the testimonies of witnesses. martial law was not necessary in Maguindanao. x x x58

REP. LAGMAN. Well, we are happy to note that there is an admission that there was no Even before the interpellations in Congress, then Executive Secretary Ermita publicly
actual rebellion in Maguindanao. But the presence of armed groups would be indicative of confirmed the inadequacies of Proclamation No. 1959:
lawless violence which is not synonymous to rebellion. As a matter of fact, the Maguindanao
situationer which was made by Police Director Andres Caro was premised on a statement We’ll have to get the report from the field from the AFP and PNP that the conditions that
that this was the worst election-related violence – an act of gross lawlessness but definitely prompted the President to issue the proclamation, have improved, and therefore, the threat
not related to rebellion. of further lawlessness and probability of rebellion is already down.59 (Emphasis supplied)
262

Significantly, at a press conference, then Secretary of Justice Agnes Devanadera declared, These shocking discoveries, however, do not amount to rebellion as defined in Article 134 of
"We noticed and observed there was a rebellion in the offing." In another press briefing, the Revised Penal Code. Based on the statements made by ranking government and military
Devanadera stated that "rebellion which does not necessarily involve a physical takeover by officials, and as clearly found by the RTC-Quezon City in Criminal Case No. Q-10-162667 and
armed elements as argued by some critics of the President’s order, was "looming affirmed by the Court of Appeals, there was no public uprising and taking arms against the
in Maguindanao."60 In short, the Department of Justice Secretary, who is the principal legal government for the purpose of removing from the allegiance to the government or its laws
officer of the Arroyo administration, publicly admitted that there was only a "looming" the territory of the Philippines or any part thereof, or depriving the Chief Executive or
rebellion, a "rebellion in the offing," in Maguindanao. Congress, wholly or partially, of any of their powers and prerogatives. The Ampatuans’
amassing of weaponry, including their collection of armored cars, tanks and patrol cars,
Likewise, in a press conference, "the AFP Chief of Staff claimed that armed groups, merely highlights this political clan’s unbelievably excessive power and influence under the
numbering between 40 to 400 men and spread out in the province, planned to prevent the Arroyo administration.
arrest of members of the Ampatuan family, the prime suspects in the Maguindanao
massacre. He stated, "Based on the reports we received, there were a lot of groupings of To repeat, only in case of actual invasion or rebellion, when public safety requires it, may the
armed groups in different places. We also received reports that they have plans to undertake President declare martial law or suspend the writ. In declaring martial law and suspending
hostile action if ever government officials, the Ampatuans particularly, were taken in the writ in Maguindanao in the absence of an actual rebellion, President Arroyo indisputably
custody. We felt this was very imminent threat, that’s why we recommended this violated the explicit provisions of Section 18, Article VII of the Constitution.
proclamation."61
Conclusion
Then Defense Secretary Norberto Gonzales was quoted as stating that the "recommendation
to declare martial law in Maguindanao is a sensitive matter that needs to be studied."62 In an Thirty-seven years after President Marcos’ Proclamation No. 1081, President Arroyo issued
interview, Gonzales said, "titingnan natin (we will see) how the situation develops Proclamation No. 1959 declaring martial law and suspending the privilege of the writ
there."63 He further stated, "As of now, I think whatever the government is doing so far is of habeas corpus in the province of Maguindanao, except in MILF identified areas. President
really effective. We will wait for the results of the work of Secretary Devanadera of Justice Marcos’ martial law, justified to counteract the Communist insurgency in the
and also Secretary Puno of DILG. So, so far maganda naman yun takbo ng ating operation country,67 turned out to be a vehicle to establish a one-man authoritarian rule in the country.
doon."64 Gonzales added, "Yung tungkol sa martial law, alam mo sensitive na bagay yan kaya Expectedly, President Arroyo’s Proclamation No. 1959 refreshed the nation’s bitter memories
pag-aaralan natin."65 of the tyranny during the Martial Law regime of President Marcos, and sparked the public’s
vigilance to prevent a possible recurrence of that horrible past.
The admissions and public statements made by members of the Cabinet, who are the
President’s alter egos, as well as the public assessments made by the highest ranking military In issuing Proclamation No. 1959, President Arroyo exercised the most awesome and
officials, clearly demonstrate that instead of being anchored on the existence of an actual powerful among her graduated Commander-in-Chief powers to suppress a supposed
rebellion, Proclamation No. 1959 was based on a mere threat, or at best an imminent threat rebellion in Maguindanao, following the massacre of 57 civilians in the worst election-related
of rebellion, or a rebellion "in the offing."66 This undeniably runs counter to the letter and violence in the country’s history. Since then, the government branded the Ampatuans, the
intent of the Constitution. A looming rebellion is analogous to imminent danger of rebellion, alleged masterminds of the massacre, as rebels orchestrating the overthrow of the Arroyo
which was deliberately eliminated by the framers of the 1987 Constitution as a ground for administration. However, the events before, during, and after the massacre negate the
the declaration of martial law precisely to avoid a repetition of the misguided and oppressive existence of an armed uprising aimed at bringing down the government, but rather point to a
martial law imposed by former President Marcos. surfeit of impunity and abuse of power of a political clan closely allied with the Arroyo
administration. In short, Proclamation No. 1959 was issued without an actual rebellion
There is absolutely nothing which shows that the Ampatuans and their armed followers, at justifying the same.
any point in time, intended to overthrow the government. On the contrary, the Ampatuans
were publicly known as very close political allies of President Arroyo. There is not a single Apparently, President Arroyo resorted to martial law and suspension of the writ, not to quell
instance where the Ampatuans denounced, expressly or impliedly, the government, or a purported rebellion because there was absolutely none, but to show her indignation over
attempted to remove allegiance to the government or its laws or to deprive the President or the gruesome massacre and her swift response in addressing the difficult situation involving
Congress of any of their powers. Based on the records, what the government clearly her close political allies. She was reported to be "under pressure to deliver, amid rising public
established, among others, were (1) the existence of the Ampatuans’ private army; and (2) outrage and international condemnation of the massacre."68 However, mounting pressure to
the Ampatuans’ vast collection of high powered firearms and ammunitions. bring the murderers to justice, without any invasion or rebellion in Maguindanao, does not
warrant the imposition of martial law or suspension of the writ. Rather, what the nation
263

expects, and what the victims and their families truly deserve, is the speedy and credible writ of habeas corpus in Maguindanao. Petitioners challenge the constitutionality of
investigation and prosecution, and eventually the conviction, of the merciless killers. Proclamation 1959.

In sum, Proclamation No. 1959 was anchored on a non-existent rebellion. Based on the ISSUE: Whether Proclamation 1959 is constitutional.
events before, during and after the Maguindanao massacre, there was obviously no rebellion
justifying the declaration of martial law and suspension of the writ. The discovery of the HELD:
Ampatuans’ private army and massive weaponry does not establish an armed public uprising
aimed at overthrowing the government. Neither do the closure of government offices and The Court deems any review of its constitutionality the equivalent of beating a dead horse.
the reluctance of the local government officials and employees to report for work indicate a Under the 1987 Constitution, the President and the Congress act in tandem in exercising the
rebellion. power to proclaim martial law or suspend the privilege of the writ of habeas corpus. They
exercise the power, not only sequentially, but in a sense jointly since, after the President has
The Constitution is clear. Only in case of actual invasion or rebellion, when public safety initiated the proclamation or the suspension, only the Congress can maintain the same based
requires it, can a state of martial law be declared or the privilege of the writ of habeas
on its own evaluation of the situation on the ground, a power that the President does not
corpus be suspended. Proclamation No. 1959 cannot be justified on the basis of a
threatened, imminent, or looming rebellion, which ground was intentionally deleted by the have.
framers of the 1987 Constitution. Considering the non-existence of an actual rebellion in
Maguindanao, Proclamation No. 1959 is unconstitutional for lack of factual basis as required Consequently, although the Constitution reserves to the SC the power to review the
under Section 18, Article VII of the Constitution for the declaration of martial law and sufficiency of the factual basis of the proclamation or suspension in a proper suit, it is implicit
suspension of the privilege of the writ of habeas corpus. that the Court must allow Congress to exercise its own review powers, which is automatic
rather than initiated. Only when Congress defaults in its express duty to defend the
Accordingly, I vote to GRANT the petitions and DECLARE Proclamation No. Constitution through such review should the SC step in as its final rampart. The constitutional
1959 UNCONSTITUTIONAL for failure to comply with Section 18, Article VII of the validity of the President’s proclamation of martial law or suspension of the writ of habeas
Constitution. corpus is first a political question in the hands of Congress before it becomes a justiciable one
in the hands of the Court.
ANTONIO T. CARPIO
Associate Justice Since Pres. Arroyo withdrew her proclamation before the joint Houses of Congress could
fulfill their automatic duty to review and validate or invalidate the same, then the petitions in
GR no. 190293 | 20 March 2012 these cases have become moot and the Court has nothing to review. The lifting of martial law
and restoration of the privilege of the writ of habeas corpus in Maguindanao was a
The President and the Congress act in tandem in exercising the power to proclaim martial law
supervening event that obliterated any justiciable controversy.
or suspend the privilege of the writ of habeas corpus.

FACTS:

On Nov. 23, 2009 heavily armed men, believed to be led by the ruling Ampatuan family,
gunned down and buried 57 innocent civilians in Maguindanao. On Dec. 4, 2009 Pres. Arroyo
issued PD Proclamation 1959 declaring martial law and suspending the privilege of the writ of
habeas corpus in Maguindanao. She submitted her report to Congress stating that she acted
based on her finding that lawless men have taken up arms in Maguindanao and risen against
the government. The Congress, in joint session, convened to review the validity of the
President’s action. However, two days later or before Congress could act, the President
issued Presidential Proclamation 1963, lifting martial law and restoring the privilege of the
264

EN BANC WHEREAS, today 23 May 2017, the same Maute terrorist group has taken over a hospital in
Marawi City, Lanao del Sur, established several checkpoints within the City, burned down
G.R. No. 231658 | July 4, 2017 certain government and private facilities and inflicted casualties on the part of Government
forces, and started flying the flag of the Islamic State of Iraq and Syria (ISIS) in several areas,
thereby openly attempting to remove from the allegiance to the Philippine Government this
REPRESENTATIVES EDCEL C. LAGMAN, TOMASITO S. VILLARIN, GARY C. ALEJANO,
part of Mindanao and deprive the Chief Executive of his powers and prerogatives to enforce
EMMANUEL A. BILLONES, AND TEDDY BRAWNER BAGUILAT, JR., Petitioners
the laws of the land and to maintain public order and safety in Mindanao, constituting the
vs.
crime of rebellion; and
HON. SALVADOR C. MEDIALDEA, EXECUTIVE SECRETARY; HON. DELFIN N. LORENZANA,
SECRETARY OF THE DEPARTMENT OF NATIONAL DEF'ENSE AND MARTIAL LAW
ADMINISTRATOR; AND GEN. EDUARDO ANO, CHIEF OF STAFF OF THE ARMED FORCES OF WHEREAS, this recent attack shows the capability of the Maute group and other rebel groups
THE PHILIPPINES AND MARTIAL LAW IMPLEMENTOR, Respondents to sow terror, and cause death and damage to property not only in Lanao del Sur but also in
other parts of Mindanao.
DECISION
NOW, THEREFORE, I, RODRIGO ROA DUTERTE, President of the Republic of the Philippines,
by virtue of the powers vested in me by the Constitution and by law, do hereby proclaim as
DEL CASTILLO, J.:
follows:

Effective May 23, 2017, and for a period not exceeding 60 days, President Rodrigo Roa
SECTION 1. There is hereby declared a state of martial law in the Mindanao group of islands
Duterte issued Proclamation No. 216 declaring a state of martial law and suspending the
for a period not exceeding sixty days, effective as of the date hereof.
privilege of the writ of habeas corpus in the whole of Mindanao.

SECTION 2. The privilege of the writ of habeas corpus shall likewise be suspended in the
The full text of Proclamation No. 216 reads as follows:
aforesaid area for the duration of the state of martial law.

WHEREAS, Proclamation No. 55, series of 2016, was issued on 04 September 2016 declaring a
DONE in the Russian Federation, this 23rd day of May in the year of our Lord, Two Thousand
state of national emergency on account of lawless violence in Mindanao;
and Seventeen.

WHEREAS, Section 18, Article VII of the Constitution provides that 'x x x In case of invasion or
Within the timeline set by Section 18, Article VII of the Constitution, the President submitted
rebellion, when the public safety requires it, he (the President) may, for a period not
to Congress on May 25, 2017, a written Report on the factual basis of Proclamation No. 216.
exceeding sixty days, suspend the privilege of the writ of habeas corpus or place the
Philippines or any part thereof under martial law x x x';
The Report pointed out that for decades, Mindanao has been plagued with rebellion and
lawless violence which only escalated and worsened with the passing of time.
WHEREAS, Article 134 of the Revised Penal Code, as amended by R.A. No. 6968, provides that
'the crime of rebellion or insurrection is committed by rising and taking arms against the
Government for the purpose of removing from the allegiance to said Government or its laws, Mindanao has been the hotbed of violent extremism and a brewing rebellion for decades. In
the territory of the Republic of the Philippines or any part thereof, of any body of land, naval more recent years, we have witnessed the perpetration of numerous acts of violence
or other armed forces, or depriving the Chief Executive or the Legislature, wholly or partially, challenging the authority of the duly constituted authorities, i.e., the Zamboanga siege, the
of any of their powers or prerogatives'; Davao bombing, the Mamasapano carnage, and the bombings in Cotabato, Sultan Kudarat,
Sulu, and Basilan, among others. Two armed groups have figured prominently in all these,
namely, the Abu Sayaff Group (ASG) and the ISIS-backed Maute Group.1
WHEREAS, part of the reasons for the issuance of Proclamation No. 55 was the series of
violent acts committed by the Maute terrorist group such as the attack on the military
outpost in Butig, Lanao del Sur in February 2016, killing and wounding several soldiers, and The President went on to explain that on May 23, 2017, a government operation to capture
the mass jailbreak in Marawi City in August 2016, freeing their arrested comrades and other the high-ranking officers of the Abu Sayyaf Group (ASG) and the Maute Group was
detainees; conducted. These groups, which have been unleashing havoc in Mindanao, however,
confronted the government operation by intensifying their efforts at sowing violence aimed
265

not only against the government authorities and its facilities but likewise against civilians and • By 1630H, the supply of power into Marawi City had been interrupted, and sporadic
their properties. As narrated in the President's Report: gunfights were heard and felt everywhere. By evening, the power outage had spread
citywide. (As of 24 May 2017, Marawi City's electric supply was still cut off, plunging the city
On 23 May 2017, a government operation to capture Isnilon Hapilon, a senior leader of the into total black-out.)
ASG, and Maute Group operational leaders, Abdullah and Omarkhayam Maute, was
confronted with armed resistance which escalated into open hostility against the • From 1800H to 1900H, the same members of the Maute Group ambushed and burned the
government. Through these groups' armed siege and acts of violence directed towards Marawi Police Station. A patrol car of the Police Station was also taken.
civilians and government authorities, institutions and establishments, they were able to take
control of major social, economic, and political foundations of Marawi City which led to its • A member of the Provincial Drug Enforcement Unit was killed during the takeover of the
paralysis. This sudden taking of control was intended to lay the groundwork for the eventual Marawi City Jail. The Maute Group facilitated the escape of at least sixty-eight (68) inmates
establishment of a DAESH wilayat or province in Mindanao. of the City Jail.

Based on verified intelligence reports, the Maute Group, as of the end of 2016, consisted of • The BJMP directed its personnel at the Marawi City Jail and other affected areas to
around two hundred sixty-three (263) members, fully armed and prepared to wage combat evacuate.
in furtherance of its aims. The group chiefly operates in the province of Lanao del Sur, but
has extensive networks and linkages with foreign and local armed groups such as the Jemaah
• By evening of 23 May 2017, at least three (3) bridges in Lanao del Sur, namely, Lilod,
Islamiyah, Mujahidin Indonesia Timur and the ASG. It adheres to the ideals being espoused
Bangulo, and Sauiaran, fell under the control of these groups. They threatened to bomb the
by the DAESH, as evidenced by, among others, its publication of a video footage declaring its
bridges to pre-empt military reinforcement.
allegiance to the DAESH. Reports abound that foreign-based terrorist groups, the ISIS (Islamic
State of Iraq and Syria) in particular, as well as illegal drug money, provide financial and
logistical support to the Maute Group. • As of 2222H, persons connected with the Maute Group had occupied several areas in
Marawi City, including Naga Street, Bangolo Street, Mapandi, and Camp Keithly, as well as
the following barangays: Basak Malutlot, Mapandi, Saduc, Lilod Maday, Bangon, Saber,
The events commencing on 23 May 2017 put on public display the groups' clear intention to
Bubong, Marantao, Caloocan, Banggolo, Barionaga, and Abubakar.
establish an Islamic State and their capability to deprive the duly constituted authorities - the
President, foremost - of their powers and prerogatives.2
• These lawless armed groups had likewise set up road blockades and checkpoints at the
Iligan City-Marawi City junction.
In particular, the President chronicled in his Report the events which took place on May 23,
2017 in Marawi City which impelled him to declare a state of martial law and suspend the
privilege of writ of habeas corpus, to wit: • Later in the evening, the Maute Group burned Dansalan College Foundation, Cathedral of
Maria Auxiliadora, the nun's quarters in the church, and the Shia Masjid Moncado Colony.
Hostages were taken from the church.
• At 1400H members of the Maute Group and ASG, along with their sympathizers,
commenced their attack on various facilities - government and privately owned - in the City
of Marawi. • About five (5) faculty members of Dansalan College Foundation had been reportedly killed
by the lawless groups.
• At 1600H around fifty (50) armed criminals assaulted Marawi City Jail being manage by the
Bureau of Jail Management and Penology (BJMP). • Other educational institutions were also burned, namely, Senator Ninoy Aquino College
Foundation and the Marawi Central Elementary Pilot School.
• The Maute Group forcibly entered the jail facilities, destroyed its main gate, and assaulted
on-duty personnel. BJMP personnel were disarmed, tied, and/or locked inside the cells. • The Maute Group also attacked Amai Pakpak Hospital and hoisted the DAESH flag there,
among other several locations. As of 0600H of 24May 2017, members of the Maute Group
were seen guarding the entry gates of Amai Pakpak Hospital. They held hostage the
• The group took cellphones, personnel-issued firearms, and vehicles (i.e., two [2] prisoner
employees of the Hospital and took over the PhilHealth office located thereat.
vans and private vehicles).
266

• The groups likewise laid siege to another hospital, Filipino-Libyan Friendship Hospital, particularly in the performance of their duties and functions, and untold hardships to the
which they later set ablaze. civilians, viz.:

• Lawless armed groups likewise ransacked the Landbank of the Philippines and Law enforcement and other government agencies now face pronounced difficulty sending
commandeered one of its armored vehicles. their reports to the Chief Executive due to the city-wide power outages. Personnel from the
BJMP have been prevented from performing their functions. Through the attack and
• Latest information indicates that about seventy-five percent (75%) of Marawi City has been occupation of several hospitals, medical services in Marawi City have been adversely
infiltrated by lawless armed groups composed of members of the Maute Group and the ASG. affected. The bridge and road blockades set up by the groups effectively deprive the
As of the time of this Report, eleven (11) members of the Armed Forces and the Philippine government of its ability to deliver basic services to its citizens. Troop reinforcements have
National Police have been killed in action, while thirty-five (35) others have been seriously been hampered, preventing the government from restoring peace and order in the area.
wounded. Movement by both civilians and government personnel to and from the city is likewise
hindered.
• There are reports that these lawless armed groups are searching for Christian communities
in Marawi City to execute Christians. They are also preventing Maranaos from leaving their The taking up of arms by lawless armed groups in the area, with support being provided by
homes and forcing young male Muslims to join their groups. foreign-based terrorists and illegal drug money, and their blatant acts of defiance which
embolden other armed groups in Mindanao, have resulted in the deterioration of public
order and safety in Marawi City; they have likewise compromised the security of the entire
• Based on various verified intelligence reports from the AFP and the PNP, there exists a
Island of Mindanao.5
strategic mass action of lawless armed groups in Marawi City, seizing public and private
facilities, perpetrating killings of government personnel, and committing armed uprising
against and open defiance of the government.3 The Report highlighted the strategic location of Marawi City and the crucial and significant
role it plays in Mindanao, and the Philippines as a whole. In addition, the Report pointed out
the possible tragic repercussions once Marawi City falls under the control of the lawless
The unfolding of these events, as well as the classified reports he received, led the President
groups.
to conclude that -

The groups' occupation of Marawi City fulfills a strategic objective because of its terrain and
These activities constitute not simply a display of force, but a clear attempt to establish the
the easy access it provides to other parts of Mindanao. Lawless armed groups have
groups' seat of power in Marawi City for their planned establishment of a DAESH wilayat or
historically used provinces adjoining Marawi City as escape routes, supply lines, and
province covering the entire Mindanao.
backdoor passages.

The cutting of vital lines for transportation and power; the recruitment of young Muslims to
Considering the network and alliance-building activities among terrorist groups, local
further expand their ranks and strengthen their force; the armed consolidation of their
criminals, and lawless armed men, the siege of Marawi City is a vital cog in attaining their
members throughout Marawi City; the decimation of a segment of the city population who
long-standing goal: absolute control over the entirety of Mindanao. These circumstances
resist; and the brazen display of DAESH flags constitute a clear, pronounced, and
demand swift and decisive action to ensure the safety and security of the Filipino people and
unmistakable intent to remove Marawi City, and eventually the rest of Mindanao, from its
preserve our national integrity.6
allegiance to the Government.

The President ended his Report in this wise:


There exists no doubt that lawless armed groups are attempting to deprive the President of
his power, authority, and prerogatives within Marawi City as a precedent to spreading their
control over the entire Mindanao, in an attempt to undermine his control over executive While the government is presently conducting legitimate operations to address the on-going
departments, bureaus, and offices in said area; defeat his mandate to ensure that all laws are rebellion, if not the seeds of invasion, public safety necessitates the continued
faithfully executed; and remove his supervisory powers over local govemments.4 implementation of martial law and the suspension of the privilege of the writ of habeas
corpus in the whole of Mindanao until such time that the rebellion is completely quelled. 7
According to the Report, the lawless activities of the ASG, Maute Group, and other criminals,
brought about undue constraints and difficulties to the military and government personnel,
267

In addition to the Report, representatives from the Executive Department, the military and Philippines".17 The Lagman Petition also avers that Lt. Gen. Salvador Mison, Jr. himself
police authorities conducted briefings with the Senate and the House of Representatives admitted that the current armed conflict in Marawi City was precipitated or initiated by the
relative to the declaration of martial law. government in its bid to capture Hapilon.18Based on said statement, it concludes that the
objective of the Maute Group's armed resistance was merely to shield Hapilon and the
After the submission of the Report and the briefings, the Senate issued P.S. Resolution No. Maute brothers from the government forces, and not to lay siege on Marawi City and
3888 expressing full support to the martial law proclamation and finding Proclamation No. remove its allegiance to the Philippine Republic.19 It then posits that if at all, there is only
216 "to be satisfactory, constitutional and in accordance with the law". In the same a threat of rebellion in Marawi City which is akin to "imminent danger" of rebellion, which is
Resolution, the Senate declared that it found "no compelling reason to revoke the same". no longer a valid ground for the declaration of martial law.20
The Senate thus resolved as follows:
Second, the Lagman Petition claims that the declaration of martial law has no sufficient
NOW, THEREFORE, BE IT RESOLVED, as it is hereby resolved, by way of the sense of the factual basis because the President's Report containef "false, inaccurate, contrived and
Senate, that the Senate finds the issuance of Proclamation No. 216 to be satisfactory, hyperbolic accounts".21
constitutional and in accordance with the law. The Senate hereby supports fully Proclamation
No. 216 and finds no compelling reason to revoke the sarne.9 It labels as false the claim in the President's Report that the Maute Group attacked Amai
Pakpak Medical Center. Citing online reports on the interview of Dr. Amer Saber (Dr. Saber),
The Senate's counterpart in the lower house shared the same sentiments. The House of the hospital's Chief, the Lagman Petition insists that the Maute Group merely brought an
Representatives likewise issued House Resolution No. 105010 "EXPRESSING THE FULL injured member to the hospital for treatment but did not overrun the hospital or harass the
SUPPORT OF THE HOUSE OF REPRESENTATIVES TO PRESIDENT RODRIGO DUTERTE AS IT hospital personnel. 22 The Lagman Petition also refutes the claim in the President's Report
FINDS NO REASON TO REVOKE PROCLAMATION NO. 216, ENTITLED 'DECLARING A STATE OF that a branch of the Landbank of the Philippines was ransacked and its armored vehicle
MARTIAL LAW AND SUSPENDING THE PRIVILEGE OF THE WRIT OF HABEAS CORPUS IN THE commandeered. It alleges that the bank employees themselves clarified that the bank was
WHOLE OF MINDANAO"'. not ransacked while the armored vehicle was owned by a third party and was empty at the
time it was commandeered.23 It also labels as false the report on the burning of the Senator
Ninoy Aquino College Foundation and the Marawi Central Elementary Pilot School. It avers
The Petitions
that the Senator Ninoy Aquino College Foundation is intact as of May 24, 2017 and that
according to Asst. Superintendent Ana Alonto, the Marawi Central Elementary Pilot School
G.R. No. 231658 (Lagman Petition) was not burned by the terrorists.24 Lastly, it points out as false the report on the beheading
of the police chief of Malabang, Lanao del Sur, and the occupation of the Marawi City Hall
On June 5, 2017, Representatives Edcel C. Lagman, Tomasito s. Villarin, Gary C. Alejano, and part of the Mindanao State University.25
Emmanuel A. Billones, and Teddy Brawner Baguilat, Jr. filed a Petition11 Under the Third
Paragraph of Section 18 of Article VII of the 1987 Constitution. Third, the Lagman Petition claims that the declaration of martial law has no sufficient factual
basis since the President's Report mistakenly included the attack on the military outpost in
First, the Lagman Petition claims that the declaration of martial law has no sufficient factual Butig, Lanao del Sur in February 2016, the mass jail break in Marawi City in August 2016, the
basis because there is no rebellion or invasion in Marawi City or in any part of Mindanao. It Zamboanga siege, the Davao market bombing, the Mamasapano carnage and other bombing
argues that acts of terrorism in Mindanao do not constitute rebellion12 since there is no proof incidents in Cotabato, Sultan Kudarat, and Basilan, as additional factual bases for the
that its purpose is to remove Mindanao or any part thereof from allegiance to the proclamation of martial law. It contends that these events either took place long before the
Philippines, its laws, or its territory.13 It labels the flying of ISIS flag by the Maute Group in conflict in Marawi City began, had long been resolved, or with the culprits having already
Marawi City and other outlying areas as mere propaganda114 and not an open attempt to been arrested.26
remove such areas from the allegiance to the Philippine Government and deprive the Chief
Executive of the assertion and exercise of his powers and prerogatives therein. It contends Fourth, the Lagman Petition claims that the declaration of martial law has no sufficient
that the Maute Group is a mere private army, citing as basis the alleged interview of Vera factual basis considering that the President acted alone and did not consult the military
Files with Joseph Franco wherein the latter allegedly mentioned that the Maute Group is establishment or any ranking official27 before making the proclamation.
more of a "clan's private militia latching into the IS brand theatrically to inflate perceived
capability".15 The Lagman Petition insists that during the briefing, representatives of the
Finally, the Lagman Petition claims that the President's proclamation of martial law lacks
military and defense authorities did not categorically admit nor deny the presence of an ISIS
sufficient factual basis owing to the fact that during the presentation before the Committee
threat in the country but that they merely gave an evasive answer16 that "there is ISIS in the
of the Whole of the House of Representatives, it was shown that the military was even
268

successful in pre-empting the ASG and the Maute Group's plan to take over Marawi City and In fine, the Cullamat Petition prays for the Court to declare Proclamation No. 216 as
other parts of Mindanao; there was absence of any hostile plan by the Moro Islamic unconstitutional or in the alternative, should the Court find justification for the declaration of
Liberation Front; and the number of foreign fighters allied with ISIS was martial law and suspension of the privilege of the writ of habeas corpus in Marawi City, to
"undetermined"28 which indicates that there are only a meager number of foreign fighters declare the same as unconstitutional insofar as its inclusion of the other parts of Mindanao.42
who can lend support to the Maute Group.29
C) G.R. No. 231774 (Mohamad Petition)
Based on the foregoing argumentation, the Lagman Petition asks the Court to: (1)"exercise
its specific and special jurisdiction to review the sufficiency of the factual basis of The Mohamad Petition, denominated as a "Petition for Review of the Sufficiency of [the]
Proclamation No. 216"; and (2) render "a Decision voiding and nullifying Proclamation No. Factual Basis of [the] Declaration of Martial Law and [the] Suspension of the Privilege of the
216" for lack of sufficient factual basis.30 Writ of Habeas Corpus,"43 labels itself as "a special proceeding"44 or an "appropriate
proceeding filed by any citizen"45 authorized under Section 18, Article VII of the Constitution.
In a Resolution31 dated June 6, 2017, the Court required respondents to comment on the
Lagman Petition and set the case for oral argument on June 13, 14, and 15, 2017. The Mohamad Petition posits that martial law is a measure of last resort46 and should be
invoked by the President only after exhaustion of less severe remedies.47 It contends that the
On June 9, 2017, two other similar petitions docketed as G.R. Nos. 231771 and 231774 were extraordinary powers of the President should be dispensed sequentially, i.e., first, the power
filed and eventually consolidated with G.R. No. 231658.32 to call out the armed forces; second, the power to suspend the privilege of the writ of habeas
corpus; and finally, the power to declare martial law.48 It maintains that the President has no
B) G.R. No. 231771 (Cullamat Petition) discretion to choose which extraordinary power to use; moreover, his choice must be
dictated only by, and commensurate to, the exigencies of the situation.49
The Cullamat Petition, "anchored on Section 18, Article VII"33 of the Constitution, likewise
seeks the nullification of Proclamation No. 216 for being unconstitutional because it lacks According to the Mohamad Petition, the factual situation in Marawi is not so grave as to
sufficient factual basis that there is rebellion in Mindanao and that public safety warrants its require the imposition of martial law.50 It asserts that the Marawi incidents "do not equate to
declaration. 34 the existence of a public necessity brought about by an actual rebellion, which would compel
the imposition of martial law or the suspension of the privilege of the writ of habeas
corpus".51 It proposes that "[m]artial law can only be justified if the rebellion or invasion has
In particular, it avers that the supposed rebellion described in Proclamation No. 216 relates
reached such gravity that [its] imposition x x x is compelled by the needs of public
to events happening in Marawi City only an not in the entire region of Mindanao. It
safety"52 which, it believes, is not yet present in Mindanao.
concludes that Proclamation No 216 "failed to show any factual basis for the imposition of
martial law in the entire Mindanao,"35 "failed to allege any act of rebellion outside Marawi
City, much less x x x allege that public safety requires the imposition o martial law in the Moreover, it alleges that the statements contained in the President's Report to the Congress,
whole of Mindanao".36 to wit: that the Maute Group intended to establish an Islamic State; that they have the
capability to deprive the duly constituted authorities of their powers and prerogatives; and
that the Marawi armed hostilities is merely a prelude to a grander plan of taking over the
The Cullamat Petition claims that the alleged "capability of the Maute Group and other rebel
whole of Mindanao, are conclusions bereft of substantiation.53
groups to sow terror and cause death and damage to property"37 does not rise to the level of
rebellion sufficient to declare martial law in the whole of Mindanao.38 It also posits that there
is no lawless violence in other parts of Mindanao similar to that in Marawi City.39 The Mohamad Petition posits that immediately after the declaration of martial law, and
without waiting for a congressional action, a suit may already be brought before the Court to
assail the sufficiency of the factual basis of Proclamation No. 216.
Moreover, the Cullamat Petition assails the inclusion of the phrase "other rebel groups" in
the last Whereas Clause of Proclamation No. 216 for being vague as it failed to identify these
rebel groups and specify the acts of rebellion that they were supposedly waging.40 Finally, in invoking this Court's power to review the sufficiency ofthe factual basis for the
declaration of martial law and the suspension of the privilege of the writ of habeas
corpus, the Mohamad Petition insists that the Court may "look into the wisdom of the
In addition, the Cullamat Petition cites alleged inaccuracies, exaggerations, and falsities in the
[President's] actions, [and] not just the presence of arbitrariness". 54 Further, it asserts that
Report of the President to Congress, particularly the attack at the Amai Pakpak Hospital, the
since it is making a negative assertion, then the burden to prove the sufficiency of the factual
ambush and burning of the Marawi Police Station, the killing of five teachers of Dansalan
basis is shifted to and lies on the respondents.55 It thus asks the Court "to compel the
College Foundation, and the attacks on various government facilities.41
269

[r]espondents to divulge relevant information"56in order for it to review the sufficiency of the declare martial law because to do so would subject the exercise of the President's discretion
factual basis. to an impossible standard.70 It reiterates that the President's decision should be guided only
by the information and data available to him at the time he made the determination.71 The
In closing, the Mohamad Petition prays for the Court to exercise its power to review, "compel OSG thus asserts that facts that were established after the declaration of martial law
respondents to present proof on the factual basis [of] the declaration of martial law and the should not be considered in the review of the sufficiency of the factual basis of the
suspension of the privilege of the writ of habeas corpus in Mindanao"57 and declare as proclamation of martial law. The OSG suggests that the assessment of after-proclamation
unconstitutional Proclamation No. 216 for lack of sufficient factual basis. facts lies with the President and Congress for the purpose of determining the propriety of
revoking or extending the martial law. The OSG fears that if the Court considers after-
proclamation-facts in its review of the sufficiency of the factual basis for the proclamation, it
The Consolidated Comment
would in effect usurp the powers of the Congress to determine whether martial law should
be revoked or extended.72
The respondents' Consolidated Comment58 was filed on June 12, 2017, as required by the
Court. Noting that the same coincided with the celebration of the 119th anniversary of the
It is also the assertion of the OSG that the President could validly rely on intelligence reports
independence of this Republic, the Office of the Solicitor General (OSG) felt that "defending
coming from the Armed Forces of the Philippines;73 and that he could not be expected to
the constitutionality of Proclamation No. 216" should serve as "a rallying call for every
personally determine the veracity of thecontents of the reports. 74 Also, since the power to
Filipino to unite behind one true flag and defend it against all threats from within and outside
impose martial law is vested solely on the President as Commander-in-Chief, the lack of
our shores".59
recommendation from the Defense Secretary, or any official for that matter, will not nullify
the said declaration, or affect its validity, or compromise the sufficiency of the factual basis.
The OSG acknowledges that Section 18, Article VII of the Constitution vests the Court with
the authority or power to review the sufficiency of the factual basis of the declaration of
Moreover, the OSG opines that the petitioners miserably failed to validly refute the facts
martial law.60 The OSG, however, posits that although Section 18, Article VII lays the basis for
cited by the President in Proclamation No. 216 and in his Report to the Congress by merely
the exercise of such authority or power, the same constitutional provision failed to specify
citing news reports that supposedly contradict the facts asserted therein or by criticizing in
the vehicle, mode or remedy through which the "appropriate proceeding" mentioned therein
piecemeal the happenings in Marawi. For the OSG, the said news articles are "hearsay
may be resorted to. The OSG suggests that the "appropriate proceeding" referred to in
evidence, twice removed,"75 and thus inadmissible and without probative value, and could
Section 18, Article VII may be availed of using the vehicle, mode or remedy of
not overcome the "legal presumption bestowed on governmental acts".76
a certiorari petition, either under Section 1 or 5, of Article VIII.61Corollarily, the OSG
maintains that the review power is not mandatory, but discretionary only, on the part of the
Court. 62 The Court has the discretion not to give due course to the petition. 63 Finally, the OSG points out that it has no duty or burden to prove that Proclamation No. 216
has sufficient factual basis. It maintains that the burden rests with the petitioners. However,
the OSG still endeavors to lay out the factual basis relied upon by the President "if only to
Prescinding from the foregoing, the OSG contends that the sufficiency of the factual basis of
remove any doubt as to the constitutionality of Proclamation No. 216".77
Proclamation No. 216 should be reviewed by the Court "under the lens of grave abuse of
discretion"64 and not the yardstick of correctness of the facts.65 Arbitrariness, not
correctness, should be the standard in reviewing the sufficiency of factual basis. The facts laid out by the OSG in its Consolidated Comment will be discussed in detail in the
Court's Ruling.
The OSG maintains that the burden lies not with the respondents but with the petitioners to
prove that Proclamation No. 216 is bereft of factual basis.1âwphi1 It thus takes issue with ISSUES
petitioners' attempt to shift the burden of proof when they asked the Court "to compel [the]
respondents to present proof on the factual basis"66 of Proclamation No. 216. For the OSG, The issues as contained in the revised Advisory78 are as follows:
"he who alleges must prove"67 and that governmental actions are presumed to be valid and
constitutional.68 1. Whether or not the petitions docketed as G.R. Nos. 231658, 231771, and 231774 are the
"appropriate proceeding" covered by Paragraph 3, Section 18, Article VII of the Constitution
Likewise, the OSG posits that the sufficiency of the factual basis must be assessed from the sufficient to invoke the mode of review required of this Court when a declaration of martial
trajectory or point of view of the President and base on the facts available to him at the time law or the suspension of the privilege of the writ of habeas corpus is promulgated;
the decision was made.69 It argues that the sufficiency of the factual basis should be
examined not based on the facts discovered after the President had made his decision to
270

2. Whether or not the President in declaring martial law and suspending the privilege of the b. for a declaration of martial law or the suspension of the privilege of the writ of habeas
writ of habeas corpus: corpus in the entire Mindanao 1 region;

a. is required to be factually correct or only not arbitrary in his appreciation of facts; 8. Whether or not terrorism or acts attributable to terrorism are equivalent to actual
rebellion and the requirements of public safety sufficient to declare martial law or suspend
b. is required to obtain the favorable recommendation thereon of the Secretary of National the privilege of the writ of habeas corpus; and
Defense;
9. Whether or not nullifying Proclamation No. 216 of 23 May 2017 will:
c. is required to take into account only the situation at the time of the proclamation, even if
subsequent events prove the situation to have not been accurately reported; a. have the effect of recalling Proclamation No. 55 s. 2016; or

3. Whether or not the power of this Court to review the sufficiency of the factual basis [of] b. also nullify the acts of the President in calling out the armed forces to quell lawless
the proclamation of martial law or the suspension of the privilege of the writ of habeas violence in Marawi and other parts of the Mindanao region.
corpus is independent of the actual actions that have been taken by Congress jointly or
separately; After the oral argument, the parties submitted their respective memoranda and
supplemental memoranda.
4. Whether or not there were sufficient factual [basis] for the proclamation of martial law or
the suspension of the privilege of the writ of habeas corpus; OUR RULING

a. What are the parameters for review? I. Locus standi of petitioners.

b. Who has the burden of proof? One of the requisites for judicial review is locus standi, i.e., "the constitutional question is
brought before [the Court] by a party having the requisite 'standing' to challenge it."79 As a
c. What is the threshold of evidence? general rule, the challenger must have "a personal and substantial interest in the case such
that he has sustained, or will sustain, direct injury as a result of its enforcement."80 Over the
5. Whether the exercise of the power of judicial review by this Court involves the calibration years, there has been a trend towards relaxation of the rule on legal standing, a prime
of graduated powers granted the President as Commander-in-Chief, namely calling out example of which is found in Section 18 of Article VII which provides that any citizen may file
powers, suspension of the privilege of the writ of habeas corpus, and declaration of martial the appropriate proceeding to assail the sufficiency of the factual basis of the declaration of
law; martial law or the suspension of the privilege of the writ of habeas corpus. "[T]he only
requisite for standing to challenge the validity of the suspension is that the challenger be a
citizen. He need not even be a taxpayer."81
6. Whether or not Proclamation No. 216 of 23 May 2017 may be considered, vague and thus
null and void:
Petitioners in the Cullamat Petition claim to be "suing in their capacities as citizens of the
Republic;"82 similarly, petitioners in the Mohamad Petition all claim to be "Filipino citizens, all
a. with its inclusion of "other rebel groups;" or
women, all of legal [age], and residents of Marawi City".83 In the Lagman Petition, however,
petitioners therein did not categorically mention that they are suing's citizens but merely
b. since it has no guidelines specifying its actual operational parameters within the entire referred to themselves as duly elected Representatives.84 That they are suing in their official
Mindanao region; capacities as Members of Congress couLd have elicited a vigorous discussion considering the
issuance by the House of Representatives of House Resolution No. 1050 expressing full
7. Whether or not the armed hostilities mentioned in Proclamation No. 216 and in the Report support to President Duterte and finding no reason to revoke Proclamation No. 216. By such
of the President to Congress are sufficient [bases]: resolution, the House of Representatives is declaring that it finds no reason to review the
sufficiency of the factual basis of the martial law declaration, which is in direct contrast to
a. for the existence of actual rebellion; or the views and arguments being espoused by the petitioners in the Lagman Petition.
Considering, however, the trend towards relaxation of the rules on legal standing, as well
271

as the transcendental issues involved in the present Petitions, the Court will exercise judicial It could not have been the intention of the framers of the Constitution that the phrase "in an
self-restraint85 and will not venture into this matter. After all, "the Court is not entirely appropriate proceeding" would refer to a Petition for Certiorari pursuant to Section 1 or
without discretion to accept a suit which does not satisfy the requirements of a [bona Section 5 of Article VIII. The standard of review in a petition for certiorari is whether the
fide] case or of standing. Considerations paramount to [the requirement of legal standing] respondent has committed any grave abuse of discretion amounting to lack or excess of
could compel assumption of jurisdiction."86 In any case, the Court can take judicial jurisdiction in the performance of his or her functions. Thus, it is not the proper tool to
cognizance of the fact that petitioners in the Lagman Petition are all citizens of the review the sufficiency of the factual basis of the proclamationor suspension. It must be
Philippines since Philippine citizenship is a requirement for them to be elected as emphasized that under Section 18, Article VII, the Court is tasked to review the sufficiency of
representatives. We will therefore consider them as suing in their own behalf as citizens of the factual basis of the President's exercise of emergency powers. Put differently, if this
this country. Besides, respondents did not question petitioners' legal standing. Court applies the standard of review used in a petition for certiorari, the same would
emasculate its constitutional task under Section 18, Article VII.
II. Whether or not the petitions are the "appropriate proceeding" covered by paragraph 3,
Section 18, Article VII of the Constitution sufficient to invoke the mode of review required by c) Purpose/significance of Section 18, Article VII is to constitutionalize the pre-Marcos martial
the Court. law ruling in In the Matter of the Petition for Habeas Corpus of Lansang.

All three petitions beseech the cognizance of this Court based on the third paragraph of The third paragraph of Section 18, Article VII was inserted by the framers of the 1987
Section 18, Article VII (Executive Department) of the 1987 Constitution which provides: Constitution to constitutionalize the pre-Marcos martial law ruling of this Court in In the
Matter of the Petition for Habeas Corpus of Lansang,92 to wit: that the factual basis of the
The Supreme Court may review, in an appropriate proceeding filed by any citizen, the declaration of martial law or the suspension of the privilege of the writ of habeas corpus is
sufficiency of the factual basis of the proclamation of martial law or the suspension of the not a political question but precisely within the ambit of judicial review.
privilege of the writ or the extension thereof, and must promulgate its decision thereon
within thirty days from its filing. "In determining the meaning, intent, and purpose of a law or constitutional provision, the
history of the times out of which it grew and to which it may be rationally supposed to bear
During the oral argument, the petitioners theorized that the jurisdiction of this Court under some direct relationship, the evils intended to be remedied, and the good to be
the third paragraph of Section 18, Article VII is sui generis.87 It is a special and specific accomplished are proper subjects of inquiry."93 Fr. Joaquin G. Bernas, S.J. (Fr. Bernas), a
jurisdiction of the Supreme Court different from those enumerated in Sections 1 and 5 of member of the Constitutional Commission that drafted the 1987 Constitution, explained:
Article VIII.88
The Commander-in-Chief provisions of the 1935 Constitution had enabled President
The Court agrees. Ferdinand Marcos to impose authoritarian rule on the Philippines from 1972 to
1986. Supreme Court decisions during that period upholding the actions taken by Mr. Marcos
made authoritarian rule part of Philippine constitutional jurisprudence. The members of the
a) Jurisdiction must be specifically conferred by theConstitution or by law.
Constitutional Commission, very much aware of these facts, went about reformulating the
Commander-in-Chief powers with a view to dismantling what had been constructed during
It is settled that jurisdiction over the subject matter is conferred only by the Constitution or the authoritarian years. The new formula included revised grounds for the activation of
by the law.89 Unless jurisdiction has been specifically conferred by the Constitution or by emergency powers, the manner of activating them, the scope of the powers, and review of
some legislative act, no body or tribunal has the power to act or pass upon a matter brought presidential action.94 (Emphasis supplied)
before it for resolution. It is likewise settled that in the absence of a clear legislative intent,
jurisdiction cannot be implied from the language of the Constitution or a statute.90 It must
To recall, the Court held in the 1951 case of Montenegro v. Castaneda95 that the authority to
appear clearly from the law or it will not be held to exist.91
decide whether there is a state of rebellion requiring the suspension of the privilege of the
writ of habeas corpus is lodged with the President and his decision thereon is final and
A plain reading of the afore-quoted Section 18, Article VII reveals that it specifically grants conclusive upon the courts. This ruling was reversed in the 1971 case of Lansang where it
authority to the Court to determine the sufficiency of the factual basis of the proclamation of was held that the factual basis of the declaration of martial law and the suspension of the
martial law or suspension of the privilege of the writ of habeas corpus. privilege of the writ of habeas corpus is not a political question and is within the ambit of
judicial review.96 However, in 1983, or after the declaration of martial law by former
b) "In an appropriate proceeding" does not refer to a petition for certiorari filed under Section President Ferdinand E. Marcos, the Court, in Garcia-Padilla v. Enrile,97 abandoned the ruling
1 or 5 of Article VIII in Lansang and reverted to Montenegro. According to the Supreme Court, the constitutional
272

power of the President to suspend the privilege of the writ of habeas corpus is not subject to Whereas, the rebellion and armed action undertaken by these lawless elements of the
judicial inquiry.98 Communists and other armed aggrupations organized to overthrow the Republic of the
Philippines by armed violence and force have assumed the magnitude of an actual state of
Thus, by inserting Section 18 in Article VII which allows judicial review of the declaration of war against our people and the Republic of the Philippines.
martial law and suspension of the privilege of the writ of habeas corpus, the framers of the
1987 Constitution in effect constitutionalized and reverted to the Lansang doctrine. And may I also call the attention of the Gentleman to General Order No. 3, also promulgated
by Ferdinand E. Marcos, in his capacity as Commander-in-Chief of all the Armed Forces of the
d) Purpose of Section 18, Article VII is to provide additional safeguard against possible abuse Philippines and pursuant to Proclamation No. 1081 dated September 21, 1972 wherein he
by the President on the exercise of the extraordinary powers. said, among other things:

Section 18, Article VII is meant to provide additional safeguard against possible abuse by the Whereas, martial law having been declared because of wanton destruction of lives and
President in the exercise of his power to declare martial law or suspend the privilege of the properties, widespread lawlessness and anarchy and chaos and disorder now prevailing
writ of habeas corpus. Reeling from the aftermath of the Marcos martial law, the framers of throughout the country, which condition has been brought about by groups of men who are
the Constitution deemed it wise to insert the now third paragraph of Section 18 of Article actively engaged in a criminal conspiracy to seize political and state power in the Philippines
VII.99 This is clear from the records of the Constitutional Commission when its members were in order to take over the government by force and violence, the extent of which has now
deliberating on whether the President could proclaim martial law even without the assumed the proportion of an actual war against our people and the legitimate government
concurrence of Congress. Thus: ...

MR. SUAREZ. Thank you, Madam President. And he gave all reasons in order to suspend the privilege of the writ of habeas corpus and
declare martial law in our country without justifiable reason. Would the Gentleman still insist
on the deletion of the phrase 'and, with the concurrence of at least a majority of all the
The Commissioner is proposing a very substantial amendment because this means that he is
members of the Congress'?
vesting exclusively unto the President the right to determine the factors which may lead to
the declaration of martial law and the suspension of the writ of habeas corpus. I suppose he
has strong and compelling reasons in seeking to delete this particular, phrase. May we be MR. MONSOD. Yes, Madam President, in the case of Mr.Marcos, he is undoubtedly an
informed of his good and substantial reasons? aberration in our history and national consciousness. But given the possibility that there
would be another Marcos, our Constitution now has sufficient safeguards. As I said, it is not
really true, as the Gentleman has mentioned, that there is an exclusive right to determine the
MR. MONSOD. This situation arises in cases of invasion or rebellion. And in previous
factual basis because the paragraph beginning on line 9 precisely tells us that the Supreme
interpellations regarding this phrase, even during the discussions on the Bill of Rights, as I
Court may review, in an appropriate proceeding filed by any citizen, the sufficiency of the
understand it, the interpretation is a situation of actual invasion or rebellion. In these
factual basis of the proclamation of martial law or the suspension of the privilege of the writ
situations, the President has to act quickly. Secondly, this declaration has a time fuse. It is
or the extension thereof and must promulgate its decision on the same within 30 days from
only good for a maximum of 60 days. At the end of 60 days, it automatically terminates.
its filing.
Thirdly, the right of the judiciary to inquire into the sufficiency of the factual basis of the
proclamation always exists, even during those first 60 days.
I believe that there are enough safeguards. The Constitution is supposed to balance the
interests of the country. And here we are trying to balance the public interest in case of
MR. SUAREZ. Given our traumatic experience during the past administration, if we give
invasion or rebellion as against the rights of citizens. And I am saying that there are enough
exclusive right to the President to determine these factors, especially the existence of an
safeguards, unlike in 1972 when Mr. Marcos was able to do all those things mentioned.100
invasion or rebellion and the second factor of determining whether the public safety requires
it or not, may I call the attention of the Gentleman to what happened to us during the past
administration. Proclamation No. 1081 was issued by Ferdinand E. Marcos in his capacity as To give more teeth to this additional safeguard, the framers of the 1987 Constitution not
President of the Philippines by virtue of the powers vested upon him purportedly under only placed the President's proclamation of martial law or suspension of the privilege of the
Article VII, Section 10 (2) of the Constitution, wherein he made this predicate under the writ of habeas corpus within the ambit of judicial review, it also relaxed the rule on standing
"Whereas" provision: by allowing any citizen to question before this Court the sufficiency of the factual basis of
such proclamation or suspension. Moreover, the third paragraph of Section 18, Article VII
veritably conferred upon any citizen a demandable right to challenge the sufficiency of the
factual basis of said proclamation or suspension. It further designated this Court as the
273

reviewing tribunal to examine, in an appropriate proceeding, the sufficiency of the factual under the third paragraph of Section 18, Article VII on top of the expanded jurisdiction of this
basis and to render its decision thereon within a limited period of 30 days from date of filing. Court.

e) Purpose of Section 18, Article VII is to curtail the extent of the powers of the President. g) Jurisdiction of the Court is not restricted to those enumerated in Sections I and 5 of
Article VIII
The most important objective, however, of Section 18, Article VII is the curtailment of the
extent of the powers of the Commander-in-Chief. This is the primary reason why the The jurisdiction of this Court is not restricted to those enumerated in Sections 1 and 5 of
provision was not placed in Article VIII or the Judicial Department but remained under Article Article VIII. For instance, its jurisdiction to be the sole judge of all contests relating to the
VII or the Executive Department. election, returns, and qualifications of the President or Vice-President can be found in the
last paragraph of Section 4, Article VII.102 The power of the Court to review on certiorari the
During the closing session of the Constitutional Commission's deliberations, President Cecilia decision, order, or ruling of the Commission on Elections and Commission on Audit can be
Muñoz Palma expressed her sentiments on the 1987 Constitution. She said: found in Section 7, Article IX(A).103

The executive power is vested in the President of the Philippines elected by the people for a h) Unique features of the third paragraph of Section 18, Article VII make it sui generis.
six-year term with no reelection for the duration of his/her life. While traditional powers
inherent in the office of the President are granted, nonetheless for the first time, there are The unique features of the third paragraph of Section 18, Article VII clearly indicate that it
specific provisions which curtail the extent of such powers. Most significant is the power of should be treated as sui generis separate and different from those enumerated in Article VIII.
the Chief Executive to suspend the privilege of the writ of habeas corpus or proclaim martial Under the third paragraph of Section 18, Article VII, a petition filed pursuant therewith will
law. follow a different rule on standing as any citizen may file it. Said provision of the Constitution
also limits the issue to the sufficiency of the factual basis of the exercise by the Chief
The flagrant abuse of that power of the Commander-in-Chief by Mr. Marcos caused the Executive of his emergency powers. The usual period for filing pleadings in Petition
imposition of martial law for more than eight years and the suspension of the privilege of the for Certiorari is likewise not applicable under the third paragraph of Section 18, Article VII
writ even after the lifting of martial law in 1981. The new Constitution now provides that considering the limited period within which this Court has to promulgate its decision.
those powers can be exercised only in two cases, invasion or rebellion when public safety
demands it, only for a period not exceeding 60 days, and reserving to Congress the power to A proceeding "[i]n its general acceptation, [is] the form in which actions are to be brought
revoke such suspension or proclamation of martial law which congressional action may not and defended, the manner of intervening in suits, of conducting them, the mode of deciding
be revoked by the President. More importantly, the action of the President is made subject them, of opposing judgments, and of executing."104In fine, the phrase "in an appropriate
to judicial review, thereby again discarding jurisprudence which render[s] the executive proceeding" appearing on the third paragraph of Section 18, Article VII refers to any action
action a political question and beyond the jurisdiction of the courts to adjudicate. initiated by a citizen for the purpose of questioning the sufficiency of the factual basis of the
exercise of the Chief Executive's emergency powers, as in these cases. It could be
For the first time, there is a provision that the state of martial law does not suspend the denominated as a complaint, a petition, or a matter to be resolved by the Court.
operation of the Constitution nor abolish civil courts or legislative assemblies, or vest
jurisdiction to military tribunals over civilians, or suspend the privilege of the writ. Please III. The power of the Court to review the sufficiency of the factual basis of the proclamation of
forgive me if, at this point, I state that this constitutional provision vindicates the dissenting martial law or the suspension of the privilege of the writ of habeas corpus under Section 18,
opinions I have written during my tenure in the Supreme Court in the martial law cases.101 Article VII of the 1987 Constitution is independent of the actions taken by Congress.

f) To interpret "appropriate proceeding" as filed under Section 1 of Article VIII would be During the oral argument,105 the OSG urged the Court to give! deference to the actions of the
contrary to the intent of the Constitution. two co-equal branches of the Government: on' the part of the President as Commander-in-
Chief, in resorting to his extraordinary powers to declare martial law and suspend the
To conclude that the "appropriate proceeding" refers to a Petition for Certiorari filed under privilege of the writ of habeas corpus; and on the part of Congress, in giving its imprimatur to
the expanded jurisdiction of this Court would, therefore, contradict the clear intention of the Proclamation No. 216 and not revoking the same.
framers of the Constitution to place additional safeguards against possible martial law abuse
for, invariably, the third paragraph of Section 18, Article VII would be subsumed under The framers of the 1987 Constitution reformulated the scope of the extraordinary powers of
Section 1 of Article VIII. In other words, the framers of the Constitution added the safeguard the President as Commander-in-Chief and the review of the said presidential action. In
274

particular, the President's extraordinary powers of suspending the privilege of the writ MS. QUESADA. But now, if they cannot meet because they have been arrested or that the
of habeas corpus and imposing martial law are subject to the veto powers of the Court and Congress has been padlocked, then who is going to declare that such a proclamation was not
Congress. warranted?

a) The judicial power to review versus the congressional power to revoke. MR. REGALADO. May I also inform Commissioner Quesada that the judiciary is not exactly
just standing by. A petition for a writ of habeas corpus, if the Members are detained, can
The Court may strike down the presidential proclamation in an appropriate proceeding filed immediately be applied for, and the Supreme Court shall also review the factual basis. x x x107
by any citizen on the ground of lack of sufficient factual basis. On the other hand, Congress
may revoke the proclamation or suspension, which revocation shall not be set aside by the c) Re-examination of the Court's pronouncement in Fortun v. President Macapagal-Arroyo
President.
Considering the above discussion, the Court finds it imperative to re-examine, reconsider,
In reviewing the sufficiency of the factual basis of the proclamation or suspension, the Court and set aside its pronouncement in Fortun v. President Macapagal-Arroyo108 to the effect
considers only the information and data available to the President prior to or at the time of that:
the declaration; it is not allowed td "undertake an independent investigation beyond the
pleadings."106 On the other hand, Congress may take into consideration not only data Consequently, although the Constitution reserves to the Supreme Court the power to review
available prior to, but likewise events supervening the declaration. Unlike the Court I which the sufficiency of the factual basis of the proclamation or suspension in a proper suit, it is
does not look into the absolute correctness of the factual basis as will be discussed below, implicit that the Court must allow Congress to exercise its own review powers, which is
Congress could probe deeper and further; it can delve into the accuracy of the facts automatic rather than initiated. Only when Congress defaults in its express duty to defend
presented before it. the Constitution through such review should the Supreme Court step in as its final rampart.
The constitutional validity of the President's proclamation of martial law or suspension of the
In addition, the Court's review power is passive; it is only initiated by the filing of a petition writ of habeas corpus is first a political question in the hands of Congress before it becomes a
"in an appropriate proceeding" by a citizen. On the other hand, Congress' review mechanism justiciable one in the hands of the Court.109
is automatic in the sense that it may be activated by Congress itself at any time after the
proclamation or suspension was made. If the Congress procrastinates or altogether fails to fulfill its duty respecting the proclamation
or suspension within the short time expected of it, then the Court can step in, hear the
Thus, the power to review by the Court and the power to revoke by Congress are not only petitions challenging the President's action, and ascertain if it has a factual basis. x x x110
totally different but likewise independent from each other although concededly, they have
the same trajectory, which is, the nullification of the presidential proclamation. Needless to By the above pronouncement, the Court willingly but unwittingly clipped its own power and
say, the power of the Court to review can be exercised independently from the power of surrendered the same to Congress as well as: abdicated from its bounden duty to review.
revocation of Congress. Worse, the Court considered' itself just on stand-by, waiting and willing to act as a substitute
in case Congress "defaults." It is an aberration, a stray declaration, which must be rectified
b) The framers of the 1987 Constitution intended the judicial power to review to be exercised and set aside in this proceeding.111
independently from the congressional power to revoke.
We, therefore, hold that the Court can simultaneously exercise its power of review with, and
If only to show that the intent of the framers of the 1987 Constitution was to vest the Court independently from, the power to revoke by Congress. Corollary, any perceived inaction or
and Congress with veto powers independently from each other, we quote the following default on the part of Congress does not deprive or deny the Court of its power to review.
exchange:
IV. The judicial power to review the sufficiency of factual basis of the declaration of martial
MS. QUESADA. Yesterday, the understanding of many was that there would be safeguards law or the suspension of the privilege of the writ of habeas corpus does not extend to the
that Congress will be able to revoke such proclamation. calibration of the President's decision of which among his graduated powers he will avail of in
a given situation.
MR. RAMA. Yes.
275

The President as the Commander-in-Chief wields the extraordinary powers of: a) calling out that he could not exercise if there is no martial law? Interestingly, these questions were also
the armed forces; b) suspending the privilege of the writ of habeas corpus; and c) declaring discussed by the framers of the 1987 Constitution, viz.:
martial law.112 These powers may be resorted to only under specified conditions.
FR. BERNAS. That same question was asked during the meetings of the Committee: What
The framers of the 1987 Constitution reformulated the powers of the Commander-in-Chief precisely does martial law add to the power of the President to call on the armed forces? The
by revising the "grounds for the activation of emergency powers, the manner of activating first and second lines in this provision state:
them, the scope of the powers, and review of presidential action." 113
A state of martial law does not suspend the operation of the Constitution, nor supplant the
a) Extraordinary powers of the President distinguished. functioning of the civil courts or legislative assemblies...

Among the three extraordinary powers, the calling out power is the most benign and involves The provision is put there, precisely, to reverse the doctrine of the Supreme Court. I think it is
ordinary police action.114 The President may resort to this extraordinary power whenever it the case of Aquino v. COMELEC where the Supreme Court said that in times of martial law,
becomes necessary to prevent or suppress lawless violence, invasion, or rebellion. "[T]he the President automatically has legislative power. So these two clauses denied that. A state
power to call is fully discretionary to the President;"115 the only limitations being that he acts of martial law does not suspend the operation of the Constitution; therefore, it does not
within permissible constitutional boundaries or in a manner not constituting grave abuse of suspend the principle of separation of powers.
discretion.116 In fact, "the actual use to which the President puts the armed forces is x x x not
subject to judicial review."117 The question now is: During martial law, can the President issue decrees? The answer we
gave to that question in the Committee was: During martial law, the President may have the
The extraordinary powers of suspending the privilege of the writ of habeas corpus and/or powers of a commanding general in a theatre of war. In actual war when there is fighting in
declaring martial law may be exercised only when there is actual invasion or rebellion, and an area, the President as the commanding general has the authority to issue orders which
public safety requires it. The 1987 Constitution imposed the following limits in the exercise of have the effect of law but strictly in a theater of war, not in the situation we had during the
these powers: "(1) a time limit of sixty days; (2) review and possible revocation by Congress; period of martial law. In other words, there is an effort here to return to the traditional
[and] (3) review and possible nullification by the Supreme Court."118 concept of martial law as it was developed especially in American jurisprudence, where
martial law has reference to the theater of war.124
The framers of the 1987 Constitution eliminated insurrection, and the phrase "imminent
danger thereof' as grounds for the suspension of the privilege of the writ of habeas corpus or FR. BERNAS. This phrase was precisely put here because we have clarified the meaning of
declaration of martial law.119 They perceived the phrase "imminent danger" to be "fraught martial law; meaning, limiting it to martial law as it has existed in the jurisprudence in
with possibilities of abuse;"120 besides, the calling out power of the President "is sufficient for international law, that it is a law for the theater of war. In a theater of war, civil courts are
handling imminent danger."121 unable to function. If in the actual theater of war civil courts, in fact, are unable to function,
then the military commander is authorized to give jurisdiction even over civilians to military
The powers to declare martial law and to suspend the privilege of the writ of habeas courts precisely because the civil courts are closed in that area. But in the general area where
corpus involve curtailment and suppression of civil rights and individual freedom. Thus, the the civil courts are open then in no case can the military courts be given jurisdiction over
declaration of martial law serves as a warning to citizens that the Executive Department has civilians. This is in reference to a theater of war where the civil courts, in fact, are unable to
called upon the military to assist in the maintenance of law and order, and while the function.
emergency remains, the citizens must, under pain of arrest and punishment, not act in a
manner that will render it more difficult to restore order and enforce the law.122 As such, MR. FOZ. It is a state of things brought about by the realities of the situation in that specified
their exercise requires more stringent safeguards by the Congress, and review by the critical area.
Court.123
FR. BERNAS. That is correct.
b) What really happens during martial law?
MR. FOZ. And it is not something that is brought about by a declaration of the Commander-
During the oral argument, the following questions cropped up: What really happens during in-Chief.
the imposition of martial law? What powers could the President exercise during martial law
276

FR. BERNAS. It is not brought about by a declaration of the Commander-in-Chief. The These extraordinary powers are conferred by the Constitution with the President as
understanding here is that the phrase 'nor authorize the conferment of jurisdiction on Commander-in-Chief; it therefore necessarily follows that the power and prerogative to
military courts and agencies over civilians' has reference to the practice under the Marcos determine whether the situation warrants a mere exercise of the calling out power; or
regime where military courts were given jurisdiction over civilians. We say here that we will whether the situation demands suspension of the privilege of the writ of habeas corpus; or
never allow that except in areas where civil courts are, in fact, unable to function and it whether it calls for the declaration of martial law, also lies, at least initially, with the
becomes necessary for some kind of court to function.125 President. The power to choose, initially, which among these extraordinary powers to wield
in a given set of conditions is a judgment call on the part of the President. As Commander-in-
A state of martial law is peculiar because the President, at such a time, exercises police Chief, his powers are broad enough to include his prerogative to address exigencies or
power, which is normally a function of the Legislature. In particular, the President exercises threats that endanger the government, and the very integrity of the State. 132
police power, with the military’s assistance, to ensure public safety and in place of
government agencies which for the time being are unable to cope with the condition in a It is thus beyond doubt that the power of judicial review does not extend to calibrating the
locality, which remains under the control of the State.126 President's decision pertaining to which extraordinary power to avail given a set of facts or
conditions. To do so would be tantamount to an incursion into the exclusive domain of the
In David v. President Macapagal-Arroyo,127 the Court, quoting Justice Vicente V. Mendoza's Executive and an infringement on the prerogative that solely, at least initially, lies with the
(Justice Mendoza) Statement before the Senate Committee on Justice on March 13, 2006, President.
stated that under a valid declaration of martial law, the President as Commander-in-Chief
may order the "(a) arrests and seizures without judicial warrants; (b) ban on public d) The framers of the 1987 Constitution intended the Congress not to interfere a priori in the
assemblies; (c) [takeover] of news media and agencies and press censorship; and (d) issuance decision-making process of the President.
of Presidential Decrees x x x".128
The elimination by the framers of the 1987 Constitution of the requirement of prior
Worthy to note, however, that the above-cited acts that the President may perform do not concurrence of the Congress in the initial imposition of martial law or suspension of the
give him unbridled discretion to infringe on the rights of civilians during martial law. This is privilege of the writ of habeas corpus further supports the conclusion that judicial review
because martial law does not suspend the operation of the Constitution, neither does it does not include the calibration of the President's decision of which of his graduated powers
supplant the operation of civil courts or legislative assemblies. Moreover, the guarantees will be availed of in a given situation. Voting 28 to 12, the framers of the 1987 Constitution
under the Bill of Rights remain in place during its pendency. And in such instance where the removed the requirement of congressional concurrence in the first imposition of martial law
privilege of the writ of habeas corpus is also suspended, such suspension applies only to and suspension of the privilege.133
those judicially charged with rebellion or offenses connected with invasion.129
MR. PADILLA.
Clearly, from the foregoing, while martial law poses the most severe threat to civil
liberties,130 the Constitution has safeguards against the President's prerogative to declare a We all agree with the suspension of the writ or the proclamation of martial law should not
state of martial law. require beforehand the concurrence of the majority of the Members of the Congress.
However, as provided by the Committee, the Congress may revoke, amend, or shorten or
c) "Graduation" of powers refers to hierarchy based on scope and effect; it does not refer to a even increase the period of such suspension.134
sequence, order, or arrangement by which the Commander-in-Chief must adhere to.
MR. NATIVIDAD. First and foremost, we agree with the Commissioner's thesis that in the first
Indeed, the 1987 Constitution gives the "President, as Commander-in- Chief, a 'sequence' of imposition of martial law there is no need for concurrence of the Members of Congress
'graduated power[s]'. From the most to the least benign, these are: the calling out power, because the provision says 'in case of actual invasion or rebellion.' If there is actual invasion
the power to suspend the privilege of the writ of habeas corpus, and the power to declare and rebellion, as Commissioner Crispino de Castro said, there is a need for immediate
martial law."131 It must be stressed, however, that the graduation refers only to hierarchy response because there is an attack. Second, the fact of securing a concurrence may be
based on scope and effect. It does not in any manner refer to a sequence, arrangement, or impractical because the roads might be blocked or barricaded. x x x So the requirement of an
order which the Commander-in-Chief must follow. This so-called "graduation of powers" initial concurrence of the majority of all Members of the Congress in case of an invasion or
does not dictate or restrict the manner by which the President decides which power to rebellion might be impractical as I can see it.
choose.
277

Second, Section 15 states that the Congress may revoke the declaration or lift the factual basis because the paragraph being on line 9 precisely tells us that the Supreme court
suspension. may review, in an appropriate proceeding filed by any citizen, the sufficiency of the factual
basis of the proclamation of martial law or the suspension of the privilege of the writ or the
And third, the matter of declaring martial law is already a justiciable question and no longer a extension thereof and must promulgate its decision on the same within 30 days from its
political one in that it is subject to judicial review at any point in time. So on that basis, I filing.
agree that there is no need for concurrence as a prerequisite to declare martial law or to
suspend the privilege of the writ of habeas corpus. x x x135 I believe that there are enough safeguards. The Constitution is supposed to balance the
interests of the country. And here we are trying to balance the public interest in case of
MR. SUAREZ. Thank you. invasion or rebellion as against the rights of citizens. x x x

The Commissioner is suggesting that in connection with Section 15, we delete the phrase MR. SUAREZ. Will that prevent a future President from doing what Mr. Marcos had done?
'and, with the concurrence of at least a majority of all the Members of the Congress...'
MR. MONSOD. There is nothing absolute in this world, and there may be another Marcos.
MR. PADILLA. That is correct especially for the initial suspension of the privilege of the writ What we are looking for are safeguards that arereasonable and, I believe, adequate at this
of habeas corpus or also the declaration of martial law. point. On the other hand, in case of invasion or rebellion, even during the first 60 days when
the intention here is to protect the country in that situation, it would be unreasonable to ask
that there should be a concurrence on the part of the Congress, which situation is
MR. SUAREZ. So in both instances, the Commissioner is suggesting that this would be an
automatically terminated at the end of such 60 days.
exclusive prerogative of the President?

MR. SUAREZ. Would the Gentleman not feel more comfortable if we provide for a legislative
MR. PADILLA. At least initially, for a period of 60 days. But even that period of 60 days may
check on this awesome power of the Chief Executive acting as Commander-in-Chief?
be shortened by the Congress or the Senate because the next sentence says that the
Congress or the Senate may even revoke the proclamation.136
MR. MONSOD. I would be less comfortable if we have a presidency that cannot act under
those conditions.
MR. SUAREZ. x x x

MR. SUAREZ. But he can act with the concurrence of the proper or appropriate authority?
The Commissioner is proposing a very substantial amendment because this means that he is
vesting exclusively unto the President the right to determine the factors which may lead to
the declaration of martial law and the suspension of the writ of habeas corpus. I suppose he MR. MONSOD. Yes. But when those situations arise, it is very unlikely that the concurrence of
has strong and compelling reasons in seeking to delete this particular phrase. May we be Congress would be available; and, secondly, the President will be able to act quickly in order
informed of his good and substantial reasons? to deal with the circumstances.

MR. MONSOD. This situation arises in cases of invasion or rebellion. And in previous MR. SUAREZ. So, we would be subordinating actual circumstances to expediency?
interpellations regarding this phrase, even during the discussions on the Bill of Rights, as I
understand it, the interpretation is a situation of actual invasion or rebellion. In these MR. MONSOD. I do not believe it is expediency when one is trying to protect the country in
situations, the President has to act quickly. Secondly, this declaration has a time fuse. It is the event of an invasion or a rebellion.137
only good for a maximum of 60 days. At the end of 60 days, it automatically terminates.
Thirdly, the right of the judiciary to inquire into the sufficiency of the factual basis of the The foregoing exchange clearly manifests the intent of the Constitution not to allow Congress
proclamation always exists, even during those first 60 days. to interfere a priori in the President's choice of extraordinary powers.

MR. MONSOD. Yes, Madam President, in the case of Mr. Marcos[,] he is undoubtedly an e) The Court must similarly and necessarily refrain from calibrating the President's decision of
aberration in our history and national consciousness. But given the possibility that there which among his extraordinary powers to avail given a certain situation or condition.
would be another Marcos, our Constitution now has sufficient safeguards. As I said, it is not
really true, as the Gentleman mentioned, that there is an exclusive right to determine the
278

It cannot be overemphasized that time is paramount in situations necessitating the V. Whether or not Proclamation No. 216 may be considered vague and thus void because of
proclamation of martial law or suspension of the privilege of the writ of habeas corpus. It was (a) its inclusion of "other rebel groups"; and (b) the absence of any guideline specifying its
precisely this time element that prompted the Constitutional Commission to eliminate the actual operational parameters within the entire Mindanao region.
requirement of 1 concurrence of the Congress in the initial imposition by the President of
martial law or suspension of the privilege of the writ of habeas corpus. Proclamation No. 216 is being facially challenged on the ground of "vagueness" by the
insertion of the phrase "other rebel groups"139 in its Whereas Clause and for lack of available
Considering that the proclamation of martial law or suspension of the privilege of the writ guidelines specifying its actual operational parameters within the entire Mindanao region,
of habeas corpus is now anchored on actual invasion or rebellion and when public safety making the proclamation susceptible to broad interpretation, misinterpretation, or
requires it, and is no longer under threat or in imminent danger thereof, there is a necessity confusion.
and urgency for the President to act quickly to protect the country.138The Court, as Congress
does, must thus accord the President the same leeway by not wading into the realm that is This argument lacks legal basis.
reserved exclusively by the Constitution to the Executive Department.
a) Void-for-vagueness doctrine.
j) The recommendation of the Defense Secretary is not a condition for the declaration of
martial law or suspension of the privilege of the writ of habeas corpus.
The void-for-vagueness doctrine holds that a law is facially invalid if "men of common
intelligence must necessarily guess at its meaning and differ as to its application."140 "[A]
Even the recommendation of, or consultation with, the Secretary of National Defense, or statute or act may be said to be vague when it lacks comprehensible standards that men of
other high-ranking military officials, is not a condition for the President to declare martial common intelligence must necessarily guess at its meaning and differ in its application. [In
law. A plain reading of Section 18, Article VII of the Constitution shows that the President's such instance, the statute] is repugnant to the Constitution in two respects: (1) it violates due
power to declare martial law is not subject to any condition except for the requirements of process for failure to accord persons, especially the parties targeted by it, fair notice of the
actual invasion or rebellion and that public safety requires it. Besides, it would be contrary to conduct to avoid; and (2) it leaves law enforcers unbridled discretion in carrying out its
common sense if the decision of the President is made dependent on the recommendation provisions and becomes an arbitrary flexing of the Government muscle."141
of his mere alter ego. Rightly so, it is only on the President and no other that the exercise of
the powers of the Commander-in-Chief under Section 18, Article VII of the Constitution is
b) Vagueness doctrine applies only in free speech cases.
bestowed.

The vagueness doctrine is an analytical tool developed for testing "on their faces" statutes in
g) In any event, the President initially employed the most benign action - the calling out
free speech cases or, as they are called in American law, First Amendment cases.142 A facial
power - before he declared martial law and suspended the privilege of the writ of habeas
challenge is allowed to be made to a vague statute and also to one which is overbroad
corpus.
because of possible "'chilling effect' on protected speech that comes from statutes violating
free speech. A person who does not know whether his speech constitutes a crime under an
At this juncture, it must be stressed that prior to Proclamation No. 216 or the declaration of overbroad or vague law may simply restrain himself from speaking in order to avoid being
martial law on May 23, 201 7, the President had already issued Proclamation No. 55 on charged of a crime. The overbroad or vague law thus chills him into silence."143
September 4, 2016, declaring a state of national emergency on account of lawless violence in
Mindanao. This, in fact, is extant in the first Whereas Clause of Proclamation No. 216. Based
It is best to stress that the vagueness doctrine has a special application only to free-speech
on the foregoing presidential actions, it can be gleaned that although there is no obligation
cases. They are not appropriate for testing the validity of penal statutes.144 Justice Mendoza
or requirement on his part to use his extraordinary powers on a graduated or sequential
explained the reason as follows:
basis, still the President made the conscious anddeliberate effort to first employ the most
benign from among his extraordinary powers. As the initial and preliminary step towards
suppressing and preventing the armed hostilities in Mindanao, the President decided to use A facial challenge is allowed to be made to a vague statute and to one which is overbroad
his calling out power first. Unfortunately, the situation did not improve; on the contrary, it because of possible 'chilling effect' upon protected speech. The theory is that ' [w]hen
only worsened. Thus, exercising his sole and exclusive prerogative, the President decided to statutes regulate or proscribe speech and no readily apparent construction suggests itself as
impose martial law and suspend the privilege of the writ of habeas corpus on the belief that a vehicle for rehabilitating the statutes in a single prosecution, the transcendent value to all
the armed hostilities in Mindanao already amount to actual rebellion and public safety society of constitutionally protected expression is deemed to justify allowing attacks on
requires it. overly broad statutes with no requirement that the person making the attack demonstrate
that his own conduct could not be regulated by a statute drawn with narrow specificity.' The
279

possible harm to society in permitting some unprotected speech to go unpunished is The contention that the phrase "other rebel groups" leaves Proclamation No. 216 open to
outweighed by the possibility that the protected speech of others may be deterred and broad interpretation, misinterpretation, and confusion, cannot be sustained.
perceived grievances left to fester because of possible inhibitory effects of overly broad
statutes. In People v. Nazario,150 the Court enunciated that:

This rationale does not apply to penal statutes. Criminal statutes have general in As a rule, a statute or act may be said to be vague when it lacks comprehensible standards
terrorem effect resulting from their very existence, and, if facial challenge is allowed for this that men 'of common intelligence must necessarily guess at its meaning and differ as to its
reason alone, the State may well be prevented from enacting laws against socially harmful application.' It is repugnant to the Constitution in two respects: (1) it violates due process for
conduct. In the area of criminal law, the law cannot take chances as in the area of free failure to accord persons, especially the parties targetted by it, fair notice of the conduct to
speech. avoid; and (2) it leaves law enforcers unbridled discretion in carrying out its provisions and
becomes an arbitrary flexing of the Government muscle.
In sum, the doctrines of strict scrutiny, overbreadth, and vagueness are analytical tools
developed for testing 'on their faces' statutes in free speech cases or, as they are called in But the act must be utterly vague on its face, that is to say, it cannot be clarified by either a
American law, First Amendment cases. They cannot be made to do service when what is saving clause or by construction. Thus, in Coates v. City of Cincinnati, the U.S. Supreme Court
involved is a criminal statute. With respect to such statute, the established rule is that'one to struck down an ordinance that had made it illegal for 'three or more persons to assemble on
whom application of a statute is constitutional will not be heard to attack the statute on the any sidewalk and there conduct themselves in a manner annoying to persons passing by.'
ground that impliedly it might also be taken as applying to other persons or other situations Clearly, the ordinance imposed no standard at all 'because one may never know in advance
in which its application might be unconstitutional.' As has been pointed out, 'vagueness what annoys some people but does not annoy others.'
challenges in the First Amendment context, like overbreadth challenges typically produce
facial invalidation, while statutes found vague as a matter of due process typically are
Coates highlights what has been referred to as a 'perfectly vague' act whose obscurity is
invalidated [only] 'as applied' to a particular defendant.' x x x145
evident on its face. It is to be distinguished, however, from legislation couched in imprecise
language - but which nonetheless specifies a standard though defectively phrased - in which
Invalidation of statutes "on its face" should be used sparingly because it results in striking case, it may be 'saved' by proper construction.151
down statutes entirely on the ground that they might beapplied to parties not before the
Court whose activities are constitutionally protected.146 "Such invalidation would constitute a
The term "other rebel groups" in Proclamation No. 216 is not at all vague when viewed in the
departure from the usual requirement of 'actual case and controversy' and permit decisions
context of the words that accompany it. Verily, the text of Proclamation No. 216 refers to
to be made in a sterile abstract context having no factual concreteness."147
"other rebel groups" found in Proclamation No. 55, which it cited by way of reference in its
Whereas clauses.
c) Proclamation No. 216 cannot be facially challenged using the vagueness doctrine.
e) Lack of guidelines/ operational parameters does not make Proclamation No. 216 vague.
Clearly, facial review of Proclamation No. 216 on the grounds of vagueness is unwarranted.
Proclamation No. 216 does not regulate speech, religious freedom, and other fundamental
Neither could Proclamation No. 216 be described as vague, and thus void, on the ground that
rights that may be facially challenged.148 What it seeks to penalize is conduct, not speech.
it has no guidelines specifying its actual operational parameters within the entire Mindanao
region. Besides, operational guidelines will serve only as mere tools for the implementation
As held by the Court in David v. President Macapagal-Arroyo,149 the facial review of of the proclamation. In Part III, we declared that judicial review covers only the sufficiency of
Proclamation No. 1017, issued by then President Gloria Macapagal-Arroyo declaring a state information or data available to or known to the President prior to, or at the time of, the
of national emergency, on ground o vagueness is uncalled for since a plain reading of declaration or suspension. And, as will be discussed exhaustively in Part VII, the review will
Proclamation No. 10171 shows that it is not primarily directed at speech or even speech- be confined to the proclamation itself and the Report submitted to Congress.
related1 conduct. It is actually a call upon the Armed Forces of the Philippines (AFP) to
prevent or suppress all forms of lawless violence. Like Proclamation No. 1017, Proclamation
Clearly, therefore, there is no need for the Court to determine the constitutionality of the
No. 216 pertains to a spectrum of conduct, not free speech, which is manifestly subject to
implementing and/or operational guidelines, general orders, arrest orders and other orders
state regulation.
issued after the proclamation for being irrelevant to its review. Thus, any act committed
under the said orders in violation of the Constitution and the laws, such as criminal acts or
d) Inclusion of "other rebel groups " does not make Proclamation No.216 vague. human rights violations, should be resolved in a separate proceeding. Finally, there is a risk
280

that if the Court wades into these areas, it would be deemed as trespassing into the sphere In Zamora, the Court categorically ruled that the Integrated Bar of the ' Philippines had failed
that is reserved exclusively for Congress in the exercise of its power to revoke. to sufficiently comply with the requisites of locus standi, as it was not able to show any
specific injury which it had suffered or could suffer by virtue of President Joseph Estrada's
VI. Whether or not nullifying Proclamation No.216 will (a) have the effect of recalling order deploying the Philippine Marines to join the PNP in visibility patrols around the
Proclamation No. 55; or (b) also nullify the acts of the President in calling out the armed metropolis.156
forces to quell lawless violence in Marawi and other parts of the Mindanao region.
This locus standi requirement, however, need not be complied with in so far as the Court's
a) The calling out power is in a different category from the power to declare martial law and jurisdiction to review the sufficiency of the factual basis of the President's declaration of
the power to suspend the privilege of the writ of habeas corpus; nullification of Proclamation martial law or suspension of the privilege ofthe writ of habeas corpus is concerned. In fact, by
No. 216 will not affect Proclamation No. 55. constitutional design, such review may be instituted by any citizen before the
Court,157 without the need to prove that he or she stands to sustain a direct and personal
injury as a consequence of the questioned Presidential act/s.
The Court's ruling in these cases will not, in any way, affect the President's declaration of a
state of national emergency on account of lawless violence in Mindanao through
Proclamation No. 55 dated September 4, 2016, where he called upon the Armed Forces and But, even assuming arguendo that the Court finds no sufficient basis for the declaration of
the Philippine National 1 Police (PNP) to undertake such measures to suppress any and all martial law in this case, such ruling could not affect the President's exercise of his calling out
forms of lawless violence in the Mindanao region, and to prevent such lawless violence from power through Proclamation No. 55.
spreading and escalating elsewhere in the Philippines.
b) The operative fact doctrine.
In Kulayan v.Tan,152 the Court ruled that the President's calling out power is in a different
category from the power to suspend the privilege of the writ of habeas corpus and the power Neither would the nullification of Proclamation No. 216 result in the nullification of the acts
to declare martial law: of the President done pursuant thereto. Under the "operative fact doctrine," the
unconstitutional statute is recognized as an "operative fact" before it is declared
x x x Congress may revoke such proclamation or suspension and the Court may review the unconstitutional.158
sufficiency of the factual basis thereof. However, there is no such equivalent provision
dealing with the revocation or review of the President's action to call out the armed forces. Where the assailed legislative or executive act is found by the judiciary to be contrary to the
The distinction places the calling out power in a different category from the power to declare Constitution, it is null and void. As the new Civil Code puts it: 'When the courts declare a law
martial law and the power to suspend the privilege of the writ of habeas corpus, otherwise, to be inconsistent with the Constitution, the former shall be void and the latter shall govern.
the framers of the Constitution would have simply lumped together the three powers and Administrative or executive acts, orders and regulations shall be valid only when they are not
provided for their revocation and review without any qualification. 153 contrary to the laws or the Constitution.' The above provision of the Civil Code reflects the
orthodox view that an unconstitutional act, whether legislative or executive, is not a law,
In other words, the President may exercise the power to call out the Armed Forces confers no rights, imposes no duties, and affords no protection. This doctrine admits of
independently of the power to suspend the privilege of the writ of habeas corpus and to qualifications, however. As the American Supreme Court stated: 'The actual existence of a
declare martial law, although, of course, it may also be a prelude to a possible future exercise statute prior to such a determination [of constitutionality], is an operative fact and may have
of the latter powers, as in this case. consequences which cannot always be erased by a new judicial declaration. The effect of the
subsequent ruling as to the invalidity may have to be considered in various aspects, - with
respect to particular regulations, individual and corporate, and particular conduct, private
Even so, the Court's review of the President's declaration of martial law and his calling out
and official.
the Armed Forces necessarily entails separate proceedings instituted for that particular
purpose.
The orthodox view finds support in the well-settled doctrine that the Constitution is supreme
and provides the measure for the validity of legislative or executive acts. Clearly then, neither
As explained in Integrated Bar of the Philippines v. Zamora,154 the President's exercise of his
the legislative nor the executive branch, and for that matter much less, this Court, has power
power to call out the armed forces to prevent or suppress lawless violence, invasion or
under the Constitution to act contrary to its terms. Any attempted exercise of power in
rebellion may only be examined by the Court as to whether such power was exercised within
violation of its provisions is to that extent unwarranted and null.
permissible constitutional limits or in a manner constituting grave abuse of discretion.155
281

The growing awareness of the role of the judiciary as the governmental organ which has the b) The "sufficiency of factual basis test".
final say on whether or not a legislative or executive measure is valid leads to a more
appreciative attitude of theemerging concept that a declaration of nullity may have legal Similarly, under the doctrine of contemporaneous construction, the framers of the 1987
consequences which the more orthodox view would deny. That for a period of time such a Constitution are presumed to know the prevailing jurisprudence at the time they were
statute, treaty, executive order, or ordinance was in 'actual existence' appears to be drafting the Constitution. Thus, the phrase "sufficiency of factual basis" in Section 18, Article
indisputable. What is more appropriate and logical then than to consider it as 'an operative VII of the Constitution should be understood as the only test for judicial review of the
fact?' (Emphasis supplied)159 President's power to declare martial law and suspend the privilege of the writ of habeas
corpus under Section 18, Article VII of the Constitution. The Court does not need to satisfy
However, it must also be stressed that this "operative fact doctrine" is not a fool-proof shield itself that the President's decision is correct, rather it only needs to determine whether the
that would repulse any challenge to acts performed during the effectivity of martial law or President's decision had sufficient factual bases.
suspension of the privilege of the writ of habeas corpus, purportedly in furtherance of
quelling rebellion or invasion, and promotion of public safety, when evidence shows We conclude, therefore, that Section 18, Article VII limits the scope of judicial review by the
otherwise. introduction of the "sufficiency of the factual basis" test.

VII. The Scope of the Power to Review. As Commander-in-Chief, the President has the sole discretion to declare martial law and/or
to suspend the privilege of the writ of habeas corpus, subject to the revocation of Congress
a) The scope of the power of review under the 1987 Constitution refers only to the and the review of this Court. Since the exercise of these powers is a judgment call of the
determination of the sufficiency of the factual basis of the declaration of martial law and President, the determination of this Court as to whether there is sufficient factual basis for
suspension of the privilege of habeas corpus. the exercise of such, must be based only on facts or information known by or available to the
President at the time he made the declaration or suspension, which facts or information are
To recall, the Court, in the case of In the Matter of the Petition for Habeas Corpus of found in the proclamation as well as the written Report submitted by him to Congress. These
Lansang,160 which was decided under the 1935 Constitution,161 held that it can inquire into, may be based on the situation existing at the time the declaration was made or past events.
within proper bounds, whether there has been adherence to or compliance with the As to how far the past events should be from the present depends on the President.
constitutionally-imposed limitations on the Presidential power to suspend the privilege of the
writ of habeas corpus.162 "Lansang limited the review function of the Court to a very Past events may be considered as justifications for the declaration and/or suspension as long
prudentially narrow test of arbitrariness."163 Fr. Bernas described the "proper bounds" as these are connected or related to the current situation existing at the time of the
in Lansang as follows: declaration.

What, however, are these 'proper bounds' on the power of the courts? The Court first gave As to what facts must be stated in the proclamation and the written Report is up to the
the general answer that its power was 'merely to check - not to supplant - the Executive, or President.165 As Commander-in-Chief, he has sole discretion to determine what to include
to ascertain merely whether he has gone beyond the constitutional limits of his and what not to include in the proclamation and the written Report taking into account the
jurisdiction, not to exercise the power vested in him or to determine the wisdom of his act. urgency of the situation as well as national security. He cannot be forced to divulge
More specifically, the Court said that its power was not 'even comparable with its power over intelligence reports and confidential information that may prejudice the operations and the
civil or criminal cases elevated thereto by appeal...in which cases the appellate court has all safety of the military.
the powers of the courtof origin,' nor to its power of quasi-judicial administrative decisions
where the Court is limited to asking whether 'there is some evidentiary basis' for the Similarly, events that happened after the issuance of the proclamation, which are included in
administrative finding. Instead, the Court accepted the Solicitor General's suggestion that the written report, cannot be considered in determining the sufficiency of the factual basis of
it 'go no further than to satisfy [itself] not that the President's decision is correct and that the declaration of martial law and/or the suspension of the privilege of the writ of habeas
public safety was endangered by the rebellion and justified the suspension of the writ, but corpus since these happened after the President had already issued the proclamation. If at
that in suspending the writ, the President did not act arbitrarily.'164 all, they may be used only as tools, guides or reference in the Court's determination of the
sufficiency of factual basis, but not as part or component of the portfolio of the factual basis
Lansang, however, was decided under the 1935 Constitution. The 1987 Constitution, by itself.
providing only for judicial review based on the determination of the sufficiency of the factual
bases, has in fact done away with the test of arbitrariness as provided in Lansang.
282

In determining the sufficiency of the factual basis of the declaration and/or the suspension, proclamation is made; and, this Court may investigate the factual background of the
the Court should look into the full complement or totality of the factual basis, and not declaration.169
piecemeal or individually. Neither should the Court expect absolute correctness of the facts
stated in the proclamation and in the written Report as the President could not be expected Hence, the maxim falsus in uno, falsus in omnibus finds no application in this case. Falsities of
to verify the accuracy and veracity of all facts reported to him due to the urgency of the and/or inaccuracies in some of the facts stated in the proclamation and the written report
situation. To require precision in the President's appreciation of facts would unduly burden are not enough reasons for the Court to invalidate the declaration and/or suspension as long
him and therefore impede the process of his decision-making. Such a requirement will as there are other facts in the proclamation and the written Report that support the
practically necessitate the President to be on the ground to confirm the correctness of the conclusion that there is an actual invasion or rebellion and that public safety requires the
reports submitted to him within a period that only the circumstances obtaining would be declaration and/or suspension.
able to dictate. Such a scenario, of course, would not only place the President in peril but
would also defeat the very purpose of the grant of emergency powers upon him, that is, to
In sum, the Court's power to review is limited to the determination of whether the President
borrow the words of Justice Antonio T. Carpio in Fortun, to "immediately put an end to the
in declaring martial law and suspending the privilege of the writ of habeas corpus had
root cause of the emergency".166 Possibly, by the time the President is satisfied with the
sufficient factual basis. Thus, our review would be limited to an examination on whether the
correctness of the facts in his possession, it would be too late in the day as the invasion or
President acted within the bounds set by the Constitution, i.e., whether the facts in his
rebellion could have already escalated to a level that is hard, if not impossible, to curtail.
possession prior to and at the time of the declaration or suspension are sufficient for him to
declare martial law or suspend the privilege of the writ of habeas corpus.
Besides, the framers of the 1987 Constitution considered intelligence reports of military
officers as credible evidence that the President ca appraise and to which he can anchor his
VIII. The parameters for determining the sufficiency of the/actual basis/or the declaration of
judgment,167 as appears to be the case here.
martial law and/or the suspension of the privilege of the writ of habeas corpus.

At this point, it is wise to quote the pertinent portions of the Dissenting Opinion of Justice
a) Actual invasion or rebellion, and public safety requirement.
Presbitero J. Velasco Jr. in Fortun:

Section 18, Article VII itself sets the parameters for determining the sufficiency of the factual
President Arroyo cannot be blamed for relying upon the information given to her by the
basis for the declaration of martial law and/or the suspension of the privilege of the writ
Armed Forces of the Philippines and the Philippine National Police, considering that the
of habeas corpus, "namely (1) actual invasion or rebellion, and (2) public safety requires the
matter of the supposed armed uprising was within their realm of competence, and that a
exercise of such power."170 Without the concurrence of the two conditions, the President's
state of emergency has also been declared in Central Mindanao to prevent lawless violence
declaration of martial law and/or suspension of the privilege of the writ of habeas
similar to the 'Maguindanao massacre,' which may be an indication that there is a threat to
corpus must be struck down.
the public safety warranting a declaration of martial law or suspension of the writ.

As a general rule, a word used in a statute which has a technical or legal meaning, is
Certainly, the President cannot be expected to risk being too late before declaring martial
construed to have the same technical or legal meaning.171 Since the Constitution did not
law or suspending the writ of habeas corpus. The Constitution, as couched, does not require
define the term "rebellion," it must be understood to have the same meaning as the crime of
precision in establishing the fact of rebellion. The President is called to act as public safety
"rebellion" in the Revised Penal Code (RPC).172
requires.168

During the July 29, 1986 deliberation of the Constitutional Commission of 1986, then
Corollary, as the President is expected to decide quickly on whether there is a need to
Commissioner Florenz D. Regalado alluded to actual rebellion as one defined under Article
proclaim martial law even only on the basis of intelligence reports, it is irrelevant, for
134 of the RPC:
purposes of the Court's review, if subsequent events prove that the situation had not been
accurately reported to him.
MR. DE LOS REYES. As I see it now, the Committee envisions actual rebellion and no longer
imminent rebellion. Does the Committee mean that there should be actual shooting or actual
After all, the Court's review is confined to the sufficiency, not accuracy, of the information at
attack on the legislature or Malacañang, for example? Let us take for example a
hand during the declaration or suspension; subsequent events do not have any bearing
contemporary event - this Manila Hotel incident, everybody knows what happened. Would
insofar as the Court's review is concerned. In any event, safeguards under Section 18, Article
the Committee consider that an actual act of rebellion?
VII of the Constitution are in place to cover such a situation, e.g., the martial law period is
good only for 60 days; Congress may choose to revoke it even immediately after the
283

MR. REGALADO. If we consider the definition of rebellion under Articles 134 and 135 of the To summarize, the parameters for determining the sufficiency of factual basis are as follows:
Revised Penal Code, that presupposes an actual assemblage of men in an armed public l) actual rebellion or invasion; 2) public safety requires it; the first two requirements must
uprising for the purposes mentioned in Article 134 and by the means employed under Article concur; and 3) there is probable cause for the President to believe that there is actual
135. x x x173 rebellion or invasion.

Thus, rebellion as mentioned in the Constitution could only refer to rebellion as defined Having laid down the parameters for review, the Court shall nowproceed to the core of the
under Article 134 of the RPC. To give it a different definition would not only create confusion controversy - whether Proclamation No. 216,Declaring a State of Martial Law and Suspending
but would also give the President wide latitude of discretion, which may be abused - a the Privilege of the Writ of Habeas Corpus in the whole of Mindanao, lacks sufficient factual
situation that the Constitution see k s to prevent.174 basis.

Article 134 of the RPC states: IX. There is sufficient factual basis for the declaration of martial law and the suspension of
the writ of habeas corpus.
Art. 134. Rebellion or insurrection; How committed. - The crime of rebellion or insurrection is
committed by rising publicly and taking arms against the Government for the purpose of At this juncture, it bears to emphasize that the purpose of judicial review is not the
removing from the allegiance to said Government or its laws, the territory of the Philippine determination of accuracy or veracity of the facts upon which the President anchored his
Islands or any part thereof, of any body of land, naval or other armed forces, depriving the declaration of martial law or suspension of the privilege of the writ of habeas corpus; rather,
Chief Executive or the Legislature, wholly or partially, of any of their powers or prerogatives. only the sufficiency of the factual basis as to convince the President that there is probable
cause that rebellion exists. It must also be reiterated that martial law is a matter ofurgency
Thus, for rebellion to exist, the following elements must be present, to wit: "(l) there is a (a) and much leeway and flexibility should be accorded the President. As such, he is not
public uprising and (b) taking arms against the Government; and (2) the purpose of the expected to completely validate all the information he received before declaring martial law
uprising or movement is either (a) to remove from the allegiance to the Government or its or suspending the privilege of the writ of habeas corpus.
laws: (i) the territory of the Philippines or any part thereof; or (ii) any body of land, naval, or
other armed forces; or (b) to deprive the Chief Executive or Congress, wholly or partially, of We restate the elements of rebellion for reference:
any of their powers and prerogatives."175
1. That there be (a) public uprising, and (b) taking up arms against the Government; and
b) Probable cause is the allowable standard of proof for the President.
2. That the purpose of the uprising or movement is either: (a) to remove from the allegiance
In determining the existence of rebellion, the President only needs to convince himself that to said Government or its laws the territory of the Philippines or any part thereof, or any
there is probable cause or evidence showing that more likely than not a rebellion was body of land, naval or other armed forces or (b) to deprive the Chief Executive or Congress,
committed or is being committed.176 To require him to satisfy a higher standard of proof wholly or partially, of any of their powers or prerogatives.178
would restrict the exercise of his emergency powers. Along this line, Justice Carpio, in his
Dissent in Fortun v. President Macapagal-Arroyo, concluded that the President needs only to Petitioners concede that there is an armed public uprising in Marawi City.179 However, they
satisfy probable cause as the standard of proof in determining the existence of either insist that the armed hostilities do not constitute rebellion in the absence of the element of
invasion or rebellion for purposes of declaring martial law, and that probable cause is the culpable political purpose, i.e., the removal from the allegiance to the Philippine Government
most reasonable, most practical and most expedient standard by which the President can or its laws: (i) the territory of the Philippines or any part thereof; or (ii) any body of land,
fully ascertain the existence or non-existence of rebellion necessary for a declaration of naval, or other armed forces; or (b) to deprive the Chief Executive or Congress, wholly or
martial law or suspension of the writ. This is because unlike other standards of proof, which, partially, of any of their powers and prerogatives.
in order to be met, would require much from the President and therefore unduly restrain his
exercise of emergency powers, the requirement of probable cause is much simpler. It merely
The contention lacks merit.
necessitates an "average man [to weigh] the facts and circumstances without resorting to the
calibration of the rules of evidence of which he has no technical knowledge. He [merely]
relies on common sense [and] x x x needs only to rest on evidence showing that, more likely a) Facts, events and information upon which the President anchored his decision to declare
than not, a crime has been committed x x x by the accused." 177 martial law and suspend the privilege of the writ of habeas corpus.
284

Since the President supposedly signed Proclamation No. 216 on May 23, 2017 at 10:00 6. Sulu bombings;190
PM,180 the Court will consider only those facts and/or events which were known to or have
transpired on or before that time, consistent with the scope of judicial review. Thus, the 7. Basilan bombings;191
following facts and/or events were deemed to have been considered by the President in
issuing Proclamation No. 216, as plucked from and extant in Proclamation No. 216 itself:
8. Attempt to capture Hapilon was confronted with armed resistance by combined forces of
ASG and the Maute Group;192
1. Proclamation No. 55 issued on September 4, 2016, declaring a state of national emergency
on account of lawless violence in Mindanao;181
9. Escalation of armed hostility against the government troops;193

2. Series of violent acts182 committed by the Maute terrorist group including:


10. Acts of violence directed not only against government authorities and establishments but
civilians as well;194
a) Attack on the military outpost in Butig, Lanao del Sur m February 2016, killing
and wounding several soldiers;
11. Takeover of major social, economic and political foundations which paralyzed Marawi
City;195
b) Mass jailbreak in Marawi City in August 2016 of the arrested comrades of the
Maute Group and other detainees;
12. The object of the armed hostilities was to lay the groundwork for the establishment of a
DAESH/ISIS wilayat or province;196
3. On May 23, 2017:183
13. Maute Group has 263 active members, armed and combat-ready;197
a) Takeover of a hospital in Marawi;
14. Extensive networks or linkages of the Maute Group with foreign and local armed
b) Establishment of several checkpoints within Marawi; groups;198

c) Burning of certain government and private facilities; 15. Adherence of the Maute Group to the ideals espoused by ISIS;199

d) Mounting casualties on the part of the government; 16. Publication of a video showing Maute Group's declaration of allegiance to ISIS;200

e) Hoisting the flag of ISIS in several areas; and 17. Foreign-based terrorist groups provide financial and logistical support to the Maute
Group;201
f) Capability of the Maute Group and other rebel groups to sow terror, and cause death and
damage to property not only in Lanao del Sur but also in other parts of Mindanao; and the 18. Events on May 23, 2017 in Marawi City, particularly:
Report184 submitted to Congress:
a) at 2:00 PM, members and sympathizers of the Maute Group and ASG attacked various
1. Zamboanga siege;185 government and privately-owned facilities;202

2. Davao bombing;186 b) at 4:00 PM, around fifty (50) armed criminals forcibly entered the Marawi City Jail;
facilitated the escape of inmates; killed a member of PDEA; assaulted and disarmed on-duty
3. Mamasapano carnage;187 personnel and/or locked them inside the cells; confiscated cellphones, personnel-issued
firearms, and vehicles;203
4. Cotabato bombings;188
c) by 4:30 PM, intem1ption of power supply; sporadic gunfights; city-wide power outage by
5. Sultan Kudarat bombings;189 evening;204
285

d) from 6:00 PM to 7:00 PM, Maute Group ambushed and burned the Marawi Police Station; After the assessment by the President of the aforementioned facts, he arrived at the
commandeered a police car;205 following conclusions, as mentioned in Proclamation No. 216 and the Report:

e) BJMP personnel evacuated the Marawi City Jail and other affected areas;206 1) The Maute Group is "openly attempting to remove from the allegiance to the Philippine
Government this part of Mindanao and deprive the Chief Executive of his powers and
f) control over three bridges in Lanao del Sur, namely, Lilod, Bangulo, and Sauiaran, was prerogatives to enforce the laws of the land and to maintain public order and safety in
taken by the rebels;207 Mindanao, constituting the crime of rebellion."221

g) road blockades and checkpoints set up by lawless armed groups at the Iligan-Marawi 2) "[L]awless armed groups have taken up arms and committed public uprising against the
junction;208 duly constituted government and against the people of Mindanao, for the purpose of
removing Mindanao - starting with the City of Marawi, Lanao del Sur - from its allegiance to
the Government and its laws and depriving the Chief Executive of his powers and
h) burning of Dansalan College Foundation, Cathedral of Maria Auxiliadora, the nuns'
prerogatives to enforce the laws of the land and to maintain public order and safety in
quarters in the church, and the Shia Masjid Moncado Colony;209
Mindanao, to the great damage, prejudice, and detriment of the people therein and the
nation as a whole."222
i) taking of hostages from the church;210
3) The May 23, 2017 events "put on public display the groups' clear intention to establish an
j) killing of five faculty members of Dansalan College foundation;211 Islamic State and their capability to deprive the duly constituted authorities - the President,
foremost - of their powers and prerogatives. "223
k) burning of Senator Ninoy Aquino College Foundation and Marawi Central Elementary Pilot
School;212 4) "These activities constitute not simply a display of force, but a clear attempt to establish
the groups' seat of power in Marawi City for their planned establishment of a
1) overrunning of Amai Pakpak Hospital;213 DAESH wilayat or province covering the entire Mindanao."224

m) hoisting the ISIS flag in several areas;214 5) "The cutting of vital lines for transportation and power; the recruitment of young Muslims
to further expand their ranks and strengthen their force; the armed consolidation of their
n) attacking and burning of the Filipino-Libyan Friendship Hospital;215 members throughout Marawi City; the decimation of a segment of the city population who
resist; and the brazen display of DAESH flags constitute a clear, pronounced, and
unmistakable intent to remove Marawi City, and eventually the rest of Mindanao, from its
o) ransacking of a branch of Landbank of the Philippines and commandeering an armored allegiance to the Government."225
vehicle;216
6) "There exists no doubt that lawless armed groups are attempting to deprive the President
p) reports regarding Maute Group's plan to execute Christians;217 of his power, authority, and prerogatives within Marawi City as a precedent to spreading
their control over the entire Mindanao, in an attempt to undermine his control over
q) preventing Maranaos from leaving their homes;218 executive departments, bureaus, and offices in said area; defeat his mandate to ensure that
all laws are faithfully executed; and remove his supervisory powers over local
r) forcing young Muslims to join their group;219 and governments."226

s) intelligence reports regarding the existence of strategic mass action of lawless armed 7) "Law enforcement and other government agencies now face pronounced difficulty sending
groups in Marawi City, seizing public and private facilities, perpetrating killings of their reports to the Chief Executive due to the city-wide power outages. Personnel from the
government personnel1 , and committing armed uprising against and open defiance of the BJMP have been prevented from performing their functions. Through the attack and
Government.220 occupation of several hospitals, medical services in Marawi City have been adversely
affected. The bridge and road blockades set up by the groups effectively deprive the
government of its ability to deliver basic services to its citizens. Troop reinforcements have
b) The President's Conclusion
286

been hampered, preventing the government from restoring peace and order in the area. President's power to act as to practically tie her hands and disable her from effectively
Movement by both civilians and government personnel to and from the city is likewise protecting the nation against threats to public safety.'
hindered."227
Neither clear and convincing evidence, which is employed in either criminal or civil cases, is
8) "The taking up of arms by lawless armed groups in the area, with support being provided indispensable for a lawful declaration of martial law or suspension of the writ. This amount of
by foreign-based terrorists and illegal drug money, and their blatant acts of defiance which proof likewise unduly restrains the President in exercising her emergency powers, as it
embolden other armed groups in Mindanao, have resulted in the deterioration of public requires proof greater than preponderance of evidence although not beyond reasonable
order and safety in Marawi City; they have likewise compromised the security of the entire doubt.
Island of Mindanao."228
Not even preponderance of evidence, which is the degree of proof necessary in civil cases, is
9) "Considering the network and alliance-building activities among terrorist groups, local demanded for a lawful declaration of martial law.
criminals, and lawless armed men, the siege f Marawi City is a vital cog in attaining their long-
standing goal: absolute control over the entirety of Mindanao. These circumstances demand Weighing the superiority of the evidence on hand, from at least two opposing sides, before
swift and decisive action to ensure the safety and security of the Filipino people and preserve she can act and impose martial law or suspend the writ unreasonably curtails the President's
our national integrity."229 emergency powers.

Thus, the President deduced from the facts available to him that there was an armed public Similarly, substantial evidence constitutes an unnecessary restriction on the President's use
uprising, the culpable purpose of which was to remove from the allegiance to the Philippine of her emergency powers. Substantial evidence is the amount of proof required in
Government a portion of its territory and to deprive the Chief Executive of any of his powers administrative or quasi-judicial cases, or that amount of relevant evidence which a
and prerogatives, leading the President to believe that there was probable cause that the reasonable mind might accept as adequate to justify a conclusion.
crime of rebellion was and is being committed and that public safety requires the imposition
of martial law and suspension of the privilege of the writ of habeas corpus.
I am of the view that probable cause of the existence of either invasion or rebellion suffices
and satisfies the standard of proof for a valid declaration of martial law and suspension of
A review of the aforesaid facts similarly leads the Court to conclude that the President, in the writ.
issuing Proclamation No. 216, had sufficient factual bases tending to show that actual
rebellion exists. The President's conclusion, that there was an armed public uprising, the
Probable cause is the same amount of proof required for the filing of a criminal information
culpable purpose of which was the removal from the allegiance of the Philippine
by the prosecutor and for the issuance of an arrest warrant by a judge. Probable cause has
Government a portion of its territory and the deprivation of the President from performing
been defined as a 'set of facts and circumstances as would lead a reasonably discreet and
his powers and prerogatives, was reached after a tactical consideration of the facts. In fine,
prudent man to believe that the offense charged in the Information or any offense included
the President satisfactorily discharged his burden of proof.
therein has been committed by the person sought to be arrested.'

After all, what the President needs to satisfy is only the standard of probable cause for a valid
In determining probable cause, the average man weighs the facts and circumstances without
declaration of martial law and suspension of the privilege of the writ of habeas corpus. As
resorting to the calibrations of the rules of evidence of which he has no technical knowledge.
Justice Carpio decreed in his Dissent in Fortun:
He relies on common sense. A finding of probable cause needs only to rest on evidence
showing that, more likely than not, a crime has been committed and that it was committed
x x x [T]he Constitution does not compel the President to produce such amount of proof as to by the accused. Probable cause demands more than suspicion; it requires less than evidence
unduly burden and effectively incapacitate her from exercising such powers. that would justify conviction.

Definitely, the President need not gather proof beyond reasonable doubt, which is the Probable cause, basically premised on common sense, is the most reasonable, most practical,
standard of proof required for convicting an accused charged with a criminal offense.x x x and most expedient standard by which the President can fully ascertain the existence or non-
existence of rebellion, necessary for a declaration of martial law x x x230
Proof beyond reasonable doubt is the highest quantum of evidence, and to require the
President to establish the existence of rebellion or invasion with such amount of proof before c) Inaccuracies, simulations, falsities, and hyperboles.
declaring martial law or suspending the writ amounts to an excessive restriction on 'the
287

The allegation in the Lagman Petition that the facts stated in Proclamation No. 216 and the the Maute Terror Groups have not taken
Report are false, inaccurate, simulated, and/or hyperbolic, does not persuade. As mentioned, over any government facility in Marawi
the Court is not concerned about absolute correctness, accuracy, or precision of the facts City.
because to do so would unduly tie the hands of the President in responding to an urgent
situation.
However, the so-called counter-evidence were derived solely from unverified news articles
Specifically, it alleges that the following facts are not true as shown by its counter- on the internet, with neither the authors nor the sources shown to have affirmed the
evidence.231 contents thereof It was not even shown that efforts were made to secure such affirmation
albeit the circumstances proved futile. As the Court has consistently ruled, news articles are
hearsay evidence, twice removed, and are thus without any probative value, unless offered
FACTUAL STATEMENTS COUNTER-EVIDENCE for a purpose other than proving the truth of the matter asserted.237 This pronouncement
applies with equal force to the Cullamat Petition which likewise submitted online news
(1) that the Maute group attacked Amai Statements made by: articles238 as basis for their claim of insufficiency of factual basis.
Pakpak Hospital and hoisted the DAESH flag (a) Dr. Amer Saber, Chief of the Hospital
there, among several locations. As of 0600H of (b) Health Secretary Paulyn Ubial;
24 May 2017, members of the Maute Group (c) PNP Spokesperson Senior Supt. Again, it bears to reiterate that the maxim falsus in uno, falsus in omnibus finds no
were seen guarding the entry gates of the Dionardo Carlos; application in these cases. As long as there are other facts in the proclamation and the
Amai Pakpak Hospital and that they held (d) AFP Public Affairs Office Chief Co. written Report indubitably showing the presence of an actual invasion or rebellion and that
hostage the employees of the Hospital and Edgard Arevalo; and public safety requires the declaration and/or suspension, the finding of sufficiency of factual
took over the PhilHealth office located thereat (e) Marawi City Mayor Majul Gandamra basis, stands.
(Proclamation No. 216 and Report); denying that the hospital was attacked by
the Maute Group citing online news articles d) Ruling in Bedol v. Commission on Elections not Applicable.
of Philstar, Sunstar, Inquirer, and Bombo
Radyo.232 Petitioners, however, insist that in Bedol v. Commission on Elections,239 news reports may be
admitted on grounds of relevance, trustworthiness, and necessity. Petitioners' reliance on
2. that the Maute Group ambushed and Statements made by PNP Director General
this case is misplaced. The Court in Bedol made it clear that the doctrine of independent
burned the Marawi Police Station Ronald dela Rosa and Marawi City Mayor
relevant statement, which is an ·exception to the hearsay rule, applies in cases "where only
(Proclamation No. 216 and the Report); Majul Gandamra in the online news reports
the fact that such statements were made is relevant, and the truth or falsity thereof is
of ABS-CBN News and CNN
immaterial."240 Here, the question is not whether such statements were made by Saber, et.
Philippines233denying that the Maute group
al., but rather whether what they said are true. Thus, contrary to the view of petitioners, the
occupied the Marawi Police Station.
exception in Bedol finds no application here.
3. that lawless armed groups likewise Statement made by the bank officials in the
ransacked the Landbank of the Philippines and on-line news article of Philstar234 that the e) There are other independent facts which support the finding that, more likely than not,
commandeered one of its armored vehicles Marawi City branch was not ransacked but rebellion exists and that public safety requires it.
(Report); sustained damages from the attacks.

4. that the Marawi Central Elementary Pilot Statements in the on-line news article of Moreover, the alleged false and/or inaccurate statements are just pieces and parcels of the
School was burned (Proclamation No. 216 and Philstar235 made by the Marawi City Schools Report; along with these alleged false data is an arsenal of other independent facts showing
the Report); Division Assistant Superintendent Ana that more likely than not, actua1 rebellion exists, and public safety requires the declaration
Alonto denying that the school was burned of martial law or suspension of the privilege of the writ of habeas corpus. To be precise, the
and Department of Education Assistant alleged false and/or inaccurate statements are only five out of the severa1 statements
Secretary Tonisito Umali stating that they bulleted in the President's Report. Notably, in the interpellation by Justice Francis H.
have not received any report of damage. Jardeleza during the second day of the oral argument, petitioner Lagman admitted that he
was not aware or that he had no personal knowledge of the other incidents cited.241 As it
5. that the Maute Group attacked various Statement in the on-line news article of thus stands, there is no question or challenge with respect to the reliability of the other
government facilities (Proclamation No. 216 Inquirer236 made by Marawi City Mayor incidents, which by themselves are ample to preclude the conclusion that the President's
and the Report). Majul Gandamra stating that the ASG and report is unreliable and that Proclamation No. 216 was without sufficient factual basis.
288

Verily, there is no credence to petitioners' claim that the bases for the President's imposition from not resuscitating the law. Yet it would appear that the constitutional writers
of martial law and suspension of the writ of habeas corpus were mostly inaccurate, entertained no doubt about the necessity and practicality of such specie of extraordinary
simulated, false and/or hyperbolic. power and thus, once again, bestowed on the Commander-in-Chief the power to declare
martial law albeit in its diluted form.
X. Public safety requires the declaration of martial law and the suspension of the privilege of
the writ of habeas corpus in the whole of Mindanao. Indeed, martial law and the suspension of the privilege of the writ of habeas corpus are
necessary for the protection of the security of the nation; suspension of the privilege of the
Invasion or rebellion alone may justify resort to the calling out power but definitely not the writ of habeas corpus is "precautionary , and although it might [curtail] certain rights of
declaration of martial law or suspension of the privilege of the writ of habeas corpus. For a individuals, [it] is for the purpose of defending and protecting the security of the state or the
declaration of martial law or suspension of the privilege of the writ of habeas corpus to be entire country and our sovereign people".253 Commissioner Ople referred to the suspension
valid, there must be a concurrence of actual rebellion or invasion and the public safety of the privilege of the writ of habeas corpus as a "form of immobilization" or "as a means of
requirement. In his Report, the President noted that the acts of violence perpetrated by the immobilizing potential internal enemies" "especially in areas like Mindanao."254
ASG and the Maute Group were directed not only against government forces or
establishments but likewise against civilians and their properties. 242 In addition and in Aside from protecting the security of the country, martial law also guarantees and promotes
relation to the armed hostilities, bomb threats were issued;243 road blockades and public safety. It is worthy of mention that rebellion alone does not justify the declaration of
checkpoints were set up;244 schools and churches were burned;245 civilian hostages were martial law or suspension of the privilege of the writ of habeas corpus; the public safety
taken and killed;246 non-Muslims or Christians were targeted;247 young male Muslims were requirement must likewise be present.
forced to join their group;248 medical services and delivery of basic services were
hampered;249 reinforcements of government troops and civilian movement were b) As Commander-in-Chief, the President receives vital, relevant, classified, and live
hindered;250 and the security of the entire Mindanao Island was compromised.251 information which equip and assist him in making decisions.

These particular scenarios convinced the President that the atrocities had already escalated In Parts IX and X, the Court laid down the arsenal of facts and events that formed the basis
to a level that risked public safety and thus impelled him to declare martial law and suspend for Proclamation No. 216. For the President, the totality of facts and events, more likely than
the privilege of the writ of habeas corpus. In the last paragraph of his Report, the President not, shows that actual rebellion exists and that public safety requires the declaration of
declared: martial law and suspension of the privilege of the writ of habeas corpus. Otherwise stated,
the President believes that there is probable cause that actual rebellion exists and public
While the government is presently conducting legitimate operations to address the on-going safety warrants the issuance of Proclamation No. 216. In turn, the Court notes that the
rebellion, if not the seeds of invasion, public safety necessitates the continued President, in arriving at such a conclusion, relied on the facts and events included in the
implementation of martial law and the suspension of the privilege of the writ of habeas Report, which we find sufficient.
corpus in the whole of Mindanao until such time that the rebellion is completely quelled. 252
To be sure, the facts mentioned in the Proclamation and the Report are far from being
Based on the foregoing, we hold that the parameters for the declaration of martial law and exhaustive or all-encompassing. At this juncture, it may not be amiss to state that as
suspension of the privilege of the writ f habeas corpus have been properly and fully complied Commander-in-Chief, the President has possession of documents and information classified
with. Proclamation No. 216 has sufficient factual basis there being probable cause to believe as "confidential", the contents of which cannot be included in the Proclamation or Report for
that rebellion exists and that public safety requires the martial law declaration and the reasons of national security. These documents may contain information detailing the position
suspension of the privilege of the writ of habeas corpus. of government troops and rebels, stock of firearms or ammunitions, ground commands and
operations, names of suspects and sympathizers, etc. , In fact, during the closed door session
XI. Whole of Mindanao held by the Court, some information came to light, although not mentioned in the
Proclamation or Report. But then again, the discretion whether to include the same in the
Proclamation or Report is the judgment call of the President. In fact, petitioners concede to
a) The overriding and paramount concern of martial law is the protection of the security of
this. During the oral argument, petitioner Lagman admitted that "the assertion of facts [in
the nation and the good and safety of the public.
the Proclamation and Report] is the call of the President."255

Considering the nation's and its people's traumatic experience martial law under the Marcos
It is beyond cavil that the President can rely on intelligence reports and classified documents.
regime, one would expect the framers of the 1987 Constitution to stop at nothing
"It is for the President as [C]ommander-in[C]hief of the Armed Forces to appraise these
289

[classified evidence or documents/]reports and be satisfied that the public safety demands e) The Constitution has provided sufficient safeguards against possible abuses of
the suspension of the writ."256 Significantly, respect to these so-called classified documents is Commander-in-Chief's powers; further curtailment of Presidential powers should not only be
accorded even "when [the] authors of or witnesses to these documents may not be discouraged but also avoided.
revealed."257
Considering the country's history, it is understandable that the resurgence of martial law
In fine, not only does the President have a wide array of information before him, he also has would engender apprehensions among the citizenry. Even the Court as an institution cannot
the right, prerogative, and the means to access vital, relevant, and confidential data, project a stance of nonchalance. However, the importance of martial law in the context of
concomitant with his position as Commander-in-Chief of the Armed Forces. our society should outweigh one's prejudices and apprehensions against it. The significance
of martial law should not be undermined by unjustified fears and past experience. After all,
c) The Court has no machinery or tool equal to that of the Commander-in-Chief to ably and martial law is critical and crucial to the promotion of public safety, the preservation of the
properly assess the ground conditions. nation's sovereignty and ultimately, the survival of our country. It is vital for the protection of
the country not only against internal enemies but also against those enemies lurking from
beyond our shores. As such, martial law should not be cast aside, or its scope and potency
In contrast, the Court does not have the same resources available to the President. However,
limited and diluted, based on bias and unsubstantiated assumptions.
this should not be considered as a constitutiona1 lapse. On the contrary, this is in line with
the function of the Court, particularly in this instance, to determine the sufficiency of factual
basis of Proclamation No. 216. As thoroughly discussed in Part VIII, the determination by the Conscious of these fears and apprehensions, the Constitution placed several safeguards
Court of the sufficiency of factual basis must be limited only to the facts and information which effectively watered down the power to declare martial law. The 1987 Constitution
mentioned in the Report and Proclamation. In fact, the Court, in David v. President "[clipped] the powers of [the] Commander-in-Chief because of [the] experience with the
Macapagal-Arroyo,258 cautioned not to "undertake an independent investigation beyond the previous regime."261 Not only were the grounds limited to actual invasion or rebellion, but its
pleadings." In this regard, "the Court will have to rely on the fact-finding capabilities of the duration was likewise fixed at 60 days, unless sooner revoked, nullified, or extended; at the
[E]xecutive [D]epartment;"259 in turn, the Executive Department will have to open its findings same time, it is subject to the veto powers of the Court and Congress.
to the Court,260 which it did during the closed door session last June 15, 2017.
Commissioner Monsod, who, incidentally, is a counsel for the Mohamad Petition, even
d) The 1987 Constitution grants to the President, as Commander-in-Chief, the discretion exhorted his colleagues in the Constitutional Convention to look at martial law from a new
to determine the territorial coverage or application of martial law or suspension of the perspective by elaborating on the sufficiency of the proposed safeguards:
privilege of the writ of habeas corpus.
MR. MONSOD. x x x
Section 18, Article VII of the Constitution states that "[i]n case of invasion or rebellion, when
the public safety requires it, [the President] may x x x suspend the privilege of writ of habeas Second, we have been given a spectre of non sequitur, that the mere declaration of martial
corpus or place the Philippines or any part thereof under martial law." Clearly, the law for a fixed period not exceeding 60 days, which is subject to judicial review, is going to
Constitution grants to the President the discretion to determine the territorial coverage of result in numerous violations of human rights, the predominance of the military forever and
martial law and the suspension of the privilege of the writ of habeas corpus. He may put the in untold sufferings. Madam President, we are talking about invasion and rebellion. We may
entire Philippines or only a part thereof under martial law. not have any freedom to speak of after 60 days, if we put as a precondition the concurrence
of Congress. That might prevent the President from acting at that time in order to meet the
This is both an acknowledgement and a recognition that it is the Executive Department, problem. So I would like to suggest that, perhaps, we should look at this in its proper
particularly the President as Commander-in-Chief, who is the repository of vital, classified, perspective. We are only looking at a very specific case. We are only looking at a case of the
and live information necessary for and relevant in calibrating the territorial application of first 60 days at its maximum. And we are looking at actual invasion and rebellion, and there
martial law and the suspension of the privilege of the writ of habeas corpus. It, too, is a are other safeguards in those cases.262
concession that the President has the tactical and military support, and thus has a more
informed understanding of what is happening on the ground. Thus, the Constitution imposed Even Bishop Bacani was convinced that the 1987 Constitution has enough safeguards against
a limitation on the period of application, which is 60 days, unless sooner nullified, revoked or presidential abuses and commission of human rights violations. In voting yes for the
extended, but not on the territorial scope or area of coverage; it merely stated "the elimination of the requirement of prior concurrence of Congress, Bishop Bacani stated, viz.:
Philippines or any part thereof," depending on the assessment of the President.
290

BISHOP BACANI. Yes, just two sentences. The reason I vote II yes is that despite my concern To illustrate: A contingent armed with high-powered firearms publicly assembled in Padre
for human rights, I believe that a good President can also safeguard human rights and human Faura, Ermita, Manila where the Court's compound is situated. They overpowered the
lives as well. And I do not want to unduly emasculate the powers of the President. Xxx263 guards, entered the Court's premises, and hoisted the ISIS flag. Their motive was
political, i.e., they want to remove from the allegiance to the Philippine government a part of
Commissioner Delos Reyes shared the same sentiment, to wit: the territory of the Philippines, particularly the Court's compound and establish it as an ISIS-
territory.
MR. DE LOS REYES. May I explain my vote, Madam President.
Based on the foregoing illustration, and vis-a-vis the nature of the crime of rebellion, could
we validly say that the rebellion is confined only within the Court's compound? Definitely
x x x The power of the President to impose martial law is doubtless of a very high and
not. The possibility that there are other rebels positioned in the nearby buildings or
delicate nature. A free people are naturally jealous of the exercise of military power, and the
compound of the Philippine General Hospital (PGH) or the Manila Science High Schoo1
power to impose martial law is certainly felt to be one of no ordinary magnitude. But as
(MSHS) could not be discounted. There is no way of knowing that all participants in the
presented by the Committee, there are many safeguards: 1) it is limited to 60 days; 2)
rebellion went and stayed inside the Court's compound.
Congress can revoke it; 3) the Supreme Court can still review as to the sufficiency of factual
basis; and 4) it does not suspend the operation of the Constitution. To repeat what I have
quoted when I interpellated Commissioner Monsod, it is said that the power to impose Neither could it be validly argued that the armed contingent positioned in PGH or MSHS
martial law is dangerous to liberty and may be abused. All powers may be abused if placed in is not engaged in rebellion because there is no publicity in their acts as, in fact, they were
unworthy hands. But it would be difficult, we think, to point out any other hands in which merely lurking inside the compound of PGH and MSHS. However, it must be pointed out that
this power will be more safe and at the same time equally effectual. When citizens of the for the crime of rebellion to be consummated, it is not required that all armed participants
State are in arms against each other and the constituted authorities are unable to execute should congregate in one place, in this case, the Court's compound, and publicly rise in arms
the laws, the action of the President must be prompt or it is of little value. against the government for the attainment of their culpable purpose. It suffices that
a portion of the contingent gathered and formed a mass or a crowd and engaged in an armed
public uprising against the government. Similarly, it cannot be validly concluded that the
At this juncture, it bears to stress that it was the collective sentiment of the framers of the
grounds on which the armed public uprising actually to6k place should be the measure of the
1987 Constitution that sufficient safeguards against possible misuse and abuse by the
extent, scope or range, of the actual I rebellion. This is logical since the other rebels
Commander-in-Chief of his extraordinary powers are already in place and that no further
positioned in PGH, MSHS, I or elsewhere, whose participation did not involve
emasculation of the presidential powers is called for in the guise of additional safeguards.
the publicity aspect of rebellion, may also be considered as engaging in the crime of
The Constitution recognizes that any further curtailment, encumbrance, or emasculation of
rebellion.
the presidential powers would not generate any good among the three co-equal branches,
and to the country and its citizens as a whole. Thus:
Proceeding from the same illustration, suppose we say that the President, after finding
probable cause that there exists actual rebellion and that public safety requires it, declares
MR. OPLE. The reason for my concern, Madam President, is that when we put all of these
martial law and suspends the writ of habeas corpus in the whole of Metro Manila, could we
encumbrances on the President and Commander-in-Chief during an actual invasion or
then say that the territorial coverage of the proclamation is too expansive?
rebellion, given an intractable Congress that may be dominated by opposition parties, we
may be actually impelling the President to use the sword of Alexander to cut the Gordian
knot by just declaring a revolutionary government that sets him free to deal with the invasion To answer this question, we revert back to the premise that the discretion to determine the
or the insurrection. x x x265 (Emphasis supplied) territorial scope of martial law lies with the President. The Constitution grants him the
prerogative whether to put the entire Philippines or any part thereof under martial law.
There is no constitutional edict that martial law should be confined only in the particular
f) Rebellion and public safety; nature, scope, and range.
place where the armed public uprising actually transpired. This is not only practical but also
logical. Martial law is an urgent measure since at stake is the nation's territorial sovereignty
It has been said that the "gravamen of the crime of rebellion is an armed public uprising and survival. As such, the President has to respond quickly. After the rebellion in the Court's
against the government;"266 and that by nature, "rebellion is x x x a crime of masses or compound, he need not wait for another rebellion to be mounted in Quezon City before he
multitudes, involving crowd action, that cannot be confined a priori, within predetermined could impose martial law thereat. If that is the case, then the President would have to wait
bounds."267 We understand this to mean that the precise extent or range of the rebellion until every remote corner in the country is infested with rebels before he could declare
could not be measured by exact metes and bounds. martial law in the entire Philippines. For sure, this is not the scenario envisioned by the
Constitution.
291

Going back to the illustration above, although the President is not required to impose martial Moreover, the President's duty to maintain peace and public safety is not limited only to the
law only within the Court's compound because it is where the armed public uprising actually place where there is actual rebellion; it extends to other areas where the present hostilities
transpired, he may do so if he sees fit. At the same time, however, he is not precluded from are in danger of spilling over. It is not intended merely to prevent the escape of lawless
expanding the coverage of martial law beyond the Court's compound. After all, rebellion is elements from Marawi City, but also to avoid enemy reinforcements and to cut their supply
not confined within predetermined bounds. lines coming from different parts of Mindanao. Thus, limiting the proclamation and/or
suspension to the place where there is actual rebellion would not only defeat the purpose of
Public safety, which is another component element for the declaration of martial law, declaring martial law, it will make the exercise thereof ineffective and useless.
"involves the prevention of and protection from events that could endanger the safety of the
general public from significant danger, injury/harm, or damage, such as crimes or g) The Court must stay within the confines of its power.
disasters."268 Public safety is an abstract term; it does not take any physical form. Plainly, its
range, extent or scope could not be physically measured by metes and bounds. The Court can only act within the confines of its power.1âwphi1 For the Court to overreach is
to infringe upon another's territory. Clearly, the power to determine the scope of territorial
Perhaps another reason why the territorial scope of martial law should not necessarily be application belongs to the President. "The Court cannot indulge in judicial legislation without
limited to the particular vicinity where the armed public uprising actually transpired, is violating the principle of separation of powers, and, hence, undermining the foundation of
because of the unique characteristic of rebellion as a crime. "The crime of rebellion consists our republican system."281
of many acts. It is a vast movement of men and a complex net of intrigues and plots. Acts
committed in furtherance of rebellion[,] though crimes in themselves[,] are deemed To reiterate, the Court is not equipped with the competence and logistical machinery to
absorbed in one single crime of rebellion."269 Rebellion absorbs "other acts committed in its determine the strategical value of other places in the military's efforts to quell the rebellion
pursuance".270 Direct assault,271murder,272 homicide,273 arson,274 robbery,275 and and restore peace. It would be engaging in an act of adventurism if it dares to embark on a
kidnapping,276 just to name a few, are absorbed in the crime of rebellion if committed in mission of deciphering the territorial metes and bounds of martial law. To be blunt about it,
furtherance of rebellion; "[i]t cannot be made a basis of a separate charge."277Jurisprudence hours after the proclamation of martial law none of the members of this Court could have
also teaches that not only common crimes may be absorbed in rebellion but also "offenses divined that more than ten thousand souls would be forced to evacuate to Iligan and
under special laws [such as Presidential Decree No. 1829]278 which are perpetrated in Cagayan de Oro and that the military would have to secure those places also; none of us
furtherance of the political offense".279 "All crimes, whether punishable under a special law could have predicted that Cayamora Maute would be arrested in Davao City or that his wife
or general law, which are me e components or ingredients, or committed in furtherance Ominta Romato Maute would be apprehended in Masiu, Lanao del Sur; and, none of us had
thereof, become absorbed in the crime of rebellion and cannot be isolated and charged as an inkling that the Bangsamoro Islamic Freedom Fighters (BIFF) would launch an attack in
separate crimes in themselves.280 Cotabato City. The Court has no military background and technical expertise to predict that.
In the same manner, the Court lacks the technical capability to determine which part of
Thus, by the theory of absorption, the crime of murder committed in Makati City, if Mindanao would best serve as forward operating base of the military in their present
committed in furtherance of the crime of rebellion being hypothetically staged in Padre endeavor in Mindanao. Until now the Court is in a quandary and can only speculate whether
Faura, Ermita, Manila, is stripped of its common complexion and is absorbed in the crime of the 60-day lifespan of Proclamation No. 216 could outlive the present hostilities in
rebellion. This all the more makes it difficult to confine the application of martial law only to Mindanao. It is on this score that the Court should give the President sufficient leeway to
the place where the armed public uprising is actually taking place. In the illustration above, address the peace and order problem in Mindanao.
Padre Faura could only be the nerve center of the rebellion but at the same time rebellion is
also happening in Makati City. Thus, considering the current situation, it will not serve any purpose if the President is
goaded into using "the sword of Alexander to cut the Gordian knot"282 by attempting to
In fine, it is difficult, if not impossible, to fix the territorial scope of martial law in direct impose another encumbrance; after all "the declaration of martial law or the suspension of
proportion to the "range" of actual rebellion and public safety simply because rebellion and the privilege of the writ of habeas corpus is essentially an executive act."283
public safety have no fixed physical dimensions. Their transitory and abstract nature defies
precise measurements; hence, the determination of the territorial scope of martial law could Some sectors, impelled perhaps by feelings of patriotism, may wish to subdue, rein in, or give
only be drawn from arbitrary, not fixed, variables. The Constitution must have considered the President a nudge, so to speak, as some sort of reminder of the nation's experience
these limitations when it granted the President wide leeway and flexibility in determining the under the Marcos-styled martial law. However, it is not fair to judge President Duterte based
territorial scope of martial law. on the ills some of us may have experienced during the Marcos-martial law era. At this point,
the Court quotes the insightful discourse of Commissioner Ople:
292

MR. OPLE. x x x a. On January 13, 2017, an improvised explosive device (IED) exploded in Barangay Campo
Uno, Lamita City, Basilan. A civilian was killed while another was wounded.290
Madam President, there is a tendency to equate patriotism with rendering the executive
branch of the government impotent, as though by reducing drastically the powers of the b. On January 19, 2017, the ASG kidnapped three Indonesians near Bakungan Island,
executive, we are rendering a service to human welfare. I think it is also important to Taganak, Tawi-Tawi.291
understand that the extraordinary measures contemplated in the Article on the Executive
pertain to a practical state of war existing in this country when national security will become c. On January 29, 2017, the ASG detonated an IED in Barangay Danapah, Albarka, Basilan
a common bond of patriotism of all Filipinos, especially if it is an actual invasion or an actual resulting in the death of two children and the wounding of three others.292
rebellion, and the President may have to be given a minimum flexibility to cope with such
unprecedented threats to the survival of a nation. I think the Commission has done so but at
d. From March to May 2017, there were eleven (11) separate instances of IED explosions by
the same time has not, in any manner, shunned the task of putting these powers under a
the BIFF in Mindanao. These resulted in the death and wounding of several personalities.293
whole system of checks and balances, including the possible revocation at any time of a
proclamation of martial law by the Congress, and in any case a definite determination of
these extraordinary powers, subject only to another extension to be determined by Congress e. On February 26, 2017, the ASG beheaded its kidnap victim, Juergen Kantner in Sulu. 294
in the event that it is necessary to do so because the emergency persists.
f. On April 11, 2017, the ASG infiltrated Inabaga, Bohol resulting in firefights between rebels
So, I think this Article on the Executive for which I voted is completely responsible; it is and government troops.295
attuned to the freedom and the rights of the citizenry. It does not render the presidency
impotent and, at the same time, it allows for a vigorous representation of the people through g. On April 13, 2017, the ASG beheaded Filipino kidnap victim Noel Besconde.296
their Congress when an emergency measure is in force and effect. 284
h. On April 20, 2017, the ASG kidnapped SSg. Anni Siraji and beheaded him three days
h) Several local armed groups have formed linkages aimed at committing rebellion and acts in later.297
furtherance thereof in the whole of Mindanao.
There were also intelligence reports from the military about offensives committed by the
With a predominantly Muslim population, Marawi City is "the only Islamic City of the ASG and other local rebel groups. All these suggest that the rebellion in Marawi has already
South."285 On April 15, 1980, it was conferred the official title of "Islamic City of spilled over to other parts of Mindanao.
Marawi."286 The city's first name, "Dansalan," "was derived from the word 'dansal', meaning
a destination point or rendezvous. Literally, it also means arrival or coming."287 Marawi lies in Moreover, considering the widespread atrocities in Mindanao and the linkages established
the heart of Mindanao. In fact, the Kilometer Zero marker in Mindanao is found in Marawi among rebel groups, the armed uprising that was initially staged in Marawi cannot be
City thereby making Marawi City the point of reference of all roads in Mindanao. justified as confined only to Marawi. The Court therefore will not simply disregard the events
that happened during the Davao City bombing, the Mamasapano massacre, the Zamboanga
Thus, there is reasonable basis to believe that Marawi is only the staging point of the City siege, and the countless bombings in Cotabato, Sultan Kudarat, Sulu, and Basilan, among
rebellion, both for symbolic and strategic reasons. Marawi may not be the target but the others.298 The Court cannot simply take the battle of Marawi in isolation. As a crime without
whole of Mindanao. As mentioned in the Report, "[l]awless armed groups have historically predetermined bounds, the President has reasonable basis to believe that the declaration of
used provinces adjoining Marawi City as escape routes, supply lines, and backdoor martial law, as well as the suspension of the privilege of the writ of habeas corpus in the
passages;"288 there is also the plan to establish a wilayat in Mindanao by staging the siege of whole of Mindanao, is most necessary, effective, and called for by the circumstances.
Marawi. The report that prior to May 23, 2017, Abdullah Maute had already dispatched some
of his men to various places in Mindanao, such as Marawi, Iligan, and Cagayan de Oro for i) Terrorism neither negates nor absorbs rebellion.
bombing operations, carnapping, and the murder of military and police personnel,289 must
also be considered. Indeed, there is some semblance of truth to the contention that Marawi
is only the start, and Mindanao the end. It is also of judicial notice that the insurgency in Mindanao has been ongoing for decades.
While some groups have sought legal and peaceful means, others have resorted to violent
extremism and terrorism. Rebellion may be subsumed under the crime of terrorism, which
Other events also show that the atrocities were not concentrated in Marawi City. Consider has a broader scope covering a wide range of predicate crimes. In fact, rebellion is only one
these: of the various means by which terrorism can be committed.299 However, while the scope of
293

terrorism may be comprehensive, its purpose is distinct and well-defined. The objective of a Let us face up to the fact that the siege in Marawi City has entered the second month and
"'terrorist" is to sow and create a condition of widespread fear among the populace in order only God or Allah knows when it would end. Let us take notice of the fact that the casualties
to coerce the government to give in to an unlawful demand. This condition of widespread of the war are mounting. To date, 418 have died. Out of that were 303 Maute rebels as
fear is traditionally achieved through bombing, kidnapping, mass killing, and beheading, against 71 government troops and 44 civilians.
among others. In contrast, the purpose of rebellion, as previously discussed, is
political, i.e., (a) to remove from the allegiance to the Philippine Government or its laws: (i) Can we not sheathe our swords and pause for a while to bury our dead, including our
the territory of the Philippines or any part thereof; (ii) any body of land, naval, or armed differences and prejudices?
forces; or (b) to deprive the Chief Executive or Congress, wholly or partially, of any of their
powers and prerogatives.
WHEREFORE, the Court FINDS sufficient factual bases for the issuance of Proclamation No.
216 and DECLARES it as CONSTITUTIONAL. Accordingly, the consolidated Petitions are
In determining what crime was committed, we have to look into the main objective of the hereby DISMISSED.
malefactors. If it is political, such as for the purpose of severing the allegiance of Mindanao
to the Philippine Government to establish a wilayat therein, the crime is rebellion. If, on the
SO ORDERED.
other hand, the primary objective is to sow and create a condition of widespread and
extraordinary fear and panic among the populace in order to coerce the government to give
in to an unlawful demand, the crime is terrorism. Here, we have already explained and ruled G.R. No. 231658, 04 July 2017, EN BANC (Del Castillo, J.)
that the President did not err in believing that what is going on in Marawi City is one
contemplated under the crime of rebellion. DOCTRINE OF THE CASE: It is difficult, if not impossible, to fix the territorial scope of martial
law in direct proportion to the "range" of actual rebellion and public safety simply because
rebellion and public safety have no fixed physical dimensions. Their transitory and abstract
In any case, even assuming that the insurgency in Marawi City can also be characterized as
nature defies precise measurements; hence, the determination of the territorial scope of
terrorism, the same will not in any manner affect Proclamation No. 216. Section 2 of Republic
martial law could only be drawn from arbitrary, not fixed, variables. The Constitution must
Act (RA) No. 9372, otherwise known as the Human Security Act of 2007 expressly provides
have considered these limitations when it granted the President wide leeway and flexibility in
that "[n]othing in this Act shall be interpreted as a curtailment, restriction or diminution of
determining the territorial scope of martial law. Moreover, the President's duty to maintain
constitutionally recognized powers of the executive branch of the government." Thus, as long
peace and public safety is not limited only to the place where there is actual rebellion; it
as the President complies with all the requirements of Section 18, Article VII, the existence of
extends to other areas where the present hostilities are in danger of spilling over. It is not
terrorism cannot prevent him from exercising his extraordinary power of proclaiming martial
intended merely to prevent the escape of lawless elements from Marawi City, but also to
' law or suspending the privilege of the writ of habeas corpus. After all, the extraordinary
avoid enemy reinforcements and to cut their supply lines coming from different parts of
powers of the President are bestowed on him by the Constitution. No act of Congress can,
Mindanao. Thus, limiting the proclamation and/or suspension to the place where there is
therefore, curtail or diminish such powers.
actual rebellion would not only defeat the purpose of declaring martial law, it will make the
exercise thereof ineffective and useless.
Besides, there is nothing in Art. 134 of the RPC and RA 9372 which states that rebellion and
terrorism are mutuallty exclusive of each other or that they cannot co-exist together. RA FACTS:
9372 does not expressly or impliedly repeal Art. 134 of the RPC. And while rebellion is one of
the predicate crimes of terrorism, one cannot absorb the other as they have different Effective May 23, 2017, and for a period not exceeding 60 days, President Rodrigo Roa
elements.300 Duterte issued Proclamation No. 216 declaring a state of martial law and suspending the
privilege of the writ of habeas corpus in the whole of Mindanao.
Verily, the Court upholds the validity of the declaration of martial law and suspension of the
privilege of the writ of habeas corpus in the entire Mindanao region. Within the timeline set by Section 18, Article VII of the Constitution, the President submitted
to Congress on May 25, 2017, a written Report on the factual basis of Proclamation No. 216.
At the end of the day, however ardently and passionately we may believe in the validity or The Report pointed out that for decades, Mindanao has been plagued with rebellion and
correctness of the varied and contentious causes or principles that we espouse, advocate or lawless violence which only escalated and worsened with the passing of time. The Report
champion, let us not forget that at this point in time we, the Filipino people, are confronted also highlighted the strategic location of Marawi City and the crucial and significant role it
with a crisis of such magnitude and proportion that we all need to summon the spirit of unity plays in Mindanao, and the Philippines as a whole. In addition, the Report pointed out the
and act as one undivided nation, if we are to overcome and prevail in the struggle at hand. possible tragic repercussions once Marawi City falls under the control of the lawless groups.
294

After the submission of the Report and the briefings, the Senate issued a resolution that it should be treated as sui generis separate and different from those enumerated in
expressing full support to the martial law proclamation and finding Proclamation No. 216 to Article VIII.
be satisfactory, constitutional and in accordance with the law. In the same Resolution, the
Senate declared that it found no compelling reason to revoke the same. The House of Under the third paragraph of Section 18, Article VII, a petition filed pursuant therewith will
Representatives likewise issued a resolution expressing its full support to the President, as it follow a different rule on standing as any citizen may file it. Said provision of the Constitution
finds no reason to revoke Proclamation No. 216. also limits the issue to the sufficiency of the factual basis of the exercise by the Chief
Executive of his emergency powers. The usual period for filing pleadings in Petition for
Invoking the third paragraph of Section 18, Article VII of the Constitution, various citizens Certiorari pursuant to Section 1 or Section 5 of Article VIII is likewise not applicable under the
filed several petitions, essentially invoking the Court’s specific and special jurisdiction to third paragraph of Section 18, Article VII considering the limited period within which the
review the sufficiency of the factual basis of Proclamation No. 216; and seeking to nullify Court has to promulgate its decision.
Proclamation No. 216 for being unconstitutional because it lacks sufficient factual basis.
In fine, the phrase “in an appropriate proceeding: appearing on the third paragraph of
ISSUES: Section 18, Article VII refers to any action initiated by a citizen for the purpose of questioning
1. Are the instant petitions the "appropriate proceeding" covered by Paragraph 3, Section 18, the sufficiency of the factual basis of the exercise of the Chief Executive's emergency powers,
Article VII of the Constitution? as in these cases. It could be denominated as a complaint, a petition, or a matter to be
resolved by the Court.
2. Is the President, in declaring martial law and suspending the privilege of the writ of habeas
corpus,: 2.
a. required to be factually correct or only not arbitrary in his appreciation of facts; A.) NO. In determining the sufficiency of the factual basis of the declaration and/or the
b. required to obtain the favorable recommendation thereon of the Secretary of National suspension, the Court should look into the full complement or totality of the factual
Defense; or basis, and not piecemeal or individually. Neither should the Court expect absolute
c. required to take into account only the situation at the time of the proclamation, even if correctness of the facts stated in the proclamation and in the written Report as the
subsequent events prove the situation to have not been accurately reported? President could not be expected to verify the accuracy and veracity of all facts reported
to him due to the urgency of the situation.
3. Is the power of the Court to review the sufficiency of the factual basis of the proclamation
of martial law or the suspension of the privilege of the writ of habeas corpus independent of To require precision in the President's appreciation of facts would unduly burden him
the actual actions that have been taken by Congress jointly or separately? and therefore impede the process of his decision-making. Such a requirement will
practically necessitate the President to be on the ground to confirm the correctness of
4. Were there sufficient factual basis for the proclamation of martial law or the suspension of the reports submitted to him within a period that only the circumstances obtaining
the privilege of the writ of habeas corpus? would be able to dictate.
a. What are the parameters for review?
b. Who has the burden of proof? B.) NO. Even the recommendation of, or consultation with, the Secretary of National
c. What is the threshold of evidence? Defense, or other high-ranking military officials, is not a condition for the President to
declare martial law.
5. Is the exercise of the power of judicial review by the Court involves the calibration of
graduated powers granted the President as Commander-in-Chief? A plain reading of Section 18, Article VII of the Constitution shows that the President's
power to declare martial law is not subject to any condition except for the requirements
6. May Proclamation No. 216 be considered, vague, and thus null and void: of actual invasion or rebellion and that public safety requires it. Besides, it would be
a. with its inclusion of “other rebel groups”; or contrary to common sense if the decision of the President is made dependent on the
b. since it has no guidelines specifying its actual operational parameters within the entire recommendation of his mere alter ego. Rightly so, it is only on the President and no other
Mindanao region? that the exercise of the powers of the Commander-in-Chief under Section 18, Article VII
of the Constitution is bestowed.
RULING:
C.) YES. Since the exercise of these powers is a judgment call of the President, the
1. YES. The unique features of the third paragraph of Section 18, Article VII clearly indicate determination of the Court as to whether there is sufficient factual basis for the exercise
295

of the power to declare martial law and/or suspend the privilege of the writ of habeas Government a portion of its territory and to deprive the Chief Executive of any of his powers
corpus, must be based only on facts or information known by or available to the and prerogative, leading the President to believe that there was probable cause that the
President at the time he made the declaration or suspension which facts or information crime of rebellion was and is being committed and that public safety requires the imposition
are found in the proclamation as well as the written Report submitted by him to of martial law and suspension of the privilege of the writ of habeas corpus.
Congress. These may be based on the situation existing at the time the declaration was
made or past events. As to how far the past events should be from the present depends A.) Section 18, Article VII itself sets the parameters for determining the sufficiency of the
on the President. factual basis for the declaration of martial law and/or the suspension of the privilege of
the writ of habeas corpus, "namely (1) actual invasion or rebellion, and (2) public safety
Similarly, events that happened after the issuance of the proclamation, which are requires the exercise of such power."170 Without the concurrence of the two conditions,
included in the written report, cannot be considered in determining the sufficiency of the the President's declaration of martial law and/or suspension of the privilege of the writ of
factual basis of the declaration of martial law and/or the suspension of the privilege of habeas corpus must be struck down.
the writ of habeas corpus since these happened after the President had already issued
A review of the aforesaid facts similarly leads the Court to conclude that the President, in
the proclamation. If at all, they may be used only as tools, guides or reference in the issuing Proclamation No. 216, had sufficient factual ' bases tending to show that actual
Court's determination of the sufficiency of factual basis, but not as part or component of rebellion exists. The President's conclusion, that there was an armed public uprising, the
the portfolio of the factual basis itself. culpable purpose of which was the removal from the allegiance of the Philippine
Government a portion of its territory and the deprivation of the President from
performing his powers and prerogatives, was reached after a tactical consideration of the
3. YES. The Court may strike down the presidential proclamation in an appropriate facts. In fine, the President satisfactorily discharged his burden of proof.
proceeding filed by any citizen on the ground of lack sufficient factual basis. On the other
hand, Congress may revoke the proclamation or suspension, which revocation shall not be B.) After all, what the President needs to satisfy is only the standard of probable cause
set aside by the President. for a valid declaration of martial law and suspension of the privilege of the writ of habeas
corpus.
In reviewing the sufficiency of the factual basis of the proclamation or suspension, the Court
considers only the information and data available to the President prior to, or at the time of C.) What the President needs to satisfy is only the standard of probable cause for a valid
the declaration; it is not allowed to “undertake an independent investigation beyond the declaration of martial law and suspension of the privilege of the writ of habeas corpus.
pleadings.” On the other hand, Congress may take into consideration not only data available
5. NO. The power of judicial review does not extend to calibrating the President's decision
prior to, but likewise events supervening the declaration. Unlike the Court which does not
pertaining to which extraordinary power to avail given a set of facts or conditions. To do so
look into the absolute correctness of the factual basis as will be discussed below, Congress would be tantamount to an incursion into the exclusive domain of the Executive and an
could probe deeper and further; it can delve into the accuracy of the facts presented before infringement on the prerogative that solely, at least initially, lies with the President.
it.
6.
In addition, the Court's review power is passive; it is only initiated by the filing of a petition A.) NO. The term "other rebel groups" in Proclamation No. 216 is not at all vague when
"in an appropriate proceeding" by a citizen. On the other hand, Congress' review mechanism viewed in the context of the words that accompany it. Verily, the text of Proclamation No.
is automatic in the sense that it may be activated by Congress itself at any time after the 216 refers to "other rebel groups" found in Proclamation No. 55, which it cited by way of
proclamation or suspension was made. reference in its Whereas clauses.

Thus, the power to review by the Court and the power to revoke by Congress are not only B.) NO. There is no need for the Court to determine the constitutionality of the implementing
totally different but likewise independent from each other although concededly, they have and/or operational guidelines, general orders, arrest orders and other orders issued after the
the same trajectory, which is, the nullification of the presidential proclamation. Needless to proclamation for being irrelevant to its review. Thus, any act committed under the said
say, the power of the Court to review can be exercised independently from the power of orders in violation of the Constitution and the laws, such as criminal acts or human rights
revocation of Congress. violations, should be resolved in a separate proceeding. Finally, there is a risk that if the
Court wades into these areas, it would be deemed a trespassing into the sphere that is
4. YES. The President deduced from the facts available to him that there was an armed public reserved exclusively for Congress in the exercise of its power to revoke.
uprising, the culpable purpose of which was to remove from the allegiance to the Philippine
296

EN BANC Senate President Aquilino "Koko" Pimentel III (Senate President Pimentel), through the Office
of the Senate Legal Counsel, separately filed their Consolidated Comment (Ex Abudanti
Cautela) on June 29, 2017.
July 25, 2017

ANTECEDENT FACTS
G.R. No. 231671

On May 23, 2017, President Duterte issued Proclamation No. 216, declaring a state of martial
ALEXANDER A. PADILLA, RENE A.V. SAGUISAG, CHRISTIAN S. MONSOD, LORETTA ANN P.
law and suspending the privilege of the writ of habeas corpus in the Mindanao group of
ROSALES, RENE B. GOROSPE, and SENATOR LEILA M. DE LIMA, Petitioners
islands on the grounds of rebellion and necessity of public safety pursuant to Article VII,
vs.
Section 18 of the 1987 Constitution.
CONGRESS OF THE PHILIPPINES, consisting of the SENATE OF THE PHILIPPINES, as
represented by Senate President Aquilino "Koko" Pimentel III, and the HOUSE OF
REPRESENTATIVES, as represented by House Speaker Pantaleon D. Alvarez, Respondents Within forty-eight (48) hours after the proclamation, or on May 25, 2017, and while the
Congress was in session, President Duterte transmitted his "Report relative to Proclamation
No. 216 dated 23 May 2017" (Report) to the Senate, through Senate President Pimentel, and
DECISION
the House of Representatives, through House Speaker Pantaleon D. Alvarez (House Speaker
Alvarez).
LEONARDO-DE CASTRO, J.:
According to President Duterte's Proclamation No. 216 and his Report to the Congress, the
These consolidated petitions under consideration essentially assail the failure and/or refusal declaration of a state of martial law and the suspension of the privilege of the writ of habeas
of respondent Congress of the Philippines (the Congress), composed of the Senate and the corpus in the whole of Mindanao ensued from the series of armed attacks, violent acts, and
House of Representatives, to convene in joint session and therein deliberate on Proclamation atrocities directed against civilians and government authorities, institutions, and
No. 216 issued on May 23, 201 7 by President Rodrigo Roa Duterte (President Duterte). establishments perpetrated by the Abu Sayyaf and Maute terrorist groups, in complicity with
Through Proclamation No. 216, President Duterte declared a state of martial law and other local and foreign armed affiliates, who have pledged allegiance to the Islamic State of
suspended the privilege of the writ of habeas corpus in the whole of Mindanao for a period Iraq and Syria (ISIS), to sow lawless violence, terror, and political disorder over the said
not e:xceeding sixty (60) days effective from the date of the proclamation's issuance. region for the ultimate purpose of establishing a DAESH wilayah or Islamic Province in
Mindanao.
In the Petition for Mandamus of Alex.antler A. Padilla (Padilla), Rene A.V. Saguisag (Saguisag),
Christian S. Monsod (Monsod), Loretta Ann P. Rosales (Rosales), Rene B. Gorospe (Gorospe), Representatives from the Executive Department, the military, and other security officials of
and Senator Leila M. De Lima (Senator De Lima), filed on June 6, 2017 and docketed as G.R. the government were thereafter invited, on separate occasions, by the Senate and the House
No. 231671 (the Padilla Petition), petitioners seek a ruling from the Court directing the of Representatives for a conference briefing regarding the circumstances, details, and
Congress to convene in joint session to deliberate on Presidential Proclamation No. 216, and updates surrounding the President's proclamation and report.
to vote thereon.1
On May 29, 2017, the briefing before the Senate was conducted, which lasted for about four
In the Petition for Certiorari and Mandamus of former Senator Wigberto E. Tanada (Tanada), (4) hours, by Secretary of National Defense Delfin N. Lorenza (Secretary Lorenzana), National
Bishop Emeritus Deogracias Iniguez (Bishop Iniguez), Bishop Broderick Pabillo (Bishop Pabillo Security Adviser and Director General of the National Security Council Hermogenes C.
), Bishop Antonio Tobias (Bishop Tobias), Mo. Adelaida Ygrubay (Mo. Y grubay), Shamah Esperon, Jr. (Secretary Esperon), and Chief of Staff of the Armed Forces of the Philippines
Bulangis (Bulangis), and Cassandra D. Deluria (Deluria), filed on June 7, 2017 and docketed as (AFP) General Eduardo M. Afio (General Año). The following day, May 30, 2017, the Senate
G.R. No. 231694 (the Tañada Petition), petitioners entreat the Court to: (a) declare the deliberated on these proposed resolutions: (a) Proposed Senate (P.S.) Resolution No.
refusal of the Congress to convene in joint session for the purpose of considering 388,3 which expressed support for President Duterte's Proclamation No. 216; and (b) P.S.
Proclamation No. 216 to be in grave abuse of discretion amounting to a lack or excess of Resolution No. 390,4 which called for the convening in joint session of the Senate and the
jurisdiction; and (b) issue a writ of mandamus directing the Congress to convene in joint House of Representatives to deliberate on President Duterte's Proclamation No. 216.
session for the aforementioned purpose.2
P.S. Resolution No. 388 was approved, after receiving seventeen (17) affirmative votes as
Respondent Congress, represented by the Office of the Solicitor General (OSG), filed against five (5) negative votes, and was adopted as Senate Resolution No.
its Consolidated Comment on June 27, 2017. Respondents Senate of the Philippines and
297

495 entitled "Resolution Expressing the Sense of the Senate Not to Revoke, at this Time, [iii] PETITIONER [DE LIMA], AS MEMBER OF CONGRESS, HAS LEGAL STANDING TO FILE THIS
Proclamation No. 216, Series of 2017, Entitled 'Declaring a State of Martial Law and PETITION.
Suspending the Privilege of the Writ of Habeas Corpus in the Whole of Mindanao.’"6
[iv] THE CASE AND THE ISSUE INVOLVED ARE RIPE FOR JUDICIAL DETERMINATION.
P.S. Resolution No. 390, on the other hand, garnered only nine (9) votes from the senators
who were in favor of it as opposed to twelve (12) votes from the senators who were against [II] THE PLAIN TEXT OF THE CONSTITUTION, SUPPORTED BY THE EXPRESS INTENT OF THE
its approval and adoption.7 FRAMERS, AND CONFIRMED BY THE SUPREME COURT, REQUIRES THAT CONGRESS CONVENE
IN JOINT SESSION TO DELIBERATE AND VOTE AS A SINGLE DELIBERATIVE BODY.
On May 31, 201 7, the House of Representatives, having previously constituted itself as a
Committee of the Whole House,8 was briefed by Executive Secretary Salvador C. Medialdea [i] THE PLAIN TEXT OF THE CONSTITUTION REQUIRES THAT CONGRESS CONVENE IN JOINT
(Executive Secretary Medialdea), Secretary Lorenzana, and other security officials for about SESSION.
six (6) hours. After the closed-door briefing, the House of Representatives resumed its
regular meeting and deliberated on House Resolution No. 1050 entitled "Resolution
[ii] THE EXPRESS INTENT OF THE FRAMERS IS FOR CONGRESS TO CONVENE IN JOINT SESSION
Expressing the Full Support of the House of Representatives to President Rodrigo Duterte as it
TO DELIBERATE AND VOTE AS A SINGLE DELIBERATIVE BODY.
Finds No Reason to Revoke Proclamation No. 216, Entitled 'Declaring a State of Martial Law
and Suspending the Privilege of the Writ of Habeas Corpus in the Whole of Mindanao.'"9 The
House of Representatives proceeded to divide its members on the matter of approving said [iii] THE SUPREME COURT CONFIRMED IN FORTUN v. GMA THAT CONGRESS HAS THE
resolution through viva voce voting. The result shows that the members who were in favor of "AUTOMATIC DUTY" TO CONVENE IN JOINT SESSION.
passing the subject resolution secured the majority vote.10
[iv] LEGISLATIVE PRECEDENT ALSO RECOGNIZES CONGRESS' DUTY TO CONVENE IN JOINT
The House of Representatives also purportedly discussed the proposal calling for a joint SESSION.
session of the Congress to deliberate and vote on President Duterte's Proclamation No. 216.
After the debates, however, the proposal was rejected.11 [III] THE REQUIREMENT TO ACT AS A SINGLE DELIBERATIVE BODY UNDER ARTICLE VII,
[SECTION] 18 OF THE CONSTITUTION IS A MANDATORY, MINISTERIAL CONSTITUTIONAL
These series of events led to the filing of the present consolidated petitions. DUTY OF CONGRESS, WHICH CAN BE COMPELLED BY MANDAMUS.12

THE PARTIES' ARGUMENTS Petitioners claim that there is an actual case or controversy in this instance and that their
case is ripe for adjudication. According to petitioners, the resolutions separately passed by
the Senate and the House of Representatives, which express support as well as the intent not
The Padilla Petition
to revoke President Duterte's Proclamation No. 216, injure their rights "to a proper [and]
mandatory legislative review of the declaration of martial law" and that the continuing
Petitioners in G.R. No. 231671 raise the question of "[w]hether Congress is required to failure of the Congress to convene in joint session similarly causes a continuing injury to their
convene in joint session, deliberate, and vote jointly under Article VII, [Section] 18 of the rights.13
Constitution" and submit the following arguments in support of their petition:
Petitioners also allege that, as citizens and taxpayers, they all have locus standi in their
[I] THE PETITION SATISFIES THE REQUISITES FOR THE EXERCISE OF THE HONORABLE COURT'S "assertion of a public right" which they have been deprived of when the Congress refused
POWER OF JUDICIAL REVIEW. and/or failed to convene in joint session to deliberate on President Duterte's Proclamation
No. 216. Senator De Lima adds that she, together with the other senators who voted in favor
[i] THERE IS AN ACTUAL CASE OR CONTROVERSY. of the resolution to convene the Congress jointly, were even effectively denied the
opportunity to perform their constitutionally-mandated duty, under Article VII, Section 18 of
[ii] PETITIONERS, AS PART OF THE PUBLIC AND AS TAXPAYERS, POSSESS LEGAL STANDING TO the Constitution, to deliberate on the said proclamation of the President in a joint session of
FILE THIS PETITION. the Congress.14
298

On the propriety of resorting to the remedy of mandamus, petitioners posit that ''the duty of III. FAIL URE TO CONVENE A JOINT SESSION DEPRIVES THE PUBLIC OF TRANSPARENT
Congress to convene in joint session upon the proclamation of martial law or the suspension PROCEEDINGS WITHIN WHICH TO BE INFORMED OF THE FACTUAL BASES OF MARTIAL LAW
of the privilege of the writ of habeas corpus does not require the exercise of discretion." Such AND THE INTENDED PARAMETERS OF ITS IMPLEMENTATION.
mandate upon the Congress is allegedly a purely ministerial act which can be compelled
through a writ of mandamus.15 IV. THE FRAMERS OF THE CONSTITUTION INTENDED THAT A JOINT SESSION OF CONGRESS BE
CONVENED IMMEDIATELY AFTER THE DECLARATION OF MARTIAL LAW. 21
As for the substantive issue, it is the primary contention of petitioners that a plain reading of
Article VII, Section 18 of the Constitution shows that the Congress is required to convene in Similar to the contentions in the Padilla Petition, petitioners maintain that they have
joint session to review Proclamation No. 216 and vote as a single deliberative body. The sufficiently shown all the essential requisites in order for this Court to exercise its power of
performance of the constitutional obligation is allegedly mandatory, not discretionary.16 judicial review, in that: (1) an actual case or controversy exists; (2) they possess the standing
to file this case; (3) the constitutionality of a governmental act has been raised at the earliest
According to petitioners, the discretionary nature of the phrase "may revoke such possible opportunity; and (4) the constitutionality of the said act is the very lis mota of the
proclamation or suspension" under Article VII, Section 18 of the Constitution allegedly petition.
pertain to the power of the Congress to revoke but not to its obligation to jointly convene
and vote - which, they stress, is mandatory. To require the Congress to convene only when it According to petitioners, there is an actual case or controversy because the failure and/or
exercises the power to revoke is purportedly absurd since the Congress, without convening in refusal of the Congress to convene jointly deprived legislators of a venue within which to
joint session, cannot know beforehand whether a majority vote in fact exists to effect a raise a motion for revocation (or even extension) of President Duterte's Proclamation No.
revocation.17 216 and the public of an opportunity to be properly informed as to the bases and particulars
thereof.22
Petitioners claim that in Fortun v. Macapagal-Arroyo,18 this Court described the "duty" of the
Congress to convene in joint session as "automatic." The convening of the Congress in joint Petitioners likewise claim to have legal standing to sue as citizens and taxpayers.
session when former President Gloria Macapagal-Arroyo (President Macapagal-Arroyo) Nonetheless, they submit that the present case calls for the Court's liberality in the
declared martial law and suspended the privilege of the writ of habeas corpus in appreciation of their locus standi given the fact that their petition presents "a question of
Maguindanao was also a legislative precedent where the Congress clearly recognized its duty first impression - one of paramount importance to the future of our democracy - as well as
to convene in joint session.19 the extraordinary nature of Martial Law itself."23

The mandate upon the Congress to convene jointly is allegedly intended by the 1986 Petitioners contend that the convening of the Congress in joint session, whenever the
Constitutional Commission (ConCom) to serve as a protection against potential abuses in the President declares martial law or suspends the privilege of the writ of habeas corpus, is a
exercise of the President's power to declare martial law and suspend the privilege of the writ public right and duty mandated by the Constitution. The writ of mandamus is, thus, the
of habeas corpus. It is "a mechanism purposely designed by the Constitution to compel "proper recourse for citizens who seek to enforce a public right and to compel the
Congress to review the propriety of the President's action x x x [and] meant to contain performance of a public duty, especially when the public right involved is mandated by the
martial law powers within a democratic framework for the preservation of democracy, Constitution."24
prevention of abuses, and protection of the people."20
For this group of petitioners, the Members of the Congress gravely abused their discretion
The Tañada Petition for their refusal to convene in joint session, underscoring that "[w]hile a writ
of mandamus will not generally lie from one branch of the government to a coordinate
The petitioners in G.R. No. 231694 chiefly opine that: branch, or to compel the performance of a discretionary act, this admits of certain
exceptions, such as in instances of gross abuse of discretion, manifest injustice, or palpable
I. A PLAIN READING OF THE 1987 CONSTITUTION LEADS TO THE INDUBITABLE CONCLUSION excess of authority, when there is no other plain, speedy and adequate remedy." 25
THAT A JOINT SESSION OF CONGRESS TO REVIEW A DECLARATION OF MARTIAL LAW BY THE
PRESIDENT IS MANDATORY. As to the merits, petitioners assert that the convening of the Congress in joint session after
the declaration of martial law is mandatory under Article VII, Section 18 of the Constitution,
II. FAIL URE TO CONVENE A JOINT SESSION DEPRIVES LAWMAKERS OF A DELIBERATIVE AND whether or not the Congress is in session or there is intent to revoke. It is their theory that a
INTERROGATORY PROCESS TO REVIEW MARTIAL LAW. joint session should be a deliberative process in which, after debate and discussion,
299

legislators can come to an informed decision as to the factual and legal bases for the regarding the factual basis of a martial law declaration may be restricted in the interest of
declaration of martial law. Moreover, "legislators who wish to revoke the martial law national security and public safety.32
proclamation should have the right to put that vote on historical record in joint session - and,
in like manner, the public should have the right to know the position of their legislators with Respondents allege that petitioners failed to present an appropriate case for mandamus to
respect to this matter of the highest national interest."26 lie. Mandamus will only issue when the act to be compelled is a clear legal duty or a
ministerial duty imposed by law upon the defendant or respondent to perform the act
Petitioners add that a public, transparent, and deliberative process is purportedly necessary required that the law specifically enjoins as a duty resulting from office, trust, or station.33
to allay the people's fears against "executive overreach." This concern allegedly cannot be
addressed by briefings in executive sessions given by representatives of the Executive Branch According to respondents, it is erroneous to assert that it is their ministerial duty to convene
to both Houses of the Congress.27 in joint session whenever martial law is proclaimed or the privilege of the writ of habeas
corpus is suspended in the absence of a clear and specific constitutional or legal provision. In
Petitioners further postulate that, based on the deliberations of the Members of the fact, Article VII, Section 18 does not use the words ''joint session" at all, much less impose the
ConCom, the phrase "voting jointly" under Article VII, Section 18 was intended to mean that convening of such joint session upon the proclamation of martial law or the suspension of
a joint session is a procedural requirement, necessary for the Congress to decide whether to the privilege of the writ of habeas corpus. What the Constitution requires is joint voting when
revoke, affirm, or even extend the declaration of martial law.28 the action of the Congress is to revoke or extend the proclamation or suspension.34

Consolidation of Respondents' Comments Indeed, prior concurrence of the Congress is not constitutionally required for the effectivity
of the proclamation or suspension. Quoting from the deliberations of the framers of the
Respondents assert firmly that there is no mandatory duty on their part to "vote jointly," Constitution pertaining to Article VII, Section 18, the Congress points out that it was the
except in cases of revocation or extension of the proclamation of martial law or the intention of the said framers to grant the President the power to declare martial law or
suspension of the privilege of the writ of habeas corpus.29 In the absence of such duty, the suspend the privilege of the writ of habeas corpus for a period not exceeding sixty (60) days
non-convening of the Congress in joint session does not pose any actual case or controversy without the concurrence of the Congress. There is absolutely nothing under the Constitution
that may be the subject of judicial review.30 Additionally, respondents argue that the that mandates the Congress to convene in joint session when their intention is merely to
petitions raise a political question over which the Court has no jurisdiction. discuss, debate, and/or review the factual and legal basis for the proclamation. That is why
the phrase "voting jointly" is limited only in case the Congress intends to revoke the
proclamation.35 In a situation where the Congress is not in session, the Constitution simply
Petitioners' avowal that they are citizens and taxpayers is allegedly inadequate to clothe
provides that the Congress must convene in accordance with its rules but does not state that
them with locus standi. Generalized interests, albeit accompanied by the assertion of a public
it must convene in joint session. Respondents further refer to the proper procedure for the
right, do not establish locus standi. Petitioners must show that they have a direct and
holding of joint sessions.
personal interest in the Congress' failure to convene in joint session, which they failed to
present herein. A taxpayer's suit is likewise proper only when there is an exercise of the
spending or taxing power of the Congress. However, in these cases, the funds used in the Respondents brush aside as mere obiter dictum the Court's pronouncement in
implementation of martial law in Mindanao are taken from those funds already appropriated the Fortun case that it is the duty of the Congress to convene upon the declaration of martial
by the Congress. Senator De Lima's averment of her locus standi as an incumbent member of law. That whether or not the Congress should convene in joint session in instances where it is
the legislature similarly lacks merit. Insofar as the powers of the Congress are not impaired, not revoking the proclamation was not an issue in that case. Moreover, the factual
there is no prejudice to each Member thereof; and even assuming arguendo that the circumstances in the Fortun case are entirely different from the present cases. The Congress
authority of the Congress is indeed compromised, Senator De Lima still does not have then issued a concurrent resolution calling for the convening of a joint session as the
standing to file the present petition for mandamus because it is not shown that she has been intention - at least as far as the Senate was concerned - was to revoke the proclamation of
allowed to participate in the Senate sessions during her incarceration. She cannot, therefore, martial law and the suspension of the privilege of the writ of habeas corpus in Maguindanao.
claim that she has suffered any direct injury from the non-convening of the Congress in joint The Fortun case then cannot be considered a legislative precedent of an "automatic
session.31 convening of a joint session by the Congress upon the President's proclamation of martial
law."36
Respondents further contend that the constitutional right to information, as enshrined under
Article III, Section 7 of the Constitution, is not absolute. Matters affecting national security Respondents argue that the remedy of certiorari is likewise unavailing. To justify judicial
are considered as a valid exception to the right to information of the public. For this reason, intervention, the abuse of discretion must be so patent and gross as to amount to an evasion
the petitioners' and the public's right to participate in the deliberations of the Congress of a positive duty or to a virtual refusal to perform a duty enjoined by law or to act at all in
300

contemplation of law, as where the power is exercised in an arbitrary and despotic manner THE COURT'S RULING
by reason of passion or hostility.37 The Congress has the duty to convene and vote jointly
only in two (2) instances, as respondents have already explained. The Congress had even The Court's jurisdiction over these consolidated petitions
issued their respective resolutions expressing their support to, as well as their intent not to
revoke, President Duterte's Proclamation No. 216. There then can be no evasion of a positive
The principle of separation of powers
duty or a virtual refusal to perform a duty on the part of the Congress if there is no duty to
begin with.38
The separation of powers doctrine is the backbone of our tripartite system of government. It
is implicit in the manner that our Constitution lays out in separate and distinct Articles the
Respondents respectfully remind the Court to uphold the "constitutional demarcation of the
powers and prerogatives of each co-equal branch of government. In Belgica v. Ochoa,41 this
three fundamental powers of government."39 The Court may not intervene in the internal
Court had the opportunity to restate:
affairs of the Legislature and it is not within the province of the courts to direct the Congress
how to do its work. Respondents stress that this Court cannot direct the Congress to convene
in joint session without violating the basic principle of the separation of powers.40 The principle of separation of powers refers to the constitutional demarcation of the three
fundamental powers of government. In the celebrated words of Justice Laurel in Angara v.
Electoral Commission, it means that the "Constitution has blocked out with deft strokes and
Subsequent Events
in bold lines, allotment of power to the executive, the legislative and the judicial
departments of the government." To the legislative branch of government, through
On July 14, 2017, petitioners in G.R. No. 231671, the Padilla Petition, filed a Manifestation, Congress, belongs the power to make laws; to the executive branch of government, through
calling the attention of the Court to the imminent expiration of the sixty (60)-day period of the President, belongs the power to enforce laws; and to the judicial branch of government,
validity of Proclamation No. 216 on July 22, 2017. Despite the lapse of said sixty (60)-day through the Court, belongs the power to interpret laws. Because the three great powers
period, petitioners exhort the Court to still resolve the instant cases for the guidance of the have been, by constitutional design, ordained in this respect, "[ e ]ach department of the
Congress, State actors, and all Filipinos. government has exclusive cognizance of matters within its jurisdiction, and is supreme within
its own sphere." Thus, "the legislature has no authority to execute or construe the law, the
On July 22, 2017, the Congress convened in joint session and, with two hundred sixty-one executive has no authority to make or construe the law, and the judiciary has no power to
(261) votes in favor versus eighteen (18) votes against, overwhelmingly approved the make or execute the law." The principle of separation of powers and its concepts of
extension of the proclamation of martial law and the suspension of the privilege of the writ autonomy and independence stem from the notion that the powers of government must be
of habeas corpus in Mindanao until December 31, 2017. divided to avoid concentration of these powers in any one branch; the division, it is hoped,
would avoid any single branch from lording its power over the other branches or the
STATEMENT OF THE ISSUES citizenry. To achieve this purpose, the divided power must be wielded by co-equal branches
of government that are equally capable of independent action in exercising their respective
mandates. Lack of independence would result in the inability of one branch of government to
After a meticulous consideration of the parties' submissions, we synthesize them into the
check the arbitrary or self-interest assertions of another or others. (Emphases supplied,
following fundamental issues:
citations omitted.)

I. Whether or not the Court has jurisdiction over the subject matter of these consolidated
Contrary to respondents' protestations, the Court's exercise of jurisdiction over these
petitions;
petitions cannot be deemed as an unwarranted intrusion into the exclusive domain of the
Legislature. Bearing in mind that the principal substantive issue presented in the cases at bar
II. Whether or not the petitions satisfy the requisites for the Court's exercise of its power of is the proper interpretation of Article VII, Section 18 of the 1987 Constitution, particularly
judicial review; regarding the duty of the Congress to vote jointly when the President declares martial law
and/or suspends the privilege of the writ of habeas corpus, there can be no doubt that the
III. Whether or not the Congress has the mandatory duty to convene jointly upon the Court may take jurisdiction over the petitions. It is the prerogative of the Judiciary to declare
President's proclamation of martial law or the suspension of the privilege of the writ "what the law is."42 It is worth repeating here that:
of habeas corpus under Article VII, Section 18 of the 1987 Constitution; and
[W]hen the judiciary mediates to allocate constitutional boundaries, it does not assert any
IV. Whether or not a writ of mandamus or certiorari may be issued in the present cases. superiority over the other departments; it does not in reality nullify or invalidate an act of the
legislature, but only asserts the solemn and sacred obligation assigned to it by the
301

Constitution to determine conflicting claims of authority under the Constitution and to On the other hand, in G.R. No. 231694, while petitioner Tañada sues in his capacity as a
establish for the parties in an actual controversy the rights which that instrument secures Filipino citizen and former legislator, his co-petitioners (Bishop Iniguez, Bishop Pabillo, Bishop
and guarantees to them. Tobias, Mo. Ygrubay, Bulangis, and Deluria) all sue in their capacity as Filipino citizens.

Political question doctrine Respondents insist that none of the petitioners have legal standing, whether as a citizen,
taxpayer, or legislator, to file the present cases.1avvphi1
Corollary to respondents' invocation of the principle of separation of powers, they argue that
these petitions involve a political question in which the Court may not interfere. It is true that The Court has consistently held that locus standi is a personal and substantial interest in a
the Court continues to recognize questions of policy as a bar to its exercise of the power of case such that the party has sustained or will sustain direct injury as a result of the
judicial review.44 However, in a long line of cases,45 we have given a limited application to the challenged governmental act. The question is whether the challenging party alleges such
political question doctrine. personal stake in the outcome of the controversy so as to assure the existence of concrete
adverseness that would sharpen the presentation of issues and illuminate the court in ruling
In The Diocese of Bacolod v. Commission on Elections,46 we emphasized that the Court's on the constitutional question posed.49
judicial power as conferred by the Constitution has been expanded to include "the duty of
the courts of justice to settle actual controversies involving rights which are legally Petitioners satisfy these standards.
demandable and enforceable, and to determine whether or not there has been a grave
abuse of discretion amounting to lack or excess of jurisdiction on the part of any branch or The Court has recognized that every citizen has the right, if not the duty, to interfere and see
instrumentality of the Government." Further, in past cases, the Court has exercised its power that a public offense be properly pursued and punished, and that a public grievance be
of judicial review noting that the requirement of interpreting the constitutional provision remedied.50 When a citizen exercises this "public right" and challenges a supposedly illegal or
involved the legality and not the wisdom of a manner by which a constitutional duty or unconstitutional executive or legislative action, he represents the public at large, thus,
power was exercised.47 clothing him with the requisite locus standi. He may not sustain an injury as direct and
adverse as compared to others but it is enough that he sufficiently demonstrates in his
In Association of Medical Clinics for Overseas Workers, Inc. (AMCOW) v. GCC Approved petition that he is entitled to protection or relief from the Court in the vindication of a public
Medical Centers Association, Inc.,48 we explained the rationale behind the Court's right.51
expanded certiorari jurisdiction. Citing former Chief Justice and Constitutional Commissioner
Roberto R. Concepcion in his sponsorship speech for Article VIII, Section 1 of the Verily, legal standing is grounded on the petitioner's personal interest in the controversy. A
Constitution, we reiterated that the courts cannot hereafter evade the duty to settle matters, citizen who files a petition before the court asserting a public right satisfies the requirement
by claiming that such matters constitute a political question. of personal interest simply because the petitioner is a member of the general public upon
which the right is vested.52 A citizen's personal interest in a case challenging an allegedly
Existence of the requisites for judicial review unconstitutional act lies in his interest and duty to uphold and ensure the proper execution
of the law.53
Petitioners' legal standing
The present petitions have been filed by individuals asserting that the Senate and the House
Petitioners in G.R. No. 231671 allege that they are suing in the following capacities: (1) of Representatives have breached an allegedly constitutional duty to convene in joint session
Padilla as a member of the legal profession representing victims of human rights violations, to deliberate on Presidential Proclamation No. 216. The citizen-petitioners' challenge of a
and a taxpayer; (2) Saguisag as a human rights lawyer, former member of the Philippine purportedly unconstitutional act in violation of a public right, done in behalf of the general
Senate, and a taxpayer; (3) Monsod as a framer of the Philippine Constitution and member of public, gives them legal standing.
the 1986 Con Com, and a taxpayer; (4) Rosales as a victim of human rights violations
committed under martial law declared by then President Ferdinand E. Marcos, and a On the other hand, Senator De Lima questions the Congress' failure to convene in joint
taxpayer; (5) Gorospe as a lawyer and a taxpayer; and (6) Senator De Lima as an incumbent session to deliberate on Proclamation No. 216, which, according to the petitioners, is the
Member of the Philippine Senate, a human rights advocate, a former Secretary of Justice, legislature's constitutional duty.
Chairperson of the Commission on Human Rights, and a taxpayer.
We have ruled that legislators have legal standing to ensure that the constitutional
prerogatives, powers, and privileges of the Members of the Congress remain
302

inviolate.54 Thus, they are allowed to question the validity of any official action - or in these There is no merit to respondents' position.
cases, inaction - which, to their mind, infringes on their prerogatives as legislators.55
For the Court to exercise its power of judicial review and give due course to the petitions, it is
Actual case or controversy sufficient that the petitioners set forth their material allegations to make out a prima
facie case for mandamus or certiorari.61 Whether the petitioners are actually and ultimately
It is long established that the power of judicial review is limited to actual cases or entitled to the reliefs prayed for is exactly what is to be determined by the Court after careful
controversies. There is an actual case or controversy where there is a conflict of legal rights, consideration of the parties' pleadings and submissions.
an assertion of opposite legal claims, where the contradiction of the rights can be interpreted
and enforced on the basis of existing law and jurisprudence.56 Liberality in cases of transcendental importance

There are two conflicting claims presented before the Court: on the one hand, the In any case, it is an accepted doctrine that the Court may brush aside procedural
petitioners' assertion that the Congress has the mandatory duty to convene in joint session technicalities and, nonetheless, exercise its power of judicial review in cases of
to deliberate on Proclamation No. 216; and, on the other, the respondents' view that so transcendental importance.
convening in joint session is discretionary on the part of the Congress.
There are marked differences between the Chief Executive's military powers, including the
Petitioners seek relief through a writ of mandamus and/or certiorari. Mandamus is a remedy power to declare martial law, as provided under the present Constitution, in comparison to
granted by law when any tribunal, corporation, board, officer, or person unlawfully neglects that granted in the 1935 Constitution. Under the 1935 Constitution,62 such powers were
the performance of an act which the law specifically enjoins as a duty resulting from an seemingly limitless, unrestrained, and purely subject to the President's wisdom and
office, trust, or station, or unlawfully excludes another from the use or enjoyment of a right discretion.
or office to which such other is entitled.57 Certiorari, as a special civil action, is available only
if: (1) it is directed against a tribunal, board, or officer exercising judicial or quasi-judicial At present, the Commander-in-Chief still possesses the power to suspend the privilege of the
functions; (2) the tribunal, board, or officer acted without or in excess of jurisdiction or with writ of habeas corpus and to proclaim martial law. However, these executive powers are now
grave abuse of discretion amounting to lack or excess of jurisdiction; and (3) there is no subject to the review of both the legislative and judicial branches. This check-and-balance
appeal nor any plain, speedy, and adequate remedy in the ordinary course of law.58 With mechanism was installed in the 1987 Constitution precisely to prevent potential abuses of
respect to the Court, however, certiorari is broader in scope and reach, and it may be issued these executive prerogatives.
to correct errors of jurisdiction committed not only by a tribunal, corporation, board, or
officer exercising judicial, quasi-judicial, or ministerial functions, but also to set right, undo,
Inasmuch as the present petitions raise issues concerning the Congress' role in our
and restrain any act of grave abuse of discretion amounting to lack or excess of jurisdiction
government's system of checks and balances, these are matters of paramount public interest
by any branch or instrumentality of the Government, even if the latter does not exercise
or issues of transcendental importance deserving the attention of the Court in view of their
judicial, quasi-judicial or ministerial functions.59
seriousness, novelty, and weight as precedents.63

As the present petitions allege an omission on the part of the Congress that constitutes
Mootness
neglect of their constitutional duties, the petitions make a prima facie case
for mandamus, and an actual case or controversy ripe for adjudication exists. When an act or
omission of a branch of government is seriously alleged to have infringed the Constitution, it The Court acknowledges that the main relief prayed for in the present petitions (i.e., that the
becomes not only the right but, in fact, the duty of the judiciary to settle the dispute. 60 Congress be directed to convene in joint session and therein deliberate whether to affirm or
revoke Proclamation No. 216) may arguably have been rendered moot by: (a) the lapse of
the original sixty (60) days that the President's martial law declaration and suspension of the
Respondents aver that the Congress cannot be compelled to do something that is
privilege of the writ of habeas corpus were effective under Proclamation No. 216; (b) the
discretionary on their part nor could they be guilty of grave abuse of discretion in the
subsequent extension by the Congress of the proclamation of martial law and the suspension
absence of any mandatory obligation to jointly convene on their part to affirm the
of the privilege of the writ of habeas corpus over the whole of Mindanao after convening in
President's proclamation of martial law. Thus, petitioners are not entitled to the reliefs
joint session on July 22, 2017; and (c) the Court's own decision in Lagman v.
prayed for in their petitions for mandamus and/or certiorari; consequently, no actual case or
Medialdea,64 wherein we ruled on the sufficiency of the factual bases for Proclamation No.
controversy exists.
216 under the original period stated therein.
303

In David v. Macapagal-Arroyo, the jurisprudential rules regarding mootness were succinctly of the writ of habeas corpus or place the Philippines or any part thereof under martial law.
summarized, thus: Within forty-eight hours from the proclamation of martial law or the suspension of the
privilege of the writ of habeas corpus, the President shall submit a report in person or in
A moot and academic case is one that ceases to present a justiciable controversy by virtue of writing to the Congress. The Congress, voting jointly, by a vote of at least a majority of all its
supervening events, so that a declaration thereon would be of no practical use or value. Members in regular or special session, may revoke such proclamation or suspension which
Generally, courts decline jurisdiction over such case or dismiss it on ground of mootness. revocation shall not be set aside by the President. Upon the initiative of the President, the
Congress may, in the same manner, extend such proclamation or suspension for a period to
be determined by the Congress, if the invasion or rebellion shall persist and public safety
The "moot and academic" principle is not a magical formula that can automatically dissuade
requires it.
the courts in resolving a case. Courts will decide cases, otherwise moot and academic, if:
first, there is a grave violation of the Constitution; second, the exceptional character of the
situation and the paramount public interest is involved; third, when constitutional issue The Congress, if not in session, shall, within twenty-four hours following such proclamation
raised requires formulation of controlling principles to guide the bench, the bar, and the or suspension, convene in accordance with its rules without need of a call.
public; and fourth, the case is capable of repetition yet evading review.65 (Emphasis supplied,
citations omitted.) The Supreme Court may review, in an appropriate proceeding filed by any citizen, the
sufficiency of the factual basis of the proclamation of martial law or the suspension of the
It cannot be gainsaid that there are compelling and weighty reasons for the Court to proceed privilege of the writ or the extension thereof, and must promulgate its decision thereon
with the resolution of these consolidated petitions on the merits. As explained in the within thirty days from its filing.
preceding discussion, these cases involve a constitutional issue of transcendental significance
and novelty. A definitive ruling from this Court is imperative not only to guide the Bench, the A state of martial law does not suspend the operation of the Constitution, nor supplant the
Bar, and the public but, more importantly, to clarify the parameters of congressional conduct functioning of the civil courts or legislative assemblies, nor authorize the conferment of
required by the 1987 Constitution, in the event of a repetition of the factual precedents that jurisdiction on military courts and agencies over civilians where civil courts are able to
gave rise to these cases. function, nor automatically suspend the privilege of the writ.

The duty of the Congress to vote jointly under Article VII, Section 18 The suspension of the privilege of the writ shall apply only to persons judicially charged for
rebellion or offenses inherent in or directly connected with invasion.
We now come to the crux of the present petitions - the issue of whether or not under Article
VII, Section 18 of the 1987 Constitution, it is mandatory for the Congress to automatically During the suspension of the privilege of the writ, any person thus arrested or detained shall
convene in joint session in the event that the President proclaims a state of martial law be judicially charged within three days, otherwise he shall be released. (Emphasis supplied.)
and/or suspends the privilege of the writ of habeas corpus in the Philippines or any part
thereof. Outside explicit constitutional limitations, the Commander-in-Chief clause in Article VII,
Section 18 of the 1987 Constitution vests on the President, as Commander-in-Chief, absolute
The Court answers in the negative. The Congress is not constitutionally mandated to convene authority over the persons and actions of the members of the armed forces,66 in recognition
in joint session except to vote jointly to revoke the President's declaration or suspension. that the President, as Chief Executive, has the general responsibility to promote public peace,
and as Commander-in-Chief, the more specific duty to prevent and suppress rebellion and
By the language of Article VII, Section 18 of the 1987 Constitution, the Congress is only lawless violence.67 However, to safeguard against possible abuse by the President of the
required to vote jointly to revoke the President's proclamation of martial law and/or exercise of his power to proclaim martial law and/or suspend the privilege of the writ
suspension of the privilege of the writ of habeas corpus. of habeas corpus, the 1987 Constitution, through the same provision, institutionalized checks
and balances on the President's power through the two other co-equal and independent
branches of government, i.e., the Congress and the Judiciary. In particular, Article VII, Section
Article VII, Section 18 of the 1987 Constitution fully reads:
18 of the 1987 Constitution requires the President to submit a report to the Congress after
his proclamation of martial law and/or suspension of the privilege of the writ of habeas
Sec. 18. The President shall be the Commander-in-Chief of allarmed forces of the Philippines corpus and grants the Congress the power to revoke, as well as extend, the proclamation
and whenever it becomes necessary, he may call out such armed forces to prevent or and/or suspension; and vests upon the Judiciary the power to review the sufficiency of the
suppress lawless violence, invasion or rebellion. In case of invasion or rebellion, when the factual basis for such proclamation and/or suspension.
public safety requires it, he may, for a period not exceeding sixty days, suspend the privilege
304

There are four provisions in Article VII, Section 18 of the 1987 Constitution specifically In Funa v. Chairman Villar,70 the Court also applied the verba legis rule in constitutional
pertaining to the role of the Congress when the President proclaims martial law and/or construction, thus:
suspends the privilege of the writ of habeas corpus, viz.:
The rule is that if a statute or constitutional provision is clear, plain and free from ambiguity,
a. Within forty-eight (48) hours from the proclamation of martial law or the suspension of it must he given its literal meaning and applied without attempted interpretation. This is
the privilege of the writ of habeas corpus, the President shall submit a report in person or in known as the plain meaning rule enunciated by the maxim verba legis non est
writing to the Congress; recedendum, or from the words of a statute there should be no departure.

b. The Congress, voting jointly, by a vote of at least a majority of all its Members in regular or The primary source whence to ascertain constitutional intent or purpose is the language of
special session, may revoke such proclamation or suspension, which revocation shall not be the provision itself. If possible, the words in the Constitution must be given their ordinary
set aside by the President; meaning, save where technical terms are employed. J.M. Tuason & Co., Inc. v. Land Tenure
Administration illustrates the verbal legis rule in this wise:
c. Upon the initiative of the_ President, the Congress may, in the same manner. extend such
proclamation or suspension for a period to be determined by the Congress, if the invasion or We look to the language of the document itself in our search for its meaning. We do not of
rebellion shall persist; and course stop there, but that is where we begin. It is to he assumed that the words in which
constitutional provisions arc couched express the objective sought to be attained. They are
d. The Congress, if not in session, shall within twenty-four hours (24) following such to be given their ordinary meaning except where technical terms are employed in which case
proclamation or suspension, convene in accordance with its rules without need of call. the significance thus attached to them prevails. As the Constitution is not primarily a lawyer's
document, it being essential for the rule of law to obtain that it should ever be present in the
people's consciousness, its language as much as possible should be understood in the sense
There is no question herein that the first provision was complied with, as within forty-eight
they have in common use. What it says according to the text of the provision to be construed
(48) hours from the issuance on May 23, 2017 by President Duterte of Proclamation No. 216,
compels acceptance and negates the power of the courts to alter it. based on the postulate
declaring a state of martial law and suspending the privilege of the writ of habeas corpus in
that the framers and the people mean what they say. Thus there are cases where the need
Mindanao, copies of President Duterte's Report relative to Proclamation No. 216 was
for construction is reduced to a minimum. (Emphases supplied.)
transmitted to and received by the Senate and the House of Representatives on May 25,
2017.
The provision in question is clear, plain, and unambiguous. In its literal and ordinary meaning,
the provision grants the Congress the power to revoke the President's proclamation of
The Court will not touch upon the third and fourth provisions as these concern factual
martial law or the suspension of the privilege of the writ of habeas corpus and prescribes
circumstances which are not availing in the instant petitions. The petitions at bar involve the
how the Congress may exercise such power, i.e., by a vote of at least a majority of all its
initial proclamation of martial law and suspension of the privilege of the writ of habeas
Members, voting jointly, in a regular or special session. The use of the word "may" in the
corpus, and not their extension; and the 17th Congress was still in session68 when President
provision - such that "[t]he Congress x x x may revoke such proclamation or suspension x x x"
Duterte issued Proclamation No. 216 on May 23, 2017.
- is to be construed as permissive and operating to confer discretion on the Congress on
whether or not to revoke,71 but in order to revoke, the same provision sets the requirement
It is the second provision that is under judicial scrutiny herein: "The Congress, voting jointly, that at least a majority of the Members of the Congress, voting jointly, favor revocation.
by a vote of at least a majority of all its Members in regular or special session, may revoke
such proclamation or suspension, which revocation shall not be set aside by the President."
It is worthy to stress that the provision does not actually refer to a "joint session." While it
may be conceded, subject to the discussions below, that the phrase "voting jointly" shall
A cardinal rule in statutory construction is that when the law is clear and free from any doubt already be understood to mean that the joint voting will be done "in joint session,"
or ambiguity, there is no room for construction or interpretation. There is only room for notwithstanding the absence of clear language in the Constitution, 72 still, the requirement
application. According to the plain-meaning rule or verba legis, when the statute is clear, that "[t]he Congress, voting jointly, by a vote of at least a majority of all its Members in
plain, and free from ambiguity, it must be given its literal meaning and applied without regular or special session, x x x" explicitly applies only to the situation when the Congress
attempted interpretation. It is expressed in the maxims index animi sermo or "speech is the revokes the President's proclamation of martial law and/or suspension of the privilege of the
index of intention[,]" and verba legis non est recedendum or "from the words of a statute writ of habeas corpus. Simply put, the provision only requires Congress to vote jointly on the
there should be no departure."69 revocation of the President's proclamation and/or suspension.
305

Hence, the plain language of the subject constitutional provision does not support the framers to make it mandatory for the Congress to convene in joint session following the
petitioners' argument that it is obligatory for the Congress to convene in joint session President's proclamation and/or suspension, so it could deliberate as a single body,
following the President's proclamation of martial law and/or suspension of the privilege of regardless of whether its Members will concur in or revoke the President's proclamation
the writ of habeas corpus, under all circumstances. and/or suspension.

The deliberations of the 1986 ConCom reveal the framers' specific intentions to (a) remove What is evident in the deliberations of the 1986 ConCom were the framers' intentions to (a)
the requirement of prior concurrence of the Congress for the effectivity of the President's remove the requirement of prior concurrence by the Congress for the effectivity of the
proclamation of martial law and/or suspension of the privilege of the writ of habeas President's proclamation of martial law and/or suspension of the privilege of the writ
corpus; and (b) grant to the Congress the discretionary power to revoke the President's of habeas corpus; and (b) grant to the Congress the discretionary power to revoke the
proclamation and/or suspension by a vote of at least a majority of its Members, voting President's proclamation and/or suspension by a vote of at least a majority of its Members,
jointly. voting jointly.

The Court recognized in Civil Liberties Union v. The Executive Secretary73 that: As the Commander-in-Chief clause was initially drafted, the President's suspension of the
privilege of the writ of habeas corpus required the prior concurrence of at least a majority of
A foolproof yardstick in constitutional construction is the intention underlying the provision all the members of the Congress to be effective. The first line read, "The President shall be
under consideration. Thus, it has been held that the Court in construing a Constitution should the commander-in-chief of all the armed forces of the Philippines and, whenever it becomes
bear in mind the object sought to be accomplished by its adoption, and the evils, if any, necessary, he may call out such armed forces to prevent or suppress lawless violence,
sought to be prevented or remedied. A. doubtful provision will be examined in the light of invasion or rebellion[;]" and the next line, "In case of invasion or rebellion, when the public
the history of the times, and the condition and circumstances under which the Constitution safety requires it, he may, for a period not exceeding sixty days, and, with the concurrence of
was framed. The object is to ascertain the reason which induced· the framers of the at least a majority of all the members of the Congress, suspend the privilege of the writ
Constitution to enact the particular provision and the purpose sought to be accomplished of habeas corpus."75
thereby, in order to construe the whole as to make the words consonant to that reason and
calculated to effect that purpose. The Commissioners, however, extensively debated on whether or not there should be prior
concurrence by the Congress, and the exchanges below present the considerations for both
However, in the same Decision, the Court issued the following caveat: sides:

While it is permissible in this jurisdiction to consult the debates and proceedings of the MR. NATIVIDAD. First and foremost, we agree with the Commissioner's thesis that in the first
constitutional convention in order to arrive at the reason and purpose of the resulting imposition of martial law there is no need for concurrence of the majority of the Members of
Constitution, resort thereto may be had only when other guides fail as said proceedings are Congress because the provision says "in case of actual invasion and rebellion." If there is
powerless to vary the terms of the Constitution when the meaning is clear. Debates in the actual invasion and rebellion, as Commissioner Crispino de Castro said, there is need for
constitutional convention "are of value as showing the views of the individual members, and immediate response because there is an attack. Second, the fact of securing a concurrence
as indicating the reasons for their votes, but they give US no light as to the views. of the large may be impractical because the roads might be blocked or barricaded. They say that in case
majority who did not talk, much less of the mass of our fellow citizens whose votes at the of rebellion, one cannot even take his car and go to the Congress, which is possible because
polls gave that instrument the force of fundamental law. We think it safer to construe the the roads are blocked or barricaded. And maybe if the revolutionaries are smart they would
constitution from what appears upon its face.'' The proper interpretation therefore depends have an individual team for each and every Member of the Congress so he would not be able
more on how it was understood by the people adopting it than in the framer's understanding to respond to a call for a session. So the requirement of an initial concurrence of the majority
thereof.74 (Emphasis supplied.) of all the Members of the Congress in case of an invasion or rebellion might be impractical as
I can see it.
As the Court established in its preceding discussion, the clear meaning of the relevant
provision in Article VU, Section 18 of the 1987 Constitution is that the Congress is only Second, Section l5states that the Congress may revoke the declaration or lift the suspension.
required to vote jointly on the revocation of the President's proclamation of martial law
and/or suspension of the privilege of the writ of habeas co1pus. Based on the Civil Liberties And third, the matter of declaring martial law is already a justiciable question and no longer a
Union case, there is already no need to look beyond the plain language of the provision and political one in that it is subject to judicial review at any point in time. So on that basis, I
decipher the intent of the framers of the 1987 Constitution. Nonetheless, the deliberations agree that there is no need for concurrence as aprerequisite to declare martial law or to
on Article VII, Section 18 of the 1986 ConCom does not reveal a manifest intent of the suspend the privilege of the writ of habeas corpus. x x x
306

MR. SUAREZ. x x x Article VII, Section 10(2) of the Constitution, wherein he made this predicate under the
"Whereas" provision.
The Commissioner is suggesting that in connection with Section 15, we delete the phrase
"and, with the concurrence of at least a majority of all the Members of the Congress..." Whereas, the rebellion and armed action undertaken by these lawless elements of the
Communists and other armed aggrupations organized to overthrow the Republic of the
MR. PADILLA. That is correct especially for the initial suspension of the privilege of the writ Philippines by armed violence and force, have assumed the magnitude of an actual state of
of habeas corpus or also the declaration of martial law. war against our people and the Republic of the Philippines.

MR. SUAREZ. So in both instances, the Commissioner is suggesting that .this would be an And may I also call the attention of the Gentleman to General Order No. 3, also promulgated
exclusive prerogative of the President? by Ferdinand E. Marcos, in his capacity as Commander-in-Chief of all the Armed Forces of the
Philippines and pursuant to Proclamation No. 1081 dated September 21, 1972 wherein he
said, among other things:
MR. PADILLA. At least initially, for a period of 60 days. But even that period of 60 days may
be shortened by the Congress or the Senate because the next sentence says that the
Congress or the Senate may even revoke the proclamation. Whereas, martial law having been declared because of wanton destruction of lives and
properties, widespread lawlessness and anarchy and chaos and disorder now prevailing
throughout the country, which condition has been brought about by groups of men who are
MR. MONSOD. x x x
actively engaged in a criminal conspiracy to seize political and state power in the Philippines
in order to take over the government by force and violence, the extent of which has now
We are back to Section 15, page 7, lines 1 and 2. I just want to reiterate my previous proposal assumed the proportion of an actual war against our people and the legitimate
to amend by deletion the phrase "and, with the concurrence of at least a majority of all the government...
members of Congress."
And he gave all reasons in order to suspend the privilege of the writ of habeas corpus and
MR. SUAREZ. x x x declare martial law in our country without justifiable reason. Would the Gentleman still insist
on the deletion of the phrase "and, with the concurrence of at least a majority of all the
The Commissioner is proposing a very substantial amendment because this means that he is members of the Congress"?
vesting exclusively unto the President the right to determine the factors which may lead to
the declaration of martial law and the suspension of the writ of habeas corpus. I suppose he MR. MONSOD. Yes, Madam President, in the case of Mr. Marcos he is undoubtedly an
has strong and compelling reasons in seeking to delete this particular phrase. May we be aberration in our history and national consciousness. But given the possibility that there
informed of his good and substantial reasons? would be another Marcos, our Constitution now has sufficient safeguards. As I said, it is not
really true, as the Gentleman has mentioned, that there is an exclusive right to determine
MR. MONSOD. This situation arises in cases of invasion or rebellion. And in previous the factual bases because the paragraph beginning on line 9 precisely tells us that the
interpellatioi1s regarding this phrase, even during the discussions on the Bill of Rights, as I Supreme Court may review, in an appropriate proceeding filed by any citizen, the sufficiency
understand it, the interpretation is a situation of actual invasion or rebellion. In these of the factual basis of the proclamation of martial law or the suspension of the privilege of
situations, the President has to act quickly. Secondly, this declaration has a time fuse. It is the writ or the extension thereof and must promulgate its decision on the same within 30
only good for a maximum of 60 days. At the end of 60 days, it automatically terminates. days from its filing.
Thirdly, the right of the judiciary to inquire into the sufficiency of the factual basis of the
proclamation always exists, even during those first 60 days. I believe that there are enough safeguards. The Constitution is supposed to balance the
interests of the country. And here we are trying to balance the public interest in case of
MR. SUAREZ. Given cur traumatic experience during the past administration, if we give invasion or rebellion as against the rights of citizens. And I am saying that there are enough
exclusive right to the President to determine these factors, especially the existence of an safeguards, unlike in 1972 when Mr. Marcos was able to do all those things mentioned.
invasion or rebellion and the second factor of determining whether the public safety requires
it or not, may I call the attention of the Gentleman to what happened to us during the past ac MR. SUAREZ. Will that prevent a future President from doing what Mr. Marcos had done?
ministration. Proclamation No. 1081 was issued by Ferdinand E. Marcos in his capacity as
President of the Philippines by virtue of the powers vested upon him purportedly under
307

MR. MONSOD. There is nothing absolute in this world, and there may be another Marcos. As the result of the foregoing, the 1987 Constitution does not provide at all for the manner of
What we are looking for are safeguards that are reasonable and, I believe, adequate at this determination and expression of concurrence (whether prior or subsequent) by the Congress
point. On the other hand, in case of invasion or rebellion, even during the first 60 days when in the President's proclamation of martial law and/or suspension of the privilege of the writ
the intention here is to protect the country in that situation, it would be unreasonable to ask of habeas corpus. In the instant cases, both Houses of the Congress separately passed
that there should be a concurrence on the part of the Congress, which situation is resolutions, in accordance with their respective rules of procedure, expressing their support
automatically terminated at the end of such 60 days. for President Duterte's Proclamation No. 216.

MR. SUAREZ. Would the Gentleman not feel more comfortable if we provide for a legislative In contrast, being one of the constitutional safeguards against possible abuse by the
check on this awesome power of the Chief Executive acting as Commander-in-Chief? President of his power to proclaim martial law and/or suspend the privilege of the writ
of habeas corpus, the 1987 Constitution explicitly provides for how the Congress may
MR. MONSOD. I would be less comfortable if we have a presidency that cannot act under exercise its discretionary power to revoke the President's proclamation and/or suspension,
those conditions. that is, "voting jointly, by a vote of at least a majority of all its Members in regular or special
session."
MR. SUAREZ. But he can act with the concurrence of the proper or appropriate authority.
The ConCom deliberations on this particular provision substantially revolved around whether
the two Houses will have to vote jointly or separately to revoke the President's proclamation
MR. MONSOD. Yes. But when those situations arise, it is very unlikely that the concurrence of
of martial law and/or suspension of the privilege of the writ of habeas corpus; but as the
Congress would be available; and, secondly, the President will be able to act quickly in order
Court reiterates, it is undisputedly for the express purpose of revoking the President's
to deal with the circumstances.
proclamation and/or suspension.

MR. SUAREZ. So, we would be subordinating actual circumstances to expediency.


Based on the ConCom deliberations, pertinent portions of which are reproduced hereunder,
the underlying reason for the requirement that the two Houses of the Congress will vote
MR. MONSOD. I do not believe it is expediency when one is trying to protect the country in jointly is to avoid the possibility of a deadlock and to facilitate the process of revocation of
the event of an invasion or a rebellion. the President's proclamation of martial law and/or suspension of the privilege of the writ
of habeas corpus:
MR. SUAREZ. No. But in both instances, we would be seeking to protect not only the country
but the rights of simple citizens. We have to balance these interests without sacrificing the MR. MONSOD. Madam President, I want to ask the Committee a clarifying question on line 4
security of the State. of page 7 as to whether the meaning here is that the majority of all the Members of each
House vote separately. Is that the intent of this phrase?
MR. MONSOD. I agree with the Gentleman that is why in the Article on the Bill of Rights,
which was approved on Third Reading, the safeguards and the protection of the citizens have FR. BERNAS. We would like a little discussion on that because yesterday we already removed
been strengthened. And on line 21 of this paragraph, I endorsed the proposed amendment of the necessity for concurrence of Congress for the initial imposition of martial law. If we
Commissioner Padilla. We are saying that those who are arrested should be judicially charged require the Senate and the House of Representatives to vote separately for purposes of
within five days; otherwise, they shall be released. So, there are enough safeguards. revoking the imposition of martial law, that will make it very difficult for Congress to revoke
the imposition of martial law and the suspension of the privilege of the writ of habeas
MR. SUAREZ. These are safeguards after the declaration of martial law and after the corpus. That is just thinking aloud. To balance the fact that the President acts unilaterally,
suspension of the writ of habeas corpus. then the Congress voting as one body and not separately can revoke the declaration of
martial law or the suspension of the privilege of the writ of habeas corpus.
MR. MONSOD. That is true.76 (Emphases supplied.)
MR. MONSOD. In other words, voting jointly.
Ultimately, twenty-eight (28) Commissioners voted to remove the requirement for prior
concurrence by the Congress for the effectivity of the President's proclamation of martial law FR. BERNAS. Jointly, yes.
and/or suspension of the privilege of the writ of habeas corpus, against only twelve (12)
Commissioners who voted to retain it.
308

MR. RODRIGO. May I comment on the statement made by Commissioner Bernas? I was a In connection with the inquiry of Commissioner Monsod, and considering the statements
Member of the Senate for 12 years. Whenever a bicameral Congress votes, it is always made by Commissioner Rodrigo, I would like to say, in reply to Commissioner Bernas, that
separately. perhaps because of necessity, we might really have to break tradition. Perhaps it would be
better to give this function of revoking the proclamation of martial law or the suspension of
For example, bills coming. from the Lower House are voted upon by the Members of the the writ or extending the same to the House of Representatives, instead of to the Congress. I
House. Then they go up to the Senate and voted upon separately. Even on constitutional feel that even the Senators would welcome this because they would feel frustrated by the
amendments, where Congress meets in joint session, the two Houses vote separately. imbalance in the number between the Senators and the Members of the House of
Representatives.
Otherwise, the Senate will be useless; it will be sort of absorbed by the House considering
that the Members of the Senate are completely outnumbered by the Members of the House. Anyway, Madam President, we have precedents or similar cases. For example, under Section
So, I believe that whenever Congress acts, it must be the two Houses voting separately. 24 of the committee report on the Legislative, appropriation, revenue or tariff bills, and bills
authorizing increase of public debt are supposed to originate exclusively in the House of
Representatives. Besides, we have always been saying that it is the Members of the House of
If the two Houses vote "jointly," it would mean mixing the 24 Senators with 250
Representatives who are mostly in touch with the people since they represent the various
Congressmen. This would result in the Senate being absorbed and controlled by the House.
districts of our country.
This violates the purpose of having a Senate.

MR. MONSOD. I would prefer to have the vote of both Houses because this is a very serious
FR. BERNAS. I quite realize that that is the practice and, precisely, in proposing this, I am
question that must be fully discussed. By limiting it alone to the House of Representatives,
consciously proposing this as an exception to this practice because of the tremendous effect
then we lose the benefit of the advice and opinion of the Members of the Senate. I would
on the nation when the privilege of the writ of habeas corpus is suspended and then martial
prefer that they would be in joint session, but I would agree with Father Bernas that they
law is imposed. Since we have allowed the President to impose martial law and suspend the
should not be voting separately as part of the option. I think they should be voting jointly, so
privilege of the writ of habeas corpus unilaterally, we should make it a little more easy for
that, in effect, the Senators will have only one vote. But at least we have the benefit of their
Congress to reverse such actions for the sake of protecting the rights of the people.
advice.

MR. RODRIGO. Maybe the way it can be done is to vest this function in just one of the
MR. RODRIGO. I was the one who proposed that the two Houses vote separately because if
Chambers - to the House alone or to the Senate alone. But to say, "by Congress," both House
they vote jointly, the Senators are absolutely outnumbered. It is insulting to the intelligence
and Senate "voting" jointly is practically a vote by the House.
of the Senators to join a session where they know they are absolutely outnumbered.
Remember that the Senators are elected at large by the whole country. The Senate is a
FR. BERNAS. I would be willing to say just the vote of the House. separate Chamber. The Senators have a longer term than the Members of the House; they
have a six-year term. They are a continuing Senate. Out of 24, twelve are elected every year.
MR. RODRIGO. That is less insulting to the Senate. However, there are other safeguards. For So, if they will participate at all, the Senate must vote separately. That is the practice
example, if, after 60 days the Congress does not act, the effectiveness of the declaration of everywhere where there are two chambers. But as I said, between having a joint session of
martial law or the suspension of the privilege of the writ ceases. Furthermore, there is the Senate and the House voting jointly where it is practically the House that will decide
recourse to the Supreme Court. alone, the lesser of two evils is just to let the House decide alone instead of insulting the
Senators by making them participate in a charade.
FR. BERNAS. I quite realize that there is this recourse to the Supreme Court and there is a
time limit, but at the same time because of the extraordinary character of this event when MR. REGALADO. May the Committee seek this clarification from Commissioner Rodrigo? This
martial law is imposed, I would like to make it easier for the representatives of the people to vC1ting is supposed to revoke the proclamation of martial Jaw. If the two Houses vote
review this very significant action taken by the President. separately and a majority is obtained in the House of Representatives for the revocation of
the proclamation of martial law but that same majority cannot be obtained in the Senate
MR. RODRIGO. Between the Senate being absorbed and controlled by the House numerically voting separately, what would be the situation?
and the House voting alone, the lesser of two evils is the latter.
MR. RODRIGO. Then the proclamation of martial law or the suspension continues for almost
MR. GUINGONA. two months. After two months, it stops. Besides, there is recourse to the Supreme Court.
309

MR. REGALADO. Therefore, that arrangement would be very difficult for the legislative since MR. MONSOD. The Gentleman is making an assumption that they will vote against each
they are voting separately and, for lack of majority in one of the Houses they are precluded other. I believe that they will discuss, probably in joint session and vote on it; then the
from revoking that proclamation. They will just, therefore, have to wait until the lapse of 60 consensus will be clear.
days.
MR. NOLLEDO. Madam President, the purpose of the amendment is really to set forth a
MR. RODRIGO. It might be difficult, yes. But remember, we speak of the Members of limitation because we have to avoid a stalemate. For example, the Lower House decides that
Congress who are elected by the people. Let us not forget that the President is also elected the declaration of martial law should be revoked, and that later on, the Senate sitting
by the people. Are we forgetting that the President is elected by the people? We seem to separately decides that it should not be revoked. It becomes inevitable that martial law shall
distrust all future Presidents just because one President destroyed our faith by his continue even if there should be no factual basis for it.
declaration of martial law. I think we are overreacting. Let us not judge all Presidents who
would henceforth be elected by the Filipino people on the basis of the abuses made by that MR. OPLE. Madam President, if this amendment is adopted, we will be held responsible for a
one President. Of course, we must be on guard; but let us not overreact. glaring inconsistency in the Constitution to a degree that it distorts the bicameral system that
we have agreed to adopt. I reiterate: If there are deadlocks, it is the responsibility of the
Let me make my position clear. I am against the proposal to make the House and the Senate presidential leadership, together with the leaders of both Houses, to overcome
vote jointly. That is an insult to the Senate. them.77 (Emphases supplied.)

MR. RODRIGO. Will the Gentleman yield to a question? When the matter was put to a vote, twenty-four (24) Commissioners voted for the two
Houses of the Congress "voting jointly" in the revocation of the President's proclamation of
MR. MONSOD. Yes, Madam President. martial law and/or suspension of the privilege of the writ of habeas corpus, and thirteen (13)
Commissioners opted for the two Houses "voting separately."
MR. RODRIGO. So, in effect, if there is a joint session composed of 250 Members of the
House plus 24 Members of the Senate, the total would be 274. The majority would be one- Yet, there was another attempt to amend the provision by requiring just the House of
half plus one. Representatives, not the entire Congress, to vote on the revocation of the President's
proclamation of martial law and/or suspension of the privilege of the writ of habeas corpus:
MR. MONSOD. So, 148 votes.
MR. RODRIGO. Madam President, may I propose an amendment?
MR. RODRIGO. And the poor Senators would be absolutely absorbed and outnumbered by
the 250 Members of the House. Is that it? MR. RODRIGO. On Section 15, page 7, line 4, I propose to change the word "Congress" to
HOUSE OF REPRESENTATIVES so that the sentence will read: "The HOUSE OF
REPRESENTATIVES, by a vote of at least a majority of all its Members in regular or special
MR. MONSOD. Yes, that is one of the implications of the suggestion and the amendment is
session, may revoke such proclamation or suspension or extend the same if the invasion or
being made nonetheless because there is a higher objective or value which is to prevent a
rebellion shall persist and public safety requires it."
deadlock that would enable the President to continue the full 60 days in case one House
revokes and the other House does not.
FR. BERNAS. Madam President, the proposed amendment is really a motion for
reconsideration. We have already decided that both Houses will vote jointly. Therefore, the
The proposal also allows the Senators to participate fully in the discussions and whether we
proposed amendment, in effect, asks for a reconsideration of that vote in order to give it to
like it or not, the Senators have very large persuasive powers because of their prestige and
the House of Representatives.
their national vote.

MR. RODRIGO. Madam President, the opposite of voting jointly is voting separately. If my
MR. RODRIGO. So, the Senators will have the "quality votes" but Members of the House will
amendment were to vote separately, then, yes, it is a motion for reconsideration. But this is
have the "quantity votes." Is that it?
another formula.

MR. DE CASTRO. What is the rationale of the amendment?


310

MR. RODRIGO. It is intended to avoid that very extraordinary and awkward provision which The provision in Article VII, Section 18 of the 1987 Constitution requiring the Congress to
would make the 24 Senators meet jointly with 250 Members of the House and make them vote jointly in a joint session is specifically for the purpose of revocation of the President's
vote jointly. What I mean is, the 24 Senators, like a drop in the bucket, are absorbed proclamation of martial law and/or suspension of the privilege of the writ of habeas
numerically by the 250 Members of the House. corpus. In the petitions at bar, the Senate and House of Representatives already separately
adopted resolutions expressing support for President Duterte's Proclamation No. 216. Given
MR. SARMIENTO. Madam President, we need the wisdom of the Senators. What is at stake is the express support of both Houses of the Congress for Proclamation No. 216, and their
the future of our country - human rights and civil liberties. If we separate the Senators, then already evident lack of intent to revoke the same, the provision in Article VII, Section 18 of
we deprive the Congressmen of the knowledge and experience of these 24 men. I think we the 1987 Constitution on revocation did not even come into operation and, therefore, there
should forget the classification of "Senators" or "Congressmen." We should all work together is no obligation on the part of the Congress to convene in joint session.
to restore democracy in our country. So we need the wisdom of 24 Senators.
Practice and logic dictate that a collegial body will first hold a meeting among its own
MR. RODRIGO. Madam President, may I just answer. This advice of the 24 Senators can be members to get a sense of the opinions of its individual members and, if possible and
sought because they are in the same building. Anyway, the provision, with the amendment necessary, reach an official stance, before convening with another collegial body. This is
of Commissioner Monsod, does not call for a joint session. It only says: "the Congress, by a exactly what the two Houses of the Congress did in these cases.
vote of at least a majority of all its Members in regular or special session" - it does not say
"joint session." So, I believe that if the Members of the House need the counsel of the The two Houses of the Congress, the Senate and the House of Representatives, immediately
Senators, they can always call on them, they can invite them.78 (Emphasis supplied.) took separate actions on President Duterte's proclamation of martial law and suspension of
the privilege of the writ of habeas corpus in Mindanao through Proclamation No. 216, in
The proposed amendment was not adopted, however, as only five (5) Commissioners voted accordance with their respective rules of procedure. The Consolidated Comment (Ex
in its favor and twenty-five (25) Commissioners voted against it. Thus, the power to revoke Abudanti Cautela), filed by the Senate and Senate President Pimentel, recounted in detail the
the President's proclamation of martial law and/or suspension of the privilege of the writ steps undertaken by both Houses of the Congress as regards Proclamation No. 216, to wit:
of habeas corpus still lies with both Houses of the Congress, voting jointly, by a vote of at
least a majority of all its Members. 2. On the date of the President's declaration of martial law and the suspension of the
privilege of the writ of habeas corpus, Congress was in session (from May 2, to June 2, 2017),
Significantly, the Commissioners only settled the manner of voting by the in its First Regular Session of the 17th Congress, as evidenced by its Legislative Calendar,
Congress, i.e., "voting jointly, by a vote of at least a majority of all its Members," in order to otherwise known as Calendar of Session as contained in Concurrent Resolution No. 3 of both
revoke the President's proclamation of martial law and/or suspension of the privilege of the the Senate and the House of Representatives.x x x
writ of habeas corpus, but they did not directly take up and specify in Article VII, Section 18
of the 1987 Constitution that the voting shall be done during a joint session of both Houses 3. During the plenary session of the Senate on the following day, 24 May 2017, privilege
of the Congress. In fact, Commissioner Francisco A. Rodrigo expressly observed that the speeches and discussions had already been made about the declaration of martial law and
provision does not call for a joint session. That the Congress will vote on the revocation of the suspension of the privilege of the writ of habeas corpus. This prompted Senator Franklin
the President's proclamation and/or suspension in a joint session can only be inferred from M. Drilon to move to invite the Secretary of National Defense, the National Security Adviser
the arguments of the Commissioners who pushed for the "voting jointly" amendment that and the Chief of Staff of the Armed Forces of the Philippines to brief the senators in closed
the Members of the House of Representatives will benefit from the advice, opinion, and/or session on what transpired in Mindanao. Submitted to a vote and there being no objection,
wisdom of the Senators, which will be presumably shared during a joint session of both the Senate approved the motion. x x x
Houses. Such inference is far from a clear mandate for the Congress to automatically
convene in joint session, under all circumstances, when the President proclaims martial law 4. On 25 May 2017, the President furnished the Senate and the House of Representatives,
and/or suspends the privilege of the writ of habeas corpus, even when Congress does not through Senate President Aquilino "Koko" Pimentel III and Speaker Pantaleon D. Alvarez,
intend to revoke the President's proclamation and/or suspension. respectively, with copies of his report (hereinafter, the "Report") detailing the factual and
legal basis for his declaration of martial law and the suspension of the privilege of the writ
There was no obligation on the part of the Congress herein to convene in joint session as the of habeas corpus in Mindanao.
provision on revocation under Article VII, Section 18 of the 1987 Constitution did not even
come into operation in light of the resolutions, separately adopted by the two Houses of the 5. On or about 25 May 2017, invitation letters were issued and sent by the Senate Secretary,
Congress in accordance with their respective rules of procedure, expressing support for Atty. Lutgardo B. Barbo to the following officials requesting them to attend a briefing for the
President Duterte's Proclamation No. 216. Senators on 29 May 2017 at 3:00 p.m. at the Senators' Lounge at the Senate in a closed door
311

session to describe what transpired in Mindanao which was the basis of the declaration of WHEREAS, the 1987 Philippine Constitution, Article VII, Section 18, provides that:
martial law in Mindanao: (a) Secretary Delfin N. Lorenzana, Secretary of National Defense
(hereinafter, "Secretary Lorenzana"); (b) Secretary Hermogenes C. Esperon, Jr., National "... in case of invasion or rebellion, when the public safety requires it, he (President) may, for
Security Adviser and Director General of the National Security Council (hereinafter, a period not exceeding sixty days, suspend the privilege of the writ of habeas corpus or place
"Secretary Esperon"); and (c) General Eduardo M. Año, Chief of Staff of the Armed Forces of the Philippines or any part thereof under martial law...";
the Philippines (hereinafter, "Gen. Año"). The said letters stated that the Senators requested
that the President's Report be explained and that more details be given about the same. Xxx
WHEREAS, President Rodrigo Roa Duterte issued Proclamation No. 216, series of 2017,
entitled "Declaring a State of Martial Law and Suspending the Privilege of the Writ of Habeas
6. On 29 May 2017, about 3:30 p.m., a closed door briefing was conducted by Secretary Corpus in the Whole of Mindanao," on May 23, 2017 (the "Proclamation");
Lorenzana, Secretary Esperon and other security officials for the Senators to brief them
about the circumstances surrounding the declaration of martial law and to inform them
WHEREAS, pursuant to his duty under the Constitution, on May 25, 2017, and within forth-
about details about the President's Report. The briefing lasted for about four (4) hours. After
eight hours after the issua.11ce of the Proclamation, President Duterte submitted to the
the briefing, the Senators had a caucus to determine what could be publicly revealed.
Senate his report on the factual and legal basis of the Proclamation;

7. On the same day, 29 May 2017, the House of Representatives resolved to constitute itself
WHEREAS, on May 29, 2017, the Senators were briefed by the Department of National
as a Committee of the Whole on 31 May 2017 to consider the President's Report.
Defense (DND), the Armed Forces of the Philippines (AFP), and by the National Security
Council (NSC) on the factual circumstances surrounding the Proclamation as well as the
8. On 30 May 2017, two (2) resolutions were introduced in the Senate about the updates on the situation in Mindanao;
proclamation of martial law. The first one was P.S. Resolution No. 388 (hereinafter, "P.S.R.
No. 388") introduced by Senators Sotto, Pimentel, Recto, Angara, Binay, Ejercito, Gatchalian,
WHEREAS, on the basis of the information received by the Senators, the Senate is convinced
Gordon, Honasan, Lacson, Legarda, Pacquiao, Villanueva, Villar and Zubiri which was entitled,
that President Duterte declared martial law and suspended the privilege of the writ
"Expressing the Sense of the Senate, Supporting the Proclamation No. 216 dated May 23,
of habeas corpus in the whole of Mindanao because actual rebellion exists and that the
2017, entitled "Declaring a State of Martial Law and Suspending the Privilege of the Writ of
public safety requires it;
Habeas Corpus in the Whole of Mindanao" and Finding no Cause to revoke the Same." The
second one was P.S. Resolution No. 390 (hereinafter, "P.S.R. No. 390") introduced by
Senators Pangilinan, Drilon, Hontiveros, Trillanes, Aquino and De Lima which was entitled, WHEREAS, the Senate, at this time, agrees that there is no compelling reason to revoke
"Resolution to Convene Congress in Joint Session and Deliberate on Proclamation No. 216 Proclamation No. 216, series of 2017;
dated 23 May 2017 entitled, "Declaring a State of Martial Law and Suspending the Privilege
of the Writ of Habeas Corpus in the Whole of Mindanao." x x x WHEREAS, the Proclamation does not suspend the operation of the Constitution, which
among others, guarantees respect for human rights and guards against any abuse or violation
9. Discussions were made on the two (2) proposed resolutions during the plenary thereof: Now, therefore, be it
deliberations of the Senate on 30 May 2017. The first resolution to be discussed was P.S.R.
No. 388. During the deliberations, amendments were introduced to it and after the Resolved, as it is hereby resolved, To express the sense of the Senate, that there is no
amendments and the debates, P.S.R. No. 388 was voted upon and it was adopted by a vote compelling reason to revoke Proclamation No. 216, series of 2017 at this time.
of seventeen (17) affirmative votes and five (5) negative votes. The amended, substituted
and approved version of P.S.R. No. 388, which was then renamed Resolution No. 49, states as Adopted. x x x"
follows:

10. Immediately thereafter, P.S.R. No. 390 was also deliberated upon. After a prolonged
RESOLUTION NO. 49 discussion, a vote was taken on it and nine (9) senators were in favor and twelve (12) were
against. As such, P.S.R. No. 390 calling for a joint session of Congress was not adopted. x x x
RESOLUTION EXPRESSING THE SENSE OF THE SENATE NOT TO REVOKE, AT THIS TIME,
PROCLAMATION NO. 216, SERIES OF 2017, ENTITLED, "DECLARING A STATE OF MARTIAL LAW 11. In the meantime, on 31 May 2017, the House of Representatives acting as a Committee
AND SUSPENDING THE PRIVILEGE OF THE WRIT OF HABEAS CORPUS IN THE WHOLE OF of the Whole was briefed for about six (6) hours by officials of the government led by
MINDANAO." Executive Secretary Salvador C. Medialdea (hereinafter, "Executive Secretary Medialdea"),
312

Secretary Lorenzana and other security officials on the factual circumstances surrounding the Except for Certain Areas) adopted on 09 December 2009. The only time that the Senate and
President's declaration of martial law and on the statements contained in the President's the House of Representatives do not adopt Rules for a joint session is when they convene on
Report. During the evening of the same day, a majority of the House of Representatives the fourth Monday of July for its regular session to receive or listen to the State of the Nation
passed Resolution No. 1050 entitled, "'Resolution Expressing the Full Support of the House of Address of the President and even then, they adopt a Concurrent Resolution to do so.
Representatives to President Rodrigo Roa Duterte As It Finds No Reason to Revoke
Proclamation No. 216 Entitled, 'Declaring A State of Martial Law and Suspending the Privilege The usual procedure for having a joint session is for both Houses to first adopt a Concurrent
of the Writ of Habeas Corpus in the Whole of Mindanao."' In the same deliberations, it was Resolution to hold a joint session. This is achieved by either of two (2) ways: (1) both the
likewise proposed that the House of Representatives call for a joint session of Congress to Senate and the House of Representatives simultaneously adopting the Concurrent Resolution
deliberate and vote on the President's declaration of martial law and the suspension of the - an example would be when the two (2) Houses inform the President that they are ready to
privilege of the writ of habeas corpus. However, after debates, the proposal was not carried. receive his State of the Nation Address or (2) For one (1) House to pass its own resolution
x x x.79 and to send it to the other House for the latter's concurrence. Once the joint session of both
Houses is actually convened, it is only then that the Senate and the House of Representatives
It cannot be disputed then that the Senate and House of Representatives placed President jointly adopt the Rules for the joint session. x x x80 (Emphases supplied.)
Duterte's Proclamation No. 216 under serious review and consideration, pursuant to their
power to revoke such a proclamation vested by the Constitution on the Congress. Each With neither Senate nor the House of Representatives adopting a concurrent resolution, no
House timely took action by accepting and assessing the President's Report, inviting over and joint session by the two Houses of the Congress can be had in the present cases.
interpellating executive officials, and deliberating amongst their fellow Senators or
Representatives, before finally voting in favor of expressing support for President Duterte's
The Court is bound to respect the rules of the Congress, a co-equal and independent branch
Proclamation No. 216 and against calling for a joint session with the other House. The
of government. Article VI, Section 16(3) of the 1987 Constitution states that "[e]ach House
prompt actions separately taken by the two Houses of the Congress on President Duterte's
shall determine the rules of its proceedings." The provision has been traditionally construed
Proclamation No. 216 belied all the purported difficulties and delays such procedures would
as a grant of full discretionary authority to the Houses of Congress in the formulation,
cause as raised in the Concurring and Dissenting Opinion of Associate Justice Marvic M.V.F.
adoption, and promulgation of its rules; and as such, the exercise of this power is generally
Leonen (Justice Leonen). As earlier pointed out, there is no constitutional provision governing
exempt from judicial supervision and interference.81 Moreover, unless there is a clear
concurrence by the Congress in the President's proclamation of martial law and/or
showing by strong and convincing reasons that they conflict with the Constitution, "all
suspension of the privilege of the writ of habeas corpus, and absent a specific mandate for
legislative acts are clothed with an armor of constitutionality particularly resilient where such
the Congress to hold a joint session in the event of concurrence, then whether or not to hold
acts follow a long-settled and well-established practice by the Legislature."82Nothing in this
a joint session under such circumstances is completely within the discretion of the Congress.
Decision should be presumed to give precedence to the rules of the Houses of the Congress
over the provisions of the Constitution. This Court simply holds that since the Constitution
The Senate and Senate President Pimentel explained in their Consolidated Comment (Ex does not regulate the manner by which the Congress may express its concurrence to a
Abudanti Cautela), that, by practice, the two Houses of the Congress must adopt a Presidential proclamation of martial law and/or suspension of the privilege of the writ
concurrent resolution to hold a joint session, and only thereafter can the Houses adopt the of habeas corpus, the Houses of the Congress have the discretion to adopt rules of procedure
rules to be observed for that particular joint session: as they may deem appropriate for that purpose.

It must be stated that the Senate and the House of Representatives have their own The Court highlights the particular circumstance herein that both Houses of Congress
respective Rules, i.e., the Rules of the Senate and the Rules of the House of Representatives. already separately expressed support for President Duterte's Proclamation No. 216, so
There is no general body of Rules applicable to a joint session of Congress. Based on revocation was not even a possibility and the provision on revocation under Article VII,
parliamentary practice and procedure, the Senate and House of Representatives only adopt Section 18 of the 1987 Constitution requiring the Congress to vote jointly in a joint session
Rules for a joint session on an ad hoc basis but only after both Houses have already agreed to never came into operation. It will be a completely different scenario if either of the Senate
convene in a joint session through a Concurrent Resolution. The Rules for a Joint Session for a or the House of Representatives, or if both Houses of the Congress, resolve/s to revoke the
particular purpose become functus officio after the purpose of the joint session has been President's proclamation of martial law and/or suspension of the privilege of the writ
achieved. Examples of these Rules for a Joint Session are (1) the Rules of the Joint Public of habeas corpus, in which case, Article VII, Section 18 of the 1987 Constitution shall apply
Session of Congress on Canvassing the Votes Cast for Presidential and Vice-Presidential and the Congress must convene in joint session to vote jointly on the revocation of the
Candidates in the May 9, 2016 Election adopted on 24 May 2016; and (2) the Rules of the proclamation and/or suspension. Given the foregoing parameters in applying Article VII,
Joint Session of Congress on Proclamation No. 1959 (Proclaiming a State of Martial Law and Section 18 of the 1987 Constitution, Justice Leonen's concern, expressed in his Concurring
Suspending the Privilege of the Writ of Habeas Corpus in the Province of Maguindanao, and Dissenting Opinion, that a deadlock may result in the future, is completely groundless.
313

The legislative precedent referred to by petitioners actually supports the position of the The Court is not persuaded.
Court in the instant cases. On December 4, 2009, then President Macapagal-Arroyo issued
Proclamation No. 1959, entitled "Proclaiming a State of Martial law and Suspending the First, the provision specially addresses the situation when the President proclaims martial
Privilege of the Writ of Habeas Corpus in the Province of Maguindanao, except for Certain law and/or suspends the privilege of the writ of habeas corpus while the Congress is in
Areas." The Senate, on December 14, 2009, adopted Resolution No. 217, entitled "Resolution recess. To ensure that the Congress will be able to act swiftly on the proclamation and/or
Expressing the Sense of the Senate that the Proclamation of Martial Law in the Province of suspension, the 1987 Constitution provides that it should convene within twenty-four (24)
Maguindanao is Contrary to the Provisions of the 1987 Constitution." Consequently, the hours without need for call. It is a whole different situation when the Congress is still in
Senate and the House of Representatives adopted Concurrent Resolutions, i.e., Senate session as it can readily take up the proclamation and/or suspension in the course of its
Concurrent Resolution No. 14 and House Concurrent Resolution No. 33, calling both Houses regular sessions, as what happened in these cases. Second, the provision only requires that
of the Congress to convene in joint session on December 9, 2009 at 4:00 p.m. at the Session the Congress convene without call, but it does not explicitly state that the Congress shall
Hall of the House of Representatives to deliberate on Proclamation No. 1959. It appears then already convene in joint session. In fact, the provision actually states that the Congress
that the two Houses of the Congress in 2009 also initially took separate actions on President "convene in accordance with its rules," which can only mean the respective rules of each
Macapagal-Arroyo's Proclamation No. 1959, with the Senate eventually adopting Resolution House as there are no standing rules for joint sessions. And third, it cannot be said herein
No. 217, expressing outright its sense that the proclamation of ma11ial law was that the Congress failed to convene immediately to act on Proclamation No. 216. Both
unconstitutional and necessarily implying that such proclamation should be revoked. With Houses of the Congress promptly took action on Proclamation No. 216, with the Senate
one of the Houses favoring revocation, and in observation of the established practice of the already issuing invitations to executive officials even prior to receiving President Duterte's
Congress, the two Houses adopted concurrent resolutions to convene in joint session to vote Report, except that the two Houses of the Congress acted separately. By initially undertaking
on the revocation of Proclamation No. 1959. separate actions on President Duterte's Proclamation No. 216 and making their respective
determination of whether to support or revoke said Proclamation, the Senate and the House
For the same reason, the Fortun case cannot be deemed a judicial precedent for the present of Representatives were only acting in accordance with their own rules of procedure and
cases. The factual background of the Fortun case is not on all fours with these cases. Once were not in any way remiss in their constitutional duty to guard against a baseless or
more, the Court points out that in the Fortun case, the Senate expressed through Resolution unjustified proclamation of martial law and/or suspension of the privilege of the writ
No. 217 its objection to President Macapagal-Arroyo's Proclamation No. 1959 for being of habeas corpus by the President.
unconstitutional, and both the Senate and the House of Representatives adopted concurrent
resolutions to convene in joint session for the purpose of revoking said proclamation; while There is likewise no basis for petitioners' assertion that without a joint session, the public
in the cases at bar, the Senate and the House of Representatives adopted Senate Resolution cannot hold the Senators and Representatives accountable for their respective positions on
No. 49 and House Resolution No. 1050, respectively, which expressed support for President President Duterte's Proclamation No. 216. Senate records completely chronicled the
Duterte's Proclamation No. 216, and both Houses of the Congress voted against calling for a deliberations and the voting by the Senators on Senate Resolution No. 49 (formerly P.S.
joint session. In addition, the fundamental issue in the Fortun case was whether there was Resolution No. 388) and P.S. Resolution No. 390. While it is true that the House of
factual basis for Proclamation No. 1959 and not whether it was mandatory for the Congress Representatives voted on House Resolution No. 1050 viva voce, this is only in accordance
to convene in joint session; and even before the Congress could vote on the revocation of with its rules. Per the Rules of the House of Representatives:
Proclamation No. 1959 and the Court could resolve the Fortun case, President Macapagal-
Arroyo already issued Proclamation No. 1963 on December 12, 2009, entitled "Proclaiming
RULE XV
the Termination of the State of Martial Law and the Restoration of the Privilege of the Writ of
Habeas Corpus in the Province of Maguindanao." Furthermore, the word "automatic" in
the Fortun case referred to the duty or power of the Congress to review the proclamation of Voting
martial law and/or suspension of the privilege of the writ of habeas corpus, rather than the
joint session of Congress.83 Sec. 115. Manner of Voting. -The Speaker shall rise and state the motion or question that is
being put to a vote in clear, precise and simple language. The Speaker shall say "as many as
Petitioners invoke the following provision also in Article VII, Section 18 of the 1987 are in favor, (as the question may be) say 'aye'". After the affirmative vote is counted, the
Constitution: "The Congress, if not in session, shall, within twenty-four hours following such Speaker shall say "as many as are opposed, (as the question may be) say 'nay"'.
proclamation or suspension convene in accordance with its rules without call." Petitioners
reason that if the Congress is not in session, it is constitutionally mandated to convene within If the Speaker doubts the result of the voting or a motion to divide the House is Carried, the
twenty-four (24) hours from the President's proclamation of martial law and/or suspension House shall divide. The Speaker shall ask those in favor to rise, to be followed by those
of the privilege of the writ of habeas corpus, then it is with all the more reason required to against. If still in doubt of the outcome or a count by tellers is demanded, the Speaker shall
convene immediately if in session. name one (1) Member from each side of the question to count the Members in the
314

affirmative and those in the negative. After the count is reported, the Speaker shall Rule XI of the Rules of the House of Representatives provides:
announce the result.
Section 82. Sessions Open to the Public. - Sessions shall be open to the public. However,
An abstention shall not be counted as a vote. Unless otherwise provided by the Constitution when the security of the State or the dignity of the House or any of its Members are affected
or by these rules, a majority of those voting, there being a quorum, shall decide the issue. by any motion or petition being considered, the House may hold executive sessions.

Sec. 116. Nominal Voting. - Upon motion of a Member, duly approved by one-fifth (1/5) of Guests and visitors in the galleries are prohibited from using their cameras and video
the Members present, there being a quorum, nominal voting on any question may be called. recorders. Cellular phones and other similar electronic devices shall be put in silent mode.
In case of nominal voting, the Secretary General shall call, in alphabetical order, the nan1es
of the Members who shall state their vote as their names are called. Section 83. Executive Sessions. - When the House decides to hold an executive session, the
Speaker shall direct the galleries and hallways to be cleared and the doors closed. Only the
Sec. 117. Second Call on Nominal Voting. - A second call on nominal voting shall be made to Secretary General, the Sergeant-at- Arms and other persons specifically authorized by the
allow Members who did not vote during the first call to vote.1avvphi1 Members who fail to House shall be admitted to the executive session. They shall preserve the confidentiality of
vote during the second call shall no longer be allowed to vote. everything read or discussed in the session. (Emphasis supplied.)

Since no one moved for nominal voting on House Resolution No. 1050, then the votes of the Rule XLVII of the Rules of the Senate similarly sets forth the following:
individual Representatives cannot be determined. It does not render though the proceedings
unconstitutional or invalid. SEC. 126. The executive sessions of the Senate shall be held always behind closed doors. In
such sessions, only the Secretary, the Sergeant-at-Arms, and/or such other persons as may
The Congress did not violate the right of the public to information when it did not convene in be authorized by the Senate may be admitted to the session hall.
joint session.
SEC. 127. Executive sessions shall be held whenever a Senator so requests it and his petition
The Court is not swayed by petitioners' argument that by not convening in joint session, the has been duly seconded, or when the security of the State or public interest so requires.
Congress violated the public's right to information because as records show, the Congress Thereupon, the President shall order that the public be excluded from the gallery and the
still conducted deliberations on President Duterte's Proclamation No. 216, albeit separately; doors of the session hall be closed.
and the public's right to information on matters of national security is not absolute. When
such matters are being taken up in the Congress, whether in separate or joint sessions, the The Senator who presented the motion shall then explain the reasons which he had for
Congress has discretion in the manner the proceedings will be conducted. submitting the same.

Petitioners contend that the Constitution requires a public deliberation process on the The minutes of the executive sessions shall be recorded m a separate book.
proclamation of martial law: one that is conducted via a joint session and by a single body.
They insist that the Congress must be transparent, such that there is an "open and robust
From afore-quoted rules, it is clear that matters affecting the security of the state are
debate," where the evaluation of the proclamation's factual bases and subsequent
considered confidential and must be discussed and deliberated upon in an executive session,
implementation shall be openly discussed and where each member's position on the issue is
excluding the public therefrom.
heard and made known to the public.

That these matters are considered confidential is in accordance with settled jurisprudence
The petitioners' insistence on the conduct of a "joint session" contemplates a mandatory
that, in the exercise of their right to information, the government may withhold certain types
joint Congressional session where public viewing is allowed.
of information from the public such as state secrets regarding military, diplomatic, and other
national security matters.85 The Court has also ruled that the Congress' deliberative process,
However, based on their internal rules, each House has the discretion over the manner by including information discussed and deliberated upon in an executive session,86may be kept
which Congressional proceedings are to be conducted. Verily, sessions are generally open to out of the public's reach.
the public,84 but each House may decide to hold an executive session due to the confidential
nature of the subject matter to be discussed and deliberated upon.
315

The Congress not only recognizes the sensitivity of these matters but also endeavors to petitioner, and to pay the damages sustained by the petitioner by reason of the wrongful
preserve their confidentiality. In fact, Rule XL VII, Section 12887 of the Rules of the Senate acts of the respondent,
expressly establishes a secrecy ban prohibiting all its members, including Senate officials and
employees, from divulging any of the confidential matters taken up by the Senate. A Senator Jurisprudence has laid down the following requirements for a petition for mandamus to
found to have violated this ban faces the possibility of expulsion from his office.88This is prosper:
consistent with the Ethical Standards Act89 that prohibits public officials and employees from
using or divulging "confidential or classified information officially known to them by reason
[T]hus, a petition for mandamus will prosper if it is shown that the subject thereof is a
of their office and not made available to the public."90
ministerial act or duty, and not purely discretionary on the part of the board, officer or
person, and that the petitioner has a well-defined, clear and certain right to warrant the
Certainly, the factual basis of the declaration of martial law involves intelligence information, grant thereof.
military tactics, and other sensitive matters that have an undeniable effect on national
security. Thus, to demand Congress to hold a public session during which the legislators shall
The difference between a ministerial and discretionary act has long been established. A
openly discuss these matters, all the while under public scrutiny, is to effectively compel
purely ministerial act or duty is one which an officer or tribunal performs in a given state of
them to make sensitive information available to everyone, without exception, and to breach
facts, in a prescribed manner, in obedience to the mandate of a legal authority, without
the recognized policy of preserving these matters' confidentiality, at the risk of being
regard to or the exercise of his own judgment upon the propriety or impropriety of the act
sanctioned, penalized, or expelled from Congress altogether.
done. If the law imposes a duty upon a public officer and gives him the right to decide how or
when the duty shall be performed, such duty is discretionary and not ministerial. The duty is
That these are the separate Rules of the two Houses of the Congress does not take away ministerial only when the discharge of the same requires neither the exercise of official
from their persuasiveness and applicability in the event of a joint session.1âwphi1 Since both discretion or judgment.92 (Emphases added.)
Houses separately recognize the policy of preserving the confidentiality of national security
matters, then in all likelihood, they will consistently observe the same in a joint session. The
It is essential to the issuance of a writ of mandamus that petitioner should have a clear legal
nature of these matters as confidential is not affected by the composition of the body that
right to the thing demanded and it must be the imperative duty of the respondent to
will deliberate upon it - whether it be the two Houses of the Congress separately or in joint
perform the act required. Mandamus never issues in doubtful cases. While it may not be
session.
necessary that the ministerial duty be absolutely expressed, it must however, be clear. The
writ neither confers powers nor imposes duties. It is simply a command to exercise a power
Also, the petitioners' theory that a regular session must be preferred over a mere briefing for already possessed and to perform a duty already imposed.93
purposes of ensuring that the executive and military officials are placed under oath does not
have merit. The Senate Rules of Procedure Governing Inquiries In Aid of Legislation91 require
Although there are jurisprudential examples of the Court issuing a writ of mandamus to
that all witnesses at executive sessions or public hearings who testify as to matters of fact
compel the fulfillment of legislative duty,94 we must distinguish the present controversy with
shall give such testimony under oath or affirmation. The proper implementation of this rule is
those previous cases. In this particular instance, the Court has no authority to compel the
within the Senate's competence, which is beyond the Court's reach.
Senate and the House of Representatives to convene in joint session absent a clear
ministerial duty on its part to do so under the Constitution and in complete disregard of the
Propriety of the issuance of a writ of mandamus or certiorari separate actions already undertaken by both Houses on Proclamation No. 216, including
their respective decisions to no longer hold a joint session, considering their respective
For mandamus to lie, there must be compliance with Rule 65, Section 3, Rules of Court, to resolutions not to revoke said Proclamation.
wit:
In the same vein, there is no cause for the Court to grant a writ of certiorari.
SECTION 3. Petition for mandamus. - When any tribunal, corporation, board, officer or
person unlawfully neglects the perfom1ance of an act which the law specifically enjoins as a As earlier discussed, under the Court's expanded jurisdiction, a petition for certiorari is a
duty resulting from an office, trust, or station, or unlawfully excludes another from the use proper remedy to question the act of any branch or instrumentality of the government on
and enjoyment of a right or office to which such other is entitled, and there is no other plain, the ground of grave abuse of discretion amounting to lack or excess of jurisdiction by any
speedy and adequate remedy in the ordinary course of law, the person aggrieved thereby branch or instrumentality of the government, even if the latter does not exercise judicial,
may file a verified petition in the proper court, alleging the facts with certainty and praying quasi-judicial or ministerial functions.95 Grave abuse of discretion implies such capricious and
that judgment be rendered commanding the respondent, immediately or at some other time whimsical exercise of judgment as to be equivalent to lack or excess of jurisdiction; in other
to be specified by the court, to do the act required to be done to protect the rights of the
316

words, power is exercised in an arbitrary or despotic manner by reason of passion, prejudice, On May 23, 2017, President Rodrigo Roa Duterte issued Proclamation No. 216, declaring a
or personal hostility; and such exercise is so patent or so gross as to amount to an evasion of state of martial law and suspending the privilege of the writ of habeas corpus in the whole of
a positive duty or to a virtual refusal either to perform the duty enjoined or to act at all in Mindanao for a period not exceeding sixty (60) days, to address the rebellion mounted by
contemplation of law.96 It bears to mention that to pray in one petition for the issuance of members of the Maute Group and Abu Sayyaf Group (ASG).
both a writ of mandamus and a writ of certiorari for the very same act - which, in the Tañada
Petition, the non-convening by the two Houses of the Congress in joint session - is On May 25, 2017, within the 48-hour period set in Section 18, Article VII of the Constitution,
contradictory, as the former involves a mandatory duty which the government branch or the President submitted to the Senate and the House of Representatives his written Report,
instrumentality must perform without discretion, while the latter recognizes discretion on citing the events and reasons that impelled him to issue Proclamation No. 216. Thereafter,
the part of the government branch or instrumentality but which was exercised arbitrarily or the Senate adopted P.S. Resolution No. 388 while the House of Representatives issued House
despotically. Nevertheless, if the Court is to adjudge the petition for certiorari alone, it still Resolution No. 1050, both expressing full support to the Proclamation and finding no cause
finds the same to be without merit. To reiterate, the two Houses of the Congress decided to to revoke the same.
no longer hold a joint session only after deliberations among their Members and putting the
same to vote, in accordance with their respective rules of procedure. Premises considered,
On July 18, 2017, the President requested the Congress to extend the effectivity of
the Congress did not gravely abuse its discretion when it did not jointly convene upon the
Proclamation No. 216. In a Special Joint Session on July 22, 2017, the Congress adopted
President's issuance of Proclamation No. 216 prior to expressing its concurrence thereto.
Resolution of Both Houses No. 2 extending Proclamation No. 216 until December 31, 2017.

WHEREFORE, the petitions are DISMISSED for lack of merit.


In a letter to the President, through Defense Secretary Lorenzana, AFP Chief of Staff General
Guerrero, recommended the further extension of martial law and suspension of the privilege
SO ORDERED. of the writ of habeas corpus in the entire Mindanao for one year beginning January 1, 2018
“for compelling reasons based on current security assessment.”
REPRESENTATIVES EDCEL LAGMAN, TOMASITO S. VILLARIN, EDGAR R. ERICE, TEDDY
BRAWNER BAGUILAT, JR., GARY C. ALEJANO, AND EMMANUEL A. BILLONES, Petitioners, vs. On the basis of this security assessment, Secretary Lorenzana wrote a similar
SENATE PRESIDENT AQUILINO PIMENTEL III, SPEAKER PANTALEON D. ALVAREZ, EXECUTIVE recommendation to the President “primarily to ensure total eradication of DAESH-inspired
SECRETARY SALVADOR C. MEDIALDEA, DEFENSE SECRETARY DELFIN N. LORENZANA, Da’awatul Islamiyah Waliyatul Masriq (DIWM), other like-minded Local/Foreign Terrorist
BUDGET SECRETARY BENJAMIN E. DIOKNO AND ARMED FORCES OF THE PHILIPPINES CHIEF Groups (L/FTGs) and Armed Lawless Groups (ALGs), and the communist terrorists (CTs) and
OF STAFF GENERAL REY LEONARDO GUERRERO, Respondents. their coddlers, supporters and financiers, and to ensure speedy rehabilitation, recovery and
reconstruction efforts in Marawi, and the attainment of lasting peace, stability, economic
GR Nos. 235935, 236061, 236145, 236155 development and prosperity in Mindanao.”

February 6, 2018 Acting on said recommendations, the President, in a letter dated December 8, 2017, asked
both the Senate and the House of Representatives to further extend the proclamation of
TOPIC: martial law extension martial law and the suspension of the privilege of the writ of habeas corpus in the entire
Mindanao for one year, from January 1, 2018 to December 31, 2018, or for such period as
the Congress may determine.
PONENTE: Tijam

On December 13, 2017, the Senate and the House of Representatives, in a joint session,
FACTS:
adopted Resolution of Both Houses No. 4 further extending the period of martial law and
suspension of the privilege of the writ of habeas corpus in the entire Mindanao for one year,
These are consolidated petitions assailing the constitutionality of the extension of the from January 1, 2018 to December 31, 2018.
proclamation of martial law and suspension of the writ of habeas corpus in the entire
Mindanao for one year from January 1 to December 31, 2018.
317

ISSUE: court can take judicial notice of the official acts of the legislative department without the
introduction of evidence.
PROCEDURAL:
Moreover, the Court noted that respondents annexed a copy of the Resolution to their
Whether or not the petitioners’ failure to attach Resolution of Both Houses No. 4 fatal to Consolidated Comment.
their petitions.
SECOND ISSUE:
Whether or not the President should be dropped as party respondent.
Whether or not the President should be dropped as party respondent. YES.
Whether or not the Congress was an indispensable party to the consolidated petitions.
The Court held that the President should be dropped as party respondent considering that he
Whether or not the Court was barred by the doctrine of conclusiveness of judgment from enjoys the presidential immunity from suit.
examining the persistence of rebellion in Mindanao.
The Court reiterated their ruling in Rubrico v. Macapagal-Arroyo, to wit:
Whether or not the petitioners may invoke the expanded (certiorari) jurisdiction of the
Supreme Court under Section 1, Article VIII of the Constitution in seeking review of the It will degrade the dignity of the high office of the President, the Head of State, if he can be
extension of Proclamation No. 216. dragged into court litigations while serving as such. Furthermore, it is important that he be
freed from any form of harassment, hindrance or distraction to enable him to fully attend to
Whether or not the manner in which Congress deliberated on the President’s request for the performance of his official duties and functions. Unlike the legislative and judicial branch,
extension of martial law is subject to judicial review. only one constitutes the executive branch and anything which impairs his usefulness in the
discharge of the many great and important duties imposed upon him by the Constitution
necessarily impairs the operation of the Government.
Whether or not the Congress has the power to extend and determine the period of martial
law and the suspension of the privilege of the writ of habeas corpus.
THIRD ISSUE:
Whether or not the President and the Congress had sufficient factual basis to extend
Proclamation No. 216. Whether or not the Congress was an indispensable party to the consolidated petitions. YES.

Whether or not there is necessity to impose tests on the choice and manner of the The Court held that in cases impugning the extension of martial law for lack of sufficient
President’s exercise of military powers. factual basis, the entire body of the Congress, composed of the Senate and the House of
Representatives, must be impleaded, being an indispensable party thereto.
Whether or not the petitioners were able to comply with all the requisites for the issuance of
an injunctive writ. The Court further ruled that in these consolidated petitions, petitioners are questioning the
constitutionality of a congressional act, specifically the approval of the President’s request to
extend martial law in Mindanao. Clearly, therefore, it is the Congress as a body, and not just
HELD:
its leadership, which has interest in the subject matter of these cases.

FIRST ISSUE:
FOURTH ISSUE:

Whether or not the petitioners’ failure to attach Resolution of Both Houses No. 4 fatal to
Whether or not the Court was barred by the doctrine of conclusiveness of judgment from
their petitions. NO.
examining the persistence of rebellion in Mindanao. NO.

The Court held that since Resolution of Both Houses No. 4 is an official act of Congress, the
they can take judicial notice thereof. Section 1, Rule 129 of the Rules of Court provides that a
318

The Court held that as to the second requirement, there was np identity of issues between The Court added that to apply the standard of review in a petition for certiorari will
the Lagman and Padilla cases, on one hand, and the case at bar. emasculate the Court’s constitutional task under Section 18, Article VII, which was precisely
meant to provide an additional safeguard against possible martial law abuse and limit the
Conclusiveness of judgment, a species of the principle of res judicata, bars the re-litigation of extent of the powers of the Commander-in-Chief.
any right, fact or matter in issue directly adjudicated or necessarily involved in the
determination of an action before a competent court in which judgment is rendered on the Finally, the Court held that a certiorari petition invoking the Court’s expanded jurisdiction is
merits. In order to successfully apply in a succeeding litigation the doctrine of conclusiveness not the proper remedy to review the sufficiency of the factual basis of the Congress’
of judgment, mere identities of parties and issues is required. extension of the proclamation of martial law or suspension of the privilege of the writ.

The issue put forth by petitioners in the earlier Lagman case, which this Court already settled, PRELIMINARIES ON MARTIAL LAW
refers to the existence of a state of rebellion which would trigger the President’s initial
declaration of martial law, whereas the factual issue in the case at bar refers to the Congressional check on martial law
persistence of the same rebellion in Mindanao which would justify the extension of martial
law.
Congressional check on the President’s martial law and suspension powers thus consists of:

The fact that petitioners are not barred from questioning the alleged persistence of the
The power to review the President’s proclamation of martial law or suspension of the
rebellion in these consolidated petitions is also supported by the transitory nature of the
privilege of the writ of habeas corpus, and to revoke such proclamation or suspension. The
Court’s judgment on the sufficiency of the factual basis for a declaration of martial law.
review is “automatic in the sense that it may be activated by Congress itself at any time after
the proclamation or suspension is made.” The Congress’ decision to revoke the proclamation
Verily, the Court’s review in martial law cases is largely dependent on the existing factual or suspension cannot be set aside by the President.
scenario used as basis for its imposition or extension. The gravity and scope of rebellion or
invasion, as the case may be, should necessarily be re-examined, in order to make a
The power to approve any extension of the proclamation or suspension, upon the President’s
justiciable determination on whether rebellion persists in Mindanao as to justify an extension
initiative, for such period as it may determine, if the invasion or rebellion persists and public
of a state of martial law.
safety requires it.

FIFTH ISSUE:
Joint executive and legislative act

Whether or not the petitioners may invoke the expanded (certiorari) jurisdiction of the
When approved by the Congress, the extension of the proclamation or suspension, as
Supreme Court under Section 1, Article VIII of the Constitution in seeking review of the
described during the deliberations on the 1987 Constitution, becomes a “joint executive and
extension of Proclamation No. 216. NO.
legislative act” or a “collective judgment” between the President and the Congress.

The Court reiterated their earlier ruling in Lagman case where they emphasized that the
SIXTH ISSUE:
Court’s jurisdiction under the third paragraph of Section 18, Article VII is special and specific,
different from those enumerated in Sections 1 and 5 of Article VIII. It was further stressed
therein that the standard of review in a petition for certiorari is whether the respondent has Whether or not the manner in which Congress deliberated on the President’s request for
committed any grave abuse of discretion amounting to lack or excess of jurisdiction in the extension of martial law is subject to judicial review. NO.
performance of his or her functions, whereas under Section 18, Article VII, the Court is tasked
to review the sufficiency of the factual basis of the President’s exercise of emergency The Court ruled that they cannot review the rules promulgated by Congress in the absence of
powers. any constitutional violation. Petitioners have not shown that the above-quoted rules of the
Joint Session violated any provision or right under the Constitution.
Hence, the Court concluded that a petition for certiorari pursuant to Section 1 or Section 5 of
Article VIII is not the proper tool to review the sufficiency of the factual basis of the Construing the full discretionary power granted to the Congress in promulgating its rules, the
proclamation of martial law or the suspension of the privilege of the writ of habeas corpus. Court, in the case of Spouses Dela Paz (Ret.) v. Senate Committee on Foreign Relations, et al.
explained that the limitation of this unrestricted power deals only with the imperatives of
319

quorum, voting and publication. It should be added that there must be a reasonable relation EIGHTH ISSUE: Whether or not the President and the Congress had sufficient factual basis to
between the mode or method of proceeding established by the rule and the result which is extend Proclamation No. 216. YES.
sought to be attained.
Section 18, Article VII of the 1987 Constitution requires two factual bases for the extension of
In the instant case, the rules in question did not pertain to quorum, voting or publication. the proclamation of martial law or of the suspension of the privilege of the writ of habeas
Furthermore, deliberations on extending martial law certainly cannot be equated to the corpus: (a) the invasion or rebellion persists; and (b) public safety requires the extension.
consideration of regular or ordinary legislation. The Congress may consider such matter as
urgent as to necessitate swift action, or it may take its time investigating the factual Rebellion persists as to satisfy the first condition for the extension of martial law or of the
situation. This Court cannot engage in undue speculation that members of Congress did not suspension of the privilege of the writ of habeas corpus.
review and study the President’s request based on a bare allegation that the time allotted for
deliberation was too short.
The reasons cited by the President in his request for further extension indicate that the
rebellion, which caused him to issue Proclamation No. 216, continues to exist and its
SEVENTH ISSUE: “remnants” have been resolute in establishing a DAESH/ISIS territory in Mindanao, carrying
on through the recruitment and training of new members, financial and logistical build-up,
Whether or not the Congress has the power to extend and determine the period of martial consolidation of forces and continued attacks.
law and the suspension of the privilege of the writ of habeas corpus. YES.
AFP General Guerrero also cited, among others, the continued armed resistance of the
Section 18, Article VII of the 1987 Constitution is indisputably silent as to how many times the DAESH-inspired DIWM and their allies. Moreover, The AFP’s data also showed that Foreign
Congress, upon the initiative of the President, may extend the proclamation of martial law or Terrorist Fighters (FTFs) are now acting as instructors to the new members of the Dawlah
the suspension of the privilege of habeas corpus. Islamiyah.

What is clear is that the ONLY limitations to the exercise of the congressional authority to Also, it does not necessarily follow that with the liberation of Marawi, the DAESH/ISIS-
extend such proclamation or suspension are (1) that the extension should be upon the inspired rebellion no longer exists. Secretary Lorenzana, during the Congress’ Joint Session on
President’s initiative; (2) that it should be grounded on the persistence of the invasion or December 13, 2017, explained that while the situation in Marawi has substantially changed,
rebellion and the demands of public safety; and (3) that it is subject to the Court’s review of the rebellion has not ceased but simply moved to other places in Mindanao.
the sufficiency of its factual basis upon the petition of any citizen.
Acts upon which extension was based posed danger to general public
Section 18, Article VII did not also fix the period of the extension of the proclamation and
suspension. However, it clearly gave the Congress the authority to decide on its duration; The Court also ruled that the acts, circumstances and events upon which the extension was
thus, the provision states that that the extension shall be “for a period to be determined by based posed a significant danger, injury or harm to the general public.
the Congress.”
The Court added that the information upon which the extension of martial law or of the
Commissioner Jose E. Suarez’s proposal to limit the extension to 60 days was not adopted by suspension of the privilege of the writ of habeas corpus shall be based principally emanate
the majority of the Commission’s members. The framers evidently gave enough flexibility on from and are in the possession of the Executive Department. Thus, “the Court will have to
the part of the Congress to determine the duration of the extension. Plain textual reading of rely on the fact-finding capabilities of the Executive Department; in tum, the Executive
Section 18, Article VII and the records of the deliberation of the Constitutional Commission Department will have to open its findings to the scrutiny of the Court.”
buttress the view that as regards the frequency and duration of the extension, the
determinative factor is as long as “the invasion or rebellion persists and public safety
The Executive Department did open its findings to the Court when the· AFP gave its “briefing”
requires” such extension.
or “presentation” during the oral arguments, presenting data, which had been vetted by the
NICA, “based on intelligence reports gathered on the ground,” from personalities they were
able to capture and residents in affected areas, declassified official documents, and
intelligence obtained by the PNP. According to the AFP, the same presentation, save for
updates, was given to the Congress. As it stands, the information thus presented has not
been challenged or questioned as regards its reliability.
320

The facts as provided by the Executive and considered by Congress amply establish that This Court cannot rely on speculations, conjectures or guesswork, but must depend upon
rebellion persists in Mindanao and public safety is significantly endangered by it. The Court, competent proof and on the basis of the best evidence obtainable under the circumstances.
thus, holds that there exists sufficient factual basis for the further extension sought by the We emphasize that the grant or denial of an injunctive writ cannot be properly resolved by
President and approved by the Congress in its Resolution of Both Houses No. 4. suppositions, deductions, or even presumptions, with no basis in evidence, for the truth must
have to be determined by the procedural rules of admissibility and proof.
NINTH ISSUE:
Incidentally, there is nothing in the Constitution, nor in any law which supports petitioners’
Whether or not there is necessity to impose tests on the choice and manner of the theory. Such purported human right violations cannot be utilized as ground either to enjoin
President’s exercise of military powers. NO. the President from exercising the power to declare martial law, or the Congress in extending
the same. To sanction petitioners’ plea would result into judicial activism, thereby going
against the principle of separation of powers.
The Court reiterated their ruling in the earlier Lagman case that the determination of which
among the Constitutionally given military powers should be exercised in a given set of factual
circumstances is a prerogative of the President. The Court’s power of review, as provided As discussed above, petitioners are not left without any recourse. Such transgressions can be
under Section 18, Article VII do not empower the Court to advise, nor dictate its own addressed in a separate and independent court action. Hence, petitioners can lodge a
judgment upon the President, as to which and how these military powers should be complaint-affidavit before the prosecutor’s office or file a direct complaint before the
exercised. appropriate courts against erring parties.

TENTH ISSUE:

Whether or not the petitioners were able to comply with all the requisites for the issuance of
an injunctive writ. NO.

By jurisprudence, to be entitled to an injunctive writ, petitioners have the burden to establish


the following requisites: (1) a right in esse or a clear and unmistakable right to be protected;
(2) a violation of that right; (3) that there is an urgent and permanent act and urgent
necessity for the writ to prevent serious damage; and (4) no other ordinary, speedy, and
adequate remedy exists to prevent the infliction of irreparable injury.

Petitioners anchored their prayer for the issuance of an injunctive writ on respondents’ gross
transgressions of the Constitution when they extended the martial law in Mindanao for one
year. The Lagman petition likewise alleges that petitioner Villarin, a Davao City resident, is
personally prejudiced by the extension or martial law in Mindanao “which would spawn
violations of civil liberties of Mindanaoans like petitioner Villarin who is a steadfast critic of
the Duterte administration and of the brutalities committed by police and military forces”.

The alleged violations of the petitioners’ civil liberties do not justify the grant of injunctive
relief. The petitioners failed to prove that the alleged violations are directly attributable to
the imposition of martial law. They likewise failed to establish the nexus between the
President’s exercise of his martial law powers and their unfounded apprehension that the
imposition “will target civilians who have no participation at all in any armed uprising or
struggle”. Incidentally, petitioners failed to state what the “civil liberties” specifically refer to,
and how the extension of martial law in Mindanao would threaten these “civil liberties” in
derogation of the rule of law. Evidently, petitioners’ right is doubtful or disputed, and can
hardly be considered a clear legal right, sufficient for the grant of an injunctive writ.
321

EN BANC In a separate move, the House Committee, through Congressman Herminio G. Teves,
requested Executive Secretary Eduardo Ermita to furnish it with "all documents on the
G.R. No. 170516 July 16, 2008 subject including the latest draft of the proposed agreement, the requests and offers
etc."2 Acting on the request, Secretary Ermita, by letter of June 23, 2005, wrote Congressman
Teves as follows:
AKBAYAN CITIZENS ACTION PARTY ("AKBAYAN"), PAMBANSANG KATIPUNAN NG MGA
SAMAHAN SA KANAYUNAN ("PKSK"), ALLIANCE OF PROGRESSIVE LABOR ("APL"), VICENTE
A. FABE, ANGELITO R. MENDOZA, MANUEL P. QUIAMBAO, ROSE BEATRIX CRUZ-ANGELES, In its letter dated 15 June 2005 (copy enclosed), [the] D[epartment of] F[oreign] A[ffairs]
CONG. LORENZO R. TANADA III, CONG. MARIO JOYO AGUJA, CONG. LORETA ANN P. explains that the Committee’s request to be furnished all documents on the JPEPA may be
ROSALES, CONG. ANA THERESIA HONTIVEROS-BARAQUEL, AND CONG. EMMANUEL JOEL J. difficult to accomplish at this time, since the proposed Agreement has been a work in
VILLANUEVA, Petitioners, progress for about three years. A copy of the draft JPEPA will however be forwarded to the
vs. Committee as soon as the text thereof is settled and complete. (Emphasis supplied)
THOMAS G. AQUINO, in his capacity as Undersecretary of the Department of Trade and
Industry (DTI) and Chairman and Chief Delegate of the Philippine Coordinating Committee Congressman Aguja also requested NEDA Director-General Romulo Neri and Tariff
(PCC) for the Japan-Philippines Economic Partnership Agreement Respondents. Commission Chairman Edgardo Abon, by letter of July 1, 2005, for copies of the latest text of
the JPEPA.
DECISION Chairman Abon replied, however, by letter of July 12, 2005 that the Tariff Commission does
not have a copy of the documents being requested, albeit he was certain that Usec. Aquino
would provide the Congressman with a copy "once the negotiation is completed." And by
CARPIO MORALES, J.:
letter of July 18, 2005, NEDA Assistant Director-General Margarita R. Songco informed the
Congressman that his request addressed to Director-General Neri had been forwarded to
Petitioners – non-government organizations, Congresspersons, citizens and taxpayers – seek Usec. Aquino who would be "in the best position to respond" to the request.
via the present petition for mandamus and prohibition to obtain from respondents the full
text of the Japan-Philippines Economic Partnership Agreement (JPEPA) including the
In its third hearing conducted on August 31, 2005, the House Committee resolved to issue a
Philippine and Japanese offers submitted during the negotiation process and all pertinent
subpoena for the most recent draft of the JPEPA, but the same was not pursued because by
attachments and annexes thereto.
Committee Chairman Congressman Teves’ information, then House Speaker Jose de Venecia
had requested him to hold in abeyance the issuance of the subpoena until the President
Petitioners Congressmen Lorenzo R. Tañada III and Mario Joyo Aguja filed on January 25, gives her consent to the disclosure of the documents.3
2005 House Resolution No. 551 calling for an inquiry into the bilateral trade agreements then
being negotiated by the Philippine government, particularly the JPEPA. The Resolution
Amid speculations that the JPEPA might be signed by the Philippine government within
became the basis of an inquiry subsequently conducted by the House Special Committee on
December 2005, the present petition was filed on December 9, 2005.4 The agreement was to
Globalization (the House Committee) into the negotiations of the JPEPA.
be later signed on September 9, 2006 by President Gloria Macapagal-Arroyo and Japanese
Prime Minister Junichiro Koizumi in Helsinki, Finland, following which the President endorsed
In the course of its inquiry, the House Committee requested herein respondent it to the Senate for its concurrence pursuant to Article VII, Section 21 of the Constitution. To
Undersecretary Tomas Aquino (Usec. Aquino), Chairman of the Philippine Coordinating date, the JPEPA is still being deliberated upon by the Senate.
Committee created under Executive Order No. 213 ("Creation of A Philippine Coordinating
Committee to Study the Feasibility of the Japan-Philippines Economic Partnership
The JPEPA, which will be the first bilateral free trade agreement to be entered into by the
Agreement")1 to study and negotiate the proposed JPEPA, and to furnish the Committee with
Philippines with another country in the event the Senate grants its consent to it, covers a
a copy of the latest draft of the JPEPA. Usec. Aquino did not heed the request, however.
broad range of topics which respondents enumerate as follows: trade in goods, rules of
origin, customs procedures, paperless trading, trade in services, investment, intellectual
Congressman Aguja later requested for the same document, but Usec. Aquino, by letter of property rights, government procurement, movement of natural persons, cooperation,
November 2, 2005, replied that the Congressman shall be provided with a copy thereof "once competition policy, mutual recognition, dispute avoidance and settlement, improvement of
the negotiations are completed and as soon as a thorough legal review of the proposed the business environment, and general and final provisions.5
agreement has been conducted."
While the final text of the JPEPA has now been made accessible to the public since
September 11, 2006,6respondents do not dispute that, at the time the petition was filed up
322

to the filing of petitioners’ Reply – when the JPEPA was still being negotiated – the initial respective legal procedures necessary for entry into force of this Agreement have been
drafts thereof were kept from public view. completed. It shall remain in force unless terminated as provided for in Article
165.11 (Emphasis supplied)
Before delving on the substantive grounds relied upon by petitioners in support of the
petition, the Court finds it necessary to first resolve some material procedural issues. President Arroyo’s endorsement of the JPEPA to the Senate for concurrence is part of the
legal procedures which must be met prior to the agreement’s entry into force.
Standing
The text of the JPEPA having then been made accessible to the public, the petition has
For a petition for mandamus such as the one at bar to be given due course, it must be become moot and academic to the extent that it seeks the disclosure of the "full text"
instituted by a party aggrieved by the alleged inaction of any tribunal, corporation, board or thereof.
person which unlawfully excludes said party from the enjoyment of a legal
right.7 Respondents deny that petitioners have such standing to sue. "[I]n the interest of a The petition is not entirely moot, however, because petitioners seek to obtain, not merely
speedy and definitive resolution of the substantive issues raised," however, respondents the text of the JPEPA, but also the Philippine and Japanese offers in the course of the
consider it sufficient to cite a portion of the ruling in Pimentel v. Office of Executive negotiations.12
Secretary8 which emphasizes the need for a "personal stake in the outcome of the
controversy" on questions of standing. A discussion of the substantive issues, insofar as they impinge on petitioners’ demand for
access to the Philippine and Japanese offers, is thus in order.
In a petition anchored upon the right of the people to information on matters of public
concern, which is a public right by its very nature, petitioners need not show that they have Grounds relied upon by petitioners
any legal or special interest in the result, it being sufficient to show that they are citizens and,
therefore, part of the general public which possesses the right.9 As the present petition is
Petitioners assert, first, that the refusal of the government to disclose the documents bearing
anchored on the right to information and petitioners are all suing in their capacity as citizens
on the JPEPA negotiations violates their right to information on matters of public
and groups of citizens including petitioners-members of the House of Representatives who
concern13 and contravenes other constitutional provisions on transparency, such as that on
additionally are suing in their capacity as such, the standing of petitioners to file the present
the policy of full public disclosure of all transactions involving public interest.14 Second, they
suit is grounded in jurisprudence.
contend that non-disclosure of the same documents undermines their right to effective and
reasonable participation in all levels of social, political, and economic decision-
Mootness making.15 Lastly, they proffer that divulging the contents of the JPEPA only after the
agreement has been concluded will effectively make the Senate into a mere rubber stamp of
Considering, however, that "[t]he principal relief petitioners are praying for is the disclosure the Executive, in violation of the principle of separation of powers.
of the contents of the JPEPA prior to its finalization between the two States parties,"10 public
disclosure of the text of the JPEPA after its signing by the President, during the pendency of Significantly, the grounds relied upon by petitioners for the disclosure of the latest text of
the present petition, has been largely rendered moot and academic. the JPEPA are, except for the last, the same as those cited for the disclosure of the Philippine
and Japanese offers.
With the Senate deliberations on the JPEPA still pending, the agreement as it now stands
cannot yet be considered as final and binding between the two States. Article 164 of the The first two grounds relied upon by petitioners which bear on the merits of respondents’
JPEPA itself provides that the agreement does not take effect immediately upon the signing claim of privilege shall be discussed. The last, being purely speculatory given that the Senate
thereof. For it must still go through the procedures required by the laws of each country for is still deliberating on the JPEPA, shall not.
its entry into force, viz:
The JPEPA is a matter of public concern
Article 164
Entry into Force
To be covered by the right to information, the information sought must meet the threshold
requirement that it be a matter of public concern. Apropos is the teaching of Legaspi v. Civil
This Agreement shall enter into force on the thirtieth day after the date on which the Service Commission:
Governments of the Parties exchange diplomatic notes informing each other that their
323

In determining whether or not a particular information is of public concern there is no rigid abandoned. Furthermore, the negotiations of the representatives of the Philippines as well
test which can be applied. ‘Public concern’ like ‘public interest’ is a term that eludes exact as of Japan must be allowed to explore alternatives in the course of the negotiations in the
definition. Both terms embrace a broad spectrum of subjects which the public may want to same manner as judicial deliberations and working drafts of opinions are accorded strict
know, either because these directly affect their lives, or simply because such matters confidentiality.22 (Emphasis and underscoring supplied)
naturally arouse the interest of an ordinary citizen. In the final analysis, it is for the courts to
determine on a case by case basis whether the matter at issue is of interest or importance, as The ground relied upon by respondents is thus not simply that the information sought
it relates to or affects the public.16(Underscoring supplied) involves a diplomatic matter, but that it pertains to diplomatic negotiations then in progress.

From the nature of the JPEPA as an international trade agreement, it is evident that the Privileged character of diplomatic negotiations
Philippine and Japanese offers submitted during the negotiations towards its execution are
matters of public concern. This, respondents do not dispute. They only claim that diplomatic
The privileged character of diplomatic negotiations has been recognized in this jurisdiction. In
negotiations are covered by the doctrine of executive privilege, thus constituting an
discussing valid limitations on the right to information, the Court in Chavez v. PCGG held that
exception to the right to information and the policy of full public disclosure.
"information on inter-government exchanges prior to the conclusion of treaties and
executive agreements may be subject to reasonable safeguards for the sake of national
Respondents’ claim of privilege interest."23 Even earlier, the same privilege was upheld in People’s Movement for Press
Freedom (PMPF) v. Manglapus24 wherein the Court discussed the reasons for the privilege in
It is well-established in jurisprudence that neither the right to information nor the policy of more precise terms.
full public disclosure is absolute, there being matters which, albeit of public concern or public
interest, are recognized as privileged in nature. The types of information which may be In PMPF v. Manglapus, the therein petitioners were seeking information from the President’s
considered privileged have been elucidated in Almonte v. Vasquez,17Chavez v. representatives on the state of the then on-going negotiations of the RP-US Military Bases
PCGG,18 Chavez v. Public Estate’s Authority,19 and most recently in Senate v. Ermita20 where Agreement.25 The Court denied the petition, stressing that "secrecy of negotiations with
the Court reaffirmed the validity of the doctrine of executive privilege in this jurisdiction and foreign countries is not violative of the constitutional provisions of freedom of speech or of
dwelt on its scope. the press nor of the freedom of access to information." The Resolution went on to state,
thus:
Whether a claim of executive privilege is valid depends on the ground invoked to justify it
and the context in which it is made.21 In the present case, the ground for respondents’ claim The nature of diplomacy requires centralization of authority and expedition of decision
of privilege is set forth in their Comment, viz: which are inherent in executive action. Another essential characteristic of diplomacy is its
confidential nature. Although much has been said about "open" and "secret" diplomacy,
x x x The categories of information that may be considered privileged includes matters of with disparagement of the latter, Secretaries of State Hughes and Stimson have clearly
diplomatic character and under negotiation and review. In this case, the privileged character analyzed and justified the practice. In the words of Mr. Stimson:
of the diplomatic negotiations has been categorically invoked and clearly explained by
respondents particularly respondent DTI Senior Undersecretary. "A complicated negotiation . . . cannot be carried through without many, many private
talks and discussion, man to man; many tentative suggestions and proposals. Delegates
The documents on the proposed JPEPA as well as the text which is subject to negotiations from other countries come and tell you in confidence of their troubles at home and of their
and legal review by the parties fall under the exceptions to the right of access to information differences with other countries and with other delegates; they tell you of what they
on matters of public concern and policy of public disclosure. They come within the coverage would do under certain circumstances and would not do under other circumstances. . . If
of executive privilege. At the time when the Committee was requesting for copies of such these reports . . . should become public . . . who would ever trust American Delegations in
documents, the negotiations were ongoing as they are still now and the text of the proposed another conference? (United States Department of State, Press Releases, June 7, 1930, pp.
JPEPA is still uncertain and subject to change. Considering the status and nature of such 282-284.)."
documents then and now, these are evidently covered by executive privilege consistent with
existing legal provisions and settled jurisprudence. There is frequent criticism of the secrecy in which negotiation with foreign powers on
nearly all subjects is concerned. This, it is claimed, is incompatible with the substance of
Practical and strategic considerations likewise counsel against the disclosure of the "rolling democracy. As expressed by one writer, "It can be said that there is no more rigid system of
texts" which may undergo radical change or portions of which may be totally silence anywhere in the world." (E.J. Young, Looking Behind the Censorship, J. B. Lippincott
324

Co., 1938) President Wilson in starting his efforts for the conclusion of the World War take essential to successful negotiation. As Sissela Bok points out, if "negotiators have more
declared that we must have "open covenants, openly arrived at." He quickly abandoned his to gain from being approved by their own sides than by making a reasoned agreement with
thought. competitors or adversaries, then they are inclined to 'play to the gallery . . .'' In fact, the
public reaction may leave them little option. It would be a brave, or foolish, Arab leader
No one who has studied the question believes that such a method of publicity is possible. In who expressed publicly a willingness for peace with Israel that did not involve the return of
the moment that negotiations are started, pressure groups attempt to "muscle in." An ill- the entire West Bank, or Israeli leader who stated publicly a willingness to remove Israel's
timed speech by one of the parties or a frank declaration of the concession which are existing settlements from Judea and Samaria in return for peace.28 (Emphasis supplied)
exacted or offered on both sides would quickly lead to widespread propaganda to block
the negotiations. After a treaty has been drafted and its terms are fully published, there is Indeed, by hampering the ability of our representatives to compromise, we may be
ample opportunity for discussion before it is approved. (The New American Government jeopardizing higher national goals for the sake of securing less critical ones.
and Its Works, James T. Young, 4th Edition, p. 194) (Emphasis and underscoring supplied)
Diplomatic negotiations, therefore, are recognized as privileged in this jurisdiction, the JPEPA
Still in PMPF v. Manglapus, the Court adopted the doctrine in U.S. v. Curtiss-Wright Export negotiations constituting no exception. It bears emphasis, however, that such privilege is
Corp.26 that the President is the sole organ of the nation in its negotiations with foreign only presumptive. For as Senate v. Ermita holds, recognizing a type of information as
countries, viz: privileged does not mean that it will be considered privileged in all instances. Only after a
consideration of the context in which the claim is made may it be determined if there is a
"x x x In this vast external realm, with its important, complicated, delicate and manifold public interest that calls for the disclosure of the desired information, strong enough to
problems, the President alone has the power to speak or listen as a representative of the overcome its traditionally privileged status.
nation. He makes treaties with the advice and consent of the Senate; but he alone
negotiates. Into the field of negotiation the Senate cannot intrude; and Congress itself is Whether petitioners have established the presence of such a public interest shall be
powerless to invade it. As Marshall said in his great argument of March 7, 1800, in the House discussed later. For now, the Court shall first pass upon the arguments raised by petitioners
of Representatives, "The President is the sole organ of the nation in its external relations, against the application of PMPF v. Manglapus to the present case.
and its sole representative with foreign nations." Annals, 6th Cong., col. 613. . . (Emphasis
supplied; underscoring in the original) Arguments proffered by petitioners against the application of PMPF v. Manglapus

Applying the principles adopted in PMPF v. Manglapus, it is clear that while the final text of Petitioners argue that PMPF v. Manglapus cannot be applied in toto to the present case,
the JPEPA may not be kept perpetually confidential – since there should be "ample there being substantial factual distinctions between the two.
opportunity for discussion before [a treaty] is approved" – the offers exchanged by the
parties during the negotiations continue to be privileged even after the JPEPA is published. It
To petitioners, the first and most fundamental distinction lies in the nature of the treaty
is reasonable to conclude that the Japanese representatives submitted their offers with the
involved. They stress that PMPF v. Manglapus involved the Military Bases Agreement which
understanding that "historic confidentiality"27 would govern the same. Disclosing these
necessarily pertained to matters affecting national security; whereas the present case
offers could impair the ability of the Philippines to deal not only with Japan but with other
involves an economic treaty that seeks to regulate trade and commerce between the
foreign governments in future negotiations.
Philippines and Japan, matters which, unlike those covered by the Military Bases Agreement,
are not so vital to national security to disallow their disclosure.
A ruling that Philippine offers in treaty negotiations should now be open to public scrutiny
would discourage future Philippine representatives from frankly expressing their views
Petitioners’ argument betrays a faulty assumption that information, to be considered
during negotiations. While, on first impression, it appears wise to deter Philippine
privileged, must involve national security. The recognition in Senate v. Ermita29 that
representatives from entering into compromises, it bears noting that treaty negotiations, or
executive privilege has encompassed claims of varying kinds, such that it may even be more
any negotiation for that matter, normally involve a process of quid pro quo, and oftentimes
accurate to speak of "executive privileges," cautions against such generalization.
negotiators have to be willing to grant concessions in an area of lesser importance in order
to obtain more favorable terms in an area of greater national interest. Apropos are the
following observations of Benjamin S. Duval, Jr.: While there certainly are privileges grounded on the necessity of safeguarding national
security such as those involving military secrets, not all are founded thereon. One example is
the "informer’s privilege," or the privilege of the Government not to disclose the identity of a
x x x [T]hose involved in the practice of negotiations appear to be in agreement that publicity
person or persons who furnish information of violations of law to officers charged with the
leads to "grandstanding," tends to freeze negotiating positions, and inhibits the give-and-
325

enforcement of that law.30 The suspect involved need not be so notorious as to be a threat to The earlier discussion on PMPF v. Manglapus36 shows that the privilege for diplomatic
national security for this privilege to apply in any given instance. Otherwise, the privilege negotiations is meant to encourage a frank exchange of exploratory ideas between the
would be inapplicable in all but the most high-profile cases, in which case not only would this negotiating parties by shielding such negotiations from public view. Similar to the privilege
be contrary to long-standing practice. It would also be highly prejudicial to law enforcement for presidential communications, the diplomatic negotiations privilege seeks, through the
efforts in general. same means, to protect the independence in decision-making of the President, particularly in
its capacity as "the sole organ of the nation in its external relations, and its sole
Also illustrative is the privilege accorded to presidential communications, which are representative with foreign nations." And, as with the deliberative process privilege, the
presumed privileged without distinguishing between those which involve matters of national privilege accorded to diplomatic negotiations arises, not on account of the content of the
security and those which do not, the rationale for the privilege being that information per se, but because the information is part of a process of deliberation which, in
pursuit of the public interest, must be presumed confidential.
x x x [a] frank exchange of exploratory ideas and assessments, free from the glare of publicity
and pressure by interested parties, is essential to protect the independence of decision- The decision of the U.S. District Court, District of Columbia in Fulbright & Jaworski v.
making of those tasked to exercise Presidential, Legislative and Judicial power. x x Department of the Treasury37enlightens on the close relation between diplomatic
x31 (Emphasis supplied) negotiations and deliberative process privileges. The plaintiffs in that case sought access to
notes taken by a member of the U.S. negotiating team during the U.S.-
French tax treatynegotiations. Among the points noted therein were the issues to be
In the same way that the privilege for judicial deliberations does not depend on the nature of
discussed, positions which the French and U.S. teams took on some points, the draft
the case deliberated upon, so presidential communications are privileged whether they
language agreed on, and articles which needed to be amended. Upholding the confidentiality
involve matters of national security.
of those notes, Judge Green ruled, thus:

It bears emphasis, however, that the privilege accorded to presidential communications is


Negotiations between two countries to draft a treaty represent a true example of a
not absolute, one significant qualification being that "the Executive cannot, any more than
deliberative process. Much give-and-take must occur for the countries to reach an
the other branches of government, invoke a general confidentiality privilege to shield its
accord. A description of the negotiations at any one point would not provide an onlooker a
officials and employees from investigations by the proper governmental institutions
summary of the discussions which could later be relied on as law. It would not be "working
into possible criminal wrongdoing." 32 This qualification applies whether the privilege is being
law" as the points discussed and positions agreed on would be subject to change at any date
invoked in the context of a judicial trial or a congressional investigation conducted in aid of
until the treaty was signed by the President and ratified by the Senate.
legislation.33

The policies behind the deliberative process privilege support non-disclosure. Much harm
Closely related to the "presidential communications" privilege is the deliberative process
could accrue to the negotiations process if these notes were revealed. Exposure of the pre-
privilege recognized in the United States. As discussed by the U.S. Supreme Court in NLRB v.
agreement positions of the French negotiators might well offend foreign governments and
Sears, Roebuck & Co,34 deliberative process covers documents reflecting advisory opinions,
would lead to less candor by the U. S. in recording the events of the negotiations
recommendations and deliberations comprising part of a process by which governmental
process. As several months pass in between negotiations, this lack of record could hinder
decisions and policies are formulated. Notably, the privileged status of such documents
readily the U. S. negotiating team. Further disclosure would reveal prematurely adopted
rests, not on the need to protect national security but, on the "obvious realization that
policies. If these policies should be changed, public confusion would result easily.
officials will not communicate candidly among themselves if each remark is a potential item
of discovery and front page news," the objective of the privilege being to enhance the quality
of agency Finally, releasing these snapshot views of the negotiations would be comparable to
decisionshttp://web2.westlaw.com/find/default.wl?rs=WLW7.07&serialnum=1975129772&f releasing drafts of the treaty, particularly when the notes state the tentative provisions
n=_top&sv=Split&tc=-1&findtype=Y&tf=-1&db=708&utid=%7b532A6DBF-9B4C-4A5A-8F16- and language agreed on. As drafts of regulations typically are protected by the deliberative
C20D9BAA36C4%7d&vr=2.0&rp=%2ffind%2fdefault.wl&mt=WLIGeneralSubscription. 35 process privilege, Arthur Andersen & Co. v. Internal Revenue Service, C.A. No. 80-705
(D.C.Cir., May 21, 1982), drafts of treaties should be accorded the same
protection. (Emphasis and underscoring supplied)
The diplomatic negotiations privilege bears a close resemblance to the deliberative process
and presidential communications privilege. It may be readily perceived that the rationale for
the confidential character of diplomatic negotiations, deliberative process, and presidential Clearly, the privilege accorded to diplomatic negotiations follows as a logical consequence
communications is similar, if not identical. from the privileged character of the deliberative process.
326

The Court is not unaware that in Center for International Environmental Law (CIEL), et al. v. It need not be stressed that in CIEL, the court ordered the disclosure of information based on
Office of U.S. Trade Representative38 – where the plaintiffs sought information relating to the its finding that the first requirement of FOIA Exemption 5 – that the documents be inter-
just-completed negotiation of a United States-Chile Free Trade Agreement – the same district agency – was not met. In determining whether the government may validly refuse disclosure
court, this time under Judge Friedman, consciously refrained from applying the doctrine of the exchanges between the U.S. and Chile, it necessarily had to deal with this requirement,
in Fulbright and ordered the disclosure of the information being sought. it being laid down by a statute binding on them.

Since the factual milieu in CIEL seemed to call for the straight application of the doctrine In this jurisdiction, however, there is no counterpart of the FOIA, nor is there any statutory
in Fulbright, a discussion of why the district court did not apply the same would help illumine requirement similar to FOIA Exemption 5 in particular. Hence, Philippine courts, when
this Court’s own reasons for deciding the present case along the lines of Fulbright. assessing a claim of privilege for diplomatic negotiations, are more free to focus directly on
the issue of whether the privilege being claimed is indeed supported by public policy, without
In both Fulbright and CIEL, the U.S. government cited a statutory basis for withholding having to consider – as the CIEL court did – if these negotiations fulfill a formal requirement
information, namely, Exemption 5 of the Freedom of Information Act (FOIA).39 In order to of being "inter-agency." Important though that requirement may be in the context of
qualify for protection under Exemption 5, a document must satisfy two conditions: (1) it must domestic negotiations, it need not be accorded the same significance when dealing with
be either inter-agency or intra-agency in nature, and (2) it must be both pre-decisional and international negotiations.
part of the agency's deliberative or decision-making process.40
There being a public policy supporting a privilege for diplomatic negotiations for the reasons
Judge Friedman, in CIEL, himself cognizant of a "superficial similarity of context" between the explained above, the Court sees no reason to modify, much less abandon, the doctrine
two cases, based his decision on what he perceived to be a significant distinction: he found in PMPF v. Manglapus.
the negotiator’s notes that were sought in Fulbright to be "clearly internal," whereas the
documents being sought in CIEL were those produced by or exchanged with an outside party, A second point petitioners proffer in their attempt to differentiate PMPF v. Manglapus from
i.e. Chile. The documents subject of Fulbright being clearly internal in character, the question the present case is the fact that the petitioners therein consisted entirely of members of the
of disclosure therein turned not on the threshold requirement of Exemption 5 that the mass media, while petitioners in the present case include members of the House of
document be inter-agency, but on whether the documents were part of the agency's pre- Representatives who invoke their right to information not just as citizens but as members of
decisional deliberative process. On this basis, Judge Friedman found that "Judge Green's Congress.
discussion [in Fulbright] of the harm that could result from disclosure therefore is
irrelevant, since the documents at issue [in CIEL] are not inter-agency, and the Court does Petitioners thus conclude that the present case involves the right of members of Congress to
not reach the question of deliberative process." (Emphasis supplied) demand information on negotiations of international trade agreements from the Executive
branch, a matter which was not raised in PMPF v. Manglapus.
In fine, Fulbright was not overturned. The court in CIEL merely found the same to be
irrelevant in light of its distinct factual setting. Whether this conclusion was valid – a question While indeed the petitioners in PMPF v. Manglapus consisted only of members of the mass
on which this Court would not pass – the ruling in Fulbright that "[n]egotiations between two media, it would be incorrect to claim that the doctrine laid down therein has no bearing on a
countries to draft a treaty represent a true example of a deliberative process" was left controversy such as the present, where the demand for information has come from
standing, since the CIEL court explicitly stated that it did not reach the question of members of Congress, not only from private citizens.
deliberative process.
The privileged character accorded to diplomatic negotiations does not ipso facto lose all
Going back to the present case, the Court recognizes that the information sought by force and effect simply because the same privilege is now being claimed under different
petitioners includes documents produced and communicated by a party external to the circumstances. The probability of the claim succeeding in the new context might differ, but
Philippine government, namely, the Japanese representatives in the JPEPA negotiations, and to say that the privilege, as such, has no validity at all in that context is another matter
to that extent this case is closer to the factual circumstances of CIELthan those of Fulbright. altogether.

Nonetheless, for reasons which shall be discussed shortly, this Court echoes the principle The Court’s statement in Senate v. Ermita that "presidential refusals to furnish information
articulated in Fulbrightthat the public policy underlying the deliberative process privilege may be actuated by any of at least three distinct kinds of considerations [state secrets
requires that diplomatic negotiations should also be accorded privileged status, even if the privilege, informer’s privilege, and a generic privilege for internal deliberations], and may be
documents subject of the present case cannot be described as purely internal in character. asserted, with differing degrees of success, in the context of either judicial or legislative
327

investigations,"41 implies that a privilege, once recognized, may be invoked under different The following statement in Chavez v. PEA, however, suffices to show that the doctrine in
procedural settings. That this principle holds true particularly with respect to diplomatic both that case and Chavez v. PCGG with regard to the duty to disclose "definite propositions
negotiations may be inferred from PMPF v. Manglapus itself, where the Court held that it is of the government" does not apply to diplomatic negotiations:
the President alone who negotiates treaties, and not even the Senate or the House of
Representatives, unless asked, may intrude upon that process. We rule, therefore, that the constitutional right to information includes official information
on on-going negotiationsbefore a final contract. The information, however, must
Clearly, the privilege for diplomatic negotiations may be invoked not only against citizens’ constitute definite propositions by the government and should not cover recognized
demands for information, but also in the context of legislative investigations. exceptions like privileged information, military and diplomatic secrets and similar matters
affecting national security and public order. x x x46 (Emphasis and underscoring supplied)
Hence, the recognition granted in PMPF v. Manglapus to the privileged character of
diplomatic negotiations cannot be considered irrelevant in resolving the present case, the It follows from this ruling that even definite propositions of the government may not be
contextual differences between the two cases notwithstanding. disclosed if they fall under "recognized exceptions." The privilege for diplomatic negotiations
is clearly among the recognized exceptions, for the footnote to the immediately quoted
As third and last point raised against the application of PMPF v. Manglapus in this case, ruling cites PMPF v. Manglapus itself as an authority.
petitioners proffer that "the socio-political and historical contexts of the two cases are
worlds apart." They claim that the constitutional traditions and concepts prevailing at the Whether there is sufficient public interest to overcome the claim of privilege
time PMPF v. Manglapus came about, particularly the school of thought that the
requirements of foreign policy and the ideals of transparency were incompatible with each It being established that diplomatic negotiations enjoy a presumptive privilege against
other or the "incompatibility hypothesis," while valid when international relations were still disclosure, even against the demands of members of Congress for information, the Court
governed by power, politics and wars, are no longer so in this age of international shall now determine whether petitioners have shown the existence of a public interest
cooperation.42 sufficient to overcome the privilege in this instance.

Without delving into petitioners’ assertions respecting the "incompatibility hypothesis," the To clarify, there are at least two kinds of public interest that must be taken into account. One
Court notes that the ruling in PMPF v. Manglapus is grounded more on the nature of treaty is the presumed public interest in favor of keeping the subject information confidential,
negotiations as such than on a particular socio-political school of thought. If petitioners are which is the reason for the privilege in the first place, and the other is the public interest in
suggesting that the nature of treaty negotiations have so changed that "[a]n ill-timed speech favor of disclosure, the existence of which must be shown by the party asking for
by one of the parties or a frank declaration of the concession which are exacted or offered on information. 47
both sides" no longer "lead[s] to widespread propaganda to block the negotiations," or that
parties in treaty negotiations no longer expect their communications to be governed by
The criteria to be employed in determining whether there is a sufficient public interest in
historic confidentiality, the burden is on them to substantiate the same. This petitioners
favor of disclosure may be gathered from cases such as U.S. v. Nixon,48 Senate Select
failed to discharge.
Committee on Presidential Campaign Activities v. Nixon,49 and In re Sealed Case.50

Whether the privilege applies only at certain stages of the negotiation process
U.S. v. Nixon, which involved a claim of the presidential communications privilege against the
subpoena duces tecum of a district court in a criminal case, emphasized the need to balance
Petitioners admit that "diplomatic negotiations on the JPEPA are entitled to a reasonable such claim of privilege against the constitutional duty of courts to ensure a fair
amount of confidentiality so as not to jeopardize the diplomatic process." They argue, administration of criminal justice.
however, that the same is privileged "only at certain stages of the negotiating process, after
which such information must necessarily be revealed to the public."43 They add that the duty
x x x the allowance of the privilege to withhold evidence that is demonstrably relevant in a
to disclose this information was vested in the government when the negotiations moved
criminal trial would cut deeply into the guarantee of due process of law and gravely impair
from the formulation and exploratory stage to the firming up of definite propositions or
the basic function of the courts. A President’s acknowledged need for confidentiality in the
official recommendations, citing Chavez v. PCGG44 and Chavez v. PEA.45
communications of his office is general in nature, whereas the constitutional need for
production of relevant evidence in a criminal proceeding is specific and central to the fair
adjudication of a particular criminal case in the administration of justice. Without access to
specific facts a criminal prosecution may be totally frustrated. The President’s broad interest
328

in confidentiality of communications will not be vitiated by disclosure of a limited number of constitutional right to effectively participate in decision-making be brought to life in the
conversations preliminarily shown to have some bearing on the pending criminal cases. context of international trade agreements.
(Emphasis, italics and underscoring supplied)
Whether it can accurately be said that the Filipino people were not involved in the JPEPA
Similarly, Senate Select Committee v. Nixon,51 which involved a claim of the presidential negotiations is a question of fact which this Court need not resolve. Suffice it to state that
communications privilege against the subpoena duces tecum of a Senate committee, spoke respondents had presented documents purporting to show that public consultations were
of the need to balance such claim with the duty of Congress to perform its legislative conducted on the JPEPA. Parenthetically, petitioners consider these "alleged consultations"
functions. as "woefully selective and inadequate."53

The staged decisional structure established in Nixon v. Sirica was designed to ensure that the AT ALL EVENTS, since it is not disputed that the offers exchanged by the Philippine and
President and those upon whom he directly relies in the performance of his duties could Japanese representatives have not been disclosed to the public, the Court shall pass upon
continue to work under a general assurance that their deliberations would remain the issue of whether access to the documents bearing on them is, as petitioners claim,
confidential. So long as the presumption that the public interest favors confidentiality can essential to their right to participate in decision-making.
be defeated only by a strong showing of need by another institution of government- a
showing that the responsibilities of that institution cannot responsibly be fulfilled without The case for petitioners has, of course, been immensely weakened by the disclosure of the
access to records of the President's deliberations- we believed in Nixon v. Sirica, and full text of the JPEPA to the public since September 11, 2006, even as it is still being
continue to believe, that the effective functioning of the presidential office will not be deliberated upon by the Senate and, therefore, not yet binding on the Philippines. Were the
impaired. x x x Senate to concur with the validity of the JPEPA at this moment, there has already been, in
the words of PMPF v. Manglapus, "ample opportunity for discussion before [the treaty] is
The sufficiency of the Committee's showing of need has come to depend, therefore, approved."
entirely on whether the subpoenaed materials are critical to the performance of its
legislative functions. The text of the JPEPA having been published, petitioners have failed to convince this Court
that they will not be able to meaningfully exercise their right to participate in decision-
In re Sealed Case52 involved a claim of the deliberative process and presidential making unless the initial offers are also published.
communications privileges against a subpoena duces tecum of a grand jury. On the claim of
deliberative process privilege, the court stated: It is of public knowledge that various non-government sectors and private citizens have
already publicly expressed their views on the JPEPA, their comments not being limited to
The deliberative process privilege is a qualified privilege and can be overcome by a sufficient general observations thereon but on its specific provisions. Numerous articles and
showing of need. This need determination is to be made flexibly on a case-by-case, ad hoc statements critical of the JPEPA have been posted on the Internet. 54 Given these
basis. "[E]ach time [the deliberative process privilege] is asserted the district court must developments, there is no basis for petitioners’ claim that access to the Philippine and
undertake a fresh balancing of the competing interests," taking into account factors such as Japanese offers is essential to the exercise of their right to participate in decision-making.
"the relevance of the evidence," "the availability of other evidence," "the seriousness of
the litigation," "the role of the government," and the "possibility of future timidity by Petitioner-members of the House of Representatives additionally anchor their claim to have
government employees. a right to the subject documents on the basis of Congress’ inherent power to regulate
commerce, be it domestic or international. They allege that Congress cannot meaningfully
Petitioners have failed to present the strong and "sufficient showing of need" referred to in exercise the power to regulate international trade agreements such as the JPEPA without
the immediately cited cases. The arguments they proffer to establish their entitlement to the being given copies of the initial offers exchanged during the negotiations thereof. In the
subject documents fall short of this standard. same vein, they argue that the President cannot exclude Congress from the JPEPA
negotiations since whatever power and authority the President has to negotiate
Petitioners go on to assert that the non-involvement of the Filipino people in the JPEPA international trade agreements is derived only by delegation of Congress, pursuant to Article
negotiation process effectively results in the bargaining away of their economic and property VI, Section 28(2) of the Constitution and Sections 401 and 402 of Presidential Decree No.
rights without their knowledge and participation, in violation of the due process clause of the 1464.55
Constitution. They claim, moreover, that it is essential for the people to have access to the
initial offers exchanged during the negotiations since only through such disclosure can their
329

The subject of Article VI Section 28(2) of the Constitution is not the power to negotiate foreign relations. In the realm of treaty-making, the President has the sole authority to
treaties and international agreements, but the power to fix tariff rates, import and export negotiate with other states.
quotas, and other taxes. Thus it provides:
Nonetheless, while the President has the sole authority to negotiate and enter into
(2) The Congress may, by law, authorize the President to fix within specified limits, and treaties, the Constitution provides a limitation to his power by requiring the concurrence of
subject to such limitations and restrictions as it may impose, tariff rates, import and export 2/3 of all the members of the Senate for the validity of the treaty entered into by him. x x x
quotas, tonnage and wharfage dues, and other duties or imposts within the framework of (Emphasis and underscoring supplied)
the national development program of the Government.
While the power then to fix tariff rates and other taxes clearly belongs to Congress, and is
As to the power to negotiate treaties, the constitutional basis thereof is Section 21 of Article exercised by the President only by delegation of that body, it has long been recognized that
VII – the article on the Executive Department – which states: the power to enter into treaties is vested directly and exclusively in the President, subject
only to the concurrence of at least two-thirds of all the Members of the Senate for the
No treaty or international agreement shall be valid and effective unless concurred in by at validity of the treaty. In this light, the authority of the President to enter into trade
least two-thirds of all the Members of the Senate. agreements with foreign nations provided under P.D. 146458 may be interpreted as an
acknowledgment of a power already inherent in its office. It may not be used as basis to hold
the President or its representatives accountable to Congress for the conduct of treaty
The doctrine in PMPF v. Manglapus that the treaty-making power is exclusive to the
negotiations.
President, being the sole organ of the nation in its external relations, was echoed in BAYAN v.
Executive Secretary56 where the Court held:
This is not to say, of course, that the President’s power to enter into treaties is unlimited but
for the requirement of Senate concurrence, since the President must still ensure that all
By constitutional fiat and by the intrinsic nature of his office, the President, as head of State,
treaties will substantively conform to all the relevant provisions of the Constitution.
is the sole organ and authority in the external affairs of the country. In many ways, the
President is the chief architect of the nation's foreign policy; his "dominance in the field of
foreign relations is (then) conceded." Wielding vast powers and influence, his conduct in the It follows from the above discussion that Congress, while possessing vast legislative powers,
external affairs of the nation, as Jefferson describes, is "executive altogether." may not interfere in the field of treaty negotiations. While Article VII, Section 21 provides for
Senate concurrence, such pertains only to the validity of the treaty under consideration, not
to the conduct of negotiations attendant to its conclusion. Moreover, it is not even Congress
As regards the power to enter into treaties or international agreements, the Constitution
as a whole that has been given the authority to concur as a means of checking the treaty-
vests the same in the President, subject only to the concurrence of at least two thirds vote
making power of the President, but only the Senate.
of all the members of the Senate. In this light, the negotiation of the VFA and the
subsequent ratification of the agreement are exclusive acts which pertain solely to the
President, in the lawful exercise of his vast executive and diplomatic powers granted him no Thus, as in the case of petitioners suing in their capacity as private citizens, petitioners-
less than by the fundamental law itself. Into the field of negotiation the Senate cannot members of the House of Representatives fail to present a "sufficient showing of need" that
intrude, and Congress itself is powerless to invade it. x x x (Italics in the original; emphasis the information sought is critical to the performance of the functions of Congress, functions
and underscoring supplied) that do not include treaty-negotiation.

The same doctrine was reiterated even more recently in Pimentel v. Executive Respondents’ alleged failure to timely claim executive privilege
Secretary57 where the Court ruled:
On respondents’ invocation of executive privilege, petitioners find the same defective, not
In our system of government, the President, being the head of state, is regarded as the sole having been done seasonably as it was raised only in their Comment to the present petition
organ and authority in external relations and is the country's sole representative with and not during the House Committee hearings.
foreign nations. As the chief architect of foreign policy, the President acts as the country's
mouthpiece with respect to international affairs. Hence, the President is vested with the That respondents invoked the privilege for the first time only in their Comment to the
authority to deal with foreign states and governments, extend or withhold recognition, present petition does not mean that the claim of privilege should not be credited.
maintain diplomatic relations, enter into treaties, and otherwise transact the business of Petitioners’ position presupposes that an assertion of the privilege should have been made
330

during the House Committee investigations, failing which respondents are deemed to have Response to the Dissenting Opinion of the Chief Justice
waived it.
We are aware that behind the dissent of the Chief Justice lies a genuine zeal to protect our
When the House Committee and petitioner-Congressman Aguja requested respondents for people’s right to information against any abuse of executive privilege. It is a zeal that We fully
copies of the documents subject of this case, respondents replied that the negotiations were share.
still on-going and that the draft of the JPEPA would be released once the text thereof is
settled and complete. There was no intimation that the requested copies are confidential in The Court, however, in its endeavor to guard against the abuse of executive privilege, should
nature by reason of public policy. The response may not thus be deemed a claim of privilege be careful not to veer towards the opposite extreme, to the point that it would strike down
by the standards of Senate v. Ermita, which recognizes as claims of privilege only those which as invalid even a legitimate exercise thereof.
are accompanied by precise and certain reasons for preserving the confidentiality of the
information being sought.
We respond only to the salient arguments of the Dissenting Opinion which have not yet been
sufficiently addressed above.
Respondents’ failure to claim the privilege during the House Committee hearings may not,
however, be construed as a waiver thereof by the Executive branch. As the immediately
1. After its historical discussion on the allocation of power over international trade
preceding paragraph indicates, what respondents received from the House Committee and
agreements in the United States, the dissent concludes that "it will be turning somersaults
petitioner-Congressman Aguja were mere requests for information. And as priorly stated, the
with history to contend that the President is the sole organ for external relations" in that
House Committee itself refrained from pursuing its earlier resolution to issue a subpoena
jurisdiction. With regard to this opinion, We make only the following observations:
duces tecum on account of then Speaker Jose de Venecia’s alleged request to Committee
Chairperson Congressman Teves to hold the same in abeyance.
There is, at least, a core meaning of the phrase "sole organ of the nation in its external
relations" which is not being disputed, namely, that the power to directly negotiate treaties
While it is a salutary and noble practice for Congress to refrain from issuing subpoenas to
and international agreements is vested by our Constitution only in the Executive. Thus, the
executive officials – out of respect for their office – until resort to it becomes necessary, the
dissent states that "Congress has the power to regulate commerce with foreign nations but
fact remains that such requests are not a compulsory process. Being mere requests, they do
does not have the power to negotiate international agreements directly."62
not strictly call for an assertion of executive privilege.

What is disputed is how this principle applies to the case at bar.


The privilege is an exemption to Congress’ power of inquiry. 59 So long as Congress itself finds
no cause to enforce such power, there is no strict necessity to assert the privilege. In this
light, respondents’ failure to invoke the privilege during the House Committee investigations The dissent opines that petitioner-members of the House of Representatives, by asking for
did not amount to a waiver thereof. the subject JPEPA documents, are not seeking to directly participate in the negotiations of
the JPEPA, hence, they cannot be prevented from gaining access to these documents.
The Court observes, however, that the claim of privilege appearing in respondents’ Comment
to this petition fails to satisfy in full the requirement laid down in Senate v. Ermita that the On the other hand, We hold that this is one occasion where the following ruling in Agan v.
claim should be invoked by the President or through the Executive Secretary "by order of the PIATCO63 – and in other cases both before and since – should be applied:
President."60 Respondents’ claim of privilege is being sustained, however, its flaw
notwithstanding, because of circumstances peculiar to the case. This Court has long and consistently adhered to the legal maxim that those that cannot be
done directly cannot be done indirectly. To declare the PIATCO contracts valid despite the
The assertion of executive privilege by the Executive Secretary, who is one of the clear statutory prohibition against a direct government guarantee would not only make a
respondents herein, without him adding the phrase "by order of the President," shall be mockery of what the BOT Law seeks to prevent -- which is to expose the government to the
considered as partially complying with the requirement laid down in Senate v. Ermita. The risk of incurring a monetary obligation resulting from a contract of loan between the project
requirement that the phrase "by order of the President" should accompany the Executive proponent and its lenders and to which the Government is not a party to -- but would also
Secretary’s claim of privilege is a new rule laid down for the first time in Senate v. Ermita, render the BOT Law useless for what it seeks to achieve –- to make use of the resources of the
which was not yet final and executory at the time respondents filed their Comment to the private sector in the "financing, operation and maintenance of infrastructure and
petition.61 A strict application of this requirement would thus be unwarranted in this case. development projects" which are necessary for national growth and development but which
the government, unfortunately, could ill-afford to finance at this point in time.64
331

Similarly, while herein petitioners-members of the House of Representatives may not have itself, that they were claiming confidentiality not only until, but even after, the conclusion
been aiming to participate in the negotiations directly, opening the JPEPA negotiations to of the negotiations.
their scrutiny – even to the point of giving them access to the offers exchanged between the
Japanese and Philippine delegations – would have made a mockery of what the Constitution Judicial deliberations do not lose their confidential character once a decision has been
sought to prevent and rendered it useless for what it sought to achieve when it vested the promulgated by the courts. The same holds true with respect to working drafts of opinions,
power of direct negotiation solely with the President. which are comparable to intra-agencyrecommendations. Such intra-agency
recommendations are privileged even after the position under consideration by the agency
What the U.S. Constitution sought to prevent and aimed to achieve in defining the treaty- has developed into a definite proposition, hence, the rule in this jurisdiction that agencies
making power of the President, which our Constitution similarly defines, may be gathered have the duty to disclose only definite propositions, and not the inter-agency and intra-
from Hamilton’s explanation of why the U.S. Constitution excludes the House of agency communications during the stage when common assertions are still being
Representatives from the treaty-making process: formulated.67

x x x The fluctuating, and taking its future increase into account, the multitudinous 3. The dissent claims that petitioner-members of the House of Representatives have
composition of that body, forbid us to expect in it those qualities which are essential to the sufficiently shown their need for the same documents to overcome the privilege. Again, We
proper execution of such a trust. Accurate and comprehensive knowledge of foreign politics; disagree.
a steady and systematic adherence to the same views; a nice and uniform sensibility to
national character, decision, secrecy and dispatch; are incompatible with a body so variable The House Committee that initiated the investigations on the JPEPA did not pursue its earlier
and so numerous. The very complication of the business by introducing a necessity of the intention to subpoena the documents. This strongly undermines the assertion that access to
concurrence of so many different bodies, would of itself afford a solid objection. The greater the same documents by the House Committee is critical to the performance of its legislative
frequency of the calls upon the house of representatives, and the greater length of time functions. If the documents were indeed critical, the House Committee should have, at the
which it would often be necessary to keep them together when convened, to obtain their very least, issued a subpoena duces tecum or, like what the Senate did in Senate v. Ermita,
sanction in the progressive stages of a treaty, would be source of so great inconvenience and filed the present petition as a legislative body, rather than leaving it to the discretion of
expense, as alone ought to condemn the project.65 individual Congressmen whether to pursue an action or not. Such acts would have served as
strong indicia that Congress itself finds the subject information to be critical to its legislative
These considerations a fortiori apply in this jurisdiction, since the Philippine functions.
Constitution, unlike that of the U.S., does not even grant the Senate the power to advise the
Executive in the making of treaties, but only vests in that body the power to concur in the Further, given that respondents have claimed executive privilege, petitioner-members of the
validity of the treaty after negotiations have been concluded.66 Much less, therefore, should House of Representatives should have, at least, shown how its lack of access to the Philippine
it be inferred that the House of Representatives has this power. and Japanese offers would hinder the intelligent crafting of legislation. Mere assertion that
the JPEPA covers a subject matter over which Congress has the power to legislate would not
Since allowing petitioner-members of the House of Representatives access to the subject suffice. As Senate Select Committee v. Nixon68 held, the showing required to overcome the
JPEPA documents would set a precedent for future negotiations, leading to the presumption favoring confidentiality turns, not only on the nature and appropriateness of
contravention of the public interests articulated above which the Constitution sought to the function in the performance of which the material was sought, but also the degree to
protect, the subject documents should not be disclosed. which the material was necessary to its fulfillment. This petitioners failed to do.

2. The dissent also asserts that respondents can no longer claim the diplomatic secrets Furthermore, from the time the final text of the JPEPA including its annexes and attachments
privilege over the subject JPEPA documents now that negotiations have been concluded, was published, petitioner-members of the House of Representatives have been free to use it
since their reasons for nondisclosure cited in the June 23, 2005 letter of Sec. Ermita, and later for any legislative purpose they may see fit. Since such publication, petitioners’ need, if
in their Comment, necessarily apply only for as long as the negotiations were still pending; any, specifically for the Philippine and Japanese offers leading to the final version of the
JPEPA, has become even less apparent.
In their Comment, respondents contend that "the negotiations of the representatives of the
Philippines as well as of Japan must be allowed to explore alternatives in the course of the In asserting that the balance in this instance tilts in favor of disclosing the JPEPA documents,
negotiations in the same manner as judicial deliberations and working drafts of opinions are the dissent contends that the Executive has failed to show how disclosing them after the
accorded strict confidentiality." That respondents liken the documents involved in the conclusion of negotiations would impair the performance of its functions. The contention,
JPEPA negotiations to judicial deliberations and working drafts of opinions evinces, by with due respect, misplaces the onus probandi. While, in keeping with the general
332

presumption of transparency, the burden is initially on the Executive to provide precise and That the Court could freely cite Curtiss-Wright – a case that upholds the secrecy of diplomatic
certain reasons for upholding its claim of privilege, once the Executive is able to show that negotiations against congressional demands for information – in the course of laying down a
the documents being sought are covered by a recognized privilege, the burden shifts to the ruling on the public right to information only serves to underscore the principle mentioned
party seeking information to overcome the privilege by a strong showing of need. earlier that the privileged character accorded to diplomatic negotiations does not ipso
facto lose all force and effect simply because the same privilege is now being claimed under
When it was thus established that the JPEPA documents are covered by the privilege for different circumstances.
diplomatic negotiations pursuant to PMPF v. Manglapus, the presumption arose that their
disclosure would impair the performance of executive functions. It was then incumbent on PMPF v. Manglapus indeed involved a demand for information from private citizens and not
petitioner- requesting parties to show that they have a strong need for the information an executive-legislative conflict, but so did Chavez v. PEA74 which held that "the
sufficient to overcome the privilege. They have not, however. [public’s] right to information . . . does not extend to matters recognized as privileged
information under the separation of powers." What counts as privileged information in an
4. Respecting the failure of the Executive Secretary to explicitly state that he is claiming the executive-legislative conflict is thus also recognized as such in cases involving the public’s
privilege "by order of the President," the same may not be strictly applied to the privilege right to information.
claim subject of this case.
Chavez v. PCGG75 also involved the public’s right to information, yet the Court recognized as
When the Court in Senate v. Ermita limited the power of invoking the privilege to the a valid limitation to that right the same privileged information based on separation of
President alone, it was laying down a new rule for which there is no counterpart even in the powers – closed-door Cabinet meetings, executive sessions of either house of Congress, and
United States from which the concept of executive privilege was adopted. As held in the 2004 the internal deliberations of the Supreme Court.
case of Judicial Watch, Inc. v. Department of Justice,69 citing In re Sealed Case,70 "the issue of
whether a President must personally invoke the [presidential communications] privilege These cases show that the Court has always regarded claims of privilege, whether in the
remains an open question." U.S. v. Reynolds,71 on the other hand, held that "[t]here must be context of an executive-legislative conflict or a citizen’s demand for information, as closely
a formal claim of privilege, lodged by the head of the department which has control over the intertwined, such that the principles applicable to one are also applicable to the other.
matter, after actual personal consideration by that officer."
The reason is obvious. If the validity of claims of privilege were to be assessed by entirely
The rule was thus laid down by this Court, not in adherence to any established precedent, different criteria in each context, this may give rise to the absurd result
but with the aim of preventing the abuse of the privilege in light of its highly exceptional where Congress would be denied access to a particular information because of a claim of
nature. The Court’s recognition that the Executive Secretary also bears the power to invoke executive privilege, but the general public would have access to the same information, the
the privilege, provided he does so "by order of the President," is meant to avoid laying down claim of privilege notwithstanding.
too rigid a rule, the Court being aware that it was laying down a new restriction on executive
privilege. It is with the same spirit that the Court should not be overly strict with applying the Absurdity would be the ultimate result if, for instance, the Court adopts the "clear and
same rule in this peculiar instance, where the claim of executive privilege occurred before present danger" test for the assessment of claims of privilege against citizens’ demands for
the judgment in Senate v. Ermitabecame final. information. If executive information, when demanded by a citizen, is privileged only when
there is a clear and present danger of a substantive evil that the State has a right to prevent,
5. To show that PMPF v. Manglapus may not be applied in the present case, the dissent it would be very difficult for the Executive to establish the validity of its claim in each
implies that the Court therein erred in citing US v. Curtiss Wright72 and the book entitled The instance. In contrast, if the demand comes from Congress, the Executive merely has to show
New American Government and Its Work73since these authorities, so the dissent claims, may that the information is covered by a recognized privilege in order to shift the burden on
not be used to calibrate the importance of the right to information in the Philippine setting. Congress to present a strong showing of need. This would lead to a situation where it would
be more difficult for Congress to access executive information than it would be for private
The dissent argues that since Curtiss-Wright referred to a conflict between the executive and citizens.
legislative branches of government, the factual setting thereof was different from that
of PMPF v. Manglapus which involved a collision between governmental power over the We maintain then that when the Executive has already shown that an information is covered
conduct of foreign affairs and the citizen’s right to information. by executive privilege, the party demanding the information must present a "strong showing
of need," whether that party is Congress or a private citizen.
333

The rule that the same "showing of need" test applies in both these contexts, however, the same would hold true even "if they simply want to know it because it interests them." As
should not be construed as a denial of the importance of analyzing the context in which an has been stated earlier, however, there is no dispute that the information subject of this case
executive privilege controversy may happen to be placed. Rather, it affirms it, for it means is a matter of public concern. The Court has earlier concluded that it is a matter of public
that the specific need being shown by the party seeking information in concern, not on the basis of any specific need shown by petitioners, but from the very nature
every particular instance is highly significant in determining whether to uphold a claim of of the JPEPA as an international trade agreement.
privilege. This "need" is, precisely, part of the context in light of which every claim of
privilege should be assessed. However, when the Executive has – as in this case – invoked the privilege, and it has been
established that the subject information is indeed covered by the privilege being claimed, can
Since, as demonstrated above, there are common principles that should be applied to a party overcome the same by merely asserting that the information being demanded is a
executive privilege controversies across different contexts, the Court in PMPF v. matter of public concern, without any further showing required? Certainly not, for that
Manglapus did not err when it cited the Curtiss-Wrightcase. would render the doctrine of executive privilege of no force and effect whatsoever as a
limitation on the right to information, because then the sole test in such controversies would
The claim that the book cited in PMPF v. Manglapus entitled The New American Government be whether an information is a matter of public concern.
and Its Work could not have taken into account the expanded statutory right to information
in the FOIA assumes that the observations in that book in support of the confidentiality of Moreover, in view of the earlier discussions, we must bear in mind that, by disclosing the
treaty negotiations would be different had it been written after the FOIA. Such assumption is, documents of the JPEPA negotiations, the Philippine government runs the grave risk of
with due respect, at best, speculative. betraying the trust reposed in it by the Japanese representatives, indeed, by the Japanese
government itself. How would the Philippine government then explain itself when that
As to the claim in the dissent that "[i]t is more doubtful if the same book be used to calibrate happens? Surely, it cannot bear to say that it just had to release the information because
the importance of the right of access to information in the Philippine setting considering its certain persons simply wanted to know it "because it interests them."
elevation as a constitutional right," we submit that the elevation of such right as a
constitutional right did not set it free from the legitimate restrictions of executive privilege Thus, the Court holds that, in determining whether an information is covered by the right to
which is itself constitutionally-based.76 Hence, the comments in that book which were cited information, a specific "showing of need" for such information is not a relevant
in PMPF v. Manglapus remain valid doctrine. consideration, but only whether the same is a matter of public concern. When, however, the
government has claimed executive privilege, and it has established that the information is
6. The dissent further asserts that the Court has never used "need" as a test to uphold or indeed covered by the same, then the party demanding it, if it is to overcome the privilege,
allow inroads into rights guaranteed under the Constitution. With due respect, we assert must show that that the information is vital, not simply for the satisfaction of its curiosity,
otherwise. The Court has done so before, albeit without using the term "need." but for its ability to effectively and reasonably participate in social, political, and economic
decision-making.79
In executive privilege controversies, the requirement that parties present a "sufficient
showing of need" only means, in substance, that they should show a public interest in favor 7. The dissent maintains that "[t]he treaty has thus entered the ultimate stage where the
of disclosure sufficient in degree to overcome the claim of privilege.77 Verily, the Court in people can exercise their right to participate in the discussion whether the Senate should
such cases engages in a balancing of interests. Such a balancing of interests is certainly not concur in its ratification or not." (Emphasis supplied) It adds that this right "will be diluted
new in constitutional adjudication involving fundamental rights. Secretary of Justice v. unless the people can have access to the subject JPEPA documents". What, to the dissent, is
Lantion,78 which was cited in the dissent, applied just such a test. a dilution of the right to participate in decision-making is, to Us, simply a recognition of the
qualified nature of the public’s right to information. It is beyond dispute that the right to
information is not absolute and that the doctrine of executive privilege is a recognized
Given that the dissent has clarified that it does not seek to apply the "clear and present
limitation on that right.
danger" test to the present controversy, but the balancing test, there seems to be no
substantial dispute between the position laid down in this ponencia and that reflected in the
dissent as to what test to apply. It would appear that the only disagreement is on the results Moreover, contrary to the submission that the right to participate in decision-making would
of applying that test in this instance. be diluted, We reiterate that our people have been exercising their right to participate in the
discussion on the issue of the JPEPA, and they have been able to articulate their different
opinions without need of access to the JPEPA negotiation documents.
The dissent, nonetheless, maintains that "it suffices that information is of public concern for
it to be covered by the right, regardless of the public’s need for the information," and that
Thus, we hold that the balance in this case tilts in favor of executive privilege.
334

8. Against our ruling that the principles applied in U.S. v. Nixon, the Senate Select case have not persuaded the Court. Moreover, petitioners – both private citizens and
Committee case, and In re Sealed Case, are similarly applicable to the present controversy, members of the House of Representatives – have failed to present a "sufficient showing of
the dissent cites the caveat in the Nixon case that the U.S. Court was there addressing only need" to overcome the claim of privilege in this case.
the President’s assertion of privilege in the context of a criminal trial, not a civil litigation nor
a congressional demand for information. What this caveat means, however, is only that That the privilege was asserted for the first time in respondents’ Comment to the present
courts must be careful not to hastily apply the ruling therein to other contexts. It does not, petition, and not during the hearings of the House Special Committee on Globalization, is of
however, absolutely mean that the principles applied in that case may never be applied in no moment, since it cannot be interpreted as a waiver of the privilege on the part of the
such contexts. Executive branch.

Hence, U.S. courts have cited U.S. v. Nixon in support of their rulings on claims of executive For reasons already explained, this Decision shall not be interpreted as departing from the
privilege in contexts other than a criminal trial, as in the case of Nixon v. Administrator of ruling in Senate v. Ermitathat executive privilege should be invoked by the President or
General Services80 – which involved former President Nixon’s invocation of executive through the Executive Secretary "by order of the President."
privilege to challenge the constitutionality of the "Presidential Recordings and Materials
Preservation Act"81 – and the above-mentioned In re Sealed Case which involved a claim of
WHEREFORE, the petition is DISMISSED.
privilege against a subpoena duces tecum issued in a grand jury investigation.

SO ORDERED.
Indeed, in applying to the present case the principles found in U.S. v. Nixon and in the other
cases already mentioned, We are merely affirming what the Chief Justice stated in his
Dissenting Opinion in Neri v. Senate Committee on Accountability82 – a case involving an G.R. No. 170516, July 16, 2008
executive-legislative conflict over executive privilege. That dissenting opinion stated that,
while Nixon was not concerned with the balance between the President’s generalized Diplomatic Negotiations are Privileged
interest in confidentiality and congressional demands for information, "[n]onetheless the
[U.S.] Court laid down principles and procedures that can serve as torch lights to illumine Executive Privilege, an Exception to Congress' Power of Inquiry
us on the scope and use of Presidential communication privilege in the case at bar."83 While
the Court was divided in Neri, this opinion of the Chief Justice was not among the points of
disagreement, and We similarly hold now that the Nixon case is a useful guide in the proper Treaty-making Power
resolution of the present controversy, notwithstanding the difference in context.
Executive Privilege vs. People's Right to Information
Verily, while the Court should guard against the abuse of executive privilege, it should also
give full recognition to the validity of the privilege whenever it is claimed within the proper FACTS:
bounds of executive power, as in this case. Otherwise, the Court would undermine its own
credibility, for it would be perceived as no longer aiming to strike a balance, but seeking This is regarding the JPEPA, the bilateral free trade agreement ratified by the President with
merely to water down executive privilege to the point of irrelevance. Japan, concerning trade in goods, rules of origin, customs procedures, paperless trading,
trade in services, investment, etc.
Conclusion
Prior to President’s signing of JPEPA in Sept. 2006, petitioners – non-government
To recapitulate, petitioners’ demand to be furnished with a copy of the full text of the JPEPA organizations, Congresspersons, citizens and taxpayers – sought via petition for mandamus
has become moot and academic, it having been made accessible to the public since and prohibition to obtain from respondents the full text of the JPEPA, including the Philippine
September 11, 2006. As for their demand for copies of the Philippine and Japanese offers and Japanese offers submitted during the negotiation process and all pertinent attachments
submitted during the JPEPA negotiations, the same must be denied, respondents’ claim of and annexes thereto. Particularly, Congress through the House Committee are calling for an
executive privilege being valid. inquiry into the JPEPA, but at the same time, the Executive is refusing to give them the said
copies until the negotiation is completed.
Diplomatic negotiations have, since the Court promulgated its Resolution in PMPF v.
Manglapus on September 13, 1988, been recognized as privileged in this jurisdiction and the
reasons proffered by petitioners against the application of the ruling therein to the present
335

ISSUES: Applying the principles adopted in PMPF v. Manglapus, it is clear that while the final text of
the JPEPA may not be kept perpetually confidential – since there should be “ample
Whether or not petitioners have legal standing opportunity for discussion before [a treaty] is approved” – the offers exchanged by the
parties during the negotiations continue to be privileged even after the JPEPA is published. It
is reasonable to conclude that the Japanese representatives submitted their offers with the
Whether or not the Philippine and Japanese offers during the negotiation process are
understanding that “historic confidentiality” would govern the same. Disclosing these offers
privileged
could impair the ability of the Philippines to deal not only with Japan but with other foreign
governments in future negotiations.
Whether or not the President can validly exclude Congress, exercising its power of inquiry
and power to concur in treaties, from the negotiation process
A ruling that Philippine offers in treaty negotiations should not be open to public scrutiny
would discourage future Philippine representatives from frankly expressing their views
RULING: during negotiations. While, on first impression, it appears wise to deter Philippine
representatives from entering into compromises, it bears noting that treaty negotiations, or
Standing any negotiation for that matter, normally involve a process of quid pro quo, and oftentimes
negotiators have to be willing to grant concessions in an area of lesser importance in order to
In a petition anchored upon the right of the people to information on matters of public obtain more favorable terms in an area of greater national interest.
concern, which is a public right by its very nature, petitioners need not show that they have
any legal or special interest in the result, it being sufficient to show that they are citizens and, Diplomatic negotiations, therefore, are recognized as privileged in this jurisdiction, the JPEPA
therefore, part of the general public which possesses the right. As the present petition is negotiations constituting no exception. It bears emphasis, however, that such privilege is
anchored on the right to information and petitioners are all suing in their capacity as citizens only presumptive. For as Senate v. Ermita holds, recognizing a type of information as
and groups of citizens including petitioners-members of the House of Representatives who privileged does not mean that it will be considered privileged in all instances. Only after a
additionally are suing in their capacity as such, the standing of petitioners to file the present consideration of the context in which the claim is made may it be determined if there is a
suit is grounded in jurisprudence. public interest that calls for the disclosure of the desired information, strong enough to
overcome its traditionally privileged status.
JPEPA, A Matter of Public Concern
Does the exception apply even though JPEPA is primarily economic and does not involve
To be covered by the right to information, the information sought must meet the threshold national security?
requirement that it be a matter of public concern xxx
While there are certainly privileges grounded on the necessity of safeguarding national
From the nature of the JPEPA as an international trade agreement, it is evident that the security such as those involving military secrets, not all are founded thereon. One example is
Philippine and Japanese offers submitted during the negotiations towards its execution are the “informer’s privilege,” or the privilege of the Government not to disclose the identity of a
matters of public concern. This, respondents do not dispute. They only claim that diplomatic person or persons who furnish information of violations of law to officers charged with the
negotiations are covered by the doctrine of executive privilege, thus constituting an enforcement of that law. The suspect involved need not be so notorious as to be a threat to
exception to the right to information and the policy of full public disclosure. national security for this privilege to apply in any given instance. Otherwise, the privilege
would be inapplicable in all but the most high-profile cases, in which case not only would this
be contrary to long-standing practice. It would also be highly prejudicial to law enforcement
Privileged Character of Diplomatic Negotiations Recognized efforts in general.

The privileged character of diplomatic negotiations has been recognized in this jurisdiction. In Also illustrative is the privileged accorded to presidential communications, which are
discussing valid limitations on the right to information, the Court in Chavez v. PCGG held that presumed privileged without distinguishing between those which involve matters of national
“information on inter-government exchanges prior to the conclusion of treaties and security and those which do not, the rationale for the privilege being that a frank exchange of
executive agreements may be subject to reasonable safeguards for the sake of national exploratory ideas and assessments, free from the glare of publicity and pressure by
interest.” interested parties, is essential to protect the independence of decision-making of those
tasked to exercise Presidential, Legislative and Judicial power.
336

In the same way that the privilege for judicial deliberations does not depend on the nature of In Chavez v. PEA and Chavez v. PCGG, the Court held that with regard to the duty to disclose
the case deliberated upon, so presidential communications are privileged whether they “definite propositions of the government,” such duty does not include recognized exceptions
involve matters of national security. like privileged information, military and diplomatic secrets and similar matters affecting
national security and public order.
It bears emphasis, however, that the privilege accorded to presidential communications is
not absolute, one significant qualification being that “the Executive cannot, any more than Treaty-making power of the President
the other branches of government, invoke a general confidentiality privilege to shield its
officials and employees from investigations by the proper governmental institutions into xxx they (petitioners) argue that the President cannot exclude Congress from the JPEPA
possible criminal wrongdoing.” This qualification applies whether the privilege is being negotiations since whatever power and authority the President has to negotiate
invoked in the context of a judicial trial or a congressional investigation conducted in aid of international trade agreements is derived only by delegation of Congress, pursuant to Article
legislation. VI, Section 28(2) of the Constitution and Sections 401 and 402 of Presidential Decree No.
1464.
Closely related to the “presidential communications” privilege is the deliberative process
privilege recognized in the United States. As discussed by the U.S. Supreme Court in NLRB v. The subject of Article VI Section 28(2) of the Constitution is not the power to negotiate
Sears, Roebuck & Co, deliberative process covers documents reflecting advisory opinions, treaties and international agreements, but the power to fix tariff rates, import and export
recommendations and deliberations comprising part of a process by which governmental quotas, and other taxes xxx.
decisions and policies are formulated. Notably, the privileged status of such documents rests,
not on the need to protect national security but, on the “obvious realization that officials will
As to the power to negotiate treaties, the constitutional basis thereof is Section 21 of Article
not communicate candidly among themselves if each remark is a potential item of discovery
VII – the article on the Executive Department.
and front page news,” the objective of the privilege being to enhance the quality of agency
decisions.
While the power then to fix tariff rates and other taxes clearly belongs to Congress, and is
exercised by the President only be delegation of that body, it has long been recognized that
The diplomatic negotiations privilege bears a close resemblance to the deliberative process
the power to enter into treaties is vested directly and exclusively in the President, subject
and presidential communications privilege. It may be readily perceived that the rationale for
only to the concurrence of at least two-thirds of all the Members of the Senate for the
the confidential character of diplomatic negotiations, deliberative process, and presidential
validity of the treaty. In this light, the authority of the President to enter into trade
communications is similar, if not identical.
agreements with foreign nations provided under P.D. 1464 may be interpreted as an
acknowledgment of a power already inherent in its office. It may not be used as basis to hold
The earlier discussion on PMPF v. Manglapus shows that the privilege for diplomatic the President or its representatives accountable to Congress for the conduct of treaty
negotiations is meant to encourage a frank exchange of exploratory ideas between the negotiations.
negotiating parties by shielding such negotiations from public view. Similar to the privilege
for presidential communications, the diplomatic negotiations privilege seeks, through the
This is not to say, of course, that the President’s power to enter into treaties is unlimited but
same means, to protect the independence in decision-making of the President, particularly in
for the requirement of Senate concurrence, since the President must still enure that all
its capacity as “the sole organ of the nation in its external relations, and its sole
treaties will substantively conform to all the relevant provisions of the Constitution.
representative with foreign nations.” And, as with the deliberative process privilege, the
privilege accorded to diplomatic negotiations arises, not on account of the content of the
information per se, but because the information is part of a process of deliberation which, in It follows from the above discussion that Congress, while possessing vast legislative powers,
pursuit of the public interest, must be presumed confidential. may not interfere in the field of treaty negotiations. While Article VII, Section 21 provides for
Senate concurrence, such pertains only to the validity of the treaty under consideration, not
to the conduct of negotiations attendant to its conclusion. Moreover, it is not even Congress
Clearly, the privilege accorded to diplomatic negotiations follows as a logical consequence
as a while that has been given the authority to concur as a means of checking the treaty-
from the privileged character of the deliberative process.
making power of the President, but only the Senate.

Does diplomatic privilege only apply to certain stages of the negotiation process?
Thus, as in the case of petitioners suing in their capacity as private citizens, petitioners-
members of the House of Representatives fail to present a “sufficient showing of need” that
337

the information sought is critical to the performance of the functions of Congress, functions consideration, but only whether the same is a matter of public concern. When, however, the
that do not include treaty-negotiation. government has claimed executive privilege, and it has established that the information is
indeed covered by the same, then the party demanding it, if it is to overcome the privilege,
Did the respondent’s alleged failure to timely claim executive privilege constitute waiver of must show that that information is vital, not simply for the satisfaction of its curiosity, but for
such privilege? its ability to effectively and reasonably participate in social, political, and economic decision-
making.
That respondent invoked the privilege for the first time only in their Comment to the present
petition does not mean that the claim of privilege should not be credited. Petitioner’s FACTS:
position presupposes that an assertion of the privilege should have been made during the
House Committee investigations, failing which respondents are deemed to have waived it. Petition for mandamus and prohibition was filed by the petitioners, as congresspersons,
citizens and taxpayers, requesting respondents to submit to them the full text of the Japan-
xxx (but) Respondent’s failure to claim the privilege during the House Committee hearings Philippines Economic Partnership Agreement (JPEPA).
may not, however, be construed as a waiver thereof by the Executive branch. xxx what
respondents received from the House Committee and petitioner-Congressman Aguja were Petitioner emphasize that the refusal of the government to disclose the said agreement
mere requests for information. And as priorly stated, the House Committee itself refrained violates there right to information on matters of public concern and of public interest. That
from pursuing its earlier resolution to issue a subpoena duces tecum on account of then the non-disclosure of the same documents undermines their right to effective and
Speaker Jose de Venecia’s alleged request to Committee Chairperson Congressman Teves to reasonable participation in all levels of social, political and economic decision making.
hold the same in abeyance.
Respondent herein invoke executive privilege. They relied on the ground that the matter
The privilege is an exemption to Congress’ power of inquiry. So long as Congress itself finds sought involves a diplomatic negotiation then in progress, thus constituting an exception to
no cause to enforce such power, there is no strict necessity to assert the privilege. In this the right to information and the policy of full disclosure of matters that are of public concern
light, respondent’s failure to invoke the privilege during the House Committee investigations like the JPEPA. That diplomatic negotiation are covered by the doctrine of executive privilege.
did not amount to waiver thereof.
Issue:
“Showing of Need” Test
Whether or not the petition has been entirely rendered moot and academic because of the
In executive privilege controversies, the requirement that parties present a “sufficient subsequent event that occurred?
showing of need” only means, in substance, that they should show a public interest in favor
of disclosure sufficient in degree to overcome the claim of privilege. Verily, the Court in such Whether the information sought by the petitioners are of public concern and are still covered
cases engages in a balancing of interests. Such a balancing of interests is certainly not new in by the doctrine of executive privilege?
constitutional adjudication involving fundamental rights.
Held:
xxx However, when the Executive has – as in this case – invoked the privilege, and it has been
established that the subject information is indeed covered by the privilege being claimed, can
On the first issue, the Supreme Court ruled that t]he principal relief petitioners are praying
a party overcome the same by merely asserting that the information being demanded is a
for is the disclosure of the contents of the JPEPA prior to its finalization between the two
matter of public concern, without any further showing required? Certainly not, for that
States parties,” public disclosure of the text of the JPEPA after its signing by the President,
would render the doctrine of executive privilege of no force and effect whatsoever as a
during the pendency of the present petition, has been largely rendered moot and academic.
limitation on the right to information, because then the sole test in such controversies would
be whether an information is a matter of public concern.
The text of the JPEPA having then been made accessible to the public, the petition has
become moot and academic to the extent that it seeks the disclosure of the “full text”
Right to information vis-a-vis Executive Privilege
thereof.

xxx the Court holds that, in determining whether an information is covered by the right to
information, a specific “showing of need” for such information is not a relevant
338

The petition is not entirely moot, however, because petitioners seek to obtain, not merely Discussion:
the text of the JPEPA, but also the Philippine and Japanese offers in the course of the
negotiations. Standing

Moving on to the second issue, The Supreme Court Ruled that Diplomatic negotiations, In a petition anchored upon the right of the people to information on matters of public
therefore, are recognized as privileged in this jurisdiction, the JPEPA negotiations constituting concern, which is a public right by its very nature, petitioners need not show that they have
no exception. It bears emphasis, however, that such privilege is only presumptive. For as any legal or special interest in the result, it being sufficient to show that they are citizens and,
Senate v. Ermita holds, recognizing a type of information as privileged does not mean that it therefore, part of the general public which possesses the right. As the present petition is
will be considered privileged in all instances. Only after a consideration of the context in anchored on the right to information and petitioners are all suing in their capacity as citizens
which the claim is made may it be determined if there is a public interest that calls for the and groups of citizens including petitioners-members of the House of Representatives who
disclosure of the desired information, strong enough to overcome its traditionally privileged additionally are suing in their capacity as such, the standing of petitioners to file the present
status. suit is grounded in jurisprudence.

The court adopted also the doctrine in PMPF v. Manglapus, Wherein petitioners were From the nature of the JPEPA as an international trade agreement, it is evident that the
seeking information from the President’s representatives on the state of the then on-going Philippine and Japanese offers submitted during the negotiations towards its execution are
negotiations of the RP-US Military Bases Agreement. The Court denied the petition, stressing matters of public concern. This, respondents do not dispute. They only claim that diplomatic
that “secrecy of negotiations with foreign countries is not violative of the constitutional negotiations are covered by the doctrine of executive privilege, thus constituting an
provisions of freedom of speech or of the press nor of the freedom of access to information. exception to the right to information and the policy of full public disclosure.

Facts: “Showing of Need” Test

Petitioners, non-government organizations, Congresspersons, citizens and taxpayers In executive privilege controversies, the requirement that parties present a “sufficient
requested, via the petition for mandamus and prohibition, to obtain from respondents the showing of need” only means, in substance, that they should show a public interest in favor
full text of the Japan-Philippines Economic Partnership Agreement (JPEPA) including the of disclosure sufficient in degree to overcome the claim of privilege. Verily, the Court in such
Philippine and Japanese offers submitted during the negotiation process and all pertinent cases engages in a balancing of interests. Such a balancing of interests is certainly not new in
attachments and annexes thereto. The Congress, through the House Committee called for an constitutional adjudication involving fundamental rights.
inquiry into the JPEPA, but at the same time, the Executive refused to give them the said
copies until the negotiation was completed.
However, when the Executive has – as in this case – invoked the privilege, and it has been
established that the subject information is indeed covered by the privilege being claimed, can
JPEPA was the bilateral free trade agreement entered between the Philippine government a party overcome the same by merely asserting that the information being demanded is a
with Japan, concerned with trade in goods, rules of origin, customs procedures, paperless matter of public concern, without any further showing required? Certainly not, for that
trading, trade in services, investment, intellectual property rights, government procurement, would render the doctrine of executive privilege of no force and effect whatsoever as a
movement of natural persons, cooperation, competition policy, mutual recognition, dispute limitation on the right to information, because then the sole test in such controversies would
avoidance and settlement, improvement of the business environment, and general and final be whether an information is a matter of public concern.
provisions.
Treaty-making power of the President
Issue:
Petitioners argue that the President cannot exclude Congress from the JPEPA negotiations
Whether or not petitioners have legal standing to request for the full text of JPEPA. since whatever power and authority the President has to negotiate international trade
agreements is derived only by delegation of Congress, pursuant to Article VI, Section 28(2) of
Whether or not the President can validly exclude Congress, exercising its power of inquiry the Constitution and Sections 401 and 402 of Presidential Decree No. 1464.
and power to concur in treaties, from the negotiation process.
339

The subject of Article VI Section 28(2) of the Constitution is not the power to negotiate Diplomatic negotiations have, since the Court promulgated its Resolution in PMPF v.
treaties and international agreements, but the power to fix tariff rates, import and export Manglapus on September 13, 1988, been recognized as privileged in this jurisdiction and the
quotas, and other taxes. reasons proffered by petitioners against the application of the ruling therein to the present
case have not persuaded the Court. Moreover, petitioners both private citizens and members
As to the power to negotiate treaties, the constitutional basis thereof is Section 21 of Article of the House of Representatives have failed to present a sufficient showing of need to
VII – the article on the Executive Department. overcome the claim of privilege in this case.

While the power then to fix tariff rates and other taxes clearly belongs to Congress, and is That the privilege was asserted for the first time in respondents Comment to the present
exercised by the President only be delegation of that body, it has long been recognized that petition, and not during the hearings of the House Special Committee on Globalization, is of
the power to enter into treaties is vested directly and exclusively in the President, subject no moment, since it cannot be interpreted as a waiver of the privilege on the part of the
only to the concurrence of at least two-thirds of all the Members of the Senate for the Executive branch.
validity of the treaty. In this light, the authority of the President to enter into trade
agreements with foreign nations provided under P.D. 1464 may be interpreted as an For reasons already explained, this Decision shall not be interpreted as departing from the
acknowledgment of a power already inherent in its office. It may not be used as basis to hold ruling in Senate v. Ermita that executive privilege should be invoked by the President or
the President or its representatives accountable to Congress for the conduct of treaty through the Executive Secretary by order of the President.
negotiations.

This is not to say, of course, that the President’s power to enter into treaties is unlimited but
for the requirement of Senate concurrence, since the President must still enure that all
treaties will substantively conform to all the relevant provisions of the Constitution.

It follows from the above discussion that Congress, while possessing vast legislative powers,
may not interfere in the field of treaty negotiations. While Article VII, Section 21 provides for
Senate concurrence, such pertains only to the validity of the treaty under consideration, not
to the conduct of negotiations attendant to its conclusion. Moreover, it is not even Congress
as a while that has been given the authority to concur as a means of checking the treaty-
making power of the President, but only the Senate.

Thus, as in the case of petitioners suing in their capacity as private citizens, petitioners-
members of the House of Representatives fail to present a “sufficient showing of need” that
the information sought is critical to the performance of the functions of Congress, functions
that do not include treaty-negotiation.

Held:

The petition was dismissed.

Petitioner’s demand to be furnished with a copy of the full text of the JPEPA has become
moot and academic, it having been made accessible to the public since September 11, 2006.
As for their demand for copies of the Philippine and Japanese offers submitted during the
JPEPA negotiations, the same must be denied, respondents claim of executive privilege being
valid.
340

EN BANC part of Philippine law only insofar as they are expressly adopted; that in its rulings in The
Holy See, v. Rosario, Jr.5 and U.S. v. Guinto6 the Court has said that international law is
G.R. No. 162230 August 13, 2014 deemed part of the Philippine law as a consequence of Statehood; that in Agustin v. Edu,7 the
Court has declared that a treaty, though not yet ratified by the Philippines, was part of the
law of the land through the Incorporation Clause; that by virtue of the Incorporation Clause,
ISABELITA C. VINUY A, Petitioners,
the Philippines is bound to abide by the erga omnesobligations arising from the jus
vs.
cogensnorms embodied in the laws of war and humanity that include the principle of the
THE HONORABLE EXECUTIVE SECRETARY ALBERTO G. ROMULO, THE HONORABLE
imprescriptibility of war crimes; that the crimes committed against petitioners are proscribed
SECRETARY OF FOREIGN AFFAIRS DELIA DOMINGOALBERT, THE HONORABLE SECRETARY
under international human rights law as there were undeniable violations of jus
OF JUSTICE MERCEDITAS N. GUTIERREZ, and THE HONORABLE SOLICITOR GENERAL
cogensnorms; that the need to punish crimes against the laws of humanity has long become
ALFREDO L. BENIPAYO, Respondents.
jus cogensnorms, and that international legal obligations prevail over national legal norms;
that the Court’s invocation of the political doctrine in the instant case is misplaced; and that
RESOLUTION the Chief Executive has the constitutional duty to afford redress and to give justice to the
victims ofthe comfort women system in the Philippines.8
BERSAMIN, J.:
Petitioners further argue that the Court has confused diplomatic protection with the broader
Petitioners filed a Motion for Reconsideration1 and a Supplemental Motion for responsibility of states to protect the human rights of their citizens, especially where the
Reconsideration,2 praying that the Court reverse its decision of April 28, 2010, and grant their rights asserted are subject of erga omnesobligations and pertain to jus cogensnorms; that
petition for certiorari. the claims raised by petitioners are not simple private claims that are the usual subject of
diplomatic protection; that the crimes committed against petitioners are shocking to the
In their Motion for Reconsideration, petitioners argue that our constitutional and conscience of humanity; and that the atrocities committed by the Japanese soldiers against
jurisprudential histories have rejected the Court’s ruling that the foreign policy prerogatives petitionersare not subject to the statute of limitations under international law.9
ofthe Executive Branch are unlimited; that under the relevant jurisprudence and
constitutional provisions, such prerogatives are proscribed by international human rights and Petitioners pray that the Court reconsider its April 28, 2010 decision, and declare: (1) that the
international conventions of which the Philippines is a party; that the Court, in holding that rapes, sexual slavery, torture and other forms of sexual violence committed against the
the Chief Executive has the prerogative whether to bring petitioners’ claims against Japan, Filipina comfort women are crimes against humanity and war crimes under customary
has read the foreign policy powers of the Office of the President in isolation from the rest of international law; (2) that the Philippines is not bound by the Treaty of Peace with Japan,
the constitutional protections that expressly textualize international human rights; that the insofar as the waiver of the claims of the Filipina comfort women against Japan is concerned;
foreign policy prerogatives are subject to obligations to promote international humanitarian (3) that the Secretary of Foreign Affairs and the Executive Secretary committed grave abuse
law as incorporated intothe laws of the land through the Incorporation Clause; that the Court of discretion in refusing to espouse the claims of Filipina comfort women; and (4) that
must re-visit its decisions in Yamashita v. Styer3 and Kuroda v. Jalandoni4 which have been petitioners are entitled to the issuance of a writ of preliminary injunction against the
noted for their prescient articulation of the import of laws of humanity; that in said decision, respondents.
the Court ruled that the State was bound to observe the laws of war and humanity; that in
Yamashita, the Court expressly recognized rape as an international crime under international Petitioners also pray that the Court order the Secretary of Foreign Affairs and the Executive
humanitarian law, and in Jalandoni, the Court declared that even if the Philippines had not Secretary to espouse the claims of Filipina comfort women for an official apology,legal
acceded or signed the Hague Convention on Rules and Regulations covering Land Warfare, compensation and other forms of reparation from Japan.10
the Rules and Regulations formed part of the law of the nation by virtue of the Incorporation
Clause; that such commitment to the laws ofwar and humanity has been enshrined in Section
In their Supplemental Motion for Reconsideration, petitioners stress that it was highly
2, Article II of the 1987 Constitution, which provides "that the Philippines…adopts the
improper for the April 28, 2010 decision to lift commentaries from at least three sources
generally accepted principles of international law as part of the law of the land and adheres
without proper attribution – an article published in 2009 in the Yale Law Journal of
to the policy of peace, equality, justice, freedom, cooperation, and amity with all nations."
International Law; a book published by the Cambridge University Press in 2005; and an article
published in 2006 in the Western ReserveJournal of International Law – and make it appear
The petitioners added that the statusand applicability of the generally accepted principles of that such commentaries supported its arguments for dismissing the petition, when in truth
international law within the Philippine jurisdiction would be uncertain without the the plagiarized sources even made a strong case in favour of petitioners’ claims.11
Incorporation Clause, and that the clause implied that the general international law forms
341

In their Comment,12 respondents disagree withpetitioners, maintaining that aside from the The Court DENIESthe Motion for Reconsiderationand Supplemental Motion for
statements on plagiarism, the arguments raised by petitioners merely rehashed those made Reconsideration for being devoid of merit.
in their June 7, 2005 Memorandum; that they already refuted such arguments in their
Memorandumof June 6, 2005 that the Court resolved through itsApril 28, 2010 decision, 1. Petitioners did not show that their resort was timely under the Rules of Court.
specifically as follows:
Petitioners did not show that their bringing ofthe special civil action for certiorariwas timely,
1. The contentions pertaining tothe alleged plagiarism were then already lodged i.e., within the 60-day period provided in Section 4, Rule 65 of the Rules of Court, to wit:
withthe Committee on Ethics and Ethical Standards of the Court; hence, the matter
of alleged plagiarism should not be discussed or resolved herein.13
Section 4. When and where position filed. – The petition shall be filed not later than sixty (60)
daysfrom notice of judgment, order or resolution. In case a motion for reconsideration or
2. A writ of certioraridid not lie in the absence of grave abuse of discretion new trial is timely filed, whether such motion is required or not, the sixty (60) day period
amounting to lack or excess of jurisdiction. Hence, in view of the failureof shall be counted from notice of the denial of said motion.
petitioners to show any arbitrary or despotic act on the part of respondents,the
relief of the writ of certiorariwas not warranted.14
As the rule indicates, the 60-day period starts to run from the date petitioner receives the
assailed judgment, final order or resolution, or the denial of the motion for reconsideration
3. Respondents hold that the Waiver Clause in the Treaty of Peace with Japan, or new trial timely filed, whether such motion is required or not. To establish the timeliness
being valid, bound the Republic of the Philippines pursuant to the international law of the petition for certiorari, the date of receipt of the assailed judgment, final order or
principle of pacta sunt servanda.The validity of the Treaty of Peace was the result resolution or the denial of the motion for reconsideration or new trial must be stated in the
of the ratification by two mutually consenting parties. Consequently, the petition;otherwise, the petition for certiorarimust be dismissed. The importance of the dates
obligations embodied in the Treaty of Peace must be carried out in accordance cannot be understated, for such dates determine the timeliness of the filing of the petition
with the common and real intention of the parties at the time the treaty was for certiorari. As the Court has emphasized in Tambong v. R. Jorge Development
concluded.15 Corporation:17

4. Respondents assert that individuals did not have direct international remedies There are three essential dates that must be stated in a petition for certiorari brought under
against any State that violated their human rights except where such remedies are Rule 65. First, the date when notice of the judgment or final order or resolution was
provided by an international agreement. Herein, neither of the Treaty of Peace and received; second, when a motion for new trial or reconsideration was filed; and third, when
the Reparations Agreement,the relevant agreements affecting herein petitioners, notice of the denial thereof was received. Failure of petitioner to comply with this
provided for the reparation of petitioners’ claims. Respondents aver that the requirement shall be sufficient ground for the dismissal of the petition. Substantial
formal apology by the Government of Japan and the reparation the Government of compliance will not suffice in a matter involving strict observance with the Rules. (Emphasis
Japan has provided through the Asian Women’s Fund (AWF) are sufficient to supplied)
recompense petitioners on their claims, specifically:
The Court has further said in Santos v. Court of Appeals:18
a. About 700 million yen would be paid from the national treasury over the next 10
years as welfare and medical services;
The requirement of setting forth the three (3) dates in a petition for certiorari under Rule 65
is for the purpose of determining its timeliness. Such a petition is required to be filed not
b. Instead of paying the money directly to the former comfort women, the services later than sixty (60) days from notice of the judgment, order or Resolution sought to be
would be provided through organizations delegated by governmental bodies in the assailed. Therefore, that the petition for certiorariwas filed forty-one (41) days from receipt
recipient countries (i.e., the Philippines, the Republic of Korea,and Taiwan); and of the denial of the motion for reconsideration is hardly relevant. The Court of Appeals was
notin any position to determine when this period commenced to run and whether the
c. Compensation would consist of assistance for nursing services (like home motion for reconsideration itself was filed on time since the material dates were not stated.
helpers), housing, environmental development, medical expenses, and medical It should not be assumed that in no event would the motion be filed later than fifteen (15)
goods.16 days. Technical rules of procedure are not designed to frustrate the ends of justice. These are
provided to effect the proper and orderly disposition of cases and thus effectively prevent
Ruling
342

the clogging of court dockets. Utter disregard of the Rules cannot justly be rationalized by Herein petitioners have not shown any compelling reason for us to relax the rule and the
harking on the policy ofliberal construction.19 requirements under current jurisprudence. x x x. (Emphasis supplied)

The petition for certioraricontains the following averments, viz: 2. Petitioners did not show that the assailed act was either judicial or quasi-judicial on the
part of respondents.
82. Since 1998, petitioners and other victims of the "comfort women system,"
approached the Executive Department through the Department of Justice in order Petitioners were required to show in their petition for certiorarithat the assailed act was
to request for assistance to file a claim against the Japanese officials and military either judicial or quasi-judicial in character. Section 1, Rule 65 of the Rules of Courtrequires
officers who ordered the establishment of the "comfort women" stations in the such showing, to wit:
Philippines;
Section 1. Petition for certiorari.—When any tribunal, board or officer exercising judicial or
83. Officials of the Executive Department ignored their request and refused to file a quasi-judicial functions has acted without or in excess of its or his jurisdiction, or with grave
claim against the said Japanese officials and military officers; abuse of discretion amounting to lack or excess of jurisdiction, and there is no appeal, nor
any plain, speedy, and adequate remedy in the ordinary course of law, a person aggrieved
84. Undaunted, the Petitioners in turnapproached the Department of Foreign thereby may file a verified petition in the proper court, alleging the facts with certainty and
Affairs, Department of Justice and Office of the of the Solicitor General to file their praying that judgment be rendered annulling or modifying the proceedings of such tribunal,
claim against the responsible Japanese officials and military officers, but their board or officer, and granting such incidental reliefs as law and justice may require.
efforts were similarly and carelessly disregarded;20
The petition shall be accompanied by a certified true copy of the judgment, order, or
The petition thus mentions the year 1998 only as the time when petitioners approached the resolution subject thereof, copies of all pleadings and documents relevant and pertinent
Department ofJustice for assistance, but does not specifically state when they received the thereto, and a sworn certification of nonforum shopping as provided in the third paragraph
denial of their request for assistance by the Executive Department of the Government. This of Section 3, Rule 46. However, petitioners did notmake such a showing.
alone warranted the outright dismissal of the petition.
3. Petitioners were not entitled to the injunction.
Even assuming that petitioners received the notice of the denial of their request for
assistance in 1998, their filing of the petition only on March 8, 2004 was still way beyond the The Court cannot grant petitioners’ prayer for the writ of preliminary mandatory injunction.
60-day period. Only the most compelling reasons could justify the Court’s acts of disregarding Preliminary injunction is merely a provisional remedy that is adjunct to the main case, and is
and lifting the strictures of the rule on the period. As we pointed out inMTM Garment Mfg. subject to the latter’s outcome. It is not a cause of action itself.22 It is provisional because it
Inc. v. Court of Appeals:21 constitutes a temporary measure availed of during the pendency of the action; and it is
ancillary because it is a mere incident in and is dependent upon the result of the main
All these do not mean, however, that procedural rules are to be ignored or disdained at will action.23Following the dismissal of the petition for certiorari, there is no more legal basis to
to suit the convenience of a party. Procedural law has its own rationale in the orderly issue the writ of injunction sought. As an auxiliary remedy, the writ of preliminary mandatory
administration of justice, namely: to ensure the effective enforcement of substantive rights injunction cannot be issued independently of the principal action. 24
by providing for a system that obviates arbitrariness, caprice, despotism, or whimsicality in
the settlement of disputes. Hence, it is a mistake to suppose that substantive law and In any event, a mandatory injunction requires the performance of a particular act. Hence, it is
procedural law are contradictory to each other, or as often suggested, that enforcement of an extreme remedy,25to be granted only if the following requisites are attendant, namely:
procedural rules should never be permitted if it would result in prejudice to the substantive
rights of the litigants. (a) The applicant has a clear and unmistakable right, that is, a right in esse;

As we have repeatedly stressed, the right to file a special civil action of certiorariis neither a (b) There is a material and substantial invasion of such right; and
natural right noran essential element of due process; a writ of certiorariis a prerogative writ,
never demandable as a matter of right, and never issued except in the exercise of judicial
discretion. Hence, he who seeks a writ of certiorarimust apply for it only in the manner and
strictly in accordance with the provisions of the law and the Rules.
343

(c) There is an urgent need for the writ to prevent irreparable injury to the
applicant; and no other ordinary, speedy, and adequate remedy exists to prevent DECISION
the infliction of irreparable injury.26
DEL CASTILLO, J.:
In Marquez v. The Presiding Judge (Hon. Ismael B. Sanchez), RTC Br. 58, Lucena City,27 we
expounded as follows: The Treaty of Peace with Japan, insofar as it barred future claims such as those
asserted by plaintiffs in these actions, exchanged full compensation of plaintiffs for
a future peace. History has vindicated the wisdom of that bargain. And while full
It is basic that the issuance of a writ of preliminary injunction is addressed to the sound
compensation for plaintiffs' hardships, in the purely economic sense, has been
discretion of the trial court, conditioned on the existence of a clear and positive right of the
denied these former prisoners and countless other survivors of the war, the
applicant which should be protected. It is an extraordinary, peremptory remedy available
immeasurable bounty of life for themselves and their posterity in a free society
only on the grounds expressly provided by law, specifically Section 3, Rule 58 of the Rules of
and in a more peaceful world services the debt.[1]
Court. Moreover, extreme caution must be observed in the exercise of such discretion. It
should be granted only when the court is fully satisfied that the law permits it and the
emergency demands it. The very foundation of the jurisdiction to issue a writ of injunction
rests in the existence of a cause of action and in the probability of irreparable injury, There is a broad range of vitally important areas that must be regularly decided by the Executive
inadequacy of pecuniary compensation, and the prevention of multiplicity of suits. Where Department without either challenge or interference by the Judiciary. One such area involves the
facts are not shown to bring the case within these conditions, the relief of injunction should
be refused.28 delicate arena of foreign relations. It would be strange indeed if the courts and the executive spoke with
different voices in the realm of foreign policy. Precisely because of the nature of the questions
Here, the Constitution has entrusted to the Executive Department the conduct of foreign
presented, and the lapse of more than 60 years since the conduct complained of, we make no attempt
relations for the Philippines. Whether or not to espouse petitioners' claim against the
Government of Japan is left to the exclusive determination and judgment of the Executive to lay down general guidelines covering other situations not involved here, and confine the opinion only
Department. The Court cannot interfere with or question the wisdom of the conduct of to the very questions necessary to reach a decision on this matter.
foreign relations by the Executive Department. Accordingly, we cannot direct the Executive
Department, either by writ of certiorari or injunction, to conduct our foreign relations with
Japan in a certain manner. Factual Antecedents

WHEREFORE, the Court DENIES the Motion for Reconsideration and Supplemental Motion for
Reconsideration for their lack of merit. This is an original Petition for Certiorari under Rule 65 of the Rules of Court with an application for the
issuance of a writ of preliminary mandatory injunction against the Office of the Executive Secretary, the
SO ORDERED.
Secretary of the Department of Foreign Affairs (DFA), the Secretary of the Department of Justice (DOJ),

EN BANC and the Office of the Solicitor General (OSG).

ISABELITA C. VINUYA, G.R. No. 162230


Petitioners, Petitioners are all members of the MALAYA LOLAS, a non-stock, non-profit organization
registered with the Securities and Exchange Commission, established for the purpose of
- versus -
providing aid to the victims of rape by Japanese military forces in the Philippines during the
THE HONORABLE EXECUTIVE Second World War.
SECRETARY ALBERTO G.
ROMULO, Respondents.
Petitioners narrate that during the Second World War, the Japanese army attacked villages and
systematically raped the women as part of the destruction of the village. Their communities were
344

bombed, houses were looted and burned, and civilians were publicly tortured, mutilated, and Respondents Arguments
slaughtered. Japanese soldiers forcibly seized the women and held them in houses or cells, where they
were repeatedly raped, beaten, and abused by Japanese soldiers. As a result of the actions of their Respondents maintain that all claims of the Philippines and its nationals relative to the war were dealt
Japanese tormentors, the petitioners have spent their lives in misery, having endured physical injuries, with in the San Francisco Peace Treaty of 1951 and the bilateral Reparations Agreement of 1956.[6]
pain and disability, and mental and emotional suffering.[2] Article 14 of the Treaty of Peace[7] provides:

Article 14. Claims and Property


Petitioners claim that since 1998, they have approached the Executive Department through the DOJ,
DFA, and OSG, requesting assistance in filing a claim against the Japanese officials and military officers a) It is recognized that Japan should pay reparations to the Allied Powers for the
who ordered the establishment of the comfort women stations in the Philippines. However, officials of damage and suffering caused by it during the war. Nevertheless it
is also recognized that the resources of Japan are not presently
the Executive Department declined to assist the petitioners, and took the position that the individual sufficient, if it is to maintain a viable economy, to make complete
claims of the comfort women for compensation had already been fully satisfied by Japans compliance reparation for all such damage and suffering and at the present
time meet its other obligations.
with the Peace Treaty between the Philippines and Japan.
b) Except as otherwise provided in the present Treaty, the Allied
Powers waive all reparations claims of the Allied Powers, other
Issues claims of the Allied Powers and their nationals arising out of any
actions taken by Japan and its nationals in the course of the
prosecution of the war, and claims of the Allied Powers for direct
Hence, this petition where petitioners pray for this court to (a) declare that respondents committed
military costs of occupation.
grave abuse of discretion amounting to lack or excess of discretion in refusing to espouse their claims for
the crimes against humanity and war crimes committed against them; and (b) compel the respondents
In addition, respondents argue that the apologies made by Japan[8] have been satisfactory, and
to espouse their claims for official apology and other forms of reparations against Japan before the
that Japan had addressed the individual claims of the women through the atonement money paid by
International Court of Justice (ICJ) and other international tribunals.
the Asian Womens Fund.

Petitioners arguments
Historical Background

Petitioners argue that the general waiver of claims made by the Philippine government in the Treaty of
The comfort women system was the tragic legacy of the Rape of Nanking. In December 1937, Japanese
Peace with Japan is void. They claim that the comfort women system established by Japan, and the
military forces captured the city of Nanking in China and began a barbaric campaign of terror known as
brutal rape and enslavement of petitioners constituted a crime against humanity,[3] sexual slavery,[4] and
the Rape of Nanking, which included the rapes and murders of an estimated 20,000 to 80,000 Chinese
torture.[5] They allege that the prohibition against these international crimes is jus cogens norms from
women, including young girls, pregnant mothers, and elderly women.[9]
which no derogation is possible; as such, in waiving the claims of Filipina comfort women and failing to
espouse their complaints against Japan, the Philippine government is in breach of its legal obligation not
In reaction to international outcry over the incident, the Japanese government sought ways to end
to afford impunity for crimes against humanity. Finally, petitioners assert that the Philippine
international condemnation[10] by establishing the comfort women system. Under this system, the
governments acceptance of the apologies made by Japan as well as funds from the Asian Womens Fund
military could simultaneously appease soldiers' sexual appetites and contain soldiers' activities within a
(AWF) were contrary to international law.
345

regulated environment.[11] Comfort stations would also prevent the spread of venereal disease among Undoubtedly frustrated by the failure of litigation before Japanese courts, victims of the comfort women
soldiers and discourage soldiers from raping inhabitants of occupied territories.[12] system brought their claims before the United States (US). On September 18, 2000, 15 comfort women
filed a class action lawsuit in the US District Court for the District of Columbia[23] "seeking money
Daily life as a comfort woman was unmitigated misery.[13] The military forced victims into barracks-style damages for [allegedly] having been subjected to sexual slavery and torture before and during World
stations divided into tiny cubicles where they were forced to live, sleep, and have sex with as many 30 War II," in violation of "both positive and customary international law." The case was filed pursuant to
soldiers per day.[14] The 30 minutes allotted for sexual relations with each soldier were 30-minute the Alien Tort Claims Act (ATCA),[24] which allowed the plaintiffs to sue the Japanese government in
increments of unimaginable horror for the women.[15] Disease was rampant.[16] Military doctors regularly a US federal district court.[25] On October 4, 2001, the district court dismissed the lawsuit due to lack of
examined the women, but these checks were carried out to prevent the spread of venereal diseases; jurisdiction over Japan, stating that [t]here is no question that this court is not the appropriate forum in
little notice was taken of the frequent cigarette burns, bruises, bayonet stabs and even broken bones which plaintiffs may seek to reopen x x x discussions nearly half a century later x x x [E]ven if Japan did
inflicted on the women by soldiers. not enjoy sovereign immunity, plaintiffs' claims are non-justiciable and must be dismissed.

Fewer than 30% of the women survived the war.[17] Their agony continued in having to suffer with the The District of Columbia Court of Appeals affirmed the lower court's dismissal of the case.[26] On appeal,
residual physical, psychological, and emotional scars from their former lives. Some returned home and the US Supreme Court granted the womens petition for writ of certiorari, vacated the judgment of the
were ostracized by their families. Some committed suicide. Others, out of shame, never returned District of Columbia Court of Appeals, and remanded the case.[27] On remand, the Court of Appeals
home.[18] affirmed its prior decision, noting that much as we may feel for the plight of the appellants, the courts of
the US simply are not authorized to hear their case.[28] The women again brought their case to the US
Efforts to Secure Reparation Supreme Court which denied their petition for writ of certiorari on February 21, 2006.

The most prominent attempts to compel the Japanese government to accept legal responsibility and Efforts at the United Nations
pay compensatory damages for the comfort women system were through a series of lawsuits,
discussion at the United Nations (UN), resolutions by various nations, and the Womens International In 1992, the Korean Council for the Women Drafted for Military Sexual Slavery by Japan (KCWS),
Criminal Tribunal. The Japanese government, in turn, responded through a series of public apologies and submitted a petition to the UN Human Rights Commission (UNHRC), asking for assistance in
the creation of the AWF.[19] investigating crimes committed by Japan against Korean women and seeking reparations for former
comfort women.[29] The UNHRC placed the issue on its agenda and appointed Radhika Coomaraswamy
Lawsuits as the issue's special investigator. In 1996, Coomaraswamy issued a Report reaffirming Japan's
responsibility in forcing Korean women to act as sex slaves for the imperial army, and made the
In December 1991, Kim Hak-Sun and two other survivors filed the first lawsuit in Japan by former following recommendations:
comfort women against the Japanese government. The Tokyo District Court however dismissed their
A. At the national level
case.[20] Other suits followed,[21] but the Japanese government has, thus far, successfully caused the
137. The Government of Japan should:
dismissal of every case.[22]
(a) Acknowledge that the system of comfort stations set up by the Japanese
Imperial Army during the Second World War was a violation of its
346

obligations under international law and accept legal responsibility for through peace treaties and reparations agreements following the war remains
that violation; equally unpersuasive. This is due, in large part, to the failure until very recently of
the Japanese Government to admit the extent of the Japanese militarys direct
(b) Pay compensation to individual victims of Japanese military sexual slavery involvement in the establishment and maintenance of these rape centres. The
according to principles outlined by the Special Rapporteur of the Sub- Japanese Governments silence on this point during the period in which peace and
Commission on Prevention of Discrimination and Protection of reparations agreements between Japan and other Asian Governments were being
Minorities on the right to restitution, compensation and rehabilitation negotiated following the end of the war must, as a matter of law and justice,
for victims of grave violations of human rights and fundamental preclude Japan from relying today on these peace treaties to extinguish liability in
freedoms. A special administrative tribunal for this purpose should be these cases.
set up with a limited time-frame since many of the victims are of a very
advanced age; 69. The failure to settle these claims more than half a century after the cessation
of hostilities is a testament to the degree to which the lives of women continue to
(c) Make a full disclosure of documents and materials in its possession with regard be undervalued. Sadly, this failure to address crimes of a sexual nature committed
to comfort stations and other related activities of the Japanese Imperial on a massive scale during the Second World War has added to the level of
Army during the Second World War; impunity with which similar crimes are committed today. The Government of
Japan has taken some steps to apologize and atone for the rape and enslavement
(d) Make a public apology in writing to individual women who have come forward of over 200,000 women and girls who were brutalized in comfort stations during
and can be substantiated as women victims of Japanese military sexual the Second World War. However, anything less than full and unqualified
slavery; acceptance by the Government of Japan of legal liability and the consequences
that flow from such liability is wholly inadequate. It must now fall to the
(e) Raise awareness of these issues by amending educational curricula to reflect Government of Japan to take the necessary final steps to provide adequate
historical realities; redress.

(f) Identify and punish, as far as possible, perpetrators involved in the recruitment
and institutionalization of comfort stations during the Second World
The UN, since then, has not taken any official action directing Japan to provide the reparations sought.
War.
Women's International War Crimes

Gay J. McDougal, the Special Rapporteur for the UN Sub-Commission on Prevention of Discrimination
Tribunal
and Protection of Minorities, also presented a report to the Sub-Committee on June 22,
1998 entitled Contemporary Forms of Slavery: Systematic Rape, Sexual Slavery and Slavery-like Practices
The Women's International War Crimes Tribunal (WIWCT) was a people's tribunal established by a
During Armed Conflict. The report included an appendix entitled An Analysis of the Legal Liability of the
number of Asian women and human rights organizations, supported by an international coalition of
Government of Japan for 'Comfort Women Stations' established during the Second World War,[30] which
non-governmental organizations.[31] First proposed in 1998, the WIWCT convened in Tokyo in 2000 in
contained the following findings:
order to adjudicate Japan's military sexual violence, in particular the enslavement of comfort women, to
bring those responsible for it to justice, and to end the ongoing cycle of impunity for wartime sexual
68. The present report concludes that the Japanese Government remains liable
for grave violations of human rights and humanitarian law, violations that amount violence against women.
in their totality to crimes against humanity. The Japanese Governments
arguments to the contrary, including arguments that seek to attack the underlying
humanitarian law prohibition of enslavement and rape, remain as unpersuasive After examining the evidence for more than a year, the tribunal issued its verdict on December 4, 2001,
today as they were when they were first raised before the Nuremberg war crimes finding the former Emperor Hirohito and the State of Japan guilty of crimes against humanity for the
tribunal more than 50 years ago. In addition, the Japanese Governments
argument that Japan has already settled all claims from the Second World War rape and sexual slavery of women.[32] It bears stressing, however, that although the tribunal included
347

prosecutors, witnesses, and judges, its judgment was not legally binding since the tribunal itself was and to restore references in Japanese textbooks to its war crimes.[36] The Dutch parliament's resolution
organized by private citizens. calls for the Japanese government to uphold the 1993 declaration of remorse made by Chief Cabinet
Secretary Yohei Kono.
Action by Individual Governments
The Foreign Affairs Committee of the United Kingdoms Parliament also produced a report in November,
On January 31, 2007, US Representative Michael Honda of California, along with six co-sponsor 2008 entitled, "Global Security: Japan and Korea" which concluded that Japan should acknowledge the
representatives, introduced House Resolution 121 which called for Japanese action in light of the pain caused by the issue of comfort women in order to ensure cooperation between Japan and Korea.
ongoing struggle for closure by former comfort women. The Resolution was formally passed on July 30,
2007,[33] and made four distinct demands: Statements of Remorse made by
representatives of the Japanese
government
[I]t is the sense of the House of Representatives that the Government of Japan (1)
should formally acknowledge, apologize, and accept historical responsibility in a
clear and unequivocal manner for its Imperial Armed Forces' coercion of young
women into sexual slavery, known to the world as comfort women, during its Various officials of the Government of Japan have issued the following public statements concerning the
colonial and wartime occupation of Asia and the Pacific Islands from the 1930s
comfort system:
through the duration of World War II; (2) would help to resolve recurring
questions about the sincerity and status of prior statements if the Prime Minister
of Japan were to make such an apology as a public statement in his official a) Statement by the Chief Cabinet Secretary Yohei Kono in 1993:
capacity; (3) should clearly and publicly refute any claims that the sexual
enslavement and trafficking of the comfort women for the Japanese Imperial
Army never occurred; and (4) should educate current and future generations The Government of Japan has been conducting a study on the issue of wartime
about this horrible crime while following the recommendations of the "comfort women" since December 1991. I wish to announce the findings as a
international community with respect to the comfort women.[34] result of that study.

As a result of the study which indicates that comfort stations were operated in
In December 2007, the European Parliament, the governing body of the European Union, drafted a extensive areas for long periods, it is apparent that there existed a great number
resolution similar to House Resolution 121.[35] Entitled, Justice for Comfort Women, the resolution of comfort women. Comfort stations were operated in response to the request of
the military authorities of the day. The then Japanese military was, directly or
demanded: (1) a formal acknowledgment of responsibility by the Japanese government; (2) a removal of
indirectly, involved in the establishment and management of the comfort stations
the legal obstacles preventing compensation; and (3) unabridged education of the past. The resolution and the transfer of comfort women. The recruitment of the comfort women was
conducted mainly by private recruiters who acted in response to the request of
also stressed the urgency with which Japan should act on these issues, stating: the right of individuals to
the military. The Government study has revealed that in many cases they were
claim reparations against the government should be expressly recognized in national law, and cases for recruited against their own will, through coaxing coercion, etc., and that, at times,
reparations for the survivors of sexual slavery, as a crime under international law, should be prioritized, administrative/military personnel directly took part in the recruitments. They lived
in misery at comfort stations under a coercive atmosphere.
taking into account the age of the survivors.
As to the origin of those comfort women who were transferred to the war areas,
excluding those from Japan, those from the Korean Peninsula accounted for a
The Canadian and Dutch parliaments have each followed suit in drafting resolutions large part. The Korean Peninsula was under Japanese rule in those days, and their
against Japan. Canada's resolution demands the Japanese government to issue a formal apology, to recruitment, transfer, control, etc., were conducted generally against their will,
through coaxing, coercion, etc.
admit that its Imperial Military coerced or forced hundreds of thousands of women into sexual slavery,
348

Undeniably, this was an act, with the involvement of the military authorities of the Solemnly reflecting upon the many instances of colonial rule and acts of
day, that severely injured the honor and dignity of many women. The aggression that occurred in modern world history, and recognizing
Government of Japan would like to take this opportunity once again to extend its that Japan carried out such acts in the past and inflicted suffering on the people of
sincere apologies and remorse to all those, irrespective of place of origin, who other countries, especially in Asia, the Members of this House hereby express
suffered immeasurable pain and incurable physical and psychological wounds as deep remorse. (Resolution of the House of Representatives adopted on June 9,
comfort women. 1995)

It is incumbent upon us, the Government of Japan, to continue to consider


seriously, while listening to the views of learned circles, how best we can express
e) Various Public Statements by Japanese Prime Minister Shinzo Abe
this sentiment.
I have talked about this matter in the Diet sessions last year, and recently as well,
and to the press. I have been consistent. I will stand by the Kono Statement. This is
We shall face squarely the historical facts as described above instead of evading
our consistent position. Further, we have been apologizing sincerely to those who
them, and take them to heart as lessons of history. We hereby reiterated our firm
suffered immeasurable pain and incurable psychological wounds as comfort
determination never to repeat the same mistake by forever engraving such issues
women. Former Prime Ministers, including Prime Ministers Koizumi and
in our memories through the study and teaching of history.
Hashimoto, have issued letters to the comfort women. I would like to be clear that
I carry the same feeling. This has not changed even slightly. (Excerpt from Remarks
As actions have been brought to court in Japan and interests have been shown in
by Prime Minister Abe at an Interview by NHK, March 11, 2007).
this issue outside Japan, the Government of Japan shall continue to pay full
attention to this matter, including private researched related thereto.
I am apologizing here and now. I am apologizing as the Prime Minister and it is as
stated in the statement by the Chief Cabinet Secretary Kono. (Excerpt from
Remarks by Prime Minister Abe at the Budget Committee, the House of
b) Prime Minister Tomiichi Murayamas Statement in 1994 Councilors, the Diet of Japan, March 26, 2007).

I am deeply sympathetic to the former comfort women who suffered hardships,


On the issue of wartime comfort women, which seriously stained the honor and
and I have expressed my apologies for the extremely agonizing circumstances into
dignity of many women, I would like to take this opportunity once again to express
which they were placed. (Excerpt from Telephone Conference by Prime Minister
my profound and sincere remorse and apologies
Abe to President George W. Bush, April 3, 2007).

I have to express sympathy from the bottom of my heart to those people who
c) Letters from the Prime Minister of Japan to Individual Comfort Women were taken as wartime comfort women. As a human being, I would like to express
The issue of comfort women, with the involvement of the Japanese military my sympathies, and also as prime minister of Japan I need to apologize to them.
authorities at that time, was a grave affront to the honor and dignity of a large My administration has been saying all along that we continue to stand by the
number of women. Kono Statement. We feel responsible for having forced these women to go
through that hardship and pain as comfort women under the circumstances at the
As Prime Minister of Japan, I thus extend anew my most sincere apologies and time. (Excerpt from an interview article "A Conversation with Shinzo Abe" by the
remorse to all the women who endured immeasurable and painful experiences Washington Post, April 22, 2007).
and suffered incurable physical and psychological wounds as comfort women.
x x x both personally and as Prime Minister of Japan, my heart goes out in
I believe that our country, painfully aware of its moral responsibilities, with feelings sympathy to all those who suffered extreme hardships as comfort women; and I
of apology and remorse, should face up squarely to its past history and accurately expressed my apologies for the fact that they were forced to endure such extreme
convey it to future generations. and harsh conditions. Human rights are violated in many parts of the world during
the 20th Century; therefore we must work to make the 21st Century a wonderful
century in which no human rights are violated. And the Government of Japan and
d) The Diet (Japanese Parliament) passed resolutions in 1995 and 2005 I wish to make significant contributions to that end. (Excerpt from Prime Minister
Abe's remarks at the Joint Press Availability after the summit meeting at Camp
David between Prime Minister Abe and President Bush, April 27, 2007).
349

whether to espouse petitioners


The Asian Women's Fund claims against Japan.

Established by the Japanese government in 1995, the AWF represented the government's concrete
Baker v. Carr[39] remains the starting point for analysis under the political question doctrine. There the US
attempt to address its moral responsibility by offering monetary compensation to victims of the comfort
Supreme Court explained that:
women system.[37] The purpose of the AWF was to show atonement of the Japanese people through
expressions of apology and remorse to the former wartime comfort women, to restore their honor, and x x x Prominent on the surface of any case held to involve a political question is
to demonstrate Japans strong respect for women.[38] found a textually demonstrable constitutional commitment of the issue to a
coordinate political department or a lack of judicially discoverable and
manageable standards for resolving it, or the impossibility of deciding without an
The AWF announced three programs for former comfort women who applied for assistance: (1) an initial policy determination of a kind clearly for non-judicial discretion; or the
impossibility of a court's undertaking independent resolution without expressing
atonement fund paying 2 million (approximately $20,000) to each woman; (2) medical and welfare lack of the respect due coordinate branches of government; or an unusual need
support programs, paying 2.5-3 million ($25,000-$30,000) for each woman; and (3) a letter of apology for unquestioning adherence to a political decision already made; or the
potentiality of embarrassment from multifarious pronouncements by various
from the Japanese Prime Minister to each woman. Funding for the program came from the Japanese departments on question.
government and private donations from the Japanese people. As of March 2006, the AWF provided 700
million (approximately $7 million) for these programs in South Korea, Taiwan, and the Philippines; 380
In Taada v. Cuenco,[40] we held that political questions refer "to those questions which, under the
million (approximately $3.8 million) in Indonesia; and 242 million (approximately $2.4 million) in
Constitution, are to be decided by the people in their sovereign capacity, or in regard to which full
the Netherlands.
discretionary authority has been delegated to the legislative or executive branch of the government. It is
concerned with issues dependent upon the wisdom, not legality of a particular measure."
On January 15, 1997, the AWF and the Philippine government signed a Memorandum of Understanding
for medical and welfare support programs for former comfort women. Over the next five years, these
Certain types of cases often have been found to present political questions.[41] One such category
were implemented by the Department of Social Welfare and Development.
involves questions of foreign relations. It is well-established that "[t]he conduct of the foreign relations of
our government is committed by the Constitution to the executive and legislative--'the political'--
Our Ruling
departments of the government, and the propriety of what may be done in the exercise of this political
power is not subject to judicial inquiry or decision."[42] The US Supreme Court has further cautioned that
Stripped down to its essentials, the issue in this case is whether the Executive Department committed
decisions relating to foreign policy are delicate, complex, and involve large elements of prophecy. They
grave abuse of discretion in not espousing petitioners claims for official apology and other forms of
are and should be undertaken only by those directly responsible to the people whose welfare they
reparations against Japan.
advance or imperil. They are decisions of a kind for which the Judiciary has neither aptitude, facilities nor
responsibility.[43]
The petition lacks merit.

From a Domestic Law Perspective,


To be sure, not all cases implicating foreign relations present political questions, and courts certainly
the Executive Department has the
exclusive prerogative to determine possess the authority to construe or invalidate treaties and executive agreements.[44]However, the
350

repudiation lead to breach of an international obligation, rupture of state


question whether the Philippine government should espouse claims of its nationals against a foreign
relations, forfeiture of confidence, national embarrassment and a plethora of
government is a foreign relations matter, the authority for which is demonstrably committed by our other problems with equally undesirable consequences.
Constitution not to the courts but to the political branches. In this case, the Executive Department has
already decided that it is to the best interest of the country to waive all claims of its nationals for
The Executive Department has determined that taking up petitioners cause would be inimical to our
reparations against Japan in the Treaty of Peace of 1951. The wisdom of such decision is not for the
countrys foreign policy interests, and could disrupt our relations with Japan, thereby creating serious
courts to question. Neither could petitioners herein assail the said determination by the Executive
implications for stability in this region. For us to overturn the Executive Departments determination
Department via the instant petition for certiorari.
would mean an assessment of the foreign policy judgments by a coordinate political branch to which
authority to make that judgment has been constitutionally committed.
In the seminal case of US v. Curtiss-Wright Export Corp.,[45] the US Supreme Court held that [t]he
President is the sole organ of the nation in its external relations, and its sole representative with foreign
In any event, it cannot reasonably be maintained that the Philippine government was without authority
relations.
to negotiate the Treaty of Peace with Japan. And it is equally true that, since time immemorial, when

It is quite apparent that if, in the maintenance of our international relations, negotiating peace accords and settling international claims:
embarrassment -- perhaps serious embarrassment -- is to be avoided and success
for our aims achieved, congressional legislation which is to be made effective x x x [g]overnments have dealt with x x x private claims as their own, treating them
through negotiation and inquiry within the international field must often accord to as national assets, and as counters, `chips', in international bargaining. Settlement
the President a degree of discretion and freedom from statutory restriction which agreements have lumped, or linked, claims deriving from private debts with
would not be admissible where domestic affairs alone involved. Moreover, he, not others that were intergovernmental in origin, and concessions in regard to one
Congress, has the better opportunity of knowing the conditions which prevail in category of claims might be set off against concessions in the other, or against
foreign countries, and especially is this true in time of war. He has his confidential larger political considerations unrelated to debts.[49]
sources of information. He has his agents in the form of diplomatic, consular and
other officials. x x x
Indeed, except as an agreement might otherwise provide, international settlements generally wipe out
the underlying private claims, thereby terminating any recourse under domestic law. In Ware v.
This ruling has been incorporated in our jurisprudence through Bayan v.
Hylton,[50] a case brought by a British subject to recover a debt confiscated by
Executive Secretary[46] and Pimentel v. Executive Secretary;[47] its overreaching principle was, perhaps,
the Commonwealth of Virginia during the war, Justice Chase wrote:
best articulated in (now Chief) Justice Punos dissent in Secretary of Justice v. Lantion:[48]

I apprehend that the treaty of peace abolishes the subject of the war, and that
x x x The conduct of foreign relations is full of complexities and consequences,
after peace is concluded, neither the matter in dispute, nor the conduct of either
sometimes with life and death significance to the nation especially in times of war.
party, during the war, can ever be revived, or brought into contest again. All
It can only be entrusted to that department of government which can act on the
violences, injuries, or damages sustained by the government, or people of either,
basis of the best available information and can decide with decisiveness. x x x It is
during the war, are buried in oblivion; and all those things are implied by the very
also the President who possesses the most comprehensive and the most
treaty of peace; and therefore not necessary to be expressed. Hence it follows,
confidential information about foreign countries for our diplomatic and consular
that the restitution of, or compensation for, British property confiscated, or
officials regularly brief him on meaningful events all over the world. He has also
extinguished, during the war, by any of the United States, could only be provided
unlimited access to ultra-sensitive military intelligence data.In fine, the presidential
for by the treaty of peace; and if there had been no provision, respecting these
role in foreign affairs is dominant and the President is traditionally accorded a
subjects, in the treaty, they could not be agitated after the treaty, by the British
wider degree of discretion in the conduct of foreign affairs. The regularity, nay,
government, much less by her subjects in courts of justice. (Emphasis supplied).
validity of his actions are adjudged under less stringent standards, lest their judicial
351

The official record of treaty negotiations establishes that a fundamental goal of the
This practice of settling claims by means of a peace treaty is certainly agreement was to settle the reparations issue once and for all. As the statement of
the chief United States negotiator, John Foster Dulles, makes clear, it was well
nothing new. For instance, in Dames & Moore v. Regan,[51] the US Supreme Court held:
understood that leaving open the possibility of future claims would be an
unacceptable impediment to a lasting peace:
Not infrequently in affairs between nations, outstanding claims by nationals of one
country against the government of another country are sources of friction Reparation is usually the most controversial aspect of
between the two sovereigns. United States v. Pink, 315 U.S. 203, 225, 62 S.Ct. 552, peacemaking. The present peace is no exception.
563, 86 L.Ed. 796 (1942). To resolve these difficulties, nations have often entered
into agreements settling the claims of their respective nationals. As one treatise On the one hand, there are claims both vast and
writer puts it, international agreements settling claims by nationals of one state just. Japan's aggression caused tremendous cost, losses and
against the government of another are established international practice suffering.
reflecting traditional international theory. L. Henkin, Foreign Affairs and the
Constitution 262 (1972). Consistent with that principle, the United States has On the other hand, to meet these claims, there stands
repeatedly exercised its sovereign authority to settle the claims of its nationals a Japan presently reduced to four home islands which are
against foreign countries. x x x Under such agreements, the President has agreed unable to produce the food its people need to live, or the
to renounce or extinguish claims of United States nationals against foreign raw materials they need to work. x x x
governments in return for lump-sum payments or the establishment of arbitration
procedures. To be sure, many of these settlements were encouraged by The policy of the United States that Japanese liability for reparations should be
the United States claimants themselves, since a claimant's only hope of obtaining sharply limited was informed by the experience of six years of United States-led
any payment at all might lie in having his Government negotiate a diplomatic occupation of Japan. During the occupation the Supreme Commander of the
settlement on his behalf. But it is also undisputed that the United States has Allied Powers (SCAP) for the region, General Douglas MacArthur, confiscated
sometimes disposed of the claims of its citizens without their consent, or even Japanese assets in conjunction with the task of managing the economic affairs of
without consultation with them, usually without exclusive regard for their the vanquished nation and with a view to reparations payments. It soon became
interests, as distinguished from those of the nation as a whole. Henkin, supra, at clear that Japan's financial condition would render any aggressive reparations
262-263. Accord, Restatement (Second) of Foreign Relations Law of the United plan an exercise in futility. Meanwhile, the importance of a stable,
States 213 (1965) (President may waive or settle a claim against a foreign state x x democratic Japan as a bulwark to communism in the region increased. At the
x [even] without the consent of the [injured] national). It is clear that the practice end of 1948, MacArthur expressed the view that [t]he use of reparations as a
of settling claims continues today. weapon to retard the reconstruction of a viable economy in Japan should be
combated with all possible means and recommended that the reparations issue
be settled finally and without delay.
Respondents explain that the Allied Powers concluded the Peace Treaty with Japan not necessarily for
That this policy was embodied in the treaty is clear not only from the negotiations
the complete atonement of the suffering caused by Japanese aggression during the war, not for the history but also from the Senate Foreign Relations Committee report
payment of adequate reparations, but for security purposes. The treaty sought to prevent the spread of recommending approval of the treaty by the Senate. The committee noted, for
example:
communism in Japan, which occupied a strategic position in the Far East. Thus, the Peace Treaty
compromised individual claims in the collective interest of the free world. Obviously insistence upon the payment of reparations in
any proportion commensurate with the claims of the
injured countries and their nationals would wreck Japan's
This was also the finding in a similar case involving American victims of Japanese slave labor during the economy, dissipate any credit that it may possess at
present, destroy the initiative of its people, and create
war.[52] In a consolidated case in the Northern District of California,[53] the court dismissed the lawsuits
misery and chaos in which the seeds of discontent and
filed, relying on the 1951 peace treaty with Japan,[54] because of the following policy considerations: communism would flourish. In short, [it] would be contrary
to the basic purposes and policy of x x x the United States
352

law. All they can do is resort to national law, if means are available, with a view to
furthering their cause or obtaining redress. The municipal legislator may lay upon
We thus hold that, from a municipal law perspective, that certiorari will not lie. As a general principle and
the State an obligation to protect its citizens abroad, and may also confer upon the
particularly here, where such an extraordinary length of time has lapsed between the treatys conclusion national a right to demand the performance of that obligation, and clothe the right
and our consideration the Executive must be given ample discretion to assess the foreign policy with corresponding sanctions. However, all these questions remain within the
province of municipal law and do not affect the position
considerations of espousing a claim against Japan, from the standpoint of both the interests of the internationally.[58] (Emphasis supplied)
petitioners and those of the Republic, and decide on that basis if apologies are sufficient, and whether
further steps are appropriate or necessary.
The State, therefore, is the sole judge to decide whether its protection will be granted, to what extent it

The Philippines is not under any is granted, and when will it cease. It retains, in this respect, a discretionary power the exercise of which
international obligation to espouse may be determined by considerations of a political or other nature, unrelated to the particular case.
petitioners claims.

The International Law Commissions (ILCs) Draft Articles on Diplomatic Protection fully support this
In the international sphere, traditionally, the only means available for individuals to bring a claim within traditional view. They (i) state that "the right of diplomatic protection belongs to or vests in the
the international legal system has been when the individual is able to persuade a government to bring a State,[59] (ii) affirm its discretionary nature by clarifying that diplomatic protection is a "sovereign
claim on the individuals behalf.[55] Even then, it is not the individuals rights that are being asserted, but prerogative" of the State;[60] and (iii) stress that the state "has the right to exercise diplomatic protection
rather, the states own rights. Nowhere is this position more clearly reflected than in the dictum of the on behalf of a national. It is under no duty or obligation to do so."[61]
Permanent Court of International Justice (PCIJ) in the 1924 Mavrommatis Palestine Concessions Case:
It has been argued, as petitioners argue now, that the State has a duty to protect its nationals and act on
By taking up the case of one of its subjects and by resorting to diplomatic action or
his/her behalf when rights are injured.[62] However, at present, there is no sufficient evidence to establish
international judicial proceedings on his behalf, a State is in reality asserting
its own right to ensure, in the person of its subjects, respect for the rules of a general international obligation for States to exercise diplomatic protection of their own nationals
international law. The question, therefore, whether the present dispute originates
abroad.[63] Though, perhaps desirable, neither state practice nor opinio juris has evolved in such a
in an injury to a private interest, which in point of fact is the case in many
international disputes, is irrelevant from this standpoint. Once a State has taken up direction. If it is a duty internationally, it is only a moral and not a legal duty, and there is no means of
a case on behalf of one of its subjects before an international tribunal, in the eyes enforcing its fulfillment.[64]
of the latter the State is sole claimant.[56]
Since the exercise of diplomatic protection is the right of the State, reliance on the right is within the
We fully agree that rape, sexual slavery, torture, and sexual violence are morally reprehensible as well as
absolute discretion of states, and the decision whether to exercise the discretion may invariably be
legally prohibited under contemporary international law.[65] However, petitioners take quite a
influenced by political considerations other than the legal merits of the particular claim.[57] As clearly
theoretical leap in claiming that these proscriptions automatically imply that that the Philippines is under
stated by the ICJ in
a non-derogable obligation to prosecute international crimes, particularly since petitioners do not
Barcelona Traction:
demand the imputation of individual criminal liability, but seek to recover monetary reparations from
The Court would here observe that, within the limits prescribed by international the state of Japan. Absent the consent of states, an applicable treaty regime, or a directive by the
law, a State may exercise diplomatic protection by whatever means and to Security Council, there is no non-derogable duty to institute proceedings against Japan.
whatever extent it thinks fit, for it is its own right that the State is asserting.
Should the natural or legal person on whose behalf it is acting consider that their Indeed, precisely because of states reluctance to directly prosecute claims against another
rights are not adequately protected, they have no remedy in international
353

state, recent developments support the modern trend to empower individuals to directly neither so clear nor so bright. Whatever the relevance of obligations erga omnes as a legal concept, its
participate in suits against perpetrators of international crimes. [66]Nonetheless, notwithstanding full potential remains to be realized in practice.[69]
an array of General Assembly resolutions calling for the prosecution of crimes against humanity and the The term is closely connected with the international law concept of jus cogens. In international law, the
strong policy arguments warranting such a rule, the practice of states does not yet support the present term jus cogens (literally, compelling law) refers to norms that command peremptory authority,
existence of an obligation to prosecute international crimes.[67] Of course a customary duty of superseding conflicting treaties and custom. Jus cogens norms are considered peremptory in the sense
prosecution is ideal, but we cannot find enough evidence to reasonably assert its existence. To the that they are mandatory, do not admit derogation, and can be modified only by general international
extent that any state practice in this area is widespread, it is in the practice of granting amnesties, norms of equivalent authority.[70]
immunity, selective prosecution, or de facto impunity to those who commit crimes against humanity.[68]
Early strains of the jus cogens doctrine have existed since the 1700s,[71] but peremptory norms began to
Even the invocation of jus cogens norms and erga omnes obligations will not alter this analysis. Even if attract greater scholarly attention with the publication of Alfred von Verdross's influential 1937 article,
we sidestep the question of whether jus cogens norms existed in 1951, petitioners have not deigned to Forbidden Treaties in International Law.[72] The recognition of jus cogens gained even more force in the
show that the crimes committed by the Japanese army violated jus cogens prohibitions at the time the 1950s and 1960s with the ILCs preparation of the Vienna Convention on the Law of Treaties
Treaty of Peace was signed, or that the duty to prosecute perpetrators of international crimes is an erga (VCLT).[73] Though there was a consensus that certain international norms had attained the status of jus
omnes obligation or has attained the status of jus cogens. cogens,[74] the ILC was unable to reach a consensus on the proper criteria for identifying peremptory
norms.
The term erga omnes (Latin: in relation to everyone) in international law has been used as a legal term After an extended debate over these and other theories of jus cogens, the ILC concluded ruefully in 1963
describing obligations owed by States towards the community of states as a whole. The concept was that there is not as yet any generally accepted criterion by which to identify a general rule of
recognized by the ICJ in Barcelona Traction: international law as having the character of jus cogens.[75] In a commentary accompanying the draft
convention, the ILC indicated that the prudent course seems to be to x x x leave the full content of this
x x x an essential distinction should be drawn between the obligations of a State
rule to be worked out in State practice and in the jurisprudence of international tribunals.[76] Thus,
towards the international community as a whole, and those arising vis--vis another
State in the field of diplomatic protection. By their very nature, the former are the while the existence of jus cogens in international law is undisputed, no consensus exists on its
concern of all States. In view of the importance of the rights involved, all States can substance,[77] beyond a tiny core of principles and rules.[78]
be held to have a legal interest in their protection; they are obligations erga
omnes.
Of course, we greatly sympathize with the cause of petitioners, and we cannot begin to comprehend the
Such obligations derive, for example, in contemporary international law, from the
outlawing of acts of aggression, and of genocide, as also from the principles and unimaginable horror they underwent at the hands of the Japanese soldiers. We are also deeply
rules concerning the basic rights of the human person, including protection from concerned that, in apparent contravention of fundamental principles of law, the petitioners appear to be
slavery and racial discrimination. Some of the corresponding rights of protection
have entered into the body of general international law others are conferred by without a remedy to challenge those that have offended them before appropriate fora. Needless to say,
international instruments of a universal or quasi-universal character. our government should take the lead in protecting its citizens against violation of their fundamental
human rights. Regrettably, it is not within our power to order the Executive Department to take up the
petitioners cause. Ours is only the power to urge and exhort the Executive Department to take up
The Latin phrase, erga omnes, has since become one of the rallying cries of those sharing a belief in the
petitioners cause.
emergence of a value-based international public order. However, as is so often the case, the reality is
354

RULING:
WHEREFORE, the Petition is hereby DISMISSED.
Petition lacks merit. From a Domestic Law Perspective, the Executive Department has the
SO ORDERED. exclusive prerogative to determine whether to espouse petitioners’ claims against Japan.
G.R. No. 162230, April 28, 2010

FACTS: Political questions refer “to those questions which, under the Constitution, are to be decided
This is an original Petition for Certiorari under Rule 65 of the Rules of Court with an by the people in their sovereign capacity, or in regard to which full discretionary authority
application for the issuance of a writ of preliminary mandatory injunction against the Office has been delegated to the legislative or executive branch of the government. It is concerned
of the Executive Secretary, the Secretary of the DFA, the Secretary of the DOJ, and the OSG. with issues dependent upon the wisdom, not legality of a particular measure.”

Petitioners are all members of the MALAYA LOLAS, a non-stock, non-profit organization One type of case of political questions involves questions of foreign relations. It is well-
registered with the SEC, established for the purpose of providing aid to the victims of rape by established that “the conduct of the foreign relations of our government is committed by the
Japanese military forces in the Philippines during the Second World War. Constitution to the executive and legislative–‘the political’–departments of the government,
and the propriety of what may be done in the exercise of this political power is not subject to
judicial inquiry or decision.” are delicate, complex, and involve large elements of prophecy.
Petitioners claim that since 1998, they have approached the Executive Department through They are and should be undertaken only by those directly responsible to the people whose
the DOJ, DFA, and OSG, requesting assistance in filing a claim against the Japanese officials welfare they advance or imperil.
and military officers who ordered the establishment of the “comfort women” stations in the
Philippines. But officials of the Executive Department declined to assist the petitioners, and
took the position that the individual claims of the comfort women for compensation had But not all cases implicating foreign relations present political questions, and courts certainly
already been fully satisfied by Japan’s compliance with the Peace Treaty between the possess the authority to construe or invalidate treaties and executive agreements. However,
Philippines and Japan. the question whether the Philippine government should espouse claims of its nationals
against a foreign government is a foreign relations matter, the authority for which is
demonstrably committed by our Constitution not to the courts but to the political branches.
Hence, this petition where petitioners pray for this court to (a) declare that respondents In this case, the Executive Department has already decided that it is to the best interest of
committed grave abuse of discretion amounting to lack or excess of discretion in refusing to the country to waive all claims of its nationals for reparations against Japan in the Treaty of
espouse their claims for the crimes against humanity and war crimes committed against Peace of 1951. The wisdom of such decision is not for the courts to question.
them; and (b) compel the respondents to espouse their claims for official apology and other
forms of reparations against Japan before the International Court of Justice (ICJ) and other
international tribunals. The President, not Congress, has the better opportunity of knowing the conditions which
prevail in foreign countries, and especially is this true in time of war. He has his confidential
sources of information. He has his agents in the form of diplomatic, consular and other
Respondents maintain that all claims of the Philippines and its nationals relative to the war officials.
were dealt with in the San Francisco Peace Treaty of 1951 and the bilateral Reparations
Agreement of 1956.
The Executive Department has determined that taking up petitioners’ cause would be
inimical to our country’s foreign policy interests, and could disrupt our relations with Japan,
On January 15, 1997, the Asian Women’s Fund and the Philippine government signed a thereby creating serious implications for stability in this region. For the to overturn the
Memorandum of Understanding for medical and welfare support programs for former Executive Department’s determination would mean an assessment of the foreign policy
comfort women. Over the next five years, these were implemented by the Department of judgments by a coordinate political branch to which authority to make that judgment has
Social Welfare and Development. been constitutionally committed.

ISSUE: From a municipal law perspective, certiorari will not lie. As a general principle, where such an
WON the Executive Department committed grave abuse of discretion in not espousing extraordinary length of time has lapsed between the treaty’s conclusion and our
petitioners’ claims for official apology and other forms of reparations against Japan. consideration – the Executive must be given ample discretion to assess the foreign policy
considerations of espousing a claim against Japan, from the standpoint of both the interests
355

of the petitioners and those of the Republic, and decide on that basis if apologies are G.R. No. 162230, April 28, 2010
sufficient, and whether further steps are appropriate or necessary.
FACTS:
In the international sphere, traditionally, the only means available for individuals to bring a
This is an original Petition for Certiorari under Rule 65 of the Rules of Court with an
claim within the international legal system has been when the individual is able to persuade
a government to bring a claim on the individual’s behalf. By taking up the case of one of its application for the issuance of a writ of preliminary mandatory injunction against the Office
subjects and by resorting to diplomatic action or international judicial proceedings on his of the Executive Secretary, the Secretary of the DFA, the Secretary of the DOJ, and the OSG.
behalf, a State is in reality asserting its own right to ensure, in the person of its subjects,
respect for the rules of international law. Petitioners are all members of the MALAYA LOLAS, a non-stock, non-profit organization
registered with the SEC, established for the purpose of providing aid to the victims of rape by
Japanese military forces in the Philippines during the Second World War.
Within the limits prescribed by international law, a State may exercise diplomatic protection
by whatever means and to whatever extent it thinks fit, for it is its own right that the State is
Petitioners claim that since 1998, they have approached the Executive Department through
asserting. Should the natural or legal person on whose behalf it is acting consider that their
rights are not adequately protected, they have no remedy in international law. All they can the DOJ, DFA, and OSG, requesting assistance in filing a claim against the Japanese officials
do is resort to national law, if means are available, with a view to furthering their cause or and military officers who ordered the establishment of the “comfort women” stations in the
obtaining redress. All these questions remain within the province of municipal law and do Philippines. But officials of the Executive Department declined to assist the petitioners, and
not affect the position internationally. took the position that the individual claims of the comfort women for compensation had
already been fully satisfied by Japan’s compliance with the Peace Treaty between the
Even the invocation of jus cogens norms and erga omnes obligations will not alter this Philippines and Japan.
analysis. Petitioners have not shown that the crimes committed by the Japanese army
violated jus cogens prohibitions at the time the Treaty of Peace was signed, or that the duty Hence, this petition where petitioners pray for this court to (a) declare that respondents
to prosecute perpetrators of international crimes is an erga omnes obligation or has attained committed grave abuse of discretion amounting to lack or excess of discretion in refusing to
the status of jus cogens. espouse their claims for the crimes against humanity and war crimes committed against
them; and (b) compel the respondents to espouse their claims for official apology and other
The term erga omnes (Latin: in relation to everyone) in international law has been used as a forms of reparations against Japan before the International Court of Justice (ICJ) and other
legal term describing obligations owed by States towards the community of states as a international tribunals.
whole. Essential distinction should be drawn between the obligations of a State towards the
international community as a whole, and those arising vis-à-vis another State in the field of Respondents maintain that all claims of the Philippines and its nationals relative to the war
diplomatic protection. By their very nature, the former are the concern of all States. In view were dealt with in the San Francisco Peace Treaty of 1951 and the bilateral Reparations
of the importance of the rights involved, all States can be held to have a legal interest in their Agreement of 1956.
protection; they are obligations erga omnes.
On January 15, 1997, the Asian Women’s Fund and the Philippine government signed a
The term “jus cogens” (literally, “compelling law”) refers to norms that command Memorandum of Understanding for medical and welfare support programs for former
peremptory authority, superseding conflicting treaties and custom. Jus cogens norms are comfort women. Over the next five years, these were implemented by the Department of
considered peremptory in the sense that they are mandatory, do not admit derogation, and Social Welfare and Development.
can be modified only by general international norms of equivalent authority
ISSUE: WON the Executive Department committed grave abuse of discretion in not espousing
WHEREFORE, the Petition is hereby DISMISSED. petitioners’ claims for official apology and other forms of reparations against Japan.

VINUYA vs. ROMULO


356

RULING: considerations of espousing a claim against Japan, from the standpoint of both the interests
of the petitioners and those of the Republic, and decide on that basis if apologies are
Petition lacks merit. From a Domestic Law Perspective, the Executive Department has the sufficient, and whether further steps are appropriate or necessary.
exclusive prerogative to determine whether to espouse petitioners’ claims against Japan.
In the international sphere, traditionally, the only means available for individuals to bring a
Political questions refer “to those questions which, under the Constitution, are to be decided claim within the international legal system has been when the individual is able to persuade
by the people in their sovereign capacity, or in regard to which full discretionary authority a government to bring a claim on the individual’s behalf. By taking up the case of one of its
has been delegated to the legislative or executive branch of the government. It is concerned subjects and by resorting to diplomatic action or international judicial proceedings on his
with issues dependent upon the wisdom, not legality of a particular measure.” behalf, a State is in reality asserting its own right to ensure, in the person of its subjects,
respect for the rules of international law.
One type of case of political questions involves questions of foreign relations. It is well-
established that “the conduct of the foreign relations of our government is committed by the Within the limits prescribed by international law, a State may exercise diplomatic protection
Constitution to the executive and legislative–‘the political’–departments of the government, by whatever means and to whatever extent it thinks fit, for it is its own right that the State is
and the propriety of what may be done in the exercise of this political power is not subject to asserting. Should the natural or legal person on whose behalf it is acting consider that their
judicial inquiry or decision.” are delicate, complex, and involve large elements of prophecy. rights are not adequately protected, they have no remedy in international law. All they can
They are and should be undertaken only by those directly responsible to the people whose do is resort to national law, if means are available, with a view to furthering their cause or
welfare they advance or imperil. obtaining redress. All these questions remain within the province of municipal law and do
not affect the position internationally.
But not all cases implicating foreign relations present political questions, and courts certainly
possess the authority to construe or invalidate treaties and executive agreements. However, Even the invocation of jus cogens norms and erga omnes obligations will not alter this
the question whether the Philippine government should espouse claims of its nationals analysis. Petitioners have not shown that the crimes committed by the Japanese army
against a foreign government is a foreign relations matter, the authority for which is violated jus cogens prohibitions at the time the Treaty of Peace was signed, or that the duty
demonstrably committed by our Constitution not to the courts but to the political branches. to prosecute perpetrators of international crimes is an erga omnes obligation or has attained
In this case, the Executive Department has already decided that it is to the best interest of the status of jus cogens.
the country to waive all claims of its nationals for reparations against Japan in the Treaty of
Peace of 1951. The wisdom of such decision is not for the courts to question. The term erga omnes (Latin: in relation to everyone) in international law has been used as a
legal term describing obligations owed by States towards the community of states as a
The President, not Congress, has the better opportunity of knowing the conditions which whole. Essential distinction should be drawn between the obligations of a State towards the
prevail in foreign countries, and especially is this true in time of war. He has his confidential international community as a whole, and those arising vis-à-vis another State in the field of
sources of information. He has his agents in the form of diplomatic, consular and other diplomatic protection. By their very nature, the former are the concern of all States. In view
officials. of the importance of the rights involved, all States can be held to have a legal interest in their
protection; they are obligations erga omnes.
The Executive Department has determined that taking up petitioners’ cause would be
inimical to our country’s foreign policy interests, and could disrupt our relations with Japan, The term “jus cogens” (literally, “compelling law”) refers to norms that command
thereby creating serious implications for stability in this region. For the to overturn the peremptory authority, superseding conflicting treaties and custom. Jus cogens norms are
Executive Department’s determination would mean an assessment of the foreign policy considered peremptory in the sense that they are mandatory, do not admit derogation, and
judgments by a coordinate political branch to which authority to make that judgment has can be modified only by general international norms of equivalent authority
been constitutionally committed.
WHEREFORE, the Petition is hereby DISMISSED.
From a municipal law perspective, certiorari will not lie. As a general principle, where such an
extraordinary length of time has lapsed between the treaty’s conclusion and our
consideration – the Executive must be given ample discretion to assess the foreign policy
357

EN BANC - Panunumpa sa Katungkulan ng Pangulo ng Pilipinas ayon sa Saligang


Batas5
G.R. No. 212426|January 12, 2016
The 1987 Constitution has "vested the executive power in the President of the Republic of
RENE A.V. SAGUISAG, WIGBERTO E. TAÑADA, FRANCISCO "DODONG" NEMENZO, JR., SR. the Philippines."6 While the vastness of the executive power that has been consolidated in
MARY JOHN MANANZAN, PACIFICO A. AGABIN, ESTEBAN "STEVE" SALONGA, H. HARRY L. the person of the President cannot be expressed fully in one provision, the Constitution has
ROQUE, JR., EVALYN G. URSUA, EDRE U. OLALIA, DR. CAROL PAGADUAN-ARAULLO, DR. stated the prime duty of the government, of which the President is the head:
ROLAND SIMBULAN, AND TEDDY CASIÑO, Petitioners,
vs. The prime duty of the Government is to serve and protect the people. The Government may
EXECUTIVE SECRETARY PAQUITO N. OCHOA, JR., DEPARTMENT OF NATIONAL DEFENSE call upon the people to defend the State and, in the fulfillment thereof, all citizens may be
SECRETARY VOLTAIRE GAZMIN, DEPARTMENT OF FOREIGN AFFAIRS SECRETARY ALBERT required, under conditions provided by law, to render personal military or civil
DEL ROSARIO, JR., DEPARTMENT OF BUDGET AND MANAGEMENT SECRETARY FLORENCIO service.7 (Emphases supplied)
ABAD, AND ARMED FORCES OF THE PHILIPPINES CHIEF OF STAFF GENERAL EMMANUEL T.
BAUTISTA, Respondents. B. The duty to protect the territory and the citizens of the Philippines, the power to call
upon the people to defend the State, and the President as Commander-in-Chief
DECISION
The duty to protect the State and its people must be carried out earnestly and effectively
SERENO, J.: throughout the whole territory of the Philippines in accordance with the constitutional
provision on national territory. Hence, the President of the Philippines, as the sole repository
The petitions1 before this Court question the constitutionality of the Enhanced Defense of executive power, is the guardian of the Philippine archipelago, including all the islands and
Cooperation Agreement (EDCA) between the Republic of the Philippines and the United waters embraced therein and all other territories over which it has sovereignty or
States of America (U.S.). Petitioners allege that respondents committed grave abuse of jurisdiction. These territories consist of its terrestrial, fluvial, and aerial domains; including its
discretion amounting to lack or excess of jurisdiction when they entered into EDCA with the territorial sea, the seabed, the subsoil, the insular shelves, and other submarine areas; and
U.S.,2 claiming that the instrument violated multiple constitutional provisions.3 In reply, the waters around, between, and connecting the islands of the archipelago, regardless of
respondents argue that petitioners lack standing to bring the suit. To support the legality of their breadth and dimensions.8
their actions, respondents invoke the 1987 Constitution, treaties, and judicial precedents. 4
To carry out this important duty, the President is equipped with authority over the Armed
A proper analysis of the issues requires this Court to lay down at the outset the basic Forces of the Philippines (AFP),9 which is the protector of the people and the state. The AFP's
parameters of the constitutional powers and roles of the President and the Senate in respect role is to secure the sovereignty of the State and the integrity of the national territory.10 In
of the above issues. A more detailed discussion of these powers and roles will be made in the addition, the Executive is constitutionally empowered to maintain peace and order; protect
latter portions. life, liberty, and property; and promote the general welfare.11

I. BROAD CONSTITUTIONAL CONTEXT OF THE POWERS OF THE PRESIDENT: DEFENSE, In recognition of these powers, Congress has specified that the President must oversee,
FOREIGN RELATIONS, AND EDCA ensure, and reinforce our defensive capabilities against external and internal threats12 and, in
the same vein, ensure that the country is adequately prepared for all national and local
emergencies arising from natural and man-made disasters.13
A. The Prime Duty of the State and the Consolidation of Executive Power in the President

To be sure, this power is limited by the Constitution itself. To illustrate, the President may
Mataimtim kong pinanunumpaan (o pinatotohanan) na tutuparin ko nang buong katapatan
call out the AFP to prevent or suppress instances of lawless violence, invasion or
at sigasig ang aking mga tungkulin bilang Pangulo (o Pangalawang Pangulo o
rebellion,14 but not suspend the privilege of the writ of habeas corpus for a period exceeding
Nanunungkulang Pangulo) ng Pilipinas, pangangalagaan at ipagtatanggol ang kanyang
60 days, or place the Philippines or any part thereof under martial law exceeding that same
Konstitusyon, ipatutupad ang mga batas nito, magiging makatarungan sa bawat tao, at
span. In the exercise of these powers, the President is also duty-bound to submit a report to
itatalaga ang aking sarili sa paglilingkod sa Bansa. Kasihan nawa aka ng Diyos.
Congress, in person or in writing, within 48 hours from the proclamation of martial law or the
suspension of the privilege of the writ of habeas corpus; and Congress may in turn revoke the
358

proclamation or suspension. The same provision provides for the Supreme Court's review of Previously, treaties under the 1973 Constitution required ratification by a majority of
the factual basis for the proclamation or suspension, as well as the promulgation of the the Batasang Pambansa,19except in instances wherein the President "may enter into
decision within 30 days from filing. international treaties or agreements as the national welfare and interest may require."20 This
left a large margin of discretion that the President could use to bypass the Legislature
C. The power and duty to conduct foreign relations altogether. This was a departure from the 1935 Constitution, which explicitly gave the
President the power to enter into treaties only with the concurrence of two-thirds of all the
Members of the Senate.21 The 1987 Constitution returned the Senate's power22 and, with it,
The President also carries the mandate of being the sole organ in the conduct of foreign
the legislative's traditional role in foreign affairs.23
relations.15 Since every state has the capacity to interact with and engage in relations with
other sovereign states,16 it is but logical that every state must vest in an agent the authority
to represent its interests to those other sovereign states. The responsibility of the President when it comes to treaties and international agreements
under the present Constitution is therefore shared with the Senate. This shared role,
petitioners claim, is bypassed by EDCA.
The conduct of foreign relations is full of complexities and consequences, sometimes with life
and death significance to the nation especially in times of war. It can only be entrusted to
that department of government which can act on the basis of the best available information II. HISTORICAL ANTECEDENTS OF EDCA
and can decide with decisiveness. x x x It is also the President who possesses the most
comprehensive and the most confidential information about foreign countries for our A. U.S. takeover of Spanish colonization and its military bases, and the transition to
diplomatic and consular officials regularly brief him on meaningful events all over the world. Philippine independence
He has also unlimited access to ultra-sensitive military intelligence data. In fine, the
presidential role in foreign affairs is dominant and the President is traditionally accorded a The presence of the U.S. military forces in the country can be traced to their pivotal victory in
wider degree of discretion in the conduct of foreign affairs. The regularity, nay, validity of his the 1898 Battle of Manila Bay during the Spanish-American War.24 Spain relinquished its
actions are adjudged under less stringent standards, lest their judicial repudiation lead to sovereignty over the Philippine Islands in favor of the U.S. upon its formal surrender a few
breach of an international obligation, rupture of state relations, forfeiture of confidence, months later.25 By 1899, the Americans had consolidated a military administration in the
national embarrassment and a plethora of other problems with equally undesirable archipelago.26
consequences.17
When it became clear that the American forces intended to impose colonial control over the
The role of the President in foreign affairs is qualified by the Constitution in that the Chief Philippine Islands, General Emilio Aguinaldo immediately led the Filipinos into an all-out war
Executive must give paramount importance to the sovereignty of the nation, the integrity of against the U.S.27 The Filipinos were ultimately defeated in the Philippine-American War,
its territory, its interest, and the right of the sovereign Filipino people to self- which lasted until 1902 and led to the downfall of the first Philippine Republic.28 The
determination.18 In specific provisions, the President's power is also limited, or at least Americans henceforth began to strengthen their foothold in the country.29 They took over
shared, as in Section 2 of Article II on the conduct of war; Sections 20 and 21 of Article VII on and expanded the former Spanish Naval Base in Subic Bay, Zambales, and put up a cavalry
foreign loans, treaties, and international agreements; Sections 4(2) and 5(2)(a) of Article VIII post called Fort Stotsenberg in Pampanga, now known as Clark Air Base.30
on the judicial review of executive acts; Sections 4 and 25 of Article XVIII on treaties and
international agreements entered into prior to the Constitution and on the presence of
When talks of the eventual independence of the Philippine Islands gained ground, the U.S.
foreign military troops, bases, or facilities.
manifested the desire to maintain military bases and armed forces in the country.31 The U.S.
Congress later enacted the Hare-Hawes-Cutting Act of 1933, which required that the
D. The relationship between the two major presidential functions and the role of the Senate proposed constitution of an independent Philippines recognize the right of the U.S. to
maintain the latter's armed forces and military bases.32 The Philippine Legislature rejected
Clearly, the power to defend the State and to act as its representative in the international that law, as it also gave the U.S. the power to unilaterally designate any part of Philippine
sphere inheres in the person of the President. This power, however, does not crystallize into territory as a permanent military or naval base of the U.S. within two years from complete
absolute discretion to craft whatever instrument the Chief Executive so desires. As previously independence.33
mentioned, the Senate has a role in ensuring that treaties or international agreements the
President enters into, as contemplated in Section 21 of Article VII of the Constitution, obtain The U.S. Legislature subsequently crafted another law called the Tydings-McDuffie Act or the
the approval of two-thirds of its members. Philippine Independence Act of 1934. Compared to the old Hare-Hawes-Cutting Act, the new
law provided for the surrender to the Commonwealth Government of "all military and other
359

reservations" of the U.S. government in the Philippines, except "naval reservations and the former' s use of the bases for military combat operations or the establishment of long-
refueling stations."34 Furthermore, the law authorized the U.S. President to enter into range missiles.53
negotiations for the adjustment and settlement of all questions relating to naval reservations
and fueling stations within two years after the Philippines would have gained Pursuant to the legislative authorization granted under Republic Act No. 9,54 the President
independence.35 Under the Tydings-McDuffie Act, the U.S. President would proclaim the also entered into the 1947 Military Assistance Agreement55 with the U.S. This executive
American withdrawal and surrender of sovereignty over the islands 10 years after the agreement established the conditions under which U.S. military assistance would be granted
inauguration of the new government in the Philippines.36 This law eventually led to the to the Philippines,56 particularly the provision of military arms, ammunitions, supplies,
promulgation of the 1935 Philippine Constitution. equipment, vessels, services, and training for the latter's defense forces.57 An exchange of
notes in 1953 made it clear that the agreement would remain in force until terminated by
The original plan to surrender the military bases changed.37 At the height of the Second any of the parties.58
World War, the Philippine and the U.S. Legislatures each passed resolutions authorizing their
respective Presidents to negotiate the matter of retaining military bases in the country after To further strengthen their defense and security relationship,59 the Philippines and the U.S.
the planned withdrawal of the U.S.38 Subsequently, in 1946, the countries entered into the next entered into the MDT in 1951. Concurred in by both the Philippine60 and the
Treaty of General Relations, in which the U.S. relinquished all control and sovereignty over U.S.61 Senates, the treaty has two main features: first, it allowed for mutual assistance in
the Philippine Islands, except the areas that would be covered by the American military bases maintaining and developing their individual and collective capacities to resist an armed
in the country.39 This treaty eventually led to the creation of the post-colonial legal regime on attack;62 and second, it provided for their mutual self-defense in the event of an armed
which would hinge the continued presence of U.S. military forces until 1991: the Military attack against the territory of either party.63 The treaty was premised on their recognition
Bases Agreement (MBA) of 1947, the Military Assistance Agreement of 1947, and the Mutual that an armed attack on either of them would equally be a threat to the security of the
Defense Treaty (MDT) of 1951.40 other.64

B. Former legal regime on the presence of U.S. armed forces in the territory of an C. Current legal regime on the presence of U.S. armed forces in the country
independent Philippines (1946-1991)
In view of the impending expiration of the 1947 MBA in 1991, the Philippines and the U.S.
Soon after the Philippines was granted independence, the two countries entered into their negotiated for a possible renewal of their defense and security relationship.65 Termed as the
first military arrangement pursuant to the Treaty of General Relations - the 1947 MBA.41 The Treaty of Friendship, Cooperation and Security, the countries sought to recast their military
Senate concurred on the premise of "mutuality of security interest,"42 which provided for the ties by providing a new framework for their defense cooperation and the use of Philippine
presence and operation of 23 U.S. military bases in the Philippines for 99 years or until the installations.66 One of the proposed provisions included an arrangement in which U.S. forces
year 2046.43 The treaty also obliged the Philippines to negotiate with the U.S. to allow the would be granted the use of certain installations within the Philippine naval base in
latter to expand the existing bases or to acquire new ones as military necessity might Subic.67 On 16 September 1991, the Senate rejected the proposed treaty.68
require.44
The consequent expiration of the 1947 MBA and the resulting paucity of any formal
A number of significant amendments to the 1947 MBA were made.45 With respect to its agreement dealing with the treatment of U.S. personnel in the Philippines led to the
duration, the parties entered into the Ramos-Rusk Agreement of 1966, which reduced the suspension in 1995 of large-scale joint military exercises.69In the meantime, the respective
term of the treaty from 99 years to a total of 44 years or until 1991.46 Concerning the number governments of the two countries agreed70 to hold joint exercises at a substantially reduced
of U.S. military bases in the country, the Bohlen-Serrano Memorandum of Agreement level.71 The military arrangements between them were revived in 1999 when they concluded
provided for the return to the Philippines of 17 U.S. military bases covering a total area of the first Visiting Forces Agreement (VFA).72
117,075 hectares.47 Twelve years later, the U.S. returned Sangley Point in Cavite City through
an exchange of notes.48 Then, through the Romulo-Murphy Exchange of Notes of 1979, the
As a "reaffirm[ation] [of the] obligations under the MDT,"73 the VFA has laid down the
parties agreed to the recognition of Philippine sovereignty over Clark and Subic Bases and the
regulatory mechanism for the treatment of U.S. military and civilian personnel visiting the
reduction of the areas that could be used by the U.S. military. 49 The agreement also provided
country.74 It contains provisions on the entry and departure of U.S. personnel; the purpose,
for the mandatory review of the treaty every five years.50 In 1983, the parties revised the
extent, and limitations of their activities; criminal and disciplinary jurisdiction; the waiver of
1947 MBA through the Romualdez-Armacost Agreement.51 The revision pertained to the
certain claims; the importation and exportation of equipment, materials, supplies, and other
operational use of the military bases by the U.S. government within the context of Philippine
pieces of property owned by the U.S. government; and the movement of U.S. military
sovereignty,52 including the need for prior consultation with the Philippine government on
vehicles, vessels, and aircraft into and within the country.75 The Philippines and the U.S. also
entered into a second counterpart agreement (VFA II), which in turn regulated the treatment
360

of Philippine military and civilian personnel visiting the U.S.76 The Philippine Senate III. ISSUES
concurred in the first VFA on 27 May 1999.77
Petitioners mainly seek a declaration that the Executive Department committed grave abuse
Beginning in January 2002, U.S. military and civilian personnel started arriving in Mindanao to of discretion in entering into EDCA in the form of an executive agreement. For this reason,
take part in joint military exercises with their Filipino counterparts.78 Called Balikatan, these we cull the issues before us:
exercises involved trainings aimed at simulating joint military maneuvers pursuant to the
MDT.79 A. Whether the essential requisites for judicial review are present

In the same year, the Philippines and the U.S. entered into the Mutual Logistics Support B. Whether the President may enter into an executive agreement on foreign military bases,
Agreement to "further the interoperability, readiness, and effectiveness of their respective troops, or facilities
military forces"80 in accordance with the MDT, the Military Assistance Agreement of 1953,
and the VFA.81 The new agreement outlined the basic terms, conditions, and procedures for
C. Whether the provisions under EDCA are consistent with the Constitution, as well as with
facilitating the reciprocal provision of logistics support, supplies, and services between the
existing laws and treaties
military forces of the two countries.82 The phrase "logistics support and services" includes
billeting, operations support, construction and use of temporary structures, and storage
services during an approved activity under the existing military arrangements.83 Already IV. DISCUSSION
extended twice, the agreement will last until 2017.84
A. Whether the essential requisites for judicial review have been satisfied
D. The Enhanced Defense Cooperation Agreement
Petitioners are hailing this Court's power of judicial review in order to strike down EDCA for
EDCA authorizes the U.S. military forces to have access to and conduct activities within violating the Constitution. They stress that our fundamental law is explicit in prohibiting the
certain "Agreed Locations" in the country. It was not transmitted to the Senate on the presence of foreign military forces in the country, except under a treaty concurred in by the
executive's understanding that to do so was no longer necessary.85 Accordingly, in June 2014, Senate. Before this Court may begin to analyze the constitutionality or validity of an official
the Department of Foreign Affairs (DFA) and the U.S. Embassy exchanged diplomatic notes act of a coequal branch of government, however, petitioners must show that they have
confirming the completion of all necessary internal requirements for the agreement to enter satisfied all the essential requisites for judicial review.93
into force in the two countries.86
Distinguished from the general notion of judicial power, the power of judicial review specially
According to the Philippine government, the conclusion of EDCA was the result of intensive refers to both the authority and the duty of this Court to determine whether a branch or an
and comprehensive negotiations in the course of almost two years.87 After eight rounds of instrumentality of government has acted beyond the scope of the latter's constitutional
negotiations, the Secretary of National Defense and the U.S. Ambassador to the Philippines powers.94 As articulated in Section 1, Article VIII of the Constitution, the power of judicial
signed the agreement on 28 April 2014.88 President Benigno S. Aquino III ratified EDCA on 6 review involves the power to resolve cases in which the questions concern the
June 2014.89 The OSG clarified during the oral arguments90 that the Philippine and the U.S. constitutionality or validity of any treaty, international or executive agreement, law,
governments had yet to agree formally on the specific sites of the Agreed Locations presidential decree, proclamation, order, instruction, ordinance, or regulation. 95 In Angara v.
mentioned in the agreement. Electoral Commission, this Court exhaustively discussed this "moderating power" as part of
the system of checks and balances under the Constitution. In our fundamental law, the role
of the Court is to determine whether a branch of government has adhered to the specific
Two petitions for certiorari were thereafter filed before us assailing the constitutionality of
restrictions and limitations of the latter's power:96
EDCA. They primarily argue that it should have been in the form of a treaty concurred in by
the Senate, not an executive agreement.
The separation of powers is a fundamental principle in our system of government. It obtains
not through express provision but by actual division in our Constitution. Each department of
On 10 November 2015, months after the oral arguments were concluded and the parties
the government has exclusive cognizance of matters within its jurisdiction, and is supreme
ordered to file their respective memoranda, the Senators adopted Senate Resolution No. (SR)
within its own sphere. But it does not follow from the fact that the three powers are to be
105.91 The resolution expresses the "strong sense"92 of the Senators that for EDCA to become
kept separate and distinct that the Constitution intended them to be absolutely unrestrained
valid and effective, it must first be transmitted to the Senate for deliberation and
and independent of each other. The Constitution has provided for an elaborate system of
concurrence.
checks and balances to secure coordination in the workings of the various departments of
361

the government. x x x. And the judiciary in turn, with the Supreme Court as the final coequal branch of government, this Court must continually exercise restraint to avoid the risk
arbiter, effectively checks the other departments in the exercise of its power to determine of supplanting the wisdom of the constitutionally appointed actor with that of its own.100
the law, and hence to declare executive and legislative acts void if violative of the
Constitution. Even as we are left with no recourse but to bare our power to check an act of a coequal
branch of government - in this case the executive - we must abide by the stringent
As any human production, our Constitution is of course lacking perfection and perfectibility, requirements for the exercise of that power under the Constitution. Demetria v.
but as much as it was within the power of our people, acting through their delegates to so Alba101 and Francisco v. House of Representatives102 cite the "pillars" of the limitations on the
provide, that instrument which is the expression of their sovereignty however limited, has power of judicial review as enunciated in the concurring opinion of U.S. Supreme Court
established a republican government intended to operate and function as a harmonious Justice Brandeis in Ashwander v. Tennessee Valley Authority.103 Francisco104 redressed these
whole, under a system of checks and balances, and subject to specific limitations and "pillars" under the following categories:
restrictions provided in the said instrument. The Constitution sets forth in no uncertain
language the restrictions and limitations upon governmental powers and agencies. If these 1. That there be absolute necessity of deciding a case
restrictions and limitations are transcended it would be inconceivable if the Constitution
had not provided for a mechanism by which to direct the course of government along
2. That rules of constitutional law shall be formulated only as required by the
constitutional channels, for then the distribution of powers would be mere verbiage, the
facts of the case
bill of rights mere expressions of sentiment, and the principles of good government mere
political apothegms. Certainly, the limitations and restrictions embodied in our Constitution
are real as they should be in any living constitution. x x x. In our case, this moderating power 3. That judgment may not be sustained on some other ground
is granted, if not expressly, by clear implication from section 2 of article VIII of [the 1935]
Constitution. 4. That there be actual injury sustained by the party by reason of the operation of
the statute
The Constitution is a definition of the powers of government. Who is to determine the
nature, scope and extent of such powers? The Constitution itself has provided for the 5. That the parties are not in estoppel
instrumentality of the judiciary as the rational way. And when the judiciary mediates to
allocate constitutional boundaries, it does not assert any superiority over the other 6. That the Court upholds the presumption of constitutionality
departments; it does not in reality nullify or invalidate an act of the legislature, but only
asserts the solemn and sacred obligation assigned to it by the Constitution to determine
conflicting claims of authority under the Constitution and to establish for the parties in an These are the specific safeguards laid down by the Court when it exercises its power of
actual controversy the rights which that instrument secures and guarantees to them. This is judicial review.105 Guided by these pillars, it may invoke the power only when the following
in truth all that is involved in what is termed "judicial supremacy" which properly is the four stringent requirements are satisfied: (a) there is an actual case or controversy; (b)
power of judicial review under the Constitution. petitioners possess locus standi; (c) the question of constitutionality is raised at the earliest
opportunity; and (d) the issue of constitutionality is the lis mota of the case.106 Of these four,
the first two conditions will be the focus of our discussion.
The power of judicial review has since been strengthened in the 1987 Constitution. The
scope of that power has been extended to the determination of whether in matters
traditionally considered to be within the sphere of appreciation of another branch of 1. Petitioners have shown the presence of an actual case or controversy.
government, an exercise of discretion has been attended with grave abuse.97 The expansion
of this power has made the political question doctrine "no longer the insurmountable The OSG maintains107 that there is no actual case or controversy that exists, since the
obstacle to the exercise of judicial power or the impenetrable shield that protects executive Senators have not been deprived of the opportunity to invoke the privileges of the institution
and legislative actions from judicial inquiry or review."98 they are representing. It contends that the nonparticipation of the Senators in the present
petitions only confirms that even they believe that EDCA is a binding executive agreement
This moderating power, however, must be exercised carefully and only if it cannot be that does not require their concurrence.
completely avoided. We stress that our Constitution is so incisively designed that it identifies
the spheres of expertise within which the different branches of government shall function It must be emphasized that the Senate has already expressed its position through SR
and the questions of policy that they shall resolve.99 Since the power of judicial review 105.108 Through the Resolution, the Senate has taken a position contrary to that of the OSG.
involves the delicate exercise of examining the validity or constitutionality of an act of a As the body tasked to participate in foreign affairs by ratifying treaties, its belief that EDCA
362

infringes upon its constitutional role indicates that an actual controversy - albeit brought to general.116 Clearly, it would be insufficient to show that the law or any governmental act is
the Court by non-Senators, exists. invalid, and that petitioners stand to suffer in some indefinite way.117 They must show that
they have a particular interest in bringing the suit, and that they have been or are about to
Moreover, we cannot consider the sheer abstention of the Senators from the present be denied some right or privilege to which they are lawfully entitled, or that they are about
proceedings as basis for finding that there is no actual case or controversy before us. We to be subjected to some burden or penalty by reason of the act complained of.118 The reason
point out that the focus of this requirement is the ripeness for adjudication of the matter at why those who challenge the validity of a law or an international agreement are required to
hand, as opposed to its being merely conjectural or anticipatory.109 The case must involve a allege the existence of a personal stake in the outcome of the controversy is "to assure the
definite and concrete issue involving real parties with conflicting legal rights and legal claims concrete adverseness which sharpens the presentation of issues upon which the court so
admitting of specific relief through a decree conclusive in nature.110 It should not equate with largely depends for illumination of difficult constitutional questions."119
a mere request for an opinion or advice on what the law would be upon an abstract,
hypothetical, or contingent state of facts.111 As explained in Angara v. Electoral The present petitions cannot qualify as citizens', taxpayers', or legislators' suits; the Senate as
Commission:112 a body has the requisite standing, but considering that it has not formally filed a pleading to
join the suit, as it merely conveyed to the Supreme Court its sense that EDCA needs the
[The] power of judicial review is limited to actual cases and controversies to be exercised Senate's concurrence to be valid, petitioners continue to suffer from lack of standing.
after full opportunity of argument by the parties, and limited further to the constitutional
question raised or the very lis mota presented. Any attempt at abstraction could only lead In assailing the constitutionality of a governmental act, petitioners suing as citizens may
to dialectics and barren legal questions and to sterile conclusions of wisdom, justice or dodge the requirement of having to establish a direct and personal interest if they show that
expediency of legislation. More than that, courts accord the presumption of constitutionality the act affects a public right.120 In arguing that they have legal standing, they claim121 that the
to legislative enactments, not only because the legislature is presumed to abide by the case they have filed is a concerned citizen's suit. But aside from general statements that the
Constitution but also because the judiciary in the determination of actual cases and petitions involve the protection of a public right, and that their constitutional rights as
controversies must reflect the wisdom and justice of the people as expressed through their citizens would be violated, they fail to make any specific assertion of a particular public right
representatives in the executive and legislative departments of the government. that would be violated by the enforcement of EDCA. For their failure to do so, the present
(Emphases supplied) petitions cannot be considered by the Court as citizens' suits that would justify a disregard
of the aforementioned requirements.
We find that the matter before us involves an actual case or controversy that is already ripe
for adjudication. The Executive Department has already sent an official confirmation to the In claiming that they have legal standing as taxpayers, petitioners122 aver that the
U.S. Embassy that "all internal requirements of the Philippines x x x have already been implementation of EDCA would result in the unlawful use of public funds. They emphasize
complied with."113 By this exchange of diplomatic notes, the Executive Department that Article X(1) refers to an appropriation of funds; and that the agreement entails a waiver
effectively performed the last act required under Article XII(l) of EDCA before the agreement of the payment of taxes, fees, and rentals. During the oral arguments, however, they
entered into force. Section 25, Article XVIII of the Constitution, is clear that the presence of admitted that the government had not yet appropriated or actually disbursed public funds
foreign military forces in the country shall only be allowed by virtue of a treaty concurred in for the purpose of implementing the agreement.123 The OSG, on the other hand, maintains
by the Senate. Hence, the performance of an official act by the Executive Department that that petitioners cannot sue as taxpayers.124Respondent explains that EDCA is neither meant
led to the entry into force of an executive agreement was sufficient to satisfy the actual case to be a tax measure, nor is it directed at the disbursement of public funds.
or controversy requirement.
A taxpayer's suit concerns a case in which the official act complained of directly involves the
2. While petitioners Saguisag et. al., do not have legal standing, they nonetheless raise illegal disbursement of public funds derived from taxation.125 Here, those challenging the act
issues involving matters of transcendental importance. must specifically show that they have sufficient interest in preventing the illegal expenditure
of public money, and that they will sustain a direct injury as a result of the enforcement of
The question of locus standi or legal standing focuses on the determination of whether those the assailed act.126 Applying that principle to this case, they must establish that EDCA
assailing the governmental act have the right of appearance to bring the matter to the court involves the exercise by Congress of its taxing or spending powers.127
for adjudication.114 They must show that they have a personal and substantial interest in the
case, such that they have sustained or are in immediate danger of sustaining, some direct We agree with the OSG that the petitions cannot qualify as taxpayers' suits. We emphasize
injury as a consequence of the enforcement of the challenged governmental act.115 Here, that a taxpayers' suit contemplates a situation in which there is already an appropriation or a
"interest" in the question involved must be material - an interest that is in issue and will be disbursement of public funds.128 A reading of Article X(l) of EDCA would show that there has
affected by the official act - as distinguished from being merely incidental or
363

been neither an appropriation nor an authorization of disbursement of funds. The cited unless they are to commit a flagrant betrayal of public trust. They are representatives of the
provision reads: sovereign people and it is their sacred duty to see to it that the fundamental law
embodying the will of the sovereign people is not trampled upon. (Emphases supplied)
All obligations under this Agreement are subject to the availability of appropriated
funds authorized for these purposes. (Emphases supplied) We emphasize that in a legislators' suit, those Members of Congress who are challenging the
official act have standing only to the extent that the alleged violation impinges on their right
This provision means that if the implementation of EDCA would require the disbursement of to participate in the exercise of the powers of the institution of which they are
public funds, the money must come from appropriated funds that are members.135 Legislators have the standing "to maintain inviolate the prerogatives, powers,
specifically authorized for this purpose. Under the agreement, before there can even be a and privileges vested by the Constitution in their office and are allowed to sue to question
disbursement of public funds, there must first be a legislative action. Until and unless the the validity of any official action, which they claim infringes their prerogatives as
Legislature appropriates funds for EDCA, or unless petitioners can pinpoint a specific item legislators."136 As legislators, they must clearly show that there was a direct injury to their
in the current budget that allows expenditure under the agreement, we cannot at this time persons or the institution to which they belong.137
rule that there is in fact an appropriation or a disbursement of funds that would justify the
filing of a taxpayers' suit. As correctly argued by respondent, the power to concur in a treaty or an international
agreement is an institutional prerogative granted by the Constitution to the Senate, not to
Petitioners Bayan et al. also claim129 that their co-petitioners who are party-list the entire Legislature. In Pimentel v. Office of the Executive Secretary, this Court did not
representatives have the standing to challenge the act of the Executive Department, recognize the standing of one of the petitioners therein who was a member of the House of
especially if it impairs the constitutional prerogatives, powers, and privileges of their office. Representatives. The petition in that case sought to compel the transmission to the Senate
While they admit that there is no incumbent Senator who has taken part in the present for concurrence of the signed text of the Statute of the International Criminal Court. Since
petition, they nonetheless assert that they also stand to sustain a derivative but substantial that petition invoked the power of the Senate to grant or withhold its concurrence in a treaty
injury as legislators. They argue that under the Constitution, legislative power is vested in entered into by the Executive Department, only then incumbent Senator Pimentel was
both the Senate and the House of Representatives; consequently, it is the entire Legislative allowed to assert that authority of the Senate of which he was a member.
Department that has a voice in determining whether or not the presence of foreign military
should be allowed. They maintain that as members of the Legislature, they have the requisite Therefore, none of the initial petitioners in the present controversy has the standing to
personality to bring a suit, especially when a constitutional issue is raised. maintain the suits as legislators.

The OSG counters130 that petitioners do not have any legal standing to file the suits Nevertheless, this Court finds that there is basis for it to review the act of the Executive for
concerning the lack of Senate concurrence in EDCA. Respondent emphasizes that the power the following reasons.
to concur in treaties and international agreements is an "institutional prerogative" granted
by the Constitution to the Senate. Accordingly, the OSG argues that in case of an allegation of In any case, petitioners raise issues involving matters of transcendental importance.
impairment of that power, the injured party would be the Senate as an institution or any of
its incumbent members, as it is the Senate's constitutional function that is allegedly being
Petitioners138 argue that the Court may set aside procedural technicalities, as the present
violated.
petition tackles issues that are of transcendental importance. They point out that the matter
before us is about the proper exercise of the Executive Department's power to enter into
The legal standing of an institution of the Legislature or of any of its Members has already international agreements in relation to that of the Senate to concur in those agreements.
been recognized by this Court in a number of cases.131 What is in question here is the alleged They also assert that EDCA would cause grave injustice, as well as irreparable violation of the
impairment of the constitutional duties and powers granted to, or the impermissible Constitution and of the Filipino people's rights.
intrusion upon the domain of, the Legislature or an institution thereof.132 In the case of suits
initiated by the legislators themselves, this Court has recognized their standing to question
The OSG, on the other hand, insists139 that petitioners cannot raise the mere fact that the
the validity of any official action that they claim infringes the prerogatives, powers, and
present petitions involve matters of transcendental importance in order to cure their inability
privileges vested by the Constitution in their office.133 As aptly explained by Justice Perfecto
to comply with the constitutional requirement of standing. Respondent bewails the overuse
in Mabanag v. Lopez Vito:134
of "transcendental importance" as an exception to the traditional requirements of
constitutional litigation. It stresses that one of the purposes of these requirements is to
Being members of Congress, they are even duty bound to see that the latter act within the protect the Supreme Court from unnecessary litigation of constitutional questions.
bounds of the Constitution which, as representatives of the people, they should uphold,
364

In a number of cases,140 this Court has indeed taken a liberal stance towards the requirement among these three great branches.143 By this division, the law implied that the divided
of legal standing, especially when paramount interest is involved. Indeed, when those who powers cannot be exercised except by the department given the power.144
challenge the official act are able to craft an issue of transcendental significance to the
people, the Court may exercise its sound discretion and take cognizance of the suit. It may do This divide continued throughout the different versions of the Philippine Constitution and
so in spite of the inability of the petitioners to show that they have been personally injured specifically vested the supreme executive power in the Governor-General of the
by the operation of a law or any other government act. Philippines,145 a position inherited by the President of the Philippines when the country
attained independence. One of the principal functions of the supreme executive is the
While this Court has yet to thoroughly delineate the outer limits of this doctrine, we responsibility for the faithful execution of the laws as embodied by the oath of office.146 The
emphasize that not every other case, however strong public interest may be, can qualify as oath of the President prescribed by the 1987 Constitution reads thus:
an issue of transcendental importance. Before it can be impelled to brush aside the essential
requisites for exercising its power of judicial review, it must at the very least consider a I do solemnly swear (or affirm) that I will faithfully and conscientiously fulfill my duties as
number of factors: (1) the character of the funds or other assets involved in the case; (2) the President (or Vice-President or Acting President) of the Philippines, preserve and defend its
presence of a clear case of disregard of a constitutional or statutory prohibition by the public Constitution, execute its laws, do justice to every man, and consecrate myself to the service
respondent agency or instrumentality of the government; and (3) the lack of any other party of the Nation. So help me God. (In case of affirmation, last sentence will be
that has a more direct and specific interest in raising the present questions.141 omitted.)147 (Emphases supplied)

An exhaustive evaluation of the memoranda of the parties, together with the oral This Court has interpreted the faithful execution clause as an obligation imposed on the
arguments, shows that petitioners have presented serious constitutional issues that provide President, and not a separate grant of power.148 Section 1 7, Article VII of the Constitution,
ample justification for the Court to set aside the rule on standing. The transcendental expresses this duty in no uncertain terms and includes it in the provision regarding the
importance of the issues presented here is rooted in the Constitution itself. Section 25, President's power of control over the executive department, viz:
Article XVIII thereof, cannot be any clearer: there is a much stricter mechanism required
before foreign military troops, facilities, or bases may be allowed in the country. The DFA has
The President shall have control of all the executive departments, bureaus, and offices. He
already confirmed to the U.S. Embassy that "all internal requirements of the Philippines x x x
shall ensure that the laws be faithfully executed.
have already been complied with."142 It behooves the Court in this instance to take a liberal
stance towards the rule on standing and to determine forthwith whether there was grave
abuse of discretion on the part of the Executive Department. The equivalent provisions in the next preceding Constitution did not explicitly require this
oath from the President. In the 1973 Constitution, for instance, the provision simply gives the
President control over the ministries.149 A similar language, not in the form of the President's
We therefore rule that this case is a proper subject for judicial review.
oath, was present in the 1935 Constitution, particularly in the enumeration of executive
functions.150 By 1987, executive power was codified not only in the Constitution, but also in
B. Whether the President may enter into an executive agreement on foreign the Administrative Code:151
military bases, troops, or facilities
SECTION 1. Power of Control. - The President shall have control of all the executive
C. Whether the provisions under EDCA are consistent with the Constitution, as departments, bureaus, and offices. He shall ensure that the laws be faithfully executed.
well as with existing laws and treaties (Emphasis supplied)

Issues B and C shall be discussed together infra. Hence, the duty to faithfully execute the laws of the land is inherent in executive power and
is intimately related to the other executive functions. These functions include the faithful
1. The role of the President as the executor of the law includes the duty to defend the execution of the law in autonomous regions;152 the right to prosecute crimes;153 the
State, for which purpose he may use that power in the conduct of foreign relations implementation of transportation projects;154 the duty to ensure compliance with treaties,
executive agreements and executive orders;155 the authority to deport undesirable
Historically, the Philippines has mirrored the division of powers in the U.S. government. aliens;156 the conferment of national awards under the President's jurisdiction;157 and the
When the Philippine government was still an agency of the Congress of the U.S., it was as an overall administration and control of the executive department.158
agent entrusted with powers categorized as executive, legislative, and judicial, and divided
365

These obligations are as broad as they sound, for a President cannot function with crippled It would therefore be remiss for the President and repugnant to the faithful-execution clause
hands, but must be capable of securing the rule of law within all territories of the Philippine of the Constitution to do nothing when the call of the moment requires increasing the
Islands and be empowered to do so within constitutional limits. Congress cannot, for military's defensive capabilities, which could include forging alliances with states that hold a
instance, limit or take over the President's power to adopt implementing rules and common interest with the Philippines or bringing an international suit against an offending
regulations for a law it has enacted.159 state.

More important, this mandate is self-executory by virtue of its being inherently executive in The context drawn in the analysis above has been termed by Justice Arturo D. Brion's
nature.160 As Justice Antonio T. Carpio previously wrote,161 Dissenting Opinion as the beginning of a "patent misconception." 165 His dissent argues that
this approach taken in analyzing the President's role as executor of the laws is preceded by
[i]f the rules are issued by the President in implementation or execution of self-executory the duty to preserve and defend the Constitution, which was allegedly overlooked.166
constitutional powers vested in the President, the rule-making power of the President is not
a delegated legislative power. The most important self-executory constitutional power of the In arguing against the approach, however, the dissent grossly failed to appreciate the
President is the President's constitutional duty and mandate to "ensure that the laws be nuances of the analysis, if read holistically and in context. The concept that the President
faithfully executed." The rule is that the President can execute the law without any cannot function with crippled hands and therefore can disregard the need for Senate
delegation of power from the legislature. concurrence in treaties167 was never expressed or implied. Rather, the appropriate reading of
the preceding analysis shows that the point being elucidated is the reality that the
The import of this characteristic is that the manner of the President's execution of the law, President's duty to execute the laws and protect the Philippines is inextricably interwoven
even if not expressly granted by the law, is justified by necessity and limited only by law, with his foreign affairs powers, such that he must resolve issues imbued with both concerns
since the President must "take necessary and proper steps to carry into execution the to the full extent of his powers, subject only to the limits supplied by law. In other words,
law."162 Justice George Malcolm states this principle in a grand manner:163 apart from an expressly mandated limit, or an implied limit by virtue of incompatibility, the
manner of execution by the President must be given utmost deference. This approach is not
different from that taken by the Court in situations with fairly similar contexts.
The executive should be clothed with sufficient power to administer efficiently the affairs of
state. He should have complete control of the instrumentalities through whom his
responsibility is discharged. It is still true, as said by Hamilton, that "A feeble executive Thus, the analysis portrayed by the dissent does not give the President authority to bypass
implies a feeble execution of the government. A feeble execution is but another phrase for a constitutional safeguards and limits. In fact, it specifies what these limitations are, how these
bad execution; and a government ill executed, whatever it may be in theory, must be in limitations are triggered, how these limitations function, and what can be done within the
practice a bad government." The mistakes of State governments need not be repeated here. sphere of constitutional duties and limitations of the President.

Every other consideration to one side, this remains certain - The Congress of the United Justice Brion's dissent likewise misinterprets the analysis proffered when it claims that the
States clearly intended that the Governor-General's power should be commensurate with his foreign relations power of the President should not be interpreted in isolation.168 The
responsibility. The Congress never intended that the Governor-General should be saddled analysis itself demonstrates how the foreign affairs function, while mostly the President's, is
with the responsibility of administering the government and of executing the laws but shorn shared in several instances, namely in Section 2 of Article II on the conduct of war; Sections
of the power to do so. The interests of the Philippines will be best served by strict adherence 20 and 21 of Article VII on foreign loans, treaties, and international agreements; Sections 4(2)
to the basic principles of constitutional government. and 5(2)(a) of Article VIII on the judicial review of executive acts; Sections 4 and 25 of Article
XVIII on treaties and international agreements entered into prior to the Constitution and on
the presence of foreign military troops, bases, or facilities.
In light of this constitutional duty, it is the President's prerogative to do whatever is legal and
necessary for Philippine defense interests. It is no coincidence that the constitutional
provision on the faithful execution clause was followed by that on the President's In fact, the analysis devotes a whole subheading to the relationship between the two major
commander-in-chief powers,164 which are specifically granted during extraordinary events of presidential functions and the role of the Senate in it.
lawless violence, invasion, or rebellion. And this duty of defending the country is unceasing,
even in times when there is no state of lawlesss violence, invasion, or rebellion. At such This approach of giving utmost deference to presidential initiatives in respect of foreign
times, the President has full powers to ensure the faithful execution of the laws. affairs is not novel to the Court. The President's act of treating EDCA as an executive
agreement is not the principal power being analyzed as the Dissenting Opinion seems to
suggest. Rather, the preliminary analysis is in reference to the expansive power of foreign
366

affairs. We have long treated this power as something the Courts must not unduly restrict. As relations, forfeiture of confidence, national embarrassment and a plethora of
we stated recently in Vinuya v. Romulo: other problems with equally undesirable consequences.169 (Emphases supplied)

To be sure, not all cases implicating foreign relations present political questions, and courts Understandably, this Court must view the instant case with the same perspective and
certainly possess the authority to construe or invalidate treaties and executive agreements. understanding, knowing full well the constitutional and legal repercussions of any judicial
However, the question whether the Philippine government should espouse claims of its overreach.
nationals against a foreign government is a foreign relations matter, the authority for which
is demonstrably committed by our Constitution not to the courts but to the political 2. The plain meaning of the Constitution prohibits the entry of foreign military bases,
branches. In this case, the Executive Department has already decided that it is to the best troops or facilities, except by way of a treaty concurred in by the Senate - a clear limitation
interest of the country to waive all claims of its nationals for reparations against Japan in the on the President's dual role as defender of the State and as sole authority in foreign
Treaty of Peace of 1951. The wisdom of such decision is not for the courts to question. relations.
Neither could petitioners herein assail the said determination by the Executive Department
via the instant petition for certiorari.
Despite the President's roles as defender of the State and sole authority in foreign relations,
the 1987 Constitution expressly limits his ability in instances when it involves the entry of
In the seminal case of US v. Curtiss-Wright Export Corp., the US Supreme Court held that foreign military bases, troops or facilities. The initial limitation is found in Section 21 of the
"[t]he President is the sole organ of the nation in its external relations, and its sole provisions on the Executive Department: "No treaty or international agreement shall be valid
representative with foreign relations." and effective unless concurred in by at least two-thirds of all the Members of the Senate."
The specific limitation is given by Section 25 of the Transitory Provisions, the full text of
It is quite apparent that if, in the maintenance of our international relations, which reads as follows:
embarrassment - perhaps serious embarrassment - is to be avoided and success for
our aims achieved, congressional legislation which is to be made effective through SECTION 25. After the expiration in 1991 of the Agreement between the Republic of the
negotiation and inquiry within the international field must often accord to the Philippines and the United States of America concerning Military Bases, foreign military
President a degree of discretion and freedom from statutory restriction which bases, troops, or facilities shall not be allowed in the Philippines except under a treaty duly
would not be admissible where domestic affairs alone involved. Moreover, he, concurred in by the Senate and, when the Congress so requires, ratified by a majority of the
not Congress, has the better opportunity of knowing the conditions which prevail in votes cast by the people in a national referendum held for that purpose, and recognized as a
foreign countries, and especially is this true in time of war. He has his confidential treaty by the other contracting State.
sources of information. He has his agents in the form of diplomatic, consular and
other officials ....
It is quite plain that the Transitory Provisions of the 1987 Constitution intended to add to the
basic requirements of a treaty under Section 21 of Article VII. This means that both
This ruling has been incorporated in our jurisprudence through Bavan v. Executive provisions must be read as additional limitations to the President's overarching executive
Secretary and Pimentel v. Executive Secretary; its overreaching principle was, perhaps, best function in matters of defense and foreign relations.
articulated in (now Chief) Justice Puno's dissent in Secretary of Justice v. Lantion:
3. The President, however, may enter into an executive agreement on foreign military
. . . The conduct of foreign relations is full of complexities and consequences, bases, troops, or facilities, if (a) it is not the instrument that allows the presence of foreign
sometimes with life and death significance to the nation especially in times of war. military bases, troops, or facilities; or (b) it merely aims to implement an existing law or
It can only be entrusted to that department of government which can act on the treaty.
basis of the best available information and can decide with decisiveness .... It is also
the President who possesses the most comprehensive and the most confidential
Again we refer to Section 25, Article XVIII of the Constitution:
information about foreign countries for our diplomatic and consular officials
regularly brief him on meaningful events all over the world. He has also unlimited
access to ultra-sensitive military intelligence data. In fine, the presidential role in SECTION 25. After the expiration in 1991 of the Agreement between the Republic of the
foreign affairs is dominant and the President is traditionally accorded a wider Philippines and the United States of America concerning Military Bases, foreign military
degree of discretion in the conduct of foreign affairs. The regularity, nay, validity bases, troops, or facilities shall not be allowed in the Philippines except under a
of his actions are adjudged under less stringent standards, lest their judicial treaty duly concurred in by the Senate and, when the Congress so requires, ratified by a
repudiation lead to breach of an international obligation, rupture of state
367

majority of the votes cast by the people in a national referendum held for that purpose, and Third, to this Court, a plain textual reading of Article XIII, Section 25, inevitably leads to the
recognized as a treaty by the other contracting State. (Emphases supplied) conclusion that it applies only to a proposed agreement between our government and a
foreign government, whereby military bases, troops, or facilities of such foreign government
In view of this provision, petitioners argue170 that EDCA must be in the form of a "treaty" duly would be "allowed" or would "gain entry" Philippine territory.
concurred in by the Senate. They stress that the Constitution is unambigous in mandating the
transmission to the Senate of all international agreements concluded after the expiration of Note that the provision "shall not be allowed" is a negative injunction. This wording signifies
the MBA in 1991 - agreements that concern the presence of foreign military bases, troops, or that the President is not authorized by law to allow foreign military bases, troops, or facilities
facilities in the country. Accordingly, petitioners maintain that the Executive Department is to enter the Philippines, except under a treaty concurred in by the Senate. Hence, the
not given the choice to conclude agreements like EDCA in the form of an executive constitutionally restricted authority pertains to the entry of the bases, troops, or facilities,
agreement. and not to the activities to be done after entry.

This is also the view of the Senate, which, through a majority vote of 15 of its members - with Under the principles of constitutional construction, of paramount consideration is the plain
1 against and 2 abstaining - says in SR 105171 that EDCA must be submitted to the Senate in meaning of the language expressed in the Constitution, or the verba legis rule.175 It is
the form of a treaty for concurrence by at least two-thirds of all its members. presumed that the provisions have been carefully crafted in order to express the objective it
seeks to attain.176 It is incumbent upon the Court to refrain from going beyond the plain
The Senate cites two constitutional provisions (Article VI, Section 21 and Article XVIII, Section meaning of the words used in the Constitution. It is presumed that the framers and the
25) to support its position. Compared with the lone constitutional provision that the Office of people meant what they said when they said it, and that this understanding was reflected in
the Solicitor General (OSG) cites, which is Article XVIII, Section 4(2), which includes the the Constitution and understood by the people in the way it was meant to be understood
constitutionality of "executive agreement(s)" among the cases subject to the Supreme when the fundamental law was ordained and promulgated.177 As this Court has often said:
Court's power of judicial review, the Constitution clearly requires submission of EDCA to the
Senate. Two specific provisions versus one general provision means that the specific We look to the language of the document itself in our search for its meaning. We do not of
provisions prevail. The term "executive agreement" is "a term wandering alone in the course stop there, but that is where we begin. It is to be assumed that the words in which
Constitution, bereft of provenance and an unidentified constitutional mystery." constitutional provisions are couched express the objective sought to be attained. They are
to be given their ordinary meaning except where technical terms are employed in which
The author of SR 105, Senator Miriam Defensor Santiago, upon interpellation even added case the significance thus attached to them prevails. As the Constitution is not primarily a
that the MDT, which the Executive claims to be partly implemented through EDCA, is already lawyer's document, it being essential for the rule of law to obtain that it should ever be
obsolete. present in the people's consciousness, its language as much as possible should be
understood in the sense they have in common use. What it says according to the text of the
provision to be construed compels acceptance and negates the power of the courts to alter
There are two insurmountable obstacles to this Court's agreement with SR 105, as well as
it, based on the postulate that the framers and the people mean what they say. Thus, these
with the comment on interpellation made by Senator Santiago.
are the cases where the need for construction is reduced to a minimum.178(Emphases
supplied)
First, the concept of "executive agreement" is so well-entrenched in this Court's
pronouncements on the powers of the President. When the Court validated the concept of
It is only in those instances in which the constitutional provision is unclear, ambiguous, or
"executive agreement," it did so with full knowledge of the Senate's role in concurring in
silent that further construction must be done to elicit its meaning.179 In Ang Bagong Bayani-
treaties. It was aware of the problematique of distinguishing when an international
OFW v. Commission on Elections,180 we reiterated this guiding principle:
agreement needed Senate concurrence for validity, and when it did not; and the Court
continued to validate the existence of "executive agreements" even after the 1987
Constitution.172 This follows a long line of similar decisions upholding the power of the it [is] safer to construe the Constitution from what appears upon its face. The proper
President to enter into an executive agreement.173 interpretation therefore depends more on how it was understood by the people adopting it
than in the framers' understanding thereof. (Emphases supplied)
Second, the MDT has not been rendered obsolescent, considering that as late as 2009,174 this
Court continued to recognize its validity. The effect of this statement is surprisingly profound, for, if taken literally, the phrase "shall
not be allowed in the Philippines" plainly refers to the entry of bases, troops, or facilities in
the country. The Oxford English Dictionary defines the word "allow" as a transitive verb that
means "to permit, enable"; "to give consent to the occurrence of or relax restraint on (an
368

action, event, or activity)"; "to consent to the presence or attendance of (a person)"; and, The aforecited Dissenting Opinion of Justice Brion disagrees with the ponencia's application
when with an adverbial of place, "to permit (a person or animal) to go, come, or be in, out, of verba legis construction to the words of Article XVIII, Section 25.187 It claims that the
near, etc."181 Black's Law Dictionary defines the term as one that means "[t]o grant, approve, provision is "neither plain, nor that simple."188 To buttress its disagreement, the dissent
or permit."182 states that the provision refers to a historical incident, which is the expiration of the 1947
MBA.189 Accordingly, this position requires questioning the circumstances that led to the
The verb "allow" is followed by the word "in," which is a preposition used to indicate "place historical event, and the meaning of the terms under Article XVIII, Section 25.
or position in space or anything having material extension: Within the limits or bounds of,
within (any place or thing)."183 That something is the Philippines, which is the noun that This objection is quite strange. The construction technique of verba legis is not inapplicable
follows. just because a provision has a specific historical context. In fact, every provision of the
Constitution has a specific historical context. The purpose of constitutional and statutory
It is evident that the constitutional restriction refers solely to the initial entry of the foreign construction is to set tiers of interpretation to guide the Court as to how a particular
military bases, troops, or facilities. Once entry is authorized, the subsequent acts are provision functions. Verba legis is of paramount consideration, but it is not the only
thereafter subject only to the limitations provided by the rest of the Constitution and consideration. As this Court has often said:
Philippine law, and not to the Section 25 requirement of validity through a treaty.
We look to the language of the document itself in our search for its meaning. We do not of
The VFA has already allowed the entry of troops in the Philippines. This Court stated in Lim v. course stop there, but that is where we begin. It is to be assumed that the words in which
Executive Secretary: constitutional provisions are couched express the objective sought to be attained. They are
to be given their ordinary meaning except where technical terms are employed in which
case the significance thus attached to them prevails. As the Constitution is not primarily a
After studied reflection, it appeared farfetched that the ambiguity surrounding the meaning
lawyer's document, it being essential for the rule of law to obtain that it should ever be
of the word "activities" arose from accident. In our view, it was deliberately made that way
present in the people's consciousness, its language as much as possible should be
to give both parties a certain leeway in negotiation. In this manner, visiting US forces may
understood in the sense they have in common use. What it says according to the text of the
sojourn in Philippine territory for purposes other than military. As conceived, the joint
provision to be construed compels acceptance and negates the power of the courts to alter
exercises may include training on new techniques of patrol and surveillance to protect the
it, based on the postulate that the framers and the people mean what they say. Thus, these
nation's marine resources, sea search-and-rescue operations to assist vessels in distress,
are the cases where the need for construction is reduced to a minimum.190(Emphases
disaster relief operations, civic action projects such as the building of school houses, medical
supplied)
and humanitarian missions, and the like.

As applied, verba legis aids in construing the ordinary meaning of terms. In this case, the
Under these auspices, the VFA gives legitimacy to the current Balikatan exercises. It is only
phrase being construed is "shall not be allowed in the Philippines" and not the preceding one
logical to assume that "Balikatan 02-1," a "mutual anti- terrorism advising, assisting and
referring to "the expiration in 1991 of the Agreement between the Republic of the
training exercise," falls under the umbrella of sanctioned or allowable activities in the context
Philippines and the United States of America concerning Military Bases, foreign military
of the agreement. Both the history and intent of the Mutual Defense Treaty and the VFA
bases, troops, or facilities." It is explicit in the wording of the provision itself that any
support the conclusion that combat-related activities -as opposed to combat itself-such as
interpretation goes beyond the text itself and into the discussion of the framers, the context
the one subject of the instant petition, are indeed authorized.184 (Emphasis supplied)
of the Constitutional Commission's time of drafting, and the history of the 1947 MBA.
Without reference to these factors, a reader would not understand those terms. However,
Moreover, the Court indicated that the Constitution continues to govern the conduct of for the phrase "shall not be allowed in the Philippines," there is no need for such reference.
foreign military troops in the Philippines,185 readily implying the legality of their initial entry The law is clear. No less than the Senate understood this when it ratified the VFA.
into the country.
4. The President may generally enter into executive agreements subject to limitations
The OSG emphasizes that EDCA can be in the form of an executive agreement, since it merely defined by the Constitution and may be in furtherance of a treaty already concurred in by
involves "adjustments in detail" in the implementation of the MDT and the VFA. 186 It points the Senate.
out that there are existing treaties between the Philippines and the U.S. that have already
been concurred in by the Philippine Senate and have thereby met the requirements of the
We discuss in this section why the President can enter into executive agreements.
Constitution under Section 25. Because of the status of these prior agreements, respondent
emphasizes that EDCA need not be transmitted to the Senate.
369

It would be helpful to put into context the contested language found in Article XVIII, Section In Commissioner of Customs v. Eastern Sea Trading, executive agreements are defined as
25. Its more exacting requirement was introduced because of the previous experience of the "international agreements embodying adjustments of detail carrying out well-established
country when its representatives felt compelled to consent to the old MBA.191 They felt national policies and traditions and those involving arrangements of a more or less
constrained to agree to the MBA in fulfilment of one of the major conditions for the country temporary nature."204 In Bayan Muna v. Romulo, this Court further clarified that executive
to gain independence from the U.S.192 As a result of that experience, a second layer of agreements can cover a wide array of subjects that have various scopes and
consent for agreements that allow military bases, troops and facilities in the country is now purposes.205 They are no longer limited to the traditional subjects that are usually covered by
articulated in Article XVIII of our present Constitution. executive agreements as identified in Eastern Sea Trading. The Court thoroughly discussed
this matter in the following manner:
This second layer of consent, however, cannot be interpreted in such a way that we
completely ignore the intent of our constitutional framers when they provided for that The categorization of subject matters that may be covered by international
additional layer, nor the vigorous statements of this Court that affirm the continued agreementsmentioned in Eastern Sea Trading is not cast in stone. x x x.
existence of that class of international agreements called "executive agreements."
As may be noted, almost half a century has elapsed since the Court rendered its decision
The power of the President to enter into binding executive agreements without Senate in Eastern Sea Trading. Since then, the conduct of foreign affairs has become more complex
concurrence is already well-established in this jurisdiction.193 That power has been alluded to and the domain of international law wider, as to include such subjects as human rights, the
in our present and past Constitutions,194 in various statutes,195 in Supreme Court environment, and the sea. In fact, in the US alone, the executive agreements executed by its
decisions,196 and during the deliberations of the Constitutional Commission.197 They cover a President from 1980 to 2000 covered subjects such as defense, trade, scientific cooperation,
wide array of subjects with varying scopes and purposes,198 including those that involve the aviation, atomic energy, environmental cooperation, peace corps, arms limitation, and
presence of foreign military forces in the country.199 nuclear safety, among others. Surely, the enumeration in Eastern Sea Trading cannot
circumscribe the option of each state on the matter of which the international agreement
As the sole organ of our foreign relations200 and the constitutionally assigned chief architect format would be convenient to serve its best interest. As Francis Sayre said in his work
of our foreign policy,201the President is vested with the exclusive power to conduct and referred to earlier:
manage the country's interface with other states and governments. Being the principal
representative of the Philippines, the Chief Executive speaks and listens for the nation; . . . It would be useless to undertake to discuss here the large variety of executive
initiates, maintains, and develops diplomatic relations with other states and governments; agreements as such concluded from time to time. Hundreds of executive agreements, other
negotiates and enters into international agreements; promotes trade, investments, tourism than those entered into under the trade-agreement act, have been negotiated with foreign
and other economic relations; and settles international disputes with other states. 202 governments. . . . They cover such subjects as the inspection of vessels, navigation dues,
income tax on shipping profits, the admission of civil air craft, custom matters and
As previously discussed, this constitutional mandate emanates from the inherent power of commercial relations generally, international claims, postal matters, the registration of
the President to enter into agreements with other states, including the prerogative to trademarks and copyrights, etc .... (Emphases Supplied)
conclude binding executive agreements that do not require further Senate concurrence. The
existence of this presidential power203 is so well-entrenched that Section 5(2)(a), Article VIII One of the distinguishing features of executive agreements is that their validity and
of the Constitution, even provides for a check on its exercise. As expressed below, executive effectivity are not affected by a lack of Senate concurrence. 206 This distinctive feature was
agreements are among those official governmental acts that can be the subject of this recognized as early as in Eastern Sea Trading (1961), viz:
Court's power of judicial review:
Treaties are formal documents which require ratification with the approval of two-thirds of
(2) Review, revise, reverse, modify, or affirm on appeal or certiorari, as the law or the Senate. Executive agreements become binding through executive action without the
the Rules of Court may provide, final judgments and orders of lower courts in: need of a vote by the Senate or by Congress.

(a) All cases in which the constitutionality or [T]he right of the Executive to enter into binding agreements without the necessity of
validity of any treaty, international or executive agreement, law, subsequent Congressional approval has been confirmed by long usage. From the earliest
presidential decree, proclamation, order, instruction, ordinance, or days of our history we have entered into executive agreements covering such subjects as
regulation is in question. (Emphases supplied) commercial and consular relations, most-favored-nation rights, patent rights, trademark and
copyright protection, postal and navigation arrangements and the settlement of
370

claims. The validity of these has never been seriously questioned by our courts. (Emphases MS. AQUINO: It is my humble submission that we should provide, unless the Committee
Supplied) explains to us otherwise, an explicit proviso which would except executive agreements from
the requirement of concurrence of two-thirds of the Members of the Senate. Unless I am
That notion was carried over to the present Constitution. In fact, the framers specifically enlightened by the Committee I propose that tentatively, the sentence should read. "No
deliberated on whether the general term "international agreement" included executive treaty or international agreement EXCEPT EXECUTIVE AGREEMENTS shall be valid and
agreements, and whether it was necessary to include an express proviso that would exclude effective."
executive agreements from the requirement of Senate concurrence. After noted
constitutionalist Fr. Joaquin Bernas quoted the Court's ruling in Eastern Sea Trading, the FR. BERNAS: I wonder if a quotation from the Supreme Court decision [in Eastern Sea
Constitutional Commission members ultimately decided that the term "international Trading] might help clarify this:
agreements" as contemplated in Section 21, Article VII, does not include executive
agreements, and that a proviso is no longer needed. Their discussion is reproduced below:207 The right of the executive to enter into binding agreements without the necessity of
subsequent Congressional approval has been confirmed by long usage. From the earliest
MS. AQUINO: Madam President, first I would like a clarification from the Committee. We days of our history, we have entered into executive agreements covering such subjects as
have retained the words "international agreement" which I think is the correct judgment on commercial and consular relations, most favored nation rights, patent rights, trademark and
the matter because an international agreement is different from a treaty. A treaty is a copyright protection, postal and navigation arrangements and the settlement of claims. The
contract between parties which is in the nature of international agreement and also a validity of this has never been seriously questioned by our Courts.
municipal law in the sense that the people are bound. So there is a conceptual difference.
However, I would like to be clarified if the international agreements include executive Agreements with respect to the registration of trademarks have been concluded by the
agreements. executive of various countries under the Act of Congress of March 3, 1881 (21 Stat. 502) . .
. International agreements involving political issues or changes of national policy and
MR. CONCEPCION: That depends upon the parties. All parties to these international those involving international agreements of a permanent character usually take the form
negotiations stipulate the conditions which are necessary for the agreement or whatever it of treaties. But international agreements embodying adjustments of detail, carrying out well
may be to become valid or effective as regards the parties. established national policies and traditions and those involving arrangements of a more or
less temporary nature usually take the form of executive agreements.
MS. AQUINO: Would that depend on the parties or would that depend on the nature of the
executive agreement? According to common usage, there are two types of executive MR. ROMULO: Is the Commissioner, therefore, excluding the executive agreements?
agreement: one is purely proceeding from an executive act which affects external relations
independent of the legislative and the other is an executive act in pursuance of legislative FR. BERNAS: What we are referring to, therefore, when we say international agreements
authorization. The first kind might take the form of just conventions or exchanges of notes which need concurrence by at least two-thirds are those which are permanent in nature.
or protocol while the other, which would be pursuant to the legislative authorization, may
be in the nature of commercial agreements.
MS. AQUINO: And it may include commercial agreements which are executive agreements
essentially but which are proceeding from the authorization of Congress. If that is our
MR. CONCEPCION: Executive agreements are generally made to implement a treaty already understanding, then I am willing to withdraw that amendment.
enforced or to determine the details for the implementation of the treaty. We are speaking
of executive agreements, not international agreements.
FR. BERNAS: If it is with prior authorization of Congress, then it does not need subsequent
concurrence by Congress.
MS. AQUINO: I am in full agreement with that, except that it does not cover the first kind of
executive agreement which is just protocol or an exchange of notes and this would be in the
MS. AQUINO: In that case, I am withdrawing my amendment.
nature of reinforcement of claims of a citizen against a country, for example.

MR. TINGSON: Madam President.


MR. CONCEPCION: The Commissioner is free to require ratification for validity insofar as the
Philippines is concerned.
THE PRESIDENT: Is Commissioner Aquino satisfied?
371

MS. AQUINO: Yes. There is already an agreement among us on the definition of "executive Executive agreements may dispense with the requirement of Senate concurrence because of
agreements" and that would make unnecessary any explicit proviso on the matter. the legal mandate with which they are concluded. As culled from the afore-quoted
deliberations of the Constitutional Commission, past Supreme Court Decisions, and works of
MR. GUINGONA: I am not clear as to the meaning of "executive agreements" because I heard noted scholars,208 executive agreements merely involve arrangements on the
that these executive agreements must rely on treaties. In other words, there must first be implementation of existing policies, rules, laws, or agreements. They are concluded (1) to
treaties. adjust the details of a treaty;209 (2) pursuant to or upon confirmation by an act of the
Legislature;210 or (3) in the exercise of the President's independent powers under the
Constitution.211 The raison d'etre of executive agreements hinges on prior constitutional or
MR. CONCEPCION: No, I was speaking about the common use, as executive agreements
legislative authorizations.
being the implementation of treaties, details of which do not affect the sovereignty of the
State.
The special nature of an executive agreement is not just a domestic variation in international
agreements. International practice has accepted the use of various forms and designations of
MR. GUINGONA: But what about the matter of permanence, Madam President? Would 99
international agreements, ranging from the traditional notion of a treaty - which connotes a
years be considered permanent? What would be the measure of permanency? I do not
formal, solemn instrument - to engagements concluded in modem, simplified forms that no
conceive of a treaty that is going to be forever, so there must be some kind of a time limit.
longer necessitate ratification.212 An international agreement may take different forms:
treaty, act, protocol, agreement, concordat, compromis d'arbitrage, convention, covenant,
MR. CONCEPCION: I suppose the Commissioner's question is whether this type of agreement declaration, exchange of notes, statute, pact, charter, agreed minute, memorandum of
should be included in a provision of the Constitution requiring the concurrence of Congress. agreement, modus vivendi, or some other form.213 Consequently, under international law,
the distinction between a treaty and an international agreement or even an executive
MR. GUINGONA: It depends on the concept of the executive agreement of which I am not agreement is irrelevant for purposes of determining international rights and obligations.
clear. If the executive agreement partakes of the nature of a treaty, then it should also be
included. However, this principle does not mean that the domestic law distinguishing treaties,
international agreements, and executive agreements is relegated to a mere variation in form,
MR. CONCEPCION: Whether it partakes or not of the nature of a treaty, it is within the power or that the constitutional requirement of Senate concurrence is demoted to an optional
of the Constitutional Commission to require that. constitutional directive. There remain two very important features that
distinguish treaties from executive agreements and translate them into terms of art in the
MR. GUINGONA: Yes. That is why I am trying to clarify whether the words "international domestic setting.
agreements" would include executive agreements.
First, executive agreements must remain traceable to an express or implied authorization
MR. CONCEPCION: No, not necessarily; generally no. under the Constitution, statutes, or treaties. The absence of these precedents puts the
validity and effectivity of executive agreements under serious question for the main function
of the Executive is to enforce the Constitution and the laws enacted by the Legislature, not to
MR. ROMULO: I wish to be recognized first. I have only one question. Do we take it, defeat or interfere in the performance of these rules.214 In turn, executive agreements
therefore, that as far as the Committee is concerned, the term "international agreements" cannot create new international obligations that are not expressly allowed or reasonably
does not include the term "executive agreements" as read by the Commissioner in that implied in the law they purport to implement.
text?
Second, treaties are, by their very nature, considered superior to executive agreements.
FR. BERNAS: Yes. Treaties are products of the acts of the Executive and the Senate215 unlike executive
agreements, which are solely executive actions.216Because of legislative participation through
The inapplicability to executive agreements of the requirements under Section 21 was again the Senate, a treaty is regarded as being on the same level as a statute.217 If there is an
recognized in Bayan v. Zamora and in Bayan Muna v. Romulo. These cases, both decided irreconcilable conflict, a later law or treaty takes precedence over one that is prior.218 An
under the aegis of the present Constitution, quoted Eastern Sea Trading in reiterating that executive agreement is treated differently. Executive agreements that are inconsistent with
executive agreements are valid and binding even without the concurrence of the Senate. either a law or a treaty are considered ineffective.219 Both types of international agreement
are nevertheless subject to the supremacy of the Constitution.220
372

This rule does not imply, though, that the President is given carte blanche to exercise this 1. Treaties, international agreements, and executive agreements are all
discretion. Although the Chief Executive wields the exclusive authority to conduct our foreign constitutional manifestations of the conduct of foreign affairs with their distinct
relations, this power must still be exercised within the context and the parameters set by the legal characteristics.
Constitution, as well as by existing domestic and international laws. There are constitutional
provisions that restrict or limit the President's prerogative in concluding international a. Treaties are formal contracts between the Philippines and other
agreements, such as those that involve the following: States-parties, which are in the nature of international agreements, and
also of municipal laws in the sense of their binding nature.226
a. The policy of freedom from nuclear weapons within Philippine territory221
b. International agreements are similar instruments, the provisions of
b. The fixing of tariff rates, import and export quotas, tonnage and wharfage dues, which may require the ratification of a designated number of parties
and other duties or imposts, which must be pursuant to the authority granted by thereto. These agreements involving political issues or changes in
Congress222 national policy, as well as those involving international agreements of a
permanent character, usually take the form of treaties. They may also
c. The grant of any tax exemption, which must be pursuant to a law concurred in by include commercial agreements, which are executive agreements
a majority of all the Members of Congress223 essentially, but which proceed from previous authorization by Congress,
thus dispensing with the requirement of concurrence by the Senate.227
d. The contracting or guaranteeing, on behalf of the Philippines, of foreign loans
that must be previously concurred in by the Monetary Board224 c. Executive agreements are generally intended to implement a treaty
already enforced or to determine the details of the implementation
thereof that do not affect the sovereignty of the State.228
e. The authorization of the presence of foreign military bases, troops, or facilities in
the country must be in the form of a treaty duly concurred in by the Senate. 225
2. Treaties and international agreements that cannot be mere executive
agreements must, by constitutional decree, be concurred in by at least two-thirds
f. For agreements that do not fall under paragraph 5, the concurrence of the
of the Senate.
Senate is required, should the form of the government chosen be a treaty.

3. However, an agreement - the subject of which is the entry of foreign military


5. The President had the choice to enter into EDCA by way of an executive agreement or a
troops, bases, or facilities - is particularly restricted. The requirements are that it be
treaty.
in the form of a treaty concurred in by the Senate; that when Congress so requires,
it be ratified by a majority of the votes cast by the people in a national referendum
No court can tell the President to desist from choosing an executive agreement over a treaty held for that purpose; and that it be recognized as a treaty by the other contracting
to embody an international agreement, unless the case falls squarely within Article VIII, State.
Section 25.
4. Thus, executive agreements can continue to exist as a species of international
As can be gleaned from the debates among the members of the Constitutional Commission, agreements.
they were aware that legally binding international agreements were being entered into by
countries in forms other than a treaty. At the same time, it is clear that they were also keen
That is why our Court has ruled the way it has in several cases.
to preserve the concept of "executive agreements" and the right of the President to enter
into such agreements.
In Bayan Muna v. Romulo, we ruled that the President acted within the scope of her
constitutional authority and discretion when she chose to enter into the RP-U.S. Non-
What we can glean from the discussions of the Constitutional Commissioners is that they
Surrender Agreement in the form of an executive agreement, instead of a treaty, and in
understood the following realities:
ratifying the agreement without Senate concurrence. The Court en banc discussed this
intrinsic presidential prerogative as follows:
373

Petitioner parlays the notion that the Agreement is of dubious validity, partaking as it does of Accordingly, in the exercise of its power of judicial review, the Court does not look into
the nature of a treaty; hence, it must be duly concurred in by the Senate. x x x x. Pressing its whether an international agreement should be in the form of a treaty or an executive
point, petitioner submits that the subject of the Agreement does not fall under any of the agreement, save in cases in which the Constitution or a statute requires otherwise. Rather, in
subject-categories that xx x may be covered by an executive agreement, such as view of the vast constitutional powers and prerogatives granted to the President in the field
commercial/consular relations, most-favored nation rights, patent rights, trademark and of foreign affairs, the task of the Court is to determine whether the international agreement
copyright protection, postal and navigation arrangements and settlement of claims. is consistent with the applicable limitations.

The categorization of subject matters that may be covered by international agreements 6. Executive agreements may cover the matter of foreign military forces if it merely
mentioned in Eastern Sea Trading is not cast in stone. There are no hard and fast rules on involves detail adjustments.
the propriety of entering, on a given subject, into a treaty or an executive agreement as an
instrument of international relations. The primary consideration in the choice of the form of The practice of resorting to executive agreements in adjusting the details of a law or a treaty
agreement is the parties' intent and desire to craft an international agreement in the form that already deals with the presence of foreign military forces is not at all unusual in this
they so wish to further their respective interests. Verily, the matter of form takes a back jurisdiction. In fact, the Court has already implicitly acknowledged this practice in Lim v.
seat when it comes to effectiveness and binding effect of the enforcement of a treaty or an Executive Secretary.231 In that case, the Court was asked to scrutinize the constitutionality of
executive agreement, as the parties in either international agreement each labor under the Terms of Reference of the Balikatan 02-1 joint military exercises, which sought to
the pacta sunt servanda principle. implement the VFA. Concluded in the form of an executive agreement, the Terms of
Reference detailed the coverage of the term "activities" mentioned in the treaty and settled
But over and above the foregoing considerations is the fact that - save for the situation and the matters pertaining to the construction of temporary structures for the U.S. troops during
matters contemplated in Sec. 25, Art. XVIII of the Constitution - when a treaty is required, the the activities; the duration and location of the exercises; the number of participants; and the
Constitution does not classify any subject, like that involving political issues, to be in the extent of and limitations on the activities of the U.S. forces. The Court upheld the Terms of
form of, and ratified as, a treaty. What the Constitution merely prescribes is that treaties Reference as being consistent with the VFA. It no longer took issue with the fact that
need the concurrence of the Senate by a vote defined therein to complete the ratification the Balikatan Terms of Reference was not in the form of a treaty concurred in by the Senate,
process. even if it dealt with the regulation of the activities of foreign military forces on Philippine
territory.
x x x. As the President wields vast powers and influence, her conduct in the external affairs of
the nation is, as Bayan would put it, "executive altogether." The right of the President to In Nicolas v. Romulo,232 the Court again impliedly affirmed the use of an executive agreement
enter into or ratify binding executive agreements has been confirmed by long practice. in an attempt to adjust the details of a provision of the VFA. The Philippines and the U.S.
entered into the Romulo-Kenney Agreement, which undertook to clarify the detention of a
In thus agreeing to conclude the Agreement thru E/N BF0-028-03, then President Gloria U.S. Armed Forces member, whose case was pending appeal after his conviction by a trial
Macapagal-Arroyo, represented by the Secretary of Foreign Affairs, acted within the scope court for the crime of rape. In testing the validity of the latter agreement, the Court precisely
of the authority and discretion vested in her by the Constitution. At the end of the day, the alluded to one of the inherent limitations of an executive agreement: it cannot go beyond
President - by ratifying, thru her deputies, the non-surrender agreement - did nothing more the terms of the treaty it purports to implement. It was eventually ruled that the Romulo-
than discharge a constitutional duty and exercise a prerogative that pertains to her Kenney Agreement was "not in accord" with the VFA, since the former was squarely
office. (Emphases supplied) inconsistent with a provision in the treaty requiring that the detention be "by Philippine
authorities." Consequently, the Court ordered the Secretary of Foreign Affairs to comply with
the VFA and "forthwith negotiate with the United States representatives for the appropriate
Indeed, in the field of external affairs, the President must be given a larger measure of
agreement on detention facilities under Philippine authorities as provided in Art. V, Sec. 10 of
authority and wider discretion, subject only to the least amount of checks and restrictions
the VFA. "233
under the Constitution.229 The rationale behind this power and discretion was recognized by
the Court in Vinuya v. Executive Secretary, cited earlier.230
Culling from the foregoing discussions, we reiterate the following pronouncements to guide
us in resolving the present controversy:
Section 9 of Executive Order No. 459, or the Guidelines in the Negotiation of International
Agreements and its Ratification, thus, correctly reflected the inherent powers of the
President when it stated that the DFA "shall determine whether an agreement is an 1. Section 25, Article XVIII of the Constitution, contains stringent requirements that
executive agreement or a treaty." must be fulfilled by the international agreement allowing the presence of foreign
374

military bases, troops, or facilities in the Philippines: (a) the agreement must be in by Philippine authorities. According to the Court, the parties "recognized the difference
the form of a treaty, and (b) it must be duly concurred in by the Senate. between custody during the trial and detention after conviction."241 Pursuant to Article V(6)
of the VFA, the custody of a U.S. military personnel resides with U.S. military authorities
2. If the agreement is not covered by the above situation, then the President may during trial. Once there is a finding of guilt, Article V(l0) requires that the confinement or
choose the form of the agreement (i.e., either an executive agreement or a treaty), detention be "by Philippine authorities."
provided that the agreement dealing with foreign military bases, troops, or
facilities is not the principal agreement that first allows their entry or presence in Justice Marvic M.V.F. Leonen's Dissenting Opinion posits that EDCA "substantially modifies or
the Philippines. amends the VFA"242and follows with an enumeration of the differences between EDCA and
the VFA. While these arguments will be rebutted more fully further on, an initial answer can
3. The executive agreement must not go beyond the parameters, limitations, and already be given to each of the concerns raised by his dissent.
standards set by the law and/or treaty that the former purports to implement; and
must not unduly expand the international obligation expressly mentioned or The first difference emphasized is that EDCA does not only regulate visits as the VFA does,
necessarily implied in the law or treaty. but allows temporary stationing on a rotational basis of U.S. military personnel and their
contractors in physical locations with permanent facilities and pre-positioned military
4. The executive agreement must be consistent with the Constitution, as well as materiel.
with existing laws and treaties.
This argument does not take into account that these permanent facilities, while built by U.S.
In light of the President's choice to enter into EDCA in the form of an executive agreement, forces, are to be owned by the Philippines once constructed. 243 Even the VFA allowed
respondents carry the burden of proving that it is a mere implementation of existing laws construction for the benefit of U.S. forces during their temporary visits.
and treaties concurred in by the Senate. EDCA must thus be carefully dissected to ascertain if
it remains within the legal parameters of a valid executive agreement. The second difference stated by the dissent is that EDCA allows the prepositioning of military
materiel, which can include various types of warships, fighter planes, bombers, and vessels,
7. EDCA is consistent with the content, purpose, and framework of the MDT and the VFA as well as land and amphibious vehicles and their corresponding ammunition.244

The starting point of our analysis is the rule that "an executive agreement xx x may not be However, the VFA clearly allows the same kind of equipment, vehicles, vessels, and aircraft
used to amend a treaty."234 In Lim v. Executive Secretary and in Nicolas v. Romulo, the Court to be brought into the country. Articles VII and VIII of the VFA contemplates that U.S.
approached the question of the validity of executive agreements by comparing them with equipment, materials, supplies, and other property are imported into or acquired in the
the general framework and the specific provisions of the treaties they seek to implement. Philippines by or on behalf of the U.S. Armed Forces; as are vehicles, vessels, and aircraft
operated by or for U.S. forces in connection with activities under the VFA. These provisions
likewise provide for the waiver of the specific duties, taxes, charges, and fees that
In Lim, the Terms of Reference of the joint military exercises was scrutinized by studying "the
correspond to these equipment.
framework of the treaty antecedents to which the Philippines bound itself,"235 i.e., the MDT
and the VFA. The Court proceeded to examine the extent of the term "activities" as
contemplated in Articles 1236 and II237 of the VFA. It later on found that the term "activities" The third difference adverted to by the Justice Leonen's dissent is that the VFA contemplates
was deliberately left undefined and ambiguous in order to permit "a wide scope of the entry of troops for training exercises, whereas EDCA allows the use of territory for
undertakings subject only to the approval of the Philippine government"238 and thereby allow launching military and paramilitary operations conducted in other states.245 The dissent of
the parties "a certain leeway in negotiation."239 The Court eventually ruled that the Terms of Justice Teresita J. Leonardo-De Castro also notes that VFA was intended for non-combat
Reference fell within the sanctioned or allowable activities, especially in the context of the activides only, whereas the entry and activities of U.S. forces into Agreed Locations were
VFA and the MDT. borne of military necessity or had a martial character, and were therefore not contemplated
by the VFA.246
The Court applied the same approach to Nicolas v. Romulo. It studied the provisions of the
VFA on custody and detention to ascertain the validity of the Romulo-Kenney This Court's jurisprudence however established in no uncertain terms that combat-related
Agreement.240 It eventually found that the two international agreements were not in accord, activities, as opposed to actual combat, were allowed under the MDT and VFA, viz:
since the Romulo-Kenney Agreement had stipulated that U.S. military personnel shall be
detained at the U.S. Embassy Compound and guarded by U.S. military personnel, instead of
375

Both the history and intent of the Mutual Defense Treaty and the VFA support the conclusion The OSG argues250 that EDCA merely details existing policies under the MDT and the VFA. It
that combat-related activities as opposed to combat itself such as the one subject of the explains that EDCA articulates the principle of defensive preparation embodied in Article II of
instant petition, are indeed authorized.247 the MDT; and seeks to enhance the defensive, strategic, and technological capabilities of
both parties pursuant to the objective of the treaty to strengthen those capabilities to
Hence, even if EDCA was borne of military necessity, it cannot be said to have strayed from prevent or resist a possible armed attack. Respondent also points out that EDCA simply
the intent of the VFA since EDCA's combat-related components are allowed under the treaty. implements Article I of the VFA, which already allows the entry of U.S. troops and personnel
into the country. Respondent stresses this Court's recognition in Lim v. Executive
Secretary that U.S. troops and personnel are authorized to conduct activities that promote
Moreover, both the VFA and EDCA are silent on what these activities actually are. Both the
the goal of maintaining and developing their defense capability.
VFA and EDCA deal with the presence of U.S. forces within the Philippines, but make no
mention of being platforms for activity beyond Philippine territory. While it may be that, as
applied, military operations under either the VFA or EDCA would be carried out in the future Petitioners contest251 the assertion that the provisions of EDCA merely implement the MDT.
the scope of judicial review does not cover potential breaches of discretion but only actual According to them, the treaty does not specifically authorize the entry of U.S. troops in the
occurrences or blatantly illegal provisions. Hence, we cannot invalidate EDCA on the basis of country in order to maintain and develop the individual and collective capacities of both the
the potentially abusive use of its provisions. Philippines and the U.S. to resist an armed attack. They emphasize that the treaty was
concluded at a time when there was as yet no specific constitutional prohibition on the
presence of foreign military forces in the country.
The fourth difference is that EDCA supposedly introduces a new concept not contemplated in
the VFA or the MDT: Agreed Locations, Contractors, Pre-positioning, and Operational
Control.248 Petitioners also challenge the argument that EDCA simply implements the VFA. They assert
that the agreement covers only short-term or temporary visits of U.S. troops "from time to
time" for the specific purpose of combined military exercises with their Filipino counterparts.
As previously mentioned, these points shall be addressed fully and individually in the latter
They stress that, in contrast, U.S. troops are allowed under EDCA to perform
analysis of EDCA's provisions. However, it must already be clarified that the terms and details
activities beyond combined military exercises, such as those enumerated in Articles 111(1)
used by an implementing agreement need not be found in the mother treaty. They must be
and IV(4) thereof. Furthermore, there is some degree of permanence in the presence of U.S.
sourced from the authority derived from the treaty, but are not necessarily expressed word-
troops in the country, since the effectivity of EDCA is continuous until terminated. They
for-word in the mother treaty. This concern shall be further elucidated in this Decision.
proceed to argue that while troops have a "rotational" presence, this scheme in fact fosters
their permanent presence.
The fifth difference highlighted by the Dissenting Opinion is that the VFA does not have
provisions that may be construed as a restriction on or modification of obligations found in
a. Admission of U.S. military and civilian personnel into Philippine territory is already allowed
existing statues, including the jurisdiction of courts, local autonomy, and taxation. Implied in
under the VFA
this argument is that EDCA contains such restrictions or modifications.249

We shall first deal with the recognition under EDCA of the presence in the country of three
This last argument cannot be accepted in view of the clear provisions of EDCA. Both the VFA
distinct classes of individuals who will be conducting different types of activities within the
and EDCA ensure Philippine jurisdiction in all instances contemplated by both agreements,
Agreed Locations: (1) U.S. military personnel; (2) U.S. civilian personnel; and (3) U.S.
with the exception of those outlined by the VFA in Articles III-VI. In the VFA, taxes are clearly
contractors. The agreement refers to them as follows:
waived whereas in EDCA, taxes are assumed by the government as will be discussed later on.
This fact does not, therefore, produce a diminution of jurisdiction on the part of the
Philippines, but rather a recognition of sovereignty and the rights that attend it, some of "United States personnel" means United States military and civilian personnel temporarily
which may be waived as in the cases under Articles III-VI of the VFA. in the territory of the Philippines in connection with activities approved by the
Philippines, as those terms are defined in the VFA.252
Taking off from these concerns, the provisions of EDCA must be compared with those of the
MDT and the VFA, which are the two treaties from which EDCA allegedly draws its validity. "United States forces" means the entity comprising United States personnel and
all property, equipment, and materiel of the United States Armed Forces present in the
territory of the Philippines.253
"Authorized presence" under the VFA versus "authorized activities" under EDCA: (1) U.S.
personnel and (2) U.S. contractors
376

"United States contractors" means companies and firms, and their employees, under 3. The following documents only, which shall be required in respect of United
contract or subcontract to or on behalf of the United States Department of Defense. United States military personnel who enter the Philippines; xx xx.
States contractors are not included as part of the definition of United States personnel in
this Agreement, including within the context of the VFA.254 4. United States civilian personnel shall be exempt from visa requirements but
shall present, upon demand, valid passports upon entry and departure of the
United States forces may contract for any materiel, supplies, equipment, and Philippines. (Emphases Supplied)
services (including construction) to be furnished or undertaken in the territory of the
Philippines without restriction as to choice of contractor, supplier, or person who By virtue of Articles I and III of the VFA, the Philippines already allows U.S. military and
provides such materiel, supplies, equipment, or services. Such contracts shall be solicited, civilian personnel to be "temporarily in the Philippines," so long as their presence is "in
awarded, and administered in accordance with the laws and regulations of the United connection with activities approved by the Philippine Government." The Philippines, through
States.255 (Emphases Supplied) Article III, even guarantees that it shall facilitate the admission of U.S. personnel into the
country and grant exemptions from passport and visa regulations. The VFA does not even
A thorough evaluation of how EDCA is phrased clarities that the agreement does not deal limit their temporary presence to specific locations.
with the entry into the country of U.S. personnel and contractors per se. While Articles
I(l)(b)256 and II(4)257 speak of "the right to access and use" the Agreed Locations, their Based on the above provisions, the admission and presence of U.S. military and civilian
wordings indicate the presumption that these groups have already been allowed entry into personnel in Philippine territory are already allowed under the VFA, the treaty supposedly
Philippine territory, for which, unlike the VFA, EDCA has no specific provision. Instead, Article being implemented by EDCA. What EDCA has effectively done, in fact, is merely provide the
II of the latter simply alludes to the VFA in describing U.S. personnel, a term defined under mechanism to identify the locations in which U.S. personnel may perform allowed activities
Article I of the treaty as follows: pursuant to the VFA. As the implementing agreement, it regulates and limits the presence of
U.S. personnel in the country.
As used in this Agreement, "United States personnel" means United States military and
civilian personnel temporarily in the Philippines in connection with activities approved by b. EDCA does not provide the legal basis for admission of U.S. contractors into Philippine
the Philippine Government. Within this definition: territory; their entry must be sourced from extraneous Philippine statutes and regulations for
the admission of alien employees or business persons.
1. The term "military personnel" refers to military members of the United States
Army, Navy, Marine Corps, Air Force, and Coast Guard. Of the three aforementioned classes of individuals who will be conducting certain activities
within the Agreed Locations, we note that only U.S. contractors are not explicitly mentioned
2. The term "civilian personnel" refers to individuals who are neither nationals of in the VFA. This does not mean, though, that the recognition of their presence under EDCA
nor ordinarily resident in the Philippines and who are employed by the United is ipso facto an amendment of the treaty, and that there must be Senate concurrence before
States armed forces or who are accompanying the United States armed forces, they are allowed to enter the country.
such as employees of the American Red Cross and the United Services
Organization.258 Nowhere in EDCA are U.S. contractors guaranteed immediate admission into the Philippines.
Articles III and IV, in fact, merely grant them the right of access to, and the authority to
Article II of EDCA must then be read with Article III of the VFA, which provides for the entry conduct certain activities within the Agreed Locations. Since Article II(3) of EDCA specifically
accommodations to be accorded to U.S. military and civilian personnel: leaves out U.S. contractors from the coverage of the VFA, they shall not be granted the same
entry accommodations and privileges as those enjoyed by U.S. military and civilian personnel
1. The Government of the Philippines shall facilitate the admission of United under the VFA.
States personnel and their departure from the Philippines in connection with
activities covered by this agreement. Consequently, it is neither mandatory nor obligatory on the part of the Philippines to admit
U.S. contractors into the country.259 We emphasize that the admission of aliens into
2. United States military personnel shall be exempt from passport and visa Philippine territory is "a matter of pure permission and simple tolerance which creates no
regulations upon enteringand departing the Philippines. obligation on the part of the government to permit them to stay."260 Unlike U.S. personnel
who are accorded entry accommodations, U.S. contractors are subject to Philippine
immigration laws.261The latter must comply with our visa and passport regulations262 and
377

prove that they are not subject to exclusion under any provision of Philippine immigration Considering that cooperation between the United States and the Republic of the
laws.263 The President may also deny them entry pursuant to his absolute and unqualified Philippines promotes their common security interests;
power to prohibit or prevent the admission of aliens whose presence in the country would be
inimical to public interest.264 Article I - Definitions

In the same vein, the President may exercise the plenary power to expel or deport U.S. As used in this Agreement, "United States personnel" means United States military and
contractors265 as may be necessitated by national security, public safety, public health, public civilian personnel temporarily in the Philippines in connection with activities approved by
morals, and national interest.266 They may also be deported if they are found to be illegal or the Philippine Government. Within this definition: xx x
undesirable aliens pursuant to the Philippine Immigration Act267 and the Data Privacy
Act.268 In contrast, Article 111(5) of the VFA requires a request for removal from the
Article II - Respect for Law
Philippine government before a member of the U.S. personnel may be "dispos[ed] xx x
outside of the Philippines."
It is the duty of United States personnel to respect the laws of the Republic of the
Philippines and to abstain from any activity inconsistent with the spirit of this agreement,
c. Authorized activities of U.S. military and civilian personnel within Philippine territory are in
and, in particular, from any political activity in the Philippines. The Government of the United
furtherance of the MDT and the VFA
States shall take all measures within its authority to ensure that this is done.

We begin our analysis by quoting the relevant sections of the MDT and the VFA that pertain
Article VII - Importation and Exportation
to the activities in which U.S. military and civilian personnel may engage:

1. United States Government equipment, materials, supplies, and other property imported
MUTUAL DEFENSE TREATY
into or acquired in the Philippines by or on behalf of the United States armed forces in
connection with activities to which this agreement applies, shall be free of all Philippine
Article II duties, taxes and other similar charges. Title to such property shall remain with the United
States, which may remove such property from the Philippines at any time, free from export
In order more effectively to achieve the objective of this Treaty, the Parties separately duties, taxes, and other similar charges. x x x.
and jointly byself-help and mutual aid will maintain and develop their individual and
collective capacity to resist armed attack. Article VIII - Movement of Vessels and Aircraft

Article III 1. Aircraft operated by or for the United States armed forces may enter the Philippines
upon approval of the Government of the Philippines in accordance with procedures
The Parties, through their Foreign Ministers or their deputies, will consult together from stipulated in implementing arrangements.
time to time regarding the implementation of this Treaty and whenever in the opinion of
either of them the territorial integrity, political independence or security of either of the 2. Vessels operated by or for the United States armed forces may enter the Philippines upon
Parties is threatened by external armed attack in the Pacific. approval of the Government of the Philippines. The movement of vessels shall be in
accordance with international custom and practice governing such vessels, and such agreed
VISITING FORCES AGREEMENT implementing arrangements as necessary. x x x (Emphases Supplied)

Preamble Manifest in these provisions is the abundance of references to the creation of further
"implementing arrangements" including the identification of "activities [to be] approved by
Reaffirming their obligations under the Mutual Defense Treaty of August 30, 1951; the Philippine Government." To determine the parameters of these implementing
arrangements and activities, we referred to the content, purpose, and framework of the MDT
and the VFA.
Noting that from time to time elements of the United States armed forces may visit the
Republic of the Philippines;
378

By its very language, the MDT contemplates a situation in which both countries shall engage visit of U.S. personnel.276 In ruling that the Terms of Reference for the Balikatan Exercises in
in joint activities, so that they can maintain and develop their defense capabilities. The 2002 fell within the context of the treaty, this Court explained:
wording itself evidently invites a reasonable construction that the joint activities shall
involve joint military trainings, maneuvers, and exercises. Both the interpretation269 and the After studied reflection, it appeared farfetched that the ambiguity surrounding the meaning
subsequent practice270 of the parties show that the MDT independently allows joint military of the word "activities" arose from accident. In our view, it was deliberately made that way
exercises in the country. Lim v. Executive Secretary271 and Nicolas v. Romulo272 recognized to give both parties a certain leeway in negotiation. In this manner, visiting US forces may
that Balikatan exercises, which are activities that seek to enhance and develop the strategic sojourn in Philippine territory for purposes other than military. As conceived, the joint
and technological capabilities of the parties to resist an armed attack, "fall squarely under exercises may include training on new techniques of patrol and surveillance to protect the
the provisions of the RP-US MDT."273 In Lim, the Court especially noted that the Philippines nation's marine resources, sea search-and-rescue operations to assist vessels in distress,
and the U.S. continued to conduct joint military exercises even after the expiration of the disaster relief operations, civic action projects such as the building of school houses, medical
MBA and even before the conclusion of the VFA.274 These activities presumably related to the and humanitarian missions, and the like.
Status of Forces Agreement, in which the parties agreed on the status to be accorded to U.S.
military and civilian personnel while conducting activities in the Philippines in relation to the
Under these auspices, the VFA gives legitimacy to the current Balikatan exercises. It is only
MDT.275
logical to assume that "Balikatan 02-1," a "mutual anti-terrorism advising, assisting and
training exercise," falls under the umbrella of sanctioned or allowable activities in the
Further, it can be logically inferred from Article V of the MDT that these joint activities may context of the agreement. Both the history and intent of the Mutual Defense Treaty and the
be conducted on Philippine or on U.S. soil. The article expressly provides that the term armed VFA support the conclusion that combat-related activities - as opposed to combat itself- such
attack includes "an armed attack on the metropolitan territory of either of the Parties, or on as the one subject of the instant petition, are indeed authorized. (Emphases Supplied)
the island territories under its jurisdiction in the Pacific or on its armed forces, public
vessels or aircraft in the Pacific." Surely, in maintaining and developing our defense
The joint report of the Senate committees on foreign relations and on national defense and
capabilities, an assessment or training will need to be performed, separately and jointly by
security further explains the wide range and variety of activities contemplated in the VFA,
self-help and mutual aid, in the territories of the contracting parties. It is reasonable to
and how these activities shall be identified:277
conclude that the assessment of defense capabilities would entail understanding the terrain,
wind flow patterns, and other environmental factors unique to the Philippines.
These joint exercises envisioned in the VFA are not limited to combat-related activities; they
have a wide range and variety. They include exercises that will reinforce the AFP's ability
It would also be reasonable to conclude that a simulation of how to respond to attacks in
to acquire new techniques of patrol and surveillance to protect the country's maritime
vulnerable areas would be part of the training of the parties to maintain and develop their
resources; sea-search and rescue operations to assist ships in distress; and disaster-relief
capacity to resist an actual armed attack and to test and validate the defense plan of the
operations to aid the civilian victims of natural calamities, such as earthquakes, typhoons
Philippines. It is likewise reasonable to imagine that part of the training would involve an
and tidal waves.
analysis of the effect of the weapons that may be used and how to be prepared for the
eventuality. This Court recognizes that all of this may require training in the area where an
armed attack might be directed at the Philippine territory. Joint activities under the VFA will include combat maneuvers; training in aircraft
maintenance and equipment repair; civic-action projects; and consultations and meetings of
the Philippine-U.S. Mutual Defense Board. It is at the level of the Mutual Defense Board-
The provisions of the MDT must then be read in conjunction with those of the VFA.
which is headed jointly by the Chief of Staff of the AFP and the Commander in Chief of the
U.S. Pacific Command-that the VFA exercises are planned. Final approval of any
Article I of the VFA indicates that the presence of U.S. military and civilian personnel in the activity involving U.S. forces is, however, invariably given by the Philippine Government.
Philippines is "in connection with activities approved by the Philippine Government." While
the treaty does not expressly enumerate or detail the nature of activities of U.S. troops in the
Siazon clarified that it is not the VFA by itself that determines what activities will be
country, its Preamble makes explicit references to the reaffirmation of the obligations of
conductedbetween the armed forces of the U.S. and the Philippines. The VFA regulates and
both countries under the MDT. These obligations include the strengthening of international
provides the legal framework for the presence, conduct and legal status of U.S.
and regional security in the Pacific area and the promotion of common security interests.
personnel while they are in the country for visits, joint exercises and other related activities.

The Court has already settled in Lim v. Executive Secretary that the phrase "activities
What can be gleaned from the provisions of the VFA, the joint report of the Senate
approved by the Philippine Government" under Article I of the VFA was intended to be
committees on foreign relations and on national defense and security, and the ruling of
ambiguous in order to afford the parties flexibility to adjust the details of the purpose of the
379

this Court in Lim is that the "activities" referred to in the treaty are meant to be specified No permanent US basing and support facilities shall be established. Temporary
and identified infurther agreements. EDCA is one such agreement. structures such as those for troop billeting, classroom instruction and messing may be set
up for use by RP and US Forces during the Exercise.
EDCA seeks to be an instrument that enumerates the Philippine-approved activities of U.S.
personnel referred to in the VFA. EDCA allows U.S. military and civilian personnel to perform The Exercise shall be implemented jointly by RP and US Exercise Co-Directors under the
"activities approved by the Philippines, as those terms are defined in the VFA"278 and clarifies authority of the Chief of Staff, AFP. In no instance will US Forces operate independently
that these activities include those conducted within the Agreed Locations: during field training exercises (FTX). AFP and US Unit Commanders will retain command
over their respective forces under the overall authority of the Exercise Co-Directors. RP and
1. Security cooperation exercises; joint and combined training activities; humanitarian US participants shall comply with operational instructions of the AFP during the FTX.
assistance and disaster relief activities; and such other activities as may be agreed upon by
the Parties279 The exercise shall be conducted and completed within a period of not more than six months,
with the projected participation of 660 US personnel and 3,800 RP Forces. The Chief of Staff,
2. Training; transit; support and related activities; refueling of aircraft; bunkering of vessels; AFP shall direct the Exercise Co-Directors to wind up and terminate the Exercise and other
temporary maintenance of vehicles, vessels, and aircraft; temporary accommodation of activities within the six month Exercise period.
personnel; communications; prepositioning of equipment, supplies, and materiel;
deployment of forces and materiel; and such other activities as the Parties may agree280 The Exercise is a mutual counter-terrorism advising, assisting and training Exercise relative
to Philippine efforts against the ASG, and will be conducted on the Island of Basilan. Further
3. Exercise of operational control over the Agreed Locations for construction activities and advising, assisting and training exercises shall be conducted in Malagutay and the Zamboanga
other types of activity, including alterations and improvements thereof281 area. Related activities in Cebu will be for support of the Exercise.

4. Exercise of all rights and authorities within the Agreed Locations that are necessary for US exercise participants shall not engage in combat, without prejudice to their right of self-
their operational control or defense, including the adoption of apfropriate measures to defense.
protect U.S. forces and contractors282
These terms of Reference are for purposes of this Exercise only and do not create additional
5. Use of water, electricity, and other public utilities283 legal obligations between the US Government and the Republic of the Philippines.

6. Operation of their own telecommunication systems, including the utilization of such II. EXERCISE LEVEL
means and services as are required to ensure the full ability to operate telecommunication
systems, as well as the use of the necessary radio spectrum allocated for this purpose284 1. TRAINING

According to Article I of EDCA, one of the purposes of these activities is to maintain and a. The Exercise shall involve the conduct of mutual military assisting,
develop, jointly and by mutual aid, the individual and collective capacities of both countries advising and trainingof RP and US Forces with the primary objective
to resist an armed attack. It further states that the activities are in furtherance of the MDT of enhancing the operational capabilities of both forces to combat
and within the context of the VFA. terrorism.

We note that these planned activities are very similar to those under the Terms of b. At no time shall US Forces operate independently within RP territory.
Reference285 mentioned in Lim. Both EDCA and the Terms of Reference authorize the U.S. to
perform the following: (a) participate in training exercises; (b) retain command over their c. Flight plans of all aircraft involved in the exercise will comply with the
forces; (c) establish temporary structures in the country; (d) share in the use of their local air traffic regulations.
respective resources, equipment and other assets; and (e) exercise their right to self-defense.
We quote the relevant portion of the Terms and Conditions as follows:286
2. ADMINISTRATION & LOGISTICS

I. POLICY LEVEL
380

a. RP and US participating forces may share, in accordance with their respective laws and EDCA is far from being permanent in nature compared to the practice of states as shown in
regulations, in the use of their resources, equipment and other assets. They will use their other defense cooperation agreements. For example, Article XIV(l) of the U.S.-Romania
respective logistics channels. x x x. (Emphases Supplied) defense agreement provides the following:

After a thorough examination of the content, purpose, and framework of the MDT and the This Agreement is concluded for an indefinite period and shall enter into force in accordance
VFA, we find that EDCA has remained within the parameters set in these two treaties. Just with the internal laws of each Party x x x. (emphasis supplied)
like the Terms of Reference mentioned in Lim, mere adjustments in detail to implement the
MDT and the VFA can be in the form of executive agreements. Likewise, Article 36(2) of the US-Poland Status of Forces Agreement reads:

Petitioners assert287 that the duration of the activities mentioned in EDCA is no longer This Agreement has been concluded for an indefinite period of time. It may be terminated
consistent with the temporary nature of the visits as contemplated in the VFA. They point by written notification by either Party and in that event it terminates 2 years after the receipt
out that Article XII(4) of EDCA has an initial term of 10 years, a term automatically renewed of the notification. (emphasis supplied)
unless the Philippines or the U.S. terminates the agreement. According to petitioners, such
length of time already has a badge of permanency.
Section VIII of US.-Denmark Mutual Support Agreement similarly provides:

In connection with this, Justice Teresita J. Leonardo-De Castro likewise argues in her
8.1 This Agreement, which consists of a Preamble, SECTIONs I-VIII, and Annexes A and B, shall
Concurring and Dissenting Opinion that the VFA contemplated mere temporary visits from
become effective on the date of the last signature affixed below and shall remain in force
U.S. forces, whereas EDCA allows an unlimited period for U.S. forces to stay in the
until terminated by the Parties, provided that it may be terminated by either Party upon 180
Philippines.288
days written notice of its intention to do so to the other Party. (emphasis supplied)

However, the provisions of EDCA directly contradict this argument by limiting itself to 10
On the other hand, Article XXI(3) of the US.-Australia Force Posture Agreement provides a
years of effectivity. Although this term is automatically renewed, the process for terminating
longer initial term:
the agreement is unilateral and the right to do so automatically accrues at the end of the 10
year period. Clearly, this method does not create a permanent obligation.
3. This Agreement shall have an initial term of 25 years and thereafter shall continue in
force, but may be terminated by either Party at any time upon one year's written notice to
Drawing on the reasoning in Lim, we also believe that it could not have been by chance that
the other Party through diplomatic channels. (emphasis supplied)
the VFA does not include a maximum time limit with respect to the presence of U.S.
personnel in the country. We construe this lack of specificity as a deliberate effort on the
part of the Philippine and the U.S. governments to leave out this aspect and reserve it for the The phrasing in EDCA is similar to that in the U.S.-Australia treaty but with a term less than
"adjustment in detail" stage of the implementation of the treaty. We interpret the half of that is provided in the latter agreement. This means that EDCA merely follows the
subsequent, unconditional concurrence of the Senate in the entire text of the VFA as an practice of other states in not specifying a non-extendible maximum term. This practice,
implicit grant to the President of a margin of appreciation in determining the duration of the however, does not automatically grant a badge of permanency to its terms. Article XII(4) of
"temporary" presence of U.S. personnel in the country. EDCA provides very clearly, in fact, that its effectivity is for an initial term of 10 years, which
is far shorter than the terms of effectivity between the U.S. and other states. It is simply
illogical to conclude that the initial, extendible term of 10 years somehow gives EDCA
Justice Brion's dissent argues that the presence of U.S. forces under EDCA is "more
provisions a permanent character.
permanent" in nature.289However, this argument has not taken root by virtue of a simple
glance at its provisions on the effectivity period. EDCA does not grant permanent bases, but
rather temporary rotational access to facilities for efficiency. As Professor Aileen S.P. Baviera The reasoning behind this interpretation is rooted in the constitutional role of the President
notes: who, as Commander-in-Chief of our armed forces, is the principal strategist of the nation
and, as such, duty-bound to defend our national sovereignty and territorial integrity;291 who,
as chief architect of our foreign relations, is the head policymaker tasked to assess, ensure,
The new EDCA would grant American troops, ships and planes rotational access to facilities of
and protect our national security and interests;292 who holds the most comprehensive and
the Armed Forces of the Philippines – but not permanent bases which are prohibited under
most confidential information about foreign countries293 that may affect how we conduct our
the Philippine Constitution - with the result of reducing response time should an external
external affairs; and who has unrestricted access to highly classified military intelligence
threat from a common adversary crystallize.290
data294 that may threaten the life of the nation. Thus, if after a geopolitical prognosis of
381

situations affecting the country, a belief is engendered that a much longer period of military 2. Prepositioning and storage of defense equipment, supplies, and materiel,
training is needed, the President must be given ample discretion to adopt necessary including delivery, management, inspection, use, maintenance, and removal of
measures including the flexibility to set an extended timetable. such equipment, supplies and materiel298

Due to the sensitivity and often strict confidentiality of these concerns, we acknowledge that 3. Carrying out of matters in accordance with, and to the extent permissible under,
the President may not always be able to candidly and openly discuss the complete situation U.S. laws, regulations, and policies299
being faced by the nation. The Chief Executive's hands must not be unduly tied, especially if
the situation calls for crafting programs and setting timelines for approved activities. These EDCA requires that all activities within Philippine territory be in accordance with Philippine
activities may be necessary for maintaining and developing our capacity to resist an armed law. This means that certain privileges denied to aliens are likewise denied to foreign military
attack, ensuring our national sovereignty and territorial integrity, and securing our national contractors. Relevantly, providing security300and carrying, owning, and possessing
interests. If the Senate decides that the President is in the best position to define in firearms301 are illegal for foreign civilians.
operational terms the meaning of temporary in relation to the visits, considered individually
or in their totality, the Court must respect that policy decision. If the Senate feels that there
The laws in place already address issues regarding the regulation of contractors. In the 2015
is no need to set a time limit to these visits, neither should we.
Foreign Investment Negative list,302 the Executive Department has already identified
corporations that have equity restrictions in Philippine jurisdiction. Of note is No. 5 on the list
Evidently, the fact that the VFA does not provide specificity in regard to the extent of the - private security agencies that cannot have any foreign equity by virtue of Section 4 of
"temporary" nature of the visits of U.S. personnel does not suggest that the duration to Republic Act No. 5487;303 and No. 15, which regulates contracts for the construction of
which the President may agree is unlimited. Instead, the boundaries of the meaning of the defense-related structures based on Commonwealth Act No. 541.
term temporary in Article I of the treaty must be measured depending on the purpose of
each visit or activity.295 That purpose must be analyzed on a case-by-case basis depending on
Hence, any other entity brought into the Philippines by virtue of EDCA must subscribe to
the factual circumstances surrounding the conclusion of the implementing agreement. While
corporate and civil requirements imposed by the law, depending on the entity's corporate
the validity of the President's actions will be judged under less stringent standards, the
structure and the nature of its business.
power of this Court to determine whether there was grave abuse of discretion remains
unimpaired.
That Philippine laws extraneous to EDCA shall govern the regulation of the activities of U.S.
contractors has been clear even to some of the present members of the Senate.
d. Authorized activities performed by US. contractors within Philippine territory - who were
legitimately permitted to enter the country independent of EDCA - are subject to relevant
Philippine statutes and regulations and must be consistent with the MDT and the VFA For instance, in 2012, a U.S. Navy contractor, the Glenn Marine, was accused of spilling fuel
in the waters off Manila Bay.304 The Senate Committee on Foreign Relations and the Senate
Committee on Environment and Natural Resources chairperson claimed environmental and
Petitioners also raise296 concerns about the U.S. government's purported practice of hiring
procedural violations by the contractor.305 The U.S. Navy investigated the contractor and
private security contractors in other countries. They claim that these contractors - one of
promised stricter guidelines to be imposed upon its contractors.306 The statement attributed
which has already been operating in Mindanao since 2004 - have been implicated in incidents
to Commander Ron Steiner of the public affairs office of the U.S. Navy's 7th Fleet - that U.S.
or scandals in other parts of the globe involving rendition, torture and other human rights
Navy contractors are bound by Philippine laws - is of particular relevance. The statement
violations. They also assert that these contractors employ paramilitary forces in other
acknowledges not just the presence of the contractors, but also the U.S. position that these
countries where they are operating.
contractors are bound by the local laws of their host state. This stance was echoed by other
U.S. Navy representatives.307
Under Articles III and IV of EDCA, U.S. contractors are authorized to perform only the
following activities:
This incident simply shows that the Senate was well aware of the presence of U.S.
contractors for the purpose of fulfilling the terms of the VFA. That they are bound by
1. Training; transit; support and related activities; refueling of aircraft; bunkering of Philippine law is clear to all, even to the U.S.
vessels; temporary maintenance of vehicles, vessels, and aircraft; temporary
accommodation of personnel; communications; prepositioning of equipment,
As applied to EDCA, even when U.S. contractors are granted access to the Agreed Locations,
supplies, and materiel; deployment of forces and materiel; and such other activities
all their activities must be consistent with Philippine laws and regulations and pursuant to
as the Parties may agree297
the MDT and the VFA.
382

While we recognize the concerns of petitioners, they do not give the Court enough The United States shall return to the Philippines any Agreed Locations, or any portion
justification to strike down EDCA. In Lim v. Executive Secretary, we have already explained thereof, including non-relocatable structures and assemblies constructed, modified, or
that we cannot take judicial notice of claims aired in news reports, "not because of any issue improved by the United States, once no longer required by United States forces for activities
as to their truth, accuracy, or impartiality, but for the simple reason that facts must be under this Agreement. The Parties or the Designated Authorities shall consult regarding the
established in accordance with the rules of evidence."308 What is more, we cannot move one terms of return of any Agreed Locations, including possible compensation for improvements
step ahead and speculate that the alleged illegal activities of these contractors in other or construction.
countries would take place in the Philippines with certainty. As can be seen from the above
discussion, making sure that U.S. contractors comply with Philippine laws is a function of law The context of use is "required by United States forces for activities under this Agreement."
enforcement. EDCA does not stand in the way of law enforcement. Therefore, the return of an Agreed Location would be within the parameters of an activity
that the Mutual Defense Board (MDB) and the Security Engagement Board (SEB) would
Nevertheless, we emphasize that U.S. contractors are explicitly excluded from the coverage authorize. Thus, possession by the U.S. prior to its return of the Agreed Location would be
of the VFA. As visiting aliens, their entry, presence, and activities are subject to all laws and based on the authority given to it by a joint body co-chaired by the "AFP Chief of Staff and
treaties applicable within the Philippine territory. They may be refused entry or expelled Commander, U.S. PACOM with representatives from the Philippines' Department of National
from the country if they engage in illegal or undesirable activities. There is nothing that Defense and Department of Foreign Affairs sitting as members."313 The terms shall be
prevents them from being detained in the country or being subject to the jurisdiction of our negotiated by both the Philippines and the U.S., or through their Designated Authorities. This
courts. Our penal laws,309 labor laws,310 and immigrations laws311 apply to them and provision, seen as a whole, contradicts petitioners' interpretation of the return as a "badge of
therefore limit their activities here. Until and unless there is another law or treaty that exclusivity." In fact, it shows the cooperation and partnership aspect of EDCA in full bloom.
specifically deals with their entry and activities, their presence in the country is subject to
unqualified Philippine jurisdiction. Second, the term "unimpeded access" must likewise be viewed from a contextual
perspective. Article IV(4) states that U.S. forces and U.S. contractors shall have "unimpeded
EDCA does not allow the presence of U.S.-owned or -controlled military facilities and bases access to Agreed Locations for all matters relating to the prepositioning and storage of
in the Philippines defense equipment, supplies, and materiel, including delivery, management, inspection, use,
maintenance, and removal of such equipment, supplies and materiel."
Petitioners Saguisag et al. claim that EDCA permits the establishment of U.S. military bases
through the "euphemistically" termed "Agreed Locations. "312 Alluding to the definition of At the beginning of Article IV, EDCA states that the Philippines gives the U.S. the authority to
this term in Article II(4) of EDCA, they point out that these locations are actually military bring in these equipment, supplies, and materiel through the MDB and SEB security
bases, as the definition refers to facilities and areas to which U.S. military forces have access mechanism. These items are owned by the U.S.,314 are exclusively for the use of the
for a variety of purposes. Petitioners claim that there are several badges of exclusivity in the U.S.315 and, after going through the joint consent mechanisms of the MDB and the SEB, are
use of the Agreed Locations by U.S. forces. First, Article V(2) of EDCA alludes to a "return" of within the control of the U.S.316 More importantly, before these items are considered
these areas once they are no longer needed by U.S. forces, indicating that there would be prepositioned, they must have gone through the process of prior authorization by the MDB
some transfer of use. Second, Article IV(4) ofEDCA talks about American forces' unimpeded and the SEB and given proper notification to the AFP.317
access to the Agreed Locations for all matters relating to the prepositioning and storage of
U.S. military equipment, supplies, and materiel. Third, Article VII of EDCA authorizes U.S. Therefore, this "unimpeded access" to the Agreed Locations is a necessary adjunct to the
forces to use public utilities and to operate their own telecommunications system. ownership, use, and control of the U.S. over its own equipment, supplies, and materiel and
must have first been allowed by the joint mechanisms in play between the two states since
a. Preliminary point on badges of exclusivity the time of the MDT and the VFA. It is not the use of the Agreed Locations that is
exclusive per se; it is mere access to items in order to exercise the rights of ownership
As a preliminary observation, petitioners have cherry-picked provisions of EDCA by granted by virtue of the Philippine Civil Code.318
presenting so-called "badges of exclusivity," despite the presence of contrary provisions
within the text of the agreement itself. As for the view that EDCA authorizes U.S. forces to use public utilities and to operate their
own telecommunications system, it will be met and answered in part D, infra.
First, they clarify the word "return" in Article V(2) of EDCA. However, the use of the word
"return" is within the context of a lengthy provision. The provision as a whole reads as Petitioners also point out319 that EDCA is strongly reminiscent of and in fact bears a one-to-
follows: one correspondence with the provisions of the 1947 MBA. They assert that both agreements
(a) allow similar activities within the area; (b) provide for the same "species of ownership"
383

over facilities; and (c) grant operational control over the entire area. Finally, they The Philippine experience with U.S. military bases under the 1947 MBA is simply not possible
argue320 that EDCA is in fact an implementation of the new defense policy of the U.S. under EDCA for a number of important reasons.
According to them, this policy was not what was originally intended either by the MDT or by
the VFA. First, in the 1947 MBA, the U.S. retained all rights of jurisdiction in and over Philippine
territory occupied by American bases. In contrast, the U.S. under EDCA does not enjoy any
On these points, the Court is not persuaded. such right over any part of the Philippines in which its forces or equipment may be found.
Below is a comparative table between the old treaty and EDCA:
The similar activities cited by petitioners321 simply show that under the MBA, the U.S. had the
right to construct, operate, maintain, utilize, occupy, garrison, and control the bases. The so-
1947 MBA/ 1946 Treaty of General Relations EDCA
called parallel provisions of EDCA allow only operational control over the Agreed Locations
specifically for construction activities. They do not allow the overarching power to operate, 1947 MBA, Art. I(1): EDCA, preamble:
maintain, utilize, occupy, garrison, and control a base with full discretion. EDCA in fact limits
the rights of the U.S. in respect of every activity, including construction, by giving the MDB
The Government of the Republic of Affirming that the Parties share an
and the SEB the power to determine the details of all activities such as, but not limited to,
the Philippines (hereinafter referred to as the understanding for the United States not to
operation, maintenance, utility, occupancy, garrisoning, and control.322
Philippines) grants to the Government of establish a permanent military presence or
the United States of America (hereinafter base in the territory of the Philippines;
The "species of ownership" on the other hand, is distinguished by the nature of the property. referred to as the United States) the right to
For immovable property constructed or developed by the U.S., EDCA expresses that retain the use of the bases in the Recognizing that all United States access to
ownership will automatically be vested to the Philippines.323 On the other hand, for movable Philippines listed in Annex A attached hereto. and use of facilities and areas will be at the
properties brought into the Philippines by the U.S., EDCA provides that ownership is retained
invitation of the Philippines and with full
by the latter. In contrast, the MBA dictates that the U.S. retains ownership over immovable
1947 MBA, Art. XVII(2): respect for the Philippine Constitution and
and movable properties.
Philippine laws;
All buildings and structures which
To our mind, both EDCA and the MBA simply incorporate what is already the law of the land
are erected by the United States in the bases EDCA, Art. II(4):
in the Philippines. The Civil Code's provisions on ownership, as applied, grant the owner of a
shall be the property of the United
movable property full rights over that property, even if located in another person's
States and may be removed by it before the "Agreed Locations" means facilities and
property.324
expiration of this Agreement or the earlier areas that are provided by the Government
relinquishment of the base on which the of the Philippines through the AFP and that
The parallelism, however, ends when the situation involves facilities that can be considered structures are situated. There shall be no United States forces, United States
immovable. Under the MBA, the U.S. retains ownership if it paid for the facility.325 Under obligation on the part of the Philippines or of contractors, and others as mutually agreed,
EDCA, an immovable is owned by the Philippines, even if built completely on the back of U.S. the United States to rebuild or repair any shall have the right to access and use
funding.326 This is consistent with the constitutional prohibition on foreign land ownership.327 destruction or damage inflicted from any pursuant to this Agreement. Such Agreed
cause whatsoever on any of the said buildings Locations may be listed in an annex to be
Despite the apparent similarity, the ownership of property is but a part of a larger whole that or structures owned or used by the United appended to this Agreement, and may be
must be considered before the constitutional restriction is violated. Thus, petitioners' points States in the bases. x x x x. further described in implementing
on operational control will be given more attention in the discussion below. The arguments arrangements.
on policy are, however, outside the scope of judicial review and will not be discussed 1946 Treaty of Gen. Relations, Art. I:
EDCA, Art. V:
Moreover, a direct comparison of the MBA and EDCA will result in several important The United States of America agrees to
distinctions that would allay suspicion that EDCA is but a disguised version of the MBA. withdraw and surrender, and does hereby 1. The Philippines shall retain ownership of
withdraw and surrender, all rights of and title to Agreed Locations.
b. There are substantial matters that the US. cannot do under EDCA, but which it was possession, supervision, jurisdiction, control
authorized to do under the 1947 MBA or sovereignty existing and exercised by the
4. All buildings, non-relocatable structures,
United States of America in and over the
384

territory and the people of the Philippine and assemblies affixed to the land in the appurtenances to such bases, and the
Islands, except the use of such bases, Agreed Locations, including ones altered or rights incident thereto, as the United
necessary appurtenances to such bases, and improved by United States forces, remain States of America, by agreement with the
the rights incident thereto, as the United the property of the Philippines.Permanent Republic of the Philippines may deem
States of America, by agreement with the buildings constructed by United States necessary to retain for the mutual
Republic of the Philippines may deem forces become the property of the protection of the Republic of the
necessary to retain for the mutual protection Philippines, once constructed, but shall be Philippines and of the United States of
of the Republic of the Philippines and of the used by United States forces until no longer America. x x x.
United States of America. x x x. required by United States forces.

Third, in EDCA, the Philippines is- guaranteed access over the entire area of the Agreed
Locations. On the other hand, given that the U.S. had complete control over its military bases
under the 1947 MBA, the treaty did not provide for any express recognition of the right of
Second, in the bases agreement, the U.S. and the Philippines were visibly not on equal access of Philippine authorities. Without that provision and in light of the retention of U.S.
footing when it came to deciding whether to expand or to increase the number of bases, as sovereignty over the old military bases, the U.S. could effectively prevent Philippine
the Philippines may be compelled to negotiate with the U.S. the moment the latter authorities from entering those bases.
requested an expansion of the existing bases or to acquire additional bases. In EDCA, U.S.
access is purely at the invitation of the Philippines.
1947 MBA EDCA

No EDCA, Art. III(5):


1947 MBA/ 1946 Treaty of General EDCA
equivalent
Relations
provision. The Philippine Designated Authority and its authorized
1947 MBA, Art.I(3): EDCA, preamble: representative shall have access to the entire area of the Agreed Locations.
Such access shall be provided promptly consistent with operational safety
The Philippines agree to enter into Recognizing that all United States access to and security requirements in accordance with agreed procedures developed
negotiations with the United States at the and use of facilities and areas will be at the by the Parties.
latter's request, to permit the United invitation of the Philippines and with full
States to expand such bases, to exchange respect for the Philippine Constitution and
Fourth, in the bases agreement, the U.S. retained the right, power, and authority over the
such bases for other bases, to acquire Philippine laws;
establishment, use, operation, defense, and control of military bases, including the limits of
additional bases, or relinquish rights to
territorial waters and air space adjacent to or in the vicinity of those bases. The only standard
bases, as any of such exigencies may be EDCA. Art. II(4): used in determining the extent of its control was military necessity. On the other hand, there
required by military necessity.
is no such grant of power or authority under EDCA. It merely allows the U.S. to exercise
"Agreed Locations" means facilities and operational control over the construction of Philippine-owned structures and facilities:
1946 Treaty of Gen. Relations, Art. I: areas that are provided by the Government of
the Philippines through the AFP and that
The United States of America agrees to United States forces, United States contractors, 1947 MBA EDCA
withdraw and surrender, and does hereby and others as mutually agreed, shall have the
1947 MBA, Art.I(2): EDCA, Art. III(4):
withdraw and surrender, all rights of right to access and use pursuant to this
possession, supervision, jurisdiction, Agreement. Such Agreed Locations may be
control or sovereignty existing and listed in an annex to be appended to this The Philippines agrees to permit The Philippines hereby grants to the United
exercised by the United States of Agreement, and may be further described in the United States, upon notice to States, through bilateral security mechanisms, such as
America in and over the territory and the implementing arrangements. the Philippines, to use such of the MDB and SEB, operational control of Agreed
people of the Philippine Islands, except those bases listed in Annex B as Locations for construction
the use of such bases, necessary the United States determines to activities and authority to undertake such
385

be required by military necessity. activities on, and make alterations and improvements 1947 MBA, Art.I(2):
to, Agreed Locations. United States forces shall
1947 MBA, Art. III(1): consult on issues regarding such construction, The Philippines agrees to permit
alterations, and improvements based on the Parties' the United States, upon notice to
shared intent that the technical requirements and the Philippines, to use such of
It is mutually agreed that
construction standards of any such projects those bases listed in Annex B as
the United Statesshall have
undertaken by or on behalf of United States forces the United States determines to
the rights, power and authority
should be consistent with the requirements and be required by military necessity.
within the bases which
standards of both Parties.
are necessary for the
establishment, use, operation and
Sixth, under the MBA, the U.S. was given the right, power, and authority to control and
defense thereof or appropriate for
prohibit the movement and operation of all types of vehicles within the vicinity of the bases.
the control thereof and all the
The U.S. does not have any right, power, or authority to do so under EDCA.
rights, power and authority within
the limits of territorial waters and
air space adjacent to, or in the 1947 MBA EDCA
vicinity of, the bases which
are necessary to provide access to 1947 MBA, Art. 111(2)(c) No
them, or appropriate for their equivalent
control. Such rights, power and authority shall include, inter alia, the right, power and provision.
authority: x x x x to control (including the right to prohibit) in so far as may be
required for the efficient operation and safety of the bases, and within the
Fifth, the U.S. under the bases agreement was given the authority to use Philippine territory
limits of military necessity, anchorages, moorings, landings, takeoffs,
for additional staging areas, bombing and gunnery ranges. No such right is given under EDCA,
movements and operation of ships and water-borne craft, aircraft and other
as seen below:
vehicles on water, in the air or on land comprising

1947 MBA EDCA


Seventh, under EDCA, the U.S. is merely given temporary access to land and facilities
1947 MBA, Art. VI: EDCA, Art. III(1): (including roads, ports, and airfields). On the other hand, the old treaty gave the U.S. the
right to improve and deepen the harbors, channels, entrances, and anchorages; and to
construct or maintain necessary roads and bridges that would afford it access to its military
The United States shall, subject to With consideration of the views of the Parties,
bases.
previous agreement with the the Philippines hereby authorizes and agrees that
Philippines, have the right to use United States forces, United States contractors, and
land and coastal sea areas of vehicles, vessels, and aircraft operated by or for United 1947 MBA EDCA
appropriate size and location for States forces may conduct the following activities with
periodic maneuvers, for respect to Agreed Locations: training; transit; support 1947 MBA, Art. III(2)(b): EDCA, Art. III(2):
additional staging areas, and related activities; refueling of aircraft; bunkering of
bombing and gunnery ranges, vessels; temporary maintenance of vehicles, vessels, Such rights, power and authority shall When requested, the Designated Authority of
and for such intermediate and aircraft; temporary accommodation of personnel; include, inter alia, the right, power and the Philippines shall assist in facilitating transit
airfields as may be required for communications; prepositioning of equipment, supplies, authority: x x x x to improve and deepen or temporary access by United States forces to
safe and efficient air operations. and materiel; deploying forces and materiel; and such the harbors, channels, entrances and public land and facilities (including roads, ports,
Operations in such areas shall be other activities as the Parties may agree. anchorages, and to construct or and airfields), including those owned or
carried on with due regard and maintain necessary roadsand bridges controlled by local governments, and to other
safeguards for the public safety. affording access to the bases. land and facilities (including roads, ports, and
airfields).
386

Eighth, in the 1947 MBA, the U.S. was granted the automatic right to use any and all public
frequency. any such projects undertaken by or on behalf
utilities, services and facilities, airfields, ports, harbors, roads, highways, railroads, bridges,
of United States forces should be consistent
viaducts, canals, lakes, rivers, and streams in the Philippines in the same manner that
with the requirements and standards of both
Philippine military forces enjoyed that right. No such arrangement appears in EDCA. In fact, it
Parties.
merely extends to U.S. forces temporary access to public land and facilities when requested:

Tenth, EDCA does not allow the U.S. to acquire, by condemnation or expropriation
1947 MBA EDCA
proceedings, real property belonging to any private person. The old military bases agreement
1947 MBA, Art. VII: EDCA, Art. III(2): gave this right to the U.S. as seen below:

It is mutually agreed that the United States may When requested, the Designated 1947 MBA EDCA
employ and use for United States military Authority of the Philippines shall assist
forces any and all public utilities, other services in facilitating transit or temporary 1947 MBA, Art. XXII(l): No
and facilities, airfields, ports, harbors, roads, access by United States forces to public equivalent
highways, railroads, bridges, viaducts, canals, land and facilities (including roads, Whenever it is necessary to acquire by provision.
lakes, rivers and streams in the Philippines under ports, and airfields), including those
conditions no less favorable than those that may owned or controlled by local
condemnation or expropriation proceedings real property belonging to any
be applicablefrom time to time to the military governments, and to other land and
private persons, associations or corporations located in bases named in
forces of the Philippines. facilities (including roads, ports, and
Annex A and Annex B in order to carry out the purposes of this Agreement,
airfields).
the Philippines will institute and prosecute such condemnation or
expropriation proceedings in accordance with the laws of the Philippines. The
Ninth, under EDCA, the U.S. no longer has the right, power, and authority to construct, United States agrees to reimburse the Philippines for all the reasonable
install, maintain, and employ any type of facility, weapon, substance, device, vessel or expenses, damages and costs therebv incurred, including the value of the
vehicle, or system unlike in the old treaty. EDCA merely grants the U.S., through bilateral property as determined by the Court. In addition, subject to the mutual
security mechanisms, the authority to undertake construction, alteration, or improvements agreement of the two Governments, the United States will reimburse the
on the Philippine-owned Agreed Locations. Philippines for the reasonable costs of transportation and removal of any
occupants displaced or ejected by reason of the condemnation or
expropriation.
1947 MBA EDCA

1947 MBA, Art. III(2)(e): EDCA, Art. III(4): Eleventh, EDCA does not allow the U.S. to unilaterally bring into the country non-Philippine
nationals who are under its employ, together with their families, in connection with the
Such rights, power and authority shall The Philippines hereby grants to the United construction, maintenance, or operation of the bases. EDCA strictly adheres to the limits
include, inter alia, the right, power and States, through bilateral security under the VFA.
authority: x x x x to construct, install, mechanisms, such as the MDB and SEB,
maintain, and employ on any base any operational control of Agreed Locations for
1947 MBA EDCA
type of facilities, weapons, substance, construction activities and authority to
device, vessel or vehicle on or under the undertake such activities on, and make 1947 MBA, Art. XI(l): EDCA, Art. II:
ground, in the air or on or under the water alterations and improvements to, Agreed
that may be requisite or appropriate, Locations. United States forces shall consult
It is mutually agreed that the United States 1. "United States personnel" means
including meteorological systems, aerial and on issues regarding such construction,
shall have the right to bring into the United States military and civilian
water navigation lights, radio and radar alterations, and improvements based on the
Philippines members of the United States personneltemporarily in the territory of
apparatus and electronic devices, of any Parties' shared intent that the technical
military forces and the United States nationals the Philippines in connection with activities
desired power, type of emission and requirements and construction standards of
employed by or under a contract with the approved by the Philippines, as those
387

United States together with their families, and terms are defined in the VFA. It is mutually agreed that the United States
technical personnel of other nationalities (not
being persons excluded by the laws of the 3. "United States contractors" means shall have the right to establish on bases, free of all licenses; fees; sales,
Philippines) in connection with the companies and firms, and their employees, excise or other taxes, or imposts; Government agencies, including
construction, maintenance, or operation of the under contract or subcontract to or on concessions, such as sales commissaries and post exchanges; messes and
bases. The United States shall make suitable behalf of the United States Department of social clubs, for the exclusive use of the United States military forces and
arrangements so that such persons may be Defense. United States contractors are not authorized civilian personnel and their families. The merchandise or services
readily identified and their status established includedas part of the definition of United sold or dispensed by such agencies shall be free of all taxes, duties and
when necessary by the Philippine authorities. States personnel in this inspection by the Philippine authorities. Administrative measures shall be
Such persons, other than members of the Agreement, including within the context taken by the appropriate authorities of the United States to prevent the resale
United States armed forces in uniform, shall of the VFA. of goods which are sold under the provisions of this Article to persons not
present their travel documents to the entitled to buy goods at such agencies and, generally, to prevent abuse of the
appropriate Philippine authorities for visas, it privileges granted under this Article. There shall be cooperation between such
being understood that no objection will be authorities and the Philippines to this end.
made to their travel to the Philippines as non-
immigrants.
In sum, EDCA is a far cry from a basing agreement as was understood by the people at the
time that the 1987 Constitution was adopted.
Twelfth, EDCA does not allow the U.S. to exercise jurisdiction over any offense committed by
any person within the Agreed Locations, unlike in the former military bases:
Nevertheless, a comprehensive review of what the Constitution means by "foreign military
bases" and "facilities" is required before EDCA can be deemed to have passed judicial
1947 MBA EDCA scrutiny.

1947 MBA, Art. XIII(l)(a): No c. The meaning of military facilities and bases
equivalent
The Philippines consents that the United provision.
An appreciation of what a military base is, as understood by the Filipino people in 1987,
would be vital in determining whether EDCA breached the constitutional restriction.
States shall have the right to exercise jurisdiction over the following
offenses: (a) Any offense committed by any person within any base except Prior to the drafting of the 1987 Constitution, the last definition of "military base" was
where the offender and offended parties are both Philippine citizens (not provided under Presidential Decree No. (PD) 1227.328 Unlawful entry into a military base is
members of the armed forces of the United States on active duty) or the punishable under the decree as supported by Article 281 of the Revised Penal Code, which
offense is against the security of the Philippines. itself prohibits the act of trespass.

Thirteenth, EDCA does not allow the U.S. to operate military post exchange (PX) facilities, Section 2 of the law defines the term in this manner: "'[M]ilitary base' as used in this
which is free of customs duties and taxes, unlike what the expired MBA expressly allowed. decree means any military, air, naval, or coast guard reservation, base, fort, camp, arsenal,
Parenthetically, the PX store has become the cultural icon of U.S. military presence in the yard, station, or installation in the Philippines."
country.
Commissioner Tadeo, in presenting his objections to U.S. presence in the Philippines before
the 1986 Constitutional Commission, listed the areas that he considered as military bases:
1947 MBA EDCA

1947 MBA, Art. XVIII(l): No 1,000 hectares Camp O'Donnel


equivalent
provision. 20,000 hectares Crow Valley Weapon's Range
388

55,000 hectares Clark Air Base the 1991 proposal, and a concrete understanding of what was constitutionally restricted. This
trend birthed the VFA which, as discussed, has already been upheld by this Court.
150 hectares Wallace Air Station
The latest agreement is EDCA, which proposes a novel concept termed "Agreed Locations."
400 hectares John Hay Air Station
By definition, Agreed Locations are
15,000 hectares Subic Naval Base
facilities and areas that are provided by the Government of the Philippines through the AFP
1,000 hectares San Miguel Naval Communication and that United States forces, United States contractors, and others as mutually agreed, shall
have the right to access and use pursuant to this Agreement. Such Agreed Locations may be
listed in an annex to be appended to this Agreement, and may be further described in
750 hectares Radio Transmitter in Capas, Tarlac
implementing arrangements.332

900 hectares Radio Bigot Annex at Bamban, Tarlac329


Preliminarily, respondent already claims that the proviso that the Philippines shall retain
ownership of and title to the Agreed Locations means that EDCA is "consistent with Article II
The Bases Conversion and Development Act of 1992 described its coverage in its Declaration of the VFA which recognizes Philippine sovereignty and jurisdiction over locations within
of Policies: Philippine territory.333

Sec. 2. Declaration of Policies. - It is hereby declared the policy of the Government to By this interpretation, respondent acknowledges that the contention of petitioners springs
accelerate the sound and balanced conversion into alternative productive uses of the Clark from an understanding that the Agreed Locations merely circumvent the constitutional
and Subic military reservations and their extensions (John Hay Station, Wallace Air Station, restrictions. Framed differently, the bone of contention is whether the Agreed Locations are,
O'Donnell Transmitter Station, San Miguel Naval Communications Station and Capas Relay from a legal perspective, foreign military facilities or bases. This legal framework triggers
Station), to raise funds by the sale of portions of Metro Manila military camps, and to apply Section 25, Article XVIII, and makes Senate concurrence a sine qua non.
said funds as provided herein for the development and conversion to productive civilian use
of the lands covered under the 194 7 Military Bases Agreement between the Philippines and
Article III of EDCA provides for Agreed Locations, in which the U.S. is authorized by the
the United States of America, as amended.330
Philippines to "conduct the following activities: "training; transit; support and related
activities; refueling of aircraft; bunkering of vessels; temporary maintenance of vehicles,
The result of the debates and subsequent voting is Section 25, Article XVIII of the vessels and aircraft; temporary accommodation of personnel; communications;
Constitution, which specifically restricts, among others, foreign military facilities or bases. At prepositioning of equipment, supplies and materiel; deploying forces and materiel; and such
the time of its crafting of the Constitution, the 1986 Constitutional Commission had a clear other activities as the Parties may agree."
idea of what exactly it was restricting. While the term "facilities and bases" was left
undefined, its point of reference was clearly those areas covered by the 1947 MBA as
This creation of EDCA must then be tested against a proper interpretation of the Section 25
amended.
restriction.

Notably, nearly 30 years have passed since then, and the ever-evolving world of military
d. Reasons for the constitutional requirements and legal standards for constitutionally
technology and geopolitics has surpassed the understanding of the Philippine people in 1986.
compatible military bases and facilities
The last direct military action of the U.S. in the region was the use of Subic base as the
staging ground for Desert Shield and Desert Storm during the Gulf War.331In 1991, the
Philippine Senate rejected the successor treaty of the 1947 MBA that would have allowed the Section 25 does not define what is meant by a "foreign military facility or base." While it
continuation of U.S. bases in the Philippines. specifically alludes to U.S. military facilities and bases that existed during the framing of the
Constitution, the provision was clearly meant to apply to those bases existing at the time and
to any future facility or base. The basis for the restriction must first be deduced from the
Henceforth, any proposed entry of U.S. forces into the Philippines had to evolve likewise,
spirit of the law, in order to set a standard for the application of its text, given the particular
taking into consideration the subsisting agreements between both parties, the rejection of
historical events preceding the agreement.
389

Once more, we must look to the 1986 Constitutional Commissioners to glean, from their SPEECH OF COMMISSIONER SUAREZ337
collective wisdom, the intent of Section 25. Their speeches are rich with history and wisdom
and present a clear picture of what they considered in the crafting the provision. MR. SUAREZ: Thank you, Madam President.

SPEECH OF COMMISSIONER REGALADO334 I am quite satisfied that the crucial issues involved in the resolution of the problem of the
removal of foreign bases from the Philippines have been adequately treated by previous
We have been regaled here by those who favor the adoption of the anti-bases provisions speakers. Let me, therefore, just recapitulate the arguments adduced in favor of a foreign
with what purports to be an objective presentation of the historical background of the bases-free Philippines:
military bases in the Philippines. Care appears, however, to have been taken to underscore
the inequity in their inception as well as their implementation, as to seriously reflect on the 1. That every nation should be free to shape its own destiny without outside
supposed objectivity of the report. Pronouncements of military and civilian officials shortly interference;
after World War II are quoted in support of the proposition on neutrality; regrettably, the
implication is that the same remains valid today, as if the world and international activity
2. That no lasting peace and no true sovereignty would ever be achieved so long as
stood still for the last 40 years.
there are foreign military forces in our country;

We have been given inspired lectures on the effect of the presence of the military bases on
3. That the presence of foreign military bases deprives us of the very substance of
our sovereignty - whether in its legal or political sense is not clear - and the theory that any
national sovereigntyand this is a constant source of national embarrassment and
country with foreign bases in its territory cannot claim to be fully sovereign or completely
an insult to our national dignity and selfrespect as a nation;
independent. I was not aware that the concepts of sovereignty and independence have now
assumed the totality principle, such that a willing assumption of some delimitations in the
exercise of some aspects thereof would put that State in a lower bracket of nationhood. 4. That these foreign military bases unnecessarily expose our country to
devastating nuclear attacks;
We have been receiving a continuous influx of materials on the pros and cons on the
advisability of having military bases within our shores. Most of us who, only about three 5. That these foreign military bases create social problems and are designed to
months ago, were just mulling the prospects of these varying contentions are now expected, perpetuate the strangle-hold of United States interests in our national economy
like armchair generals, to decide not only on the geopolitical aspects and contingent and development;
implications of the military bases but also on their political, social, economic and cultural
impact on our national life. We are asked to answer a plethora of questions, such as: 1) 6. That the extraterritorial rights enjoyed by these foreign bases operate
whether the bases are magnets of nuclear attack or are deterrents to such attack; 2) whether to deprive our country of jurisdiction over civil and criminal offenses committed
an alliance or mutual defense treaty is a derogation of our national sovereignty; 3) whether within our own national territory and against Filipinos;
criticism of us by Russia, Vietnam and North Korea is outweighed by the support for us of the
ASEAN countries, the United States, South Korea, Taiwan, Australia and New Zealand; and 4) 7. That the bases agreements are colonial impositions and dictations upon our
whether the social, moral and legal problems spawned by the military bases and their helpless country; and
operations can be compensated by the economic benefits outlined in papers which have
been furnished recently to all of us.335
8. That on the legal viewpoint and in the ultimate analysis, all the bases
agreements are null and void ab initio, especially because they did not count the
Of course, one side of persuasion has submitted categorical, unequivocal and forceful sovereign consent and will of the Filipino people.338
assertions of their positions. They are entitled to the luxury of the absolutes. We are urged
now to adopt the proposed declaration as a "golden," "unique" and "last" opportunity for
Filipinos to assert their sovereign rights. Unfortunately, I have never been enchanted by In the real sense, Madam President, if we in the Commission could accommodate the
superlatives, much less for the applause of the moment or the ovation of the hour. Nor do I provisions I have cited, what is our objection to include in our Constitution a matter as
look forward to any glorious summer after a winter of political discontent. Hence, if I may priceless as the nationalist values we cherish? A matter of the gravest concern for the safety
join Commissioner Laurel, I also invoke a caveat not only against the tyranny of labels but and survival of this nation indeed deserves a place in our Constitution.
also the tyranny of slogans.336
390

x x x Why should we bargain away our dignity and our self-respect as a nation and the future Allow me to say in summation that I am for the retention of American military bases in the
of generations to come with thirty pieces of silver?339 Philippines provided that such an extension from one period to another shall be concluded
upon concurrence of the parties, and such extension shall be based on justice, the
SPEECH OF COMMISSIONER BENNAGEN340 historical amity of the people of the Philippines and the United States and their common
defense interest.348
The underlying principle of military bases and nuclear weapons wherever they are found and
whoever owns them is that those are for killing people or for terrorizing humanity. This SPEECH OF COMMISSIONER ALONTO349
objective by itself at any point in history is morally repugnant. This alone is reason enough for
us to constitutionalize the ban on foreign military bases and on nuclear weapons.341 Madam President, sometime ago after this Commission started with this task of framing a
constitution, I read a statement of President Aquino to the effect that she is for the removal
SPEECH OF COMMISSIONER BACANI342 of the U.S. military bases in this country but that the removal of the U.S. military bases
should not be done just to give way to other foreign bases. Today, there are two world
superpowers, both vying to control any and all countries which have importance to their
x x x Hence, the remedy to prostitution does not seem to be primarily to remove the
strategy for world domination. The Philippines is one such country.
bases because even if the bases are removed, the girls mired in poverty will look for their
clientele elsewhere. The remedy to the problem of prostitution lies primarily elsewhere - in
an alert and concerned citizenry, a healthy economy and a sound education in values. 343 Madam President, I submit that I am one of those ready to completely remove any vestiges
of the days of enslavement, but not prepared to erase them if to do so would merely leave a
vacuum to be occupied by a far worse type.350
SPEECH OF COMMISSIONER JAMIR344

SPEECH OF COMMISSIONER GASCON351


One of the reasons advanced against the maintenance of foreign military bases here is that
they impair portions of our sovereignty. While I agree that our country's sovereignty should
not be impaired, I also hold the view that there are times when it is necessary to do so Let us consider the situation of peace in our world today. Consider our brethren in the
according to the imperatives of national interest. There are precedents to this effect. Thus, Middle East, in Indo-China, Central America, in South Africa - there has been escalation of
during World War II, England leased its bases in the West Indies and in Bermuda for 99 years war in some of these areas because of foreign intervention which views these conflicts
to the United States for its use as naval and air bases. It was done in consideration of 50 through the narrow prism of the East-West conflict. The United States bases have been used
overaged destroyers which the United States gave to England for its use in the Battle of the as springboards for intervention in some of these conflicts. We should not allow ourselves
Atlantic. to be party to the warlike mentality of these foreign interventionists. We must always be
on the side of peace – this means that we should not always rely on military solution.352
A few years ago, England gave the Island of Diego Garcia to the United States for the latter's
use as a naval base in the Indian Ocean. About the same time, the United States obtained x x x The United States bases, therefore, are springboards for intervention in our own
bases in Spain, Egypt and Israel. In doing so, these countries, in effect, contributed to the internal affairs and in the affairs of other nations in this region.
launching of a preventive defense posture against possible trouble in the Middle East and in
the Indian Ocean for their own protection.345 Thus, I firmly believe that a self-respecting nation should safeguard its fundamental
freedoms which should logically be declared in black and white in our fundamental law of the
SPEECH OF COMMISSIONER TINGSON346 land - the Constitution. Let us express our desire for national sovereignty so we may be able
to achieve national self-determination. Let us express our desire for neutrality so that we
may be able to follow active nonaligned independent foreign policies. Let us express our
In the case of the Philippines and the other Southeast Asian nations, the presence of
desire for peace and a nuclear-free zone so we may be able to pursue a healthy and tranquil
American troops in the country is a projection of America's security interest. Enrile said that
existence, to have peace that is autonomous and not imposed. 353
nonetheless, they also serve, although in an incidental and secondary way, the security
interest of the Republic of the Philippines and the region. Yes, of course, Mr. Enrile also
echoes the sentiments of most of us in this Commission, namely: It is ideal for us as an SPEECH OF COMMISSIONER TADEO354
independent and sovereign nation to ultimately abrogate the RP-US military treaty and, at
the right time, build our own air and naval might.347 Para sa magbubukid, ano ha ang kahulugan ng U.S. military bases? Para sa
magbubukid, ang kahulugan nito ay pagkaalipin. Para sa magbubukid, ang pananatili
391

ng U.S. military bases ay tinik sa dibdib ng sambayanang Pilipinong patuloy na nakabaon. Third, the continued exercise by the United States of extraterritoriality despite the
Para sa sambayanang magbubukid, ang ibig sabihin ng U.S. military bases ay batong pabigat condemnations of such practice by the world community of nations in the light of
na patuloy na pinapasan ng sambayanang Pilipino. Para sa sambayanang magbubukid, ang overwhelming international approval of eradicating all vestiges of colonialism.358
pananatili ng U.S. military bases ay isang nagdudumilat na katotohanan ng patuloy na
paggahasa ng imperyalistang Estados Unidos sa ating Inang Bayan - economically, Sixth, the deification of a new concept called pragmatic sovereignty, in the hope that such
politically and culturally. Para sa sambayanang magbubukid ang U.S. military can be wielded to force the United States government to concede to better terms and
bases ay kasingkahulugan ng nuclear weapon - ang kahulugan ay magneto ng conditions concerning the military bases agreement, including the transfer of complete
isang nuclear war. Para sa sambayanang magbubukid, ang kahulugan ng U.S. military control to the Philippine government of the U.S. facilities, while in the meantime we have to
bases ay isang salot.355 suffer all existing indignities and disrespect towards our rights as a sovereign nation.

SPEECH OF COMMISSIONER QUESADA356 Eighth, the utter failure of this forum to view the issue of foreign military bases as
essentially a question of sovereignty which does not require in-depth studies or analyses
The drift in the voting on issues related to freeing ourselves from the instruments of and which this forum has, as a constituent assembly drafting a constitution, the expertise
domination and subservience has clearly been defined these past weeks. and capacity to decide on except that it lacks the political will that brought it to existence and
now engages in an elaborate scheme of buck-passing.
So for the record, Mr. Presiding Officer, I would like to declare my support for the
committee's position to enshrine in the Constitution a fundamental principle forbidding Without any doubt we can establish a new social order in our country, if we reclaim, restore,
foreign military bases, troops or facilities in any part of the Philippine territory as a clear and uphold and defend our national sovereignty. National sovereignty is what the military bases
concrete manifestation of our inherent right to national self-determination, independence issue is all about. It is only the sovereign people exercising their national sovereignty who
and sovereignty. can design an independent course and take full control of their national destiny.359

Mr. Presiding Officer, I would like to relate now these attributes of genuine nationhood to SPEECH OF COMMISSIONER P ADILLA360
the social cost of allowing foreign countries to maintain military bases in our country.
Previous speakers have dwelt on this subject, either to highlight its importance in relation to Mr. Presiding Officer, in advocating the majority committee report, specifically Sections 3
the other issues or to gloss over its significance and !llake this a part of future negotiations. 357 and 4 on neutrality, nuclear and bases-free country, some views stress sovereignty of the
Republic and even invoke survival of the Filipino nation and people.361
Mr. Presiding Officer, I feel that banning foreign military bases is one of the solutions and is
the response of the Filipino people against this condition and other conditions that have REBUTTAL OF COMMISSIONER NOLLEDO362
already been clearly and emphatically discussed in past deliberations. The deletion,
therefore, of Section 3 in the Constitution we are drafting will have the following
The anachronistic and ephemeral arguments against the provisions of the committee report
implications:
to dismantle the American bases after 1991 only show the urgent need to free our country
from the entangling alliance with any power bloc.363
First, the failure of the Constitutional Commission to decisively respond to the continuing
violation of our territorial integrity via the military bases agreement which permits the
xx x Mr. Presiding Officer, it is not necessary for us to possess expertise to know that the so-
retention of U.S. facilities within the Philippine soil over which our authorities have no
called RP-US Bases Agreement will expire in 1991, that it infringes on our sovereignty and
exclusive jurisdiction contrary to the accepted definition of the exercise of sovereignty.
jurisdiction as well as national dignity and honor, that it goes against the UN policy of
disarmament and that it constitutes unjust intervention in our internal affairs.364 (Emphases
Second, consent by this forum, this Constitutional Commission, to an exception in the Supplied)
application of a provision in the Bill of Rights that we have just drafted regarding equal
application of the laws of the land to all inhabitants, permanent or otherwise, within its
The Constitutional Commission eventually agreed to allow foreign military bases, troops, or
territorial boundaries.
facilities, subject to the provisions of Section 25. It is thus important to read its discussions
carefully. From these discussions, we can deduce three legal standards that were articulated
by the Constitutional Commission Members. These are characteristics of any agreement that
the country, and by extension this Court, must ensure are observed. We can thereby
392

determine whether a military base or facility in the Philippines, which houses or is accessed Catholic Apostolic Administrator of Davao, Inc. v. Land Registration Commission, we stated
by foreign military troops, is foreign or remains a Philippine military base or facility. The legal that the constitutional proscription on property ownership is not violated despite the foreign
standards we find applicable are: independence from foreign control, sovereignty and national's control over the property.370
applicable law, and national security and territorial integrity.
EDCA, in respect of its provisions on Agreed Locations, is essentially a contract of use and
i. First standard: independence from foreign control access. Under its pertinent provisions, it is the Designated Authority of the Philippines that
shall, when requested, assist in facilitating transit or access to public land and
Very clearly, much of the opposition to the U.S. bases at the time of the Constitution's facilities.371 The activities carried out within these locations are subject to agreement as
drafting was aimed at asserting Philippine independence from the U.S., as well as control authorized by the Philippine govemment.372 Granting the U.S. operational control over these
over our country's territory and military. locations is likewise subject to EDCA' s security mechanisms, which are bilateral procedures
involving Philippine consent and cooperation.373 Finally, the Philippine Designated Authority
or a duly designated representative is given access to the Agreed Locations.374
Under the Civil Code, there are several aspects of control exercised over property.

To our mind, these provisions do not raise the spectre of U.S. control, which was so feared by
Property is classified as private or public.365 It is public if "intended for public use, such as
the Constitutional Commission. In fact, they seem to have been the product of deliberate
roads, canals, rivers, torrents, ports and bridges constructed by the State, banks, shores,
negotiation from the point of view of the Philippine government, which balanced
roadsteads, and others of similar character[,]" or "[t]hose which belong to the State, without
constitutional restrictions on foreign military bases and facilities against the security needs of
being for public use, and are intended for some public service or for the development of the
the country. In the 1947 MBA, the U.S. forces had "the right, power and authority x x x to
national wealth. "366
construct (including dredging and filling), operate, maintain, utilize, occupy, garrison and
control the bases."375 No similarly explicit provision is present in EDCA.
Quite clearly, the Agreed Locations are contained within a property for public use, be it
within a government military camp or property that belongs to the Philippines.1avvphi1
Nevertheless, the threshold for allowing the presence of foreign military facilities and bases
has been raised by the present Constitution. Section 25 is explicit that foreign military bases,
Once ownership is established, then the rights of ownership flow freely. Article 428 of the troops, or facilities shall not be allowed in the Philippines, except under a treaty duly
Civil Code provides that "[t]he owner has the right to enjoy and dispose of a thing, without concurred in by the Senate. Merely stating that the Philippines would retain ownership
other limitations than those established by law." Moreover, the owner "has also a right of would do violence to the constitutional requirement if the Agreed Locations were simply to
action against the holder and possessor of the thing in order to recover it." become a less obvious manifestation of the U.S. bases that were rejected in 1991.

Philippine civil law therefore accords very strong rights to the owner of property, even When debates took place over the military provisions of the Constitution, the committee
against those who hold the property. Possession, after all, merely raises a disputable rejected a specific provision proposed by Commissioner Sarmiento. The discussion
presumption of ownership, which can be contested through normal judicial processes.367 illuminates and provides context to the 1986 Constitutional Commission's vision of control
and independence from the U.S., to wit:
In this case, EDCA explicitly provides that ownership of the Agreed Locations remains with
the Philippine govemment.368 What U.S. personnel have a right to, pending mutual MR. SARMIENTO: Madam President, my proposed amendment reads as follows: "THE STATE
agreement, is access to and use of these locations.369 SHALL ESTABLISH AND MAINTAIN AN INDEPENDENT AND SELF-RELIANT ARMED FORCES OF
THE PHILIPPINES." Allow me to briefly explain, Madam President. The Armed Forces of the
The right of the owner of the property to allow access and use is consistent with the Civil Philippines is a vital component of Philippine society depending upon its training, orientation
Code, since the owner may dispose of the property in whatever way deemed fit, subject to and support. It will either be the people's protector or a staunch supporter of a usurper or
the limits of the law. So long as the right of ownership itself is not transferred, then whatever tyrant, local and foreign interest. The Armed Forces of the Philippines' past and recent
rights are transmitted by agreement does not completely divest the owner of the rights over experience shows it has never been independent and self-reliant. Facts, data and statistics
the property, but may only limit them in accordance with law. will show that it has been substantially dependent upon a foreign power. In March 1968,
Congressman Barbero, himself a member of the Armed Forces of the Philippines, revealed
Hence, even control over the property is something that an owner may transmit freely. This top secret documents showing what he described as U.S. dictation over the affairs of the
act does not translate into the full transfer of ownership, but only of certain rights. In Roman Armed Forces of the Philippines. He showed that under existing arrangements, the United
States unilaterally determines not only the types and quantity of arms and equipments
393

that our armed forces would have, but also the time when these items are to be made This logic was accepted in Tañada v. Angara, in which the Court ruled that independence
available to us. It is clear, as he pointed out, that the composition, capability and schedule does not mean the absence of foreign participation:
of development of the Armed Forces of the Philippines is under the effective control of the
U.S. government.376 (Emphases supplied) Furthermore, the constitutional policy of a "self-reliant and independent national
economy" does not necessarily rule out the entry of foreign investments, goods and
Commissioner Sarmiento proposed a motherhood statement in the 1987 Constitution that services. It contemplates neither "economic seclusion" nor "mendicancy in the international
would assert "independent" and "self-reliant" armed forces. This proposal was rejected by community." As explained by Constitutional Commissioner Bernardo Villegas, sponsor of this
the committee, however. As Commissioner De Castro asserted, the involvement of the constitutional policy:
Philippine military with the U.S. did not, by itself, rob the Philippines of its real
independence. He made reference to the context of the times: that the limited resources of Economic self reliance is a primary objective of a developing country that is keenly aware of
the Philippines and the current insurgency at that time necessitated a strong military overdependence on external assistance for even its most basic needs. It does not mean
relationship with the U.S. He said that the U.S. would not in any way control the Philippine autarky or economic seclusion; rather, it means avoiding mendicancy in the international
military despite this relationship and the fact that the former would furnish military community. Independence refers to the freedom from undue foreign control of the national
hardware or extend military assistance and training to our military. Rather, he claimed that economy, especially in such strategic industries as in the development of natural resources
the proposal was in compliance with the treaties between the two states. and public utilities.378 (Emphases supplied)

MR. DE CASTRO: If the Commissioner will take note of my speech on U.S. military bases on 12 The heart of the constitutional restriction on foreign military facilities and bases is therefore
September 1986, I spoke on the selfreliance policy of the armed forces. However, due to very the assertion of independence from the U.S. and other foreign powers, as independence is
limited resources, the only thing we could do is manufacture small arms ammunition. We exhibited by the degree of foreign control exerted over these areas.1âwphi1 The essence of
cannot blame the armed forces. We have to blame the whole Republic of the Philippines for that independence is self-governance and self-control.379 Independence itself is "[t]he state
failure to provide the necessary funds to make the Philippine Armed Forces self-reliant. or condition of being free from dependence, subjection, or control. "380
Indeed that is a beautiful dream. And I would like it that way. But as of this time, fighting an
insurgency case, a rebellion in our country - insurgency - and with very limited funds and very
Petitioners assert that EDCA provides the U.S. extensive control and authority over Philippine
limited number of men, it will be quite impossible for the Philippines to appropriate the
facilities and locations, such that the agreement effectively violates Section 25 of the 1987
necessary funds therefor. However, if we say that the U.S. government is furnishing us the
Constitution.381
military hardware, it is not control of our armed forces or of our government. It is in
compliance with the Mutual Defense Treaty. It is under the military assistance program that
it becomes the responsibility of the United States to furnish us the necessary hardware in Under Article VI(3) of EDCA, U.S. forces are authorized to act as necessary for "operational
connection with the military bases agreement. Please be informed that there are three (3) control and defense." The term "operational control" has led petitioners to regard U.S.
treaties connected with the military bases agreement; namely: the RP-US Military Bases control over the Agreed Locations as unqualified and, therefore, total.382 Petitioners contend
Agreement, the Mutual Defense Treaty and the Military Assistance Program. that the word "their" refers to the subject "Agreed Locations."

My dear Commissioner, when we enter into a treaty and we are furnished the military This argument misreads the text, which is quoted below:
hardware pursuant to that treaty, it is not in control of our armed forces nor control of our
government. True indeed, we have military officers trained in the U.S. armed forces school. United States forces are authorized to exercise all rights and authorities within Agreed
This is part of our Military Assistance Program, but it does not mean that the minds of our Locations that are necessary for their operational control or defense, including taking
military officers are for the U.S. government, no. I am one of those who took four courses in appropriate measure to protect United States forces and United States contractors. The
the United States schools, but I assure you, my mind is for the Filipino people. Also, while we United States should coordinate such measures with appropriate authorities of the
are sending military officers to train or to study in U.S. military schools, we are also sending Philippines.
our officers to study in other military schools such as in Australia, England and in Paris. So, it
does not mean that when we send military officers to United States schools or to other A basic textual construction would show that the word "their," as understood above, is a
military schools, we will be under the control of that country. We also have foreign officers in possessive pronoun for the subject "they," a third-person personal pronoun in plural form.
our schools, we in the Command and General Staff College in Fort Bonifacio and in our Thus, "their" cannot be used for a non-personal subject such as "Agreed Locations." The
National Defense College, also in Fort Bonifacio.377 (Emphases supplied) simple grammatical conclusion is that "their" refers to the previous third-person plural noun,
394

which is "United States forces." This conclusion is in line with the definition of operational Petitioners assert that beyond the concept of operational control over personnel, qualifying
control. access to the Agreed Locations by the Philippine Designated Authority with the phrase
"consistent with operational safety and security requirements in accordance with agreed
a. U.S. operational control as the exercise of authority over U.S. personnel, and not over the procedures developed by the Parties" leads to the conclusion that the U.S. exercises effective
Agreed Locations control over the Agreed Locations.389 They claim that if the Philippines exercises possession
of and control over a given area, its representative should not have to be authorized by a
special provision.390
Operational control, as cited by both petitioner and respondents, is a military term referring
to
For these reasons, petitioners argue that the "operational control" in EDCA is the "effective
command and control" in the 1947 MBA.391 In their Memorandum, they distinguish effective
[t]he authority to perform those functions of command over subordinate forces involving
command and control from operational control in U.S. parlance.392 Citing the Doctrine for the
organizing and employing commands and forces, assigning tasks, designating objective, and
Armed Forces of the United States, Joint Publication 1, "command and control (C2)" is
giving authoritative direction necessary to accomplish the mission. 383
defined as "the exercise of authority and direction by a properly designated commander over
assigned and attached forces in the accomplishment of the mission x x x."393 Operational
At times, though, operational control can mean something slightly different. In JUSMAG control, on the other hand, refers to "[t]hose functions of command over assigned forces
Philippines v. National Labor Relations Commission, the Memorandum of Agreement involving the composition of subordinate forces, the assignment of tasks, the designation of
between the AFP and JUSMAG Philippines defined the term as follows:384 objectives, the overall control of assigned resources, and the full authoritative direction
necessary to accomplish the mission."394
The term "Operational Control" includes, but is not limited to, all personnel administrative
actions, such as: hiring recommendations; firing recommendations; position classification; Two things demonstrate the errors in petitioners' line of argument.
discipline; nomination and approval of incentive awards; and payroll computation.
Firstly, the phrase "consistent with operational safety and security requirements in
Clearly, traditional standards define "operational control" as personnel control. Philippine accordance with agreed procedures developed by the Parties" does not add any qualification
law, for instance, deems operational control as one exercised by police officers and civilian beyond that which is already imposed by existing treaties. To recall, EDCA is based upon prior
authorities over their subordinates and is distinct from the administrative control that they treaties, namely the VFA and the MDT.395 Treaties are in themselves contracts from which
also exercise over police subordinates.385 Similarly, a municipal mayor exercises operational rights and obligations may be claimed or waived.396 In this particular case, the Philippines has
control over the police within the municipal government,386 just as city mayor possesses the already agreed to abide by the security mechanisms that have long been in place between
same power over the police within the city government.387 the U.S. and the Philippines based on the implementation of their treaty relations.397

Thus, the legal concept of operational control involves authority over personnel in a Secondly, the full document cited by petitioners contradicts the equation of "operational
commander-subordinate relationship and does not include control over the Agreed Locations control" with "effective command and control," since it defines the terms quite
in this particular case. Though not necessarily stated in EDCA provisions, this interpretation is differently, viz:398
readily implied by the reference to the taking of "appropriate measures to protect United
States forces and United States contractors."
Command and control encompasses the exercise of authority, responsibility, and direction by
a commander over assigned and attached forces to accomplish the mission. Command at all
It is but logical, even necessary, for the U.S. to have operational control over its own forces, levels is the art of motivating and directing people and organizations into action to
in much the same way that the Philippines exercises operational control over its own units. accomplish missions. Control is inherent in command. To control is to manage and direct
forces and functions consistent with a commander's command authority. Control of forces
For actual operations, EDCA is clear that any activity must be planned and pre-approved by and functions helps commanders and staffs compute requirements, allocate means, and
the MDB-SEB.388 This provision evinces the partnership aspect of EDCA, such that both integrate efforts. Mission command is the preferred method of exercising C2. A complete
stakeholders have a say on how its provisions should be put into effect. discussion of tenets, organization, and processes for effective C2 is provided in Section B,
"Command and Control of Joint Forces," of Chapter V "Joint Command and Control."
b. Operational control vis-à-vis effective command and control
Operational control is defined thus:399
395

OPCON is able to be delegated from a lesser authority than COCOM. It is the authority to operational control over the Agreed Locations for construction activities, then it is quite
perform those functions of command over subordinate forces involving organizing and logical to conclude that it is not exercised over other activities.
employing commands and forces, assigning tasks, designating objectives, and giving
authoritative direction over all aspects of military operations and joint training necessary to Limited control does not violate the Constitution. The fear of the commissioners was total
accomplish the mission. It should be delegated to and exercised by the commanders of control, to the point that the foreign military forces might dictate the terms of their acts
subordinate organizations; normally, this authority is exercised through subordinate JFCs, within the Philippines.404 More important, limited control does not mean an abdication or
Service, and/or functional component commanders. OPCON provides authority to organize derogation of Philippine sovereignty and legal jurisdiction over the Agreed Locations. It is
and employ commands and forces as the commander considers necessary to accomplish more akin to the extension of diplomatic courtesies and rights to diplomatic agents,405 which
assigned missions. It does not include authoritative direction for logistics or matters of is a waiver of control on a limited scale and subject to the terms of the treaty.
administration, discipline, internal organization, or unit training. These elements of COCOM
must be specifically delegated by the CCDR. OPCON does include the authority to delineate
This point leads us to the second standard envisioned by the framers of the Constitution: that
functional responsibilities and operational areas of subordinate JFCs.
the Philippines must retain sovereignty and jurisdiction over its territory.

Operational control is therefore the delegable aspect of combatant command, while


ii. Second standard: Philippine sovereignty and applicable law
command and control is the overall power and responsibility exercised by the commander
with reference to a mission. Operational control is a narrower power and must be given,
while command and control is plenary and vested in a commander. Operational control does EDCA states in its Preamble the "understanding for the United States not to establish a
not include the planning, programming, budgeting, and execution process input; the permanent military presence or base in the territory of the Philippines." Further on, it
assignment of subordinate commanders; the building of relationships with Department of likewise states the recognition that "all United States access to and use of facilities and areas
Defense agencies; or the directive authority for logistics, whereas these factors are included will be at the invitation of the Philippines and with full respect for the Philippine Constitution
in the concept of command and control.400 and Philippine laws."

This distinction, found in the same document cited by petitioners, destroys the very The sensitivity of EDCA provisions to the laws of the Philippines must be seen in light of
foundation of the arguments they have built: that EDCA is the same as the MBA. Philippine sovereignty and jurisdiction over the Agreed Locations.

c. Limited operational control over the Agreed Locations only for construction activitites Sovereignty is the possession of sovereign power,406 while jurisdiction is the conferment by
law of power and authority to apply the law.407 Article I of the 1987 Constitution states:
As petitioners assert, EDCA indeed contains a specific provision that gives to the U.S.
operational control within the Agreed Locations during construction activities. 401 This The national territory comprises the Philippine archipelago, with all the islands and waters
exercise of operational control is premised upon the approval by the MDB and the SEB of the embraced therein, and all other territories over which the Philippines has sovereignty or
construction activity through consultation and mutual agreement on the requirements and jurisdiction, consisting of its terrestrial, fluvial, and aerial domains, including its territorial
standards of the construction, alteration, or improvement.402 sea, the seabed, the subsoil, the insular shelves, and other submarine areas. The waters
around, between, and connecting the islands of the archipelago, regardless of their breadth
and dimensions, form part of the internal waters of the Philippines. (Emphasis supplied)
Despite this grant of operational control to the U.S., it must be emphasized that the grant is
only for construction activities. The narrow and limited instance wherein the U.S. is given
operational control within an Agreed Location cannot be equated with foreign military From the text of EDCA itself, Agreed Locations are territories of the Philippines that the U.S.
control, which is so abhorred by the Constitution. forces are allowed to access and use.408 By withholding ownership of these areas and
retaining unrestricted access to them, the government asserts sovereignty over its territory.
That sovereignty exists so long as the Filipino people exist.409
The clear import of the provision is that in the absence of construction activities, operational
control over the Agreed Location is vested in the Philippine authorities. This meaning is
implicit in the specific grant of operational control only during construction activities. The Significantly, the Philippines retains primary responsibility for security with respect to the
principle of constitutional construction, "expressio unius est exclusio alterius," means the Agreed Locations.410Hence, Philippine law remains in force therein, and it cannot be said that
failure to mention the thing becomes the ground for inferring that it was deliberately jurisdiction has been transferred to the U.S. Even the previously discussed necessary
excluded.403Following this construction, since EDCA mentions the existence of U.S. measures for operational control and defense over U.S. forces must be coordinated with
Philippine authorities.411
396

Jurisprudence bears out the fact that even under the former legal regime of the MBA, inconsistent with the Purposes of the United Nations."418 Any unlawful attack on the
Philippine laws continue to be in force within the bases.412 The difference between then and Philippines breaches the treaty, and triggers Article 51 of the same charter, which guarantees
now is that EDCA retains the primary jurisdiction of the Philippines over the security of the the inherent right of individual or collective self-defence.
Agreed Locations, an important provision that gives it actual control over those locations.
Previously, it was the provost marshal of the U.S. who kept the peace and enforced Moreover, even if the lawfulness of the attack were not in question, international
Philippine law in the bases. In this instance, Philippine forces act as peace officers, in stark humanitarian law standards prevent participants in an armed conflict from targeting non-
contrast to the 1947 MBA provisions on jurisdiction.413 participants. International humanitarian law, which is the branch of international law
applicable to armed conflict, expressly limits allowable military conduct exhibited by forces of
iii. Third standard: must respect national security and territorial integrity a participant in an armed conflict.419 Under this legal regime, participants to an armed
conflict are held to specific standards of conduct that require them to distinguish between
The last standard this Court must set is that the EDCA provisions on the Agreed Locations combatants and non-combatants,420 as embodied by the Geneva Conventions and their
must not impair or threaten the national security and territorial integrity of the Philippines. Additional Protocols.421

This Court acknowledged in Bayan v. Zamora that the evolution of technology has essentially Corollary to this point, Professor John Woodcliffe, professor of international law at the
rendered the prior notion of permanent military bases obsolete. University of Leicester, noted that there is no legal consensus for what constitutes a base, as
opposed to other terms such as "facilities" or "installation."422 In strategic literature, "base" is
defined as an installation "over which the user State has a right to exclusive control in an
Moreover, military bases established within the territory of another state is no longer viable
extraterritorial sense."423 Since this definition would exclude most foreign military
because of the alternatives offered by new means and weapons of warfare such as nuclear
installations, a more important distinction must be made.
weapons, guided missiles as well as huge sea vessels that can stay afloat in the sea even for
months and years without returning to their home country. These military warships are
actually used as substitutes for a land-home base not only of military aircraft but also of For Woodcliffe, a type of installation excluded from the definition of "base" is one that does
military personnel and facilities. Besides, vessels are mobile as compared to a land-based not fulfill a combat role. He cites an example of the use of the territory of a state for training
military headquarters.414 purposes, such as to obtain experience in local geography and climactic conditions or to carry
out joint exercises.424 Another example given is an advanced communications technology
installation for purposes of information gathering and communication.425 Unsurprisingly, he
The VFA serves as the basis for the entry of U.S. troops in a limited scope. It does not allow,
deems these non-combat uses as borderline situations that would be excluded from the
for instance, the re-establishment of the Subic military base or the Clark Air Field as U.S.
functional understanding of military bases and installations.426
military reservations. In this context, therefore, this Court has interpreted the restrictions on
foreign bases, troops, or facilities as three independent restrictions. In accord with this
interpretation, each restriction must have its own qualification. By virtue of this ambiguity, the laws of war dictate that the status of a building or person is
presumed to be protected, unless proven otherwise.427 Moreover, the principle of distinction
requires combatants in an armed conflict to distinguish between lawful targets428 and
Petitioners quote from the website http://en.wikipedia.org to define what a military base
protected targets.429 In an actual armed conflict between the U.S. and a third state, the
is.415 While the source is not authoritative, petitioners make the point that the Agreed
Agreed Locations cannot be considered U.S. territory, since ownership of territory even in
Locations, by granting access and use to U.S. forces and contractors, are U.S. bases under a
times of armed conflict does not change.430
different name.416 More important, they claim that the Agreed Locations invite instances of
attack on the Philippines from enemies of the U.S.417
Hence, any armed attack by forces of a third state against an Agreed Location can only be
legitimate under international humanitarian law if it is against a bona fide U.S. military base,
We believe that the raised fear of an attack on the Philippines is not in the realm of law, but
facility, or installation that directly contributes to the military effort of the U.S. Moreover, the
of politics and policy. At the very least, we can say that under international law, EDCA does
third state's forces must take all measures to ensure that they have complied with the
not provide a legal basis for a justified attack on the Philippines.
principle of distinction (between combatants and non-combatants).

In the first place, international law disallows any attack on the Agreed Locations simply
There is, then, ample legal protection for the Philippines under international law that would
because of the presence of U.S. personnel. Article 2(4) of the United Nations Charter states
ensure its territorial integrity and national security in the event an Agreed Location is
that "All Members shall refrain in their international relations from the threat or use of force
subjected to attack. As EDCA stands, it does not create the situation so feared by petitioners -
against the territorial integrity or political independence of any state, or in any other manner
397

one in which the Philippines, while not participating in an armed conflict, would construction and use of temporary structures incident to operations support), training
be legitimately targeted by an enemy of the U.S.431 services, x x x storage services, x x x during an approved activity." 443 These logistic supplies,
support, and services include temporary use of "nonlethal items of military equipment which
In the second place, this is a policy question about the wisdom of allowing the presence of are not designated as significant military equipment on the U.S. Munitions List, during an
U.S. personnel within our territory and is therefore outside the scope of judicial review. approved activity."444 The first Mutual Logistics Support Agreement has lapsed, while the
second one has been extended until 2017 without any formal objection before this Court
from the Senate or any of its members.
Evidently, the concept of giving foreign troops access to "agreed" locations, areas, or facilities
within the military base of another sovereign state is nothing new on the international plane.
In fact, this arrangement has been used as the framework for several defense cooperation The provisions in EDCA dealing with Agreed Locations are analogous to those in the
agreements, such as in the following: aforementioned executive agreements. Instead of authorizing the building of temporary
structures as previous agreements have done, EDCA authorizes the U.S. to build permanent
structures or alter or improve existing ones for, and to be owned by, the Philippines. 445 EDCA
1. 2006 U.S.-Bulgaria Defense Cooperation Agreement432
is clear that the Philippines retains ownership of altered or improved facilities and newly
constructed permanent or non-relocatable structures.446 Under EDCA, U.S. forces will also be
2. 2009 U.S.-Colombia Defense Cooperation Agreement433 allowed to use facilities and areas for "training; x x x; support and related activities; x x x;
temporary accommodation of personnel; communications" and agreed activities.447
3. 2009 U.S.-Poland Status of Forces Agreement434
Concerns on national security problems that arise from foreign military equipment being
4. 2014 U.S.-Australia Force Posture Agreement435 present in the Philippines must likewise be contextualized. Most significantly, the VFA
already authorizes the presence of U.S. military equipment in the country. Article VII of the
5. 2014 U.S.-Afghanistan Security and Defense Cooperation Agreement436 VFA already authorizes the U.S. to import into or acquire in the Philippines "equipment,
materials, supplies, and other property" that will be used "in connection with activities"
contemplated therein. The same section also recognizes that "[t]itle to such property shall
In all of these arrangements, the host state grants U.S. forces access to their military remain" with the US and that they have the discretion to "remove such property from the
bases.437 That access is without rental or similar costs to the U.S.438 Further, U.S. forces are Philippines at any time."
allowed to undertake construction activities in, and make alterations and improvements to,
the agreed locations, facilities, or areas.439 As in EDCA, the host states retain ownership and
jurisdiction over the said bases.440 There is nothing novel, either, in the EDCA provision on the prepositioning and storing of
"defense equipment, supplies, and materiel,"448 since these are sanctioned in the VFA. In
fact, the two countries have already entered into various implementing agreements in the
In fact, some of the host states in these agreements give specific military-related rights to the past that are comparable to the present one. The Balikatan 02-1 Terms of Reference
U.S. For example, under Article IV(l) of the US.-Bulgaria Defense Cooperation mentioned in Lim v. Executive Secretary specifically recognizes that Philippine and U.S. forces
Agreement, "the United States forces x x x are authorized access to and may use agreed "may share x x x in the use of their resources, equipment and other assets." Both the 2002
facilities and areas x x x for staging and deploying of forces and materiel, with the purpose of and 2007 Mutual Logistics Support Agreements speak of the provision of support and
conducting x x x contingency operations and other missions, including those undertaken in services, including the "construction and use of temporary structures incident to operations
the framework of the North Atlantic Treaty." In some of these agreements, host countries support" and "storage services" during approved activities.449 These logistic supplies,
allow U.S. forces to construct facilities for the latter’s exclusive use.441 support, and services include the "temporary use of x x x nonlethal items of military
equipment which are not designated as significant military equipment on the U.S. Munitions
Troop billeting, including construction of temporary structures, is nothing new. In Lim v. List, during an approved activity."450Those activities include "combined exercises and
Executive Secretary, the Court already upheld the Terms of Reference of Balikatan 02- training, operations and other deployments" and "cooperative efforts, such as humanitarian
1, which authorized U.S. forces to set up "[t]emporary structures such as those for troop assistance, disaster relief and rescue operations, and maritime anti-pollution operations"
billeting, classroom instruction and messing x x x during the Exercise." Similar provisions are within or outside Philippine territory.451 Under EDCA, the equipment, supplies, and materiel
also in the Mutual Logistics Support Agreement of 2002 and 2007, which are essentially that will be prepositioned at Agreed Locations include "humanitarian assistance and disaster
executive agreements that implement the VFA, the MDT, and the 1953 Military Assistance relief equipment, supplies, and materiel. "452 Nuclear weapons are specifically excluded from
Agreement. These executive agreements similarly tackle the "reciprocal provision of logistic the materiel that will be prepositioned.
support, supplies, and services,"442 which include "[b ]illeting, x x x operations support (and
398

Therefore, there is no basis to invalidate EDCA on fears that it increases the threat to our preferential use of Philippine suppliers.461 Hence, the provision on the assumption of tax
national security. If anything, EDCA increases the likelihood that, in an event requiring a liability does not constitute a tax exemption as petitioners have posited.
defensive response, the Philippines will be prepared alongside the U.S. to defend its islands
and insure its territorial integrity pursuant to a relationship built on the MDT and VFA. Additional issues were raised by petitioners, all relating principally to provisions already
sufficiently addressed above. This Court takes this occasion to emphasize that the agreement
8. Others issues and concerns raised has been construed herein as to absolutely disauthorize the violation of the Constitution or
any applicable statute. On the contrary, the applicability of Philippine law is explicit in EDCA.
A point was raised during the oral arguments that the language of the MDT only refers to
mutual help and defense in the Pacific area.453 We believe that any discussion of the EPILOGUE
activities to be undertaken under EDCA vis-a-vis the defense of areas beyond the Pacific is
premature. We note that a proper petition on that issue must be filed before we rule The fear that EDCA is a reincarnation of the U.S. bases so zealously protested by noted
thereon. We also note that none of the petitions or memoranda has attempted to discuss personalities in Philippine history arises not so much from xenophobia, but from a genuine
this issue, except only to theorize that the U.S. will not come to our aid in the event of an desire for self-determination, nationalism, and above all a commitment to ensure the
attack outside of the Pacific. This is a matter of policy and is beyond the scope of this judicial independence of the Philippine Republic from any foreign domination.
review.
Mere fears, however, cannot curtail the exercise by the President of the Philippines of his
In reference to the issue on telecommunications, suffice it to say that the initial impression of Constitutional prerogatives in respect of foreign affairs. They cannot cripple him when he
the facility adverted to does appear to be one of those that require a public franchise by way deems that additional security measures are made necessary by the times. As it stands, the
of congressional action under Section 11, Article XII of the Constitution. As respondents Philippines through the Department of Foreign Affairs has filed several diplomatic protests
submit, however, the system referred to in the agreement does not provide against the actions of the People's Republic of China in the West Philippine Sea;462 initiated
telecommunications services to the public for compensation. 454 It is clear from Article VIl(2) arbitration against that country under the United Nations Convention on the Law of the
of EDCA that the telecommunication system is solely for the use of the U.S. and not the Sea;463 is in the process of negotiations with the Moro Islamic Liberation Front for peace in
public in general, and that this system will not interfere with that which local operators use. Southern Philippines,464 which is the subject of a current case before this Court; and faces
Consequently, a public franchise is no longer necessary. increasing incidents of kidnappings of Filipinos and foreigners allegedly by the Abu Sayyaf or
the New People's Army.465 The Philippine military is conducting reforms that seek to ensure
Additionally, the charge that EDCA allows nuclear weapons within Philippine territory is the security and safety of the nation in the years to come.466 In the future, the Philippines
entirely speculative. It is noteworthy that the agreement in fact specifies that the must navigate a world in which armed forces fight with increasing sophistication in both
prepositioned materiel shall not include nuclear weapons.455Petitioners argue that only strategy and technology, while employing asymmetric warfare and remote weapons.
prepositioned nuclear weapons are prohibited by EDCA; and that, therefore, the U.S. would
insidiously bring nuclear weapons to Philippine territory.456 The general prohibition on Additionally, our country is fighting a most terrifying enemy: the backlash of Mother Nature.
nuclear weapons, whether prepositioned or not, is already expressed in the 1987 The Philippines is one of the countries most directly affected and damaged by climate
Constitution.457 It would be unnecessary or superfluous to include all prohibitions already in change. It is no coincidence that the record-setting tropical cyclone Yolanda (internationally
the Constitution or in the law through a document like EDCA. named Haiyan), one of the most devastating forces of nature the world has ever seen hit the
Philippines on 8 November 2013 and killed at least 6,000 people.467 This necessitated a
Finally, petitioners allege that EDCA creates a tax exemption, which under the law must massive rehabilitation project.468 In the aftermath, the U.S. military was among the first to
originate from Congress. This allegation ignores jurisprudence on the government's extend help and support to the Philippines.
assumption of tax liability. EDCA simply states that the taxes on the use of water, electricity,
and public utilities are for the account of the Philippine Government.458 This provision That calamity brought out the best in the Filipinos as thousands upon thousands volunteered
creates a situation in which a contracting party assumes the tax liability of the their help, their wealth, and their prayers to those affected. It also brought to the fore the
other.459 In National Power Corporation v. Province of Quezon, we distinguished between value of having friends in the international community.
enforceable and unenforceable stipulations on the assumption of tax liability. Afterwards, we
concluded that an enforceable assumption of tax liability requires the party assuming the
In order to keep the peace in its archipelago in this region of the world, and to sustain itself
liability to have actual interest in the property taxed.460 This rule applies to EDCA, since the
at the same time against the destructive forces of nature, the Philippines will need friends.
Philippine Government stands to benefit not only from the structures to be built thereon or
Who they are, and what form the friendships will take, are for the President to decide. The
improved, but also from the joint training with U.S. forces, disaster preparation, and the
399

only restriction is what the Constitution itself expressly prohibits. It appears that this must specifically show that they have sufficient interest in preventing the illegal expenditure
overarching concern for balancing constitutional requirements against the dictates of of public money, and that they will sustain a direct injury as a result of the enforcement of
necessity was what led to EDCA. the assailed act. Applying that principle to this case, they must establish that EDCA involves
the exercise by Congress of its taxing or spending powers. A reading of the EDCA, however,
As it is, EDCA is not constitutionally infirm. As an executive agreement, it remains consistent would show that there has been neither an appropriation nor an authorization of
with existing laws and treaties that it purports to implement. disbursement.
ISSUE 3:W/N the petitions qualify as “legislator’s suit”
WHEREFORE, we hereby DISMISS the petitions. No. The power to concur in a treaty or an international agreement is an institutional
prerogative granted by the Constitution to the Senate. In a legislator’s suit, the injured party
SO ORDERED. would be the Senate as an institution or any of its incumbent members, as it is the Senate’s
constitutional function that is allegedly being violated. Here, none of the petitioners, who are
GR 212426 Jan 12, 2016 former senators, have the legal standing to maintain the suit.

FACTS: ISSUE 4:W/N the SC may exercise its Power of Judicial Review over the case

Petitioners, as citizens, taxpayers and former legislators, questioned before the SC the Yes. Although petitioners lack legal standing, they raise matters of transcendental
constitutionality of EDCA (Enhanced Defense Cooperation Agreement), an agreement importance which justify setting aside the rule on procedural technicalities. The challenge
entered into by the executive department with the US and ratified on June 6, 2014. Under raised here is rooted in the very Constitution itself, particularly Art XVIII, Sec 25 thereof,
the EDCA, the PH shall provide the US forces the access and use of portions of PH territory, which provides for a stricter mechanism required before any foreign military bases, troops or
which are called Agreed Locations. Aside from the right to access and to use the Agreed facilities may be allowed in the country. Such is of paramount public interest that the Court is
Locations, the US may undertake the following types of activities within the Agreed behooved to determine whether there was grave abuse of discretion on the part of the
Locations: security cooperation exercises; joint and combined training activities; Executive Department.
humanitarian and disaster relief activities; and such other activities that as may be agreed
upon by the parties.
Mainly, petitioners posit that the use of executive agreement as medium of agreement with
US violated the constitutional requirement of Art XVIII, Sec 25 since the EDCA involves
foreign military bases, troops and facilities whose entry into the country should be covered
by a treaty concurred in by the Senate. The Senate, through Senate Resolution 105, also
expressed its position that EDCA needs congressional ratification.
ISSUE 1:W/N the petitions as “citizen’s suit” satisfy the requirements of legal standing in
assailing the constitutionality of EDCA
No. In assailing the constitutionality of a governmental act, petitioners suing as citizens may
dodge the requirement of having to establish a direct and personal interest if they show that
the act affects a public right. But here, aside from general statements that the petitions
involve the protection of a public right, and that their constitutional rights as citizens would
be violated, the petitioners failed to make any specific assertion of a particular public right
that would be violated by the enforcement of EDCA. For their failure to do so, the present
petitions cannot be considered by the Court as citizens’ suits that would justify a disregard of
the aforementioned requirements.
ISSUE 2:W/N the petitioners have legal standing as “taxpayers”
No. Petitioners cannot sue as taxpayers because EDCA is neither meant to be a tax measure,
nor is it directed at the disbursement of public funds.
A taxpayer’s suit concerns a case in which the official act complained of directly involves the
illegal disbursement of public funds derived from taxation. Here, those challenging the act
400

EN BANC the Secretary of Justice to institute criminal and administrative cases against
SPOUSES RENATO CONSTANTINO, JR. and
respondents for acts which circumvent or negate the provisions Art. XII of the Constitution.[1]
LOURDES CONSTANTINO. Petitioners,
Parties and Facts
- versus -

Respondents. October 13, 2005 The petition was filed on 17 July 1992 by petitioners spouses Renato Constantino, Jr. and

Lourdes Constantino and their minor children, Renato Redentor, Anna Marika Lissa, Nina
DECISION
Elissa, and Anna Karmina, Filomeno Sta. Ana III, and the Freedom from Debt Coalition, a non-
TINGA, J.:
stock, non-profit, non-government organization that advocates a pro-people and just

Philippine debt policy.[2] Named respondents were the then Governor of the Bangko Sentral
The quagmire that is the foreign debt problem has especially confounded developing nations
ng Pilipinas, the Secretary of Finance, the National Treasurer, and the Philippine Debt
around the world for decades. It has defied easy solutions acceptable both to debtor
Negotiation Chairman Emmanuel V. Pelaez.[3] All respondents were members of the
countries and their creditors. It has also emerged as cause celebre for various political
Philippine panel tasked to negotiate with the countrys foreign creditors pursuant to the
movements and grassroots activists and the wellspring of much scholarly thought and
Financing Program.
debate.

The operative facts are sparse and there is little need to elaborate on them.
The present petition illustrates some of the ideological and functional differences between

experts on how to achieve debt relief. However, this being a court of law, not an academic
The Financing Program was the culmination of efforts that began during the term of former
forum or a convention on development economics, our resolution has to hinge on the
President Corazon Aquino to manage the countrys external debt problem through a
presented legal issues which center on the appreciation of the constitutional provision that
negotiation-oriented debt strategy involving cooperation and negotiation with foreign
empowers the President to contract and guarantee foreign loans. The ultimate choice is
creditors.[4]Pursuant to this strategy, the Aquino government entered into three
between a restrictive reading of the constitutional provision and an alimentative application
restructuring agreements with representatives of foreign creditor governments during the
thereof consistent with time-honored principles on executive power and the alter
period of 1986 to 1991.[5] During the same period, three similarly-oriented restructuring
ego doctrine.
agreements were executed with commercial bank creditors.[6]

This Petition for Certiorari, Prohibition and Mandamus assails said contracts which were
On 28 February 1992, the Philippine Debt Negotiating Team, chaired by respondent Pelaez,
entered into pursuant to the Philippine Comprehensive Financing Program for 1992
negotiated an agreement with the countrys Bank Advisory Committee, representing all
(Financing Program or Program). It seeks to enjoin respondents from executing additional
foreign commercial bank creditors, on the Financing Program which respondents
debt-relief contracts pursuant thereto. It also urges the Court to issue an order compelling
characterized as a multi-option financing package.[7] The Program was scheduled to be
401

executed on 24 July 1992 by respondents in behalf of the Republic. Nonetheless, petitioners Philippine debts with an interest reduction bond; or to exchange the same Philippine debts

alleged that even prior to the execution of the Program respondents had already with a principal collateralized interest reduction bond.[15]

implemented its buyback component when on 15 May 1992, the Philippines bought
Issues for Resolution
back P1.26 billion of external debts pursuant to the Program.[8]

Petitioners raise several issues before this Court.


The petition sought to enjoin the ratification of the Program, but the Court did not issue any

injunctive relief. Hence, it came to pass that the Program was signed in London as scheduled. First, they object to the debt-relief contracts entered into pursuant to the Financing Program
The petition still has to be resolved though as petitioners seek the annulment of as beyond the powers granted to the President under Section 20,

any and all acts done by respondents, their subordinates and any other public officer Article VII of the Constitution.[16] The provision states that the President may contract or
pursuant to the agreement and program in question.[9]Even after the signing of the guarantee foreign loans in behalf of the Republic. It is claimed that the buyback and
Program, respondents themselves acknowledged that the remaining principal objective of securitization/bond conversion schemes are neither loans nor guarantees, and hence beyond
the petition is to set aside respondents actions.[10] the power of the President to execute.

Petitioners characterize the Financing Program as a package offered to the countrys foreign Second, according to petitioners even assuming that the contracts under the Financing
creditors consisting of two debt-relief options.[11] The first option was a cash buyback of Program are constitutionally permissible, yet it is only the President who may exercise the
portions of the Philippine foreign debt at a discount. [12] The second option allowed creditors power to enter into these contracts and such power may not be delegated to respondents.
to convert existing Philippine debt instruments into any of three kinds of
Third, petitioners argue that the Financing Program violates several constitutional policies
bonds/securities: (1) new money bonds with a five-year grace period and 17 years final
and that contracts executed or to be executed pursuant thereto were or will be done by
maturity, the purchase of which would allow the creditors to convert their eligible debt
respondents with grave abuse of discretion amounting to lack or excess of jurisdiction.
papers into bearer bonds with the same terms; (2) interest-reduction bonds with a maturity

of 25 years; and (3) principal-collateralized interest-reduction bonds with a maturity of 25 Petitioners contend that the Financing Program was made available for debts that were
years.[13] either fraudulently contracted or void. In this regard, petitioners rely on a 1992 Commission

on Audit (COA) report which identified several behest loans as either contracted or
On the other hand, according to respondents the Financing Program would cover about U.S.
guaranteed fraudulently during the Marcos regime. [17] They posit that since these and other
$5.3 billion of foreign commercial debts and it was expected to deal comprehensively with
similar debts, such as the ones pertaining to the Bataan Nuclear Power Plant, [18] were eligible
the commercial bank debt problem of the country and pave the way for the countrys access
for buyback or conversion under the Program, the resultant relief agreements pertaining
to capital markets.[14] They add that the Program carried three basic options from which

foreign bank lenders could choose, namely: to lend money, to exchange existing restructured
402

thereto would be void for being waivers of the Republics right to repudiate the void or disbursed, or that public money is being deflected to any improper purpose, or that there is a

fraudulently contracted loans. wastage of public funds through the enforcement of an invalid or unconstitutional law.[22]

For their part, respondents dispute the points raised by petitioners. They also question the Moreover, a ruling on the issues of this case will not only determine the validity or invalidity

standing of petitioners to institute the present petition and the justiciability of the issues of the subject pre-termination and bond-conversion of foreign debts but also create a

presented. precedent for other debts or debt-related contracts executed or to be executed in behalf of

the President of the Philippines by the Secretary of Finance. Considering the reported
The Court shall tackle the procedural questions ahead of the substantive issues.
Philippine debt of P3.80 trillion as of November 2004, the foreign public borrowing

The Courts Rulings component of which reached P1.81 trillion in November, equivalent to 47.6% of total

government borrowings,[23] the importance of the issues raised and the magnitude of the
Standing of Petitioners
public interest involved are indubitable.

The individual petitioners are suing as citizens of the Philippines; those among them who are
Thus, the Courts cognizance of this petition is also based on the consideration that the
of age are suing in their additional capacity as taxpayers.[19] It is not indicated in what
determination of the issues presented will have a bearing on the state of the countrys
capacity the Freedom from Debt Coalition is suing.
economy, its international financial ratings, and perhaps even the Filipinos way of life. Seen

Respondents point out that petitioners have no standing to file the present suit since the rule in this light, the transcendental importance of the issues herein presented cannot be

allowing taxpayers to assail executive or legislative acts has been applied only to cases where doubted.

the constitutionality of a statute is involved. At the same time, however, they urge this Court
Where constitutional issues are properly raised in the context of alleged facts, procedural
to exercise its wide discretion and waive petitioners lack of standing. They invoke the
questions acquire a relatively minor significance.[24] We thus hold that by the very nature of
transcendental importance of resolving the validity of the questioned debt-relief contracts
the power wielded by the President, the effect of using this power on the economy, and the
and others of similar import.
well-being in general of the Filipino nation, the Court must set aside the procedural barrier of

The recent trend on locus standi has veered towards a liberal treatment in taxpayers standing and rule on the justiciable issues presented by the parties.

suits. In Tatad v. Garcia Jr.,[20] this Court reiterated that the prevailing doctrines in taxpayers
Ripeness/Actual Case Dimension
suits are to allow taxpayers to question contracts entered into by the national government or

government owned and controlled corporations allegedly in contravention of Even as respondents concede the transcendental importance of the issues at bar, in

law.[21] A taxpayer is allowed to sue where there is a claim that public funds are illegally their Rejoinder they ask this Court to dismiss the Petition. Allegedly, petitioners arguments
403

are mere attempts at abstraction.[25] Respondents are correct to some degree. Several issues, community its resolve not to recognize a certain set of illegitimate loans, adverse

as shall be discussed in due course, are not ripe for adjudication. repercussions[27] would come into play. Dr. Felipe Medalla, former Director General of the

National Economic Development Authority, has warned, thus:


The allegation that respondents waived the Philippines right to repudiate void and
One way to reduce debt service is to repudiate debts, totally or
fraudulently contracted loans by executing the debt-relief agreements is, on many levels, not
selectively. Taken to its limit, however, such a strategy would put
justiciable. the Philippines at such odds with too many enemies. Foreign commercial
banks by themselves and without the cooperation of creditor
In the first place, records do not show whether the so-called behest loansor other allegedly governments, especially the United States, may not be in a position to
inflict much damage, but concerted sanctions from commercial banks,
void or fraudulently contracted loans for that matterwere subject of the debt-relief contracts multilateral financial institutions and creditor governments would affect
entered into under the Financing Program. not only our sources of credit but also our access to markets for our
exports and the level of development assistance. . . . [T]he country might
face concerted sanctions even if debts were repudiated only selectively.
Moreover, asserting a right to repudiate void or fraudulently contracted loans begs the

question of whether indeed particular loans are void or fraudulently contracted. Fraudulently The point that must be stressed is that repudiation is not an attractive
alternative if net payments to creditors in the short and medium-run can
contracted loans are voidable and, as such, valid and enforceable until annulled by the be reduced through an agreement (as opposed to a unilaterally set ceiling
courts. On the other hand, void contracts that have already been fulfilled must be declared on debt service payments) which provides for both rescheduling of
principal and capitalization of interest, or its equivalent in new loans,
void in view of the maxim that no one is allowed to take the law in his own which would make it easier for the country to pay interest.[28]
hands.[26] Petitioners theory depends on a prior annulment or declaration of nullity of the

pre-existing loans, which thus far have not been submitted to this Court. Additionally, void Sovereign default is not new to the Philippine setting. In October 1983, the Philippines

contracts are unratifiable by their very nature; they are null and void ab initio. Consequently, declared a moratorium on principal payments on its external debts that eventually lasted

from the viewpoint of civil law, what petitioners present as the Republics right to repudiate is four years,[29] that virtually closed the countrys access to new foreign money[30] and drove

yet a contingent right, one which cannot be allowed as an anticipatory basis for annulling the investors to leave the Philippine market, resulting in some devastating consequences.[31] It

debt-relief contracts. Petitioners contention that the debt-relief agreements are tantamount would appear then that this beguilingly attractive and dangerously simplistic solution

to waivers of the Republics right to repudiate so-called behest loans is without legal deserves the utmost circumspect cogitation before it is resorted to.

foundation.
In any event, the discretion on the matter lies not with the courts but with the executive.

It may not be amiss to recognize that there are many advocates of the position that the Thus, the Program was conceptualized as an offshoot of the decision made by then President

Republic should renege on obligations that are considered as illegitimate. However, should Aquino that the Philippines should recognize its sovereign debts[32] despite the controversy

the executive branch unilaterally, and possibly even without prior court determination of the that engulfed many debts incurred during the Marcos era. It is a scheme whereby the

validity or invalidity of these contracts, repudiate or otherwise declare to the international


404

would have the effect of increasing the foreign debt, and containing
Philippines restructured its debts following a negotiated approach instead of a default
other matters as may be provided by law.
approach to manage the bleak Philippine debt situation.
On Bond-conversion
As a final point, petitioners have no real basis to fret over a possible waiver of the right to

repudiate void contracts. Even assuming that spurious loans had become the subject of debt- Loans are transactions wherein the owner of a property allows another party to use the

relief contracts, respondents unequivocally assert that the Republic did not waive any right property and where customarily, the latter promises to return the property after a specified

to repudiate void or fraudulently contracted loans, it having incorporated a no-waiver clause period with payment for its use, called interest.[34] On the other hand, bonds are interest-

in the agreements.[33] bearing or discounted government or corporate securities that obligate the issuer to pay the

bondholder a specified sum of money, usually at specific intervals, and to repay the principal
Substantive Issues
amount of the loan at maturity.[35] The word bond means contract, agreement, or guarantee.

It is helpful to put the matter in perspective before moving on to the merits. The Financing All of these terms are applicable to the securities known as bonds. An investor who

Program extinguished portions of the countrys pre-existing loans through either debt purchases a bond is lending money to the issuer, and the bond represents the issuers

buyback or bond-conversion. The buyback approach essentially pre-terminated portions of contractual promise to pay interest and repay principal according to specific terms. A short-

public debts while the bond-conversion scheme extinguished public debts through the term bond is often called a note.[36]

obtention of a new loan by virtue of a sovereign bond issuance, the proceeds of which in turn

were used for terminating the original loan. The language of the Constitution is simple and clear as it is broad. It allows the President to

contract and guarantee foreign loans. It makes no prohibition on the issuance of certain
First Issue: The Scope of Section 20, Article VII
kinds of loans or distinctions as to which kinds of debt instruments are more onerous than

For their first constitutional argument, petitioners submit that the buyback and bond- others. This Court may not ascribe to the Constitution meanings and restrictions that would

conversion schemes do not constitute the loan contract or guarantee contemplated in the unduly burden the powers of the President. The plain, clear and unambiguous language of

Constitution and are consequently prohibited. Sec. 20, Art. VII of the Constitution the Constitution should be construed in a sense that will allow the full exercise of the power

provides, viz: provided therein. It would be the worst kind of judicial legislation if the courts were to

misconstrue and change the meaning of the organic act.


The President may contract or guarantee foreign loans in behalf of the
Republic of the Philippines with the prior concurrence of the Monetary The only restriction that the Constitution provides, aside from the prior concurrence of the
Board and subject to such limitations as may be provided under law. The
Monetary Board, is that the loans must be subject to limitations provided by law. In this
Monetary Board shall, within thirty days from the end of every quarter of
the calendar year, submit to the Congress a complete report of its regard, we note that Republic Act (R.A.) No. 245 as amended by Pres. Decree (P.D.) No. 142,
decisions on applications for loans to be contracted or guaranteed by the
s. 1973, entitled An Act Authorizing the Secretary of Finance to Borrow to Meet Public
government or government-owned and controlled corporations which
405

Expenditures Authorized by Law, and for Other Purposes, allows foreign loans to be the Philippines wants to pay in advance, it can buy out its bonds in the market; if interest

contracted in the form of, inter alia, bonds. Thus: rates go down but the Philippines does not have money to retire the bonds, it can replace

Sec. 1. In order to meet public expenditures authorized by law or to the old bonds with new ones; if it defaults on the bonds, the bondholders shall organize and
provide for the purchase, redemption, or refunding of any obligations, bring about a re-negotiation or settlement.[43] In fact, several countries have restructured
either direct or guaranteed of the Philippine Government, the Secretary
of Finance, with the approval of the President of the Philippines, after their sovereign bonds in view either of inability and/or unwillingness to pay the
consultation with the Monetary Board, is authorized to borrow from indebtedness.[44] Petitioners have not presented a plausible reason that would preclude the
time to time on the credit of the Republic of the Philippines such sum or
sums as in his judgment may be necessary, and to issue therefor Philippines from acting in a similar fashion, should it so opt.
evidences of indebtedness of the Philippine Government."
This theory may even be dismissed in a perfunctory manner since petitioners are merely
Such evidences of indebtedness may be of the following types:
expecting that the Philippines would opt to restructure the bonds but with the negotiable
c. Treasury bonds, notes, securities or other evidences of indebtedness
character of the bonds, would be prevented from so doing. This is a contingency which
having maturities of one year or more but not exceeding twenty-five
years from the date of issue. (Emphasis supplied.) petitioners do not assert as having come to pass or even imminent. Consummated acts of the

executive cannot be struck down by this Court merely on the basis of petitioners anticipatory
Under the foregoing provisions, sovereign bonds may be issued not only to supplement
cavils.
government expenditures but also to provide for the purchase,[37] redemption,[38] or

refunding[39] of any obligation, either direct or guaranteed, of the Philippine Government.


On the Buyback Scheme
Petitioners, however, point out that a supposed difference between contracting a loan and

issuing bonds is that the former creates a definite creditor-debtor relationship between the In their Comment, petitioners assert that the power to pay public debts lies with Congress

parties while the latter does not.[40] They explain that a contract of loan enables the debtor and was deliberately withheld by the Constitution from the President.[45] It is true that in the

to restructure or novate the loan, which benefit is lost upon the conversion of the debts to balance of power between the three branches of government, it is Congress that manages

bearer bonds such that the Philippines surrenders the novatable character of a loan contract the countrys coffers by virtue of its taxing and spending powers. However, the law-making

for the irrevocable and unpostponable demandability of a bearer bond. [41] Allegedly, the authority has promulgated a law ordaining an automatic appropriations provision for debt

Constitution prohibits the President from issuing bonds which are far more onerous than servicing[46] by virtue of which the President is empowered to execute debt payments

loans.[42] without the need for further appropriations. Regarding these legislative enactments, this

Court has held, viz:


This line of thinking is flawed to say the least. The negotiable character of the subject bonds
Congress deliberates or acts on the budget proposals of the President,
is not mutually exclusive with the Republics freedom to negotiate with bondholders for the and Congress in the exercise of its own judgment and wisdom formulates
an appropriation act precisely following the process established by the
revision of the terms of the debt. Moreover, the securities market provides some flexibilityif
406

Constitution, which specifies that no money may be paid from the


The afore-quoted provisions of law specifically allow the President to pre-terminate debts
Treasury except in accordance with an appropriation made by law.
without further action from Congress.
Debt service is not included in the General Appropriation Act, since
authorization therefor already exists under RA Nos. 4860 and 245, as
Petitioners claim that the buyback scheme is neither a guarantee nor a loan since its
amended, and PD 1967. Precisely in the light of this subsisting
authorization as embodied in said Republic Acts and PD for debt service, underlying intent is to extinguish debts that are not yet due and demandable. [48] Thus, they
Congress does not concern itself with details for implementation by the
suggest that contracts entered pursuant to the buyback scheme are unconstitutional for not
Executive, but largely with annual levels and approval thereof upon due
deliberations as part of the whole obligation program for the year. Upon being among those contemplated in Sec. 20, Art. VII of the Constitution.
such approval, Congress has spoken and cannot be said to have
delegated its wisdom to the Executive, on whose part lies the Buyback is a necessary power which springs from the grant of the foreign borrowing power.
implementation or execution of the legislative wisdom.[47]
Every statute is understood, by implication, to contain all such provisions as may be

Specific legal authority for the buyback of loans is established under Section 2 of Republic Act necessary to effectuate its object and purpose, or to make effective rights, powers, privileges

(R.A.) No. 240, viz: or jurisdiction which it grants, including all such collateral and subsidiary consequences as

may be fairly and logically inferred from its terms.[49] The President is not empowered to
Sec. 2. The Secretary of Finance shall cause to be paid out of
any moneys in the National Treasury not otherwise borrow money from foreign banks and governments on the credit of the Republic only to be
appropriated, or from any sinking funds provided for the
left bereft of authority to implement the payment despite appropriations therefor.
purpose by law, any interest falling due, or accruing, on any
portion of the public debt authorized by law. He shall also
cause to be paid out of any such money, or from any such Even petitioners concede that [t]he Constitution, as a rule, does not enumeratelet alone
sinking funds the principal amount of any obligations which enumerate allthe acts which the President (or any other public officer) may not do,[50] and
have matured, or which have been called for redemption or for
which redemption has been demanded in accordance with [t]he fact that the Constitution does not explicitly bar the President from exercising a power
terms prescribed by him prior to date of issue: Provided, does not mean that he or she does not have that power.[51] It is inescapable from the
however, That he may, if he so chooses and if the holder is
willing, exchange any such obligation with any other direct or standpoint of reason and necessity that the authority to contract foreign loans and
guaranteed obligation or obligations of the Philippine guarantees without restrictions on payment or manner thereof coupled with the availability
Government of equivalent value. In the case of interest-bearing
obligations, he shall pay not less than their face value; in the of the corresponding appropriations, must include the power to effect payments or to make
case of obligations issued at a discount he shall pay the face payments unavailing by either restructuring the loans or even refusing to make any payment
value at maturity; or, if redeemed prior to maturity, such
portion of the face value as is prescribed by the terms and altogether.
conditions under which such obligations were originally
issued. (Emphasis supplied.) More fundamentally, when taken in the context of sovereign debts, a buyback is simply the

purchase by the sovereign issuer of its own debts at a discount. Clearly then, the objection to

the validity of the buyback scheme is without basis.


407

Second Issue: Delegation of Power cabinet positions and the respective expertise which the holders thereof are accorded and

would unduly hamper the Presidents effectivity in running the government.


Petitioners stress that unlike other powers which may be validly delegated by the President,
Necessity thus gave birth to the doctrine of qualified political agency, later adopted in Villena
the power to incur foreign debts is expressly reserved by the Constitution in the person of
v. Secretary of the Interior[55] from American jurisprudence, viz:
the President. They argue that the gravity by which the exercise of the power will affect the
With reference to the Executive Department of the government, there is
Filipino nation requires that the President alone must exercise this power. They submit that
one purpose which is crystal-clear and is readily visible without the
the requirement of prior concurrence of an entity specifically named by the Constitutionthe projection of judicial searchlight, and that is the establishment of a single,
not plural, Executive. The first section of Article VII of the Constitution,
Monetary Boardreinforces the submission that not respondents but the President alone and
dealing with the Executive Department, begins with the enunciation of
personally can validly bind the country. the principle that "The executive power shall be vested in a President of
the Philippines." This means that the President of the Philippines is the
Petitioners position is negated both by explicit constitutional[52] and legal[53] imprimaturs, as Executive of the Government of the Philippines, and no other. The heads
of the executive departments occupy political positions and hold office in
well as the doctrine of qualified political agency. an advisory capacity, and, in the language of Thomas Jefferson, "should
be of the President's bosom confidence" (7 Writings, Ford ed., 498), and,
The evident exigency of having the Secretary of Finance implement the decision of the in the language of Attorney-General Cushing (7 Op., Attorney-General,
453), "are subject to the direction of the President." Without minimizing
President to execute the debt-relief contracts is made manifest by the fact that the process
the importance of the heads of the various departments, their
of establishing and executing a strategy for managing the governments debt is deep within personality is in reality but the projection of that of the President. Stated
otherwise, and as forcibly characterized by Chief Justice Taft of the
the realm of the expertise of the Department of Finance, primed as it is to raise the required
Supreme Court of the United States, "each head of a department is, and
amount of funding, achieve its risk and cost objectives, and meet any other sovereign debt must be, the President's alter ego in the matters of that department
where the President is required by law to exercise authority" (Myers vs.
management goals.[54]
United States, 47 Sup. Ct. Rep., 21 at 30; 272 U. S., 52 at 133; 71 Law. ed.,
160).[56]
If, as petitioners would have it, the President were to personally exercise every aspect of the

foreign borrowing power, he/she would have to pause from running the country long enough As it was, the backdrop consisted of a major policy determination made by then President
to focus on a welter of time-consuming detailed activitiesthe propriety of Aquino that sovereign debts have to be respected and the concomitant reality that
incurring/guaranteeing loans, studying and choosing among the many methods that may be the Philippines did not have enough funds to pay the debts. Inevitably, it fell upon the
taken toward this end, meeting countless times with creditor representatives to negotiate, Secretary of Finance, as the alter ego of the President regarding the sound and efficient
obtaining the concurrence of the Monetary Board, explaining and defending the negotiated management of the financial resources of the Government,[57] to formulate a scheme for the
deal to the public, and more often than not, flying to the agreed place of execution to sign implementation of the policy publicly expressed by the President herself.
the documents. This sort of constitutional interpretation would negate the very existence of
408

Nevertheless, there are powers vested in the President by the Constitution which may not be Another important qualification must be made. The Secretary of Finance or any

delegated to or exercised by an agent or alter ego of the President. Justice Laurel, in designated alter ego of the President is bound to secure the latters prior consent to or

his ponencia in Villena, makes this clear: subsequent ratification of his acts. In the matter of contracting or guaranteeing foreign loans,

Withal, at first blush, the argument of ratification may seem plausible the repudiation by the President of the very acts performed in this regard by the alter
under the circumstances, it should be observed that there are certain ego will definitely have binding effect. Had petitioners herein succeeded in demonstrating
acts which, by their very nature, cannot be validated by subsequent
approval or ratification by the President. There are certain constitutional that the President actually withheld approval and/or repudiated the Financing Program,
powers and prerogatives of the Chief Executive of the Nation which must there could be a cause of action to nullify the acts of respondents. Notably though,
be exercised by him in person and no amount of approval or ratification
will validate the exercise of any of those powers by any other person. petitioners do not assert that respondents pursued the Program without prior authorization
Such, for instance, in his power to suspend the writ of habeas corpus and of the President or that the terms of the contract were agreed upon without the Presidents
proclaim martial law (PAR. 3, SEC. 11, Art. VII) and the exercise by him of
the benign prerogative of mercy (par. 6, sec. 11, idem).[58] authorization. Congruent with the avowed preference of then President Aquino to honor and

restructure existing foreign debts, the lack of showing that she countermanded the acts of
These distinctions hold true to this day. There are certain presidential powers which arise out respondents leads us to conclude that said acts carried presidential approval.
of exceptional circumstances, and if exercised, would involve the suspension of fundamental
With constitutional parameters already established, we may also note, as a source of
freedoms, or at least call for the supersedence of executive prerogatives over those
suppletory guidance, the provisions of R.A. No. 245. The afore-quoted Section 1 thereof
exercised by co-equal branches of government. The declaration of martial law, the
empowers the Secretary of Finance with the approval of the President and after
suspension of the writ of habeas corpus, and the exercise of the pardoning power
consultation[59] of the Monetary Board, to borrow from time to time on the credit of the
notwithstanding the judicial determination of guilt of the accused, all fall within this special
Republic of the Philippines such sum or sums as in his judgment may be necessary, and to
class that demands the exclusive exercise by the President of the constitutionally vested
issue therefor evidences of indebtedness of the Philippine Government. Ineluctably then,
power. The list is by no means exclusive, but there must be a showing that the executive
while the President wields the borrowing power it is the Secretary of Finance who normally
power in question is of similar gravitas and exceptional import.
carries out its thrusts.
We cannot conclude that the power of the President to contract or guarantee foreign debts
In our recent rulings in Southern Cross Cement Corporation v. The Philippine Cement
falls within the same exceptional class. Indubitably, the decision to contract or guarantee
Manufacturers Corp.,[60] this Court had occasion to examine the authority granted by
foreign debts is of vital public interest, but only akin to any contractual obligation undertaken
Congress to the Department of Trade and Industry (DTI) Secretary to impose safeguard
by the sovereign, which arises not from any extraordinary incident, but from the established
measures pursuant to the Safeguard Measures Act. In doing so, the Court was impelled to
functions of governance.
construe Section 28(2), Article VI of the Constitution, which allowed Congress, by law, to

authorize the President to fix within specified limits, and subject to such limitations and
409

restrictions as it may impose, tariff rates, import and export quotas, tonnage and wharfage the absence of proof and even allegation to the contrary, should be regarded in a fashion

dues, and other duties or imposts within the framework of the national development congruent with the presumption of regularity bestowed on acts done by public officials.

program of the Government.[61]


Moreover, in praying that the acts of the respondents, especially that of the Secretary of

While the Court refused to uphold the broad construction of the grant of power as preferred Finance, be nullified as being in violation of a restrictive constitutional interpretation,

by the DTI Secretary, it nonetheless tacitly acknowledged that Congress could designate the petitioners in effect would have this Court declare R.A. No. 245 unconstitutional. We will not

DTI Secretary, in his capacity as alter ego of the President, to exercise the authority vested on strike down a law or provisions thereof without so much as a direct attack thereon when

the chief executive under Section 28(2), Article VI.[62] At the same time, the Court simple and logical statutory construction would suffice.

emphasized that since Section 28(2), Article VI authorized Congress to impose limitations and

restrictions on the authority of the President to impose tariffs and imposts, the DTI Secretary Petitioners also submit that the unrestricted character of the Financing Program violates the

was necessarily subjected to the same restrictions that Congress could impose on the framers intent behind Section 20, Article VII to restrict the power of the President. This

President in the exercise of this taxing power. intent, petitioners note, is embodied in the proviso in Sec. 20, Art. VII, which states that said

power is subject to such limitations as may be provided under law. However, as previously
Similarly, in the instant case, the Constitution allocates to the President the exercise of the
discussed, the debt-relief contracts are governed by the terms of R.A. No. 245, as amended
foreign borrowing power subject to such limitations as may be provided under law.
by P.D. No. 142 s. 1973, and therefore were not developed in an unrestricted setting.
Following Southern Cross, but in line with the limitations as defined in Villena, the
Third Issue: Grave Abuse of Discretion and
presidential prerogative may be exercised by the Presidents alter ego, who in this case is the

Secretary of Finance. Violation of Constitutional Policies

It bears emphasis that apart from the Constitution, there is also a relevant statute, R.A. No.
We treat the remaining issues jointly, for in view of the foregoing determination, the general
245, that establishes the parameters by which the alter ego may act in behalf of the
allegation of grave abuse of discretion on the part of respondents would arise from the
President with respect to the borrowing power. This law expressly provides that the
purported violation of various state policies as expressed in the Constitution.
Secretary of Finance may enter into foreign borrowing contracts. This law neither amends

nor goes contrary to the Constitution but merely implements the subject provision in a
Petitioners allege that the Financing Program violates the constitutional state policies to
manner consistent with the structure of the Executive Department and the alter ego doctine.
promote a social order that will ensure the prosperity and independence of the nation and
In this regard, respondents have declared that they have followed the restrictions provided
free the people from poverty,[64] foster social justice in all phases of national
under R.A. No. 245,[63] which include the requisite presidential authorization and which, in
development,[65] and develop a self-reliant and independent national economy effectively

controlled by Filipinos;[66] thus, the contracts executed or to be executed pursuant thereto


410

were or would be tainted by a grave abuse of discretion amounting to lack or excess of Assuming the accuracy of the foregoing for the nonce, despite the watered-down parameters

jurisdiction. of petitioners computations, we can make no conclusion other than that respondents efforts

were geared towards debt-relief with marked positive results and towards achieving the
Respondents cite the following in support of the propriety of their acts: [67] (1) a Department
constitutional policies which petitioners so hastily declare as having been violated by
of Finance study showing that as a result of the implementation of voluntary debt reductions
respondents. We recognize that as with other schemes dependent on volatile market and
schemes, the countrys debt stock was reduced by U.S. $4.4 billion as of December
economic structures, the contracts entered into by respondents may possibly have a net
1991;[68] (2) revelations made by independent individuals made in a hearing before the
outflow and therefore negative result. However, even petitioners call this latter event the
Senate Committee on Economic Affairs indicating that the assailed agreements would bring
worst-case scenario. Plans are seldom foolproof. To ask the Court to strike down debt-relief
about substantial benefits to the country;[69] and (3) the Joint Legislative-Executive Foreign
contracts, which, according to independent third party evaluations using historically-
Debt Councils endorsement of the approval of the financing package containing the debt-
suggested rates would result in substantial debt-relief,[76] based merely on the possibility of
relief agreements and issuance of a Motion to Urge the Philippine Debt Negotiating Panel to
petitioners worst-case scenario projection, hardly seems reasonable.
continue with the negotiation on the aforesaid package.[70]

Even with these justifications, respondents aver that their acts are within the arena of Moreover, the policies set by the Constitution as litanized by petitioners are not a panacea
political questions which, based on the doctrine of separation of powers,[71] the judiciary that can annul every governmental act sought to be struck down. The gist of petitioners
must leave without interference lest the courts substitute their judgment for that of the arguments on violation of constitutional policies and grave abuse of discretion boils down to
official concerned and decide a matter which by its nature or law is for the latter alone to their allegation that the debt-relief agreements entered into by respondents do not deliver
decide.[72] the kind of debt-relief that petitioners would want. Petitioners cite the aforementioned

article in stating that that the agreement achieves little that cannot be gained through less
On the other hand, in furtherance of their argument on respondents violation of
complicated means like postponing (rescheduling) principal payments,[77] thus:
constitutional policies, petitioners cite an article of Jude Esguerra, The 1992 Buyback and
[T]he price of success in putting together this debt-relief package
Securitization Agreement with Philippine Commercial Bank Creditors,[73] in illustrating a best-
(indicates) the possibility that a simple rescheduling agreement may well
case scenario in entering the subject debt-relief agreements. The computation results in a turn out to be less expensive than this comprehensive debt-relief
package. This means that in the next six years the humble and simple
yield of $218.99 million, rather than the $2,041.00 million claimed by the debt
rescheduling process may well be the lesser evil because there is that
negotiators.[74] On the other hand, the worst-case scenario allegedly is that a net amount of distinct possibility that less money will flow out of the country as a result.

$1.638 million will flow out of the country as a result of the debt package.[75]
411

Constantino v. CuisiaG.R. No. 106064. October 13, 2005


Note must be taken that from these citations, petitioners submit that there is possibly a

better way to go about debt rescheduling and, on that basis, insist that the acts of Facts:

respondents must be struck down. These are rather tenuous grounds to condemn the
Petition for certiorari, prohibition andmandamus of the Philippine ComprehensiveProgram
subject agreements as violative of constitutional principles. for 1992.Petitioners are members of the non-government organization, Freedom from
DebtCoalition, which advocates a “pro-people and just Philippine debt policy.” They question
the Financing Program started by then President Corazon Aquino, characterized as a “multi
Conclusion option financing package”, wherein thePresident entered into three restructuringagreements
with foreign creditor governments.Petitioners stress that unlike otherpowers which may be
validly delegated by thePresident, the power to incur foreign debts isexpressly reserved by
The raison d etre of the Financing Program is to manage debts incurred by the Philippines in
the Constitution in theperson of the President.
a manner that will lessen the burden on the Filipino taxpayersthus the term debt-relief
Issue:
agreements. The measures objected to by petitioners were not aimed at incurring more

debts but at terminating pre-existing debts and were backed by the know-how of the WON the President can delegate thepower to incur foreign debts to other executiveagencies.

countrys economic managers as affirmed by third party empirical analysis.


WON the President can borrow to meet publicexpenditures in the form of bonds.

That the means employed to achieve the goal of debt-relief do not sit well with petitioners is Held:
beyond the power of this Court to remedy. The exercise of the power of judicial review is
Petition DISMISSED.
merely to checknot supplantthe Executive, or to simply ascertain whether he has gone

beyond the constitutional limits of his jurisdiction but not to exercise the power vested in Ratio/Doctrine:

him or to determine the wisdom of his act.[78] In cases where the main purpose is to nullify Yes, based on the Doctrine of Qualified Political Agency. Each head of the department is and
must be, the President’s alter ego in the matters of that departmentwhere the President is
governmental acts whether as unconstitutional or done with grave abuse of discretion, there
required by law toexercise authority.Yes, based on RA 245, as amended byPD 142, s.1973.
is a strong presumption in favor of the validity of the assailed acts. The heavy onus is in on
472 SCRA 505 – Political Law – Constitutional Law – The Executive Department – Qualified
petitioners to overcome the presumption of regularity.
Political Agency – when not applied – Borrowing Powers of the President
During the Corazon Aquino regime, her administration came up with a scheme to reduce the
We find that petitioners have not sufficiently established any basis for the Court to declare country’s external debt. The solution resorted to was to incur foreign debts. Three
the acts of respondents as unconstitutional. restructuring programs were sought to initiate the program for foreign debts – they are
basically buyback programs and bond-conversion programs. The spouses Renato
WHEREFORE the petition is hereby DISMISSED. No costs. Constantino, Jr. and Lourdes Constantino, as a taxpayers, and in behalf of their minor
children who are Filipino citizens, together with FFDC (Freedom From Debt Coalition) averred
that the buyback and bond-conversion schemes were onerous and they do not constitute the
SO ORDERED. loan “contract” or “guarantee” contemplated in Sec. 20, Art. VII of the Constitution. And
assuming that the President has such power, unlike other powers which may be validly
delegated by the President, the power to incur foreign debts is expressly reserved by the
Constitution in the person of the President, hence, the respondents herein, Central Bank
Governor Josse Cuisia et al, cannot incur debts for the Philippines or such power can be
412

delegated to them. Constantino argue that the gravity by which the exercise of the power President of the constitutionally vested power. The list is by no means exclusive, but there
will affect the Filipino nation requires that the President alone must exercise this power. must be a showing that the executive power in question is of similar gravitas and exceptional
They argue that the requirement of prior concurrence of an entity specifically named by the import.
Constitution–the Monetary Board–reinforces the submission that not respondents but the
President “alone and personally” can validly bind the country. Hence, they would like Cuisia
et al to stop acting pursuant to the said scheme.
ISSUE: Whether or not the President of the Philippines can validly delegate her debt power
to the respondents.
HELD:
Yes. There is no question that the president has borrowing powers and that the President
may contract or guarantee foreign loans in behalf of this country with prior concurrence of
the Monetary Board. It makes no distinction whatsoever and the fact that a debt or a loan
may be onerous is irrelevant. On the other hand, the President can delegate this power to
her direct subordinates. The evident exigency of having the Secretary of Finance implement
the decision of the President to execute the debt-relief contracts is made manifest by the
fact that the process of establishing and executing a strategy for managing the government’s
debt is deep within the realm of the expertise of the Department of Finance, primed as it is
to raise the required amount of funding, achieve its risk and cost objectives, and meet any
other sovereign debt management goals. If the President were to personally exercise every
aspect of the foreign borrowing power, he/she would have to pause from running the
country long enough to focus on a welter of time-consuming detailed activities–the propriety
of incurring/guaranteeing loans, studying and choosing among the many methods that may
be taken toward this end, meeting countless times with creditor representatives to
negotiate, obtaining the concurrence of the Monetary Board, explaining and defending the
negotiated deal to the public, and more often than not, flying to the agreed place of
execution to sign the documents. This sort of constitutional interpretation would negate the
very existence of cabinet positions and the respective expertise which the holders thereof
are accorded and would unduly hamper the President’s effectivity in running the
government. The act of the Cuisia et al are not unconstitutional.

Exception
There are certain acts which, by their very nature, cannot be validated by subsequent
approval or ratification by the President. There are certain constitutional powers and
prerogatives of the Chief Executive of the Nation which must be exercised by him in person
and no amount of approval or ratification will validate the exercise of any of those powers by
any other person. Such, for instance, in his power to suspend the writ of habeas corpus and
proclaim martial law and the exercise by him of the benign prerogative of pardon (mercy).
There are certain presidential powers which arise out of exceptional circumstances, and if
exercised, would involve the suspension of fundamental freedoms, or at least call for the
supersedence of executive prerogatives over those exercised by co-equal branches of
government. The declaration of martial law, the suspension of the writ of habeas corpus, and
the exercise of the pardoning power notwithstanding the judicial determination of guilt of
the accused, all fall within this special class that demands the exclusive exercise by the
413

SECOND DIVISION of the petitioners. The property is covered by a Transfer Certificate of Title No. 143713 of the
Registry of Deeds of the Province of Bukidnon.
G.R. No. 131457 April 24, 1998
2. In 1984, the land was leased as a pineapple plantation to the Philippine Packing
HON. CARLOS O. FORTICH, PROVINCIAL GOVERNOR OF BUKIDNON, HON. REY B. BAULA, Corporation, now Del Monte Philippines, Inc. (DMPI), a multinational corporation, for a
MUNICIPAL MAYOR OF SUMILAO, BUKIDNON, NQSR MANAGEMENT AND DEVELOPMENT period of ten (10) years under the Crop Producer and Grower's Agreement duly annotated in
CORPORATION, petitioners, the certificate of title. The lease expired in April, 1994.
vs.
HON. RENATO C. CORONA, DEPUTY EXECUTIVE SECRETARY, HON. ERNESTO D. GARILAO, 3. In October, 1991, during the existence of the lease, the Department of Agrarian Reform
SECRETARY OF THE DEPARTMENT OF AGRARIAN REFORM, respondents. (DAR) placed the entire 144-hectare property under compulsory acquisition and assessed the
land value at P2.38 million.4
MARTINEZ, J.:
4. NQSRMDC resisted the DAR's action. In February, 1992, it sought and was granted by the
The dramatic and well-publicized hunger strike staged by some alleged farmer-beneficiaries DAR Adjudication Board (DARAB), through its Provincial Agrarian Reform Adjudicator
in front of the Department of Agrarian Reform compound in Quezon City on October 9, 1997 (PARAD) in DARAB Case No. X-576, a writ of prohibition with preliminary injunction which
commanded nationwide attention that even church leaders and some presidential ordered the DAR Region X Director, the Provincial Agrarian Reform Officer (PARO) of
candidates tried to intervene for the strikers' "cause." Bukidnon, the Municipal Agrarian Reform Office (MARO) of Sumilao, Bukidnon, the Land
Bank of the Philippines (Land Bank), and their authorized representatives "to desist from
pursuing any activity or activities" concerning the subject land "until further orders."5
The strikers protested the March 29, 1996 Decision 1 of the Office of the President (OP),
issued through then Executive Secretary Ruben D. Torres in OP Case No. 96-C-6424, which
approved the conversion of a one hundred forty-four (144)-hectare land from agricultural to 5. Despite the DARAB order of March 31, 1992, the DAR Regional Director issued a
agro-industrial/institutional area. This led the Office of the President, through then Deputy memorandum, dated May 21, 1992, directing the Land Bank to open a trust account for
Executive Secretary Renato C. Corona, to issue the so-called "Win-Win" Resolution 2 on P2.38 million in the name of NQSRMDC and to conduct summary proceedings to determine
November 7, 1997, substantially modifying its earlier Decision after it had already become the just compensation of the subject property. NQSRMDC objected to these moves and filed
final and executory. The said Resolution modified the approval of the land conversion to on June 9, 1992 an Omnibus Motion to enforce the DARAB order of March 31, 1992 and to
agro-industrial area only to the extent of forty-four (44) hectares, and ordered the remaining nullify the summary proceedings undertaken by the DAR Regional Director and Land Bank on
one hundred (100) hectares to be distributed to qualified farmer-beneficiaries. the valuation of the subject property.

But, did the "Win-Win" Resolution culminate in victory for all the contending parties? 6. The DARAB, on October 22, 1992, acted favorably on the Omnibus Motion by (a) ordering
the DAR Regional Director and Land Bank "to seriously comply with the terms of the order
dated March 31, 1992;" (b) nullifying the DAR Regional Director's memorandum, dated May
The above-named petitioners cried foul. They have come to this Court urging us to annul and
21, 1992, and the summary proceedings conducted pursuant thereto; and (c) directing the
set aside the "Win-Win" Resolution and to enjoin respondent Secretary Ernesto D. Garilao of
Land Bank "to return the claim folder of Petitioner NQSRMDC's subject Property to the DAR
the Department of Agrarian Reform from implementing the said Resolution.
until further orders."6

Thus, the crucial issue to be resolved in this case is: What is the legal effect of the "Win-Win"
7. The Land Bank complied with the DARAB order and cancelled the trust account it opened
Resolution issued by the Office of the President on its earlier Decision involving the same
in the name of petitioner NQSRMDC. 7
subject matter, which had already become final and executory?

8. In the meantime, the Provincial Development Council (PDC) of Bukidnon, headed by


The antecedent facts of this controversy, as culled from the pleadings, may be stated as
Governor Carlos O. Fortich, passed Resolution No. 6,8 dated January 7, 1993, designating
follows:
certain areas along Bukidnon-Sayre Highway as part of the Bukidnon Agro-Industrial Zones
where the subject property is situated.
1. This case involves a 144-hectare land located at San Vicente, Sumilao, Bukidnon, owned by
the Norberto Quisumbing, Sr. Management and Development Corporation (NQSRMDC), one
414

9. What happened thereafter is well-narrated in the OP (TORRES) Decision of March 29, The said NQSRMDC Proposal was, per Certification dated January 4, 1995, adopted by the
1996, pertinent portions of which we quote: Department of Trade and Industry, Bukidnon Provincial Office, as one of its flagship projects.
The same was likewise favorably recommended by the Provincial Development Council of
Pursuant to Section 20 of R.A. No. 7160, otherwise known as the Local Government Bukidnon; the municipal, provincial and regional office of the DAR; the Regional Office
Code, the Sangguniang Bayan of Sumilao, Bukidnon, on March 4, 1993, enacted (Region X) of the DENR (which issued an Environmental Compliance Certificate on June 5,
Ordinance No. 24 converting or re-classifying 144 hectares of land in Bgy. San 1995); the Executive Director, signing "By Authority of PAUL G. DOMINGUEZ," Office of the
Vicente, said Municipality, from agricultural to industrial/institutional with a view President — Mindanao; the Secretary of DILG; and Undersecretary of DECS Wilfredo D.
of providing an opportunity to attract investors who can inject new economic Clemente.
vitality, provide more jobs and raise the income of its people.
In the same vein, the National Irrigation Administration, Provincial Irrigation Office,
Parenthetically, under said section, 4th to 5th class municipalities may authorize Bagontaas Valencia, Bukidnon, thru Mr. Julius S. Maquiling, Chief, Provincial Irrigation Office,
the classification of five percent (5%) of their agricultural land area and provide for interposed NO. OBJECTION to the proposed conversion "as long as the development cost of
the manner of their utilization or disposition. the irrigation systems thereat which is P2,377.00 per hectare be replenished by the
developer . . . ." Also, the Kisolon-San Vicente Irrigators Multi Purpose Cooperative, San
Vicente, Sumilao, Bukidnon, interposed no objection to the proposed conversion of the land
On 12 October 1993, the Bukidnon Provincial Land Use Committee approved the
in question "as it will provide more economic benefits to the community in terms of outside
said Ordinance. Accordingly, on 11 December 1993, the instant application for
investments that will come and employment opportunities that will be generated by the
conversion was filed by Mr. Gaudencio Beduya in behalf of NQSRMDC/BAIDA
projects to be put up . . . .
(Bukidnon Agro-Industrial Development Association).

On the same score, it is represented that during the public consultation held at the Kisolan
Expressing support for the proposed project, the Bukidnon Provincial Board, on the
Elementary School on 18 March 1995 with Director Jose Macalindong of DAR Central Office
basis of a Joint Committee Report submitted by its Committee on Laws, Committee
and DECS Undersecretary Clemente, the people of the affected barangay rallied behind their
on Agrarian Reform and Socio-Economic Committee approved, on 1 February 1994,
respective officials in endorsing the project.
the said Ordinance now docketed as Resolution No. 94-95. The said industrial area,
as conceived by NQSRMDC (project proponent) is supposed to have the following
components: Notwithstanding the foregoing favorable recommendation, however, on November 14, 1994,
the DAR, thru Secretary Garilao, invoking its powers to approve conversion of lands under
Section 65 of R.A. No. 6657, issued an Order denying the instant application for the
1. Development Academy of Mindanao which constitutes following: Institute for
conversion of the subject land from agricultural to agro-industrial and, instead, placed the
Continuing Higher Education; Institute for Livelihood Science (Vocational and
same under the compulsory coverage of CARP and directed the distribution thereof to all
Technical School); Institute for Agribusiness Research; Museum, Library, Cultural
qualified beneficiaries on the following grounds:
Center, and Mindanao Sports Development Complex which covers an area of 24
hectares;
1. The area is considered as a prime agricultural land with irrigation facility;
2. Bukidnon Agro-Industrial Park which consists of corn processing for corn oil, corn
starch, various corn products; rice processing for wine, rice-based snacks, 2. The land has long been covered by a Notice of Compulsory Acquisition (NCA);
exportable rice; cassava processing for starch, alcohol and food delicacies;
processing plants, fruits and fruit products such as juices; processing plants for 3. The existing policy on withdrawal or lifting on areas covered by NCA is not
vegetables processed and prepared for market; cold storage and ice plant; cannery applicable;
system; commercial stores; public market; and abattoir needing about 67 hectares;
4. There is no clear and tangible compensation package arrangements for the
3. Forest development which includes open spaces and parks for recreation, horse- beneficiaries;
back riding, memorial and mini-zoo estimated to cover 33 hectares; and
5. The procedures on how the area was identified and reclassified for agro-
4. Support facilities which comprise the construction of a 360-room hotel, industrial project has no reference to Memo Circular No. 54, Series of 1993, E.O.
restaurants, dormitories and a housing project covering an area of 20 hectares. No. 72, Series of 1993, and E.O. No. 124, Series of 1993.
415

A Motion for Reconsideration of the aforesaid Order was filed on January 9, 1995 hand, distributing the land to would-be beneficiaries (who are not even tenants, as
by applicant but the same was denied (in an Order dated June 7, 1995). 9 there are none) does not guarantee such benefits.

10. Thus, the DAR Secretary ordered the DAR Regional Director "to proceed with the Nevertheless, on the issue that the land is considered a prime agricultural land with
compulsory acquisition and distribution of the property." 10 irrigation facility it maybe appropriate to mention that, as claimed by petitioner,
while it is true that there is, indeed, an irrigation facility in the area, the same
11. Governor Carlos O. Fortich of Bukidnon appealed" the order of denial to the Office of the merely passes thru the property (as a right of way) to provide water to the
President and prayed for the conversion/reclassification of the subject land as the same ricelands located on the lower portion thereof. The land itself, subject of the
would be more beneficial to the people of Bukidnon. instant petition, is not irrigated as the same was, for several years, planted with
pineapple by the Philippine Packing Corporation.
12. To prevent the enforcement of the DAR Secretary's order, NQSRMDC, on June 29, 1995,
filed with the Court of Appeals a petition for certiorari, prohibition with preliminary On the issue that the land has long been covered by a Notice of Compulsory
injunction, 12 docketed as CA-G.R. SP No. 37614. Acquisition (NCA) and that the existing policy on withdrawal or lifting on areas
covered by NCA is not applicable, suffice it to state that the said NCA was declared
null and void by the Department of Agrarian Reform Adjudication Board (DARAB) as
13. Meanwhile, on July 25, 1995, the Honorable Paul G. Dominguez, then Presidential
early as March 1, 1992. Deciding in favor of NQSRMDC, the DARAB correctly
Assistant for Mindanao, after conducting an evaluation of the proposed project, sent a
pointed out that under Section 8 of R.A. No. 6657, the subject property could not
memorandum 13 to the President favorably endorsing the project with a recommendation
validly be the subject of compulsory acquisition until after the expiration of the
that the DAR Secretary reconsider his decision in denying the application of the province for
lease contract with Del Monte Philippines, a Multi-National Company, or until April
the conversion of the land.
1994, and ordered the DAR Regional Office and the Land Bank of the Philippines,
both in Butuan City, to "desist from pursuing any activity or activities covering
14. Also, in a memorandum 14 to the President dated August 23, 1995, the Honorable Rafael petitioner's land.
Alunan III, then Secretary of the Department of the Interior and Local Government (DILG),
recommended the conversion of the subject land to industrial/institutional use with a
On this score, we take special notice of the fact that the Quisumbing family has
request that the President "hold the implementation of the DAR order to distribute the land
already contributed substantially to the land reform program of the government,
in question."
as follows: 300 hectares of rice land in Nueva Ecija in the 70's and another 400
hectares in the nearby Municipality of Impasugong, Bukidnon, ten(10) years ago,
15. On October 23, 1995, the Court of Appeals, in CA-G.R. SP No. 37614, issued a for which they have not received "just compensation" up to this time.
Resolution 15 ordering the parties to observe status quo pending resolution of the petition. At
the hearing held in said case on October 5, 1995, the DAR, through the Solicitor General,
Neither can the assertion that "there is no clear and tangible compensation
manifested before the said court that the DAR was merely "in the processing stage of the
package arrangements for the beneficiaries' hold water as, in the first place, there
applications of farmers-claimants" and has agreed to respect status quo pending the
are no beneficiaries to speak about, for the land is not tenanted as already stated.
resolution of the petition. 16

Nor can procedural lapses in the manner of identifying/reclassifying the subject


16. In resolving the appeal, the Office of the President, through then Executive Secretary
property for agro-industrial purposes be allowed to defeat the very purpose of the
Ruben D. Torres, issued a Decision in OP Case No. 96-C-6424, dated March 29, 1996,
law granting autonomy to local government units in the management of their local
reversing the DAR Secretary's decision, the pertinent portions of which read:
affairs. Stated more simply, the language of Section 20 of R.A. No. 7160, supra, is
clear and affords no room for any other interpretation. By unequivocal legal
After a careful evaluation of the petition vis-a-vis the grounds upon which the mandate, it grants local government units autonomy in their local affairs including
denial thereof by Secretary Garilao was based, we find that the instant application the power to convert portions of their agricultural lands and provide for the
for conversion by the Municipality of Sumilao, Bukidnon is impressed with merit. manner of their utilization and disposition to enable them to attain their fullest
To be sure, converting the land in question from agricultural to agro-industrial development as self-reliant communities.
would open great opportunities for employment and bring about real development
in the area towards a sustained economic growth of the municipality. On the other
WHEREFORE, in pursuance of the spirit and intent of the said legal mandate and in
view of the favorable recommendations of the various government agencies
416

abovementioned, the subject Order, dated November 14, 1994 of the Hon. October 10, 1997, some persons claiming to be farmer-beneficiaries of the NQSRMDC
Secretary, Department of Agrarian Reform, is hereby SET ASIDE and the instant property filed a motion for intervention (styled as Memorandum In Intervention) in O.P. Case
application of NQSRMDC/BAIDA is hereby APPROVED. 17 No. 96-C-6424, asking that the OP Decision allowing the conversion of the entire 144-hectare
property be set aside. 25
17. On May 20, 1996, DAR filed a motion for reconsideration of the OP decision.
24. President Fidel V. Ramos then held a dialogue with the strikers and promised to resolve
18. On September 11, 1996, in compliance with the OP decision of March 29, 1996, their grievance within the framework of the law. He created an eight (8)-man Fact Finding
NQSRMDC and the Department of Education, Culture and Sports (DECS) executed a Task Force (FFTF) chaired by Agriculture Secretary Salvador Escudero to look into the
Memorandum of Agreement whereby the former donated four (4) hectares from the subject controversy and recommend possible solutions to the problem. 26
land to DECS for the establishment of the NQSR High School. 18
25. On November 7, 1997, the Office of the President resolved the strikers' protest by issuing
When NQSRMDC was about to transfer the title over the 4-hectare donated to DECS, it the so-called "Win/Win" Resolution penned by then Deputy Executive Secretary Renato C.
discovered that the title over the subject property was no longer in its name. It soon found Corona, the dispositive portion of which reads:
out that during the pendency of both the Petition for Certiorari, Prohibition, with Preliminary
Injunction it filed against DAR in the Court of Appeals and the appeal to the President filed by WHEREFORE, premises considered, the decision of the Office of the President,
Governor Carlos O. Fortich, the DAR, without giving just compensation, caused the through Executive Secretary Ruben Torres, dated March 29, 1996, is hereby
cancellation of NQSRMDC's title on August 11, 1995 and had it transferred in the name of the MODIFIED as follows:
Republic of the Philippines under TCT No. T-50264 19 of the Registry of Deeds of Bukidnon.
Thereafter, on September 25, 1995, DAR caused the issuance of Certificates of Land 1. NQSRMDC's application for conversion is APPROVED only with respect to the
Ownership Award (CLOA) No. 00240227 and had it registered in the name of 137 farmer- approximately forty-four (44) hectare portion of the land adjacent to the highway,
beneficiaries under TCT No. AT-3536 20 of the Registry of Deeds of Bukidnon. as recommended by the Department of Agriculture.

19. Thus, on April 10, 1997, NQSRMDC filed a complaint 21 with the Regional Trial Court (RTC) 2. The remaining approximately one hundred (100) hectares traversed by an
of Malaybalay, Bukidnon (Branch 9), docketed as Civil Case No. 2687-97, for annulment and irrigation canal and found to be suitable for agriculture shall be distributed to
cancellation of title, damages and injunction against DAR and 141 others. The RTC then qualified farmer-beneficiaries in accordance with RA 6657 or the Comprehensive
issued a Temporary Restraining Order on April 30, 1997 22and a Writ of Preliminary Injunction Agrarian Reform Law with a right of way to said portion from the highway provided
on May 19, 1997, 23 restraining the DAR and 141 others from entering, occupying and/or in the portion fronting the highway. For this purpose, the DAR and other concerned
wresting from NQSRMDC the possession of the subject land. government agencies are directed to immediately conduct the segregation survey
of the area, valuation of the property and generation of titles in the name of the
20. Meanwhile, on June 23, 1997, an Order 24 was issued by then Executive Secretary Ruben identified farmer-beneficiaries.
D. Torres denying DAR's motion for reconsideration for having been filed beyond the
reglementary period of fifteen (15) days. The said order further declared that the March 29, 3. The Department of Agrarian Reform is hereby directed to carefully and
1996 OP decision had already become final and executory. meticulously determine who among the claimants are qualified farmer-
beneficiaries.
21. The DAR filed on July 11, 1997 a second motion for reconsideration of the June 23, 1997
Order of the President. 4. The Department of Agrarian Reform is hereby further directed to expedite
payment of just compensation to NQSRMDC for the portion of the land to be
22. On August 12, 1997, the said writ of preliminary injunction issued by the RTC was covered by the CARP, including other lands previously surrendered by NQSRMDC
challenged by some alleged farmers before the Court of Appeals through a petition for CARP coverage.
for certiorari and prohibition, docketed as CA-G.R. SP No. 44905, praying for the lifting of the
injunction and for the issuance of a writ of prohibition from further trying the RTC case. 5. The Philippine National Police is hereby directed to render full assistance to the
Department of Agrarian Reform in the implementation of this Order.
23. On October 9, 1997, some alleged farmer-beneficiaries began their hunger strike in front
of the DAR Compound in Quezon City to protest the OP Decision of March 29, 1996. On
417

We take note of the Memorandum in Intervention filed by 113 farmers These are the preliminary issues which must first be resolved, including the incident on the
on October 10, 1997 without ruling on the propriety or merits thereof motion for intervention filed by the alleged farmer-beneficiaries.
since it is unnecessary to pass upon it at this time.
Anent the first issue, in order to determine whether the recourse of petitioners is proper or
SO ORDERED. 27 not, it is necessary to draw a line between an error of judgment and an error of jurisdiction.
An error of judgment is one which the court may commit in the exercise of its jurisdiction,
A copy of the "Win-Win" Resolution was received by Governor Carlos O. Fortich of Bukidnon, and which error is reviewable only by an appeal. 35 On the other hand, an error of
Mayor Rey B. Baula of Sumilao, Bukidnon, and NQSRMDC on November 24, 1997 28 and, on jurisdiction is one where the act complained of was issued by the court, officer or a quasi-
December 4, 1997, they filed the present petition for certiorari, prohibition (under Rule 65 of judicial body without or in excess of jurisdiction, or with grave abuse of discretion which is
the Revised Rules of Court) and injunction with urgent prayer for a temporary restraining tantamount to lack or in excess of jurisdiction. 36 This error is correctable only by the
order and/or writ of preliminary injunction (under Rule 58, ibid.), against then Deputy extraordinary writ of certiorari. 37
Executive Secretary Renato C. Corona and DAR Secretary Ernesto D. Garilao.
It is true that under Rule 43, appeals from awards, judgments, final orders or resolutions of
On December 12, 1997, a Motion For Leave To Intervene 29 was filed by alleged farmer- any quasi-judicial agency exercising quasi-judicial functions, 38 including the Office of the
beneficiaries, through counsel, claiming that they are real parties in interest as they were President, 39 may be taken to the Court of Appeals by filing a verified petition for
"previously identified by respondent DAR as agrarian reform beneficiaries on the 144- review 40 within fifteen (15) days from notice of the said judgment, final order or
hectare" property subject of this case. The motion was vehemently opposed 30 by the resolution, 41 whether the appeal involves questions of fact, of law, or mixed questions of
petitioners. fact and law. 42

In seeking the nullification of the "Win-Win" Resolution, the petitioners claim that the Office However, we hold that, in this particular case, the remedy prescribed in Rule 43 is
of the President was prompted to issue the said resolution "after a very well-managed inapplicable considering that the present petition contains an allegation that the challenged
hunger strike led by fake farmer-beneficiary Linda Ligmon succeeded in pressuring and/or resolution is "patently illegal" 43 and was issued with "grave abuse of discretion" and "beyond
politically blackmailing the Office of the President to come up with this purely political his (respondent Secretary Renato C. Corona's) jurisdiction" 44 when said resolution
decision to appease the 'farmers,' by reviving and modifying the Decision of 29 March substantially modified the earlier OP Decision of March 29, 1996 which had long become
1996 which has been declared final and executory in an Order of 23 June 1997. . . ."31 Thus, final and executory. In other words, the crucial issue raised here involves an error of
petitioners further allege, respondent then Deputy Executive Secretary Renato C. Corona jurisdiction, not an error of judgment which is reviewable by an appeal under Rule 43. Thus,
"committed grave abuse of discretion and acted beyond his jurisdiction when he issued the the appropriate remedy to annul and set aside the assailed resolution is an original special
questioned Resolution of 7 November 1997. . . ." 32 They availed of this extraordinary writ civil action for certiorari under Rule 65, as what the petitioners have correctly done. The
of certiorari "because there is no other plain, speedy and adequate remedy in the ordinary pertinent portion of Section 1 thereof provides:
course of law."33 They never filed a motion for reconsideration of the subject Resolution
"because (it) is patently illegal or contrary to law and it would be a futile exercise to seek a Sec. 1. Petition for certiorari. — When any tribunal, board or officer exercising
reconsideration. . . ." 34 judicial or quasi-judicial functions has acted without or in excess of its or his
jurisdiction, or with grave abuse of discretion amounting to lack or excess of
The respondents, through the Solicitor General, opposed the petition and prayed that it be jurisdiction, and there is no appeal, or any plain, speedy, and adequate remedy in
dismissed outright on the following grounds: the ordinary course of law, a person aggrieved thereby may file a verified petition
in the proper court, alleging the facts with certainty and praying that judgment be
rendered annulling or modifying the proceedings of such tribunal, board or officer,
(1) The proper remedy of petitioners should have been to file a petition for review directly
and granting such incidental reliefs as law and justice may require.
with the Court of Appeals in accordance with Rule 43 of the Revised Rules of Court;

The office of a writ of certiorari is restricted to truly extraordinary cases — cases in


(2) The petitioners failed to file a motion for reconsideration of the assailed "Win-Win"
which the act of the lower court or quasi-judicial body is wholly void. 45
Resolution before filing the present petition; and

The aforequoted Section 1 of Rule 65 mandates that the person aggrieved by the assailed
(3) Petitioner NQSRMDC is guilty of forum-shopping.
illegal act "may file a verified petition (for certiorari) in the proper court." The proper court
where the petition must be filed is stated in Section 4 of the same Rule 65 which reads:
418

Sec. 4. Where petition filed. — The petition may be filed not later than sixty (60) . . . . A direct invocation of the Supreme Court's original jurisdiction to issue these
days from notice of the judgment, order or resolution sought to be assailed in the writs should be allowed only when there are special and important reasons
Supreme Court or, if it relates to the acts or omissions of a lower court or of a therefor, clearly and specifically set out in the petition. This is established policy. It
corporation, board, officer or person, in the Regional Trial Court exercising is a policy that is necessary to prevent inordinate demands upon the Court's time
jurisdiction over the territorial area as defined by the Supreme Court. It may also and attention which are better devoted to those matters within its exclusive
be filed in the Court of Appeals whether or not the same is in aid of its appellate jurisdiction, and to prevent further over-crowding of the Court's docket.
jurisdiction, or in the Sandiganbayan if it is in aid of its jurisdiction. If it involves the
acts or omissions of a quasi-judicial agency, and unless otherwise provided by law Pursuant to said judicial policy, we resolve to take primary jurisdiction over the present
or these Rules, the petition shall be filed in and cognizable only by the Court of petition in the interest of speedy justice 55 and to avoid future litigations so as to promptly
Appeals. (4a) put an end to the present controversy which, as correctly observed by petitioners, has
sparked national interest because of the magnitude of the problem created by the issuance
Under the above-qouted Section 4, the Supreme Court, Court of Appeals and Regional Trial of the assailed resolution. Moreover, as will be discussed later, we find the assailed
Court have original concurrent jurisdiction to issue a writ resolution wholly void and requiring the petitioners to file their petition first with the Court
of certiorari, 46 prohibition 47 and mandamus. 48 But the jurisdiction of these three (3) courts of Appeals would only result in a waste of time and money.
are also delineated in that, if the challenged act relates to acts or omissions of a lower court
or of a corporation, board, officer or person, the petition must be filed with the Regional Trial That the Court has the power to set aside its own rules in the higher interests of justice is
Court which exercises jurisdiction over the territorial area as defined by the Supreme Court. well-entrenched, in our jurisprudence. We reiterate what we said in Piczon vs. Court of
And if it involves the act or omission of a quasi-judicial agency, the petition shall be filed only Appeals: 56
with the Court of Appeals, unless otherwise provided by law or the Rules of Court. We have
clearly discussed this matter of concurrence of jurisdiction in People
Be it remembered that rules of procedure are but mere tools designed to facilitate
vs. Cuaresma, et. al.,49 through now Chief Justice Andres R. Narvasa, thus:
the attainment of justice. Their strict and rigid application, which would result in
technicalities that tend to frustrate rather than promote substantial justice, must
. . . . This Court's original jurisdiction to issue writs of certiorari (as well as always be avoided. Time and again, this Court has suspended its own rules and
prohibition, mandamus, quo warranto, habeas corpus and injunction) is not excepted a particular case from their operation whenever the higher interests of
exclusive. It is shared by this Court with Regional Trial Courts (formerly Courts of justice so require. In the instant petition, we forego a lengthy disquisition of the
First Instance), which may issue the writ, enforceable in any part of their respective proper procedure that should have been taken by the parties involved and proceed
regions. It is also shared by this Court, and by the Regional Trial Court, with the directly to the merits of the case.
Court of Appeals (formerly, Intermediate Appellate Court), although prior to the
effectivity of Batas Pambansa Bilang 129 on August 14, 1981, the latter's
As to the second issue of whether the petitioners committed a fatal procedural lapse when
competence to issue the extraordinary writs was restricted to those "in aid of its
they failed to file a motion for reconsideration of the assailed resolution before seeking
appellate jurisdiction." This concurrence of jurisdiction is not, however, to be taken
judicial recourse, suffice it to state that the said motion is not necessary when the
as according to parties seeking any of the writs an absolute, unrestrained freedom
questioned resolution is a patent nullity, 57 as will be taken up later.
of choice of the court to which application therefor will be directed. There is after
all a hierarchy of courts. That hierarchy is determinative of the venue of appeals,
and should also serve as a general determinant of the appropriate forum for With respect to the third issue, the respondents claim that the filing by the petitioners of: (a)
petitions for the extraordinary writs. A becoming regard for that judicial hierarchy a petition for certiorari, prohibition with preliminary injunction (CA-G.R. SP No. 37614) with
most certainly indicates that petitions for the issuance of extraordinary writs the Court of Appeals; (b) a complaint for annulment and cancellation of title, damages and
against first level ("inferior") courts should be filed with the Regional Trial Court, injunction against DAR and 141 others (Civil Case No. 2687-97) with the Regional Trial Court
and those against the latter, with the Court of Appeals. (Citations omitted) of Malaybalay, Bukidnon; and (c) the present petition, constitute forum shopping.

But the Supreme Court has the full discretionary power to take cognizance of the petition We disagree.
filed directly to it if compelling reasons, or the nature and importance of the issues raised,
warrant. This has been the judicial policy to be observed and which has been reiterated in The rule is that:
subsequent cases, namely: 50 Uy vs. Contreras, et. al., 51 Torres vs. Arranz, 52 Bercero vs. De
Guzman, 53 and Advincula vs. Legaspi, et. al. 54 As we have further stated in Cuaresma:
419

There is forum-shopping whenever, as a result of an adverse opinion in one forum, of Farmer-Beneficiaries. Apparently, the alleged master list was made pursuant to the
a party seeks a favorable opinion (other than by appeal or certiorari) in another. directive in the dispositive portion of the assailed "Win-Win" Resolution which directs the
The principle applies not only with respect to suits filed in the courts but also in DAR "to carefully and meticulously determine who among the claimants are qualified farmer-
connection with litigation commenced in the courts while an administrative beneficiaries." However, a perusal of the said document reveals that movants are those
proceeding is pending, as in this case, in order to defeat administrative processes purportedly "Found Qualified and Recommended for Approval." In other words, movants are
and in anticipation of an unfavorable administrative ruling and a favorable court merely recommendee farmer-beneficiaries.
ruling. This specially so, as in this case, where the court in which the second suit
was brought, has no jurisdiction (citations omitted). The rule in this jurisdiction is that a real party in interest is a party who would be benefited or
injured by the judgment or is the party entitled to the avails of the suit. Real interest means
The test for determining whether a party violated the rule against forum shopping a present substantial interest, as distinguished from a mere expectancy or a future,
has been laid down in the 1986 case of Buan vs. Lopez (145 SCRA 34), . . . and that contingent, subordinate or consequential interest. 59 Undoubtedly, movants' interest over
is, forum shopping exists where the elements of litis pendentia are present or the land in question is a mere expectancy. Ergo, they are not real parties in interest.
where a final judgment in one case will amount to res judicatain the other, as
follows: Furthermore, the challenged resolution upon which movants based their motion is, as
intimated earlier, null and void. Hence, their motion for intervention has no leg to stand on.
There thus exists between the action before this Court and RTC Case No.
86-36563 identity of parties, or at least such parties as represent the Now to the main issue of whether the final and executory Decision dated March 29, 1996 can
same interests in both actions, as well as identity of rights asserted and still be substantially modified by the "Win-Win" Resolution.
relief prayed for, the relief being founded on the same facts, and the
identity on the two preceding particulars is such that any judgment
We rule in the negative.
rendered in the other action, will, regardless of which party is successful,
amount to res adjudicata in the action under consideration: all the
requisites, in fine, of auter action pendant. 58 The rules and regulations governing appeals to the Office of the President of the Philippines
are embodied in Administrative Order No. 18. Section 7 thereof provides:
It is clear from the above-quoted rule that the petitioners are not guilty of forum shopping.
The test for determining whether a party has violated the rule against forum shopping is Sec. 7. Decisions/resolutions/orders of the Office of the President shall, except as
where a final judgment in one case will amount to res adjudicata in the action under otherwise provided for by special laws, become final after the lapse of fifteen (15)
consideration. A cursory examination of the cases filed by the petitioners does not show that days from receipt of a copy thereof by the parties, unless a motion for
the said cases are similar with each other. The petition for certiorari in the Court of Appeals reconsideration thereof is filed within such period.
sought the nullification of the DAR Secretary's order to proceed with the compulsory
acquisition and distribution of the subject property. On the other hand, the civil case in RTC Only one motion for reconsideration by any one party shall be allowed and
of Malaybalay, Bukidnon for the annulment and cancellation of title issued in the name of entertained, save in exceptionally meritorious cases. (Emphasis ours).
the Republic of the Philippines, with damages, was based on the following grounds: (1) the
DAR, in applying for cancellation of petitioner NQSRMDC's title, used documents which were It is further provided for in Section 9 that "The Rules of Court shall apply in a suppletory
earlier declared null and void by the DARAB; (2) the cancellation of NQSRMDC's title was character whenever practicable.
made without payment of just compensation; and (3) without notice to NQSRMDC for the
surrender of its title. The present petition is entirely different from the said two cases as it
seeks the nullification of the assailed "Win-Win" Resolution of the Office of the President When the Office of the President issued the Order dated June 23, 1997 declaring the
dated November 7, 1997, which resolution was issued long after the previous two cases were Decision of March 29, 1996 final and executory, as no one has seasonably filed a motion for
instituted. reconsideration thereto, the said Office had lost its jurisdiction to re-open the case, more so
modify its Decision. Having lost its jurisdiction, the Office of the President has no more
authority to entertain the second motion for reconsideration filed by respondent DAR
The fourth and final preliminary issue to be resolved is the motion for intervention filed by Secretary, which second motion became the basis of the assailed "Win-Win" Resolution.
alleged farmer-beneficiaries, which we have to deny for lack of merit. In their motion, Section 7 of Administrative Order No. 18 and Section 4, Rule 43 of the Revised Rules of Court
movants contend that they are the farmer-beneficiaries of the land in question, hence, are mandate that only one (1) motion for reconsideration is allowed to be taken from the
real parties in interest. To prove this, they attached as Annex "I" in their motion a Master List Decision of March 29, 1996. And even if a second motion for reconsideration was permitted
420

to be filed in "exceptionally meritorious cases," as provided in the second paragraph of Separate Opinions
Section 7 of AO 18, still the said motion should not have been entertained considering that
the first motion for reconsideration was not seasonably filed, thereby allowing the Decision PUNO, J., separate opinion;
of March 29, 1996 to lapse into finality. Thus, the act of the Office of the President in re-
opening the case and substantially modifying its March 29, 1996 Decision which had already
The salient facts are well established. The instant controversy originated from an application
become final and executory, was in gross disregard of the rules and basic legal precept that
for land use conversion filed on December 11, 1993 before the DAR by Mr. Gaudencio
accord finalityto administrative determinations.
Beduya in behalf of the Bukidnon Agro-Industrial Development Association (BAIDA) and
petitioner NQSR Management and Development Corporation concerning its 144-hectars land
In San Luis, et al. vs. Court of Appeals, et al. 60 we held: in San Vicente, Sumilao Bukidnon. In and Oder 1 dated November 14, 1994, DAR Secretary
Ernesto D. Garilao denied the application for conversion of the land from agricultural to agro-
Since the decisions of both the Civil Service Commission and the Office of industrial use and ordered its distibution to qualified landless farmers. BAIDA and NQSR
the President had long become final and executory, the same can no Management and Development Corporation filed a motion for reconsideration 2 dated
longer be reviewed by the courts. It is well-established in our January 9, 1995, which was, however, denied in an Order 3 dated June 7, 1995, which was,
jurisprudence that the decisions and orders of administrative agencies, however, denied in an Order 3dated June 7, 1995. Thereafter, Bukidnon Governor Carlos O.
rendered pursuant to their quasi-judicial authority, have upon their Fortich sent a letter 4 to President Fidel V. Ramos requesting him to suspend the Garilao
finality, the force and binding effect of a final judgment within the Order and to confirm the ordinance enacted by the Sangguniang Bayan of Sumilao converting
purview of the doctrine of res judicata [Brillantes v. Castro, 99 Phil. 497 the subject land from agricultural to insdustrial/institutional land. Acting on the letter, then
(1956), Ipekdijna Merchandizing Co., Inc. v. Court of Tax Appeals, G.R. No. Executive Secretary Torres reversed the Garilao Order and upheld the power of local
L-15430, September 30, 1963, 9 SCRA 72.] The rule of res judicata which government units to convert portions of their agricultural lands into industrial
forbids the reopening of a matter once judicially determined by areas. 5 Respondent DAR Secretary Garilao filed a motion for reconsiderations, admittedly
competent authority applies as well to the judicial and quasi-judicial acts tardy, which was denied by then Executive Secretary Torres on the ground that his March 29,
of public, executive or administrative officers and boards acting within 1996 decision had already become final and executory in view of the lapse of the fifteen-day
their jurisdiction as to the judgments of courts having general judicial period for filling a motion for reconsideration. A second motion for reconsideration was filed
powers [Brillantes v. Castro, supra at 503]. during the pendency of which President Ramos constituted the Presidential Fact-Finding Task
Force. On November 7, 1997, Deputy Executive Secretary Corona issued the herein-assailed
The orderly administration of justice requires that the judgments/resolutions of a court or "win-win" resolution which, pursuant to the recommendations of the task force, substantially
quasi-judicial body must reach a point of finality set by the law, rules and regulations. The modified the Torres decision by awarding one (100) hectares of the Sumilao property to the
noble purpose is to write finis to disputes once and for all. 61 This is a fundamental principle qualified farmer beneficiaries and allocating only forty four (44) hectares for the
in our justice system, without which there would no end to litigations. Utmost respect and establishment of an industrial and commercial zone.
adherence to this principle must always be maintained by those who wield the power of
adjudication. Any act which violates such principle must immediately be struck down. In our decison promulgated in Baguio City on April 24, 1998, we annulled the "win-win"
resolution on the ground that public respondent Deputy Executive Secretary Renato C.
Therefore, the assailed "Win-Win" Resolution which substantially modified the Decision of Corona committed grave abuse of discretion in modifying an already final and executory
March 29, 1996 after it has attained finality, is utterly void. Such void resolution, as aptly decison of then Executive Secretary Ruben D. Torres. It is undisputed that the Department of
stressed by Justice Thomas A. Street 62 in a 1918 case, 63 is "a lawless thing, which can be Agrarian Reform (DAR) failed to comply with the fifteen-day period for filling a motion for
treated as an outlaw and slain at sight, or ignored wherever and whenever it exhibits its reconsideration. 6 It received the Torres decison on April 10, 1996 but transmitted its for
head." 64 mailing to the Office of the President only on May 23, 1997. 7 The Office of the President
received the motion on July 14, 1997. Forthwith, we applied the rule on finality of
administrative determinations and upheld the policy of setting an end to litigation as an
WHEREFORE, the present petition is hereby GRANTED. The challenged Resolution dated
indispensable aspect of orderly administration of justice. In their motions for
November 7, 1997, issued by the Office of the President in OP Case No. 96-C-6424, is hereby
reconsideration, respondent and intervenors protest the technical basis of our decision.
NULLIFIED and SET ASIDE. The Motion For Leave To Intervene filed by alleged farmer-
beneficiaries is hereby DENIED.
I vote to grant their motions for reconsideration and remand the case to the Court of
Appeals.
No pronouncement as to costs. SO ORDERED.
421

First. It is true that procedural rules are necessary to secure just speedy and inexpensive This presidential power of control over the executive branch of
disposition of every action and proceeding. 8 Procedure, however, is only a means to an government extends over all executive officers from Cabinet Secretary to
end, 9and they may be suspended when they subvert the interests of justice. It is sel-evident the lowliest clerk and has been held by us, in the landmark case of
that the prerogative to suspend procedural rules or to grant an exception in a particular case Mondono vs. Silvosa to mean "the power of [the President] to alter or
lies in the authority that promulgated the rules. 10 modify or nullify or set aside what a subordinate officer had done in the
performance of his duties and to substitute the judgment of the former
Rules concerning pleading, practice and procedure in all courts are promulgated by this with that of the latter." It is said to be at the very "heart of the meaning
Court. 11On the other hand, it is the President as administrative head who is vested by the of Chief Executive".
Administrative Code of 1987 to promulgate rules relating to governmental operations,
including administrative procedure. These rules take the form of administrative Equally well accepted, as a corollary rule to the control powers of the
orders. 12 This power is necessary for the President to discharge his constitutional duty of President, is the "Doctrine of Qualified Political Agency." As the President
faithfully executing our law. 13 Under exceptional circumstances, this Court has suspended its cannot be expected to exercise his control powers all the same time and
rules to prevent miscarriage of justice. In the same breath, we should hold that the President in person, he will have to delegate some of them to his Cabinet members.
has the power to suspend the effectivity of administrative rules of procedure when they
hamper, defeat or in any way undermine the effective enforcement of the laws of the land. Under this doctrine, which recognizes the establishment of a single
Indeed, we already recognize that Congress can suspend its own rules if doing so will enable executive, "all executive and administrative organizations are adjuncts of
it to facilitate its task of lawmaking. The three great branches of our government are co- the Executive Department, the heads of the various executives
equal and within their own sphere they have the same responsibility to promote the good of departments are assitants and agents of the Chief Executive, and, except
our people. There is no reason to withhold the power to suspend rules from the President in cases where the Chief Executive is required by the Constitution or law
and grant it alone to the two other branches of government. to act in person o[r] the exigencies of the situation demand that he act
personally, the multifarious executive and administrative functions of the
A closer scrutiny of the records in the instant case reveals that the fifteen-day rule for filling a Chief Executive are performed by and through the executive
motion for reconsideration under Section 7 of Administrative Order No. 18 was suspended by departments, and the acts of the Secretaries of such departments,
the President when he constituted, on October 15, 1997 or some six (6) months after the performed and promulgated in the regular course of business, are, unless
promulgation of the Torres decision, the Presidential Fact-Finding Task Force to conduct a dissapproved or reprobated by the Chief Executive presumptively the
comprehensive review of the proper land use of the 144-hectare Sumilao property. At that acts of the Chief Executive". . . .
time, then Executive Secretary Torres had already denied the first motion for reconsideration
of the DAR on the ground that his March 29, 1997 decision had already become final and Thus, and in short, "the President's power of control is directly exercised
executory. This notwithstanding, the President treated the case as still open and stated in his by him over the members of the Cabinet who, in turn, and by his
memorandum that the findings of the Presidential Fact-Finding Task Force" will be inputs to authority, control the bureaus and other offices under their respective
the resolution of the case now pending at the Office of the President regarding the said land" jurisdictions in the executive department." 16
(emphasis ours). 15 The President took cognizance of the special circumstances surrounding
the tardy filing by the DAR of its motion for reconsideration. The DAR lawyers assigned to the
By suspending the fifteen-day period for filing a motion for reconsideration and re-
Sumilao case received the Torres decision only, after the lapse of the reglementary fifteen-
opening the Torres decision, the President clearly exercised his control power over
day period for appeal. The copy of the decision intended for them was passed from one
an alter-ego within the framework of a constitutional and presidential system of
office to another, e.g., the Records Section of the DAR, the Office of the DAR Secretary, the
governance.
Bureau of Agrarian Legal Assistance, before it finally reached the DAR Litigation Office. It
does not appear to be just that DAR will be made to lose a significant case because of
bureaucratic lapses. Viewed in this context, we should rule that the President suspended the The President's suspension of the fifteen-day rule for filing a motion for reconsideration
effectivity of Section 7 of Administrative Order No. 18 and that his exercise of discreation in cannot be characterized as arbitrary. The Sumilao problem raises fundamental issues which
this regard cannot be assailed as whimsical. conflict between land reform and the industrialization of the countryside, the power of
control by the President over his alter-ego vis-a-vis the power of local governments to
convert agricultural land to industrial land. The resolution of these issues has far reaching
I also repectfully submit this act of the President also finds full sanction under the corollary
implications on the issues of our land reform program. Indeed, their successful resolution can
principles of presidential power of control and qualified political agency.
bring peace or rebellion in our contryside. The President should not be frustrated by an
administrative procedural rule that he himself promulgated, from formulating a creative,
422

legal solution to the Sumilao problem. There is no denying the liberal interpretation equally Third. Considering the special circumstances of the case as detailed above, it would better
accorded to both administrative and judicial rules in order to promote their object to the serve the ends of justice to obtain a definitive resolution of the issues raised in the instant
extent that technicality be not a bar to the vindication of a legitimate grievance. We have petition and remand the same to the Court of Appeals where jurisdiction over this appeal
trumpeted the truism that when technicality ceases to be an aid to justice, the courts are lies. Noteworthy, is the pendency in the Court of Appeals of two more cases involving the
justified in excepting from its operation a paricular case. 17 We ought not to deny the same Sumilao property: (1) Petition for Certiorari and Prohibition, entitled, "N.Q.S.R. Management
power to the Chief Executive who heads a co-equal branch of government. & Development Corporation and Bukidnon Agro-Industrial Association, Petitioners, vs. Hon.
Ernesto Garilao, Secretary of the Department of Agrarian Reform; Rogelio E. Tamin, DAR
Second. The petitioners are estopped from assailing the authority of the Office of the Regional Director, Region X; Nicanor Peralta, Provincial Agrarian Reform Officer, Region X;
President to re-open the Sumilao case and resolve it based on the report of the Presidential Dolores Apostol, Municipal Agrarian Reform Officer, Sumilao, Bukidnon, Respondent;" 28 and
Fact-Finding Task Force. Undeniably, petitioners participated in the processes conducted by (2) Petition, for Certiorari and Prohibition, entitled, "Rodolfo Buclasan, et al., Petitioners, vs.
the task force. Their participation in the administrative proceedings without raising any Hon. Leonardo N. Demecillo, as Judge of RTC, Malaybalay, Bukidnon, Branch IX and NQSR
objection thereto, bars them from raising any jurisdictional infirmity after an adverse Management and Development Corporation,
decision is rendered against them. 18Petitioners Carlos O. Fortich and Rey B. Baula, Bukidnon Respondent. 29
Governor and Sumilao Mayor, respectively, were named members of the task force. 19 The
president ordered the task force to confer with the representatives of, among others, the The remand of the instant petition to the Court of Appeals would enable said court to
landowner, namely, petitioner NQSR Management and Development Corporation. 20 In a consolidate the same with the two other cases pending there which undoubtedly
letter dated October 20, 1997 addressed to the President, the counsel for NQSR contemplate of the same factual milieu and raise invariably the same issues as in this
Management and Development Corporation expressed its reluctance "to comment on the petition, leaving no room for further confusion that will surely be wrought by the rendition of
merits and demerits of the (motion for intervention and motion to admit additional evidence conflicting decisions affecting a single controversy.
filed by the farmer beneficiaries] out of respect of the Regional Trial Court and the Court of
Appeals where these cases are presently pending. 21 NQSR Management and Development For the above reasons, I vote to grant the motions for reconsideration filed by the
Corporation, however, did not question the authority of the President to constitute the task respondents and the intervenors who should be allowed to intervene pursuant to sec, 1, Rule
force despite its express adherence to the declaration made by then Executive Secretary 19 and to remand the instant petition to the Court of Appeals for appropriate proceedings.
Torres as to the finality of his March 29, 1997 decision. It was confident that its interests
would be promoted and protected by Bukidnon Governor Fortich who himself filed the
Separate Opinions
appeal from the order of DAR Secretary Garilao 22 and Sumilao Mayor Baula who certified as
correct Resolution No. 24 approved by the Sangguniang Bayan of Sumilao on March 4, 1993
converting the 144-hectare property from agricultural to industrial/institutional land. 23 But PUNO, J., separate opinion;
when "win-win" resolution was issued by the Office of the President on November 7, 1997,
allowing the conversion into industrial land of only forty four (44) hectares of the 114- The salient facts are well established. The instant controversy originated from an application
hectare Sumilao property and ordering the distribution of the rest to qualified farmer for land use conversion filed on December 11, 1993 before the DAR by Mr. Gaudencio
beneficiaries, petitioners were flabbergasted. Mr. Norberto Quisumbing, Jr. could hardly hide Beduya in behalf of the Bukidnon Agro-Industrial Development Association (BAIDA) and
his disdain over that resolution in his letter to the provincial agrarian reform officer petitioner NQSR Management and Development Corporation concerning its 144-hectars land
protesting as absurd and arbitrary the valuation of the 100 hectares at P5.1 million pesos. in San Vicente, Sumilao Bukidnon. In and Oder 1 dated November 14, 1994, DAR Secretary
That resolution was allegedly an "unprecedented turn-around which is most difficult for the Ernesto D. Garilao denied the application for conversion of the land from agricultural to agro-
discerning public to appreciate 24 industrial use and ordered its distibution to qualified landless farmers. BAIDA and NQSR
Management and Development Corporation filed a motion for reconsideration 2 dated
The "win-win" resolution being adverse to petitioners, they now assail the authority of the January 9, 1995, which was, however, denied in an Order 3 dated June 7, 1995, which was,
President to modify the Torres decision. Under the above-mentioned circumstances, however, denied in an Order 3dated June 7, 1995. Thereafter, Bukidnon Governor Carlos O.
however, the principle of estoppel applies to effectively bar petitioners from raising the issue Fortich sent a letter 4 to President Fidel V. Ramos requesting him to suspend the Garilao
of jurisdiction. 25While lack of jurisdiction of the court or quasi-judicial body may be assailed Order and to confirm the ordinance enacted by the Sangguniang Bayan of Sumilao converting
at any stage, a party's active participation in the proceedings before it will estop him from the subject land from agricultural to insdustrial/institutional land. Acting on the letter, then
assailing its lack of jurisdiction.26 This Court has always frowned upon the undesirable Executive Secretary Torres reversed the Garilao Order and upheld the power of local
practice of a party submitting his case for decision and then accepting the judgment, only if government units to convert portions of their agricultural lands into industrial
favorable, and attacking it for lack of jurisdiction when adverse. 27 areas. 5 Respondent DAR Secretary Garilao filed a motion for reconsiderations, admittedly
tardy, which was denied by then Executive Secretary Torres on the ground that his March 29,
423

1996 decision had already become final and executory in view of the lapse of the fifteen-day A closer scrutiny of the records in the instant case reveals that the fifteen-day rule for filling a
period for filling a motion for reconsideration. A second motion for reconsideration was filed motion for reconsideration under Section 7 of Administrative Order No. 18 was suspended by
during the pendency of which President Ramos constituted the Presidential Fact-Finding Task the President when he constituted, on October 15, 1997 or some six (6) months after the
Force. On November 7, 1997, Deputy Executive Secretary Corona issued the herein-assailed promulgation of the Torres decision, the Presidential Fact-Finding Task Force to conduct a
"win-win" resolution which, pursuant to the recommendations of the task force, substantially comprehensive review of the proper land use of the 144-hectare Sumilao property. At that
modified the Torres decision by awarding one (100) hectares of the Sumilao property to the time, then Executive Secretary Torres had already denied the first motion for reconsideration
qualified farmer beneficiaries and allocating only forty four (44) hectares for the of the DAR on the ground that his March 29, 1997 decision had already become final and
establishment of an industrial and commercial zone. executory. This notwithstanding, the President treated the case as still open and stated in his
memorandum that the findings of the Presidential Fact-Finding Task Force" will be inputs to
In our decison promulgated in Baguio City on April 24, 1998, we annulled the "win-win" the resolution of the case now pending at the Office of the President regarding the said land"
resolution on the ground that public respondent Deputy Executive Secretary Renato C. (emphasis ours). 15 The President took cognizance of the special circumstances surrounding
Corona committed grave abuse of discretion in modifying an already final and executory the tardy filing by the DAR of its motion for reconsideration. The DAR lawyers assigned to the
decison of then Executive Secretary Ruben D. Torres. It is undisputed that the Department of Sumilao case received the Torres decision only, after the lapse of the reglementary fifteen-
Agrarian Reform (DAR) failed to comply with the fifteen-day period for filling a motion for day period for appeal. The copy of the decision intended for them was passed from one
reconsideration. 6 It received the Torres decison on April 10, 1996 but transmitted its for office to another, e.g., the Records Section of the DAR, the Office of the DAR Secretary, the
mailing to the Office of the President only on May 23, 1997. 7 The Office of the President Bureau of Agrarian Legal Assistance, before it finally reached the DAR Litigation Office. It
received the motion on July 14, 1997. Forthwith, we applied the rule on finality of does not appear to be just that DAR will be made to lose a significant case because of
administrative determinations and upheld the policy of setting an end to litigation as an bureaucratic lapses. Viewed in this context, we should rule that the President suspended the
indispensable aspect of orderly administration of justice. In their motions for effectivity of Section 7 of Administrative Order No. 18 and that his exercise of discreation in
reconsideration, respondent and intervenors protest the technical basis of our decision. this regard cannot be assailed as whimsical.

I vote to grant their motions for reconsideration and remand the case to the Court of I also repectfully submit this act of the President also finds full sanction under the corollary
Appeals. principles of presidential power of control and qualified political agency.

First. It is true that procedural rules are necessary to secure just speedy and inexpensive This presidential power of control over the executive branch of
disposition of every action and proceeding. 8 Procedure, however, is only a means to an government extends over all executive officers from Cabinet Secretary to
end, 9and they may be suspended when they subvert the interests of justice. It is sel-evident the lowliest clerk and has been held by us, in the landmark case of
that the prerogative to suspend procedural rules or to grant an exception in a particular case Mondono vs. Silvosa to mean "the power of [the President] to alter or
lies in the authority that promulgated the rules. 10 modify or nullify or set aside what a subordinate officer had done in the
performance of his duties and to substitute the judgment of the former
with that of the latter." It is said to be at the very "heart of the meaning
Rules concerning pleading, practice and procedure in all courts are promulgated by this
of Chief Executive".
Court. 11On the other hand, it is the President as administrative head who is vested by the
Administrative Code of 1987 to promulgate rules relating to governmental operations,
including administrative procedure. These rules take the form of administrative Equally well accepted, as a corollary rule to the control powers of the
orders. 12 This power is necessary for the President to discharge his constitutional duty of President, is the "Doctrine of Qualified Political Agency." As the President
faithfully executing our law. 13 Under exceptional circumstances, this Court has suspended its cannot be expected to exercise his control powers all the same time and
rules to prevent miscarriage of justice. In the same breath, we should hold that the President in person, he will have to delegate some of them to his Cabinet members.
has the power to suspend the effectivity of administrative rules of procedure when they
hamper, defeat or in any way undermine the effective enforcement of the laws of the land. Under this doctrine, which recognizes the establishment of a single
Indeed, we already recognize that Congress can suspend its own rules if doing so will enable executive, "all executive and administrative organizations are adjuncts of
it to facilitate its task of lawmaking. The three great branches of our government are co- the Executive Department, the heads of the various executives
equal and within their own sphere they have the same responsibility to promote the good of departments are assitants and agents of the Chief Executive, and, except
our people. There is no reason to withhold the power to suspend rules from the President in cases where the Chief Executive is required by the Constitution or law
and grant it alone to the two other branches of government. to act in person o[r] the exigencies of the situation demand that he act
personally, the multifarious executive and administrative functions of the
424

Chief Executive are performed by and through the executive Corporation, however, did not question the authority of the President to constitute the task
departments, and the acts of the Secretaries of such departments, force despite its express adherence to the declaration made by then Executive Secretary
performed and promulgated in the regular course of business, are, unless Torres as to the finality of his March 29, 1997 decision. It was confident that its interests
dissapproved or reprobated by the Chief Executive presumptively the would be promoted and protected by Bukidnon Governor Fortich who himself filed the
acts of the Chief Executive". . . . appeal from the order of DAR Secretary Garilao 22 and Sumilao Mayor Baula who certified as
correct Resolution No. 24 approved by the Sangguniang Bayan of Sumilao on March 4, 1993
Thus, and in short, "the President's power of control is directly exercised converting the 144-hectare property from agricultural to industrial/institutional land. 23 But
by him over the members of the Cabinet who, in turn, and by his when "win-win" resolution was issued by the Office of the President on November 7, 1997,
authority, control the bureaus and other offices under their respective allowing the conversion into industrial land of only forty four (44) hectares of the 114-
jurisdictions in the executive department." 16 hectare Sumilao property and ordering the distribution of the rest to qualified farmer
beneficiaries, petitioners were flabbergasted. Mr. Norberto Quisumbing, Jr. could hardly hide
his disdain over that resolution in his letter to the provincial agrarian reform officer
By suspending the fifteen-day period for filing a motion for reconsideration and re-
protesting as absurd and arbitrary the valuation of the 100 hectares at P5.1 million pesos.
opening the Torres decision, the President clearly exercised his control power over
That resolution was allegedly an "unprecedented turn-around which is most difficult for the
an alter-ego within the framework of a constitutional and presidential system of
discerning public to appreciate 24
governance.

The "win-win" resolution being adverse to petitioners, they now assail the authority of the
The President's suspension of the fifteen-day rule for filing a motion for reconsideration
President to modify the Torres decision. Under the above-mentioned circumstances,
cannot be characterized as arbitrary. The Sumilao problem raises fundamental issues which
however, the principle of estoppel applies to effectively bar petitioners from raising the issue
conflict between land reform and the industrialization of the countryside, the power of
of jurisdiction. 25While lack of jurisdiction of the court or quasi-judicial body may be assailed
control by the President over his alter-ego vis-a-vis the power of local governments to
at any stage, a party's active participation in the proceedings before it will estop him from
convert agricultural land to industrial land. The resolution of these issues has far reaching
assailing its lack of jurisdiction.26 This Court has always frowned upon the undesirable
implications on the issues of our land reform program. Indeed, their successful resolution can
practice of a party submitting his case for decision and then accepting the judgment, only if
bring peace or rebellion in our contryside. The President should not be frustrated by an
favorable, and attacking it for lack of jurisdiction when adverse. 27
administrative procedural rule that he himself promulgated, from formulating a creative,
legal solution to the Sumilao problem. There is no denying the liberal interpretation equally
accorded to both administrative and judicial rules in order to promote their object to the Third. Considering the special circumstances of the case as detailed above, it would better
extent that technicality be not a bar to the vindication of a legitimate grievance. We have serve the ends of justice to obtain a definitive resolution of the issues raised in the instant
trumpeted the truism that when technicality ceases to be an aid to justice, the courts are petition and remand the same to the Court of Appeals where jurisdiction over this appeal
justified in excepting from its operation a paricular case. 17 We ought not to deny the same lies. Noteworthy, is the pendency in the Court of Appeals of two more cases involving the
power to the Chief Executive who heads a co-equal branch of government. Sumilao property: (1) Petition for Certiorari and Prohibition, entitled, "N.Q.S.R. Management
& Development Corporation and Bukidnon Agro-Industrial Association, Petitioners, vs. Hon.
Ernesto Garilao, Secretary of the Department of Agrarian Reform; Rogelio E. Tamin, DAR
Second. The petitioners are estopped from assailing the authority of the Office of the
Regional Director, Region X; Nicanor Peralta, Provincial Agrarian Reform Officer, Region X;
President to re-open the Sumilao case and resolve it based on the report of the Presidential
Dolores Apostol, Municipal Agrarian Reform Officer, Sumilao, Bukidnon, Respondent;" 28 and
Fact-Finding Task Force. Undeniably, petitioners participated in the processes conducted by
(2) Petition, for Certiorari and Prohibition, entitled, "Rodolfo Buclasan, et al., Petitioners, vs.
the task force. Their participation in the administrative proceedings without raising any
Hon. Leonardo N. Demecillo, as Judge of RTC, Malaybalay, Bukidnon, Branch IX and NQSR
objection thereto, bars them from raising any jurisdictional infirmity after an adverse
Management and Development Corporation,
decision is rendered against them. 18Petitioners Carlos O. Fortich and Rey B. Baula, Bukidnon
Respondent. 29
Governor and Sumilao Mayor, respectively, were named members of the task force. 19 The
president ordered the task force to confer with the representatives of, among others, the
landowner, namely, petitioner NQSR Management and Development Corporation. 20 In a The remand of the instant petition to the Court of Appeals would enable said court to
letter dated October 20, 1997 addressed to the President, the counsel for NQSR consolidate the same with the two other cases pending there which undoubtedly
Management and Development Corporation expressed its reluctance "to comment on the contemplate of the same factual milieu and raise invariably the same issues as in this
merits and demerits of the (motion for intervention and motion to admit additional evidence petition, leaving no room for further confusion that will surely be wrought by the rendition of
filed by the farmer beneficiaries] out of respect of the Regional Trial Court and the Court of conflicting decisions affecting a single controversy.
Appeals where these cases are presently pending. 21 NQSR Management and Development
425

For the above reasons, I vote to grant the motions for reconsideration filed by the executory, was in gross disregard of the rules and basic legal precept that accord finality to
respondents and the intervenors who should be allowed to intervene pursuant to sec, 1, Rule administrative determinations.
19 and to remand the instant petition to the Court of Appeals for appropriate proceedings.
The orderly administration of justice requires that the judgments/resolutions of a court or
289 SCRA 624, April 24, 1998 quasi-judicial body must reach a point of finality set by the law, rules and regulations. The
noble purpose is to write finis to disputes once and for all
TOPIC: Finality of Judgement; Administrative Law
This case involves a land located at San Vicente, Sumilao, Bukidnon, owned by the Norberto
DOCTRINE: The orderly administration of justice requires that the judgements/resolutions of Quisumbing, Sr. Management and Development Corporation (NQSRMDC), one of the
a court or quasi-judicial body must reach a point of finality set by the law, rules and petitioners. The property is covered by a Transfer Certificate of Title No. 14371 of the
regulations; a resolution which substantially modifies a decision after it has attained finality is Registry of Deeds of the Province of Bukidnon.
utterly void. When an administrative agency's decision becomes final and executory and no
one has seasonably filed a motion for reconsideration thereto, the said agency has lost its In 1984, the land was leased as a pineapple plantation to the Philippine Packing Corporation,
jurisdiction to re-open the case, more so modify its decision. now Del Monte Philippines, Inc. (DMPI), a multinational corporation, for a period of ten (10)
years under the Crop Producer and Growers Agreement duly annotated in the certificate of
FACTS: title. The lease expired in April, 1994.
On March 29, 1996, the Office of the President (OP) issued a decision converting a large
parcel of land from agricultural land to agro-industrial/institutional area. Because of this, a
In October, 1991, during the existence of the lease, the Department of Agrarian Reform
group of farmer-beneficiaries staged a hunger strike in front of the Department of Agrarian
(DAR) placed the entire 144-hectare property under compulsory acquisition and assessed the
Reform (DAR) Compound in Quezon City in October 9, 1997. The strike generated a lot of
land value at P2.38 million.
publicity and even a number of Presidential Candidates (for the upcoming 1998 elections)
intervened on behalf of the farmers.
When NQSRMDC was about to transfer the title over the 4-hectare donated to DECS, it
Because of this “blackmail”, the OP re-opened the case and through Deputy Executive discovered that the title over the subject property was no longer in its name. It soon found
Secretary Renato C. Corona issued the so-called, “politically motivated”, “win-win” resolution out that during the pendency of both the Petition for Certiorari, Prohibition, with Preliminary
on November 7, 1997, substantially modifying its 1996 decision after it had become final and Injunction it filed against DAR in the Court of Appeals and the appeal to the President filed by
executory. Governor Carlos O. Fortich, the DAR, without giving just compensation, caused the
cancellation of NQSRMDCs title on August 11, 1995 and had it transferred in the name of the
ISSUE: WON the “win-win” resolution, issued after the original decision had become final and Republic of the Philippines under TCT No. T-50264 of the Registry of Deeds of Bukidnon.
executory, had any legal effect. Thereafter, on September 25, 1995, DAR caused the issuance of Certificates of Land
Ownership Award (CLOA) No. 00240227 and had it registered in the name of 137 farmer-
HELD: beneficiaries under TCT No. AT-3536 of the Registry of Deeds of Bukidnon.
No; When the OP issued the Order dated June 23,1997 declaring the Decision of March 29,
1996 final and executory, as no one has seasonably filed a motion for reconsideration NQSRMDC filed a complaint with the Regional Trial Court (RTC) of Malaybalay, Bukidnon
thereto, the said Office had lost its jurisdiction to re-open the case, more so modify its docketed as Civil Case No. 2687-97, for annulment and cancellation of title, damages and
Decision. Having lost its jurisdiction, the Office of the President has no more authority to injunction against DAR and 141 others. The RTC then issued a Temporary Restraining Order
entertain the second motion for reconsideration filed by respondent DAR Secretary, which and a Writ of Preliminary Injunction on May 19, 1997, restraining the DAR and 141 others
second motion became the basis of the assailed “Win-Win” Resolution. Section 7 of from entering, occupying and/or wresting from NQSRMDC the possession of the subject land.
Administrative Order No. 18 and Section 4, Rule 43 of the Revised Rules of Court mandate
that only one (1) motion for reconsideration is allowed to be taken from the Decision of Meanwhile, an Order was issued by then Executive Secretary Ruben D. Torres denying DARs
March 29, 1996. And even if a second motion for reconsideration was permitted to be filed in motion for reconsideration for having been filed beyond the reglementary period of fifteen
“exceptionally meritorious cases,” as provided in the second paragraph of Section 7 of AO 18, (15) days. The said order further declared that the March 29, 1996 OP decision had already
still the said motion should not have been entertained considering that the first motion for become final and executory.
reconsideration was not seasonably filed, thereby allowing the Decision of March 29, 1996 to
lapse into finality. Thus, the act of the Office of the President in re-opening the case and
substantially modifying its March 29,1996 Decision which had already become final and
426

On December 12, 1997, a Motion For Leave To Intervene was filed by alleged farmer- fifteen (15) days from notice of the said judgment, final order or resolution, whether the
beneficiaries, through counsel, claiming that they are real parties in interest as they were appeal involves questions of fact, of law, or mixed questions of fact and law.
previously identified by respondent DAR as agrarian reform beneficiaries on the 144-hectare
property subject of this case. The motion was vehemently opposed by the petitioners. However, in this particular case, the remedy prescribed in Rule 43 is inapplicable considering
that the present petition contains an allegation that the challenged resolution is patently
In seeking the nullification of the Win-Win Resolution, the petitioners claim that the Office of illegal and was issued with grave abuse of discretion and beyond his (respondent Secretary
the President was prompted to issue the said resolution after a very well-managed hunger Renato C. Coronas) jurisdiction when said resolution substantially modified the earlier OP
strike led by fake farmer-beneficiary Linda Ligmon succeeded in pressuring and/or politically Decision of March 29, 1996 which had long become final and executory. In other words, the
blackmailing the Office of the President to come up with this purely political decision to crucial issue raised here involves an error of jurisdiction, not an error of judgment which is
appease the farmers, by reviving and modifying the Decision of 29 March 1996 which has reviewable by an appeal under Rule 43. Thus, the appropriate remedy to annul and set aside
been declared final and executory in an Order of 23 June 1997. Thus, petitioners further the assailed resolution is an original special civil action for certiorari under Rule 65, as what
allege, respondent then Deputy Executive Secretary Renato C. Corona committed grave the petitioners have correctly done. The pertinent portion of Section 1 thereof provides:
abuse of discretion and acted beyond his jurisdiction when he issued the questioned
Resolution of 7 November 1997. They availed of this extraordinary writ of certiorari because SECTION 1. Petition for certiorari. When any tribunal, board or officer exercising judicial or
there is no other plain, speedy and adequate remedy in the ordinary course of law. They quasi-judicial functions has acted without or in excess of its or his jurisdiction, or with grave
never filed a motion for reconsideration of the subject Resolution because (it) is patently abuse of discretion amounting to lack or excess of jurisdiction, and there is no appeal, or any
illegal or contrary to law and it would be a futile exercise to seek reconsideration. plain, speedy, and adequate remedy in the ordinary course of law, a person aggrieved
thereby may file a verified petition in the proper court, alleging the facts with certainty and
Issue: praying that judgment be rendered annulling or modifying the proceedings of such tribunal,
board or officer, and granting such incidental reliefs as law and justice may require.
1) Whether or not the proper remedy of petitioners should have been to file a petition for
review directly with the Court of Appeals in accordance with Rule 43 of the Revised Rules of The office of a writ of certiorari is restricted to truly extraordinary cases in which the act of
Court; the lower court or quasi-judicial body is wholly void.

(2) Whether or not the petitioners failed to file a motion for reconsideration of the assailed The aforequoted Section 1 of Rule 65 mandates that the person aggrieved by the assailed
Win-Win Resolution before filing the present petition; and illegal act may file a verified petition (for certiorari) in the proper court. The proper court
where the petition must be filed is stated in Section 4 of the same Rule 65 which reads:
(3) Whether or not Petitioner NQSRMDC is guilty of forum-shopping.
SEC. 4. Where petition filed.- The petition may be filed not later than sixty (60) days from
Held: notice of the judgment, order or resolution sought to be assailed in the Supreme Court or, if
it relates to the acts or omissions of a lower court or of a corporation, board, officer or
person, in the Regional Trial Court exercising jurisdiction over the territorial area as defined
1. In order to determine whether the recourse of petitioners is proper or not, it is necessary
by the Supreme Court. It may also be filed in the Court of Appeals whether or not the same is
to draw a line between an error of judgment and an error of jurisdiction.
in aid of its appellate jurisdiction, or in the Sandiganbayan if it is in aid of its jurisdiction. If it
involves the acts or omissions of a quasi-judicial agency, and unless otherwise provided by
An error of judgment is one which the court may commit in the exercise of its jurisdiction, law or these Rules, the petition shall be filed in and cognizable only by the Court of Appeals.
and which error is reviewable only by an appeal. On the other hand, an error of jurisdiction is
one where the act complained of was issued by the court, officer or a quasi-judicial body
Under the above-quoted Section 4, the Supreme Court, Court of Appeals and Regional Trial
without or in excess of jurisdiction, or with grave abuse of discretion which is tantamount to
Court have original concurrent jurisdiction to issue a writ of certiorari, prohibition and
lack or in excess of jurisdiction. This error is correctable only by the extraordinary writ of
mandamus. But the jurisdiction of these three (3) courts are also delineated in that, if the
certiorari.
challenged act relates to acts or omissions of a lower court or of a corporation, board, officer
or person, the petition must be filed with the Regional Trial Court which exercises jurisdiction
It is true that under Rule 43, appeals from awards, judgments, final orders or resolutions of over the territorial area as defined by the Supreme Court. And if it involves the act or
any quasi-judicial agency exercising quasi-judicial functions, including the Office of the omission of a quasi-judicial agency, the petition shall be filed only with the Court of Appeals,
President, may be taken to the Court of Appeals by filing a verified petition for review within unless otherwise provided by law or the Rules of Court. We have clearly discussed this matter
427

of concurrence of jurisdiction in People vs. Cuaresma, et. al., through now Chief Justice Secretary, which second motion became the basis of the assailed Win-Win Resolution.
Andres R. Narvasa, thus: Section 7 of Administrative Order No. 18 and Section 4, Rule 43 of the Revised Rules of Court
mandate that only one (1) motion for reconsideration is allowed to be taken from the
This Courts original jurisdiction to issue writs of certiorari (as well as prohibition, mandamus, Decision of March 29, 1996. And even if a second motion for reconsideration was permitted
quo warranto, habeas corpus and injunction) is not exclusive. It is shared by this Court with to be filed in exceptionally meritorious cases, as provided in the second paragraph of Section
Regional Trial Courts , which may issue the writ, enforceable in any part of their respective 7 of AO 18, still the said motion should not have been entertained considering that the first
regions. It is also shared by this Court, and by the Regional Trial Court, with the Court of motion for reconsideration was not seasonably filed, thereby allowing the Decision of March
Appeals, although prior to the effectivity of Batas Pambansa Bilang 129, the latters 29, 1996 to lapse into finality. Thus, the act of the Office of the President in re-opening the
competence to issue the extraordinary writs was restricted to those in aid of its appellate case and substantially modifying its March 29,1996 Decision which had already become final
jurisdiction. This concurrence of jurisdiction is not, however, to be taken as according to and executory, was in gross disregard of the rules and basic legal precept that accord finality
parties seeking any of the writs an absolute, unrestrained freedom of choice of the court to to administrative determinations.
which application therefor will be directed.
In San Luis, et al. vs. Court of Appeals, et al. we held:
But the Supreme Court has the full discretionary power to take cognizance of the petition
filed directly to it if compelling reasons, or the nature and importance of the issues rose, Since the decisions of both the Civil Service Commission and the Office of the President had
warrant. This has been the judicial policy to be observed. long become final and executory, the same can no longer be reviewed by the courts. It is
well-established in our jurisprudence that the decisions and orders of administrative
Pursuant to said judicial policy, we resolve to take primary jurisdiction over the present agencies, rendered pursuant to their quasi-judicial authority, have upon their finality, the
petition in the interest of speedy justice and to avoid future litigations so as to promptly put force and binding effect of a final judgment within the purview of the doctrine of res judicata
an end to the present controversy which, as correctly observed by petitioners, has sparked The rule of res judicata which forbids the reopening of a matter once judicially determined by
national interest because of the magnitude of the problem created by the issuance of the competent authority applies as well to the judicial and quasi-judicial acts of public, executive
assailed resolution. Moreover, as will be discussed later, we find the assailed resolution or administrative officers and boards acting within their jurisdiction as to the judgments of
wholly void and requiring the petitioners to file their petition first with the Court of Appeals courts having general judicial powers.
would only result in a waste of time and money.
The orderly administration of justice requires that the judgments/resolutions of a court or
2. The rules and regulations governing appeals to the Office of the President of the quasi-judicial body must reach a point of finality set by the law, rules and regulations. The
Philippines are embodied in Administrative Order No. 18. Section 7 thereof provides: noble purpose is to write finis to disputes once and for all. This is a fundamental principle in
our justice system, without which there would be no end to litigations. Utmost respect and
adherence to this principle must always be maintained by those who wield the power of
SEC. 7. Decisions/resolutions/orders of the Office of the President shall, except as otherwise
adjudication. Any act which violates such principle must immediately be struck down.
provided for by special laws, become final after the lapse of fifteen (15) days from receipt of
a copy thereof by the parties, unless a motion for reconsideration thereof is filed within such
period. 3. There is forum-shopping whenever, as a result of an adverse opinion in one forum, a party
seeks a favorable opinion (other than by appeal or certiorari) in another. The principle
applies not only with respect to suits filed in the courts but also in connection with litigation
Only one motion for reconsideration by any one party shall be allowed and entertained, save
commenced in the courts while an administrative proceeding is pending, as in this case, in
in exceptionally meritorious cases.
order to defeat administrative processes and in anticipation of an unfavorable administrative
ruling and a favorable court ruling. This specially so, as in this case, where the court in which
It is further provided for in Section 9 that The Rules of Court shall apply in a suppletory the second suit was brought, has no jurisdiction.
character whenever practicable.
The test for determining whether a party violated the rule against forum shopping has been
When the Office of the President issued the Order dated June 23,1997 declaring the Decision laid down in the 1986 case of Buan vs. Lopez and that is, forum shopping exists where the
of March 29, 1996 final and executory, as no one has seasonably filed a motion for elements of litis pendentia are present or where a final judgment in one case will amount to
reconsideration thereto, the said Office had lost its jurisdiction to re-open the case, more so res judicata in the other, as follows:
modify its Decision. Having lost its jurisdiction, the Office of the President has no more
authority to entertain the second motion for reconsideration filed by respondent DAR
428

There thus exists between the action before this Court and RTC Case No. 86-36563 identity of
parties, or at least such parties as represent the same interests in both actions, as well as
identity of rights asserted and relief prayed for, the relief being founded on the same facts,
and the identity on the two preceding particulars is such that any judgment rendered in the
other action, will, regardless of which party is successful, amount to res adjudicata in the
action under consideration: all the requisites, in fine, of auter action pendant.

It is clear from the above-quoted rule that the petitioners are not guilty of forum shopping.
The test for determining whether a party has violated the rule against forum shopping is
where a final judgment in one case will amount to res adjudicata in the action under
consideration. A cursory examination of the cases filed by the petitioners does not show that
the said cases are similar with each other. The petition for certiorari in the Court of Appeals
sought the nullification of the DAR Secretarys order to proceed with the compulsory
acquisition and distribution of the subject property. On the other hand, the civil case in RTC
of Malaybalay, Bukidnon for the annulment and cancellation of title issued in the name of
the Republic of the Philippines, with damages, was based on the following grounds: (1) the
DAR, in applying for cancellation of petitioner NQSRMDCs title, used documents which were
earlier declared null and void by the DARAB; (2) the cancellation of NQSRMDCs title was
made without payment of just compensation; and (3) without notice to NQSRMDC for the
surrender of its title. The present petition is entirely different from the said two cases as it
seeks the nullification of the assailed Win-Win Resolution of the Office of the President dated
November 7, 1997, which resolution was issued long after the previous two cases were
instituted.
429

Saturnino C. Ocampo, et al. v. Rear Admiral Ernesto C. Enriquez, et al. Bayani (LNMB), in compliance with the verbal order of President Duterte to fulfill his election
campaign promise to that effect.
G.R. No. 225973, 8 November 2016, EN BANC (Peralta, J.)
On August 9, 2016, AFP Rear Admiral Ernesto C. Enriquez issued the corresponding directives
DOCTRINE OF THE CASE: to the Philippine Army Commanding General.

While the Constitution is a product of our collective history as a people, its entirety should Dissatisfied with the foregoing issuance, various parties filed several petitions for certiorari,
not be interpreted as providing guiding principles to just about anything remotely related to prohibition and mandamus, essentially arguing that the decision to have the remains of
the Martial Law period such as the proposed Marcos burial at the LNMB. former President Marcos interred at the LNMB violated various laws; that Marcos is not
entitled to be interred at the LNMB; and that the Marcos family has already waived such
To apply the standard that the LNMB is reserved only for the "decent and the brave" or
burial.
"hero" would be violative of public policy as it will put into question the validity of the burial
of each and every mortal remains resting therein, and infringe upon the principle of ISSUES:
separation of powers since the allocation of plots at the LNMB is based on the grant of
authority to the President under existing laws and regulations. 1. Did the issuance of the assailed memorandum and directive violate the Constitution,
domestic and international laws?
Dishonorable discharge through a successful revolution is an extra-constitutional and direct
sovereign act of the people which is beyond the ambit of judicial review, let alone a mere 2. Have historical facts, laws enacted to recover ill-gotten wealth from the Marcoses and
administrative regulation. It is undeniable that former President Marcos was forced out of their cronies, and the decisions of the Court on the Marcos regime nullified his entitlement
office by the people through the so-called EDSA Revolution. Said political act of the people as a soldier and former President to internment at the LNMB?
should not be automatically given a particular legal meaning other than its obvious
consequence - that of ousting him as president. To do otherwise would lead the Court to the 3. Has the Marcos family waived the burial of former President Marcos at the LNMB by virtue
treacherous and perilous path of having to make choices from multifarious inferences or of their agreement with the Government of the Republic of the Philippines as regards the
theories arising from the various acts of the people. return and internment of his remains in the Philippines?

Moreover, under the Administrative Code, the President has the power to reserve for public RULINGS:
use and for specific public purposes any of the lands of the public domain and that the
1. NO, the assailed memorandum and directive, being the President’s decision, to bury
reserved land shall remain subject to the specific public purpose indicated until otherwise
Marcos at the
provided by law or proclamation. At present, there is no law or executive issuance specifically
excluding the land in which the LNMB is located from the use it was originally intended by
LNMB is in accordance with the Constitution, domestic and international laws.
the past Presidents.
1987 Constitution. While the Constitution is a product of our collective history as a people, its
The allotment of a cemetery plot at the LNMB for Marcos as a former President and
entirety should not be interpreted as providing guiding principles to just about anything
Commander-in-Chief, a legislator, a Secretary of National Defense, military personnel, a
remotely related to the Martial Law period such as the proposed Marcos burial at the LNMB.
veteran, and a Medal of Valor awardee, whether recognizing his contributions or simply his
status as such, satisfies the public use requirement. Tañada v. Angara already ruled that the provisions in Article II of the Constitution are not
selfexecuting.
FACTS:
The reasons for denying a cause of action to an alleged infringement of broad constitutional
On August 7, 2016, Secretary of National Defense Delfin N. Lorenzana issued a memorandum
principles are sourced from basic considerations of due process and the lack of judicial
to the Chief of Staff of the Armed Forces of the Philippines (AFP), General Ricardo R. Visaya,
authority to wade“into the uncharted ocean of social and economic policy making.”
regarding the interment of former President Ferdinand E. Marcos at the Libingan ng Mga
430

In the same vein, Sec. 1 of Art. XI of the Constitution is not a self-executing provision. The Violations of International Humanitarian Law, and the Updated Set of Principles for the
Court also found the reliance on Sec. 3(2) of Art. XIV and Sec. 26 of Art. XVIII of the Protection and Promotion of Human Rights Through Action to Combat Impunity.
Constitution to be misplaced, with such provisions bearing no direct or indirect prohibition to
Marcos’ interment at the LNMB. The Court also found no violation of President Duterte’s When the Filipinos regained their democratic institutions after the successful People Power
mandate under Sec. 17, Art. VII of the Constitution to take necessary and proper steps to Revolution that culminated on February 25, 1986, the three branches of the government
carry into execution the law. have done their fair share to respect, protect and fulfill the country's human rights
obligations.
RA No. 289 (An Act Providing For the Construction of A National Pantheon for Presidents of
the Philippines, National Heroes and Patriots of the Country) Ocampo, et al. also invoked RA The 1987 Constitution contains provisions that promote and protect human rights and social
289, which authorized the construction of a National Pantheon as the burial place of the justice. As to judicial remedies, aside from the writs of habeas corpus, amparo, and habeas
mortal remains of all the Presidents of the Philippines, national heroes and patriots, as well data, the Supreme Court promulgated AO No. 25-2007, which provides rules on cases
as a Board on National Pantheon to implement the said law. involving extra-judicial killings of political ideologists and members of the media. On the part
of the Executive Branch, it issued a number of administrative and executive orders. Congress
Ocampo, et al. are mistaken. Both in their pleadings and during the oral arguments, they has passed several laws affecting human rights.
miserably failed to provide legal and historical bases as to their supposition that the LNMB
and the National Pantheon are one and the same. To date, the Congress has deemed it wise Contrary to Ocampo, et al.’s postulation, our nation's history will not be instantly revised by a
not to appropriate any funds for its construction or the creation of the Board on National single resolve of President Duterte, acting through the Enriquez, et al., to bury Marcos at the
Pantheon. This is indicative of the legislative will not to pursue, at the moment, the LNMB.
establishment of a singular interment place for the mortal remains of all Presidents of the
Whether Ocampo, et al. admit it or not, the lessons of Martial Law are already engraved,
Philippines, national heroes, and patriots.
albeit in varying degrees, in the hearts and minds of the present generation of Filipinos. As to
Even if the Court treats R.A. No. 289 as relevant to the issue, still, Ocampo, et al.'s allegations the unborn, it must be said that the preservation and popularization of our history is not the
must fail. To apply the standard that the LNMB is reserved only for the "decent and the sole responsibility of the Chief Executive; it is a joint and collective endeavor of every
brave" or "hero" would be violative of public policy as it will put into question the validity of freedom-loving citizen of this country.
the burial of each and every mortal remains resting therein, and infringe upon the principle
2. NO, Marcos remains to be qualified to be interred at the LNMB.
of separation of powers since the allocation of plots at the LNMB is based on the grant of
authority to the President under existing laws and regulations.
Under AFP Regulations G-161-375, the following are eligible for interment at the LNMB:
RA No. 10368 (Human Rights Victims Reparation and Recognition Act of 2013)
(a)Medal of Valor Awardees;

Ocampo, et al. also invoked RA 10368, modifiying AFP Regulations G-161-375, which they
(b) Presidents or Commanders-in-Chief, AFP;
interpreted as implicitly disqualifying Marcos’ burial at the LNMB because the legislature, a
co-equal branch of the government, has statutorily declared his tyranny as a deposed (c) Secretaries of National Defense;
dictator and has recognized the heroism and sacrifices of the Human Rights Violations
Victims (HRVVs). (d) Chiefs of Staff, AFP;

International Human Rights Laws (e) General/Flag Officers of the AFP;

Ocampo, et al. argued that the burial of Marcos at the LNMB will violate the rights of the (f) Active and retired military personnel of the AFP to include active draftees and trainees
HRVVs to “full” and “effective” reparation, provided under the International Covenant on who died in line of duty, active reservists and CAFGU Active Auxiliary (CAA) who died in
Civil and Political Rights (ICCPR), the Basic Principles and Guidelines on the Right to a Remedy combat operations or combat related activities;
and Reparation for Victims of Gross Violations of International Human Rights Law and Serious
431

(g) Former members of the AFP who laterally entered or joined the PCG and the PNP; (h) Aside from being eligible for burial at the LNMB, Marcos possessed none of the
Veterans of Philippine Revolution of 1890, WWI, WWII and recognized guerillas; (i) disqualifications stated in AFP Regulations G-161-375. He was neither convicted by final
Government Dignitaries, Statesmen, National Artists and other deceased persons whose judgment of the offense involving moral turpitude nor dishonorably
interment or reinternment has been approved by the Commander-in-Chief, Congress or the separated/reverted/discharged from active military service. Despite ostensibly persuasive
Secretary of National Defense; and g) Former Presidents, Secretaries of Defense, Dignitaries, arguments as to gross human rights violations, massive graft and corruption, and dubious
Statesmen, National Artists, widows of Former Presidents, Secretaries of National Defense military records, the 1986 popular uprising as a clear sign of Marcos’ discharge from the AFP,
and Chief of Staff. the fact remains that Marcos was not convicted by final judgment of any offense involving
moral turpitude. The various cases cited by Ocampo, et al., which were decided with finality
Similar to AFP Regulations G-161-374, the following are not qualified to be interred in the by courts here and abroad, have no bearing in this case since they are merely civil in nature;
LNMB: (a) Personnel who were dishonorably separated/reverted/discharged from the hence, cannot and do not establish moral turpitude.
service; and (b) Authorized personnel who were convicted by final judgment of an offense
involving moral turpitude. To the Court’s mind, the word "service" should be construed as that rendered by a military
person in the AFP, including civil service, from the time of his/her commission, enlistment,
In the absence of any executive issuance or law to the contrary, the AFP Regulations G-161- probation, training or drafting, up to the date of his/her separation or retirement from the
375 remains to be the sole authority in determining who are entitled and disqualified to be AFP. Civil service after honorable separation and retirement from the AFP is outside the
interred at the LNMB. Interestingly, even if they were empowered to do so, former context of "service" under AFP Regulations G-161-375. Hence, it cannot be conveniently
Presidents Corazon C. Aquino and Benigno Simeon C. Aquino III, who were themselves claimed that Marcos' ouster from the presidency during the EDSA Revolution is tantamount
aggrieved at the Martial Law, did not revise the rules by expressly prohibiting the burial of to his dishonorable separation, reversion or discharge from the military service.
Marcos at the LNMB.
Not being a military person who may be prosecuted before the court martial, the President
It is not contrary to the "well-established custom," as the dissent described it, to argue that can hardly be deemed "dishonorably separated/reverted/discharged from the service" as
the word "bayani" in the LNMB has become a misnomer since while a symbolism of heroism contemplated by AFP Regulations G-161-375. Dishonorable discharge through a successful
may attach to the LNMB as a national shrine for military memorial, the same does not revolution is an extraconstitutional and direct sovereign act of the people which is beyond
automatically attach to its feature as a military cemetery and to those who were already laid the ambit of judicial review, let alone a mere administrative regulation. It is undeniable that
or will be laid therein. former President Marcos was forced out of office by the people through the so-called EDSA
Revolution. Said political act of the people should not be automatically given a particular
Whether or not the extension of burial privilege to civilians is unwarranted and should be
legal meaning other than its obvious consequence - that of ousting him as president. To do
restricted in order to be consistent with the original purpose of the LNMB is immaterial and
otherwise would lead the Court to the treacherous and perilous path of having to make
irrelevant to the issue at bar since it is indubitable that Marcos had rendered significant
choices from multifarious inferences or theories arising from the various acts of the people.
active military service and military-related activities. Ocampo, et al. did not dispute that
Marcos was a former President and Commander-in-Chief, a legislator, a Secretary of National 3. NO, the Marcoses are not deemed to have waived the former President’s burial at the
Defense, a military personnel, a veteran, and a Medal of Valor awardee. LNMB.

For his alleged human rights abuses and corrupt practices, the Court may disregard Marcos The presidential power of control over the Executive Branch of Government is a self-
as a President and Commander-in-Chief, but the Court cannot deny him the right to be executing provision of the Constitution and does not require statutory implementation, nor
acknowledged based on the other positions he held or the awards he received. In this sense, may its exercise be limited, much less withdrawn, by the legislature. This is why President
the Court agreed with the proposition that Marcos should be viewed and judged in his Duterte is not bound by the alleged 1992 Agreement between former President Ramos and
totality as a person. While he was not all good, he was not pure evil either. Certainly, just a the Marcos family to have the remains of
human who erred like us.
Marcos interred in Batac, Ilocos Norte. As the incumbent President, he is free to amend,
revoke or rescind political agreements entered into by his predecessors, and to determine
432

policies which he considers, based on informed judgment and presumed wisdom, will be Marcos burial case: Ocampo vs. Enriquez case digest
most effective in carrying out his mandate.
October 9, 2017
Moreover, under the Administrative Code, the President has the power to reserve for public
use and for specific public purposes any of the lands of the public domain and that the Facts:
reserved land shall remain subject to the specific public purpose indicated until otherwise
provided by law or proclamation. During the campaign period for the 2016 Presidential Election, then candidate Rodrigo R.
Duterte publicly announced that he would allow the burial former President Ferdinand E.
At present, there is no law or executive issuance specifically excluding the land in which the Marcos at the Libingan ng Mga Bayani ("LNMB"). Duterte won the May 9, 2016 elections.
LNMB is located from the use it was originally intended by the past Presidents. The allotment
of a cemetery plot at the LNMB for Marcos as a former President and Commander-in-Chief, a On August 7, 2016, Defense Secretary Delfin N. Lorenzana issued a Memorandum to AFP
legislator, a Secretary of National Defense, military personnel, a veteran, and a Medal of Chief of Staff General Ricardo R. Visaya regarding the interment of former President
Valor awardee, whether recognizing his contributions or simply his status as such, satisfies Ferdinand E. Marcos at the Libingan ng Mga Bayani.
the public use requirement.
On August 9, 2016, AFP Rear Admiral Ernesto C. Enriquez issued a directive to the Philippine
Presumption of regularity in the performance of official duty prevails over Ocampo, et al.'s Army on the Funeral Honors and Service for President Marcos.
highly disputed factual allegation that, in the guise of exercising a presidential prerogative,
the Chief Executive is actually motivated by utang na loob (debt of gratitude) and bayad Dissatisfied with the foregoing issuance, the petitioners filed a Petition for Certiorari and
utang (payback) to the Marcoses. Prohibition and Petition for Mandamus and Prohibition with the Court.

Conclusion ISSUES:

In sum, there is no clear constitutional or legal basis to hold that there was a grave abuse of 1) Whether respondents Defense Secretary and AFP Rear Admiral committed grave abuse of
discretion amounting to lack or excess of jurisdiction which would justify the Court to discretion when they issued the assailed memorandum and directive in compliance with the
interpose its authority to check and override an act entrusted to the judgment of another verbal order of President Duterte to implement his election campaign promise to have the
branch. Truly, the President's discretion is not totally unfettered. "Discretion is not a free- remains of Marcos interred at the LNMB?
spirited stallion that runs and roams wherever it pleases but is reined in to keep it from
straying. In its classic formulation, 'discretion is not unconfined and vagrant' but 'canalized 2) Whether the issuance and implementation of the assailed memorandum and directive
violated the Constitution, and domestic and international laws?
within banks that keep it from overflowing."' At bar, President Duterte, through Enriquez, et
al., acted within the bounds of the law and jurisprudence.
3) Whether historical facts, laws enacted to recover ill-gotten wealth from the Marcoses and
their cronies, and the pronouncements of the Court on the Marcos regime have nullified his
Notwithstanding the call of human rights advocates, the Court must uphold what is legal and
entitlement as a soldier and former President to interment at the LNMB?
just. And that is not to deny Marcos of his rightful place at the LNMB. For even the framers of
our Constitution intend that full respect for human rights is available at any stage of a
4) Whether the Marcos family is deemed to have waived the burial of the remains of former
person's development, from the time he or she becomes a person to the time he or she President Marcos at the LNMB after they entered into an agreement with the Government of
leaves this earth. the Republic of the Philippines as to the conditions and procedures by which his remains shall
be brought back to and interred in the Philippines?
There are certain things that are better left for history - not this Court - to adjudge. The Court
could only do so much in accordance with the clearly established rules and principles. Beyond
that, it is ultimately for the people themselves, as the sovereign, to decide, a task that may
require the better perspective that the passage of time provides. In the meantime, the RULING:
country must move on and let this issue rest.
433

The Supreme Court denied the petitions. Exhaustion of administrative remedies

Procedural issues Petitioners violated the exhaustion of administrative remedies. Contrary to their claim of lack
of plain, speedy, adequate remedy in the ordinary course of law, petitioners should be
Political question faulted for failing to seek reconsideration of the assailed memorandum and directive before
the Secretary ofNational Defense. The Secretary of National Defense should be given
opportunity to correct himself, if warranted, considering that AFP Regulations G 161-375 was
The Court agrees with the OSG that President Duterte's decision to have the remains of
issued upon his order. Questions on the implementation and interpretation thereof demand
Marcos interred at the LNMB involves a political question that is not a justiciable
the exercise of sound administrative discretion, requiring the special knowledge, experience
controversy. In the excercise of his powers under the Constitution and the Administrative
and services of his office to determine technical and intricate matters of fact. If petitioners
Code of 1987 to allow the interment of Marcos at the LNMB, which is a land of the public
would still be dissatisfied with the decision of the Secretary, they could elevate the matter
domain devoted for national military cemetery and military shrine purposes, President
before the Office of the President which has control and supervision over the Department of
Duterte decided a question of policy based on his wisdom that it shall promote national
National Defense (DND).
healing and forgiveness. There being no taint of grave abuse in the exercise of such
discretion, as discussed below, President Duterte's decision on that political question is
outside the ambit of judicial review. Hierarchy of Courts

Locus standi While direct resort to the Court through petitions for the extraordinary writs of certiorari,
prohibition and mandamus are allowed under exceptional cases, which are lacking in this
case, petitioners cannot simply brush aside the doctrine of hierarchy of courts that requires
Petitioners have no legal standing to file the petitions for certiorari, prohibition and
such petitions to be filed first with the proper Regional Trial Court (RTC). The RTC is not just a
mandamus because they failed to show that they have suffered or will suffer direct and
trier of facts, but can also resolve questions of law in the exercise of its original and
personal injury as a result of the interment of Marcos at the LNMB.
concurrent jurisdiction over petitions for certiorari, prohibition and mandamus, and has the
power to issue restraining order and injunction when proven necessary.
Petitioners cannot also file as taxpayers. They merely claim illegal disbursement of public
funds, without showing that Marcos is disqualified to be interred at the LNMB by either
Substantive issues
express or implied provision of the Constitution, the laws or jurisprudence.

I. The President's decision to bury Marcos at the LNMB is in accordance with the
Petitioners Saguisag, et al., as members of the Bar, failed to disclose the direct or potential
Constitution, the law and jurisprudence.
injury which they may suffer as a result of the act complained of. Their interest in this case is
too general and shared by other groups, such that their duty to uphold the rule of law,
without more, is inadequate to clothe them with requisite legal standing. While the Constitution is a product of our collective history as a people, its entirety should
not be interpreted as providing guiding principles to just about anything remotely related to
the Martial Law period such as the proposed Marcos burial at the LNMB.
Petitioners also failed to prove that the case is of transcendental importance. At this point in
time, the interment of Marcos at a cemetery originally established as a national military
cemetery and declared a national shrine would have no profound effect on the political, Section 1 of Article XI of the Constitution is not a self-executing provision considering that a
economic, and other aspects of our national life considering that more than twenty-seven law should be passed by the Congress to clearly define and effectuate the principle
(27) years since his death and thirty (30) years after his ouster have already passed. embodied therein. Pursuant thereto, Congress enacted the Code of Conduct on Ethical
Significantly, petitioners failed to demonstrate a clear and imminent threat to their Standards for Public Officials and Employees, the Ombudsman Act of 1989, Plunder Act, and
fundamental constitutional rights. Anti-Red Tape Act of 2007. To complement these statutes, the Executive Branch has issued
various orders, memoranda, and instructions relative to the norms of behavior/code of
conduct/ethical standards of officials and employees; workflow charts/public transactions;
rules and policies on gifts and benefits; whistle blowing and reporting; and client feedback
program
As to petitioners Senator De Lima and Congressman Lagman, they failed to show that the
burial of Marcos encroaches on their prerogatives as legislators.
434

Petitioners' reliance on Sec. 3(2) of Art. XIV and Sec. 26 of Art. XVIII of the Constitution is also The Court cannot subscribe to petitioners' logic that the beneficial provisions of R.A. No.
misplaced. Sec. 3(2) of Art. XIV refers to the constitutional duty of educational institutions in 10368 are not exclusive as it includes the prohibition on Marcos' burial at the LNMB. It would
teaching the values of patriotism and nationalism and respect for human rights, while Sec. 26 be undue to extend the law beyond what it actually contemplates. With its victim-oriented
of Art. XVIII is a transitory provision on sequestration or freeze orders in relation to the perspective, our legislators could have easily inserted a provision specifically proscribing
recovery of Marcos' ill-gotten wealth. Clearly, with respect to these provisions, there is no Marcos' interment at the LNMB as a "reparation" for the Human Rights Violations Victims
direct or indirect prohibition to Marcos' interment at the LNMB. (HRVVs). The law is silent and should remain to be so. This Court cannot read into the law
what is simply not there. It is irregular, if not unconstitutional, for Us to presume the
The second sentence of Sec. 17 of Art. VII is likewise not violated by public respondents. legislative will by supplying material details into the law. That would be tantamount to
Being the Chief Executive, the President represents the government as a whole and sees to it judicial legislation.
that all laws are enforced by the officials and employees of his or her department. Under the
Faithful Execution Clause, the President has the power to take "necessary and proper steps" The enforcement of the HRVV s' rights under R.A. No 10368 will surely not be impaired by
to carry into execution the law. The mandate is self-executory by virtue of its being the interment of Marcos at the LNMB. As opined by the OSG, the assailed act has no causal
inherently executive in nature and is intimately related to the other executive functions. It is connection and legal relation to the law. The subject memorandum and directive of public
best construed as an imposed obligation, not a separate grant of power. The provision simply respondents do not and cannot interfere with the statutory powers and functions of the
underscores the rule of law and, corollarily, the cardinal principle that the President is not Board and the Commission. More importantly, the HRVVs' entitlements to the benefits
above the laws but is obliged to obey and execute them. provided for by R.A. No 10368 and other domestic laws are not curtailed. R.A. No. 10368
does not amend or repeal, whether express or implied, the provisions of the Administrative
There is no violation of RA 289. Code or AFP Regulations G 161-375.

Petitioners miserably failed to provide legal and historical bases as to their supposition that There is no violation of International Human Rights Laws.
the LNMB and the National Pantheon are one and the same. This is not at all unexpected
because the LNMB is distinct and separate from the burial place envisioned in R.A. No 289. The nation's history will not be instantly revised by a single resolve of President Duterte,
The parcel of land subject matter of President Quirino's Proclamation No. 431, which was acting through the public respondents, to bury Marcos at the LNMB. Whether petitioners
later on revoked by President Magsaysay's Proclamation No. 42, is different from that admit it or not, the lessons of Martial Law are already engraved, albeit in varying degrees, in
covered by Marcos' Proclamation No. 208. The National Pantheon does not exist at present. the hearts and minds of the present generation of Filipinos. As to the unborn, it must be said
To date, the Congress has deemed it wise not to appropriate any funds for its construction or that the preservation and popularization of our history is not the sole responsibility of the
the creation of the Board on National Pantheon. This is indicative of the legislative will not to Chief Executive; it is a joint and collective endeavor of every freedom-loving citizen of this
pursue, at the moment, the establishment of a singular interment place for the mortal country.
remains of all Presidents of the Philippines, national heroes, and patriots.
Notably, complementing the statutory powers and functions of the Human Rights Victims'
Furthermore, to apply the standard that the LNMB is reserved only for the "decent and the Claims Board and the HRVV Memorial Commission in the memorialization of HRVV s, the
brave" or "hero" would be violative of public policy as it will put into question the validity of National Historical Commission of the Philippines (NHCP), formerly known as the National
the burial of each and every mortal remains resting therein, and infringe upon the principle Historical Institute (NHJ), is mandated to act as the primary government agency responsible
of separation of powers since the allocation of plots at the LNMB is based on the grant of for history and is authorized to determine all factual matters relating to official Philippine
authority to the President under existing laws and regulations. Also, the Court shares the history.
view of the OSG that the proposed interment is not equivalent to the consecration of
Marcos' mortal remains. The act in itself does not confer upon him the status of a "hero." II. The President's decision to bury Marcos at the LNMB is not done whimsically, capriciously
Despite its name, which is actually a misnomer, the purpose of the LNMB, both from legal or arbitrarily, out of malice, ill will or personal bias.
and historical perspectives, has neither been to confer to the people buried there the title of
"hero" nor to require that only those interred therein should be treated as a "hero." Lastly,
The LNMB was not expressly included in the national shrines enumerated in PD 105
petitioners' repeated reference to a "hero's burial" and "state honors," without showing
proof as to what kind of burial or honors that will be accorded to the remains of Marcos, is
speculative until the specifics of the interment have been finalized by public respondents. P.D. No. 105 does not apply to the LNMB. Despite the fact that P.D. No. 208 predated P.D.
No. 105, the LNMB was not expressly included in the national shrines enumerated in the
latter. The proposition that the LNMB is implicitly covered in the catchall phrase "and others
No violation of RA 10639 .
435

which may be proclaimed in the future as National Shrines" is erroneous because: (1) As motivated by utang na loob (debt of gratitude) and bayad utang (payback) to the Marcoses.
stated, Marcos issued P.D. No. 208 prior to P.D. No. 105; (2) Following the canon of statutory As the purpose is not self-evident, petitioners have the burden of proof to establish the
construction known as ejusdem generis, 138 the LNMB is not a site "of the birth, exile, factual basis of their claim. They failed. Even so, this Court cannot take cognizance of factual
imprisonment, detention or death of great and eminent leaders of the nation,"; and (3) Since issues since We are not a trier of facts.
its establishment, the LNMB has been a military shrine under the jurisdiction of the PVAO.
AFP Regulations G 161-375 must be sustained.
Assuming that P.D. No. 105 is applicable, the descriptive words "sacred and hallowed" refer
to the LNMB as a place and not to each and every mortal remains interred therein. Hence, Under AFP Regulations G 161-375, the following are eligible for interment at the LNMB: (a)
the burial of Marcos at the LNMB does not diminish said cemetery as a revered and Medal of Valor Awardees; (b) Presidents or Commanders-in-Chief, AFP; ( c) Secretaries of
respected ground. Neither does it negate the presumed individual or collective "heroism" of National Defense; ( d) Chiefs of Staff, AFP; ( e) General/Flag Officers of the AFP; (f) Active and
the men and women buried or will be buried therein. The "nation's esteem and reverence for retired military personnel of the AFP to include active draftees and trainees who died in line
her war dead, " as originally contemplated by President Magsaysay in issuing Proclamation of duty, active reservists and CAFGU Active Auxiliary (CAA) who died in combat operations or
No. 86, still stands unaffected. That being said, the interment of Marcos, therefore, does not combat related activities; (g) Former members of the AFP who laterally entered or joined the
constitute a violation of the physical, historical, and cultural integrity of the LNMB as a PCG and the PNP; (h) Veterans of Philippine Revolution of 1890, WWI, WWII and recognized
national military shrine. guerillas; (i) Government Dignitaries, Statesmen, National Artists and other deceased persons
whose interment or reinterment has been approved by the Commander-in-Chief, Congress
The LNMB is considered as a national shrine for military memorials. The PVAO, which is or the Secretary of National Defense; and G) Former Presidents, Secretaries of Defense,
empowered to administer, develop, and maintain military shrines, is under the supervision Dignitaries, Statesmen, National Artists, widows of Former Presidents, Secretaries of National
and control of the DND. The DND, in tum, is under the Office of the President. Defense and Chief of Staff.

The presidential power of control over the Executive Branch of Government is a self- Similar to AFP Regulations G 161-374, the following are not qualified to be interred in the
executing provision of the Constitution and does not require statutory implementation, nor LNMB: (a) Personnel who were dishonorably separated/reverted/discharged from the
may its exercise be limited, much less withdrawn, by the legislature. This is why President service; and (b) Authorized personnel who were convicted by final judgment of an offense
Duterte is not bound by the alleged 1992 Agreement between former President Ramos and involving moral turpitude.
the Marcos family to have the remains of Marcos interred in Batac, Ilocos Norte. As the
incumbent President, he is free to amend, revoke or rescind political agreements entered In the absence of any executive issuance or law to the contrary, the AFP Regulations G 161-
into by his predecessors, and to determine policies which he considers, based on informed 375 remains to be the sole authority in determining who are entitled and disqualified to be
judgment and presumed wisdom, will be most effective in carrying out his mandate. interred at the LNMB. Interestingly, even if they were empowered to do so, former
Presidents Corazon C. Aquino and Benigno Simeon C. Aquino III, who were themselves
Moreover, under the Administrative Code, the President has the power to reserve for public aggrieved at the Martial Law, did not revise the rules by expressly prohibiting the burial of
use and for specific public purposes any of the lands of the public domain and that the Marcos at the LNMB. The validity of AFP Regulations G 161-375 must, therefor, be sustained
reserved land shall remain subject to the specific public purpose indicated until otherwise for having been issued by the AFP Chief of Staff acting under the direction of the Secretary of
provided by law or proclamation. At present, there is no law or executive issuance National Defense, who is the alter ego of the President.
specifically excluding the land in which the LNMB is located from the use it was originally
intended by the past Presidents. The allotment of a cemetery plot at the LNMB for Marcos as AFP Regulations G 161-375 should not be stricken down in the absence of clear and
a former President and Commander-in-Chief, a legislator, a Secretary of National Defense, a unmistakable showing that it has been issued with grave abuse of discretion amounting to
military personnel, a veteran, and a Medal of Valor awardee, whether recognizing his lack or excess of jurisdiction. Neither could it be considered ultra vires for purportedly
contributions or simply his status as such, satisfies the public use requirement. The providing incomplete, whimsical, and capricious standards for qualification for burial at the
disbursement of public funds to cover the expenses incidental to the burial is granted to LNMB.
compensate him for valuable public services rendered.
It is not contrary to the "well-established custom," as the dissent described it, to argue that
Likewise, President Duterte's determination to have Marcos' remains interred at the LNMB the word "bayani" in the LNMB has become a misnomer since while a symbolism of heroism
was inspired by his desire for national healing and reconciliation. Presumption of regularity in may attach to the LNMB as a national shrine for military memorial, the same does not
the performance of official duty prevails over petitioners' highly disputed factual allegation automatically attach to its feature as a military cemetery and to those who were already laid
that, in the guise of exercising a presidential prerogative, the Chief Executive is actually or will be laid therein. As stated, the purpose of the LNMB, both from the legal and historical
436

perspectives, has neither been to confer to the people buried there the title of "hero" nor to (Presidents Quirino, Garcia, and Macapagal). All of them were not convicted of a crime
require that only those interred therein should be treated as a "hero." involving moral turpitude. In addition, the classification between a military personnel and a
former President is germane to the purposes of Proclamation No. 208 and P.D. No. 1076.
In fact, the privilege of internment at the LNMB has been loosen up through the years. Since While the LNMB is a national shrine for military memorials, it is also an active military
1986, the list of eligible includes not only those who rendered active military service or cemetery that recognizes the status or position held by the persons interred therein.
military-related activities but also non-military personnel who were recognized for their
significant contributions to the Philippine society (such as government dignitaries, statesmen, Likewise, Marcos was honorably discharged from military service. PVAO expressly recognized
national artists, and other deceased persons whose interment or reinterment has been him as a retired veteran pursuant to R.A. No. 6948, as amended. Petitioners have not shown
approved by the Commander-in-Chief, Congress or Secretary of National Defense). In 1998, that he was dishonorably discharged from military service under APP Circular 17, Series of
the widows of former Presidents, Secretaries of National Defense and Chief of Staff were 1987 (Administrative Discharge Prior to Expiration of Term of Enlistment) for violating
added to the list. Whether or not the extension of burial privilege to civilians is unwarranted Articles 94, 95 and 97 of the Articles of War. The NHCP study is incomplete with respect to
and should be restricted in order to be consistent with the original purpose of the LNMB is his entire military career as it failed to cite and include the official records of the AFP.
immaterial and irrelevant to the issue at bar since it is indubitable that Marcos had rendered
significant active military service and military-related activities. The word "service" in AFP Regulations G 161-375 should be construed as that rendered by a
military person in the AFP, including civil service, from the time of his/her commission,
Petitioners did not dispute that Marcos was a former President and Commander-in-Chief, a enlistment, probation, training or drafting, up to the date of his/her separation or retirement
legislator, a Secretary of National Defense, a military personnel, a veteran, and a Medal of from the AFP. Civil service after honorable separation and retirement from the AFP is outside
Valor awardee. For his alleged human rights abuses and corrupt practices, we may disregard the context of "service" under AFP Regulations G 161-375.
Marcos as a President and Commander-in-Chief, but we cannot deny him the right to be
acknowledged based on the other positions he held or the awards he received. In this sense, Hence, it cannot be conveniently claimed that Marcos' ouster from the presidency during the
We agree with the proposition that Marcos should be viewed and judged in his totality as a EDSA Revolution is tantamount to his dishonorable separation, reversion or discharge from
person. While he was not all good, he was not pure evil either. Certainly, just a human who the military service. The fact that the President is the Commander-in-Chief of the AFP under
erred like us. the 1987 Constitution only enshrines the principle of supremacy of civilian authority over the
military. Not being a military person who may be prosecuted before the court martial, the
Aside from being eligible for burial at the LNMB, Marcos possessed none of the President can hardly be deemed "dishonorably separated/reverted/discharged from the
disqualifications stated in AFP Regulations G 161-3 7 5. He was neither convicted by final service" as contemplated by AFP Regulations G 161-375. Dishonorable discharge through a
judgment of the offense involving moral turpitude nor dishonorably successful revolution is an extra-constitutional and direct sovereign act of the people which is
separated/reverted/discharged from active military service. beyond the ambit of judicial review, let alone a mere administrative regulation.

The fact remains that Marcos was not convicted by final judgment of any offense involving It is undeniable that former President Marcos was forced out of office by the people through
moral turpitude. No less than the 1987 Constitution mandates that a person shall not be held the so-called EDSA Revolution. Said political act of the people should not be automatically
to answer for a criminal offense without due process of law. given a particular legal meaning other than its obvious consequence - that of ousting him as
president. To do otherwise would lead the Court to the treacherous and perilous path of
Also, the equal protection clause is not violated. Generally, there is no property right to having to make choices from multifarious inferences or theories arising from the various acts
safeguard because even if one is eligible to be buried at the LNMB, such fact would only give of the people. It is not the function of the Court, for instance, to divine the exact implications
him or her the privilege to be interred therein. Unless there is a favorable recommendation or significance of the number of votes obtained in elections, or the message from the
from the Commander- in-Chief, the Congress or the Secretary of National Defense, no right number of participants in public assemblies. If the Court is not to fall into the pitfalls of
can be said to have ripen. Until then, such inchoate right is not legally demandable and getting embroiled in political and oftentimes emotional, if not acrimonious, debates, it must
enforceable. remain steadfast in abiding by its recognized guiding stars - clear constitutional and legal
rules - not by the uncertain, ambiguous and confusing messages from the actions of the
people.
Assuming that there is a property right to protect, the requisites of equal protection clause
are not met. 181 In this case, there is a real and substantial distinction between a military
personnel and a former President. The conditions of dishonorable discharge under the
Articles of War attach only to the members of the military. There is also no substantial
distinction between Marcos and the three Philippine Presidents buried at the LNMB

Das könnte Ihnen auch gefallen